Ketabton.com (c) ketabton.com: The Digital Library

SIXTH EDITION

LANGEQ&A™

USMLE STEP2CK

Carlyle H. Chan, MD, FAPA Department of Psychiatry & Behavioral Medicine Medical College of Wisconsin Milwaukee, Wisconsin

New York Chicago San Francisco Lisbon London Madrid Mexico City Milan New Delhi San Juan Seoul Singapore Sydney Toronto (c) ketabton.com: The Digital Library

Copyright © 2008 by The McGraw-Hill Companies, Inc. All rights reserved. Manufactured in the United States of America. Except as permitted under the United States Copyright Act of 1976, no part of this publication may be reproduced or distributed in any form or by any means, or stored in a database or retrieval system, without the prior written permission of the publisher.

0-07-164318-4

The material in this eBook also appears in the print version of this title: 0-07-149400-6.

All trademarks are trademarks of their respective owners. Rather than put a trademark symbol after every occurrence of a trademarked name, we use names in an editorial fashion only, and to the benefit of the trademark owner, with no intention of infringement of the trademark. Where such designations appear in this book, they have been printed with initial caps.

McGraw-Hill eBooks are available at special quantity discounts to use as premiums and sales promotions, or for use in corporate training programs. For more information, please contact George Hoare, Special Sales, at [email protected] or (212) 904-4069.

TERMS OF USE

This is a copyrighted work and The McGraw-Hill Companies, Inc. (“McGraw-Hill”) and its licensors reserve all rights in and to the work. Use of this work is subject to these terms. Except as permitted under the Copyright Act of 1976 and the right to store and retrieve one copy of the work, you may not decompile, disassemble, reverse engineer, reproduce, modify, create derivative works based upon, transmit, distribute, disseminate, sell, publish or sublicense the work or any part of it without McGraw-Hill’s prior consent. You may use the work for your own noncommercial and personal use; any other use of the work is strictly prohibited. Your right to use the work may be terminated if you fail to comply with these terms.

THE WORK IS PROVIDED “AS IS.” McGRAW-HILL AND ITS LICENSORS MAKE NO GUARANTEES OR WARRANTIES AS TO THE ACCURACY, ADEQUACY OR COMPLETENESS OF OR RESULTS TO BE OBTAINED FROM USING THE WORK, INCLUDING ANY INFORMATION THAT CAN BE ACCESSED THROUGH THE WORK VIA HYPERLINK OR OTHERWISE, AND EXPRESSLY DISCLAIM ANY WARRANTY, EXPRESS OR IMPLIED, INCLUDING BUT NOT LIMITED TO IMPLIED WARRANTIES OF MERCHANTABILITY OR FITNESS FOR A PARTICULAR PURPOSE. McGraw-Hill and its licensors do not warrant or guarantee that the functions contained in the work will meet your requirements or that its operation will be uninterrupted or error free. Neither McGraw-Hill nor its licensors shall be liable to you or anyone else for any inaccuracy, error or omission, regardless of cause, in the work or for any damages resulting therefrom. McGraw-Hill has no responsibility for the content of any information accessed through the work. Under no circumstances shall McGraw-Hill and/or its licensors be liable for any indirect, incidental, special, punitive, consequential or similar damages that result from the use of or inability to use the work, even if any of them has been advised of the possibility of such damages. This limitation of liability shall apply to any claim or cause whatsoever whether such claim or cause arises in contract, tort or otherwise.

DOI: 10.1036/0071494006 (c) ketabton.com: The Digital Library

Professional

Want to learn more? We hope you enjoy this McGraw-Hill eBook! If you’d like more information about this book, its author, or related books and websites, please click here. (c) ketabton.com: The Digital Library

For more information about this title, click here

Contents

Contributors ...... vii

Preface ...... ix

Review Preparation Guide ...... xi

Standard Abbreviations ...... xvii

1. Internal Medicine ...... 1 MICHAEL H. BONNER, MD AND ANN M. MAGUIRE, MD, MPH Questions ...... 1 Answers and Explanations ...... 25 Bibliography ...... 45

2. Obstetrics and Gynecology ...... 47 JAMES AIMAN, MD AND MICHAEL R. LUND, MD Questions ...... 47 Answers and Explanations ...... 65 Bibliography ...... 86

3. Pediatrics ...... 87 RAINER GEDEIT, MD Questions ...... 87 Answers and Explanations ...... 107 Bibliography ...... 127

4. Preventive Medicine ...... 129 STEPHEN K. LIU, MD, MPH Questions ...... 129 Answers and Explanations ...... 146 Bibliography ...... 163

5. Psychiatry ...... 165 JON A. LEHRMANN, MD Questions ...... 165 Answers and Explanations ...... 180 Bibliography ...... 195

iii (c) ketabton.com: The Digital Library

iv Contents

6. Surgery ...... 197 KELLI R. BROWN, MD AND TRAVIS WEBB, MD Questions ...... 197 Answers and Explanations ...... 228 Bibliography ...... 252

7. Practice Test 1...... 253 Questions ...... 253 Answers and Explanations ...... 261 Bibliography ...... 270 Subject List: Practice Test 1...... 271

8. Practice Test 2...... 273 Questions ...... 273 Answers and Explanations ...... 282 Bibliography ...... 290 Subject List: Practice Test 2...... 291

9. Practice Test 3...... 293 Questions ...... 293 Answers and Explanations ...... 302 Bibliography ...... 309 Subject List: Practice Test 3...... 310

10. Practice Test 4...... 311 Questions...... 311 Answers and Explanations ...... 319 Bibliography ...... 328 Subject List: Practice Test 4...... 329

11. Practice Test 5...... 331 Questions ...... 331 Answers and Explanations ...... 340 Bibliography ...... 348 Subject List: Practice Test 5...... 349

12. Practice Test 6...... 351 Questions ...... 351 Answers and Explanations ...... 360 Bibliography ...... 368 Subject List: Practice Test 6...... 369

13. Practice Test 7...... 371 Questions ...... 371 Answers and Explanations ...... 381 Bibliography ...... 388 Subject List: Practice Test 7...... 389 (c) ketabton.com: The Digital Library

Contents v

14. Practice Test 8...... 391 Questions ...... 391 Answers and Explanations ...... 402 Bibliography ...... 410 Subject List: Practice Test 8...... 411

Index ...... 413

Color insert appears between pages 350 and 351. (c) ketabton.com: The Digital Library

This page intentionally left blank (c) ketabton.com: The Digital Library

Contributors

James Aiman, MD Jon A. Lehrmann, MD Professor Residency Director Department of Obstetrics and Gynecology Department of Psychiatry and Behavioral Medicine Medical College of Wisconsin Medical College of Wisconsin Senior Staff Physician Residency Director Department of Obstetrics and Gynecology Division of Mental Health Froedtert Hospital and Children’s Hospital Zablocki Veterans Administration Medical Center of Wisconsin Milwaukee, Wisconsin Milwaukee, Wisconsin Stephen K. Liu, MD, MPH Michael H. Bonner, MD Assistant Professor Assistant Professor of Medicine Department of Medicine Division of General Internal Medicine Dartmouth Medical School Department of Medicine Hanover, New Hampshire Medical College of Wisconsin Hospitalist Milwaukee, Wisconsin Section of Hospital Medicine Department of Medicine Kellie R. Brown, MD Dartmouth-Hitchcock Medical Center Assistant Professor Lebanon, New Hampshire Division of Vascular Surgery Department of Surgery Michael R. Lund, MD Medical College of Wisconsin Assistant Professor Chief, Division of Vascular Surgery Department of Obstetrics and Gynecology Zablocki Veterans Administration Medical Center Medical College of Wisconsin Milwaukee, Wisconsin Milwaukee, Wisconsin

Rainer Gedeit, MD Ann M. Maguire, MD, MPH Associate Professor Assistant Professor Division of Critical Care Division of General Internal Medicine Department of Pediatrics Department of Medicine Medical College of Wisconsin Medical College of Wisconsin Clinical Director, Pediatric Critical Care Milwaukee, Wisconsin Children’s Hospital of Wisconsin Milwaukee, Wisconsin Travis Webb, MD Assistant Professor Division of Trauma and Critical Care Department of Surgery Medical College of Wisconsin Milwaukee, Wisconsin

vii

Copyright © 2008 by The McGraw-Hill Companies, Inc. Click here for terms of use. (c) ketabton.com: The Digital Library

This page intentionally left blank (c) ketabton.com: The Digital Library

Preface

Taking licensing examinations is a stressful but nec- the six clinical disciplines, at the end we have pro- essary endeavor. Extensive clinical exposure and vided a complete practice test in eight parts, cover- comprehensive study, including rehearsing the ing all areas. examination process through a review book such as The contributors are experienced educators and this one, can aid in the preparation. Such reviews clinicians, several of whom have been either clerk- can help you identify areas of content weakness as ship directors or medical student education direc- well as provide you an opportunity to familiarize tors for their respective specialties. yourself with the test format. We believe this book will provide you with a The questions in this book were constructed valuable tool to assess your readiness to take the according to the parameters set forth in the USMLE exam. We hope you will find the questions, expla- Step 2 CK Bulletin. All of the subjects, types of ques- nations, and format to be of assistance to you in tions, and techniques that will be encountered on your review. Good luck! the USMLE Step 2 CK have been updated and pre- sented in this review. In addition to the questions in Carlyle H. Chan, MD, FAPA

ix

Copyright © 2008 by The McGraw-Hill Companies, Inc. Click here for terms of use. (c) ketabton.com: The Digital Library

This page intentionally left blank (c) ketabton.com: The Digital Library

Review Preparation Guide

This book is designed for those preparing for the 2. Pediatrics (focusing on pediatric content and United States Medical Licensing Examination tasks and competencies) (USMLE) Step 2 CK. It provides a comprehensive 3. Internal Medicine (including infectious disease, review, with more than 1000 clinical science multiple- immunology, and allergy; diseases of the respi- choice questions and referenced, paragraph-length ratory, cardiovascular, hematopoietic, gastroin- explanations of each answer. The last section of the testinal, renal, musculoskeletal, nervous, and book consists of two integrated practice tests for integumentary systems; nutritional, metabolic, self-assessment purposes. endocrine, oncologic, and fluid and electrolyte dis- This introduction provides information on orders; clinical pharmacology; legal medicine) question types, question-answering strategies, 4. Surgery (including the general topics of physiology, specifics on the USMLE Step 2 CK, and various anesthesiology, wounds, neoplasms, and forensic ways to use this review. medicine; specific surgical treatment of the various body systems) 5. Psychiatry (including theories; social, community, The United States Medical Licensing and family relationships; assessment techniques; Examination Step 2 CK psychopathology; interventions; ethical and legal aspects of psychiatry) The USMLE Step 2 CK is currently a 1-day (8-hours) 6. Preventive Medicine (including biostatistics, epi- computerized examination consisting of approxi- demiology, disease control, provision of health mately 400 multiple-choice questions testing your services, and ethical and legal aspects of medicine) knowledge in the clinical sciences. 7. Chapters 7–14. Practice Tests (includes 368 ques- tions from all six clinical sciences presented in an integrated format) Organization of this Book Each section is authored by an experienced teacher in the discipline. However, you will find that This book is organized to cover sequentially each of the author covers material of a general nature appro- the clinical science areas specified by the National priate for Step 2. As a result, the basic concepts of Board of Medical Examiners (NBME). There are six clinical pathophysiology are covered. As in the exam- sections, one for each of the clinical sciences, and an ination itself, topics that might be classified as gener- integrated practice test section at the end of the al or internal medicine are included in each section. review. The sections are as follows: Each of the chapters is organized in the follow- ing order: 1. Obstetrics and Gynecology (including biology of reproduction; fetus, placenta, and newborn; pri- 1. Questions mary care of the OB/GYN patient; normal and 2. Answers and Explanations abnormal clinical obstetrics; clinical gynecology) 3. Bibliography

xi

Copyright © 2008 by The McGraw-Hill Companies, Inc. Click here for terms of use. (c) ketabton.com: The Digital Library

xii Review Preparation Guide

These sections and how you might use them are The most likely cause is discussed below. (A) polycystic ovary disease Question Format (B) ovarian tumor (C) adrenal tumor The style and presentation of the questions have (D) Cushing’s disease been fully revised to conform with the USMLE. This will enable readers to familiarize themselves with (E) familial hirsutism the types of questions to be expected and practice In the question above, the key word is “most.” answering questions in each format. Following the Although ovarian tumors, adrenal tumors, and answer to each question, a reference refers the reader Cushing disease are causes of hirsutism (described to a particular and easily available text for further in the stem of the question), polycystic ovary dis- reference and reading. ease is a much more common cause. Familial hir- Each chapter contains multiple-choice questions sutism is not associated with the menstrual irregu- (or items). Most of these are one best answer–single- larities mentioned. Thus, the most likely cause of item questions, some are one best answer–matching the manifestations described can only be “(A) poly- sets, and some are comparison–matching set ques- cystic ovary disease.” tions. In some cases, a group of two or three ques- tions may be related to one situation. In addition, some questions have illustrations (graphs, x-rays, Strategies for Answering One Best tables, or line drawings) that require understanding Answer–Single-Item Questions and interpretation. Because the USMLE seems to 1. Remember that only one choice can be the correct prefer questions requiring judgment and critical answer. thinking in the context of clinical situations, we 2. Read the question carefully to be sure that you have attempted to emphasize these questions. understand what is being asked. 3. Quickly read each choice for familiarity. (This One Best Answer–Single-Item Question. Most of important step is often not done by test takers.) the questions are posed in the A-type, or “one best 4. Go back and consider each choice individually. answer–single-item” format. This is the most popu- 5. If a choice is partially correct, tentatively consider lar question format in most exams. It generally con- it to be incorrect. (This step will help you eliminate sists of a brief statement, followed by five options of choices and increase your odds of choosing the which only ONE is entirely correct. The options on correct answer.) the USMLE are lettered A, B, C, D, and E. Although 6. Consider the remaining choices and select the one the format for this question type is straightfor- you think is the answer. At this point, you may ward, these questions can be difficult, because want to scan the item quickly to ensure you some of the distractors may be partially right. The understand the question and your answer. instructions you will see for this type of question 7. If you do not know the answer, make an educated will generally appear as below: guess. Your score is based on the number of correct answers, not the number you get incorrect. DIRECTIONS: Each of the numbered items or Do not leave any blanks. incomplete statements in this section is followed 8. The actual examination is timed for an average of by answers or by completions of the statement. 1.2 minutes per question. It is important to be Select the ONE lettered answer or completion that thorough to understand the questions, but it is is BEST in each case. equally important for you to keep moving. The following is an example of this question type: One Best Answer–Matching Set Questions. This 1. An obese 21-year-old woman complains of format presents lettered options followed by sever- increased growth of coarse hair on her upper lip, al items related to a common topic. The directions chin, chest, and abdomen. She also notes men- you will generally see for this type of question are strual irregularity with periods of ? as follows: (c) ketabton.com: The Digital Library

Review Preparation Guide xiii

DIRECTIONS (Questions 2 through 4): Each set of Extended One Best Answer–Matching/Choosing matching questions in this section consists of a list Questions. The USMLE Step 2 uses a new type of of lettered options followed by several numbered matching question that is similar to the one above items. For each item, select the ONE best lettered but can contain up to 26 lettered options followed option that is most closely associated with it. Each by several items. The directions you will see for this lettered option may be selected once, more than type of question will generally read the same as once, or not at all. those listed for the best answer–matching sets, because this is another version of the same question. Below is an example of this type of question. An example of this type of question is: For each adverse drug reaction listed below, select the antibiotic with which it is most closely associated. (A) sarcoidosis (B) tuberculosis (A) tetracycline (C) histoplasmosis (B) chloramphenicol (D) coccidioidomycosis (C) clindamycin (E) amyloidosis (D) gentamicin (F) bacterial pneumonia 1. Bone marrow suppression (G) mesothelioma (H) carcinoma 2. Pseudomembranous enterocolitis (I) fibrosing alveolitis 3. Acute fatty necrosis of liver (J) silicosis Note that unlike the single-item questions, the choices in the matching sets precede the actual ques- 4. A right lower lobectomy specimen contains a tions. However, as with the single-item questions, solitary 1.2-cm-diameter solid nodule. The only one choice can be correct for a given question. center of the nodule is fibrous. The periphery has granulomatous inflammation. With special stains, multiple 2- to 5-μm budding yeasts are Strategies for Answering One Best evident within the nodule. Acid-fast stains are Answer–Matching Set Questions negative. 1. Remember that the lettered choices are followed 5. A left upper lobectomy specimen is received by the numbered questions. containing a 4.6-cm nodule with central cystic 2. As with single-item questions, only one answer degeneration. Microscopically, the nodule is is correct for each item. composed of anaplastic squamous cells. Similar 3. Quickly read each choice for familiarity. abnormal cells are seen in a concomitant biopsy 4. Read the question carefully to be sure you of a hilar lymph node. understand what is being asked. 5. Go back and consider each choice individually. 6. After a long history of multiple myeloma, a 6. If a choice is partially correct for a particular 67-year-old male is noted to have abundant item, tentatively consider it to be incorrect. (This acellular eosinophilic deposits around the pul- step will help you eliminate choices and increase monary microvasculature at autopsy. A Congo your odds of choosing the correct answer.) red special stain demonstrates apple green 7. Consider the remaining choices, and select the birefringence. one you think is correct. 7. A large pleural-based lesion is found on 8. If you do not know the answer, make an educated chest x-ray of an asbestos worker. Electron guess. Your score is based on the number of correct microscopy of the biopsy shows abundant long answers, not the number you get incorrect. Do microvilli. not leave any blanks. 9. Again, the actual examination allows an average Note that, as with other matching sets, the lettered of 1.2 minutes per question. options are listed first. (c) ketabton.com: The Digital Library

xiv Review Preparation Guide

Strategies for Answering Extended One Best For example, by checking off your incorrect Answer–Matching/Choosing Questions answers on, say, the preventive medicine list, you may find that a pattern develops in that you are 1. Read the lettered options through first. incorrect on most or all of the biostatistics questions. 2. Work with one item at a time. In this case, you could note the references (in the 3. Read the item through, then go back to the explanation section) for your incorrect answers and options and consider each choice individually. read those sources. You might also want to purchase 4. As with the other question types, if the choice is a biostatistics text or review book to do a much partially correct, tentatively consider it to be more in-depth review. We think that you will find incorrect. these subject lists very helpful, and we urge you to 5. Consider the remaining choices and select the use them. answer. 6. Remember to make a selection for each item. Practice Tests 7. Again, the test allows for 1.2 minutes per item. The eight blocks of practice tests at the end of the Answers, Explanations, and Bibliography book consist of questions from each of the six clini- cal sciences. The questions are grouped according to In each of the sections of this book, the question sec- question type (one best answer–single-item, one best tions are followed by a section containing the answer–matching sets, and comparison/matching answers, explanations, and bibliogaphy to the ques- sets, with the subject areas integrated. This format tions. This section: (1) tells you the answer to each mimics the actual exam and enables you to test your question; (2) gives you an explanation/review of skill at answering questions in all of the clinical sci- why the answer is correct, background information ences under simulated examination conditions. on the subject matter, and why the other answers The practice test section is organized in the fol- are incorrect; and (3) tells you where you can find lowing format: questions, answers and explana- more in-depth information on the subject matter in tions, and bibliogrpahy, and subject lists (which, other books and/or journals. We encourage you to here, will also list the major subject heading). use this section as a basis for further study and understanding. If you choose the correct answer to a question, you can then read the explanation: (1) for reinforce- How to Use this Book ment; and (2) to add to your knowledge about the subject matter (remember that the explanations usu- There are two logical ways to get the most value ally tell not only why the answer is correct, but also from this book. We call them Plan A and Plan B. why the other choices are incorrect). If you choose In Plan A, you go straight to the practice tests the wrong answer to a question, you can read the and complete them. After taking the practice tests, explanation for a learning/reviewing discussion of you check your answers and then tick off those you the material in the question. Furthermore, you can got wrong on the subject lists on pages 309–310 and note the reference cited (e.g., Last, pp. 478–484), look 365–366. The number of questions you got wrong up the full source in the Bibliography at the end of will be a good indicator of your initial knowledge the section (e.g., Wallace RB, Doebbeling BN, eds. state, and the types of questions you got wrong will Maxcy-Rosenau-Last Textbook of Public Health & help point you in the right direction for further Preventive Medicine, 14th ed. Stamford, CT: Appleton preparation and review. At this point, you can use & Lange, 1998.), and refer to the pages cited for a the first six sections of the book, with the lists and more in-depth discussion. discussions, to help you improve your areas of rela- tive weakness. Subject Lists In Plan B, you go through the clinical science At the end of the practice tests of this book is a sub- sections (from OB/GYN to preventive medicine), ject list for each subject area. These subject lists will checking off your answers, and then compare your help point out your areas of relative weakness, and choices with the answers and discussions in the thus help you focus your review. book. Once you’ve completed this process, you can (c) ketabton.com: The Digital Library

Review Preparation Guide xv

take the practice tests, check your answers as us can sit for an exam without study. In this case, described above, and see how well prepared you you will have done some reviewing (from superfi- are at this point. If you still have a major weakness, cial to in-depth), and your practice tests will reflect it should be apparent in time for you to take reme- this studying time. If, after reviewing the six clinical dial action. science sections and taking the practice tests, your In Plan A, by taking the practice tests first, you scores still indicate some weaknesses, you can then get quick feedback regarding your initial areas of go back into the clinical science sections and sup- strength and weakness. You may find that you plement your review with your texts. know all of the material very well, indicating that perhaps only a cursory review of the six clinical sci- ence sections is necessary. This, of course, would be Specific Information on the Step 2 Examination good to know early on in your exam preparation. On the other hand, you may find that you have many areas of weakness (say, for example, in all of The official source of all information with respect to pediatrics and psychiatry and in some of the sub- the United States Medical Licensing Examination specialties of preventive medicine). In this case, you Step 2 is the National Board of Medical Examiners could then focus on these areas in your review—not (NBME), 3930 Chestnut Street, Philadelphia, PA just with this book, but also with textbooks of pedi- 19104. Established in 1915, the NBME is a voluntary, atrics and psychiatry. nonprofit, independent organization whose sole It is, however, unlikely that you will not do function is the design, implementation, distribution, some studying before taking the USMLE Step 2 CK and processing of a vast bank of question items, cer- (especially because you have this book). Therefore, tifying examinations, and evaluative services in the it may be more realistic to take the practice tests after professional medical field. Contact the NBME for you have reviewed the six clinical science sections information on registration requirements for the (as in Plan B). This, of course, will probably give you USMLE Step 2. a more realistic test-type situation, because few of (c) ketabton.com: The Digital Library

This page intentionally left blank (c) ketabton.com: The Digital Library

Standard Abbreviations

ACTH: adrenocorticotropic hormone IgA, etc.: immunoglobulin A, etc. ADH: antidiuretic hormone IM: intramuscular(ly) ADP: adenosine diphosphate IQ: intelligence quotient AFP: a-fetoprotein IU: international unit AMP: adenosine monophosphate IV: intravenous(ly) ATP: adenosine triphosphate KUB: kidney, ureter, and bladder ATPase: adenosine triphosphatase LDH: lactic dehydrogenase bid: two times a day LH: luteinizing hormone BP: blood pressure LSD: lysergic acid diethylamide BUN: blood urea nitrogen mRNA: messenger RNA CT: computed tomography PO: oral(ly) CBC: complete blood count PRN: as needed CCU: coronary care unit CNS: central nervous system RBC: red blood cell CPK: creatine phosphokinase RNA: ribonucleic acid CSF: cerebrospinal fluid RNase: ribonuclease rRNA: ribosomal RNA DNA: deoxyribonucleic acid DNase: deoxyribonuclease SC: subcutaneous(ly) SGOT: serum glutamic oxaloacetic transaminase ECG: electrocardiogram SGPT: serum glutamic pyruvic transaminase EDTA: ethylenediaminetetraacetate EEG: electroencephalogram TB: tuberculosis ER: emergency room tRNA: transfer RNA TSH: thyroid-stimulating hormone FSH: follicle-stimulating hormone WBC: white blood cell GI: gastrointestinal GU: genitourinary Hb: hemoglobin HCG: human chorionic gonadotropin Hct: hematocrit

xvii

Copyright © 2008 by The McGraw-Hill Companies, Inc. Click here for terms of use. (c) ketabton.com: The Digital Library

This page intentionally left blank (c) ketabton.com: The Digital Library

CHAPTER 1 Internal Medicine Michael H. Bonner, MD and Ann M. Maguire, MD, MPH

Questions

Questions 1 through 3 2. Which of the following examination findings would this patient most likely have? 1. You evaluate a 38-year-old man who complains of muscle weakness. Her appearance is remark- (A) proximal muscle weakness able for a periorbital heliotrope rash with (B) distal muscle weakness edema and erythema on his upper chest, neck, (C) ataxic gait and face (Figure 1-1). Which of the following is (D) hyperactive deep tendon reflexes the most likely diagnosis? (E) inflamed small joints

3. Which of the following blood parameters is likely to be elevated? (A) serum creatinine (B) serum potassium (C) serum sodium (D) rheumatoid factor (E) creatinine phosphokinase

4. A 47-year-old man with diabetes and hyper- tension travels with his family to Mexico. The next morning after eating out at a local restau- rant and despite drinking bottled water, he develops severe crampy abdominal pain and watery, frequent diarrhea. Which of the fol- lowing is the best approach for his care? (A) ciprofloxacin × 3 days (B) penicillin × 5 days FIG. 1-1 (Reproduced, with permission, from Hurwitz RM, Hood AF. × Pathology of the Skin. Stamford, CT: Appleton & Lange, 1998:41.) (C) tetracycline 3 days (D) observation of symptoms (E) metronidazole × 10 days (A) polymyositis (B) dermatomyositis (C) spinocerebellar degeneration (D) vasculitis (E) rheumatoid arthritis

1

Copyright © 2008 by The McGraw-Hill Companies, Inc. Click here for terms of use. (c) ketabton.com: The Digital Library

2 1: Internal Medicine

5. Which of the following is the most likely diag- 7. Which of the following is the prophylactic nosis for the ulcerated lesion on the person’s antibiotic of choice for dental procedures? cheek shown in Figure 1-2? (A) amoxicillin (B) vancomycin (C) cephalexin (D) penicillin (E) clindamycin

8. In patients who are not intravenous (IV) drug users and who do not have prosthetic valves, which of the following organisms is the most common cause of bacterial endocarditis? (A) Enterococcus (B) Streptococcus (C) gram-negative bacilli (D) Candida (E) Pseudomonas

9. A 36-year-old female complains of 5 days of fever, nasal congestion, sinus pressure, and postnasal drip. On examination, nasal dis- charge is yellow and the posterior pharynx is FIG. 1-2 (Reproduced, with permission, from Fitzpatrick TB. Color slightly erythematous. Tapping over the max- Atlas and Synopsis of Clinical Dermatology, 2nd ed. New York, NY: illary sinuses elicits mild pain. Which of the McGraw-Hill, 1994.) following is the most appropriate treatment for this patient? (A) squamous cell carcinoma (A) treatment of symptoms (analgesics, (B) malignant melanoma antipyretics, decongestants) (C) benign ulcerated nevus (B) a 7-day course of amoxicillin (D) basal cell carcinoma (C) a 10-day course of amoxicillin clavulanic (E) hemangioma acid (D) a 14-day course of clarithromycin Questions 6 through 8 Questions 10 through 12 A dentist asks you to evaluate a 42-year-old woman before tooth extraction. You make the diagnosis of Marfan syndrome in a very tall 22-year-old man with long, thin extremities. 6. Which of the following would prompt you to prescribe prophylactic antibiotics? 10. What other finding is associated with this disease? (A) midsystolic click at the left sternal border (B) insulin-dependent diabetes (A) family history in 100% of the patients (C) a prior history of infective endocarditis (B) upward subluxation of the lenses (D) a history of congestive heart failure (C) mental retardation (D) malar rash (E) S4 gallop (E) increased length of trunk compared with the limbs (c) ketabton.com: The Digital Library

Questions: 5–15 3

11. The major cause of morbidity and mortality in 15. A 59-year-old woman complains of shortness Marfan patients is cardiac. Which of the fol- of breath and aching left-sided chest pain that lowing is a common complication? radiates to the left shoulder. Physical exami- nation shows no abnormalities; her CXRs are (A) pulmonary stenosis shown in Figure 1-3. Which of the following (B) ventricular septal defect (VSD) statements is true concerning this disease? (C) pulmonary hypertension (D) aortic root dilatation (E) coronary artery disease (CAD)

12. Which of the following is the best way to mon- itor these patients for cardiovascular changes? (A) electrocardiogram (ECG) (B) chest x-ray (CXR) (C) angiography (D) pulmonary function tests (E) echocardiography

13. A 30-year-old woman is visiting you in your primary care office as a new patient. Overall, she is healthy. On taking a family history, you learn that her mother was diagnosed with col- orectal cancer at the age of 50. When should this patient start being screened for colorectal cancer? (A) there is no proven benefit for colorectal cancer screening (B) at age 40 (C) at age 50 (D) at age 60 (E) at age 30

14. A 70-year-old man presents to urgent care com- plaining of a painful, swollen left knee. He pre- viously has had no problems with this knee. Three days prior to onset, he went out dancing for 2–3 hours but recalls no specific injury. Examination of the knee reveals a moderate- FIG. 1-3 sized effusion and mild pain with any range of motion. Plain x-ray shows no fracture. Which (A) This tumor frequently metastasizes to of the following is the best next management? distant sites. (B) Direct exposure to asbestos is required. (A) MRI of knee (C) Most cases are associated with recent, (B) aspiration of effusion fluid massive exposure to asbestos. (C) rest, ice, and leg elevation (D) Diffuse forms may be cured by (D) physical therapy referral chemotherapy alone. (E) arthroscopy (E) Localized forms may be cured by surgery alone. (c) ketabton.com: The Digital Library

4 1: Internal Medicine

16. A 23-year-old woman presents with “skipped 18. A 19-year-old high school senior complains of heartbeats” and on cardiac examination is feeling “fat and ugly” despite being extremely found to have a midsystolic click followed by thin. She takes small amounts of food at meals a late systolic murmur. Echocardiogram shows and occasionally gags herself to induce vomit- prolapse of the mitral valve. Which of the fol- ing after meals. Which of the following is lowing is true about this condition? commonly associated with this disorder? (A) Mitral valve prolapse is present in up to (A) menorrhagia 10% of the population. (B) metrorrhagia (B) Mitral valve prolapse is more common (C) loss of body hair in men. (D) bradycardia (C) Prophylaxis against bacterial endocardi- (E) thrombocytopenia tis is never recommended. (D) Risk of pulmonary embolism is high. Questions 19 and 20 (E) Ventricular arrhythmias do not occur. A 59-year-old woman had a left modified radical 17. A 57-year-old man complains of worsening mastectomy for intraductal carcinoma 2 years pre- headache, nausea, and vomiting for 2 months. viously. She presents with confusion, lethargy, and On examination, he is lethargic, confused, and thigh pain. X-rays reveal a lytic lesion in the shaft of has right-sided weakness. While waiting for a the femur. computed tomography (CT) scan, he develops status epilepticus, suffers cardiorespiratory 19. Which of the following blood abnormalities is arrest, and dies. His brain at autopsy is shown most likely? in Figure 1-4. Which of the following is the (A) high glucose most likely diagnosis? (B) low calcium (A) glioma (C) high potassium (B) meningioma (D) high calcium (C) craniopharyngioma (E) low magnesium (D) pituitary adenoma (E) acoustic neuroma

FIG. 1-4 (c) ketabton.com: The Digital Library

Questions: 16–25 5

20. Which of the following is the most appropriate healthy and has no risk factors for CAD. Which initial therapy? of the following statements is correct? (A) radiotherapy to the femur (A) Hypertriglyceridemia is a strong inde- (B) vigorous saline infusion pendent risk factor for premature CAD. (C) tamoxifen (B) Dietary modification is usually sufficient. (D) chemotherapy (C) High triglyceride levels are associated (E) glucocorticoids with elevated high-density lipoprotein (HDL) levels. 21. A 55-year-old retired policeman has had hyper- (D) Hypertriglyceridemia is usually associ- tension for about 15 years for which he takes ated with skin lesions. hydralazine. He has a 35 pack-year tobacco his- (E) Control of triglyceride levels can pre- tory and continues to smoke one pack a day. vent attacks of acute pancreatitis in On his visit, he complains about the appear- patients with extreme hypertriglyc- ance of his nose (Figure 1-5) and asks if some- eridemia. thing can be done to decrease the redness. Which of the following statements is correct? 23. A 60-year-old patient with long-standing dia- betes has a creatinine of 3.6, which has been stable for several years. Which of the following antibiotics requires the most dosage modifica- tion in chronic renal failure? (A) tetracycline (B) gentamicin (C) erythromycin (D) nafcillin (E) chloramphenicol

24. A 57-year-old man is on maintenance hemodialysis for chronic renal failure. Which of the following metabolic derangements can be anticipated? (A) hypercalcemia FIG. 1-5 (Reproduced, with permission, from Hurwitz RM, Hood AF. Pathology of the Skin. Stamford, CT: Appleton & Lange, 1998:307.) (B) hypophosphatemia (C) osteomalacia (D) vitamin D excess (A) Hydralazine does not play a role in his nasal erythema. (E) hypoparathyroidism (B) Smoking probably aggravates the dilata- 25. Which of the following is a degenerative dis- tion of the blood vessels on his nose. ease of the central nervous system (CNS) (C) He should avoid alcohol and spicy caused by infectious proteins called prions? foods. (D) There is no effective topical therapy. (A) Creutzfeldt-Jakob disease (CJD) (E) Laser therapy will worsen the erythema. (B) Alzheimer’s disease (C) Parkinson’s disease 22. A 46-year-old attorney is noted to have normal (D) Cushing disease cholesterol levels but a very high fasting (E) Guillain-Barré syndrome triglyceride level of 1600. He is otherwise (c) ketabton.com: The Digital Library

6 1: Internal Medicine

26. A 25-year-old man was admitted to the inten- 29. A 20-year-old female presents to the office com- sive care unit with a severe head injury, with plaining that her right eye has been itchy and fracture of the base of the skull. Approximately watery. The patient reports that the onset was 18 hours after the injury, he developed abrupt. The patient is noted to be afebrile with polyuria. Urine osmolality was 150 mOsm/L normal vital signs. Examination discloses a red and serum osmolality was 350 mOsm/L. IV eye with watery discharge. Minimal preauric- fluids were stopped, and 3 hours later, urine ular adenopathy is also found on examination. output and urine osmolality remained Tonometry is normal. Profuse tearing is noted unchanged. Five units of vasopressin were (Figure 1-6). Which of the following is the most intravenously administered. Urine osmolality likely diagnosis? increased to 300 mOsm/L. Which of the fol- lowing is the most likely diagnosis? (A) central diabetes insipidus (B) nephrogenic diabetes insipidus (C) water intoxication (D) solute overload (E) syndrome of inappropriate antidiuretic hormone secretion (SIADH)

27. A 70-year-old man with a 60 pack-year smoking history presents with cough and weight loss. He describes recent diffuse darkening of his skin and his CXR shows a mass suspicious for lung cancer in the left hilum. His laboratory tests FIG. 1-6 (Reproduced, with permission, from 1994 Managing the reveal hypokalemia. Which of the following is Red Eye: A Slide Script Program, San Francisco, American Academy the most likely histology of his lung cancer? of Ophthalmology, from Jenson HB, Baltimore RS. Pediatric Infectious Diseases. Stamford, CT: Appleton & Lange, 1995.) (A) adenocarcinoma (B) small cell (A) viral conjunctivitis (C) squamous cell (B) bacterial conjunctivitis (D) mesothelioma (C) foreign body reaction (E) glioblastoma (D) allergic conjunctivitis 28. A 47-year-old man is postoperative day number (E) acute open-angle glaucoma 2 after an open cholecystectomy. He becomes short of breath and a medicine consultation is 30. A 22-year-old man complains of low back pain called to evaluate. Vital signs include a tem- and stiffness that is worse on arising and perature of 100°F, pulse rate of 110/min, blood improves with exercise. On examination, he is pressure (BP) of 110/60 mmHg, and respira- found to have limited mobility of the sacroiliac tory rate of 24/min. Blood gas shows a pH of joints and lumbar spine. A serum test for his- tocompatibility antigen HLA-B27 is positive. 7.52, carbon dioxide of 28, PO2 of 58, and cal- culated bicarbonate of 20. What is the primary What is the most common extraskeletal mani- acid-base disorder in this patient? festation of this disease? (A) metabolic acidosis (A) premature cataracts (B) respiratory acidosis (B) splenomegaly (C) metabolic alkalosis (C) acute iritis (D) respiratory alkalosis (D) aortic insufficiency (E) metabolic and respiratory acidosis (E) pulmonary fibrosis (c) ketabton.com: The Digital Library

Questions: 26–32 7

Questions 31 and 32 32. Which of the following is the most appropriate next step in management? A 54-year-old man presents to the emergency department complaining of epigastric discomfort, (A) trial of antacid immediately which began while he was walking his dog after (B) reassurance and arrange outpatient dinner about one-half hour earlier. He has not follow-up received medical care for several years. On exami- (C) arrange for cardiac intensive care bed nation, he is moderately obese and in obvious dis- (D) begin thrombolytic therapy in the emer- comfort and seems restless. His BP is 160/98 gency department mmHg, and his examination is otherwise unre- (E) arrange for urgent echocardiogram markable. His ECG is seen in Figure 1-7.

31. Which of the following is the most likely diag- nosis? (A) gastroesophageal reflux (B) costochondritis (C) pericarditis (D) inferior wall myocardial infarction (E) anterolateral myocardial infarction

I aVR V1 V4

II aVL V2 V5

aVF V3 V6 III

VI

II

V5

FIG. 1-7 (c) ketabton.com: The Digital Library

8 1: Internal Medicine

33. A 59-year-old woman who lives independently 35. A 75-year-old man who developed diabetes and had been healthy, presents to the emer- within the last 6 months was found to be jaun- gency department with cough and fever. She diced. He has remained asymptomatic, except related she was well until 2 days before when for weight loss of about 10 lbs in 6 months. On she noted onset of fever, chills, and cough pro- physical examination, he is found to have a ductive of yellow sputum. On examination, you nontender, globular, right upper quadrant mass note a tired appearing woman with BP of that moves with respiration. A CT scan shows 160/90, pulse of 105, and respiratory rate of 32. enlargement of the head of the pancreas, with You start her on ceftriaxone and azithromycin no filling defects in the liver. What is the most and admit her to the hospital. Which of the fol- likely cause of his painless jaundice? lowing factors is a poor prognostic sign in com- (A) malignant biliary structure munity-acquired pneumonia? (B) carcinoma of the head of the pancreas (A) age less than 60 (C) choledocholithiasis (B) systolic BP = 160 mmHg (D) cirrhosis of the liver (C) leukocytosis = 15,000 (E) pancreatitis (D) respiratory rate = 32 (E) mycoplasma pneumonia infected 36. Which of the following is a useful clue to the diagnosis of Legionella pneumonia? 34. A 32-year-old woman is referred to you by her dermatologist for further evaluation. She devel- (A) diarrhea oped these changes gradually in the last year. (B) rash Her hands are seen in Figure 1-8. What other (C) pedal edema associated disease is most likely? (D) elevated serum glucose (E) photophobia

37. A 60-year-old previously healthy man presents with massive rectal bleeding. Which of the fol- lowing is the most likely diagnosis? (A) diverticulosis of the colon (B) ulcerative colitis (C) external hemorrhoid (D) ischemic colitis (E) carcinoma of the colon

38. A 24-year-old man runs a marathon on an unusually hot and muggy day. Several hours FIG. 1-8 (Reproduced, with permission, from Bondi EE, Jegasothy BV, and Lazarus GS. Dermatology Diagnosis & Therapy. Stamford, later he becomes ill with fever, weakness, and CT: Appleton & Lange, 1991.) painful swollen legs and passes dark brown urine. Which of the following is a common (A) acquired immune deficiency syndrome finding with this disorder? (AIDS) (A) Urine orthotoluidine (Hematest) reac- (B) Addison’s disease tion will be negative. (C) lymphoma (B) Serum will be pink. (D) primary biliary cirrhosis (C) Serum creatine phosphokinase levels (E) Hashimoto’s thyroiditis will be elevated. (c) ketabton.com: The Digital Library

Questions: 33–42 9

(D) Serum haptoglobin levels will be 42. A 73-year-old man has been experiencing elevated. increasing drowsiness and incoherence. He has (E) Serum potassium levels will be lowered. a history of arrhythmias and has fallen twice in the past 2 weeks. There are no focal deficits on 39. While examining a 46-year-old woman, you hear neurologic examination. A contrast CT scan of a diastolic murmur that is increased when the the head is shown in Figure 1-9. Which of the patient is in the left lateral decubitus position. following is the treatment of choice? You ask her to run in place for 3 minutes, and the murmur is found to be accentuated as well by exercise. What is the most likely valvular defect? (A) aortic regurgitation (B) mitral stenosis (C) tricuspid stenosis (D) pulmonic regurgitation (E) VSD

Questions 40 and 41

A 70-year-old man presents with shuffling gait, FIG. 1-9 tremor, masked facies, and rigidity which have pro- gressed over the last 9 months. Parkinson’s disease is diagnosed. (A) parenteral antibiotics 40. In this patient, which neurotransmitter defi- (B) antifungal therapy ciency primarily is responsible for his symp- (C) neurosurgical evacuation of the clot toms? (D) observation and a repeat CT scan in 1 month (A) acetylcholine (E) fibrinolytic therapy (B) epinephrine (C) norepinephrine (D) dopamine (E) cortisol

41. Which of the following is not true about Parkinson’s disease? (A) Over 1 million people in North America have Parkinson’s disease. (B) Mortality is higher in patients with Parkinson’s disease when compared to age-matched controls. (C) The classic triad of major signs of Parkinson’s disease is memory loss, rigidity, and akinesia. (D) The tremor in Parkinson’s disease is typically an intention tremor. (E) Over 90% of patients with Parkinson’s disease have a good initial response to levodopa. (c) ketabton.com: The Digital Library

10 1: Internal Medicine

43. A 63-year-old man complains of sudden onset 44. A 54-year-old woman with diabetes is noted to of right-sided headache while at work. He rap- have BP in the range of 140/90 mmHg on sev- idly becomes confused and lethargic. On exam- eral occasions. Which of the following is the ination, he is hemiparetic and has bilateral best next step in management? Babinski signs. A CT scan of the head is shown (A) initiate antihypertensive therapy in Figure 1-10. What is the patient most likely to have? (B) advise weight loss and recheck BP in 3 months (C) advise regular exercise and recheck BP in 3 months (D) no further intervention is necessary (E) follow-up in 6 months for recheck of BP

45. A 44-year-old man undergoes evaluation for worsening headaches. His posteroanterior and lateral arteriograms are shown in Figure 1-11. Which of the following is the patient most likely to develop? (A) hypopituitarism FIG. 1-10 (B) subarachnoid hemorrhage (C) hypercalcemia (D) tentorial herniation (A) an arteriovenous malformation (E) chronic meningitis (B) a carotid occlusion (C) hypertension Questions 46 through 48 (D) an underlying malignancy (E) abnormal clotting studies A 35-year-old pharmacist complains of “hurting all over.” Her pain is particularly bad in her upper back and shoulders, and she notes morning stiffness. On

FIG. 1-11 (c) ketabton.com: The Digital Library

Questions: 43–50 11

examination, her joints are not inflamed, but she has symmetric “tender points” in the posterior neck, anterior chest, lateral buttocks, medial knees, and lateral elbows. You make a preliminary diagnosis of fibromyalgia.

46. Which of the following is another characteris- tic symptom associated with this syndrome? (A) sleep disturbance (B) fever (C) rash on the extremities (D) muscle weakness (E) migratory joint inflammation

47. Which one of the following diagnostic tests should you order? (A) Lyme titers (B) electromyelography (C) sedimentation rate (D) spine radiographs (E) screening test for depression

48. Which of the following is the most appropriate FIG. 1-12 therapeutic recommendation? (A) avoid most physical activity (A) diplopia (B) trial of amoxicillin (B) transient monocular blindness (C) benzodiazepine in low doses for sleep (C) ataxia (D) low-dose steroid (D) vertigo (E) low-dose antidepressant (E) dysarthria

49. A 62-year-old man is undergoing neurologic Questions 50 and 51 evaluation. His arteriogram demonstrates the A 30-year-old woman who has been human lesion shown in Figure 1-12. Which of the fol- immunodeficiency virus (HIV) positive for 4 years lowing deficits is compatible with this lesion? was recently diagnosed with AIDS.

50. Which of the following meets the criteria for the case definition? (A) oral thrush (B) herpes zoster (C) persistent lymphadenopathy (D) peripheral neuropathy (E) pulmonary tuberculosis (c) ketabton.com: The Digital Library

12 1: Internal Medicine

51. Which of the following immunologic abnor- malities would be expected? (A) increased numbers of CD4+ (helper) T cells (B) decreased number of CD8+ (suppressor) T cells (C) cutaneous anergy to usual skin test antigens (D) normal B-cell function (E) increased natural killer cell function

52. When you examine the back of an elderly gen- tleman, you note multiple brown papules and nodules having a “stuck on” appearance. These are shown in Figure 1-13. The patient tells you FIG. 1-13 (Reproduced, with permission, from Hurwitz RM, Hood AF. they have been there for years. Which of the fol- Pathology of the Skin. Stamford, CT: Appleton & Lange, 1998:346.) lowing is the most likely diagnosis? (A) melanocytic nevi Questions 53 and 54 (B) actinic keratoses A 58-year-old man is establishing care with you (C) seborrheic keratoses because his insurance changed. His old records (D) seborrheic dermatitis have not yet arrived, but he is complaining of pal- (E) malignant melanoma pitations and lightheadedness, so you order the ECG shown in Figure 1-14.

I aVR V1 V4

II aVL V2 V5

III aVF V3 V6

VI

II

V5

FIG. 1-14 (c) ketabton.com: The Digital Library

Questions: 51–59 13

53. What is the underlying abnormality? 57. A 62-year-old woman with a long-standing his- tory of diabetes and hypertension presents for (A) right bundle branch block (RBBB) evaluation of hyperkalemia. Her room air arte- (B) left bundle branch block (LBBB) rial blood gas (ABG) and electrolytes are the (C) accelerated junctional rhythm following: (D) left anterior fascicular block pH 7.38/PCO2 34/PO2 89 (E) intraventricular conduction delay Na 140 Cl 106 BUN 51 K 5.9 CO 20 Cr 2.8 54. Which of the following is the most likely prob- 2 lem associated with this pattern? Which of the following is the underlying renal abnormality? (A) congenital heart disease (B) severe aortic valve disease (A) renal tubular acidosis (RTA), type 2 (C) hypokalemia (B) focal segmental glomerulonephritis (D) atrial septal defect (ASD) (C) interstitial nephritis (E) VSD (D) RTA, type 4 (E) Barter syndrome 55. In a patient infected with HIV, which of the following laboratory parameters provides the Questions 58 and 59 most useful information about the current A 72-year-old man has the sudden onset of supra- immunologic status of the patient? pubic pain and oliguria. His temperature is 38.0°C (A) HIV RNA level (100.4°F), pulse is 100/min, respiration rate is (B) white blood cell (WBC) count 12/min, and BP is 110/72 mmHg. Abdominal (C) CD4+ T-cell count examination is remarkable only for a tender, dis- + tended urinary bladder. (D) CD8 T-cell count (E) p24 antigen level 58. Which of the following is the most appropriate immediate management of this patient? 56. During a routine checkup, a 45-year-old execu- tive is found to have hypercalcemia. Subsequent (A) plain x-ray of the abdomen workup reveals elevated parathormone, (B) abdominal ultrasonography decreased phosphorus, elevated chloride, and (C) urethral catheter normal blood urea nitrogen (BUN), and creati- (D) IV furosemide nine in serum. Urinary calcium is above normal levels. What is the most likely etiology? (E) intravenous pyelogram (IVP) (A) multiple myeloma 59. Which of the following is the most likely cause (B) primary hyperparathyroidism of this condition? (C) hypervitaminosis D (D) sarcoidosis (A) urinary tract infection (E) milk alkali syndrome (B) prostatic hypertrophy (C) posterior urethral valves (D) renal carcinoma (E) renal arterial occlusion (c) ketabton.com: The Digital Library

14 1: Internal Medicine

60. A 42-year-old woman is noted to have a 63. A 46-year-old woman presents with a 4-hour multinodular goiter on examination. She has history of left flank pain with fever and chills. no symptoms and is clinically euthyroid. On examination, her temperature is 103°F, pulse rate is 120/min, respiratory rate is Which of the following statements about 40/min, and supine BP is 80/40 mmHg. She Hashimoto’s thyroiditis is true? has marked tenderness over the left flank and left upper quadrant of the abdomen without (A) The condition is associated with prior rebound. Urinalysis shows multiple red blood radioactive exposure. cells (RBCs), multiple WBCs, and WBC casts. (B) Patients diagnosed with this disorder Which of the following is the most likely diag- have an increased incidence of thyroid nosis? cancer. (C) Corticosteroids are helpful in controlling (A) appendicitis the progression of the disease. (B) pyelonephritis (D) Antinuclear antibodies are pathogno- (C) cholelithiasis monic for this disease. (D) diverticulitis (E) Hashimoto’s thyroiditis is an autoim- (E) pelvic inflammatory disease mune disease. DIRECTIONS (Questions 64 through 78): Each set 61. A 55-year-old man complains of severe of matching questions in this section consists of a headaches over the past few weeks. Similar list of lettered options followed by several num- episodes have occurred in past years. Which of bered items. For each item, select the ONE best let- the following supports the diagnosis of cluster tered option that is most closely associated with it. headaches? Each lettered option may be selected once, more than once, or not at all. (A) Pain-free intervals can last for days and then recur. Questions 64 through 66 (B) Attacks of daily pain last for 4–8 weeks. (C) The most common location of pain is For each patient with hepatitis, select the most occipital. likely type of viral hepatitis. (D) Women are affected twice as commonly (A) hepatitis A as men. (B) hepatitis B (E) Caffeine is the most effective treatment (C) hepatitis C for an acute attack. 64. A 45-year-old woman presents with fever, 62. On the second day after an appendectomy, a anorexia, nausea, and diarrhea. Other patients 33-year-old man complains of chest pain. Vital in the community have presented similarly and signs are: temperature 102°F, BP 130/70 have a common exposure at a local restaurant. mmHg, pulse rate 100/min, and respiration rate 22/min. Room air ABG reveals a pH of 65. A 55-year-old male who is a former IV drug 7.50, PCO of 29, and PO of 49. His WBC count 2 2 user presents with jaundice, ascites, and leg is elevated and CXR shows a right lower lobe edema. A CT scan of the abdomen reveals a infiltrate. Which of the following is the most malignant-appearing mass in the liver. likely diagnosis? (A) pulmonary embolism 66. A 43-year-old prison cook becomes ill with (B) myocardial infarction jaundice, malaise, and fever. Shortly there- (C) pneumonia after, multiple prison inmates develop similar symptoms. (D) asthma (E) congestive heart failure (c) ketabton.com: The Digital Library

Questions: 60–80 15

Questions 67 through 73 Questions 74 and 76

For each item, select the ONE best lettered option For each clinical setting described below, select the that is most closely associated with it. Each lettered set of ABG determinations with which it is most heading may be selected once, more than once, or likely to be associated. not at all. pH PaO2 PaCO2 (A) Cushing’s syndrome (A) 7.23 64 80 (B) Addison’s disease (B) 7.39 88 40 (C) Klinefelter syndrome (C) 7.22 74 33 (D) hyperparathyroidism (D) 7.54 75 24 (E) hypothyroidism (E) 7.37 67 52 (F) pheochromocytoma (G) acromegaly 74. A 60-year-old man with morbid obesity (H) diabetes insipidus (I) diabetes mellitus 75. A 30-year-old woman with salicylate intoxica- tion (J) polycystic 76. A 29-year-old diabetic with blood glucose of 67. A 42-year-old obese woman complains of hir- 280 mg/dL sutism, amenorrhea, and difficulty becoming pregnant. Questions 77 through 80

68. A 47-year-old man complains of fatigue and For each antihypertensive agent listed below, select dizziness. On laboratory evaluation, he is noted the set of undesirable side effects with which it is to have significant hyponatremia and hyper- most commonly associated. kalemia. (A) cough, hyperkalemia, angioedema 69. A 55-year-old woman is having episodic pal- (B) positive Coombs test, hemolytic anemia, pitations, headaches, and sweating. On exam- hepatitis ination, her BP is elevated. (C) hypokalemia, hyperuricemia, hyper- glycemia 70. A 48-year-old man is being evaluated for dia- (D) peripheral edema, flushing, and consti- betes and hypertension. On examination, he pation has mandibular enlargement and large hands. (E) increased angina, tachycardia, systemic lupus erythematosus (SLE) 71. A 35-year-old woman complains of constipa- tion, hair loss, and dry skin. 77. hydrochlorothiazide

72. A 45-year-old woman has noticed changes in 78. hydralazine the fat distribution on her body with excess fat over the posterior neck and upper back. On 79. lisinopril examination, she has high BP and abdominal striae. Laboratory evaluation shows a high glu- 80. amlodipine cose intolerance.

73. A 55-year-old man complains of abdominal pain and is found to have a kidney stone. Laboratory evaluation reveals hypercalcemia. (c) ketabton.com: The Digital Library

16 1: Internal Medicine

DIRECTIONS (Questions 81 through 114): Each of (A) hypertension: systolic BP 160 mmHg the numbered items or incomplete statements in (B) hyperreflexia this section is followed by answers or completions (C) fever with temperature 102°F of the statement. Select the ONE lettered answer (D) elevated platelet count 400,000 or completion that is BEST in each case. (E) hypercalcemia

81. A 34-year-old male presents to your clinic with 85. A 45-year-old man with HIV is being evalu- an acute upper respiratory infection (URI). He ated in the clinic. His HIV diagnosis was made has a nonproductive cough and no fever. This 6 months ago and he wants to know more patient is immunocompetent and has no about medication treatment options. Which of underlying heart or lung disease. Which of the the following is an indication to initiate HIV following is the most appropriate treatment? medication treatment? (A) 7 days of a macrolide antibiotic (A) CD4 count less than 700 (B) 7 days of a quinolone antibiotic (B) HIV viral load less than 55,000 (C) 5 days of a macrolide antibiotic (C) CD4 count greater than 700 (D) 5 days of a quinolone antibiotic (D) history of hepatitis A (E) rest and fluids (E) HIV viral load greater than 55,000

82. A 40-year-old patient of yours is planning to 86. A 24-year-old female is infected with HIV from climb Mt. Everest. Which of the following is an unprotected sexual exposure. What is the considered an important risk factor for high- median time for this patient to develop clinical altitude pulmonary edema? disease if she is not treated? (A) warm temperature (A) 6 months (B) history of asthma (B) 1 year (C) rate of ascent (C) 5 years (D) tall stature (D) 10 years (E) rate of descent (E) 15 years

Questions 83 and 84 87. A 52-year-old woman has had diabetes mellitus since childhood. She has controlled her glu- 83. A previously healthy 19-year-old woman has a cose well and kept her glycohemoglobin sudden onset of headache, profound myalgias, (HgbA1C) below 7% (normal, 2–6%). For profuse vomiting, and diarrhea. The woman is which of the following complications is she still near the end of her menstrual period and is at risk, despite excellent glucose control? using tampons. She appears to be suffering from toxic shock syndrome (TSS). Which of the (A) autonomic dysfunction following is the most likely skin finding? (B) coronary heart disease (A) papular rash on the trunk (C) blindness (B) scaly rash on the face (D) peripheral neuropathy (C) pustular rash on the extremities (E) peripheral vascular disease (D) macular erythroderma 88. A middle-aged White male presents to your (E) heliotrope facial rash office complaining of arthralgias, diarrhea, abdominal pain, and weight loss. On exami- 84. Which of the following is another common nation, you note generalized increased skin finding in TSS and is part of the case definition? pigmentation. Which of the following is true regarding Whipple disease? (c) ketabton.com: The Digital Library

Questions: 81–94 17

(A) Acute renal failure is a common compli- (A) Parkinson’s disease cation. (B) anxiety disorder (B) This disease usually strikes young (C) meningioma adults before the third decade. (D) Alzheimer’s disease (C) It is predominantly a disease of women. (E) dysthymia (D) Microscopic examination of duodenal biopsies show extensive periodic acid- Questions 92 through 94 Schiff (PAS) positive material in the lam- ina propria and villous atrophy. 92. A 44-year-old secretary presents with a fever of (E) It is associated with gram-positive 103°F, headache, and stiff neck. You entertain a cocci. diagnosis of bacterial meningitis and begin antibiotics immediately. With bacterial menin- Questions 89 and 90 gitis, which of the following is a likely finding in the cerebrospinal fluid (CSF)? 89. A 42-year-old patient suffering from alcoholism (A) leukocytes between 100 and 500/mm has advanced liver disease with ascites. He is hospitalized for agitation and bizarre behavior. (B) CSF pressure between 100 and 120 mmH O Which of the following findings is most help- 2 ful in making the diagnosis of hepatic (C) negative Gram stain encephalopathy? (D) glucose >120 mg/dL (A) jaundice (E) protein levels >45 mg/dL (B) asterixis of the hands 93. In this otherwise healthy adult woman, what is (C) spider angiomas on the face and chest the most likely infecting organism? (D) heme-positive stool (A) group B Streptococcus (E) positive fluid wave on abdominal exam- ination (B) Staphylococcus aureus (C) Haemophilus influenzae 90. In the patient above, his blood ammonia level (D) Streptococcus pneumoniae is twice his baseline. Which of the following is (E) Listeria monocytogenes a likely precipitating factor? (A) bleeding esophageal varices 94. In the adult neutropenic patient, which of the following is the most likely organism to cause (B) noncompliance with diuretic therapy bacterial meningitis? (C) excessive lactulose therapy (D) insufficient protein ingestion (A) group B Streptococcus (E) recent alcohol ingestion (B) S. aureus (C) H. influenzae 91. A 78-year-old woman comes to your primary (D) S. pneumoniae care office practice with her son who is con- (E) L. monocytogenes cerned about changes in her mood. She is less interested in going out to dinner and does not want to visit family or friends. Her language skills seem to have deteriorated over the last few years and her memory is not as sharp. Her gait and motor strength are normal. Which of the following is the most likely diagnosis? (c) ketabton.com: The Digital Library

18 1: Internal Medicine

95. A 50-year-old woman complains of worsening dyspnea of 1-month duration, but is otherwise asymptomatic. Lung examination is normal; her CXR is shown in Figure 1-15. Which of the following is the most likely diagnosis?

FIG. 1-16

(A) increased arterial carbon dioxide pres-

sure (PaCO2) (B) normal compliance (C) decreased carbon monoxide diffusing FIG. 1-15 capacity (DLCO) (D) increased vital capacity (A) pulmonary tuberculosis (E) increased oxygen saturation with (B) lung metastases exercise (C) sarcoidosis 97. A 23-year-old man presents complaining of (D) mycoplasma pneumonia severe crampy abdominal pain and blood in (E) silicosis his stool over the past 2 days. A similar episode occurred a few months ago and spontaneously 96. A 63-year-old man complains of a new cough resolved. No history of travel. Abdominal x-ray and of breathlessness after walking up a flight shows mild colonic dilatation. Which of the of stairs. Chest examination reveals late inspi- following is the most likely diagnosis? ratory crackles but no wheezes. There is a mild clubbing of the fingers. His CXR is shown in (A) ulcerative colitis Figure 1-16. Which of the following would be (B) viral gastroenteritis found on pulmonary function testing (PFT)? (C) irritable bowel syndrome (D) celiac sprue (E) Whipple disease

98. A 60-year-old man presents with a nonproduc- tive cough for a week and generalized malaise. He also has noted some abdominal pain asso- ciated with diarrhea for the past few days. His (c) ketabton.com: The Digital Library

Questions: 95–104 19

temperature is 101.5°F and clinical examination (C) prior history of radiation exposure is unremarkable. A CXR shows a left lower lobe (D) cryptorchidism infiltrate. His urinalysis shows 50 RBCs, and (E) maternal diethylstilbestrol (DES) during his BUN (30) and creatinine (1.6) are both mildly pregnancy elevated. In light of the extrapulmonary symp- toms and signs, which of the following is the 101. What is the most common cell type in testicu- most likely cause of his pneumonia? lar cancer? (A) Pseudomonas aeruginosa (A) choriocarcinoma (B) S. aureus (B) embryonal cell (C) H. influenzae (C) seminoma (D) S. pneumoniae (D) teratocarcinoma (E) Legionella (E) endodermal sinus

99. A 63-year-old man with chronic bronchitis pres- 102. What serum marker can be used to monitor ents to the emergency department with wors- therapy? ening shortness of breath. He is dyspneic, his respiratory rate is 32/min, and he has periph- (A) carcinoembryonic antigen (CEA) eral cyanosis. A chest examination reveals (B) human chorionic gonadotropin (hCG) increased anteroposterior diameter and scat- (C) sedimentation rate tered rhonchi, but no wheezes or evidence of (D) lactic dehydrogenase (LDH) consolidation. His ABG determinations on (E) prostate-specific antigen (PSA) room air are pH of 7.36, arterial oxygen pressure (PaO ) of 40 mmHg, and PaCO of 47 mmHg. 2 2 103. A 55-year-old man with a 50 pack-year history He is given oxygen by face mask while await- of smoking presents with hemoptysis. CXR ing a CXR. His respiratory rate falls to 12/min, shows a left upper lobe mass and laboratory but his ABGs on oxygen are now pH of 7.31, evaluation reveals hypercalcemia. Which of the PaO of 62 mmHg, and PaCO of 58 mmHg. 2 2 following is the most likely diagnosis? Which of the following is the most appropriate next step in the management of this patient? (A) small cell lung cancer (B) tuberculosis (A) repeat the ABG (C) squamous cell lung cancer (B) initiate mechanical ventilation (D) adenocarcinoma of the lung (C) obtain a CXR (E) metastatic testicular cancer (D) check the oxygen delivery system (E) decrease the fraction of inspired oxygen 104. A 25-year-old man has the sudden onset of (FIO ) 2 chest pain on the right side and dyspnea. On CXR, his trachea is deviated to the left. Which Questions 100 through 102 of the following should be anticipated on 100. A 26-year-old man presents with a hard, pain- examination? less testicular mass. At operation, frozen sec- (A) rales on the left tion reveals testicular cancer. Which of the (B) rales on the right following is a risk factor? (C) hyperresonance on the left (A) family history of testicular cancer (D) distant breath sounds on the right (B) masturbation (E) pleural friction rub on the left (c) ketabton.com: The Digital Library

20 1: Internal Medicine

105. A 65-year-old woman with a long history of CVP = central venous pressure, CO = cardiac uncontrolled hypertension and valvular heart output, SVR = systemic vascular resistance,

disease presents for evaluation. She is fatigued SVO2 = venous O2 saturation and complains of swelling in her legs and short- ness of breath. Which of the following is the dis- 109. A 35-year-old man presents with acute low tinguishing feature of left ventricular failure? back pain after lifting a couch in his home. Pain is in the lumbosacral area and increases with (A) elevated liver enzymes walking and bending. Examination reveals (B) pulmonary edema paraspinal muscle spasm and tenderness and (C) ascites negative straight leg raise bilaterally. Lower (D) peripheral edema extremity strength is intact. Which of the fol- (E) jugular venous distention lowing is the best next step in managing this patient? Questions 106 and 107 (A) bed rest for 1 week A 42-year-old man admitted with a high fever and (B) referral to an orthopedic specialist leukocytosis is transferred to the intensive care unit (C) x-ray of lumbosacral spine in shock. (D) treatment with anti-inflammatory med- ication and gradual return to normal 106. Which of the following is a common finding in activity the early stages of septic shock? (E) referral to a pain clinic (A) reduced cardiac output (B) bradycardia 110. A 54-year-old woman is brought to the ER with palpitations and dizziness. She has a history (C) decreased systemic vascular resistance of arrhythmia. Adenosine is given and the (SVR) patient converts to a sinus rhythm. With which (D) hypertension of the following rhythms did this patient most (E) metabolic alkalosis likely present to the ER?

107. Which of the following is an appropriate initial (A) ventricular tachycardia therapy for both septic shock and cardiogenic (B) atrial fibrillation shock? (C) atrial flutter (A) prophylactic antibiotics (D) paroxysmal supraventricular tachycardia (B) beta blockers (E) ventricular fibrillation (C) volume resuscitation with crystalloid 111. A 48-year-old man complains of fatigue and fluids shortness of breath. His hematocrit is 32% and (D) mechanical ventilation hemoglobin is 10.3 g/100 mL. Peripheral blood (E) diuretics smear reveals macrocytosis. His serum vita-

min B12 level is 90 pg/mL (normal, 170–940); 108. Which of the following patterns is most con- serum folate level is 6 ng/mL (normal, 2–14). sistent with the physiologic profile of cardio- Which of the following is the most likely cause genic shock? of this patient’s symptoms? (A) poor dietary habits CVP CO SVR SVO2 (B) colonic diverticulosis (A) ↓↓↑ ↓ (C) regional enteritis (B) ↑↓↑ ↓ (D) chronic constipation (C) ↓↑↓ ↑ (E) vagotomy (D) ↓↓↓ ↓ (c) ketabton.com: The Digital Library

Questions: 105–116 21

112. A 62-year-old man presents with weakness and (A) chronic pulmonary emboli aching in his hips and shoulders which has (B) obstructive lung disease progressed over the last few months. He (C) adult-onset asthma reports generalized fatigue and malaise. (D) tuberculosis Workup includes a normal complete blood count, kidney and liver tests, and a sedimenta- (E) interstitial lung disease (ILD) tion rate of 102. Which of the following is the most likely diagnosis? 115. A 24-year-old man is found to be seropositive for HIV on a military induction screening test. (A) SLE Which of the following opportunistic infections (B) diabetes is most likely to develop in this patient? (C) Wegener’s granulomatosis (A) Pneumocystis carinii pneumonia (D) polymyalgia rheumatica (B) Candida albicans fungemia (E) Graves’ disease (C) disseminated Mycobacterium avium- intracellulare infection 113. A 33-year-old woman complains of general- (D) cryptococcal meningitis ized, throbbing headache that is worse in the morning and with coughing. She occasionally (E) cytomegalovirus retinitis feels dizzy and nauseated. Examination is sig- nificant only for obesity and bilateral 116. A 56-year-old man complains of fatigue, dysp- papilledema. A CT scan of the head is normal. nea on exertion, and palpitations. He has had At lumbar puncture, the opening pressure is a murmur since childhood. Examination reveals a lift at the left sternal border, split S , 220 mmH O; CSF is clear, with protein of 12 1 2 and fixed splitting of S . There is a grade 3/6 mg/100 mL (normal, 15–45), glucose of 68 2 mg/100 mL (normal, 45–80), and no cells are midsystolic pulmonic murmur and a 1/6 mid- seen. Which of the following is the most likely diastolic tricuspid murmur at the lower left diagnosis? sternal border. CXR shows right ventricular enlargement and prominent pulmonary arter- (A) migraine headache ies. An ECG demonstrates atrial fibrillation (B) multiple sclerosis with a RBBB. Which of the following is the (C) malignant carcinomatosis most likely diagnosis? (D) pseudotumor cerebri (A) coarctation of the aorta (E) glaucoma (B) ASD (C) patent ductus arteriosus 114. A 55-year-old man is being evaluated for gradu- (D) tetralogy of Fallot ally increasing shortness of breath. He does not smoke and does not have any significant envi- (E) VSD ronmental exposures. Cardiac workup is unre- markable and pulmonary function tests reveal decreased lung volumes and decreased total lung capacity (TLC) and vital capacity. Which of the following is the most likely diagnosis? (c) ketabton.com: The Digital Library

22 1: Internal Medicine

DIRECTIONS (Questions 117 through 122): For (normal, 135–148). Serum osmolality is 260, each item, select the ONE best lettered option that urine osmolality is 450, and urine sodium is is most closely associated with it. Each lettered 80. Which of the following is the most likely option may be selected once, more than once, or cause of this patient’s lethargy? not at all. (A) hyperglycemia (A) rheumatoid arthritis (B) hyperlipidemia (B) SLE (C) hyperproteinemia (C) Wegener’s granulomatosus (D) SIADH (D) polyarteritis nodosa (E) diabetes insipidus (E) Goodpasture syndrome 124. Which of the following is the most common (F) fibromyalgia cause of nephrotic syndrome? (G) osteoarthritis (OA) (A) diabetes mellitus (H) giant cell arteritis (B) Hodgkin’s lymphoma (I) sarcoidosis (C) heroin abuse 117. A 70-year-old female with headaches and (D) malignant hypertension fevers notices visual changes. Her erythrocyte (E) renal failure sedimentation rate (ESR) is 125. Questions 125 and 126 118. A 40-year-old male with purulent, bloody sinus drainage, hemoptysis, and rising creatinine. 125. A 33-year-old woman experiences visions of flashing lights followed by throbbing left-sided 119. A 32-year-old White female with anemia, temporal pain and nausea. Which of the fol- arthralgia, alopecia, and RBC casts on urinalysis. lowing is the most likely diagnosis? (A) tension headache 120. A 38-year-old African American female with (B) transient ischemic attack (TIA) shortness of breath and bilateral hilar adenopa- (C) temporal arteritis thy on CXR. (D) migraine headache 121. A 45-year-old female with pain and visible (E) cluster headache swelling in her metacarpal-phalangeal joints bilaterally. Her distal interphalangeal joints 126. This patient is most likely to benefit from acute (DIPs) appear normal. treatment with which of the following sub- stances? 122. A 60-year-old obese male with Heberden’s (A) propranolol nodes on his hands and chronic, severe left- (B) prednisone sided knee pain. (C) sumatriptan DIRECTIONS (Questions 123 through 144): Each (D) heparin of the numbered items or incomplete statements (E) oxygen in this section is followed by answers or comple- tions of the statement. Select the ONE lettered Questions 127 and 128 answer or completion that is BEST in each case. A 27-year-old female complains of dysuria and uri- nary frequency. Urinalysis reveals 10–20 WBCs per 123. A 92-year-old man is referred from his nursing high-power field and numerous gram-negative bac- home for evaluation of lethargy. Examination is teria. She denies fevers, chills, and has no flank pain unrevealing, but laboratory results are signifi- or tenderness. cant for a serum sodium level of 118 meq/L (c) ketabton.com: The Digital Library

Questions: 117–132 23

127. Which of the following statements concerning (A) The majority of renal stones are radiolu- urinary tract infections is true? cent. (A) A single dose of an antibiotic may be (B) Radiolucent stones are usually com- sufficient treatment. posed of uric acid. (B) Pregnant women with bacteriuria (C) Staghorn calculi are associated with acid should not be treated if asymptomatic. urine. (C) Patients with flank pain or fever should (D) Radiopaque stones usually contain cys- be hospitalized. tine. (D) Hematuria indicates renal involvement. (E) Urate stones are associated with alkaline urine. (E) Urologic investigation is indicated after the treatment course is completed. 131. The patient spontaneously passes the stone, which is found to contain calcium oxalate. 128. Which of the following bacteria is most likely Which of the following is the most likely cause responsible for this patient’s urinary tract infec- of this stone? tion? (A) chronic urinary tract infection (A) Klebsiella (B) vitamin D excess (B) Chlamydia (C) primary hyperparathyroidism (C) Escherichia coli (D) idiopathic hypercalciuria (D) Pseudomonas (E) RTA (E) Candida 132. A 30-year-old woman comes to your office for 129. A 49-year-old woman with a history of evaluation of fatigue and shortness of breath on migraine headaches reports 6 days of persistent exertion. Past medical history is unremarkable. headache, nausea, and recurrent vomiting. On Physical examination is remarkable only for examination, the patient is orthostatic. mild pallor. Lung and cardiovascular exami- Electrolytes show a bicarbonate of 42 and a nation are normal. Laboratory tests show a blood gas is obtained revealing a pH of 7.53, hematocrit of 28 with a mean corpuscular carbon dioxide of 53, and PO of 85. What is the 2 volume of 72. WBC count and platelet count underlying acid-base abnormality? are normal. On taking further history from the (A) metabolic acidosis patient, which of the following patient ques- (B) metabolic alkalosis tions would most likely confirm a diagnosis? (C) respiratory acidosis (A) What is your family history of colon (D) respiratory alkalosis cancer? (E) respiratory alkalosis and metabolic (B) What is your family history of heart acidosis disease? (C) How much alcohol do you drink? Questions 130 and 131 (D) Do you have attacks of pain in your 130. A 28-year-old man has the acute onset of col- joints? icky pain in the left costovertebral angle radi- (E) How heavy are your menstrual periods? ating into the groin, as well as gross hematuria. Abdominal x-ray discloses a stone in the left ureter. Which of the following is true concern- ing this disease? (c) ketabton.com: The Digital Library

24 1: Internal Medicine

133. A 54-year-old man complains of cough, (C) metabolic acidosis shortness of breath, and pleuritic left-sided (D) metabolic alkalosis chest pain. Examination and CXR are com- patible with a large left-sided pleural effu- Questions 136 and 137 sion. At thoracentesis, the pleural fluid is straw colored and slightly turbid, with a WBC A 17-year-old girl notes an enlarging lump in her count of 53,000/mL, RBC count of 1200/mL, neck. On examination, her thyroid gland is twice glucose of 42 mg/100 mL, total protein of the normal size, firm to rubbery, multilobular, non- 5 g/100 mL, LDH of 418 IU/L, and pH of 7.2. tender, and freely mobile. There is no adenopathy. Simultaneous serum total protein is 8 g/100 Family history is positive for both hypo- and hyper- mL (normal, 6–8 g/100 mL), and serum LDH thyroidism. Her serum triiodothyronine (T3) and level is 497 IU/L (normal, 52–149 IU/L). thyroxine (T4) levels are low normal, and serum Gram stain is positive for gram-negative rods. thyroid-stimulating hormone (TSH) is high normal. Which of the following is the most likely Technetium scan shows nonuniform uptake. Serum cause of his pleural effusion? and antithyroglobulin titer is strongly positive.

(A) parapneumonic effusion 136. What will thyroid biopsy of this patient most (B) congestive heart failure likely disclose? (C) malignant effusion (A) giant cell granulomas and necrosis (D) trauma (B) polymorphonuclear cells and bacteria (E) nephrotic syndrome (C) diffuse fibrous replacement 134. A young woman with a history of seizures has (D) lymphocytic infiltration a series of grand mal seizures in the emergency (E) parafollicular cells room. She is lethargic and has a nonfocal neu- rologic examination. Her blood gas reveals a 137. Which of the following is the most appropriate treatment for this patient? pH of 7.12, carbon dioxide of 48, PO2 of 86, and calculated bicarbonate of 16. How would you (A) corticosteroids best characterize her underlying acid-base (B) antibiotics problem? (C) thyroid hormone (A) respiratory acidosis (D) radioactive iodine (B) metabolic and respiratory acidosis (E) surgery (C) metabolic acidosis and respiratory alka- losis 138. An obese 21-year-old woman complains of (D) metabolic alkalosis and respiratory increased growth of coarse hair on her lip, chin, acidosis chest, and abdomen. She also notes menstrual (E) metabolic acidosis irregularity with periods of amenorrhea. Which of the following is the most likely cause of this 135. A 43-year-old man with AIDS complains of patient’s symptoms? shortness of breath and worsening diarrhea. (A) polycystic ovary disease His temperature is 98°F, respiration rate is (B) an ovarian tumor 26/min, pulse rate is 100/min, and BP is 100/70 (C) an adrenal tumor mmHg. His lung and heart examination are unremarkable. A room air ABG reveals: pH (D) Cushing disease (E) familial hirsutism 7.10/PCO2 5/PO2 130/calculated bicarbonate 6. What is the primary acid-based disorder? (A) respiratory acidosis (B) respiratory alkalosis (c) ketabton.com: The Digital Library

Answers and Explanations

1. (B) Although locally aggressive and destructive, they rarely metastasize. Figure 1-2 shows a 2. (A) large ulcer with a rodent-like appearance with nodules at the border. Squamous cell carci- 3. (E) noma would be in the differential, but it does not ulcerate as often and is characterized by Explanations 1 through 3 being hard nodules. Malignant melanomas are usually nodular and pigmented. It would be The heliotrope, purple periorbital rash is seen highly unusual for a benign nevus to ulcerate with dermatomyositis and may even precede and have this appearance. A hemangioma is a the muscle involvement. On examination, these red vascular lesion. (Kasper et al., 2005, pp. 497–498) patients will usually show proximal muscle weakness and may complain of difficulty get- 6. (C) ting up from a chair, climbing stairs, and raising the arms over the head. Ataxia may be present 7. (A) with cerebellar lesions. Deep tendon reflexes should be normal and there is no joint inflam- 8. (B) mation. Polymyalgia rheumatica generally occurs in older people but is not associated with Explanations 6 through 8 muscle weakness. Spinocerebellar degenera- tion, vasculitis, and rheumatoid arthritis are not Guidelines for antibiotic prophylaxis of infec- associated with this rash. Creatine phosphoki- tive endocarditis (IE) underwent a major nase is usually markedly elevated and muscle revision in 2007. Prophylaxis is now only rec- biopsy will confirm the diagnosis. Serum crea- ommended for those patients at highest risk of tinine, sodium, and potassium should be IE including patients with a prosthetic valve, normal, and the rheumatoid factor should not history of IE, cardiac transplant patients that be elevated. (Kasper et al., 2005, pp. 2540–2544) develop valvulopathy, cyanotic congenital heart disease that remains unrepaired, cyanotic con- 4. (A) Ciprofloxacin is the drug of choice in a dose genital heart disease that has been repaired with of 500 mg bid for 1–3 days because most cases of a prosthesis during the first 6 months after the travelers’ diarrhea are from E. coli. This patient’s procedure or if a defect remains at the site of the symptoms are moderate to severe and warrant prosthesis after 6 months. Congestive heart fail-

antibiotic treatment which will decrease the fre- ure, an S4 gallop, and diabetes do not increase quency of bowel movements and duration of risk. Recommended antibiotic coverage for illness. Erythromycin and tetracycline are effec- high-risk patients before dental procedures is tive for Vibrio which is an uncommon cause of amoxicillin 2 g PO 1 hour before the procedures. travelers’ diarrhea. Metronidazole is used for Penicillin-allergic patients can receive clar- Clostridium difficile enteritis. (Kasper et al., 2005, ithromycin, cephalexin, cefadroxil, or clin- pp. 727–728) damycin as prophylaxis. Streptococci and S. aureus are responsible for the majority of commu- 5. (D) Basal cell carcinoma is the most common nity-acquired native valve endocarditis cases. form of skin cancer and can present as an iso- In IV drug abusers, S. aureus is responsible for lated papule or nodule or with ulceration. more than 50% of cases, and Candida and

25 (c) ketabton.com: The Digital Library

26 1: Internal Medicine

Pseudomonas for about 6% each. Patients with VSD, pulmonary hypertension, and CAD are not prior endocarditis are at high risk. Bacterial increased with Marfan syndrome. Echocardio- endocarditis carries a mortality rate of about graphy should be performed to follow the course 25%, and prevention is of paramount impor- of the heart. The other tests will not reveal aortic tance. In S. aureus endocarditis in injection drug root dilatation or aneurysm formation. (Kasper et al., users, mortality is only 10–15%. As many as 40% 2005, pp. 2329–2330) of cases occur without underlying heart disease. VSD, patent ductus arteriosus, and tetralogy of 13. (B) Screening should begin approximately 10 Fallot are most commonly associated; whereas, years before the age of diagnosis of colorectal ASD is rarely a predisposing factor. (Kasper et al., cancer in a first-degree (parent or sibling) rela- 2005, pp. 731–740) tive. Given that this patient’s mother was diag- nosed at age 50, this patient should start 9. (A) The Clinical Practice Guidelines of the screening at age 40. The natural history of a colon American College of Physicians (endorsed by polyp to develop into cancer is thought to be 10 Centers for Disease Control [CDC], American years. Colorectal cancer screening has proven Academy of Family Physicians, and the mortality benefit. (Smith et al., 2006, pp. 27–43) Infectious Diseases Society of America) state that most cases of acute sinusitis in the ambulatory 14. (B) The presence of effusion generally signifies setting are due to viral infection and do not significant disease. Aspiration of the effusion require antibiotic treatment or radiography. will help in evaluation for hemarthrosis, septic Therapy should be targeted toward symptoms arthritis, and inflammatory crystal disease. Each and often include decongestants, antipyretics, of these is important to identify and treat early. and analgesics. Antibiotic treatment should be An MRI and/or arthroscopy would be later con- reserved for patients who have symptoms siderations. Orthopedic referral likely would be that persist or worsen (generally for more necessary. (Ferrari and Bach, 1998, pp. 52–63) than 7 days). (Piccirillo, 2004, pp. 902–910) 15. (E) The x-ray in Figure 1-3 shows a large, pleu- 10. (B) ral-based tumor in the left upper chest; this is most likely a mesothelioma. The tumor is 11. (D) locally invasive, so there are no signs of extrathoracic disease. Direct exposure or con- 12. (E) tact with asbestos is not required-tumors have occurred in families of asbestos workers. The Explanations 10 through 12 exposure may be brief and mild, and there is Marfan syndrome is an inherited disorder of con- typically a long latent period before appear- nective tissue, but at least one-fourth of patients do ance of the tumor, about 20–40 years. Surgery not have an affected parent. Abnormal metabo- is curative in local cases. Diffuse malignant lism of collagen or elastin is suspected as the mesothelioma responds poorly to all treatments cause. Clinical features involve the eyes (upward (surgery, radiotherapy, and chemotherapy). subluxation of the lenses, myopia), cardiovascu- (Kasper et al., 2005, p. 1523, 1567) lar system (aortic dilation, regurgitation, and aneurysms; mitral valve prolapse), and the skele- 16. (A) Mitral valve prolapse can be diagnosed by ton (arachnodactyly, pectus deformity, joint auscultation and echocardiogram in as much as laxity). Mental retardation and malar rash are 10% of the population. They may be asympto- not associated with Marfan. A frequent finding is matic or complain of atypical chest pain, palpi- increased length of the limbs as compared with tation, shortness of breath, or weakness. An the trunk. Aortic root dilatation is a serious increasing number of complications are being complication that can lead to aortic regurgita- recognized. Although they occur infrequently, tion, dissection, and even rupture. Mitral valve they may be life threatening and demand care- prolapse is also seen, but pulmonary stenosis, ful evaluation of individuals at risk. Both (c) ketabton.com: The Digital Library

Answers: 9–22 27

supraventricular and ventricular arrhythmias 19. (D) occur, as may sudden death. Mitral insufficiency, if present, is usually insignificant but may 20. (B) progress and require valve replacement. There is an increased risk of infective endocarditis. Intra- Explanations 19 and 20 atrial thrombus formation may occur, predispos- ing to cerebral and peripheral embolism. Because Hypercalcemia is a common complication of the clot originates in the left atrium, however, pul- malignancy. Mechanisms include bone metas- monary embolism does not occur more frequently tases, humoral secretion (e.g., osteoclast-acti- vating factor), prostaglandin, or ectopic in these patients. (Kasper et al., 2005, pp. 1395–1396) parathormone production and immobilization. 17. (A) The autopsy specimen illustrated in Figure 1- Hypercalcemia is often manifested by confu- 4 contains a large, multicolored, irregular tumor sion and lethargy. The other metabolic abnor- invading the left hemisphere. There is hemor- malities usually are not associated with rhage, necrosis, and surrounding edema. The confusion. Therapy is directed at increasing clinical and pathologic findings are most com- renal calcium clearance and inhibiting further patible with a diagnosis of malignant glioma bone resorption. Saline infusion raises the (astrocytoma). Glioma is a highly malignant glomerular filtration rate and decreases cal- tumor of astrocytic cells and is the most common cium reabsorption in the proximal tubule. primary brain tumor. It infiltrates widely, often Under life-threatening circumstances, the infu- involving multiple lobes, as well as the opposite sion may need to be aggressive, as much as 6 L hemisphere via the corpus callosum. Prognosis is of saline daily plus furosemide. Radiotherapy poor, with an average survival time of 6 months will do nothing for the calcium. Tamoxifen is an after diagnosis. Meningiomas are benign primary antiestrogen used in the treatment of breast brain tumors that are usually slow growing and carcinoma and other malignancies. When used occur outside of the hemispheres, where they are in the presence of bone metastases, it may con- well encapsulated and compressed but do not tribute to hypercalcemia. Chemotherapy will invade brain tissue. Craniopharyngiomas arise not decrease the calcium levels. Glucocorticoids from remnants of Rathke’s pouch (the cranio- have an antitumor effect and reduce tumor pro- pharyngeal anlage). They are usually benign, well duction of humoral mediators, but act slowly. encapsulated, and found in or near the sella (Kasper et al., 2005, pp. 2257–2261) turcica. Acoustic neuromas arise from the root of the eighth cranial nerve in the cerebellopontine 21. (C) He should avoid alcohol and spicy foods angle. Like meningiomas, they are encapsulated because these along with the heat, emotional and compressed rather than invade brain stress, and hot temperature foods can aggravate rosacea. Hydralazine is a vasodilator and could substance. (Kasper et al., 2005, pp. 2452–2457) worsen his nasal erythema. Smoking vasocon- 18. (D) The history of severe, self-induced weight stricts rather than dilates blood vessels. loss with an abnormal attitude toward food, Metronidazole gel is an effective topical therapy. weight, and body image in an adolescent female Laser therapy is usually done after the other inter- strongly suggests anorexia nervosa. Common ventions have been tried. (Kasper et al., 2005, p. 295) symptoms are amenorrhea, not menorrhagia or metrorrhagia, constipation, and cold intoler- 22. (E) Hypertriglyceridemia has not been shown to ance. Examination frequently reveals cachexia, be a strong independent risk factor for CAD, hypothermia, bradycardia, hypotension, hyper- however, epidemiologic data do suggest a rela- carotenemic skin, and increased lanugo-like tionship. According to the National Cholesterol body hair. Decreased thyroid and pituitary Education Program, when tirglycerides are above function are evident on laboratory tests, but 200 mg/dL then non-HDL (total HDL) choles- thrombocytopenia and anemia are not terol becomes a pharmacologic treatment target. Severely elevated triglycerides (1000 mg/dL) are common. (Kasper et al., 2005, pp. 430–432) (c) ketabton.com: The Digital Library

28 1: Internal Medicine

a recognized risk factor for attacks of acute pan- and 75 years of age and present with dementia creatitis, and control of the triglycerides can pre- and clonus. The disease is progressive and death vent these attacks. Diet alone is usually not occurs generally within a year of symptom sufficient at these high levels. A National onset. CJD is found throughout the world, and Institutes of Health Consensus Conference has no consistent etiologic agent has been identi- recommended that treatment be initiated in all fied. (Kasper et al., 2005, pp. 2495–2500) patients with triglycerides greater than 500 mg/ 100 mL to prevent acute pancreatitis. Skin lesions 26. (A) Diabetes insipidus, a deficiency of pituitary are not present with hypertriglyceridemia. (Kasper antidiuretic hormone (ADH) (arginine vaso- et al., 2005, pp. 1430–1432) pressin), causes water loss because of failure to facilitate reabsorption of water in the distal 23. (B) Many drugs require dosage modifications tubules and collecting ducts of the kidneys. In in chronic renal insufficiency. Bioavailability, central diabetes insipidus, there is impaired distribution, action, and elimination of drugs production of vasopressin, and in nephrogenic all may be altered. Drugs that are nephrotoxic diabetes insipidus, the distal renal tubules are may be contraindicated or used only with refractory to vasopressin. In central diabetes extreme care in renal insufficiency. The amino- insipidus, urine osmolality remains unchanged. glycosides, vancomycin, ampicillin, most If water intoxication were present, stopping IV cephalosporins, methicillin, penicillin G, sul- fluids should have increased urine osmolality. fonamides, and trimethoprim all should be With solute overload, serum osmolality would given in reduced dosage to patients with chronic have been higher. In SIADH, urine osmolality is renal failure. The aminoglycosides and van- usually higher than serum osmolality. (Kasper comycin can be nephrotoxic and should be used et al., 2005, pp. 251–258, 2098–2100) with caution in renal insufficiency. The small group of antibiotics not needing dosage modifi- 27. (B) Endocrine syndromes are seen in 12% of cation includes chloramphenicol, erythromycin, patients with lung cancer. Squamous cell car- the isoxazolyl penicillins (nafcillin and oxacillin), cinoma is associated with PTH-related peptide. and moxifloxacin. (Kasper et al., 2005, p. 1662, 19) Adrenocorticotrophic hormone (ACTH) and ADH secretion can be associated with small 24. (C) Chronic renal failure treated with hemodial- cell lung carcinoma. ACTH-secreting tumors ysis results in predictable metabolic abnormali- are associated with darkening of the skin and ties. The kidneys fail to excrete phosphate, hypokalemia. (Kasper et al., 2005, pp. 508–509) leading to hyperphosphatemia, and fail to syn-

thesize 1,25(OH)2D3. Vitamin D deficiency causes 28. (D) This patient has an elevated pH (normal is impaired intestinal calcium absorption. 7.40) indicating alkalosis. A low carbon dioxide Phosphate retention, defective intestinal absorp- level is consistent with a respiratory etiology of tion, and skeletal resistance to parathyroid the alkalosis. This occurs when alveolar venti-

hormone (PTH) all result in hypocalcemia. lation is increased relative to CO2 production. Hypocalcemia causes secondary hyperparathy- Causes may include fever, anxiety, pain, pul- roidism, and the excess PTH production worsens monary, and/or neurologic conditions. In a the hyperphosphatemia by increasing phos- metabolic alkalosis, a high bicarbonate is seen phorus release from bone. These derangements (a bicarbonate of 20 is low normal). (Paulson, impair collagen synthesis and maturation, result- 1999, pp. 103–109) ing in skeletal abnormalities collectively referred to as renal osteodystrophy. Osteomalacia, 29. (A) Viral (follicular) conjunctivitis most often osteosclerosis, and osteitis fibrosa cystica may presents with minimal discharge and itching as all be seen. (Kasper et al., 2005, pp. 1656–1657) compared to the moderate-to-profuse dis- charge of bacterial conjunctivitis. While mild 25. (A) CJD is caused by an infectious protein pain and photophobia may be noted in viral, called a prion. Patients typically are between 50 bacterial, fungal, and allergic conjunctivitis, (c) ketabton.com: The Digital Library

Answers: 23–35 29

preauricular adenopathy is common in viral up to 50% when administered within the first and fungal conjunctivitis only. Allergic con- hour of symptoms, so time is of the essence. junctivitis presents with minimal discharge and Arranging for a bed may waste time for limiting marked itching. The patient’s young age and infarct size. The ECG would obviously preclude normal eye pressure (tonometry) helps to rule the other two options: immediate trial of antacid or out glaucoma. (Kasper et al., 2005, p. 166) reassurance and arranging outpatient follow-up. (Kasper et al., 2005, pp. 1316–1318, 1444–1449) 30. (C) The clinical features of the patient described in the question are most compatible with anky- 33. (D) Respiratory rate >30 is a poor prognostic losing spondylitis, an inflammatory arthritis that sign in community-acquired pneumonia. Other occurs most often in young men. Early findings patient factors include age greater than 65 years, of low back pain and stiffness may progress to coexisting illness such as cancer, liver disease, involve the entire spine with straightening (poker congestive heart failure (CHF), renal disease, spine). The most common extraskeletal manifes- systolic BP less than 90 mmHg, temperature tation is acute anterior iridocyclitis, occurring in greater than 40°C. Laboratory findings associ- 25–30% of patients. Additional manifestations, ated with poor prognosis include arterial pH which occur rarely, include heart block, aortitis <7.35, BUN >30, sodium less than 130, glucose with aortic insufficiency, and upper-lobe pul- >250, and hematocrit <30%. These factors are monary fibrosis. Splenomegaly is associated with often used to calculate the PORT (Pneumonia rheumatoid arthritis (Felty syndrome) but is not Outcomes Research Team) pneumonia sever- a feature of ankylosing spondylitis, nor are ity index score which can be an aid in making cataracts. (Kasper et al., 2005, pp. 1993–1995) treatment decisions. S. pneumoniae, Legionella, and S. aureus are the pathogens associated with 31. (D) poor prognosis, not Mycoplasma. (Kasper et al., 2005, pp. 1530–1533) 32. (D) 34. (E) Up to 30% of cases of acquired vitiligo are Explanations 31 and 32 associated with thyroid disease, especially Hashimoto’s thyroiditis. It also may occur with This ECG reveals ST-segment elevation in II, III, pernicious anemia, diabetes, and other autoim- and AVF, indicating acute injury of the inferior mune disorders. Vitiligo has not been reported wall of the myocardium. Inferior wall ischemia with AIDS. Addison’s disease, lymphoma, and can be perceived as pain in the epigastric area. biliary cirrhosis can be associated with hyper- Anterolateral myocardial infarction would pigmentation. (Kasper et al., 2005, pp. 285–301) show loss of R-wave progression in V4 through V . Pericarditis would show diffuse ST seg- 6 35. (B) Adenocarcinoma of the pancreas arises ment elevation in limb and precordial leads. from ductal epithelium. Because of fibrous Although his symptoms could suggest gas- tissue formation, the terminal bile duct troesophageal reflux, this ECG shows this a occludes, causing jaundice. Typically, in the cardiac event. Costochondritis is not present by early stages, the patient is free of pain. With examination. When ST segment elevation is pres- invasion of retroperitoneal structures, the ent, a patient should be considered a candidate patient may sometimes have severe and con- for reperfusion therapy or primary percutaneous stant pain. Often, patients have a history of intervention (PCI) such as angioplasty and stent- weight loss and present with unexplained dia- ing. If no contraindications are present and PCI betes. Because of gradual obstruction, the gall- is unavaliable, thrombolytic therapy should bladder distends, unless it has lost its ideally be initiated within 30 minutes, right in distensibility because of previous scarring. the emergency department. The goal of both Malignant biliary stricture, choledocholithia- thrombolysis and PCI is prompt restoration of sis, and cirrhosis of the liver are ruled out by coronary arterial patency. Thrombolytic ther- the appearance of the CT. Pancreatitis is rarely apy can reduce the risk of in-hospital death by (c) ketabton.com: The Digital Library

30 1: Internal Medicine

associated with jaundice and would be painful. valve, as in exercise. Tricuspid stenosis is heard (Kasper et al., 2005, pp. 537–538) best at the lower left sternal border. Aortic regurgitation is generally reduced by dynamic 36. (A) The spectrum of infection with Legionella exercise due to shortened diastole. A VSD may organisms ranges from asymptomatic sero- be small, and causes a systolic murmur; its conversion to Pontiac fever (a flu-like illness) to murmur will fade with maneuvers favoring for- full-blown pneumonia. Cough is usually non- ward flow, such as vasodilatation with amyl productive initially. Malaise, myalgia, and nitrate. The murmur of aortic stenosis is systolic headache are common. The diagnosis of and will grow louder with increased flow across Legionella infection is suggested by extrapul- the valve, as with amyl nitrate; it will diminish monary signs and symptoms, including diar- with maneuvers that decrease flow across the rhea, abdominal pain, azotemia, and hematuria. valve, as in stage two of the Valsalva maneuver. (Kasper et al., 2005, pp. 870–874) (Kasper et al., 2005, pp. 1307–1311)

37. (A) The causes of lower gastrointestinal (GI) 40. (D) bleeding include hemorrhoids and anal fissure diverticulosis, carcinoma, vascular ectasia, coli- 41. (D) tis, and polyps. Carcinoma of the colon usually causes chronic GI bleeding, resulting in anemia. Explanations 40 and 41 Diverticulosis and vascular ectasia are common causes of massive GI bleeding in the elderly Deficiency of dopamine primarily is responsi- patient. Inflammatory bowel disease can also ble for the signs and symptoms of Parkinson’s cause massive GI bleeding but is more frequent disease. Specifically, the loss of dopamine from in younger age group patients. Most patients the substantia nigra is thought to be primarily with ischemic colitis will be quite sick and will responsible for the akinesia and rigidity. have had symptoms before the onset of bleeding. Tremor, akinesia, and rigidity are the classic (Kasper et al., 2005, pp. 235–238) triad of signs seen in Parkinson’s disease. The tremor typically is a resting tremor; often a “pill 38. (C) The clinical features of the patient described rolling” tremor is seen in the hand. Well over in the question are characteristic of rhabdomy- 90% of patients with Parkinson’s disease do olysis with myoglobinuria. Skeletal muscle have a good initial response to levodopa. (Lang injury releases large amounts of myoglobin into and Lozano, 1998, pp. 1130–1143) the circulation, and myoglobinuria produces a positive orthotoluidine reaction. Because myo- 42. (C) The CT scan shown in Figure 1-9 demon- globin is quickly cleared from serum by the strates a smooth, biconvex lens-shaped mass in kidneys, the serum does not turn pink, as it the periphery of the right temporoparietal region. does with hemoglobinemia. Muscle damage This picture is characteristic of a subdural leads to elevated creatine phosphokinase levels hematoma that is a result of laceration of veins and hyperkalemia. Myoglobin does not bind to bridging the subdural space. Unlike an epidural haptoglobin as does hemoglobin, so serum hematoma, which expands quickly and pro- haptoglobin levels are normal. The major com- gresses rapidly to coma, a subdural hematoma is plication of rhabdomyolysis is acute renal failure. initially limited in size by increased intracranial (Kasper et al., 2005, p. 744) pressure and expands slowly. Symptoms may follow the inciting trauma by several weeks. 39. (B) Heart sounds and murmurs can often be Altered mental status is often more prominent accentuated by various physiologic and phar- than focal signs and may progress from confusion macologic maneuvers. These maneuvers aid in to stupor to coma. Treatment consists of evacua- the differentiation of multiple valvular and tion of the clot via burr holes. Antibiotics and other organic lesions from ordinary sounds. antifungal agents have no role, and fibrinolytic Mitral stenosis is a diastolic murmur that grows therapy or delay in treatment could be harmful. louder with increased flow across the stenotic (Kasper et al., 2005, pp. 2449–2450) (c) ketabton.com: The Digital Library

Answers: 36–51 31

43. (C) The history and physical examination of the Explanations 46 through 48 patient described in the question suggest either an intracerebral hemorrhage or a completed Sleep disturbance is a characteristic symptom ischemic stroke. The CT scan that accompanies associated with fibromyalgia. Patients awaken the question demonstrates a large hemorrhage in feeling tired. The examination, other than ten- the region of the right basal ganglia with a sur- derness in 14 specific, symmetrical points, is rounding zone of edema and narrowing of the usually normal. Fever, rash on the extremities, ventricle. Patients with intracerebral hemorrhage muscle weakness, and migratory joint inflam- often have a preceding history of hypertension. mation point to Lyme disease or other rheuma- Carotid occlusion, malignancy, arteriovenous tologic disorders. A sedimentation rate should malformation, and coagulopathy all are much be normal. If elevated, it may point to another less likely causes of this disorder. In general, only diagnosis. Lyme titers are not indicated unless cerebellar hemorrhages and cerebral hemor- the patient has symptoms or history suggestive rhages that are easily reached are surgically evac- of the disease. Electromyelography and spine uated. Most intracerebral hemorrhages are radiographs are typically normal and unnec- essary for help in establishing the diagnosis. managed with general supportive care. (Kasper et al., 2005, p. 257, 2372) Depression can be associated with pain, but screening for it early on does not make sense 44. (A) The Hypertension Optimal Treatment and might offend the patient. Low-dose anti- Study and the U.K. Prospective Diabetes depressants often help to correct the sleep pat- Study both showed benefit in targeting BP to tern and result in relief of pain. Nonsteroidal the normal range in patients with diabetes anti-inflammatory agents can also be used as (i.e., 130/85 mmHg). This patient has multiple needed; low-dose steroid is not indicated. readings of 140/90 mmHg and should be Exercise is also helpful, and patients should be treated with antihypertensive medication. encouraged to stay physically active. Amoxicillin (Kasper et al., 2005, pp. 1470–1480) is not used for fibromyalgia. Benzodiazepines have addictive potential and lose their effective- 45. (B) The arteriograms in Figure 1-11 demonstrate ness for sleep after a few weeks. (Kasper et al., 2005, a large aneurysm arising from the basilar artery. pp. 2055–2057) Intracranial aneurysms occasionally present with new onset or worsening of headaches or 49. (B) The cerebral arteriogram shown in Figure 1-12 may be asymptomatic and found coincidentally reveals severe stenosis of the common carotid during evaluation of an unrelated disorder. artery proximal to its bifurcation, as well as Frequently, they leak or rupture, resulting in a small lesions in the more distal vessels. subarachnoid hemorrhage with sudden onset Common manifestations are transient monoc- of severe headache and meningeal symptoms ular blindness (amaurosis fugax), hemiparesis, and signs (e.g., nuchal rigidity, photophobia). hemisensory loss, aphasia, and homonymous Rapid progression to stroke, coma, or death may visual field defects. Ataxia would be an unusual follow. Intracranial aneurysms are not usually feature of carotid disease and, if present, would associated with hypercalcemia, hypopituitarism, suggest involvement of the vertebrobasilar or chronic meningitis and rarely cause tentorial arteries, which results in dysarthria, diplopia, herniation without rupturing. Surgical and vertigo. (Kasper et al., 2005, pp. 2383–2384) approaches to intracranial aneurysms include 50. (E) excision and ligation. (Kasper et al., 2005, p. 1481, 86)

46. (A) 51. (C)

47. (C) Explanations 50 and 51 The new case definition for AIDS in 1993 added 48. (E) pulmonary tuberculosis, invasive cervical (c) ketabton.com: The Digital Library

32 1: Internal Medicine

cancer, and recurrent pneumonia. This CDC QRS duration substantially, but are associated classification system is divided into three cate- with shifts in the frontal plane QRS axis (left gories: category A is symptomatic infection axis deviation). With intraventricular conduc- with HIV and includes acute illness and per- tion delay, the QRS is between 100 and 120 ms. sistent lymphadenopathy; category B includes LBBB is a marker of one of four conditions: conditions attributed to HIV infection, such as severe aortic valve disease, ischemic heart dis- oral thrush, herpes zoster, and peripheral neu- ease, long-standing hypertension, and car- ropathy; category C is the AIDS surveillance diomyopathy. RBBB is seen more commonly cases. Anergy to common skin test antigens is than LBBB in patients without structural heart a common finding with HIV infection. There is disease, although RBBB also occurs with con- a decline in CD4 cell numbers, a relative genital heart disease and ASD or valvular heart

increase in the number of T8 cells, which results disease. Hyper- but not hypokalemia may cause in a decreased T4:T8 ratio of less than 1. intraventricular conduction delay. Myocarditis Functional abnormalities occur in both B cells does not usually lead to LBBB. (Kasper et al., 2005, and natural killer cells, which accounts for the pp. 1315–1316) increase in certain bacterial infections seen in + advanced HIV disease. Elevation of beta2 sub- 55. (C) The CD4 T-cell count provides information microglobulin, a serologic finding reflecting on the current immunologic status of a patient immunologic dysfunction, is a fairly reliable infected with HIV. HIV RNA level measures marker of progressive immunologic decline viral load and predicts what will happen to the and the subsequent development of AIDS. CD4+ count in the near future. WBC count is a (Kasper et al., 2005, pp. 1071–1122) nonspecific marker for infection. The p24 anti- gen assay is used for direct detection of HIV. 52. (C) This man has multiple seborrheic keratoses, CD8+ count typically is not as important in which are very common, benign pigmented monitoring immunologic status though the tumors that occur after age 30, especially on CD4+/CD8+ ratio sometimes is used. (Kasper the trunk and face. Melanocytic nevi are usu- et al., 2005, pp. 1071–1122) ally small, circumscribed, pigmented macules or papules, rather than large “stuck-on” nod- 56. (B) Primary hyperparathyroidism is character- ules. Actinic keratoses are red, scaly (not dark) ized by hypercalcemia, hypophosphatemia, lesions on the face and arms that are from sun- hyperchloremia, increased urinary calcium induced damage. Seborrheic dermatitis is a red, excretion, and an increase in serum parathor- scaly rash along the scalp, eyebrows, and mone level. Multiple myeloma is associated with nasolabial folds. Malignant melanoma would hypercalcemia when there are many lytic lesions. be in the differential if it were a single lesion. Chronic ingestion of 50–100 times the normal (Kasper et al., 2005, pp. 285, 497–503) requirement of vitamin D is required to produce hypercalcemia in normal people, so hypervita- 53. (B) minosis D is rare and parathormone levels would be suppressed. With milk alkali syn- 54. (B) drome, which is caused by excess ingestion of calcium and absorbable antacids, parathormone Explanations 53 and 54 levels would also be suppressed. In sarcoidosis, about 10% of patients have hypercalcemia attrib- The prolonged, negative QRS vector anteriorly utable to increased intestinal absorption of cal- (V −V ) and wide notched R waves in V and 1 3 5 cium and increased production of 1,25(OH)2D. V are characteristic for LBBB. In RBBB, there is 6 (Kasper et al., 2005, pp. 2252–2263) an rSR’ complex in V1 and QRS pattern in V6. Accelerated junctional rhythm would not have 57. (D) In type 4 RTA associated with diabetes and P waves. Partial blocks, such as left anterior hypertension, damage to the juxtaglomerular fascicular block, generally do not prolong the apparatus in the glomeruli leads to decreased (c) ketabton.com: The Digital Library

Answers: 52–66 33

renin production. This results in a state of 61. (B) Men are affected by cluster headaches seven hypoaldosteronism, causing hyperkalemia and to eight times more often than women. Attacks a nongap metabolic acidosis. (Kasper et al., 2005, are typically periorbital and may involve the pp. 1699–1700) temporal area. Cluster headaches typically occur daily over a 4- to 8-week period followed 58. (C) by pain-free intervals averaging 1 year. The periodicity of the attacks is often striking, with 59. (B) daily recurrences of pain often at the same hour. Pathogenesis is thought to be due to Explanations 58 and 59 abnormal serotonergic neurotransmission. One hundred percent oxygen inhalation is most Acute oliguria is a medical emergency requir- effective acutely. Drugs used to prevent cluster ing the immediate identification of any cor- attacks include lithium, ergotamine, sodium rectable cause. Distention of the urinary valproate, and prednisone. (Kasper et al., 2005, bladder indicates bladder outlet obstruction. pp. 85–94) Immediate management should be the passage of a urethral catheter to relieve the obstruction 62. (C) Asthma is characterized by bronchospasm and provide urine for examination. An abdom- and typically causes a prolonged expiratory inal flat plate, ultrasonography, or IVP may phase and retention of carbon dioxide. This yield a diagnosis but delay the relief of obstruc- degree of hypoxia (PO2 of 49) generally would tion. Furosemide may be harmful if given not be seen with asthma. Pulmonary embolism, while the bladder is obstructed. Bladder outlet congestive heart failure, myocardial infarction, obstruction may be caused by prostatic hyper- or pneumonia all could lead to this degree of trophy or prostatitis, stones, clots, malignancy, hypoxia. The constellation of fever, elevated or urethral stricture; it may also be neurogenic. white count, and localized infiltrate on CXR is Posterior urethral valves are a congenital defect most characteristic of pneumonia. The most that could cause obstruction in children but likely etiology is perioperative aspiration rarely in adults. Renal carcinoma would not related to general anesthesia. (Kasper et al., 2005, cause outlet obstruction. Renal arterial occlu- pp. 209–210, 1535–1538) sion can cause acute renal failure but not obstructive uropathy. If urethral catheteriza- 63. (B) Detection of leukocyte casts is pathogno- tion fails to relieve the obstruction, further eval- monic for pyelonephritis. Common symptoms uation, including radiographic or ultrasound include fever, shaking chills, nausea, vomiting, studies, is in order. Suprapubic cystostomy and diarrhea. Flank tenderness over the affected may be necessary to empty the bladder. (Kasper kidney is very common. (Kasper et al., 2005, et al., 2005, pp. 1644–1652) pp. 1717–1720)

60. (E) Hashimoto’s thyroiditis, an autoimmune 64. (A) condition, is the leading cause of multinodular goiter in the United States. Although not 65. (C) unique to this condition, antimicrosomal anti- bodies are found in 70–95% of patients. 66. (A) Antinuclear antibodies are associated with SLE. Although an autoimmune process, steroids are Explanations 64 through 66 of no benefit in this condition. One-third of patients experience progressive loss of glan- Hepatitis A is transmitted almost exclusively dular function, and eventually become by the fecal-oral route. Large outbreaks have hypothyroid, but there is no increased inci- been linked to contaminated food products. dence of thyroid cancer. (Kasper et al., 2005, Intrafamily and intrainstitutional spread also is pp. 2109–2113) common. Clinical severity usually is mild, and (c) ketabton.com: The Digital Library

34 1: Internal Medicine

hepatitis A does not progress to chronicity. 71. (E) The most common symptoms seen in Hepatitis C more commonly progresses to patients with hypothyroidism include tired- chronicity (50–70% develop chronic hepatitis ness, weakness, dry skin, feeling cold, hair loss, and 80–90% of these patients have evidence for difficulty concentrating with poor memory, chronic infection). Hepatitis C can lead to cir- constipation, and weight gain. (Kasper et al., 2005, rhosis and hepatocellular carcinoma. Chronicity pp. 2109–2111) occurs in only 1–10% of patients with hepatitis B. (Kasper et al., 2005, pp. 1822–1845) 72. (A) Cushing’s syndrome is caused by overpro- duction of cortisol by the adrenal gland. 67. (J) Polycystic ovarian syndrome is character- Centripetal obesity occurs in 97% of patients, ized by infertility, hirsutism, obesity, and increased body weight in 94%, fatigability and amenorrhea or . These patients weakness in 87%, and hypertension in 82%. have chronic and experience with- Impaired glucose tolerance is common and drawal bleeding after progestogen administra- attributable to increased hepatic gluconeogen- tion. (Kasper et al., 2005, pp. 2204–2205) esis and insulin resistance. (Kasper et al., 2005, pp. 2134–2137) 68. (B) Addison disease, or primary adrenocortical deficiency, commonly results in a low serum 73. (D) In primary hyperparathyroidism, increased Na+, a low serum Cl−, and a high serum K+. levels of PTH lead to hypercalcemia and Aldosterone deficiency causes loss of sodium hypophosphatemia. Patient symptoms and in the urine and contributes to the hyper- signs on presentation include kidney stones, kalemia. The original description of Addison peptic ulcers, mental status changes and less disease summarizes well the key clinical find- commonly, extensive bony resorption. (Kasper ings: general languor and debility, feebleness of et al., 2005, pp. 2252–2256) the heart’s action, irritability of the stomach, and a peculiar change of the color of the skin. 74. (E) The hyperpigmentation of the skin is often seen on the elbows and in the creases of the hands. 75. (D) (Kasper et al., 2005, pp. 2141–2143) 76. (C) 69. (F) Pheochromocytoma is a secondary cause of hypertension. Pheochromocytomas secrete cat- Explanations 74 through 76 echolamines; 80% are unilateral, solitary tumors, most often located in the adrenal gland. Patients ABG determinations are essential in the diag- present with episodes of headache, palpitations, nosis of respiratory and acid-based disturbances. Extremely obese patients suffer from increased sweats, and a sense of apprehension. (Kasper et al., 2005, pp. 2148–2152) work of breathing, as well as elevation of the diaphragm with decrease in lung volume. The 70. (G) Acromegaly is caused by oversecretion of resultant hypoventilation is characterized by growth hormone, usually from a pituitary ade- carbon dioxide retention leading to chronic res- noma. Clinical manifestations are indolent, piratory acidosis with metabolic compensation often not diagnosed for 10 or more years. Acral (ABG set E in the question). When associated bony overgrowth leads to frontal bossing, with somnolence, excessive appetite, and poly- increased hand and foot size, and mandibular cythemia, this is known as the pickwickian syn- enlargement. Growth hormone excess also drome. Modest weight loss can lead to commonly affects the cardiovascular system. dramatic improvement in respiratory func- Upper airway obstruction with sleep apnea is tioning. The earliest derangement in salicylate poisoning is hyperventilation, resulting in present in up to 60% of patients. (Kasper et al., decreased PaCO and increased arterial pH 2005, pp. 2090–2092) 2 (ABG set D). Eventually, there is CNS depression (c) ketabton.com: The Digital Library

Answers: 67–85 35

with somnolence and hypoventilation resulting use of decongestants and nonsteroidal anti- in respiratory acidosis. Diabetic ketoacidosis inflammatory drugs. Other therapies directed may cause acute metabolic acidosis. In a at specific symptoms are often useful, including healthy young adult with no lung disease, dextromethorphan for cough and lozenges appropriate respiratory compensation occurs with topical anesthetic for sore throat. Clinical (ABG set C). Without insulin to reverse this trials of zinc, vitamin C, echinacea, and other process, the patient may go on to develop alternative remedies have revealed no consis- worsening acidosis and an inability to com- tent benefit for the treatment of nonspecific pensate adequately. ABG set A reflects acute URI. (Kasper et al., 2005, p. 185) respiratory acidosis (hypoventilation) without metabolic compensation. ABG set B is normal. 82. (C) Incidence of high-altitude pulmonary edema (Kasper et al., 2005, pp. 263–271) is related to the rate of ascent, altitude reached, and degree of exertion. Cold temperature 77. (C) increases pulmonary artery pressure through sympathetic stimulation and is an independent 78. (E) risk factor. Recurrence rate may be as high as 60% in persons with a previous history of high- 79. (A) altitude pulmonary edema. These individuals have reduced ventilatory response to hypoxia 80. (D) and an exaggerated pulmonary pressor response to exercise and hypoxia. (Kasper et al., 2005, p. 1617) Explanations 77 through 80 83. (D) All of the drugs used to treat hypertension can cause adverse reactions, ranging from 84. (C) trivial to life threatening. Thiazide diuretics are associated with hypokalemia, causing Explanations 83 and 84 arrhythmias; hyperuricemia causing gout; and hyperglycemia due to insulin resistance. Toxin-producing S. aureus organisms have been The vasodilator hydralazine can cause tachy- implicated in the pathogenesis of TSS and are cardia with increased angina and a lupus- frequently cultured from the vagina and like syndrome. As many as 10% of patients of affected women. There is no diagnostic labo- on angiotensin-converting enzyme (ACE) ratory test, and diagnosis is based on the typical inhibitors develop an annoying dry cough. clinical findings. Diffuse macular erythroderma Because they block aldosterone, they can lead (sunburn-like rash) occurs in the first few days to hyperkalemia. ACE inhibitors cause 10–25% of illness, followed by desquamation, usually of all cases of angioedema. Peripheral edema is of the palms and soles 1–2 weeks later. Fever, the most commonly reported side effect of cal- hypotension, and multiorgan-system involve- cium channel blockers, especially amlodipine ment (GI, CNS, muscular, renal, hepatic, hema- and nifedipine. Constipation, flushing, and tologic) are also part of the case definition. dizziness are also frequently reported. Platelet counts are usually reduced below Coombs’-positive hemolytic anemia and hepa- 100,000. Disorientation may occur but without titis are idiosyncratic reactions to the central such focal neurologic signs as hyperreflexia. adrenergic-stimulant methyldopa. (Kasper et al., Complications include shock, arrhythmias, renal 2005, pp. 1472–1478) failure, respiratory failure, and coagulopathy. Hypercalcemia is not a part of the picture. (Kasper 81. (E) Antibiotics have no role in the treatment of et al., 2005, p. 819) uncomplicated nonspecific URI. In the absence of clinical evidence of bacterial infection, treat- 85. (E) Adherence to a drug regimen is critical to ment remains entirely symptom-based with prevent antiretroviral drug resistance. Treatment (c) ketabton.com: The Digital Library

36 1: Internal Medicine

usually should be offered to patients who are Explanations 89 and 90 symptomatic from their HIV infection. Asymptomatic patients should have antiretro- Hepatic encephalopathy is a syndrome of viral therapy offered if their CD4 counts are declining intellectual function, altered state of less than 350 or plasma HIV viral load is greater consciousness, and neurologic abnormalities than 50,000 copies. A high viral load (>100,000 in the setting of advanced liver disease. Other copies) correlates with poor prognosis and an findings include hyperactivity, delirium, agita- increased likelihood of developing opportunis- tion, and personality changes, progressing to confusion, somnolence, and coma. Asterixis tic infections. (Kasper et al., 2005, pp. 1076–1136) (lapses of sustained muscle contraction) or 86. (D) The median time from initial infection with “flapping tremor” is common. Jaundice, spider HIV to the development of clinical disease is 10 angiomas, and ascites can be present in alco- years. The rate of disease progression is highly holic liver disease without the presence of variable and directly correlates with HIV RNA encephalopathy. Precipitating factors must be levels. With high levels of HIV RNA, the dis- looked for and reversed if possible. GI bleeding ease progresses faster. During the asympto- (due to esophageal varices, gastritis, ulcer, and matic period of HIV infection, CD4+ cells so forth) increases the nitrogen load in the gut decline at an average rate of 50 μL per year. and reduces cerebral perfusion. Excessive Some patients do not progress and show little diuresis with prerenal azotemia increases extra- + hepatic circulation of urea and ammonia pro- if any decline in CD4 counts over time. (Kasper et al., 2005, pp. 1076–1105) duction, so noncompliance with diuretics would decrease ammonia levels. Lactulose 87. (B) Diabetes mellitus is associated with hyper- acidifies the stool, traps ammonia and other glycemia, disease of the microvasculature nitrogenous substances, and decreases their (retinopathy, nephropathy, neuropathy), and absorption from the gut so excessive lactulose macrovascular disease, including CAD and would decrease ammonia levels. Excessive pro- peripheral vascular disease, The Diabetes tein intake is a common precipitant. (Kasper et al., Control and Complications Trial and other 2005, pp. 1867–1869) studies have demonstrated that tight control can decrease complications of microvascular 91. (D) Classic features of Alzheimer’s syndrome disease significantly, but it does not seem to include amnestic memory impairment, deteri- reduce CAD mortality to the same extent. oration of language, and visuospatial deficits. (Kasper et al., 2005, pp. 2161–2169) Gait disturbances and motor and sensory changes are uncommon until late phases of the 88. (D) Whipple disease is a systemic illness char- syndrome. Mood change and apathy are com- acterized by arthralgias, diarrhea, abdominal monly seen in early stages of Alzheimer’s syn- pain, and weight loss. The usual patient is a drome and typically continue for the duration middle-aged White male. Reported in 1907 by of the disease. Psychotic features may be seen George Whipple, it has been associated with a in middle and late phases of the syndrome. gram-positive bacillus related to Actinomycetes. (Kasper et al., 2005, pp. 2398–2401) The disease can affect nearly every organ system, although it usually involves the GI 92. (E) The Gram stain is positive in three-fourths tract, heart, and CNS. Renal failure is not a of bacterial meningitis cases. Leukocyte counts average between 5000 and 20,000; CSF pres- common complication. (Kasper et al., 2005, pp. 1774–1775) sure is consistently elevated usually above 180 mmH2O; glucose levels are usually lower than 89. (B) 40 mg/dL, or less than 40% of blood glucose; and protein levels are higher than 45 mg/dL in 90. (A) 90% of cases. (Kasper et al., 2005, pp. 2471–2475) (c) ketabton.com: The Digital Library

Answers: 86–99 37

93. (D) S. pneumoniae is the most common cause of decreased lung volumes (vital capacity, TLC) adult meningitis in people over 30 and and decreased compliance. Loss of the alveolar accounts for about 15% of cases. H. influenzae is capillary bed leads to decreased carbon monox- the most common cause in children over 1 ide diffusing capacity. Arterial oxygen pressure

month old. Group B Streptococcus is an impor- (PaO2) may be normal at rest but is decreased tant cause of neonatal meningitis, but is very with exercise. Arterial carbon dioxide pressure

rare in adults. Staphylococcus, E. coli, and (PaCO2) may be normal or decreased because Klebsiella may be seen with penetrating head of hyperventilation, but it is not usually wounds or postneurosurgical procedures. elevated in pure ILD. (Kasper et al., 2005, (Kasper et al., 2005, pp. 2471–2477) pp. 1554–1560)

94. (E) Although Listeria still represents only a frac- 97. (A) Ulcerative colitis typically presents between tion of total cases (about 10%) of meningitis, it is the ages of 15 and 25 years with symptoms of seen in diabetes and cancer patients, alcoholic, diarrhea with blood and abdominal pain. elderly, and immunocompromised patients. Involvement begins in the rectum and is limited (Kasper et al., 2005, pp. 2471–2477) to the colon. The recurrent episodes and hema- tochezia make inflammatory bowel disease 95. (B) The CXR shown in Figure 1-16 contains most likely. (Kasper et al., 2005, pp. 1776–1789) multiple bilateral pulmonary parenchymal nodules varying in size and shape, most com- 98. (E) The spectrum of infection with Legionella patible with metastatic disease to the lungs. organisms ranges from asymptomatic serocon- Other possibilities are bronchogenic carcinoma version to Pontiac fever (a flu-like illness) to or fungal granulomas (e.g., histoplasmosis or full-blown pneumonia. Cough is usually non- coccidiosis). Sarcoidosis usually presents with productive initially. Malaise, myalgia, and bilateral hilar adenopathy and rarely with mul- headache are common. The diagnosis of tiple pulmonary nodules. Tuberculosis pres- Legionella infection is suggested by extrapul- ents with a cavitating lesion, pleural effusion, monary signs and symptoms, including diar- or miliary pattern. Typical findings in silicosis rhea, abdominal pain, azotemia, and hematuria. are diffuse nodular fibrosis and eggshell calci- (Kasper et al., 2005, pp. 871–874) fication of hilar or bronchopulmonary lymph nodes. The CXR of patients with mycoplasma 99. (E) Patients with advanced chronic obstructive pneumonia usually shows patchy infiltrates pulmonary disease (COPD) are at risk for involving the lower lobes and spreading from development of acute respiratory failure. the hila. The finding of metastatic nodules on Common precipitants are infections, increased CXR should prompt a search for the primary secretions, and superimposed bronchospasm. tumor. (Kasper et al., 2005, p. 562, 1497–1498, 1556) Oxygen therapy is effective in reversing the hypoxemia associated with respiratory failure. 96. (C) The CXR shown in Figure 1-16 shows a dif- A risk of such therapy peculiar to patients with fuse reticulonodular pattern consistent with COPD is worsening hypercapnia. Affected ILD. The hilar nodes are enlarged, suggesting patients are thought to have lost their respira- lymphadenopathy. This is a nonspecific pic- tory center’s sensitivity to hypercapnia, so that ture and may be caused by a large number of their primary stimulus to breathe is hypox- diseases. Occupational exposure to dust, gas, or emia. When the hypoxemia is corrected, they fumes; sarcoidosis; idiopathic pulmonary fibro- may lose their stimulus to breathe and develop sis; and lung disease associated with the rheu- carbon dioxide narcosis with worsening aci- matic diseases are the more common factors. dosis, confusion, stupor, and eventually coma. Despite the diverse causes, there is a common Because of this, the usual approach is to begin

pathogenesis: injury leads to alveolitis, which with a low fraction of inspired oxygen (FIO2) progresses to fibrosis. Abnormalities on PFT are and increase gradually. Serial ABGs are

also similar: restrictive disease characterized by obtained to ensure that as PaO2 improves, (c) ketabton.com: The Digital Library

38 1: Internal Medicine

PaCO2 does not increase. In some cases, even common among patients with lung cancer. the lowest FIO2 causes carbon dioxide reten- Endocrine paraneoplastic syndromes are seen tion, and mechanical ventilation is required. In in 12% of patients. Hypercalcemia may result

the case presented in the question, a lower FIO2 from ectopic production of PTH or PTH-related should be used before mechanical ventilation is peptides by squamous cell carcinomas. Small initiated. (Kasper et al., 2005, pp. 1547–1554) cell cancers may secrete ACTH or excessive amounts of ADH leading to hyponatremia and 100. (D) SIADH. Other nonendocrine paraneoplastic manifestations may include anorexia, cachexia, 101. (C) weight loss, fever, suppressed immunity. peripheral neuropathy, and the myasthenic 102. (B) Eaton-Lambert syndrome. (Kasper et al., 2005, pp. 506–516) Explanations 100 through 102 104. (D) In the patient described in the question, the Testicular cancer is the most common cancer in movement of the trachea to the left suggests a men between the ages of 20 and 40. difference between right and left pleural pres- Predisposing factors include cryptorchidism, sures, either a reduction in pressure on the left hernias, and testicular atrophy. Abdominal or a rise in pressure on the right. The acute onset testes are at higher risk than inguinal cryp- of right-sided chest pain in an otherwise healthy torchid testes. Family history of testicular or young man suggests a pneumothorax. On the prostate cancer, radiation exposure, or maternal side of the pneumothorax, we would expect DES seems to play no role. Testicular cancers increased resonance and distant or absent are divided into nonseminoma and seminoma breath sounds because of the air trapped in the subtypes. Seminoma represents about 50% of pleural space between the lung and chest wall all tumors and generally follows a more indo- and possible compression of the normal lung. lent course. The primary tumor is treated by No rales or rhonchi would be expected. A pleu- inguinal orchiectomy regardless of cell type. ral friction rub suggests an inflammatory Pure seminomas do not require retroperitoneal process involving the left chest, a finding not lymph node dissection, because radiation is likely on the basis of the patient’s presentation. usually adequate therapy. Nonseminomatous (Kasper et al., 2005, pp. 77–81, 1565–1569) testicular tumors (embryonal cell, teratocarci- noma, choriocarcinoma, endodermal sinus) are 105. (B) Physical findings are often helpful in dis- usually treated by retroperitoneal dissection. tinguishing left-sided from right-sided heart Serum alpha-fetoprotein (AFP) and hCG levels failure. When the left ventricle is either over- are markers that are important for diagnosis loaded or weakened, patients develop dyspnea and as prognostic indicators and are used to and orthopnea as a result of pulmonary con- monitor therapy. Serum LDH level is often ele- gestion. When the underlying abnormality is vated with bulky tumors but is not as specific primarily right ventricular failure symptoms as either AFP or hCG. CEA is a nonspecific related to pulmonary congestion are uncommon marker elaborated by many adenocarcinomas. and patients experience edema, venous dis- PSA is a marker associated with prostate tention, and hepatic congestion. Liver enzymes cancer. (Kasper et al., 2005, pp. 550–553) may be elevated secondary to hepatic conges- tion. (Kasper et al., 2005, pp. 1367–1378) 103. (C) Although 5–15% of patients with lung cancer are identified while they are asympto- 106. (C) matic, most patients present with signs and symptoms including cough, hemoptysis, wheeze, 107. (C) stridor, shortness of breath, and postobstructive pneumonia. Paraneoplastic syndromes are (c) ketabton.com: The Digital Library

Answers: 100–113 39

Explanations 106 and 107 110. (D) The majority of paroxysmal supraventric- ular tachycardias respond to adenosine, The usual early hemodynamic response to because they involve a re-entrant circuit includ- sepsis is a hyperdynamic circulation. This ing the atrioventricular node. Adenosine is includes tachycardia, elevated cardiac output, ineffective in the termination of the majority and decreased systemic resistance. Septic shock of other atrial or ventricular tachycardias that may then progress with intractable hypoten- do not involve the AV node, although it may sion, metabolic acidosis, reduced cardiac output, slow the ventricular response to an atrial tachy- oliguria, and death. The initial resuscitation of cardia. (Kasper et al., 2005, p. 1342–1351) patients with all forms of shock requires rapid expansion of circulating blood volume to help 111. (C) The most common causes of megaloblastic maintain BP and tissue perfusion. This is usually anemia are folate and vitamin B deficiencies. achieved with the infusion of crystalloid fluids. 12 Vitamin B12 deficiency rarely results from inad- When septic shock is suspected, cultures of equate intake, but has been associated with blood, urine, and other sources along with strict vegetarianism. Decreased absorption may antibiotic therapy targeted toward the most be due to insufficient intrinsic factor (as in per- likely source is critical. Mechanical ventilation nicious anemia and after gastrectomy), malab- may be required when altered mental status, sorption of the intrinsic factor-vitamin B12 acidosis, and hypoxia are present. Beta-blockers complex in the terminal ileum (as in regional and diuretics may have specific indications that enteritis, sprue, pancreatitis, and after ileec- cardiac ischemia and pulmonary edema are tomy), or competition for vitamin B12 by gut present. (Kasper et al., 2005, pp. 1600–1606) bacteria (as in the blind loop syndrome and Diphyllobothrium latum infections). Because 108. (B) Patients with cardiogenic shock (B) demon- diverticulosis and constipation do not interfere strate a pattern with increased CVP, low CO, with stomach or small-bowel functioning, they increased SVR, and decreased SVO2. Those are not causes of vitamin B12 deficiency. (Kasper with hypovolemic shock (A) demonstrate low et al., 2005, pp. 601–607) CVP, low CO, increased SVR, and decreased SVO . The distinguishing feature of early septic 2 112. (D) Polymyalgia rheumatica is characterized shock (C) is an increased cardiac output. by bilateral aching and stiffness of the proximal Patients with neurogenic shock (D) have severe parts of the arms and thighs with associated loss of vasomotor tone leading to the unusual weakness and generalized fatigue. The sedi- combination of low SVR and low CO. (Kasper mentation rate typically is significantly ele- et al., 2005, pp. 1600–1606) vated. This condition can coexist with the syndrome of temporal arteritis (aka giant cell 109. (D) In younger patients, low back pain tends to arteritis) which is a chronic vasculitis of large- be mild and self-limited, typically resolving in and medium-sized vessels, usually including 4–6 weeks. Patients should be encouraged to cranial branches of the aortic arch arteries. remain active and symptom control can be Common symptoms among patients with over- achieved with pain medications. Low back lapping temporal arteritis are headache and pain is the leading cause of work-related dis- sudden loss of vision. Graves’ disease is an ability in the United States. The absence of autoimmune thyroid disease and Wegner gran- alarm symptoms such as unilateral or bilateral ulomatosis typically involves the sinuses, lungs, leg weakness and bladder, bowel or sexual dys- and/or kidneys. (Kasper et al., 2005, pp. 2008–2009) function makes a cauda equine syndrome or other spinal cord injury unlikely. Age >50, con- 113. (D) Pseudotumor cerebri is a disorder of stant pain at night, history of cancer, unex- increased intracranial pressure that has no plained weight loss, and lack of response to obvious cause. The typical patient is an obese conservative therapy make further investigation young woman who complains of headache and including radiologic evaluation appropriate. is found to have papilledema. Slight decrease in (Kasper et al., 2005, pp. 94–104) (c) ketabton.com: The Digital Library

40 1: Internal Medicine

visual fields and enlargement of blind spots 116. (B) ASD is the second most common form of may also be observed. Neurologic examination congenital heart disease in adults, after a bicus- is otherwise normal, and the patient appears to pid aortic valve. The murmur heard in child- be healthy. CSF is under increased pressure hood is often considered innocent, and and may have slightly low protein concentra- symptoms do not appear until adulthood. A tion, but is otherwise normal. CT scan, arteri- left-to-right shunt of blood between the atria ogram, and other x-ray studies are usually causes right ventricular overload and increased normal. The most serious complication is pulmonary circulation. These result in the clas- severe visual loss, which occurs in about 10% of sic findings of a pulmonic systolic ejection affected persons. Treatment with a carbonic murmur, late pulmonic valve closure with

anhydrase inhibitor decreases intracranial pres- wide splitting of S2, and a tricuspid flow sure by decreasing production of CSF. Weight murmur. CXR has signs of cardiomegaly and loss is important but often unsuccessful. If the pulmonary overcirculation. Characteristic ECG carbonic anhydrase inhibitor and weight loss changes are atrial fibrillation and an incom- fail, or if visual loss develops, lumboperitoneal plete or complete RBBB. In the more common shunting or optic nerve sheath fenestration are ostium secundum type of ASD, there is often important maneuvers to prevent blindness. right axis deviation; whereas, the ostium (Kasper et al., 2005, pp. 169–170) primum type has a left axis deviation pattern. Coarctation of the aorta, patent ductus arterio- 114. (E) ILD is characterized by decreased lung vol- sus, and VSDs are not associated with the find- umes including decreased TLC, forced expira- ings of the patient described in the question,

tory volume in 1 second (FEV1), and forced and tetralogy of Fallot would not present in vital capacity (FVC). Such patients also have adulthood. (Kasper et al., 2005, pp. 1385–1386) reduced diffusing capacity (DLCO) due to alve- olar damage. Causes of ILD include pulmonary 117. (H) Giant cell arteritis (aka temporal arteritis) diseases such as idiopathic pulmonary fibrosis, occurs almost exclusively in patients over 55 connective tissue disorders, and drugs. years of age. The classic clinical findings are fever, Extrapulmonary issues from poor breathing anemia, high ESR, and headache. Other mani- mechanics such as scoliosis or myasthenia festations may include malaise, fatigue, anorexia, gravis may also result in a restrictive pattern on weight loss, sweat, and arthralgias. Headache PFT testing, however DLCO is normal. may be associated with a tender, thickened, or Obstructive lung disease is characterized by nodular temporal artery. Diagnosis often is made

decreased FEV1 and TLC that is normal or clinically and can be confirmed by a temporal increased. (Kasper et al., 2005, pp. 1554–1560) artery biopsy. Patients respond well to glucocor- ticoid therapy. (Kasper et al., 2005, pp. 2008–2009) 115. (A) Although thrush and esophagitis attributable to C. albicans are common manifestations of 118. (C) Wegener granulomatosis affects males and AIDS-related immunodeficiency, fungal dis- females equally, can occur at any age, and is semination and sepsis are extremely rare. extremely rare in African Americans. Patients Although not uncommon, opportunistic infec- typically present with severe upper respiratory tions in these patients, disseminated M. avium- tract symptoms. Nasal perforation and saddle intracellulare, cryptococcal meningitis, and nose deformity may occur. Pulmonary involve- cytomegalovirus retinitis, are less common than ment (cough, hemoptysis, or dyspnea are P. carinii pneumonia. Prior to the HAART (highly common presenting symptoms/signs) is pres- active antiretroviral therapy) era, PCP (phency- ent in 85–90% of patients, renal disease in clidine) was the AIDS defining illness in about 70–80% of patients, eye involvement in 50–55%, 50% of AIDS patients not on PCP prophylaxis. and skin lesions in 40–50%. Antineutrophil While the number of deaths from PCP has cytoplasmic antibodies (C-ANCA) is positive in decreased dramatically, it is still a leading cause 90% of patients. ESR typically is high. (Kasper et al., of death in AIDS. (Kasper et al., 2005, pp. 1104–1120) 2005, pp. 2004–2007) (c) ketabton.com: The Digital Library

Answers: 114–126 41

119. (B) SLE is characterized by tissue and cell hyponatremia are associated with diuretic use, damage from pathogenic autoantibodies and diarrhea, and dehydration. Expanded ECF immune complexes. Ninety percent of patients volume due to decreased effective circulating are women in childbearing years and the disease volume and increased ADH secretion may is more common in African Americans. Multiple result in edema and hyponatremia as often organ system manifestations can occur, includ- seen with congestive heart failure, cirrhosis of ing musculoskeletal (arthralgias, myalgias), the liver, and nephritic syndrome. Euvolemia cutaneous (malar rash, photosensitivity, hair and hyponatremia can be due to hypothy- loss), renal (nephritis, nephritic syndrome), roidism, adrenal insufficiency, and other con- nervous (seizures, headaches), cardiopulmonary ditions associated with the SIADH. Diabetes (pericarditis, pleuritis), hematologic (anemia, insipidus is a cause of hypernatremia. (Kasper leukopenia). (Kasper et al., 2005, pp. 1960–1967) et al., 2005, pp. 254–256, 2102–2104)

120. (I) Sarcoidosis is a relatively common disease. 124. (A) The nephrotic syndrome is characterized In the United States, most patients are African by proteinuria of greater than 3 g/day. Hypo- American (ratio ranges from 10:1 to 17:1). The albuminemia, edema, and hyperlipidemia, disease is systemic, and the lung is almost lipiduria, and hypercoagulability are other always affected (primarily an ILD). Seventy- defining features. Six entities account for five to ninety percent of patients have enlarged >90% of cases of nephrotic syndrome in adults: intrathoracic lymph nodes, usually the hilar minimal change disease (MCD), focal and seg- nodes. (Kasper et al., 2005, pp. 2017–2023) mental glomerulosclerosis (FSGS), membra- nous glomerulopathy, membranoproliferative 121. (A) Rheumatoid arthritis affects approximately glomerulonephritis (MPGN), diabetic nephropa- 0.8% of the population. Women are affected thy, and amyloidosis. In North America, the most three times more often than men. Synovial common cause of nephrotic syndrome is dia- inflammation causes tenderness, swelling, and betes mellitus. Heroin use has been associated limitation of motion. Most often the arthritis is with FSGS and lymphoma has been associated symmetric and characteristically involves joints with glomerular disorders. Some of the glomeru- such as the wrist and the proximal interpha- lar disorders above may present with nephrotic langeal and metacarpophalangeal joints. (Kasper syndrome and if untreated progress to renal et al., 2005, pp. 1968–1977) failure. (Kasper et al., 2005, pp. 1684–1685)

122. (G) Risk factors for OA include age, obesity, 125. (D) major trauma, and repetitive joint use. Bony enlargements of the DIP joint (Heberden’s 126. (C) nodes) are the most common form of idiopathic OA. Obesity is a risk factor for knee OA. With Explanations 125 and 126 severe OA of the knee in particular, obesity is thought to play a large role in pathogenesis. The typical migraine attack consists of a visual (Kasper et al., 2005, pp. 2036–2045) aura with flashes, scintillating scotomata (field loss), or fortification spectra followed by a throb- 123. (D) Hyponatremia is a common metabolic bing unilateral temporal headache. There may be derangement. Facititious hyponatremia is seen associated vestibular, GI, or neurologic symp- with severe hyperlipidemia or hyperproteine- toms. Attacks are often precipitated by stress, mia (which lower plasma water content) and fatigue, or foods that contain tyramine (e.g., with hyperglycemia due to water movement cheese, yogurt, nuts) or phenylethylamine (wine, out of cells. Most patients with hyponatremia chocolate). Symptoms peak within an hour of are hypoosmolar and the diagnosis is based on onset and persist for hours to days. A positive an estimation of extracellular fluid (ECF) family history is found in as many as 50% of volume status. Decreased ECF volume and cases. Tension headaches are more often bilateral (c) ketabton.com: The Digital Library

42 1: Internal Medicine

and described as band like or vise like and are include Klebsiella, Proteus, and Enterobacter not usually associated with visual auras. TIAs species. (Kasper et al., 2005, pp. 1715–1721) more typically present as transient monocular blindness without aura or headache. Temporal 129. (B) The pH of 7.53 indicates alkalosis as the arteritis may present as painless loss of vision primary disorder (normal pH is 7.40). A high without aura, but is usually in older people. bicarbonate is consistent with a metabolic cause Cluster headaches are much more common in of the alkalosis. The high carbon dioxide of 53 men. Sumatriptan and the other triptans work by (normal is 40) is compensating for the primary inhibiting the release of vasoactive peptides, pro- disorder (alkalosis) in an attempt to bring the moting vasoconstriction, and blocking brainstem pH closer to normal. Metabolic alkalosis results pain pathways. Sumatriptan comes in oral, from renal bicarbonate reabsorption. Processes injectable, and nasal spray form. Ergotamine tar- which maintain persistent high reclamation of trate, antiemetics, and analgesics may also be bicarbonate include dehydration, hypokalemia, used in the acute treatment of migraine hypercapnea, and mineralocorticoid excess. headache. Prophylactic medications such as (Kasper et al., 2005, pp. 263–271) beta-blockers, tricyclic antidepressants, calcium channel blockers, and anticonvulsants are inef- 130. (B) fective for acute attacks. Avoidance of known precipitants and control of stress are also 131. (D) important in prevention. (Kasper et al., 2005, pp. 85–94) Explanations 130 and 131

127. (A) More than 90% of renal stones are visible on a plain abdominal x-ray, and the majority contain 128. (C) calcium oxalate. Staghorn calculi usually contain magnesium ammonium phosphate (triple phos- Explanations 127 and 128 phate or struvite) and are associated with alka- line urine. This is commonly encountered in Urinary tract infections are extremely common chronic urinary tract infections with urea-split- in young women. For simple infections uncom- ting bacteria (especially Proteus species). plicated by fever, chills, or flank pain, a single Radiolucent stones often contain urea, which is dose of an antibiotic may be curative. In the associated with acidic urine. A small percent- presence of symptoms suggesting renal age (fewer than 10%) of renal stones contain cys- parenchymal infection (i.e., pyelonephritis), tine. The most common cause of calcium stone treatment should continue for as long as 2 disease is idiopathic hypercalciuria. Almost half weeks, and parenteral antibiotics may be these patients will excrete more than 4 mg of required (e.g., fluoroquinolone). Bacteriuria in calcium/kg body weight/24 h in the absence pregnant women should be treated regardless of of hypercalcemia. Causes of hypercalciuria to symptoms; whereas, bacteriuria in patients with be ruled out are sarcoidosis, hyperparathy- indwelling catheters should probably be treated roidism, and Paget’s disease of bone. Idiopathic only in the presence of symptoms. Chronic sup- hypercalciuria is believed to result from either pressive antibiotic therapy in the latter group increased GI absorption of calcium, increased has not been shown to be useful. Radiologic calcium resorption from bone, or excessive renal investigation for underlying anatomic abnor- calcium leakage into the urine. (Kasper et al., 2005, malities should be undertaken in girls up to age pp. 1710–1714) 6, in all males after their first infection, and in women of any age with recurrent urinary tract 132. (E) Iron-deficiency anemia characteristically is infections. The most common pathogen is E. coli, a hypochromic, microcytic anemia. Causes of accounting for greater than 80% of infections. iron-deficiency anemia include menstrual loss, Other organisms frequently encountered inadequate diet, malabsorption, chronic (c) ketabton.com: The Digital Library

Answers: 127–137 43

inflammation, and chronic blood loss. Colon 135. (C) The pH is 7.10, which indicates the primary cancer could lead to chronic blood loss and iron- disorder to be an acidosis. The low bicarbonate deficiency anemia. This, however, would be and the low carbon dioxide both are indicative very uncommon in a young patient without a of a metabolic cause for the acidosis. For the family history of colon cancer. Alcohol causes a primary cause of the acidosis to be respiratory, macrocytic anemia. (Kasper et al., 2005, pp. 586–592) the carbon dioxide would need to be greater than 40. In this case, the patient is compensat- 133. (A) Although the differential diagnosis of a ing for the metabolic acidosis due to chronic pleural effusion is large, the diagnostic possi- diarrhea by hyperventilation. (Kasper et al., 2005, bilities may be narrowed by classifying the pp. 263–271) fluid as transudative or exudative. Exudates are characterized by a pleural fluid-to-serum 136. (D) protein ratio greater than 0.5, pleural fluid LDH greater than 200 IU/L, or pleural fluid-to- 137. (C) serum LDH ratio greater than 0.6. Other common findings in exudative effusions are a Explanations 136 and 137 WBC count greater than 1000/mL, glucose less than 60 mg/100 mL, and grossly hemor- The patient described in the question most rhagic fluid. Causes of transudative effusions likely has Hashimoto’s thyroiditis, also called include CHF, nephrotic syndrome, cirrhosis autoimmune or chronic lymphocytic thyroidi- with ascites, and myxedema. Causes of exuda- tis. It is the most common cause of thyroiditis in tive fluid include parapneumonic effusion, the United States and is encountered more fre- neoplasm, pulmonary infarction, tuberculosis, quently in women than in men. Patients note and fungal infection among others. A low pleu- progressive thyromegaly but are usually euthy- ral fluid pH (<7.30) limits the differential diag- roid at the outset. Hypothyroidism may appear nosis to empyema, carcinoma, collagen years later, often heralded by an elevated serum vascular disease, esophageal rupture, tubercu- TSH level. Diagnosis is based on the history, losis, or hemothorax. Uncomplicated parap- examination, heterogeneous uptake on thyroid neumonic effusions have WBC counts under scan, and the presence of antithyroid and 40,000/mL, normal glucose levels, and a pH antithyroglobulin antibodies. If the diagnosis under 7.30; a positive Gram stain or culture is still in doubt, needle biopsy will demonstrate constitutes a complicated parapneumonic effu- lymphocyte infiltration, sometimes in sheets or sion. These tend to loculate and form adhe- forming germinal centers. Subacute (de Quervain, sions if not immediately and thoroughly granulomatous) thyroiditis will show polymor- phonuclear cells, necrosis, and giant cells. Bacteria drained by chest tube placement. (Kasper et al., 2005, pp. 1565–1569) may not be present in acute suppurative thy- roiditis. Thyroid infiltration and replacement by 134. (B) The pH is 7.12, indicating acidosis as the rock-hard, woody, fibrous tissue is typical of primary disorder. A low bicarbonate is consis- Riedel’s struma. C-cell hyperplasia is associ- tent with a metabolic cause of the acidosis and ated with medullary thyroid carcinoma. a high carbon dioxide is consistent with a res- Hashimoto’s thyroiditis is treated with thyroid piratory cause of the acidosis. Therefore, both hormone. Lower doses (0.10–0.15 mg/day) of are contributing as primary problems. The levothyroxine are used to treat hypothyroidism metabolic source likely is lactic acidosis from alone; whereas, higher doses (0.15–0.30 muscle breakdown resulting from the seizures. mg/day) suppress TSH release and diminish The respiratory source likely is related to the goiter size. Partial resection may result in patient’s postictal state and hypoventilation enlargement of the remaining gland. Steroids, antibiotics, and radioiodine have no role in after the seizures. (Kasper et al., 2005, pp. 263–271) therapy. (Kasper et al., 2005, pp. 2109–2113) (c) ketabton.com: The Digital Library

44 1: Internal Medicine

138. (A) As many as 85% of women with hirsutism, involved in estrogen biosynthesis. Diagnosis obesity, and menstrual irregularities have is based on an elevated LH level, decreased polycystic ovary disease (Stein-Leventhal follicle-stimulating hormone (FSH) level, and syndrome). Women with this disorder have an LH/FSH ratio greater than 2:5. Combination chronic anovulation and frequent infertility estrogen-progestin therapy suppresses the despite the presence of adequate amounts of androgen production. Less common causes of estrogen. Excessive luteinizing hormone (LH) hirsutism are drug induced (e.g., testosterone, response to gonadotropin-releasing hormone is anabolic steroids), adrenal tumor or hyperpla- thought by many to be the primary problem, sia, Cushing disease, and ovarian tumors. resulting in ovarian theca-cell hyperplasia and Familial hirsutism is not associated with men- hypersecretion of androgens. Others have strual abnormalities or obesity. (Kasper et al., 2005, found deficiencies of the ovarian enzymes pp. 275–278, 2204–2205) (c) ketabton.com: The Digital Library

BIBLIOGRAPHY

Bartlett JG, Breiman RF, Mandell LA, et al. Hackett PH, Roach RC. High-altitude illness. N Engl Community-acquired pneumonia in adults: guide- J Med 2001;345:107–114. lines for management. Clin Infect Dis 1998;26: Hogan DE. The emergency department approach to 811–838. diarrhea. Emerg Clin North Am 1996;14:673–692. Byrne TN. Spinal cord compression from epidural Hurst JW. Medicine for the Practicing Physician. metastases. N Engl J Med 1992;327:614–619. Stamford, CT: Appleton & Lange, 1996. Cummings JL. Alzheimer’s disease. N Engl J Med Kasper DL, Braunwald E, Fauci A, et al. Harrison’s 2004;351:56–67. Principles of Internal Medicine, 16th ed. New York, Department of Health and Human Services, Henry J. NY: McGraw-Hill, 2005. Kaiser Family Foundation. Guidelines for the use Lang AE, Lozano AM. Parkinson’s disease. Part 1. of antiretroviral agents in HIV-1-infected adults [Review] N Engl J Med 1998;339(15):1044–1053. and adolescents. Bethesda, MD: Department of Lang AE, Lozano AM. Parkinson’s disease. Part 2. Health and Human Services, Henry J. Kaiser [Review] N Engl J Med 1998;339(16):1130–1143. Family Foundation, March 23, 2004, p. 97 [355 ref- Light RW. Parapneumonic effusions and empyema: erences]. current management strategies. J Crit Illn 1995;10: Ferrari JD, Bach BR. Knee pain in adults: when to 832–842. manage, when to refer. J Musculoskel Med 1998; Paulson WD. Identifying acid-base disorders: a sys- 15(3):52–63. tematic approach. J Crit Illn 1999;14(2):103–109. Freedberg IM, Eisen AZ, Wolff K, et al. Fitzpatrick’s Piccirillo JF. Acute bacterial sinusitis. N Engl J Med Dermatology in General Medicine, 6th ed. New York, 2004;351:902–910. NY: McGraw-Hill, 2003. Relman DA, Schmidt TM, MacDermott RP, et al. Giammarco R, Edmeads J, Dodick D. Critical Identification of the uncultured bacillus of Decisions in Headache Management. Hamilton, Ont.: Whipple’s disease. N Engl J Med 1992;327:293–301. BC Decker, 1998. Smith RA, Cokkinides V, Eyre HJ. American Cancer Gonzales R, Bartlett J, Besser R, et al. Principles of Society guidelines for the early detection of cancer, appropriate antibiotic use for treatment of uncom- 2006. CA Cancer J Clin 2006;56:11–25. plicated acute bronchitis: background. Ann Intern Med 2001;134:521–529. Steere AC. Lyme disease. N Engl J Med 2001;345: 115–125. Goroll AH, Lawrence AM, Mulley AG. Primary Care Medicine, 3rd ed. Philadelphia, PA: JB Lippincott, Toy EC, Patlan JT, Cruse SE, et al. Case Files: Internal 1995. Medicine. New York, NY: McGraw-Hill, 2004.

45 (c) ketabton.com: The Digital Library

This page intentionally left blank (c) ketabton.com: The Digital Library

CHAPTER 2 Obstetrics and Gynecology James Aiman, MD and Michael R. Lund, MD

Questions

1. A 27-year-old woman has used oral contracep- (C) computed tomography (CT) scan of her tives (OCs) without problems for 5 years. head However, she just read an article about com- (D) a serum estradiol concentration plications of OCs in a popular women’s mag- (E) a serum follicle-stimulating hormone azine and asks you about the risks and hazards (FSH) concentration of taking OCs. You correctly tell her which of the following? 3. The tests you ordered are normal for a prepu- (A) The risk of developing ovarian cancer is bertal girl. Which of the following is the most increased. likely diagnosis? (B) The risk of developing pelvic inflamma- (A) ingestion of the mother’s OC pills tory disease (PID) is increased. (B) a granulosa cell tumor (C) The risk of developing endometrial can- (C) 21-hydroxylase deficiency cer is decreased. (D) polycystic ovary syndrome (D) The risk of bearing a child with major (E) premature thelarche congenital anomalies is increased if taken while pregnant. 4. Which of the following is the most appropriate (E) The risk of ectopic pregnancy is management of this girl? increased. (A) pituitary suppression with a Questions 2 through 4 gonadotropin-releasing hormone (GnRH) agonist The mother of a 3-year-old girl brings her daughter to (B) laparoscopy see you because the girl developed breasts 6 months (C) assurance that the condition is benign ago. The girl has had no , and there is and self-limiting no pubic hair. She takes no medication. (D) corticosteroid suppression of adrenal 2. Which of the following is the most appropriate function next diagnostic step? (E) breast biopsy (A) an ultrasound of the pelvis (B) a pelvic examination under general anesthesia

47

Copyright © 2008 by The McGraw-Hill Companies, Inc. Click here for terms of use. (c) ketabton.com: The Digital Library

48 2: Obstetrics and Gynecology

5. After an appropriate diagnostic evaluation, a (D) Intrauterine infection with herpes is 59-year-old woman with postmenopausal bleed- common after 20 weeks in women with ing had a total abdominal hysterectomy and primary herpes. bilateral salpingo-oophorectomy (TAH-BSO). (E) Pitocin induction of labor should be The pathologic diagnosis is adenocarcinoma of started within 4 hours after ruptured the . An endometrial adenocarci- amniotic membranes in a woman at noma that is confined to the and extends term with active genital herpes. more than 50% through the is at which stage? 8. A 63-year-old woman has a 3-cm pruritic lesion (A) IC on her right labia majora that she has noted for approximately 9 months. She has been treated (B) IIA with various topical creams and ointments for (C) IIB vulvar candidiasis without resolution of her (D) IIIA symptoms or lesion. When you examine this (E) IVA woman, the lesion is still present. Which of the following is the most appropriate intervention? 6. A 39-year-old woman at 16 weeks’ gestation (A) Papanicolaou (Pap) smear of the lesion complains of headaches, blurred vision, and epigastric pain. Her blood pressure is now (B) colposcopy of the lesion 156/104 mmHg. Her uterine fundus is palpa- (C) biopsy of the lesion ble 22 cm above her symphysis pubis. Fetal (D) wide local excision of the lesion heart tones could not be heard with a handheld (E) vulvectomy Doppler. She has 3+ proteinuria. Which of the following is the most likely diagnosis? Questions 9 through 11

(A) anencephaly A 48-year-old woman had a biopsy of a friable, (B) twin gestation bleeding lesion on her cervix. She had not had a (C) maternal renal disease pelvic examination or Pap smear for about 12 years. (D) hydatidiform mole The biopsy is reported as invasive squamous cell (E) gestational diabetes mellitus carcinoma of the cervix. On bimanual examination, there is induration to the side wall of her pelvis. 7. A 23-year-old woman develops painful vulvar vesicles that contain intranuclear inclusions on 9. Which of the following is the stage of her cer- cytologic examination. She is 22 weeks’ preg- vical cancer? nant. Which of the following statements about (A) IA genital herpes is correct? (B) IB (A) Acyclovir should be prescribed from 36 (C) IIB gestational weeks until after delivery in (D) IIIB women with primary herpes anytime (E) IV during pregnancy. (B) Herpes cultures from the cervix should 10. To complete the staging of her cancer according be obtained weekly beginning at 36 to International Federation of Gynecology and weeks’ gestation. Obstetrics (FIGO) standards, she should have (C) An active genital herpetic lesion any which of the following? time after 20 weeks’ gestation requires a (A) lymphangiogram cesarean section. (B) pelvic venogram (C) cystoscopy (c) ketabton.com: The Digital Library

Questions: 5–16 49

(D) magnetic resonance imaging (MRI) scan 14. At a follow-up routine prenatal visit, the uter- of her abdomen ine fundus of a healthy 23-year-old pregnant (E) laparoscopy woman is palpated halfway between her sym- physis pubis and umbilicus. Which of the fol- 11. Which of the following is the most important lowing is the most appropriate test to order at prognostic factor for 5-year survival after this stage of her pregnancy? appropriate treatment of cervical cancer? (A) serum human immunodeficiency virus (A) presence of high-risk strains of human (HIV) titer papilloma virus (HPV) (B) glucose tolerance test (B) stage of the cancer (C) amniocentesis (C) age of the patient (D) maternal serum alpha-fetoprotein (D) histologic grade of the tumor (MSAFP) (E) presence of positive regional (pelvic) (E) cervical culture for group B Streptococcus lymph nodes (GBS)

12. A 35-year-old G3P3 woman has been experi- 15. A 58-year-old woman with stage II epithelial encing bilateral breast pain for the past year. ovarian cancer undergoes successful surgical Breast examination and mammography are debulking followed by chemotherapy with car- normal. Conservative measures have failed. boplatin and radiation therapy. Subsequently, Which of the following medications is most she develops nonpitting edema of both legs likely to bring relief? and pain and numbness in her legs. Which of the following is the most likely cause of her (A) clomiphene pain and numbness? (B) tamoxifen (A) nerve damage caused by the pelvic lym- (C) danazol phadenectomy (D) hydrochlorothiazide (B) lymphedema (E) medroxyprogesterone (C) carboplatin therapy 13. A 23-year-old married woman consults you (D) radiation therapy because she and her husband have never con- (E) recurrent ovarian cancer summated their marriage because she has severe pain with attempts at vaginal penetra- 16. A 13-year-old girl had growth of breast buds at tion. Her pelvic examination is normal except 11 years, followed by the appearance of pubic for involuntary tightening of her vaginal mus- hair between the ages of 11 1/2 and 12 years. cles when you attempt to insert a speculum. Which pubertal event is most likely to occur Which of the following conditions would best next? be treated with the use of vaginal dilators? (A) beginning of accelerated growth (A) primary (B) menarche (B) (C) Tanner stage 5 breast development (C) deep-thrust (D) maximal growth rate (D) anorgasmia (E) Tanner stage 5 pubic hair (E) vulvar vestibulitis (c) ketabton.com: The Digital Library

50 2: Obstetrics and Gynecology

17. A 17-year-old G1P1001 is now 5 weeks post- family history is negative for hirsutism. The hir- partum after a routine vaginal delivery. She sutism is confirmed by your examination. Her calls your office to report a 3-week history of pelvic examination is normal other than a mild male difficulty sleeping and “feeling blue.” On fur- pubic hair pattern. ther questioning, she reports difficulty con- centrating, very poor appetite, occasional 20. Which of the following is the most appropriate wishes that she had never become pregnant, next step in her evaluation? and feelings of guilt about those wishes. She (A) serum prolactin concentration has not left her home in more than a week because she “just can’t find the energy to go (B) 24-hour urine for 17-ketosteroid anywhere.” This patient’s symptoms are most excretion consistent with: (C) serum dehydroepiandrosterone sulfate (DHEAS) concentration (A) postpartum blues (D) CT scan of the pituitary sella (B) normal adolescent adjustment to (E) pelvic ultrasound motherhood (C) postpartum depression 21. Your evaluation of this hirsute, amenorrheic (D) hypothyroidism woman is normal except for a significantly (E) postpartum psychosis increased serum DHEAS concentration. Additional history discloses that her menses 18. Labor and vaginal delivery occur successfully have always been somewhat irregular since in a 29-year-old woman after administration menarche at age 10 years. She has a 23-year-old of oxytocin (Pitocin) for 9 hours. Spontaneous sister with irregular menstrual intervals and onset of labor at term is the result of which of hirsutism to a lesser degree. This patient has a the following? blood pressure of 96/64 mmHg. Which of the following is the most likely diagnosis? (A) cortisol production in the amniotic cavity (A) polycystic ovary syndrome (B) prostaglandin release from the fetal (B) 21-hydroxylase deficiency membranes (C) 11-hydroxylase deficiency (C) prolactin produced in the decidua (D) 17-hydroxylase deficiency (D) fetal pituitary secretion of oxytocin from (E) Sertoli-Leydig cell tumor the neurohypophysis (E) events that are currently uncertain 22. A 31-year-old pregnant woman 6–7 weeks from her last menses comes to the emergency 19. At 24 weeks’ gestation, where are most fetal department of your hospital complaining of red blood cells produced? lower abdominal pain for 3 hours. The pain is diffused in the lower abdomen but worse on (A) the yolk sac the right side. Her serum human chorionic (B) spleen gonadotropin (hCG) concentration is 9600 (C) bone marrow mIU/mL. Which of the following is the (D) liver strongest evidence that she has a tubal ectopic (E) lymph nodes pregnancy? (A) absence of an extrauterine sac on Questions 20 and 21 ultrasonography A 27-year-old woman with amenorrhea of 6 (B) absence of blood on culdocentesis months’ duration relates a 4-month growth of thick, (C) absence of a mass on bimanual black hair on her face, chest, and abdomen. She examination takes no medications with androgenic effects. Her (c) ketabton.com: The Digital Library

Questions: 17–28 51

(D) absence of an intrauterine sac on ultra- 26. An 18-year-old nullipara has suddenly stopped sonography menstruating. She recently lost 8.6 kg when (E) her hCG concentration she started long-distance running. The labora- tory test most consistent with her cause of sec- 23. A 22-year-old primiparous woman is in pre- ondary amenorrhea is which of the following? mature labor at 30 weeks’ gestation. Despite (A) a serum prolactin level of 86 ng/mL administration of tocolytic agents, it seems she (normal <20) will deliver soon. Pulmonary maturity might (B) a serum LH level of 48 mIU/mL be enhanced by the administration of which of (normal 6–35) the following drugs? (C) a serum estradiol level of 128 pg/mL (A) magnesium sulfate (normal 40–300) (B) betamethasone (D) a serum FSH level of 3 mIU/mL (C) hydroxyprogesterone (normal 5–18) (D) chloroprocaine (E) a serum testosterone level of 156 ng/dL (E) digitalis (normal 40–110)

24. A 28-year-old woman with 28-day menstrual 27. A 22-year-old woman with cystic fibrosis is cycle is attempting to conceive and is consid- engaged to be married and asks you about ering the use of a home ovulation predictor kit childbearing. How should you advise her? to time intercourse at ovulation. She asks you (A) An amniocentesis should be done to what day of her menstrual cycle her luteinizing detect fetal cystic fibrosis. hormone (LH) peak is most likely to occur. (B) Pregnancy is contraindicated because What should you tell her? maternal mortality is significantly (A) day 12 increased. (B) day 14 (C) Her children have a 25% chance of hav- (C) day 18 ing cystic fibrosis. (D) day 20 (D) Pregnancy and delivery are usually suc- (E) day 27 cessful with special care and precau- tions. 25. A 48-year-old woman with five children com- (E) She should use nasal oxygen through- plains of urinary incontinence with coughing out pregnancy to minimize fetal hypox- and stair climbing. She likely has genuine stress emia. urinary incontinence if which of the following is true? 28. On the first pelvic examination of an 18-year- old nulligravida, a soft, fluctuant mass is found (A) Loss of urine is secondary to involun- in the superior aspect of the right labia majora. tary bladder contractions. This is asymptomatic. She tells you it has been (B) Loss of urine is associated with a strong present for several years and seems to be desire to void immediately. enlarging slightly. There is no defect in the (C) Loss of urine occurs in relation to anxi- inguinal ring. Which of the following is the ety or depression. most likely diagnosis? (D) Loss of urine occurs when intravesical (A) vulvar varicosities pressure exceeds maximal urethral pres- (B) inguinal hernia sure. (C) femoral hernia (E) Loss of urine is due to increased intrav- esical pressure associated with bladder (D) cyst of the canal of Nuck distention. (E) granuloma inguinale (c) ketabton.com: The Digital Library

52 2: Obstetrics and Gynecology

29. Your patient has just had twins and wonders if (A) The vaginal contraceptive ring is there is any way to determine whether the changed weekly for 3 consecutive twins are identical. You correctly tell her which weeks, then removed for 1 week to of the following? allow for withdrawal bleeding. (A) Close examination of the placenta can (B) Because of effects on the cytochrome often provide this answer. P450 system, Depo-Provera should not be used in patients taking antiepileptic (B) There is no way to tell unless one is a drugs (e.g., phenytoin). girl and one a boy. (C) Amenorrhea while using the lev- (C) Only matching of human lymphocyte onorgestrel intrauterine system (IUD) antigens could determine this with should raise concern immediately for certainty. ectopic pregnancy. (D) Identical twins occur only once in about (D) A diaphragm should be inserted no 80 births of twins. more than 6 hours before intercourse (E) It is unlikely because the birth weights and should remain in place about 6 differed by more than 200 g. hours after intercourse. (E) Failure rate for tubal ligation over 10 30. An 11-year-old girl has her first menses. Both years is less than 1 pregnancy per 1000 ovaries contain approximately how many surgeries performed. oocytes? (A) 7 million 33. A 24-year-old nullipara is being evaluated for (B) 1 million infertility. On pelvic examination, she has a (C) 500,000 single cervix. A diagnostic laparoscopy shows a double uterine fundus. Which of the follow- (D) 50,000 ing is the most likely diagnosis of her uterine (E) 5000 anomaly? 31. A healthy 29-year-old gravida 2 woman at 39 (A) septate uterus weeks has been in labor for 3 hours. She had a (B) unicornuate uterus positive vaginal-anal culture for GBS at 37 (C) bicornuate uterus weeks’ gestation. Which one of the following (D) didelphic uterus statements is correct? (E) a diethylstilbestrol (DES) exposed (A) Asymptomatic rectovaginal colonization uterus is present in 60% of pregnant women. (B) The transmission rate from mother to 34. A 58-year-old G6P4Ab2 diabetic woman who baby is approximately 25%. weighs 122.6 kg (270 lb) has her first episode of (C) A rectovaginal culture should have been vaginal bleeding in 5 years. Her physician per- obtained at the first prenatal visit. forms an outpatient operative hysteroscopy and dilatation and curettage (D&C). Which of (D) Neonatal sepsis occurs in 1% of colo- the following is an indication for the procedure nized mothers. and the most likely diagnosis? (E) Treatment with penicillin in labor is nec- essary only for heavy colonized mothers. (A) endometrial cancer because of her high parity 32. Which of the following statements is true (B) endometrial cancer because of her obesity regarding contraception? (C) cervical cancer because of her age (D) cervical cancer because of her diabetes (E) ovarian cancer because of her obesity (c) ketabton.com: The Digital Library

Questions: 29–39 53

35. A pregnant woman is being followed by a 38. A 31-year-old primigravida develops gesta- nephrologist for chronic glomerulonephritis. tional diabetes mellitus and is managed appro- Which of the following findings is normal at 28 priately during pregnancy. She asks you about weeks’ gestation? the consequences of gestational diabetes to her and her fetus. Which one of the following state- (A) blood pressure of 132/86 mmHg ments is correct? (B) blood urea nitrogen (BUN) of 21 mg/100 mL (A) The risk of fetal anomalies is increased. (C) serum creatinine of 1.1 mg/100 mL (B) The risk of stillbirth is increased if her (D) glomerular filtration rate (GFR) of fasting blood sugars are elevated. 130 mL/min (C) The risk of a growth-restricted newborn (E) glycosuria with a plasma glucose of is increased. 130 mg/100 mL (D) Insulin is the preferred treatment to maintain euglycemia. 36. A 25-year-old woman has a positive cervical (E) The risk of fetal macrosomia is not culture for Neisseria gonorrhoeae. She has had increased with gestational diabetes. at least two positive cultures for gonorrhea treated in the past. She is afebrile and has no Questions 39 and 40 symptoms. The incidence of penicillin-resistant gonorrhea in some areas of the United States is 39. A 22-year-old G3P1102 is admitted to the Labor currently as great as 10%. Because of this, the and Delivery ward at 28 weeks’ gestation com- recommended treatment for gonorrhea plaining of watery . You con- includes which of the following? firm the diagnosis of preterm premature rupture of amniotic membranes (PPROM). (A) 125 mg intramuscular ceftriaxone as a Fetal monitoring demonstrates reassuring fetal single dose heart tones and no contractions are noted. The (B) 1 g spectinomycin patient is understandably concerned and asks (C) 2 g ampicillin orally as a single dose you why this happened and what this means (D) 2 g intramuscular cefoxitin for her pregnancy. Which of the following should you tell her? (E) 2 g metronidazole as a single dose (A) The incidence of PPROM is directly cor- 37. A 37-year-old pregnant woman with type 2 related to maternal age. diabetes mellitus and chronic hypertension is (B) Most patients with PPROM before 35 weeks’ pregnant. Which of the following is 30 weeks will remain pregnant until the best test to screen for fetal well-being? at least 34 weeks. (A) nonstress test (NST) (C) Management at home is a reasonable (B) oxytocin challenge test option for most patients until the onset of contractions. (C) amniocentesis (D) Patients with are at (D) fetal movement counting increased risk for PPROM during preg- (E) fetal biophysical profile nancy. (E) Pulmonary hypoplasia is a common complication of PPROM at this gesta- tional age. (c) ketabton.com: The Digital Library

54 2: Obstetrics and Gynecology

40. Which of the following is the most appropriate (A) Because fibroids are responsive to sex therapy for this woman? steroids, treatment with GnRH agonists (e.g., leuprolide) will produce up to a (A) begin antibiotic therapy to prolong the 50% reduction in volume. latency period until labor begins (B) Treatment with leuprolide appears to be (B) immediate cesarean delivery to prevent long lasting, making this an attractive umbilical cord prolapse alternative to hysterectomy or myomec- (C) induction of labor to prevent intra- tomy. amniotic infection (C) Myomectomy (i.e., removal of uterine (D) amniocentesis to determine fetal lung fibroids without removal of the uterus) maturity status is replacing hysterectomy as it is associ- (E) placement of a cervical cerclage to pre- ated with less complications and less vent preterm delivery blood loss. (D) Because it requires no abdominal or Questions 41 and 42 uterine incisions, uterine artery 41. A 38-year-old G4P3013 woman is seeing you embolization is the preferred method of for her annual gynecologic examination. She treatment for women who desire future has no specific complaints, but notes that her pregnancy. menses have gradually become heavier over (E) Any leiomyoma larger than 5 cm should the past 2–3 years. Your pelvic examination is be removed by either hysterectomy or normal aside from an enlarged uterus, which myomectomy to rule out leiomyosarcoma. you estimate at 12 weeks’ size. Office ultra- sonography confirms that she has multiple 43. A 23-year-old G1P0010 woman complains of uterine fibroids. Which of the following state- severe dysmenorrhea (i.e., pain with menses). ments is true regarding leiomyomata? She misses work for the first 3 days of her men- strual cycle almost every month and states that (A) OCs cause leiomyomata to grow more this has been a problem for about 3 years, now rapidly. getting worse. She is not currently sexually (B) Leiomyomata not removed by hysterec- active and is not using any hormonal contra- tomy may eventually degenerate into ception. She complains of occasional deep malignant tumors (i.e., leiomyosarcoma). abdominal pain after bowel movements. (C) Submucosal fibroids are more likely to Subsequent laparoscopic evaluation reveals cause painful, heavy periods than are . Which of the following is a true subserosal fibroids. statement regarding this condition? (D) Leiomyomata occur in up to 5% of all (A) Smoking is a strong risk factor for the women. development of endometriosis. (E) Typical bleeding abnormalities seen (B) There is a direct relationship between with uterine fibroids are heavy men- perceived pain and the amount of strual bleeding as well as frequent inter- endometriosis noted at the time of menstrual bleeding episodes. surgery. (C) Most women with endometriosis also 42. Your patient comes back 6 months later with a have infertility. calendar demonstrating continued worsening of her menstrual bleeding, now 10 days in (D) Medical and surgical management are duration and requiring one pad hourly during equally effective in restoring fertility in her heaviest days. Which of the following the patient with endometriosis. statements are true regarding treatment of (E) The most common sites of endometrio- leiomyomata? sis implants are the ovaries and anterior and posterior cul-de-sacs. (c) ketabton.com: The Digital Library

Questions: 40–48 55

44. You are seeing a 38-year-old woman for her Questions 46 and 47 annual gynecologic examination. She asks you for some information regarding the HPV vac- 46. A 35-year-old primigravid woman with a his- cine and whether you think it would be appro- tory of cyclic menses at 28- to 30-day intervals priate for her 17-year-old daughter. Which of began her last menses on August 18. A home the following statements regarding the quadri- pregnancy test was positive on September 20. valent human papillomavirus vaccine and At her first prenatal visit, she asks you what the HPV is true? duration of pregnancy is and what her due date is. You tell her that the average number of (A) The vaccine is recommended for women days from the onset of menses to delivery is ages 11–26 but can be given as young as which of the following? age 9. (A) 250 (B) After vaccination, women no longer need routine Pap smears. (B) 260 (C) The vaccine is given every month for (C) 270 3 months. (D) 280 (D) The vaccine is prepared from the pro- (E) 290 teins of four oncogenic (e.g., high-risk for cervical cancer) strains of HPV. 47. With reference to the above patient, you tell (E) Women with a prior history of abnormal her that her expected delivery date (EDD) is Pap smears are not candidates for vacci- which of the following? nation. (A) May 18 (B) August 18 45. A 19-year-old primigravid woman at 39 weeks’ (C) May 11 gestation is in active labor, and her cervix is (D) August 11 4 cm dilated, 90% effaced. Her amniotic mem- branes have been ruptured for 4 hours. (E) May 25 Contractions are strong at 2- to 3-minute inter- (F) June 1 vals and of 60- to 70-second duration. For the past 30 minutes, repetitive variable decelera- Questions 48 and 49 tions of the fetal heart rate have occurred. They A 44-year-old woman had a normal Pap smear have lasted 60–90 seconds, and the fetal heart 3 years ago. Her menstrual periods occur monthly rate has dropped as low as 60 beats per minute and last 5 days. She has had intermenstrual and post- (BPM). You explain that there is a risk that the coital spotting intermittently for the past 6 months. baby will become hypoxic and recommend a The pelvic examination is normal. cesarean section. She refuses. Which of the fol- lowing is the most appropriate course of 48. Which of the following is the most appropriate action? test to perform? (A) obtain permission for the cesarean (A) an endometrial biopsy section from her mother (B) an endocervical curettage (B) perform a cesarean section as an emergency (C) a conization of the cervix (C) obtain a court order permitting a (D) a Pap smear cesarean section (E) a hysteroscopy (D) counsel her carefully about the fetal risks but accede to her wishes (E) assign her care to another obstetrician (c) ketabton.com: The Digital Library

56 2: Obstetrics and Gynecology

49. All tests performed on the woman were (C) intravesical instillation of methylene normal. She returns 1 year later for her annual blue gynecologic examination. On speculum exam- (D) the Q-tip test ination, she has a visible 7-mm lesion on her (E) measurement of residual urine volume cervix that bleeds on contact. Which of the fol- lowing is the most appropriate procedure to 53. A 48-year-old G5P5 woman has genuine stress perform? incontinence (GSI). Kegel exercises have not (A) colposcopy helped, and her incontinence is gradually (B) cervical biopsy worsening. Her urethrovesical junction (UVJ) is prolapsed into the vagina, and her urethral clo- (C) Pap smear sure pressure is normal. Which of the following (D) conization of the cervix procedures will most likely cure her inconti- (E) vaginal hysterectomy nence?

50. A 24-year-old woman lost her previous two (A) retropubic urethropexy pregnancies at approximately 20 weeks’ gesta- (B) anterior colporrhaphy tion, without having noted any contractions. (C) suburethral sling procedure She is currently at 15 weeks’ gestation and (D) needle suspension of paraurethral tissue denies having uterine contractions. Her cervix (E) paraurethral collagen injections is undilated and uneffaced. Which of the fol- lowing is the most appropriate management 54. On a routine annual examination, a 43-year- of this patient? old woman is found to have a 2-cm mass in the (A) bed rest lateral aspect of her right breast. Which of the (B) terbutaline following is the most appropriate next step in management? (C) hydroxyprogesterone (D) DES (A) repeat the breast examination after her (E) a cervical cerclage next menses (B) mammography 51. A 24-year-old woman has a MSAFP of 0.5 (C) fine-needle aspiration MOM (multiples of the median) at 17 weeks’ (D) open biopsy gestation. Which of the following fetal abnor- (E) segmental resection malities is most likely to occur with this MSAFP? 55. A 37-year-old pregnant woman has a genetic (A) spina bifida amniocentesis at 16 weeks’ gestation. A concur- (B) omphalocele rent ultrasound shows normal fetal anatomy. Her prenatal course has been unremarkable. (C) gastroschisis Her prenatal laboratory tests include a B-negative (D) bladder exstrophy blood type, a negative rubella antibody titer, a (E) trisomy 21 negative hepatitis B surface antigen, and a hematocrit of 31%. Which of the following is the 52. A 69-year-old woman with diabetes mellitus most appropriate management for this woman? complains of urinary incontinence. Her dia- betes is well controlled with oral hypoglycemic (A) rubella immunization at the time of the agents. She has no complaints other than the amniocentesis wetness. Which of the following tests is most (B) a serologic test for the presence of likely to demonstrate the cause? hepatitis B surface antibody (A) urinalysis (B) urine culture and sensitivity (c) ketabton.com: The Digital Library

Questions: 49–60 57

(C) a follow-up ultrasound in 1 week to 58. Which of the following is the treatment of assess for intra-amniotic bleeding choice during pregnancy for this woman? (D) administration of Rh immune globulin (A) coumadin at the time of the amniocentesis (B) heparin (E) chorionic villus biopsy at the time of the (C) aspirin amniocentesis (D) tissue plasminogen activator (TPA) Questions 56 through 58 (E) vena caval filter

A 23-year-old pregnant woman at 5 postmenstrual 59. A 19-year-old primigravida at term has been 1 weeks took coumadin until about 3 days after her completely dilated for 2 /2 hours. The vertex is menses was due. She has monthly menses. A home at 2 to 3 station, and the position is occiput pos- pregnancy test was positive on the day she took terior. She complains of exhaustion and is coumadin. She takes coumadin because of a history unable to push effectively to expel the fetus. of deep vein thrombosis and pulmonary embolism. She has an anthropoid pelvis. Which of the fol- She is concerned that the coumadin will cause birth lowing is the most appropriate management defects. to deliver the fetus?

56. You tell her that the conceptus is most suscep- (A) immediate low transverse cesarean tible to teratogenesis at what stage of preg- section nancy? (B) immediate classical cesarean section (C) apply forceps and deliver the baby as an (A) between menses and ovulation occiput posterior (B) from ovulation to implantation (D) apply Kielland forceps to rotate the (C) between implantation and the day of baby to occiput anterior expected menses (E) cut a generous episiotomy to make her (D) between the day of expected menses pushing more effective and 12 postmenstrual weeks (E) during the second and third trimesters Questions 60 and 61

57. You advise this woman to do which of the A 22-year-old woman and her 24-year-old partner following? have been attempting to conceive for 12 months. They have sexual intercourse two to three times per (A) Abort the pregnancy because the fetus is week and use no contraception or coital lubricants. likely to have birth defects. She has never been pregnant and her husband has (B) Have an ultrasound in 1–2 weeks to fathered no pregnancies. She has no history to sug- search for fetal anomalies. gest damage to her Fallopian tubes and her menses (C) Have a genetic amniocentesis at 16 post- occur at 28- to 31-day intervals. menstrual weeks. (D) Begin prenatal care because the proba- 60. What is the probability of conception per bility of birth defects is low. ovulation in normally fertile couples? (E) Take 10 mg vitamin K to reverse the (A) 5% effects of coumadin. (B) 10% (C) 20% (D) 35% (E) 50% (c) ketabton.com: The Digital Library

58 2: Obstetrics and Gynecology

61. The statistic in question 60 is fecundability. concerned that the drug will cause fetal Fecundity is defined as the probability of abnormalities. Which of the following defects having a liveborn child per ovulation. What is is the most common anomaly associated with the fecundity of normally fertile couples? phenytoin? (A) 5% (A) atrial septal defect (B) 10% (B) ventricular septal defect (C) 20% (C) cleft lip/palate (D) 35% (D) spina bifida (E) 50% (E) hydrocephalus

Questions 62 and 63 65. When counseling pregnant women about the dangers of drug use during pregnancy, they A 46-year-old G3P3 woman has had postcoital spot- should be told that the rate of spontaneous ting for 6 months. On pelvic examination, she has a major malformations in newborns is what? fungating, exophytic lesion arising from her cervix that is approximately 2 cm in diameter. Biopsy of this (A) less than 1% lesion is interpreted as invasive squamous cell carci- (B) 2–4% noma of the cervix. There is no evidence of extension (C) 6–8% of the cancer onto the vagina. The parametria are (D) 10–12% indurated on bimanual examination, though not to (E) more than 12% the pelvic sidewall. CT scan of her pelvis and abdomen discloses enlarged paraaortic lymph nodes Questions 66 and 67 and metastatic lesions in the parenchyma of her liver. A 34-year-old woman just delivered a 4100-g boy 1 62. Which of the following is the FIGO stage of after a 15-hour labor, including a 2 /2-hour second her cancer? stage. During the repair of a midline episiotomy, (A) IA there is a marked increase in the amount of vaginal bleeding. (B) IB (C) IIB 66. Which of the following is the most common (D) IIIB cause of immediate postpartum hemorrhage? (E) IVB (A) retained placental fragments 63. This woman’s childbearing is complete. She is (B) uterine atony a healthy woman who is close to ideal body (C) cervical laceration weight, exercises regularly, and does not (D) vaginal laceration smoke. Which of the following is the most (E) disseminated intravascular coagulation appropriate treatment of this woman? (A) TAH-BSO 67. Which of the following is the best immediate management of the probable cause of this post- (B) radical hysterectomy with pelvic and partum hemorrhage? paraaortic lymph node dissection (C) pelvic exenteration (A) massage and compression of the uterine (D) multiagent chemotherapy fundus (E) combined brachytherapy and external (B) intravenous administration of 20 units radiation therapy of oxytocin (C) abdominal hysterectomy 64. A pregnant woman has been taking pheny- (D) uterine artery embolization toin (Dilantin) for a seizure disorder. She is (E) hypogastric artery ligation (c) ketabton.com: The Digital Library

Questions: 61–75 59

Questions 68 through 72 72. Which of the following is the best predictor of survival after appropriate treatment of breast A 55-year-old woman has a bloody discharge from cancer? her left breast. A mammogram discloses a cluster of microcalcifications 3 cm beneath her left nipple. (A) an initial tumor 1.5 cm or smaller in diameter 68. Which of the following is the best next step in (B) the presence of estrogen receptors her evaluation? (C) the presence of progesterone receptors (A) cytologic evaluation of the nipple dis- (D) a well-differentiated tumor charge (E) axillary nodes negative for cancer (B) fine-needle aspiration under radiologic guidance 73. A 39-year-old woman known to have fibrocys- (C) MRI of the breast and axillary nodes tic disease of the breast complains of persistent fullness and pain in both breasts. Which of the (D) image-guided percutaneous biopsy of following drugs will be most effective in reliev- the left breast ing her symptoms? (E) segmental mastectomy (A) tamoxifen 69. Which of the following is the principal advan- (B) bromocriptine tage of a fine-needle aspiration of a breast mass? (C) medroxyprogesterone acetate (A) It reassures the patient if it is negative. (D) danazol (B) It reduces the number of open breast (E) hydrochlorothiazide biopsies. (C) It differentiates between noninvasive 74. A couple consults you because each has neu- and invasive cancer. rofibromatosis and wish to know what their reproductive possibilities are. You should tell (D) It replaces the need for subsequent them which of the following? mammography. (E) It helps to determine the extent of in (A) The disease is lethal and results in spon- situ breast carcinoma. taneous abortion of homozygous fetuses. (B) 25% of the females will be affected. 70. Which of the following factors is associated (C) 50% of all offspring will be homozygous with the greatest lifetime risk for developing for the abnormal gene. breast cancer? (D) 75% or more of their offspring will have (A) obesity the disease. (B) early menarche (E) 25% of their offspring will be unaffected. (C) late menopause 75. A 26-year-old woman complains of a vaginal (D) age discharge causing burning and itching of the (E) having a mother with a history of breast perineum. The pH of the discharge is 4.5. cancer Which of the following is the most likely cause of her discharge? 71. Which of the following is the most common type of breast cancer? (A) monilial (B) trichomonas vaginitis (A) inflammatory carcinoma (C) chlamydial (B) lobular carcinoma in situ (D) gonococcal cervicitis (C) lobular infiltrating carcinoma (E) bacterial vaginosis (D) infiltrating ductal carcinoma (E) ductal carcinoma in situ (c) ketabton.com: The Digital Library

60 2: Obstetrics and Gynecology

Questions 76 and 77 (C) 17α-hydroxylase deficiency (D) congenital absence of the vasa deferentia 76. A wet smear of a vaginal discharge is illus- trated in Figure 2-1. Which of the following is (E) Klinefelter syndrome the most likely cause of the discharge? Questions 79 and 80

79. A healthy 27-year-old male and his partner have been attempting to conceive for more than 1 year. As part of their evaluation he has a semen analysis. His ejaculate volume is 3.5 mL, sperm concentration is 8 million/mL, sperm motility is 65%, oval forms comprise 60% of the sperm, and fructose is present in the ejacu- late. Which of these semen parameters is abnor- mal and suggests that this couple’s infertility may be due to a male factor? (A) ejaculate volume (B) sperm concentration (C) sperm motility (D) sperm morphology

FIG. 2-1 (E) presence of fructose

80. The man is treated with clomiphene for a pre- (A) monilial vaginitis sumptive diagnosis of male factor infertility. (B) trichomonas vaginitis Though clomiphene is an unproven and unap- (C) Chlamydia trachomatis proved therapy for male infertility, what is the earliest that a semen analysis should be (D) N. gonorrhoeae done to detect an improvement in his semen (E) bacterial vaginosis parameters?

77. Which of the following is the most appropriate (A) 33 days treatment for the discharge illustrated in (B) 53 days Figure 2-1? (C) 73 days (A) clindamycin (D) 90 days (B) erythromycin (E) 120 days (C) metronidazole Questions 81 through 83 (D) miconazole (E) doxycycline A 39-year-old pregnant woman with chronic hyper- tension and one prior pregnancy is now at 38 78. A 37-year-old man and his wife seek help for weeks’ gestation. She comes to labor and delivery their 5-year history of primary infertility. Her with profuse vaginal bleeding and abdominal pain infertility investigation is normal. However, the of sudden onset. husband has an ejaculate volume of 0.4 mL, and there are no sperm in the ejaculate. A qualitative 81. Which of the following is the most likely test for fructose in the semen is negative. Which diagnosis? of the following is the most likely diagnosis? (A) bloody show (A) germ cell aplasia (B) vaginal laceration from coitus (B) bilateral occlusion of the vasa deferentia (C) cervicitis (c) ketabton.com: The Digital Library

Questions: 76–88 61

(D) placenta previa 85. Which one of the following laboratory tests is (E) placental abruption found in women with the androgen insensitiv- ity syndrome? 82. If the patient has a placental abruption, which (A) normal adult female testosterone of the following is the most likely risk factor? concentrations (A) advanced maternal age (B) normal adult male testosterone (B) low parity concentrations (C) coitus immediately before the onset of (C) low LH concentrations bleeding (D) elevated prolactin concentrations (D) hypertension (E) low TSH (thyroid-stimulating hormone) (E) a step aerobic class immediately before concentrations the onset of bleeding 86. Which one of the following laboratory tests is 83. This patient has an external fetal monitor increased in women with gonadal dysgenesis? placed. Uterine tone seems to be increased, and (A) serum FSH concentrations there are occasional variable decelerations of (B) serum TSH concentrations the fetal heart to 90 BPM. Which of the follow- ing is the most appropriate management? (C) serum prolactin concentrations (D) serum estradiol concentrations (A) tocolysis with a β-receptor agonist (E) serum testosterone concentrations (B) Pitocin induction of labor (C) continued monitoring of mother and 87. A 45-year-old woman has bilateral breast pain baby that is most severe premenstrually. On palpa- (D) amniotomy tion, there is excessive nodularity, tenderness, (E) cesarean section and cystic areas that diminish in size after menses. Which of the following is the most Questions 84 through 86 likely diagnosis?

84. A pediatrician asks you to see a 13-year-old (A) fibrocystic disease girl who has not begun to menstruate and has (B) fibroadenomas a 6-month history of pelvic and lower abdom- (C) intraductal papilloma inal pain at approximately 1-month interval. (D) breast cancer She has Tanner stage 3 pubic hair and breast (E) engorgement attributable to increased development. Her growth spurt occurred about prolactin 1 year ago and her current height is 65 in. (165 cm). She has never had sexual intercourse. 88. Which feature of fibrocystic disease of the On examination of her external genitalia the breast is associated with the greatest risk of presence of Tanner stage 3 pubic hair is con- developing breast cancer? firmed. The labia are normal. There is no obvi- ous vaginal opening and there is bulging (A) number of nodules between the labia minora. Which one of the (B) serous nipple discharge following is the most likely diagnosis? (C) size of the dominant mass (A) androgen insensitivity syndrome (D) presence of epithelial hyperplasia (B) (ovulation pain) (E) presence of a palpable axillary node (C) Müllerian and vaginal agenesis (D) gonadal dysgenesis (E) imperforate hymen (c) ketabton.com: The Digital Library

62 2: Obstetrics and Gynecology

89. A 35-year-old woman at 30 weeks’ gestation is the most accurate definition of engagement discovers a lump in her left breast. Examination in a woman with a vertex presentation? reveals a 2–3 cm, firm nodule in the upper (A) The vertex has passed through the outer quadrant. Which of the following is the pelvic inlet. most appropriate next step in the management of this patient? (B) The vertex reaches the pelvic floor. (C) The biparietal diameter has passed (A) observation until after delivery through the pelvic inlet. (B) thermography (D) The biparietal diameter has reached the (C) application of hot packs pelvic floor. (D) breast ultrasound (E) The vertex is at plus 1 station. (E) fine-needle aspiration 93. In which of the following circumstances is the 90. A 1-cm carcinoma of the breast is diagnosed by administration of anti-D immune globulin not an excisional biopsy in a 36-year-old woman at necessary? 14 weeks’ gestation. The axillary nodes are neg- (A) threatened abortion and first-trimester ative. Which of the following is the best man- bleeding agement of this patient? (B) genetic amniocentesis at 16 weeks’ (A) terminate the pregnancy immediately gestation and treat the breast cancer (C) at 28 weeks (B) monitor the mass throughout pregnancy (D) at 40 weeks with the onset of labor with serial breast ultrasounds (E) after delivery of an Rh-positive fetus (C) induce labor at 34 weeks’ gestation, then give chemotherapy 94. She has an uncomplicated spontaneous vaginal (D) perform a cesarean delivery at 36 weeks delivery with an estimated blood loss of 450 and treat the breast cancer mL. Her infant is Rh positive and she receives (E) modified radical mastectomy at the time 300 μg of Rh immunoglobulin (one vial) within of diagnosis 72 hours after delivery. This dose is sufficient to prevent Rh isoimmunization after what amount Questions 91 through 94 of fetal red cells enters the maternal circulation? A woman at 31 weeks’ gestation complains of feel- (A) 5 mL ing dizzy and lightheaded when she lies on her (B) 30 mL back. She is Rh negative but denies vaginal bleeding, (C) 100 mL abdominal trauma, or abdominal pain. The diagno- (D) 200 mL sis is probably the supine hypotensive syndrome. (E) 300 mL 91. This results in which of the following? 95. Why does ligation of the hypogastric (internal (A) a decreased fetal heart rate iliac) artery effectively control intractable pelvic (B) an increased frequency of uterine hemorrhage? contractions (A) There is no collateral circulation to the (C) a decreased tolerance to pain uterus. (D) a decreased effect of epidural analgesia (B) Uterine blood flow is stopped. (E) an increased risk of placental abruption (C) Arterial pulse pressure to the uterus is reduced. 92. During this woman’s labor, the nurse describes (D) Clotting in uterine capillaries is enhanced. the presenting part as engaged when the (E) Blood flow is shunted to the ovarian woman is 6 cm dilated. Which of the following veins. (c) ketabton.com: The Digital Library

Questions: 89–101 63

Questions 96 through 99 seen on speculum examination. The uterus is pres- ent but small. There are no adnexal masses. A 53-year-old woman with five adult children com- plains of losing urine shortly after coughing or 99. Which of the following is the most likely jumping. She occasionally loses urine while lying in diagnosis? bed if she happens to cough vigorously. She is unable to stop the urine once it has begun to flow. (A) anorexia nervosa (B) androgen insensitivity syndrome 96. Which of the following is the most likely diag- (C) Turner syndrome (gonadal dysgenesis) nosis? (D) Müllerian agenesis (A) GSI (E) premature ovarian failure (B) a (C) a urethrovaginal fistula 100. What is the most likely karyotype of this 16 year old? (D) an atonic bladder (E) detrusor dyssynergia (A) 46,XX (B) 46,XY 97. Which of the following would most likely con- (C) 45,X firm the cause of this woman’s incontinence? (D) 47,XX trisomy 21 (A) a urine culture (E) 47,XXY (B) a Q-tip test (C) urethroscopy 101. Which of the following is the treatment of choice for this patient? (D) urethrocystometry (E) an intravenous pyelography (IVP) (A) estrogen replacement therapy (B) estrogen plus progestin therapy 98. Which of the following is the treatment of (C) clomiphene choice for this woman’s urinary incontinence? (D) Follistim (recombinant FSH) (A) a course of nitrofurantoin (E) gonadectomy (B) a selective serotonin reuptake inhibitor (SSRI) DIRECTIONS (Questions 102 through 110): For (C) oxybutynin chloride (Ditropan) each item select the ONE best lettered option that is the most likely diagnosis of vaginal bleeding in (D) vaginal hysterectomy and anterior pregnancy. Each lettered option may be selected colporrhaphy once, more than once, or not at all. (E) a Marshall-Marchetti-Krantz urethropexy (A) threatened abortion (B) gestational trophoblastic disease Questions 99 through 101 (C) cervicitis A 16-year-old girl presents because she has not (D) placenta previa begun to menstruate. Also, breast development and (E) placental abruption pubic hair have not developed. She is 59 in. (150 cm) (F) uterine rupture tall and weighs 115 lbs (52 kg). On examination, her vital signs are normal. She has skin folds on the lat- (G) placenta accreta eral sides of her neck. She has evidence of cubitus (H) uterine inversion valgus. Breasts are Tanner stage 1 and the nipples (I) uterine atony appear to be spaced wider than average. Pubic hair (J) vaginal laceration is Tanner stage 1. The external genitalia are normal. (K) tubal pregnancy The vagina is of normal depth and a small cervix is (c) ketabton.com: The Digital Library

64 2: Obstetrics and Gynecology

102. A 28-year-old pregnant woman at 32 weeks’ 107. A 21-year-old woman last menstruated 6 weeks gestation suddenly begins profuse, painless ago. Her menses are usually every 28–30 days. vaginal bleeding. Her prenatal care began at 7 She has a past history of chlamydia. One week weeks and had been uncomplicated. She last ago she had a positive home pregnancy test. had sexual intercourse 7 days ago. She denies She complains of mild left lower quadrant pain. abdominal trauma. Her uterus is soft and non- Quantitative serum chorionic gonadotropin tender, and the fetal heart rate is 132 BPM. (hCG) concentrations 2 days ago and today are 6850 and 7685 mIU/mL, respectively. No 103. A 23-year-old pregnant woman at 16 weeks’ intrauterine pregnancy is identified by trans- gestation begins to have light vaginal bleeding vaginal ultrasound. several hours after sexual intercourse. She has no abdominal discomfort. Her prenatal course 108. A 29-year-old pregnant woman at 38 weeks’ has been uneventful and fetal heart tones were gestation presents to your labor and delivery heard at 12 weeks. Her uterus is midway unit complaining of dizziness, heavy vaginal between her symphysis pubis and umbilicus, bleeding, and loss of fetal movement. She had and is soft and nontender. Fetal heart tones are been having uterine contractions for approxi- now 148 BPM. Her cervix is undilated. mately 4 hours, but these stopped when the bleeding began. Her previous pregnancy was 104. A 15-year-old girl began her last menstrual delivered by classical cesarean section because period (LMP) 14 weeks ago. She had a positive of a transverse lie. pregnancy test 6 weeks ago. On examination, her uterine fundus is at the level of the umbili- 109. A 31-year-old woman has an uncomplicated cus. Fetal heart tones are not heard and no fetus labor and vaginal delivery of a healthy 3400-g is seen by abdominal ultrasound. male infant. However, her placenta has not yet delivered 2 hours after the delivery of her child. 105. A 39-year-old pregnant woman with chronic Under appropriate anesthesia manual extrac- hypertension at 37 weeks’ gestation presents tion of the placenta is attempted, but the pla- with heavy vaginal bleeding and complains of centa is removed in fragments. She continues to lower abdominal pain. She also has noted have excessive vaginal bleeding after manual diminished fetal movement since the bleeding removal of her placenta. Her first child was began 3 hours previously. On physical exami- delivered by a low transverse cesarean section nation her uterus is firm (rock hard) and very because of fetal distress. tender. Fetal heart tones are in the range of 160–170 BPM. 110. A 34-year-old woman, gravida 5, has a 17-hour first stage, a 3.5-hour second stage ending with 106. A 32-year-old pregnant woman has a low for- a spontaneous vaginal delivery of a 4400-g ceps vaginal delivery after a 3-hour second infant, and a 15-minute third stage of labor. stage of labor. One hour later the nurse informs Immediately after the placenta delivers, she has you she has excessive vaginal bleeding. On profuse vaginal bleeding. On examination, her examination, her uterus is firm and nontender, perineum is intact and there are no vaginal or and the uterine fundus is at the umbilicus. cervical lacerations. Her uterus is soft and the uterine fundus is 4–5 cm above her umbilicus. (c) ketabton.com: The Digital Library

Answers and Explanations

1. (C) The incidence of ovarian cancer in OC users sensitivity of breast tissue to the low levels of is 50% less than that found in nonusers. The circulating estradiol in prepubertal girls. The incidence of PID is also decreased by 50% in disorder occurs most commonly before the OC users. The risk of endometrial cancer is age of 3 years. The estradiol concentration decreased by 50% after 1 year of OC use, and may be normal in young girls ingesting estro- the protective effect seems to persist after stop- gen if the serum estrogen concentration is not ping the OC. In well-controlled studies, there is obtained at the time the estrogen is ingested. A no increase in the risk of having a child with a negative medication history is helpful to major malformation, cardiac malformation, or exclude this possibility. The absence of a pal- limb abnormality. The risk of ectopic pregnancy pable lower abdominal mass and a prepuber- is reduced by 90%, perhaps because the risk of tal concentration of estradiol exclude a any pregnancy approaches zero when the OC granulosa cell tumor. Adrenal 21-hydroxylase is taken correctly. (Speroff and Fritz, 2005, pp. deficiency and polycystic ovary syndrome are 894–895, 901–902, 904–905) function disorders that require the stimulation of adrenocorticotropic hormone (ACTH) and 2. (D) Breast development in an infant or young pituitary gonadotropins (FSH and LH), respec- child is the consequence of increased estrogen tively, to become clinically apparent. Neither secretion, exposure to exogenous estrogens, or disorder appears until after the onset of increased response of breast tissue to normal, puberty. Moreover, both are associated with prepubertal amounts of estrogen. After exclud- androgen excess and masculinization, not ing exposure to exogenous estrogens (e.g., OCs, estrogen excess and precocious breast devel- estrogen creams), increased response to estro- opment. (Speroff and Fritz, 2005, pp. 372–384) gen is more common than increased estrogen secretion from the ovaries or adrenal glands 4. (C) Premature thelarche is a benign, self-limited when breast development is the only sign of disorder that does not progress. Breast devel- precocious puberty. The uterus and adnexa can opment may actually regress, though the be palpated abdominally in prepubertal girls if regression may not be complete. The girl and they are pathologically enlarged. For this her parents should be assured that the events of reason, an estrogen-secreting ovarian tumor puberty will be normal at a normal age. (granulosa cell is the most common type) is Examination of the girl should be repeated at usually palpable, and an ultrasound examina- 3- to 6-month intervals for about 1 year to be tion is unnecessary. For the same reason, a certain that additional pubertal events do not pelvic examination under anesthesia is not nec- occur (such as growth of pubic hair, acceler- essary, especially if the serum estradiol con- ated linear growth, and vaginal bleeding). centration is normal. CT scan of the head and Because pituitary and adrenal functions are a serum FSH concentration are unnecessary if normal for a prepubertal girl, therapy with a breast development is the only sign of preco- GnRH agonist (Lupron, Synarel, and so forth) cious puberty, and the serum estradiol concen- or a corticosteroid is ineffective and inappro- tration is normal in the prepubertal range. priate. Although breast cancer is a rare possi- (Speroff and Fritz, 2005, pp. 372–384) bility in prepubertal girls, the presence of bilateral breast buds effectively excludes this 3. (E) Premature thelarche is a disorder that diagnosis. A breast biopsy may destroy breast probably occurs as a consequence of increased analge, and these girls will not have breast

65 (c) ketabton.com: The Digital Library

66 2: Obstetrics and Gynecology

development at puberty. (Speroff and Fritz, 2005, fetus is increased until the pregnancy is within pp. 372–384) 2–4 weeks of delivery. Intrauterine infections via transplacental or transmembrane trans- 5. (A) In general, gynecologic cancers confined mission of the herpesvirus are rare. Most infec- to the organ of origin are stage I. Thus, this tions of the infant occur after passage through patient has a stage I cancer. In 1988, FIGO an infected birth canal. A cesarean section revised the staging of endometrial cancer from should be performed at term if the amniotic a clinical staging to surgical staging. Cancer membranes rupture in a woman with an active limited to the endometrium is stage IA. herpetic lesion in the genital area, regardless of Myometrial invasion less than 50% is stage IB, the duration of membrane rupture. (ACOG and myometrial invasion more than 50%, but Practice Bulletin, 2007) not involving the serosa, is stage IC. (Hoskins et al., 2005, p. 829) 8. (C) Vulvar carcinoma must be considered in any postmenopausal woman with pruritus, 6. (D) The onset of preeclampsia before the 20th especially in the presence of a visible lesion. week of pregnancy is clinically seen only with a The appropriate management is to biopsy the hydatidiform mole. Advanced maternal age, lesion after disinfecting the area and infiltrating uterine size greater than gestational weeks, and with 1% Xylocaine. A 3–4 mm dermal punch is the absence of a fetal heartbeat are added fea- useful to obtain the biopsy. Colposcopy alone is tures to suggest gestational trophoblastic dis- less reliable for vulvar lesions compared to cer- ease. Hydramnios, which can be associated with vical abnormalities because the technique anencephaly and other fetal developmental requires the topical application of 3–5% acetic abnormalities, also predisposes to preeclampsia, acid, which penetrates a keratinized squamous but its onset does not occur before 24 weeks. epithelium (the vulva) less than a nonkera- Renal disease, diabetes mellitus, and chronic tinized squamous epithelium (the cervix). hypertension also increase the likelihood of Multiple biopsies should be obtained for a preeclampsia, but not before 24 weeks of preg- large, confluent lesion or a multifocal vulvar nancy. The incidence of preeclampsia is increased lesion. Wide local excision may be appropriate in twin gestation, but again, its onset is not before for small lesions, but is more difficult in an out- 24 weeks. (Cunningham et al., 2005, pp. 276–277) patient setting. Vulvectomy is reserved for women with biopsy-proven vulvar carcinoma. 7. (A) Acyclovir prescribed from 36 gestational (Hoskins et al., 2005, pp. 669–670) weeks until after delivery reduces the proba- bility of a cesarean section, although in one 9. (D) Cancer of the cervix that has not invaded study of a small number of patients there were cervical stroma is stage 0 carcinoma in situ. no cases of neonatal herpes in either the treat- Cancer that has invaded the cervical stroma ment or control group. Nonetheless, acyclovir but has not spread beyond the cervix is stage I. (a class C drug) and newer antiviral drugs Involvement of the upper vagina or parametria (valacyclovir, famciclovir, both class B) should (but not to the pelvic sidewall) is stage II. Stage be given to women with either a primary out- III is involvement of the lower third of the break or a recurrence during pregnancy. vagina (IIIA) or parametria to the pelvic side- Weekly cultures are unreliable to exclude active wall (IIIB). Extension outside the reproductive herpes lesions in pregnancy and are not rec- tract is stage IV. (Hoskins et al., 2005, pp. 743–748) ommended for basing a decision to perform a cesarean section. A cesarean section should be 10. (C) The intent of staging is to judge the results performed if a woman develops an active cer- of various treatments and to compare treatment vical or vaginal lesion at term. However, geni- results worldwide. Because advanced proce- tal herpetic lesions before 36 weeks do not dures such as venography, lymphangiography, necessitate a cesarean section, because there is MRI or CT scans, and laparoscopy are not uni- no evidence that vertical transmission to the versally available, staging of cervical cancer (c) ketabton.com: The Digital Library

Answers: 5–14 67

remains primarily clinical. Such tests as cys- pathology, such as fibroids, endometriosis, toscopy, proctosigmoidoscopy, barium enema, ovarian cysts, pelvic adhesions, and others. IVP, and plain radiographs of the abdomen and Anorgasmia is usually a psychological disor- chest are permitted. Evidence of mucosal der best dealt with by a psychologist or psy- cancer confirmed by biopsy at the time of cys- chiatrist expert in sexual counseling and toscopy changes her diagnosis to stage IV cer- therapy. Sexual or physical abuse should be vical cancer. (Hoskins et al., 2005, pp. 743–748) considered and the woman questioned about this possibility. Vulvar vestibulitis is an inflam- 11. (B) Stage of the cancer is the most important matory condition of uncertain (perhaps multi- prognostic factor. Women with stage IAcervical ple) etiology and is usually treated with topical cancer have a 95% 5-year survival. This glucocorticoids. Failure of medical therapy may decreases progressively to 80, 64, 38, and 14% require surgical excision of the affected area. for stages IB, II, III, and IV, respectively. The (McGuire and Hawton, 2004, pp. 1–24) other choices are prognostic factors but relate to the stage of the cancer. Women with high-risk 14. (D) The fundal height corresponds to 16 gesta- serotypes (strains) of HPV tend to develop cer- tional weeks. Between 15 and 20 weeks, screen- vical neoplasia at a younger age. More ing for open neural tube defects should be advanced stages of cervical cancer tend to have offered. In addition to MSAFP, the American less differentiated tumors and a greater proba- College of Obstetricians and Gynecologists rec- bility of pelvic and paraaortic lymph node ommends hCG and unconjugated estriol to metastasis. Pelvic lymph node metastasis will screen for Down syndrome and trisomy 18 as be found in 5% or fewer of women with stage I well. This triad of tests is called a triple screen or cervical cancer, with a progressive increase as triple marker screen. Reported sensitivity of the the stage advances to 55% of women with stage triple screen is between 57 and 67% and the false IV cervical cancer. (Hoskins et al., 2005, pp. 758–763) positive rate is 5%. An abnormal result must be evaluated further by ultrasonography to identify 12. (B) Breast discomfort is a problem premenstru- the presence or absence of open neural tube ally for many women. Simple palliative meas- defects or abdominal wall defects (increased ures include administration of vitamin E, 600 MSAFP) or trisomy disorder (decreased MSAFP units daily, and limiting methylxanthines by and unconjugated estriol, increased hCG). In eliminating coffee and other caffeine-containing skilled hands, an ultrasound reduces the risk of substances, although the mechanism of action is such an anomaly by 95%. If the diagnosis is still not well understood. Danazol (Danocrine), in uncertain, the woman should be offered amnio- doses of 200–400 mg daily, is often effective in centesis for measurement of alpha-fetoprotein relieving breast pain. Clomiphene may have (AFP) and acetylcholinesterase activity estrogenic side effects and worsen breast pain. (increased in neural tube defects) and karyotype Many women receiving progesterone note of fetal skin cells. Although testing for HIV can breast discomfort secondary to fluid retention. be done any time, it is most appropriate at the There is no evidence that diuretics such as first prenatal visit, because earlier onset of pro- hydrochlorothiazide relieve breast pain. In one phylaxis with acquired immune deficiency syn- comparison study, tamoxifen (a selective estro- drome (AIDS) drugs reduces the risk of gen receptor modulator) was more effective transmission to the fetus significantly. Routine than danazol. (Speroff and Fritz, 2005, pp. 588–589) culture for GBS is not recommended because of the high recurrence rate after treatment and the 13. (B) Vaginismus is the painful, involuntary low attack rate to the fetus. Amniocentesis is spasm of the musculature of the pelvis and not a screening procedure and is reserved for lower third of the vagina. It may respond to those women with a specific indication, such as properly administered dilator therapy, although elevated MSAFP, low MSAFP (risk of Down there is a lack of conclusive evidence. Deep- syndrome), advanced maternal age, and others. thrust dyspareunia is often the result of pelvic A glucose tolerance test may be appropriate if (c) ketabton.com: The Digital Library

68 2: Obstetrics and Gynecology

there is a clinical indication for diabetes mellitus: condition that occurs in up to 0.2% of deliver- previous macrosomic infant or stillbirth, strong ies and is marked by confusion, bizarre behavior, family history of diabetes mellitus, persistent disordered thoughts, delusions, and hallucina- glycosuria, previous gestational diabetes, or ele- tions. There is a high risk of suicide or harm to vated random serum glucose concentration. others and immediate psychiatric care is (Creasy et al., 2004, pp. 237–239; Cunningham et al., required. Postpartum mood disorders are more 2005, pp. 207–213) common in adolescents but cannot be blamed on “typical teenage adjustment.” This patient’s 15. (C) The nonpitting edema of her legs is likely the case fits most closely the definition of postpar- result of lymphedema. This may cause discom- tum depression, using the same DSM-IV fort or pain in her legs, but not hypesthesia. The (Diagnostic and Statistical Manual of Mental most likely cause of the peripheral neuropathy Disorders, 4th Edition) criteria as major depres- is the carboplatin. Toxicity at doses higher than sion. Five lakh postpartum women in the 100 mg/m2 limit its use and also limit the abil- United States have postpartum depression ity to study various doses alone and in combi- annually. Treatment should include antide- nation with other chemotherapeutic agents, pressant therapy (generally using SSRIs) and such as paclitaxel (Taxol), which may also cause psychotherapy as indicated. (Creasy et al., 2004, peripheral neuropathy. Nonetheless, the com- pp. 1193–2000) bination of tumor debulking, pelvic and paraaortic lymph node dissection, combination 18. (E) Although many mechanisms involving the paclitaxel and carboplatin, and radiation offers fetal pituitary axis, placental membranes, the longest disease-free interval. (Hoskins et al., decidual secretions, and fetal-placental inter- 2005, pp. 498–499) action have been investigated, no mechanism has been established for the initiation of labor 16. (D) The mean age of onset of any pubertal event in humans. Cortisol mechanisms probably ini- is approximately 11 years, beginning with the tiate labor in sheep, and sheep generally are appearance of breast buds. Pubic hair appears the experimental animals used to study human approximately 6 months later, and this is fol- parturition. It has, however, been shown that lowed by the peak height velocity (greatest rate this mechanism does not incite labor in of linear growth per unit time). Six to 12 months humans. Women who are pregnant with an later, menstrual bleeding begins. Increased rate anencephalic fetus often do not begin labor of growth begins early in the pubertal process. until after 42 weeks, but they do begin sponta- The sequence of pubertal events and the neous labor even in the absence of a fetal pitu- approximate age of appearance of each event is itary gland. The most current thinking is that a sufficiently predictable that significant varia- fetal-placental-uterine interaction initiates tion in age of onset or sequence should lead to labor. It is uncertain exactly how the pieces fit an evaluation of a cause of abnormal puberty. in this puzzle. (There is a comprehensive and (Speroff and Fritz, 2005, pp. 365–372) complex review of the physiologic and bio- chemical processes of human parturition in 17. (C) Postpartum mood disorders are much more Cunningham et al.) (Cunningham et al., 2005, common than previously believed. Postpartum pp. 151–186; Scott et al., 2003, p. 35) blues (also called maternity blues or baby blues) occurs in most women within the first 2 weeks 19. (D) The first site of hematopoiesis in the fetus is of delivery and is characterized by irritability, the yolk sac. Between 12 and 24 weeks’ gestation, mood lability, and anxiety. This condition is gen- the fetal liver makes the largest contribution. erally resolved within 2 weeks. Hypothyroidism After 28 weeks, the fetal bone marrow is the most can mimic postpartum depression or contribute important site. (Cunningham et al., 2005, pp. 103–106) to it, but this diagnosis is based on laboratory studies and is not as common as postpartum 20. (C) Hirsutism occurs when a woman is depression itself. Psychosis is a very serious exposed to increased amounts of biologically (c) ketabton.com: The Digital Library

Answers: 15–24 69

active androgens, or when hair follicles are are unable to produce androgens or estrogens extrasensitive to normal amounts of androgens. in normal amounts. Women with 21-hydroxy- Women with regular menstrual intervals usu- lase deficiency may have salt wasting and ally have familial hirsutism, and it usually begins hypotension if the enzyme deficiency is suffi- at or soon after puberty. Hirsutism associated ciently severe. (Speroff and Fritz, 2005, pp. 510–511) with menstrual disturbances usually means exposure to increased amounts of androgens, 22. (D At serum hCG concentrations above the dis- either endogenous secretion from the ovaries or criminatory zone (usually about 4000 mIU/mL), adrenal glands or ingestion of a drug with transvaginal sonography should reveal an androgenic effects. The amenorrhea suggests intrauterine pregnancy. The absence of such a increased androgen exposure, while the nega- finding suggests either an extrauterine preg- tive drug history suggests an endogenous nancy or a spontaneous abortion. Higher levels source. Testosterone may arise from the ovaries, of hCG are necessary before an extrauterine ges- the adrenal glands, and from extraglandular for- tational sac may be seen by sonography. At each mation. A serum testosterone concentration is week of gestation, hCG concentrations normally not helpful to distinguish which source of andro- vary by a large amount. For this reason, a single gen is responsible for hirsutism. Furthermore, measurement is not helpful, although serial the serum testosterone level is often mislead- measurements to determine whether the hCG ingly low, because increased production rates of fails to double in 48 hours is helpful to suggest testosterone stimulate an increase in the rate of a failing pregnancy (ectopic or intrauterine). removal (the metabolic clearance rate) of testos- Nonclotting blood obtained from the cul-de-sac terone from the circulation. The degree of hir- by a culdocentesis may be the result of a rup- sutism is the best gauge of the amount of tured ectopic pregnancy or a ruptured ovarian excessive androgen production. Most virilizing cyst. An adnexal mass is palpated in only 50% of ovarian tumors are palpable in young women, women with an ectopic pregnancy. (Scott et al., and a pelvic ultrasound is useful only when the 2003, pp. 92–94) bimanual examination is inadequate. Elevated prolactin levels may cause amenorrhea but do 23. (B) The only agents currently recognized to not cause hirsutism. There is no use for meas- enhance production of fetal pulmonary surfactant urement of urinary androgen (17-ketosteroid or are glucocorticoids. There is good evidence that 17-ketogenic steroid) excretion in modern gyne- pulmonary immaturity is reduced by 50% when cology. The best next step is to measure a serum corticosteroids are given to mothers at a gesta- DHEAS concentration, because it is elevated in tional age less than 31 weeks. Also, there is evi- adrenal disorders and normal or only slightly dence that neonatal death is decreased by about elevated in ovarian causes of hirsutism. (Speroff 50% with corticosteroid therapy, and other major and Fritz, 2005, pp. 504–519) infant morbidity is reduced as well (intraven- tricular hemorrhage, necrotizing enterocolitis). 21. (B) A history of irregular menses from menar- To achieve these benefits, delivery must be che suggests a functional disorder, such as delayed 48 hours. Of the agents listed, magne- polycystic ovary syndrome or attenuated adre- sium sulfate can prevent eclamptic seizures and nal hyperplasia attributable to an inherited may inhibit uterine contractions. The other enzyme deficiency. The absence of a unilateral agents have no role in the treatment of fetal ovarian mass on pelvic examination, the posi- lung immaturity. (Creasy et al., 2004, pp. 456–460) tive family history, and the early menarche favor a diagnosis of attenuated adrenal hyper- 24. (B) The LH surge classically triggers ovulation plasia over that of a virilizing ovarian tumor. 14 days before the onset of the subsequent men- Women with 17-hydroxylase deficiency or strual period. Subtract 14 days from the typical 11-hydroxylase deficiency are hypertensive. cycle length to estimate the cycle day of the LH Women with 17-hydroxylase deficiency are surge and ovulation. It is pertinent to remind also sexually infantile, not hirsute, because they this woman that her probability of conceiving in (c) ketabton.com: The Digital Library

70 2: Obstetrics and Gynecology

each cycle is no higher than 15–20%, even with had a previously affected child. It is becoming intercourse timed to the preovulatory LH the standard of care to screen routinely preg- surge. (Speroff and Fritz, 2005, pp. 212–216) nant women for the cystic fibrosis gene. Currently, routine screening will identify 25. (D) GSI occurs when there is immediate invol- approximately 80% of carriers of the cystic fibro- untary loss of urine with increased intravesical sis gene. (Creasy et al., 2004, pp. 968–971) pressure greater than maximal urethral pres- sure in the absence of detrusor contractions. 28. (D) The most likely diagnosis is a cyst of the These women can usually stop the flow of urine canal of Nuck. These arise from inclusions of by voluntary contraction of the muscles that the peritoneum at the inferior insertion of the close the urethra. Loss of urine with a strong round ligament into the labia majora. They are desire to void immediately suggests urge incon- analogous to a spermatic cord hydrocele and tinence, often occurring as a result of detrusor are typically found at the superior aspect of contractions. Loss of urine associated with the labia majora. Vulvar varicosities usually seemingly unrelated conditions should raise involve most of the labia, occur in older and the suspicion of a drug-associated incontinence. parous women, and have a classical “bag of Maximal bladder distention and greatly worms” appearance. Given the physical find- increased bladder capacity suggest a diagnosis ings, a hernia is unlikely. An ultrasound may be of an atonic bladder with overflow inconti- useful to distinguish a hernial sac from a cyst of nence. (Scott et al., 2003, pp. 845–846, 849–856) the canal of Nuck. One-third of women with a cyst of the canal of Nuck may have a coexistent 26. (D) Women with amenorrhea owing to weight inguinal hernia. (Scott et al., 2003, p. 622) loss and stress have decreased hypothalamic secretion of GnRH, and secondarily decreased 29. (C) Different-sex twins must be dizygous. serum levels of FSH and LH. As a consequence, Prenatal ultrasound can detect monochorionic, serum estradiol levels will be low. While monoamnionic twins, and these must be women with weight loss amenorrhea may have monozygous. For same-sex twins, careful exam- mild hirsutism, it is probably the result of a ination of the amniotic membranes after birth decreased estrogen secretion and decreased can reveal monozygous twins if the placental estrogen:androgen ratio, rather than an membranes are monochorionic. Dichorionic increase in serum testosterone levels. (Speroff membranes can occur with either monozygous and Fritz, 2005, pp. 438–449) or dizygous twins. Ultimately, assessment of DNA polymorphism is the best way to deter- 27. (D) With improved care, women with cystic mine twin zygosity. (Creasy et al., 2004, pp. 65–66) fibrosis now survive into the reproductive age and are capable of carrying a pregnancy suc- 30. (C) The maximum number of oocytes is 6–7 cessfully. No special precautions such as pro- million at approximately 20 gestational weeks. longed hospitalization, oxygen supplementation, At birth, the number of oocytes has decreased bed rest, or others are necessary. Likewise, there to about 1 million, and the number at puberty is no need for routine cesarean section or other is 300,000–500,000. Women at menopause still labor modifications, except ensuring adequate have a small number of oocytes, a number hydration and normal serum electrolytes. An insufficient to produce an amount of estrogen amniocentesis is unnecessary. There is no con- to prevent vasomotor symptoms. By simple stituent of amniotic fluid that is diagnostic of mathematics, women lose approximately 1000 cystic fibrosis. Also, the fetus is at risk for cystic oocytes per menstrual cycle: one by ovulation fibrosis only if the father is a carrier. If not, the and the remainder by follicular atresia. (Speroff fetus will be a carrier only. Chorionic villus and Fritz, 2005, pp. 106–107) biopsy can be done to determine whether the fetus has cystic fibrosis if the father carries one 31. (D) Approximately 20% of pregnant women of the 150+ alleles for cystic fibrosis or the couple have positive rectovaginal cultures. Vertical (c) ketabton.com: The Digital Library

Answers: 25–35 71

transmission rate from mother to baby is Müllerian ducts (didelphic uterus, with two approximately 75%. A rectovaginal culture is and two vaginal canals separated by a not indicated at the first prenatal visit because longitudinal septum); (C) partial fusion of the the pregnant woman may subsequently con- Müllerian ducts (bicornuate uterus); (D) failure vert from a negative to a positive culture. For of dissolution of the fused medial walls of the this reason, all pregnant women should have a Müllerian ducts (septate uterus); and (E) DES rectovaginal culture at 36–37 gestational weeks. exposed uterus. A fetus exposed to DES (or any If the woman goes into preterm labor before a estrogen) in the first trimester will often culture is obtained, she should be treated with develop a T-shaped uterine cavity. The patho- penicillin (gentamicin if she is penicillin aller- physiology of this abnormality is unknown. In gic). The rate of neonatal sepsis is 1% of colo- this patient, a single cervix and a double uter- nized mothers, a serious infection that may ine fundus indicate a bicornuate uterus. (Speroff cause pneumonia, meningitis, and death. and Fritz, 2005, pp. 1079–1080) (Creasy et al., 2004, pp. 754–757) 34. (B) Obesity, advanced age, and hepatic disease 32. (D) Diaphragms can be used successfully for are associated with an increased risk of contraception with proper patient education endometrial adenocarcinoma. While post- and motivation. Proper fitting is most impor- menopausal bleeding is most commonly tant for efficacy. The diaphragm should be caused by atrophic changes in the genital tract, coated with spermicide prior to insertion cancer must be considered. Cervical cytology (within the dome and along the rim), and and examination of endometrial histology are inserted no more than 6 hours before inter- absolutely indicated. The risk of endometrial course is planned. Conversely, the device cancer is increased approximately threefold in should be left in place at least 6 hours but no diabetic women, and obese women have a more than 24 hours after intercourse. If multi- three- to fourfold increased risk. High parity is ple episodes of intercourse take place, addi- a risk factor for cervical cancer; low parity is a tional spermicide should be used. The vaginal risk factor for ovarian and endometrial cancer. ring is designed to be worn for 3 consecutive Postmenopausal bleeding is a sign of ovarian weeks (i.e., one ring, not three rings changed cancer only if the malignancy secretes estro- weekly) and then removed for 1 week. Depo- gen to stimulate the endometrium. An office Provera does not affect liver enzymes but actu- endometrial biopsy has a sensitivity of about ally increases the seizure threshold, making it 90%. If postmenopausal bleeding persists, a a great choice for patients with seizure disor- D&C with hysteroscopy should be done. A D&C ders. Amenorrhea is common (20–60%) with alone samples about 50% of the endometrium. patients using the levonorgestrel IUD, and is so For this reason, many gynecologists are per- effective in preventing pregnancy that this forming a hysteroscopy and directed endome- symptom should not raise alarm (as long as trial biopsy in addition to a D&C. (Hoskins et al., other pregnancy symptoms—e.g., nausea and 2005, p. 824) breast tenderness—are not present). Over 10 years, tubal ligation failure rates approach 1 35. (D) Blood pressure tends to drop slightly in per 100 procedures (as opposed to 1 per 1000). normal pregnancy. This woman’s blood pres- (Speroff and Fritz, 2005, p. 842, 946, 962, 984, 1007) sure of 132/86 mmHg is definitely higher than would be expected and suggests the possibility 33. (C) A single cavity uterus forms from fusion of of chronic hypertension. Because the GFR in paired Müllerian ducts followed by dissolu- pregnancy increases normally by as much as tion of the fused medial walls. Uterine anom- 50% to a peak of approximately 160 mL/min, alies can be divided into five distinct categories: serum creatinine and BUN should be less than (A) failure of formation of one or both 0.9 and 13 mg/100 mL, respectively. The Müllerian ducts (unicornuate uterus or absent observed values in this patient are elevated uterus, respectively); (B) failure of fusion of the for pregnancy. The renal threshold for glucose (c) ketabton.com: The Digital Library

72 2: Obstetrics and Gynecology

normally decreases in pregnancy. Therefore, infant is increased in women with long-standing glycosuria does not always mean diabetes in diabetes and vascular disease, but not in pregnancy. Several plasma glucose measure- women with gestational diabetes. There is a ments should be obtained in pregnant women slight increase in the frequency of fetal macro- with glycosuria to correlate urinary and plasma somia (birth weight over 4000 g), though shoul- glucose levels. (Creasy et al., 2004, pp. 111–118) der dystocia and brachial plexus injury are infrequent. (Cunningham et al., 2005, pp. 1170–1176) 36. (A) The current treatment guideline from the Centers for Disease Control and Prevention for 39. (D) Preterm premature rupture of membranes uncomplicated gonococcal infections is ceftri- is a relatively common condition, affecting axone 125 mg IM one time. Cefixime 400 mg 3–18.5% of all pregnancies. It is estimated that orally is an alternative. Each is given as a single 30% of all preterm deliveries result from dose. Importantly, the quinolone class, for PPROM. There are multiple etiologies for example, ciprofloxacin is no longer considered PPROM, including ascending vaginal infec- appropriate treatment for gonococcal infections tion. Carriers of GBS, bacterial vaginosis, and due to drug resistance. To the chosen drug is gonorrhea are all at increased risk for PPROM. added azithromycin, 1 g orally, or doxycycline, Maternal age is not a risk factor, nor is parity, 100 mg orally twice daily for 7 days. The maternal weight, maternal weight gain, or second drug is added to treat C. trachomatis, trauma. According to most experts, patients which is present in almost 50% of women with with this condition should be managed in the gonorrhea. Sexual partners should be treated at hospital due to the high risk for amniotic infec- the same time. (CDC MMWR 2006: Vol. 55, pp. tion, preterm labor, and umbilical cord com- 42–49; CDC MMWR 2006: Vol. 56, pp. 332–336) pression or prolapse. Pulmonary hypoplasia and fetal compression malformations are seen 37. (E) Of the choices listed, a biophysical profile is when rupture of membranes occurs in the pre- the best assessment of fetal well-being. This viable period (less than 24 weeks). The duration assesses multiple fetal variables: breathing of latency (time from rupture of membranes to movement, body or limb movements, tone and delivery) varies inversely with gestational age. posture, fetal heart rate pattern, and amniotic In other words, at term, labor generally begins fluid volume. A NST, oxytocin challenge test, within hours. However, even at 28 weeks, up to and fetal movement counts assess only one 90% of patients will go into labor within 1 week. determinant of fetal well-being. An amniocen- (Creasy et al., 2004, pp. 723–739) tesis has no value in assessing fetal well-being, but may be appropriate to determine fetal lung 40. (A) Multiple randomized-controlled trials have maturity if induction of labor before 40 gesta- now demonstrated the benefit of administer- tional weeks is indicated because of her chronic ing antibiotics to women with PPROM at less illnesses. Fetal Doppler studies to assess sys- than 32 weeks’ gestation. Most importantly, tolic:diastolic (SD) ratio may be a better test of these drugs prolong the latent period until labor fetal well-being and a significant decrease or begins, but reductions have also been noted in reversal of the ratio is an indication for delivery. maternal infection, fetal infection, fetal respira- (Creasy et al., 2004, pp. 362–376) tory distress syndrome, and fetal intraventric- ular hemorrhage. Commonly used antibiotics 38. (B) Unlike women with overt or pregestational are ampicillin and erythromycin, but efficacy diabetes mellitus, the risk of fetal anomalies is has been noted with many different regimens. not increased in women with gestational dia- Cesarean delivery at this point is not indicated, betes. Stillbirth rates are increased in women but might need to be performed in case of non- with gestational diabetes if their fasting plasma reassuring fetal status or malpresentation (e.g., glucose concentrations are elevated, but not breech). Induction of labor generally takes place with elevated postprandial glucose concentra- between 32 and 34 weeks if the patient’s status tions only. The risk of a growth-restricted remains stable, or sooner in the event of amniotic (c) ketabton.com: The Digital Library

Answers: 36–44 73

infection or other concerns. Amniocentesis may and completely reversible. Therefore, this ther- be performed to look for evidence of amniotic apy is useful as an adjunct to surgery in improv- infection, but the likelihood of fetal lung matu- ing hemoglobin or allowing a vaginal approach rity at this point is remote. Patients with pre- rather than abdominal. No specific size limit viously placed cervical cerclages may be exists for removal of a single myoma. (Scott et al., candidates for expectant management with the 2003, pp. 875–887) cerclage in place, but it would be inappropriate to place a cerclage after PPROM. (Creasy et al., 43. (E) Endometriosis is a complex condition affect- 2004, pp. 728–739) ing women of reproductive age, most com- monly diagnosed between the ages of 20 and 40. 41. (C) Leiomyomata lead to more hysterectomies True incidence rates are hard to estimate because than any other gynecologic condition and can be direct visualization is necessary to confirm the found in up to 50% of all women. Most are diagnosis, but the disease likely affects 7–10% of asymptomatic, but symptoms can include heavy women. Because smoking is associated with menstrual bleeding, pelvic pain, “pressure reduced body mass index and reduced estro- symptoms,” and even preterm birth. Symptoms gen levels, it is negatively associated with the depend on where the fibroids are located—those risk of endometriosis. Symptoms include dys- closest to the endometrial cavity cause more menorrhea (most common), dyspareunia (i.e., problems with bleeding and dysmenorrhea, pain with intercourse), or dyschezia (i.e., pain while those closest to the serosa can be expected with defecation). There appears to be little cor- to cause more pressure symptoms such as blad- relation between the amount of disease noted at der frequency or constipation. However, the typ- the time of surgery and the patient’s report of ical bleeding disturbance associated with uterine pain. Infertility affects up to 30–50% of women fibroids is heavy regular bleeding as opposed to with endometriosis and surgical resection bleeding or spotting between periods, which appears to be more effective in promoting fer- deserves separate evaluation. Leiomyosarcomas tility than does medical management alone. In are malignant tumors that appear very similar to order, the most common sites for endometriosis benign leiomyomata—however, they are now implants are the ovaries, anterior and posterior felt to arise de novo and not from degeneration cul-de-sacs, broad ligaments, uterosacral liga- of a benign fibroid. Leiomyomata do seem to ments, uterus, tubes, colon, and appendix. (Scott respond to sex steroids but oral contraceptives et al., 2003, pp. 713–720) do not cause more rapid growth or regression. (Scott et al., 2003, pp. 869–875) 44. (A) The quadrivalent human papillomavirus (under the name Gardasil) was licensed for use 42. (A) As mentioned, leiomyomata account for in June 2006. It is currently recommended for more hysterectomies than any other gynecologic routine vaccination of young women ages disorder, but alternative treatments continue to 11–12 with “catch-up” vaccination for women be explored. Myomectomy (e.g., removal of 13–26. It can be given as young as age 9. The uterine fibroids) can be performed via laparo- vaccine is three separate 0.5-mL doses, given at tomy, laparoscopy, hysteroscopy, or even vagi- 0, 2 and 6 months apart. If one dose is delayed, nally. However, it is generally not considered a there is no need to restart the schedule. Rather, “simpler” or safer procedure than hysterectomy, the doses should be given as soon as possible. and bleeding can be excessive. Uterine artery The vaccine is targeted against the L1 proteins embolization is an angiographic procedure cur- of HPV types 6, 11, 16, and 18. HPV 6 and 11 are rently reserved for women who do not desire “low-risk” strains causing genital condylo- future pregnancy, as the effects of embolizing mata, and HPV 16 and 18 are “high-risk” onco- both uterine arteries and then allowing preg- genic strains responsible for 70% of all cervical nancy is uncertain. GnRH agonists are useful in cancer. Guidelines for screening for cervical reducing uterine bleeding and reducing fibroid cancer have not changed—patients should still volume up to 50%, but this effect is short-lived be advised to have routine Pap smear screening (c) ketabton.com: The Digital Library

74 2: Obstetrics and Gynecology

and HPV screening as indicated. Women with a and 297 days after the onset of the LMP. prior history of abnormal Pap smears are still (Cunningham et al., 2005, pp. 208–209) candidates for the vaccine, as it is unlikely that they have been exposed to all four strains. (Centers 47. (E) Calculation of the EDD is by using for Disease Control and Prevention, 2007, pp. 1–23) Naegele’s rule. Seven days are added to the first day of the LMP, then 3 months are sub- 45. (D) In many states, a pregnant woman under tracted. Application of Naegele’s rule is accu- the age of 21 years is considered an emanci- rate only for women who have regular pated minor and is the only person who may menstrual cycle intervals of 28–30 days. make legal decisions pertaining to the preg- (Cunningham et al., 2005, pp. 208–209) nancy. Although an immediate cesarean sec- tion is indicated because of the severe fetal 48. (D) Postcoital spotting and intermenstrual heart rate decelerations, to perform it without spotting in a woman with cyclic menses is sug- her permission violates the ethical principle of gestive of a cervical abnormality, rather than an autonomy. This is a principle that states that endometrial hormonal abnormality. In the human beings should have their wishes absence of a visible lesion, a Pap smear that respected as autonomous persons if they are includes cells from both the ectocervix and capable of self-determination. Obtaining a endocervix is the preferred method of evalua- court order may fulfill the ethical principle of tion, especially when the woman has no history beneficence, a physician acting to do no harm of cervical pathology or a normal Pap smear in and to help the patient. In this situation, the the recent past. Endometrial biopsy and hys- ethical (moral) decision is complicated by a teroscopy assess the endometrium, not the conflict between beneficence and autonomy. cervix. A conization of the cervix should be However, proceeding with a cesarean section reserved for women with documented cervical exposes the obstetrician to a legal charge of neoplasia when determination of the extent of battery. Assigning her care to another physician the lesion is necessary. An endocervical curet- is a standard and accepted solution when there tage is usually reserved for women with unsat- is a moral conflict between patient and physi- isfactory Pap smears and persistent abnormal cian. However, this is not an acceptable option bleeding. (Scott et al., 2003, p. 925) in an emergency situation. The obstetrician is at risk for abandonment. Although not a satisfy- 49. (B) Pap smear and colposcopy are screening ing choice, the choice most ethically sound is to tests appropriate when there is no visible cer- counsel her carefully, but eventually accede to vical pathology. In the presence of a lesion, her wishes. Placing her in the lateral position, pathologic evaluation is necessary to make a giving her oxygen by mask, and providing ade- diagnosis. An office cervical biopsy is the pro- quate intravenous hydration should be insti- cedure of choice to establish the diagnosis. If tuted to minimize the risk of fetal hypoxia. the diagnosis from the biopsy is cancer, a (Scott et al., 2003, pp. 1037–1042) conization of the cervix is indicated to deter- mine the extent of the disease surface spread as 46. (D) The mean duration of human pregnancy is well as depth of stromal invasion. As a gen- 266 days from conception. To this is added 14 eral principle, cytology is a screening tool, not days for the interval between the onset of the a diagnostic test, and any visible lesion (vulvar, last menses and the conception date. Thus, it is vaginal, or cervical) should be biopsied for a important to ascertain the range of days for definitive diagnosis. (Scott et al., 2003, pp. 926–927) each woman’s menstrual cycles. The more vari- able a woman’s menstrual cycles are, the less 50. (E) The patient described in the question has a certain is the estimated due date calculated classic history of an incompetent cervix: expul- from the LMP. The standard deviation of preg- sion of a fetus without labor. It is believed to be nancy duration is ±17 days. Thus, 95% of caused by previous cervical trauma, DES expo- human pregnancies will deliver between 263 sure, or, most commonly, a congenital defect in (c) ketabton.com: The Digital Library

Answers: 45–54 75

cervical stroma. In the absence of preterm labor, with overflow incontinence from those with there is no indication for terbutaline or other GSI; the latter are able to voluntarily increase tocolytic agents. DES is contraindicated in urethral pressure enough to stop urine flow. pregnancy, but was used in the past to treat Cystitis commonly causes urgency and repeated pregnancy loss. Hydroxyprogesterone increased urinary frequency, but not inconti- is a progestational compound that is being nence. Urinalysis and urine culture are not used by some hospitals for patients in prema- likely to be revealing in this patient, but should ture labor, but its use is controversial. Bed rest be done routinely in all incontinent women. is occasionally encouraged by some practi- Instillation of methylene blue into the bladder tioners for patients with a history of prema- after placement of a vaginal tampon should be ture deliveries. The probability of a successful done when a vesicovaginal fistula is suspected. pregnancy after a cervical cerclage increases This occurs most often following gynecologic from 20% to approximately 80%. It is crucial to surgery and should be suspected in women eliminate the possibility of preterm labor before complaining of constant urine leakage. The placing a cerclage. (Creasy et al., 2004, pp. 603–619) Q-tip test is useful to demonstrate posterior urethral rotation found in women with GSI. 51. (E) Production of AFP begins in the yolk sac (Scott et al., 2003, pp. 849–856) and then moves to the fetal liver and, to a lesser extent, the fetal gastrointestinal tract. Choices 53. (A) In a patient with GSI, a retropubic approach (A) through (D) result in an increased maternal offers the best long-term cure of the inconti- serum AFP (MOM greater than 2.0) because all nence. The Burch procedure and the Marshall- are open defects of the fetus that result in an Marchetti-Krantz procedure are the most increase in amniotic fluid concentrations and common retropubic procedures. With an ante- then maternal serum concentrations of AFP. rior colporrhaphy, plication sutures are placed Trisomy 21 (Down syndrome) is associated at the UVJ in an effort to support and elevate it. with a decreased MSAFP. In clinical practice, Long-term results are not as good as a retrop- measurement of MSAFP is combined with ubic urethropexy or a suburethral sling. A sub- serum chorionic gonadotropin (hCG) and urethral sling procedure is used when urethral

unconjugated estriol (E3). These three tests are closing pressure is low, less than 20 cmH2O. A commonly called a triple screen or triple needle suspension procedure is most often marker screen and together improve the sensi- done when there is associated genital prolapse tivity over each test alone. Approximately 60% with potential incontinence. Collagen injections of trisomy 21 fetuses in women under age 35 at the UVJ have been attempted to obstruct the years, and more than 75% in women over 35 urethra partially. Incontinent patients who may will be detected using a multiple marker benefit the most from collagen injections are screening test. Screening for these defects is those with intrinsic sphincter deficiency and a most sensitive between 16 and 18 gestational fixed bladder neck. (Scott et al., 2003, pp. 856–866) weeks, but the test should be offered to all preg- nant women between 15 and 22 gestational 54. (C) The presence of a dominant mass requires weeks. (Cunningham et al., 2005, pp. 318–324) immediate evaluation. While all women with a dominant mass should have a mammogram, 52. (E) The combination of aging and diabetes sug- this is a screening test. A fine-needle aspiration gests the likelihood of a neurologic defect in the is a diagnostic tool that will resolve whether the bladder, resulting in overflow incontinence. This mass is cystic or solid. Any fluid or tissue occurs when the detrusor muscle becomes hypo- obtained should be sent for cytologic evalua- tonic or atonic. Such women complain of voiding tion to further aid in the diagnosis. If clear or small amounts but still having the feeling of a full cloudy fluid is aspirated, and the mass disap- bladder. In addition, these women are inconti- pears, the woman should have a repeat breast nent of small amounts of urine and are unable to examination in 1 month. If the mass remains stop the flow. This helps to distinguish those after aspiration, if the fluid is bloody, or if there (c) ketabton.com: The Digital Library

76 2: Obstetrics and Gynecology

is a residual mass on a follow-up visit in 1 month, 57. (D) From the information in question 56, it is an open biopsy should be done. A segmental apparent that the fetus is relatively resistant to resection is a therapeutic option for a circum- teratogenic effects of drugs until about 2 weeks scribed carcinoma, but is not an appropriate after conception. A recommendation to abort diagnostic tool. (Scott et al., 2003, pp. 895–898) the pregnancy cannot be made on medical probability, although the woman may choose 55. (D) Rh immune globulin should always be this, because she does not wish to take any administered to an Rh-negative pregnant chance of having an affected child. Ultrasound woman who sustains any trauma or has any is incapable of detecting anomalies until at least type of invasive procedure, such as an amnio- 12–14 postmenstrual weeks. The fetal warfarin centesis. Detectable fetomaternal hemorrhage syndrome does not cause chromosomal abnor- occurs in 6% of women having an amniocen- malities, and a genetic amniocentesis is not tesis and 1% of Rh-negative women will indicated. Vitamin K reverses the anticoagu- develop Rh isoimmunization after amniocen- lant effects of coumadin but does not alter the tesis (without Rh immune globulin). The risk that the fetus will develop anomalies. immune globulin reduces the risk of subse- (Cunningham et al., 2005, pp. 342–344, 349) quent Rh sensitization during the pregnancy, which could result in severe erythroblastosis 58. (B) Heparin is the drug of choice for anticoagu- fetalis. Although chorionic villus biopsy might lation in pregnancy. Little of it crosses the pla- be an alternative to amniocentesis, it is done centa, and it is not associated with congenital earlier in pregnancy, and occasionally must be birth defects. Experience with low molecular followed by an amniocentesis after 14 weeks’ weight heparin in pregnancy is increasing and gestation because of the possibility that mater- appears to be safe for mother and fetus. In full nal decidua was analyzed. Rubella immuniza- therapeutic doses, low molecular weight heparin tion should be given after delivery to avoid the offers the advantage of less or no monitoring of theoretical risk of a congenital rubella syndrome its anticoagulant effect. Coumadin readily from the administration of the live vaccine. The crosses the placenta and is associated with birth presence of hepatitis B surface antibody sug- defects in 15–25% of fetuses exposed throughout gests immunity to hepatitis B but is unrelated to the first trimester. Aspirin is ineffective as an amniocentesis. Intra-amniotic bleeding is a anticoagulant, although the risk of maternal or complication of amniocentesis but occurs at the fetal bleeding (e.g., placental abruption, fetal time of the procedure. The amniotic fluid will intracranial bleeding) is increased. There is no appear bloody. (Creasy et al., 2004, pp. 262–263) clinical experience with TPA in pregnancy. Because pregnancy itself is a thrombogenic con- 56. (D) The conceptus is remarkably resistant to the dition, anticoagulation throughout pregnancy is toxic and teratogenic effects of most drugs until indicated. Vena caval filters offer no advantage about 2 postconceptual weeks (4 postmenstrual over heparin and require an invasive procedure. weeks). Although certain drugs may be toxic to (Cunningham et al., 2005, pp. 1081–1084) oocytes, their effect will be to prevent conception or cause an early spontaneous abortion. The 59. (C) The station of the vertex indicates that the developing conceptus is not exposed to mater- fetal head is on the perineum. A cesarean sec- nal toxins or teratogens until after implantation tion, either low transverse or classical, is inap- and establishment of a blood supply from propriate unless an operative vaginal delivery mother to fetus. Even after implantation, the is unsuccessful. In women with an anthropoid fetus is relatively resistant to teratogens for pelvis, the transverse, interspinous diameter about 1 week. Organogenesis is complete by of the bony pelvis is narrow, and the antero- the end of the first trimester. Congenital posterior diameter of the pelvis is relatively abnormalities are, therefore, unlikely in the long. In this circumstance, a forceps rotation second and third trimesters. (Cunningham et al., should not be done and delivery should be in 2005, pp. 342–344) the occiput posterior. The indication for forceps (c) ketabton.com: The Digital Library

Answers: 55–63 77

is maternal exhaustion; women with an anthro- 62. (C) Cervical cancer is currently the only female poid pelvis usually have a spontaneous vaginal reproductive tract cancer staged clinically delivery. In women with a gynecoid pelvis, the according to FIGO standards. FIGO also transverse and anteroposterior diameters are requires that the clinical staging be based on more equal, and rotation of the fetal head to technologies generally available worldwide, occiput anterior would be an acceptable choice. including third world countries. For this Soft-tissue resistance to delivery is not great reason, lymphangiography, angiography, CT enough that an episiotomy will permit slight or MRI scans, laparoscopy, or hysteroscopy expulsive efforts by the mother to deliver the are not permitted to stage cervical cancer. fetal head. (Cunningham et al., 2005, pp. 511–513, Stage I cancer is confined to the cervix. Stage 548–555) IA is microscopic cancer without a visible lesion. Stage IB is macroscopic cancer visible 60. (C) Fecundability is the ability to achieve a to the eye. Stage IB is further subdivided into pregnancy, and the rate per ovulation in cou- stage IB1 (clinically visible lesion 4.0 cm or ples with no impediment to fertility is approx- less in greatest dimension) and stage IB2 (clin- imately 20–25%. This figure is reduced as the ically visible lesion more than 4.0 cm in great- woman enters her early 30s, if sexual inter- est dimension). Stages II–IV have spread course occurs fewer than one to two times per beyond the cervix. Stage IIB is lateral spread week, or if the couple use coital lubricants in into the parametria, but not extending to the the periovulatory part of her menstrual cycle. pelvic sidewall. Because of the presence of Using the 20% fecundability rate, 20 of 100 cou- abnormal paraaortic lymph nodes and ples will achieve a pregnancy in the first men- hepatic changes consistent with metastases, strual cycle of effort. In the second cycle, 20% of she is actually a stage IVB. (Hoskins et al., 2005, the remaining 80 couples will achieve a preg- pp. 747–751) nancy, that is 16 women. This is a cumulative pregnancy rate of 36%. Twenty percent of the 63. (E) Although this 46-year-old woman is staged remaining 64 women will conceive in the third as a IIB, she should be treated as a stage IVB cycle, approximately 13 women. The cumula- because of the findings on CT scan. Methods of tive conception rate after three cycles is approx- staging that are similar allow institutions to imately 49%. Continuing this calculation, compare results of treatment without having to 90–95% of normally fertile couples will achieve account for different staging procedures and a pregnancy within 12 ovulations. Based on criteria. A simple TAH-BSO is appropriate ther- this calculation, infertility is defined as the apy only for women with carcinoma in situ of inability to conceive after 12 ovulatory cycles. the cervix (CIN III, stage 0). Women with stage (Speroff and Fritz, 2005, p. 1013) I or IIA may be treated with radical hysterec- tomy or with radiation therapy. Beyond stage 61. (B) While the probability of conception per IIA, only radiation therapy is acceptable. A ovulation is approximately 20%, about 50% pelvic exenteration is indicated when there is of all conceptions in humans end as a preg- a central recurrence after maximal dose radi- nancy loss. Of this percentage, 35% of preg- ation therapy. Platinum-based chemotherapy nancies end so early that women never realize has been used for women with metastases or they conceived. The remaining 15% are rec- recurrence after radiation therapy. It is con- ognized pregnancy losses, most in the first sidered palliative. Also, some suggest that a trimester. Stated otherwise, only 10% of lymphadenectomy be performed before the women who conceive will have a liveborn start of radiation. Recently, several have used child (50% of the 20% conception rate per ovu- chemotherapy as primary therapy for bulk lation). Fecundity in the future (probability of disease. There are no randomized-controlled having a liveborn child) is not reduced after a trials to document that chemotherapy is supe- single pregnancy loss. (Speroff and Fritz, 2005, rior to surgery or radiation. (Hoskins et al., 2005, p. 1013) pp. 763–767) (c) ketabton.com: The Digital Library

78 2: Obstetrics and Gynecology

64. (C) As many as 30% of fetuses exposed to of the myometrium. Persistent bleeding from phenytoin had minor craniofacial and digital the uterus despite these measures may indicate anomalies. Cleft lip/palate, hypertelorism, uterine rupture, retained placental fragments, broad nasal bridge, and epicanthal folds are the or placenta accreta. If a careful curettage of the craniofacial anomalies observed. Hypoplasia of uterine lining fails to remove any placental frag- the distal phalanges and nails are the digital ments and decrease uterine bleeding, hypogas- anomalies. In addition, these infants may have tric artery ligation or a hysterectomy must be growth and cognitive deficiencies. Trimetha- considered. (Scott et al., 2003, pp. 48–49) dione, another anticonvulsant, causes similar anomalies. Spina bifida occurs in 1–2% of 68. (D) Both the bloody nipple discharge and the infants whose mothers took valproic acid microcalcifications are indications for a breast during pregnancy. (Cunningham et al., 2005, pp. biopsy. Although there are benign-appearing 348–349; Scott et al., 2003, p. 138) radiographic calcifications, clusters of calcifi- cation are associated with a 25% chance of a 65. (B) Approximately 3% of live born infants have cancer. An image-guided percutaneous biopsy a major congenital anomaly detected at birth. is preferred because a fine-needle biopsy has The incidence increases to 6–7% later in child- about a 20% false negative rate. Cytology is a hood. Chromosomal and single-gene defects screening tool. In the presence of significant account for 10–25% of human malformations. risk factors for cancer, a tissue diagnosis is Fetal infections (3–5%), maternal disease (4%), mandatory. Imaging studies are also screening and drugs and medications (<1%) account for tools with a false negative and a false positive the remaining recognized causes of human rate, making such studies inappropriate for malformations. Sixty-five to seventy-five diagnosis. (Scott et al., 2003, pp. 893–898) percent of malformations have an unknown or multifactorial etiology. (Creasy et al., 2004, 69. (B) The advantages of a fine-needle aspiration pp. 329–330, 1296) of a breast mass are that it can distinguish between a cystic and solid lesion, and it reduces 66. (B) The main mechanism by which hemostasis the number of open breast biopsies when it is is achieved following delivery is contraction positive for cancer. However, a negative needle of the myometrium to compress the uterine biopsy is nondiagnostic (and nonreassuring), vessels that had been supplying the placenta. and an open biopsy is still necessary. A fine- Lack of effective myometrial contraction (i.e., needle biopsy does not differentiate between uterine atony) is the major cause of postpartum noninvasive and invasive cancer, nor does it hemorrhage. If the uterus is found to be firmly delineate the extent of in situ disease. Most contracted, then other factors, such as cervical breast surgeons will not perform definitive sur- or vaginal lacerations or a coagulopathy, must gery (e.g., mastectomy or lumpectomy with be sought. (Scott et al., 2003, pp. 48–49) lymph node dissection) without histologic con- firmation of cancer: core-needle biopsy, surgical 67. (A) Immediate management is bimanual mas- biopsy, or frozen section at the time of lumpec- sage and compression of the uterine fundus by tomy or mastectomy. (Scott et al., 2003, pp. 895–896) placing one fist into the anterior vaginal fornix and the other hand abdominally posterior to the 70. (D) The factor associated with the greatest life- uterus. The uterine massage is often enough to time risk for developing breast cancer is age of cause myometrial contractions and slowing of the woman. Hereditary breast cancers account the bleeding. Oxytocin or an ergot alkaloid (e.g., for 5–10% of all breast cancers and give the methylergonovine) should then be administered woman a relative risk of approximately 2. The if bimanual massage of the uterus is ineffective. relative risk is 4 with two first-degree relatives. Insertion of a gauze pack is never indicated Increased lifetime estrogen exposure is a minor because it is rarely effective. It may actually risk factor for breast cancer. Obesity, early worsen the bleeding by preventing contraction menarche, late menopause, and low parity are (c) ketabton.com: The Digital Library

Answers: 64–75 79

associated with an increased lifetime estrogen toms has been achieved in approximately 70% exposure and are minor risk factors for breast of women in small studies, and seems to be cancer. The prevalence of breast cancer more effective in women with cyclic rather than increased from 30 per 10,000 women years (no continuous pain. Bromocriptine inhibits pro- hormone replacement) to 38 per 10,000 women lactin secretion, not recognized as a cause of years (women on hormone replacement), fibrocystic breast disease and mastodynia. Oral according to data from the Women’s Health progestins (e.g., medroxyprogesterone acetate), Initiative. This increase was not statistically sig- depot medroxyprogesterone acetate (Depo- nificant, but the hormone arms of the study Provera), or OCs may provide symptomatic were stopped after 5.5–6 years because a relief, but symptoms usually return after these prestudy threshold defined by the investiga- are stopped. Hydrochlorothiazide provides tors’ Data Safety Monitoring Board was unpredictable relief of symptoms. (Speroff and exceeded. Stated otherwise, the risk of breast Fritz, 2005, pp. 588–589) cancer increases from 3.3 to 4.1 per 1000 women using hormone replacement. Estrogens are con- 74. (D) This is an autosomal dominant disorder. sidered promoters of breast cancer rather than Both parents are carriers of the abnormal gene inducers or initiators. (Hoskins et al., 2005, p. 1081; (N), which is on chromosome 17. If each parent Scott et al., 2003, pp. 890–893) is a heterozygote (Nn, where n is the normal gene), 25% of their offspring will have a normal 71. (D) Infiltrating (invasive) ductal carcinoma genotype, nn. Fifty percent of their offspring will accounts for 65–80% of all breast carcinomas. be affected heterozygotes (Nn), and the remain- Infiltrating lobular carcinoma accounts for ing 25% will be homozygous affected (NN). If 10–14%, and the others 5% or less. (Scott et al., either parent or both are homozygous for the 2003, p. 901) abnormal gene (NN), 100% of the offspring will be affected. As an autosomal disorder, there is no 72. (A) In women with a primary tumor of 1–1.5 sex predilection; males and females are affected cm or less, the recurrence risk is approxi- with equal frequency. Prenatal diagnosis is avail- mately 15%, primarily because the likelihood able. (Creasy et al., 2004, p. 260) of complete excision is greatest. Women with positive sex-steroid receptors in the tumor 75. (A) The normal pH of the vagina is 3.8–4.2. In also have a better prognosis for several rea- women with a vaginal discharge, a pH less sons: the tumor tends to be better differenti- than 5.0 suggests monilial vaginitis or a phys- ated, and 5-year therapy with tamoxifen iologic discharge of normal squamous cells significantly reduces recurrence. Although desquamated from the vaginal epithelium. A survival decreases as the number of positive pH greater than 5.0 suggests some type of bac- axillary lymph nodes increases, the role of terial infection, such as bacterial vaginosis or sentinel lymph node biopsy and axillary lym- trichomonas vaginitis. The diagnosis of bac- phadenectomy has become controversial. terial vaginosis is based on the presence of 3 There appears to be a trend against axillary of 4 characteristics: pH greater than 4.5, a lymphadenectomy, especially with ductal car- homogenous thin appearance of the vaginal cinoma in situ less than 1.5 cm diameter. discharge, a fishy amine odor after the addi- (Hoskins et al., 2005, pp. 1116–1119) tion of 10% potassium hydroxide (KOH) to the discharge, and clue cells present in 20–50% 73. (D) Danazol, in oral doses of 100, 200, or 400 of vaginal epithelial cells. Clue cells are bac- mg daily for 4–6 months, relieves breast pain teria adherent to the surface of vaginal epithe- and reduces nodularity in 90% of women. The lial cells. Lactobacilli are absent from the beneficial effects often last for several months vagina in women with bacterial vaginosis. after discontinuation of the drug. Tamoxifen is Both chlamydia and gonorrhea infect the a synthetic antiestrogen that competes with cervix and do not change the vaginal pH. (Scott estrogen receptors in the breast. Relief of symp- et al., 2003, pp. 585–589) (c) ketabton.com: The Digital Library

80 2: Obstetrics and Gynecology

76. (E) Clue cells are shown in Figure 2-1. This secretion of normal amounts of cortisol, andro- indicates bacterial vaginosis. Clue cells are gens, and estrogen, but an increased secretion vaginal squamous cells with indistinct margins of mineralocorticoids. Men with Klinefelter that are studded extensively with coccobacilli. syndrome have patent vasa deferentia and Trichomonas infection is caused by a unicellu- seminal vesicles; their ejaculate volumes will be lar protozoon. The organism on wet smear with normal and contain fructose. (Speroff and Fritz, normal saline is fusiform, slightly larger than 2005, pp. 1143–1146) white blood cells, and has flagella at one end. The flagella cause the motion on wet smear 79. (B) The WHO suggests a minimal sperm con- that is diagnostic. Monilial vaginitis is best centration of 20 million/mL for normal con- demonstrated by placing a small amount of ception rates of 15–20% per ovulation. the discharge in 10% KOH and observing for However, sperm motility (percentage and branching hyphae. N. gonorrhoeae and C. tra- velocity; >50% with forward progression), chomatis cannot be seen on a wet smear. (Scott sperm morphology (30% or more oval forms, et al., 2003, pp. 588–589) using strict criteria), coital frequency, and others must be considered. Stated otherwise, a 77. (C) The treatment of choice for bacterial vagi- sperm concentration of 10 million/mL may be nosis is metronidazole, also an effective treat- associated with normal fertility if the sperm ment for trichomonas vaginitis. The dose is motility and morphology are better than aver- 375–500 mg orally twice daily for 1 week. A age and coital frequency is three to four times single daily dose of 750 mg was recently per week. Any abnormality of a semen analy- approved. Vaginal metronidazole gel or clin- sis should be confirmed by a repeat semen damycin cream are also approved forms of analysis no sooner than 4 weeks after the initial treatment. Concurrent therapy of the male part- analysis. (Speroff and Fritz, 2005, p. 1144) ner is controversial. Treatment in pregnancy is recommended, because there is a potential 80. (D) The cycle of spermatogenesis is 73 ± 5 days. association of bacterial vaginosis and preterm This is the time required for maturation of labor and delivery. (Cunningham et al., 2005, pp. spermatogonia to spermatozoa. The cycle is at 1319–1320; Scott et al., 2003, pp. 588–589) different stages along the seminiferous tubules, necessary to ensure the presence of sperm in 78. (D) The normal ejaculate volume is 2–5 mL, each ejaculate. Further, spermatozoa require and the bulk of the ejaculate is from the semi- approximately 3 weeks to traverse the ductal nal vesicles. The reduced ejaculate volume may system and appear in the ejaculate. Knowing be the result of an incomplete collection or may this has important implications: any therapy indicate absence of the seminal vesicles. intended to stimulate spermatogenesis must be Fructose is the reducing sugar produced by the continued for at least the duration of one sper- seminal vesicles, and its absence establishes a matogenic cycle to determine whether there is diagnosis of congenital bilateral absence of the a beneficial effect. While impaired spermatoge- vasa deferentia and seminal vesicles. This nesis may occur late in the cycle of spermato- explains the azoospermia (absence of sperm; genesis and improvement with clomiphene aspermia is absence of an ejaculate). Men with may occur sooner, the semen analysis should germ cell aplasia have only Sertoli cells in their still be delayed for 90 days to provide better seminiferous tubules. Their ejaculate volumes evidence for the presence or absence of are normal, and fructose is present. Likewise, improvement in the semen analysis. Results men with occlusion of the vasa deferentia will must be interpreted with caution because there be azoospermic but have a normal ejaculate is great biological variability in semen param- volume containing fructose. Men with 17α- eters: what is interpreted as a therapeutic effect hydroxylase deficiency will have hypertension, may only be natural biological variation. be sexually infantile, and have azoospermia, Ultimately, pregnancy is the only meaningful because the enzyme deficiency prevents the measure of treatment success and pregnancy (c) ketabton.com: The Digital Library

Answers: 76–87 81

may have occurred despite the therapy, not 84. (E) The presence of pubertal events except because of it. (Speroff and Fritz, 2005, pp. 1135–1139) external menstruation and the bulging of the hymen causing retrograde menstruation as a 81. (E) Painful vaginal bleeding is most likely the cause of her cyclic pain establish the diagnosis result of placental abruption, premature sepa- of imperforate hymen. Strictly, these women ration of the placenta. Bloody show is a normal have cryptomenorrhea, not amenorrhea. sign of impending or early labor. The bleeding Women with mittelschmerz (ovulation pain) is scant and intermingled with clear mucus. menstruate cyclically. While pubertal events Bleeding from a vaginal laceration following except menstruation occur normally in women coitus is not associated with abdominal pain. A with Müllerian/vaginal agenesis, cyclic pain history of coitus followed immediately by is absent because the uterus is hypoplastic or bleeding suggests this diagnosis. Bleeding from absent and there is no endometrium to shed. cervicitis is most often spotting and not associ- Women with gonadal dysgenesis do not ated with abdominal pain. Classically, bleeding develop breasts or pubic hair and have pri- with a placenta previa is painless. (Creasy et al., mary amenorrhea. The most common form of 2004, pp. 713–720) gonadal dysgenesis is 45,X Turner syndrome and affected females are less than 62 in. tall. 82. (D) Maternal hypertension is the most common (Emans et al., 2005, pp. 239–240) risk factor for a placental abruption. The rela- tive risk is 3.8 for parous women and 1.6 for 85. (B) The androgen insensitivity syndrome is nulliparous women. In one published report, the result of abnormal testosterone receptors half of the women with an abruption severe in genetic males with functioning testes. The enough to kill the fetus had hypertension, and testes secrete normal amounts of testos- half of these had evidence of chronic vascular terone after puberty and adult male concen- disease. Advanced maternal age without con- trations are found. Serum LH concentrations founding factors such as diabetes or hyperten- are normal or even elevated, presumably sion is not a risk factor for placental abruption. because of the testosterone receptor defect in High parity is associated with an increased risk the hypothalamus and pituitary gland. As a of placental abruption. Vigorous coitus can result, secretion of LH is often increased. cause a vaginal laceration, but not abdominal Serum prolactin and TSH concentrations are pain. While blunt abdominal trauma may normal because absent testosterone recep- cause a placental abruption, routine forms of tors have no effect on pituitary secretion exercise are not a risk factor for placental of these hormones. (Speroff and Fritz, 2005, abruption. (Creasy et al., 2004, pp. 713–715) pp. 421–423)

83. (E) At term, a placental abruption severe 86. (A) Women with gonadal dysgenesis have enough to cause fetal distress warrants imme- fibrous streaks instead of functioning gonads. diate delivery. If the pregnancy is remote from As a result secretion of estradiol and testos- term, temporizing measures may be consid- terone is decreased. Serum FSH concentrations ered, such as observation. However, delivery are increased because of the lack of negative should be achieved if the mother becomes feedback to the hypothalamus and pituitary as hemodynamically unstable. Tocolysis is inef- a result of diminished sex steroid secretion by fective in relaxing the uterus and has the the streak gonads. Other pituitary hormones, added disadvantage of causing vasodilation such as TSH and prolactin are not regulated of an already under-filled vascular system. by sex steroids and their concentrations are Amniotomy and Pitocin induction will not normal. (Emans et al., 2005, pp. 237–239; Speroff and cause delivery rapidly enough to prevent fur- Fritz, 2005, pp. 345–348) ther deterioration of the fetus. Evidence of fetal distress makes continued monitoring unac- 87. (A) The classic symptom of fibrocystic breast ceptable. (Creasy et al., 2004, pp. 716–717) disease is cyclic bilateral breast pain. The pain (c) ketabton.com: The Digital Library

82 2: Obstetrics and Gynecology

and associated diffuse breast engorgement is 90. (E) Breast cancers in young women tend to be most severe premenstrually. Cystic changes aggressive tumors and estrogen receptor neg- palpated premenstrually typically are smaller ative, both of which worsen the prognosis. For postmenstrually. Fibroadenomas are firm, rub- this reason, the cancer should be treated surgi- bery, freely mobile, solid, and usually solitary cally, usually a modified radical mastectomy to masses. Intraductal papilloma does not cause minimize the need for adjuvant radiation or diffuse breast symptoms. Spontaneous and chemotherapy with wide local excision or a intermittent nipple discharge is the classic lumpectomy. There is little convincing evidence sign of an intraductal papilloma. Intraductal that termination of pregnancy improves the carcinoma is more likely if there is a discharge prognosis. (Cunningham et al., 2005, pp. 1259–1261; from multiple ducts. Breast cancer should be Hoskins et al., 2005, pp. 1296–1298) suspected when a solitary firm nodule does not change throughout the menstrual cycle. A 91. (A) In late pregnancy, the large uterus com- mammogram is helpful, but any suspicious monly compresses the inferior vena cava and mass should be biopsied. Hyperprolactinemia impedes return of blood from the lower can cause breast engorgement, but the pain is extremities to the heart. This may be sufficient usually mild, and cystic areas tend not to vary to reduce cardiac output. In approximately 10% in size. (Scott et al., 2003, p. 898) of women, arterial hypotension occurs, which can result in diminished uteroplacental blood 88. (D) Fibrocystic disease includes a variety of flow and a decreased fetal heart rate. None of histologic findings. Typical is proliferation and the other options occur as a result of this syn- hyperplasia of the lobular, ductal, and acinar drome. Management is to have the woman roll epithelium. Histologic variants include vari- on to her side or lean forward if she is sitting. able-sized cysts, adenosis, fibrosis, duct ecta- Both these maneuvers cause the uterus to fall sia, apocrine metaplasia, and others. Ductal away from the inferior vena cava. (Cunningham epithelial hyperplasia and apocrine metaplasia et al., 2005, p. 135) with atypia are the findings associated with the greatest risk of subsequent breast cancer. 92. (C) The strict definition of the cardinal move- The presence of histologic atypia increases the ment of labor, called engagement, is given as woman’s chance of breast cancer fivefold. (Scott choice C. Among other things, this means that et al., 2003, p. 898) the presenting part is fixed in the true pelvis and a prolapsed umbilical cord is unlikely to 89. (E) Breast cancer is rare in women younger occur. Often, the fetal head is considered to be than 35 years, and the approximate incidence engaged when the vertex is at 0 station, the of breast cancer in pregnancy is 1 in 3000 level of the ischial spines. Although engage- deliveries. Survival rates from breast cancer in ment is conclusive evidence of an adequate pregnancy are less than in nonpregnant pelvic inlet, its absence is not always indicative women of comparable age. This is due to of pelvic contraction. Nevertheless, the inci- delayed diagnosis, not a biological effect of dence of pelvic contraction is higher in primi- pregnancy. Pregnant women, when matched gravid women whose presenting part is not for disease stage, have identical survival rates engaged. (Cunningham et al., 2005, pp. 426–427) as nonpregnant women. Despite the preg- nancy, the presence of a dominant mass 93. (D) Anti-D immune globulin should be given requires histologic evaluation. Temporizing at the time of any vaginal bleeding, trauma, or measures (choices A through D) are inappro- invasive procedure (e.g., amniocentesis) during priate in the presence of a dominant mass. pregnancy. Although maternal isoimmuniza- Imaging studies of the breast during pregnancy tion usually occurs as a result of fetomaternal are difficult to interpret because of the ductular transfusion at the time of delivery, a small per- and glandular hypertrophy of pregnancy. centage of women become isoimmunized (Hoskins et al., 2005, pp. 1296–1298) during pregnancy. Anti-D immune globulin is (c) ketabton.com: The Digital Library

Answers: 88–99 83

routinely given to unsensitized Rh-negative urethrovaginal fistula will complain of a women at 28 weeks’ gestation to reduce this watery vaginal discharge. Women with an risk. Anti-D immune globulin must also be atonic bladder typically void small amounts administered within 72 hours after the birth of and complain that the bladder still feels full an Rh-positive infant. Administration at 40 (which it is). This is a disorder seen in women weeks’ gestation before the onset of labor is with neurologic dysfunction of the bladder, unnecessary if the infant is Rh negative and such as multiple sclerosis and diabetic neu- may be ineffective if the infant is Rh positive ropathy. (Scott et al., 2003, pp. 849–856) and there is a significant fetomaternal transfu- sion. (Cunningham et al., 2005, p. 671) 97. (D) Although a urine culture is a standard part of the evaluation of women with loss of urine, 94. (B) One vial of Rh immunoglobulin will pre- this woman’s history is not consistent with vent Rh isoimmunization if the amount of fetal acute cystitis. A Q-tip test is done to assess the whole blood entering the maternal circulation angle the urethra makes with the horizontal in is 30 mL or less. If the woman has a condition the relaxed and voiding circumstances. Though where a greater amount of fetal blood may intended to differentiate GSI from other causes enter the maternal circulation, the amount of of incontinence, it has not proved to be suf- fetal blood should be estimated by submitting ficiently sensitive to make this distinction a maternal blood sample for a Kleihauer-Betke reliably. Urethroscopy is appropriate if a ure- test. The dose of Rh immunoglobulin is then throvaginal fistula or urethral diverticulum is based on the results of this test. (Scott et al., 2003, suspected. Urethrocystometry is one name for pp. 317–319) a test that measures the pressure-volume rela- tionship in the bladder. It should be done in 95. (C) Bilateral hypogastric artery ligation con- most women with incontinence as the most verts the arterial system into a venous system; sensitive test to distinguish the various causes thereby, reducing the pulse pressure by as of incontinence. An IVP is of little value in much as 85%. Subsequent menstrual function determining the cause of incontinence. (Scott et al., and fertility are normal, in part because of the 2003, pp. 849–856) rich collateral circulation to the uterus. The pro- cedure is successful in approximately 50% of 98. (C) Antibiotics are useful only when there is cases. The procedure is not technically easy to evidence of cystitis. SSRIs are antidepressants perform, and an intimate knowledge of the that have not been shown to improve inconti- local anatomy is essential to prevent injury to nence with detrusor dyssynergia. Surgery is of the hypogastric vein or ureter. Uterine hemor- no value and may actually worsen inconti- rhage not controlled by other means requires a nence in women with detrusor dyssynergia. hysterectomy. (Creasy et al., 2004, pp. 942–943) Bladder retraining, in which the patient embarks on a programmed progressive length- 96. (E) Two clues to the diagnosis of detrusor ening of the interval of voiding, forms the basis dyssynergia are loss of urine in the recumbent of therapy. While such retraining is occurring, position and inability to stop the urine loss once the use of anticholinergic drugs, such as oxy- the stream has begun. Generally, large volumes butynin chloride (Ditropan), propantheline of urine are lost because of the inability to stop (Pro-Banthine), or flavoxate (Urispas), seems the flow of urine. With GSI, urine is lost only in to improve the results over use of any one the upright position when intra-abdominal and alone. (Scott et al., 2003, pp. 859–860) intravesical pressure exceeds urethral closing pressure, such as with coughing. Women with 99. (C) The delayed puberty, short stature, web GSI are able to stop the flow of urine voluntar- neck, increased carrying angle (cubitus valgus), ily and, therefore, the volume of urine lost is widely spaced nipples are classic signs of small. With GSI, urine loss with coughing is Turner syndrome, gonadal dysgenesis. Women immediate. Women with a vesicovaginal or with anorexia nervosa are of normal height and (c) ketabton.com: The Digital Library

84 2: Obstetrics and Gynecology

have a history of weight loss below 15% of or vaginal ultrasound should be done because ideal body weight. Androgen insensitivity syn- these may damage the placenta and cause fur- drome is not a consideration because these ther bleeding and fetal compromise. Delivery is women lack a uterus and cervix, and have a by cesarean section. (Cunningham et al., 2005, short vagina. The Müllerian ducts develop to pp. 819–823) form the Fallopian tubes, uterus, cervix, and upper vagina. The presence of a cervix and 103. (C) Light vaginal bleeding following sexual uterus excludes this diagnosis. Strictly, gonadal intercourse is likely of cervical origin because of dysgenesis is a type of premature ovarian fail- the increased blood flow to the pelvic organs in ure, developing before the age of puberty to pregnancy and eversion of the endocervix that result in primary amenorrhea. However, pre- occurs as a result of the increased estrogen pro- mature ovarian failure is most often a postpu- duction. The absence of abdominal pain and bertal event and these women usually tenderness, the appropriate size of the uterus menstruate for a variable period of time. (Speroff and the spotting rather than overt bleeding and Fritz, 2005, pp. 345–346) tend to exclude threatened abortion as the cause of this woman’s bleeding. (Scott et al., 2003, 100. (C) The short stature and other features of pp. 643–649) Turner syndrome are strong presumptive evi- dence of a 45,X karyotype. A 46,XX and a 46,XY 104. (B) Gestational trophoblastic disease is more karyotype may be found in women with common in women at the extremes of repro- gonadal dysgenesis. Their clinical presentation ductive age. Uterine size larger than her gesta- is absence of pubertal events, but lack of the tional age calculated from her last menses, the physical features of Turner syndrome. Most absence of fetal heart tones, and the absence of helpful in excluding either of these karyotypes a fetus by ultrasound confirm the diagnosis. is the short stature: those with 46,XX or 46,XY Treatment is evacuation of the uterus. gonadal dysgenesis are of normal height. There (Cunningham et al., 2005, pp. 274–279) is nothing in the clinical presentation to suggest 47,XX trisomy 21. The phenotype of those with 105. (E) The features that make placental abruption a 47,XXY karyotype is male, Klinefelter syn- the most likely diagnosis are the amount of drome. (Speroff and Fritz, 2005, pp. 345–348) bleeding, chronic hypertension, and an exces- sively firm (even titanic) uterus that is tender to 101. (B) The streak gonads of women with gonadal palpation. Women with a placental abruption dysgenesis will never secrete sex hormones. may have no bleeding if placental separation Combined estrogen plus progestin is the cor- occurs only in the center of the placenta. The rect choice to stimulate pubertal development. other signs of pain and tenderness will be Estrogen alone,in long term, increases the risk present. (Cunningham et al., 2005, pp. 811–819) that these women will develop endometrial adenocarcinoma. Clomiphene and Follistim are 106. (J) The use of forceps and the presence of a fertility-promoting agents used to stimulate firm, nontender uterus of appropriate size sug- ovulation in anovulatory women who have gest vaginal laceration as the cause of postpar- ovarian follicles. Women with Turner syn- tum bleeding. The diagnosis is confirmed by a drome have few or no follicles and oocytes. speculum examination. A cervical laceration Gonadectomy is necessary only when a Y chro- should also be considered and the cervix care- mosome is present in a woman with gonadal fully inspected at the time of the speculum dysgenesis. (Speroff and Fritz, 2005, pp. 349–350) examination. (Cunningham et al., 2005, pp. 556–557)

102. (D) Painless vaginal bleeding in the third 107. (K) Vaginal spotting accompanied by pelvic trimester is most often due to a placenta previa. discomfort in a woman with a prior sexually The diagnosis is easily confirmed by abdominal transmitted infection suggests the diagnosis of ultrasound. No vaginal/cervical examination an ectopic (tubal) pregnancy. Serum hCG (c) ketabton.com: The Digital Library

Answers: 100–110 85

concentrations should increase by at least 67% 109. (G) Placenta accreta is suggested by the diffi- in 2 days during the first 6–8 weeks of preg- culty with manual removal of the placenta in a nancy. The subnormal increase in this woman’s woman with a prior cesarean section. Placenta serum hCG concentrations increases the prob- accrete is also more common over any previous ability of an ectopic pregnancy. However, uterine incision, such as a myomectomy. approximately 15% of women with a normal Placenta accrete is also more common in intrauterine pregnancy will have a subnormal women with placenta previa and there is rise in hCG concentrations. Also, approxi- greater than an eightfold increase in women mately 15% of women with an ectopic preg- with an AFP higher than 2.5 MOM. The safest nancy will have a 48-hour rise in serum hCG and most appropriate treatment is a hysterec- concentrations greater than 67%. At hCG con- tomy. (Cunningham et al., 2005, pp. 830–833) centrations over 5000 mIU/mL, the absence of an intrauterine pregnancy by transvaginal 110. (I) Prolonged labor with delivery of a macro- ultrasound provides additional evidence for somic fetus (greater than 4000 g) in a highly the diagnosis of an ectopic pregnancy. (Scott parous woman are the risk factors for uterine et al., 2003, pp. 92–94) atony. The diagnosis is confirmed by a boggy, noncontracted uterus that is larger than 108. (F) The clinical features that make uterine rup- expected after a normal delivery. Treatment is ture the most likely diagnosis are profuse a combination of manual massage of the bleeding coincident with cessation of uterine uterus, oxytocin, blood transfusion to maintain contractions and labor in a woman with a prior hemodynamic stability, and careful inspection classical cesarean section (vertical incision in of the vagina, cervix, and uterus to exclude a the uterine fundus). Uterine rupture is uncom- vaginal or cervical laceration or retained pla- mon in women with a prior low transverse cental fragments. Ergot derivatives or cesarean section and rare in women with no prostaglandins should be administered if the scar on her uterus. The standard of care is that above measures fail to cause the myometrium all women with a previous classical cesarean to contract. Uterine artery embolization is an section be delivered by repeat cesarean section unproven therapy. A hysterectomy is necessary at term before the onset of labor. (Cunningham if all measures fail to stop the postpartum hem- et al., 2005, pp. 615–616) orrhage. (Cunningham et al., 2005, pp. 826–830) (c) ketabton.com: The Digital Library

BIBLIOGRAPHY

ACOG Committee on Practice Bulletins. ACOG Cunningham FG, Leveno KJ, Bloom SL, et al. Williams Practice Bulletin No. 82: Management of herpes in Obstetrics, 22nd ed. New York, NY: McGraw Hill, pregnancy. Obstet Gynecol 109:1489–98, June 2007. 2005. Centers for Disease Control and Prevention. MMWR Emans SJ, Laufer MR, Goldstein DP. Pediatric and Morbidity and Mortality Weekly Report. Adolescent Gynecology, 5th ed. Philadelphia, PA: Recommendations and Reports. Sexually Transmitted Lippincott Williams and Wilkins, 2005. Diseases Guidelines, Vol. 55, No. RR-11. August 4, Hoskins WJ, Perez CA, Young RC, et al. Principles 2006, pp. 42–49. and Practice of Gynecologic Oncology, 4th ed. Center for Disease Control and Prevention. MMWR Philadelphia, PA: Lippincott Williams & Wilkins, Morbidity and Mortality Weekly Report. Update to 2005. CDC’s Sexually Transmitted Diseases Guidelines: McGuire H, Hawton K. Interventions for vaginis- Fluoroquinolones No Longer Recommended for mus. Cochrane Database of Systematic Reviews 2001, Treatment of Gonococcal Infections, Vol. 56, No. 14, Issue 2. Article No.: CD001760.DOI:10.1002/ 2006, pp. 332–336. 14651858.CD001760. Center for Disease Control and Prevention. MMWR Scott JR, Gibbs RS, Karlan BY, et al., eds. Danforth’s Morbidity and Mortality Weekly Report. Obstetrics and Gynecology, 9th ed. Philadelphia, PA: Quadrivalent Human Papillomavirus Vaccine: Lippincott Williams & Wilkins, 2003. Recommendations of the Advisory Committee on Speroff L, Fritz MA. Clinical Gynecologic Endocrinology Immunization Practices (ACIP), Vol. 56, No. RR-2, and Infertility, 7th ed. Philadelphia, PA: Lippincott March 23, 2007. Williams & Wilkins, 2005. Creasy RK, Resnik R, Iams JD. Maternal–Fetal Medicine: Principles and Practice, 5th ed. Philadelphia, PA: W.B. Saunders, 2004.

86 (c) ketabton.com: The Digital Library

CHAPTER 3 Pediatrics Rainer Gedeit, MD

Questions

1. A 2-year-old child was recently adopted from 3. In an adolescent presenting with pityriasis India. She appears to be healthy, and there are rosea, which of the following would be an no abnormal symptoms. Her weight and height appropriate blood test to order? are at 25th percentile for age. Her examination (A) Venereal Disease Research Laboratory is normal. On screening, you find a positive (VDRL) TB skin test using purified protein derivative (PPD) with 20 mm induration. She has a history (B) complete blood count (CBC) of receiving a BCG vaccination at birth. Your (C) hepatitis A immunoglobulin M (IgM) management plan is to do which of the fol- (D) fluorescent antinuclear antibody (FANA) lowing? (E) glucose (A) Obtain a chest x-ray and treat only if this is abnormal. 4. A parent brings in a 5-year-old boy being treated for acute lymphocytic leukemia (ALL). (B) Obtain a chest x-ray and initiate pro- He states a friend who is staying with them at phylactic treatment with isoniazid their home has just come down with chicken (INH). pox. Your patient has not had chicken pox or (C) Repeat the test in 3–6 months. received immunization with varicella vaccine. (D) Attribute the positive PPD to the BCG What is the appropriate treatment? vaccination and do serial yearly x-rays. (A) acyclovir given IV (E) Obtain sputum cultures. (B) varicella vaccine 2. A baby is born to a mother who is positive for (C) varicella immune globulin (VZIG) hepatitis B surface antigen (HBsAg). Your plan (D) varicella vaccine and VZIG is to do which of the following? (E) acyclovir given IV for 7 days, varicella (A) Give the infant a hepatitis B immuniza- vaccine, and VZIG tion. 5. A young mother claims that her 4-week-old (B) Give the infant hepatitis B immune child sleeps best on his stomach. You tell her globulin (HBIG). that the safest sleep position for infants is (C) Give the infant a hepatitis B immuniza- which of the following? tion and HBIG. (D) Obtain liver function tests and hepatitis (A) on the back serology of the infant. (B) on the stomach (E) Give the HBIG only if the child is posi- (C) on the side tive for HBsAg. (D) on the back with the head elevated by a pillow (E) in the parents’ bed

87

Copyright © 2008 by The McGraw-Hill Companies, Inc. Click here for terms of use. (c) ketabton.com: The Digital Library

88 3: Pediatrics

6. You receive a call from the parents of a 1 year (C) a rear-facing car seat suitable for a old who is due for his well-child visit next larger child (20–40 lbs), in the rear seat week. They have just received a letter from of the car their daycare center that an employee has hep- (D) a forward-facing car seat suitable for a atitis A. Which of the following is the best treat- larger child (20–40 lbs), in the rear seat ment plan? of the car (A) Give hepatitis A immune globulin and (E) a forward-facing car seat suitable for a hepatitis A vaccine. larger child (20–40 lbs), in the front seat (B) Treat with hepatitis A immune globulin. of the car (C) Obtain hepatitis A serology and give 9. A 2-month-old infant is brought to the emer- hepatitis A vaccine. gency department with irritability and lethargy. (D) Give hepatitis A vaccine. The parents state that he was well until he (E) No treatment is needed. rolled off the couch on to the floor yesterday. On examination, he is inconsolable and 7. A 5-year-old child was hit in the right eye by a afebrile. The fontanels are full and tense. He toy. He is rubbing at his eye, which is watering has a generalized tonic-clonic seizure. Which of profusely. There is a small abrasion at the the following is the most important initial diag- corner of the eye. He is mildly photophobic, but nostic study to order? his pupils are equal, symmetric, and reactive to light and accommodation. His vision is normal. (A) serum calcium, phosphorus, and mag- Which of the following is the most appropriate nesium levels next step in the management of this patient? (B) analysis of cerebrospinal fluid (CSF) (C) cranial computed tomography (CT) scan (A) Perform a fluorescein dye stain of the cornea to determine if there is a corneal (D) serum ammonia level abrasion. (E) serum acetaminophen level (B) Refer him immediately to an ophthal- mologist. 10. A 7-year-old girl presents with hives, which developed after a bee sting. She has no other (C) Irrigate the eye with sterile normal symptoms. The hives resolve with diphenhy- saline. dramine. Which of the following is the most (D) Discharge him to home with antibiotic appropriate management? eye ointment. (E) Apply a patch to the eye and follow-up (A) Write a prescription for diphenhy- in a week. dramine in case she is bitten again. (B) Provide an Epi-pen Jr (epinephrine auto 8. A 9-month-old male is in for a well-child injector) to be carried at all times, as checkup. He is greater than 90th percentile for well as a prescription for diphenhy- height, and he weighs 25 lbs. He no longer fits dramine. in his infant car seat, which is only recom- (C) Admit to the hospital for observation mended for use by children under 20 lbs. for delayed hypersensitivity symptoms. Which of the following is the safest car seat (D) Refer her to an allergist for desensitiza- option for him? tion. (A) to remain in the rear-facing infant seat (E) Order a skin-prick test with until he is 1 year old, in the rear seat of hymenoptera venom. the car (B) turn the infant seat to face forward, in the rear seat of the car (c) ketabton.com: The Digital Library

Questions: 6–16 89

11. A 2-year-old boy has had a purulent drainage sexual maturity rating (Tanner) is stage II for from the right nostril for a week. He is afebrile both genitalia and pubic hair growth. Initial man- and has had no associated symptoms, such as agement should include which of the following? cough. Which of the following is the most likely (A) magnetic resonance imaging of the head diagnosis? (B) urine drug screen for marijuana (A) sinusitis (C) chromosome analysis (B) nasal polyps (D) reassurance that this is a normal condi- (C) an upper respiratory infection tion (D) a foreign body in the right nostril (E) ultrasound imaging of the abdomen and (E) allergic rhinitis testes

Questions 12 and 13 15. A mother brings in her 3-year-old girl because she felt a smooth mass on the left side of her 12. A 5-year-old boy has a history of bed-wetting belly when she was giving her a bath. Which of about four to five times a week. He has recently the following is the most likely diagnosis? begun to attend kindergarten. He was toilet trained (dry during the day) by age 3 but has (A) Wilms tumor never been consistently dry at night. He denies (B) neuroblastoma any dysuria or frequency. There is no history of (C) acute lymphoblastic leukemia increased thirst or frequent urination. The uri- (D) Hodgkin’s disease nalysis is negative for blood, protein, glucose, (E) hepatoblastoma or ketones; there are no white cells or bacteria; the specific gravity is 1.020. Which of the fol- Questions 16 through 19 lowing is the most likely diagnosis? (A) a urinary tract infection (UTI) An 11-month-old girl presents to your office with a fever of 39°C she has had for 2 days. She has also (B) primary nocturnal enuresis vomited frequently and had decreased fluid intake. (C) secondary enuresis caused by stress of She looked tired and ill but on examination, had no the new school apparent source of infection. She appeared to be (D) diabetes mellitus 5–10% dehydrated. (E) diabetes insipidus 16. You decide to obtain a urine specimen for 13. The parents request some treatment for this analysis and culture. Which of the following is condition. Which of the following is the most the best method? appropriate treatment for a child of this age? (A) Collect a midstream “clean catch” speci- (A) bladder stretching exercises men. (B) intranasal DDAVP (desmopressin (B) Collect a catheterized specimen. acetate) (C) Place an adhesive bag to collect urine. (C) imipramine (D) Obtain urine from a diaper. (D) conditioning therapy with a bed-wetting (E) Collect urine after she urinates in a alarm potty chair. (E) reassurance of the parents and restric- tion of fluids before bedtime

14. A 14-year-old boy complains of breast enlarge- ment on the left side. He denies pain, discharge, or any drug use. He is on no medications and is otherwise healthy. On physical examination, his (c) ketabton.com: The Digital Library

90 3: Pediatrics

17. Her urinalysis shows a urine specific gravity of (C) human herpesvirus, type 6 1.030, trace blood, and protein. Nitrite and (D) respiratory syncytial virus leukocyte esterase are both positive. Microscopic (E) rotavirus examination of unspun urine shows >100 white blood cells (WBCs) and 0–5 red blood cells Questions 21 and 22 (RBCs) per high-power field, as well as many bacteria. A urine culture is sent. Which of the fol- 21. A 10-year-old boy comes to your office in the lowing is the most appropriate management winter with a sore throat he has had for 2 days. plan? In addition, he has had fever, headache, and abdominal pain. He does not have any allergies (A) Treat only if the culture is positive. to medications. On examination, he has a tem- (B) Admit for intravenous (IV) hydration perature of 38.6°C, an erythematous pharynx, and IV antibiotics. and tender cervical adenopathy. A rapid screen- (C) Treat with intramuscular ceftriaxone ing test for group A streptococcus is performed and have her follow-up in the office the and is positive. Which of the following would following day. be the most appropriate antimicrobial agent? (D) Treat with trimethoprim- (A) erythromycin sulfamethoxazole, and have her follow- up in the office the following day. (B) penicillin (E) Prescribe amoxicillin and start oral (C) trimethoprim-sulfamethoxazole hydration. (D) azithromycin (E) cefaclor 18. Her urine culture is positive at 24 hours. Which of the following is the most likely organism? 22. The same child returns to your office the next day. He has taken the medication you pre- (A) Klebsiella scribed. He is feeling a little better. His fever (B) Escherichia coli has resolved, but he has developed a rash. His (C) Staphylococcus saprophyticus examination is unchanged, except that he is (D) Proteus afebrile and has a fine, papular rash over his (E) Enterococcus body, which is accentuated in his axilla and groin. Which of the following is the most likely 19. After the infection has been treated, which one cause of his rash? of the following tests should be considered? (A) allergic reaction to the antibiotic (A) no tests are needed (B) rash from the antibiotic seen in patients (B) renal ultrasound with mononucleosis (C) voiding cystourethrogram (VCUG) (C) scarlet fever (D) renal ultrasound and a VCUG (D) serum sickness (E) radionucleotide renal scan (E) viral exanthem typical of enterovirus

20. A 3-month-old infant is brought to your office in the winter with a history of 1 day of vomit- ing, followed by 3 days of diarrhea. She has had six to eight stools per day, which are loose and foul smelling. On examination, she looks well. Which of the following viruses is the most likely cause of her illness? (A) adenovirus (B) enterovirus (c) ketabton.com: The Digital Library

Questions: 17–26 91

Questions 23 and 24 24. As you continue your physical examination, you remember that congenital heart disease is An 8-year-old girl presents for a checkup. She is common in this particular syndrome. Which new to your practice. The mother states that she has of the following is the most likely congenital always been small for her age; otherwise, she has heart defect in patients with this syndrome? been well. She is short and has a height age of 4 years, 4 months. You note some abnormalities in her (A) supravalvular aortic stenosis general appearance (Figure 3-1). (B) atrioventricular (AV) canal defects (C) coarctation of the aorta (D) pulmonary valvular stenosis (E) mitral valve prolapse

Questions 25 and 26

A 12-year-old boy comes to the clinic for a sports physical. He is new to your practice. He comes with his foster mother, who states that he was recently placed in her care because of his mother’s problems with drug abuse. Although a complete medical his- tory is not available, she knows that he has not received regular care. He does not have any chronic medical problems. She also knows that his father died of heart disease when he was 35. On physical examination, the boy’s height is greater than the 95th percentile. His arm span exceeds his height.

25. Which of the following is the most likely cause of his tall stature? (A) Ehlers-Danlos syndrome (B) Kleinfelter syndrome (C) Marfan syndrome FIG. 3-1 (Reproduced, with permission, from Grumbach, Barr. Recent Prog Horm Res 14:255, 1958, with permission from Elsevier (D) Noonan syndrome Copyright © Elsevier 1958.) (E) Williams syndrome

26. As you continue your physical examination, 23. Which of the following is the most likely you remember that congenital heart disease is diagnosis? common in this particular syndrome. Which (A) Marfan syndrome of the following is the most likely congenital heart defect in patients with this syndrome? (B) Noonan syndrome (C) trisomy 21 (A) supravalvular aortic stenosis (D) Turner syndrome (B) AV canal defects (E) Williams syndrome (C) coarctation of the aorta (D) pulmonary valvular stenosis (E) mitral valve prolapse (c) ketabton.com: The Digital Library

92 3: Pediatrics

27. A 17-year-old girl comes to the clinic with sev- (A) incision and drainage eral weeks of joint pain and rash. The joint pain (B) oral amoxicillin is most prominent in the hands. She states that (C) IV naficillin the pain is most severe in the morning and (D) selenium sulfide shampoo twice a week tends to improve over the day. She has noted some swelling of her fingers. She has also had (E) oral griseofulvin and selenium sulfide a rash on her face that becomes more promi- shampoo twice weekly nent when she is outdoors. She states that sun- light tends to bother her eyes. On further 30. A 4-year-old child presents to your office in questioning, she states that she has not felt well July with a history of a low-grade fever (38.1°C) for several months. She has had intermittent and “sores” in his mouth for 2 days. He has fever, has been more tired than usual, and has been refusing to eat but has been drinking an lost weight although she has not been restrict- adequate amount of liquids. On examination, ing her diet. On physical examination, she he is afebrile and seems well hydrated. He has looks tired. She has lost 5 lbs since her last visit ulcers on his tongue and posterior pharynx, 1 year ago. She has an erythematous rash on which are 4 mm in diameter. You also note a her cheeks. She has several shallow ulcers in few vesicles on his hands and feet, which are her mouth. She has fusiform swelling of her 3–4 mm in size and mildly tender. Which of fingers and pain with movement of her fin- the following is the most likely diagnosis? gers. Which of the following is the most likely (A) herpes simplex virus (HSV) diagnosis? (B) coxsackie virus (A) systemic lupus erythematosus (SLE) (C) aphthous ulcers (B) dermatomyositis (D) Behçet syndrome (C) juvenile rheumatoid arthritis (E) traumatic ulcers (D) rheumatic fever (E) Lyme disease 31. A 10-year-old boy comes to the office with fever and chills for 5 days and myalgia. He has 28. A 2-year-old girl has severe dental caries of the recently returned from a 2-week vacation to upper and lower incisors. Her teeth are New England with his family. On physical brushed twice daily with a small amount of examination he has mild splenomegaly. Which fluoride-containing toothpaste. What is the of the following is the most likely cause of his feeding practice most likely to result in this symptoms? pattern of dental caries? (A) Kawasaki disease (A) drinking juice from a cup at snack time (B) pneumococcus (B) drinking juice from a bottle at snack (C) babesiosis time (D) leptospirosis (C) drinking milk from a bottle at meal time (E) psittacosis (D) prolonged breast-feeding beyond the first year 32. A 6-year-old girl has a low-grade fever, (E) drinking a bottle of juice in bed headache, and nasal congestion. She has a flushed face and has developed a lacy reticular 29. An 8-year-old African American boy is brought rash on the trunk and extensor surface of her in for evaluation of a mass on the scalp. On arms and legs. Palms and soles are spared. Her examination, he is afebrile and nontoxic. There mother has been ill with a low-grade fever and is a boggy mass on his scalp with alopecia. His some joint stiffness and pain. Which of the fol- posterior cervical lymph nodes are enlarged lowing is the most likely diagnosis? but nontender. Which of the following is the (A) rubella most appropriate treatment? (B) measles (c) ketabton.com: The Digital Library

Questions: 27–38 93

(C) scarlet fever (C) congenital syphilis (D) roseola infantum (D) rickets (E) erythema infectiosum (fifth disease) (E) chondrodystrophy

33. A 3-year-old boy has had fever for 4 days. On 36. A 14-year-old boy presents with sudden onset physical examination he has bilateral cervical of pain and swelling of his right testicle. There lymphadenopathy, injected pharynx, and dry was no history of trauma, he is not sexually cracked lips. A throat swab is done and the active, and denies any history of penile dis- rapid strep test is negative. The child is sent charge. On examination, the scrotum is swollen home and advised to follow-up if symptoms and tender. The cremasteric reflex is absent. A worsen. The child is brought back 2 days later testicular flow scan shows a “cold spot” or with all previous findings including a macu- absent flow to the affected side. Which of the lopapular rash, swollen hands, and conjunc- following is the most likely cause? tivitis. Which of the following is the most likely (A) inguinal hernia diagnosis? (B) hydrocele (A) Scarlett fever (C) epididymitis (B) Kawasaki disease (D) testicular torsion (C) toxic shock syndrome (E) torsion of the appendix testis (D) infectious mononucleosis (E) erythema infectiosum 37. Otitis media occurring during the first 8 weeks of life deserves special consideration, because 34. A 5-year-old febrile child presents with the bacteria responsible for infections during swelling of the right eyelid. Proptosis and lim- this time may be different from those that affect itation of ocular movements is noted. Which of older infants and children. Which of the fol- the following is the most likely diagnosis? lowing organisms is the most likely to cause otitis media in these infants? (A) retinoblastoma (B) orbital cellulitis (A) Chlamydia trachomatis (C) periorbital cellulitis (B) E. coli (D) neuroblastoma (C) Neisseria gonorrhoeae (E) hyphema (D) Treponema pallidum (E) Toxoplasma gondii 35. A 1-year-old African American infant is in for well-child care. He is primarily breast-fed. His 38. Among the conditions that cause edema of the parents do not give him much solid food eyelids is orbital cellulitis. This is a serious because he has no teeth. He receives no med- infection that must be recognized early and ications or supplements. His parents are con- treated aggressively if complications are to be cerned about his bowed legs. On examination, avoided. Which of the following features is you note some other bony abnormalities useful in differentiating orbital cellulitis from including frontal bossing, enlargement of the periorbital (preseptal) cellulitis? costochondral junctions, a protuberant ster- (A) proptosis num (pigeon chest), and severe bowing of the legs. You obtain x-rays to confirm your clinical (B) elevated WBC count diagnosis and also note a healing fracture of the (C) fever left femur. Which of the following is the most (D) lid swelling likely diagnosis? (E) conjunctival inflammation (A) osteogenesis imperfecta (B) scurvy (c) ketabton.com: The Digital Library

94 3: Pediatrics

39. A 6-month-old infant is diagnosed with her within normal limits. You obtain a diet history, first episode of otitis media. She does not have which reveals that he drinks about 30–40 oz of any allergies to medications. Which of the fol- whole cow’s milk a day. He eats no meat and some lowing medications would be the recom- fruits and vegetables. mended initial therapy for this infant? 42. Which of the following is the most likely cause? (A) amoxicillin (B) amoxicillin-clavulanic acid (A) sickle cell anemia (C) cephalexin (B) thalassemia major (D) ceftriaxone (C) lead poisoning (E) erythromycin (D) iron-deficiency anemia (E) anemia of chronic disease 40. A 5-year-old pedestrian is hit by a car in a mall parking lot and he is brought to the emergency 43. The most effective next step in management department. There was loss of consciousness would be to obtain which of the following? for less than 1 minute. On evaluation, the child (A) iron studies—serum iron, total iron has no neurologic deficits and a CT scan of the binding capacity, ferritin head reveals no intracranial abnormalities and no obvious skull fractures. The parents want to (B) reticulocyte count know what possible long-term problems there (C) hemoglobin electrophoresis might be. You remember that problems after (D) a repeat hemoglobin in 1 month after head trauma may include the development of treatment with folic acid seizures and that the risk of developing post- (E) a repeat hemoglobin in 1 month after traumatic epilepsy is increased by which of the treatment with iron following? (A) a brief loss of consciousness 44. A week-old infant presents blood in his stools. He was born at home, with the father assisting (B) an acute intracranial hemorrhage in the delivery; no physician or midwife was (C) retrograde amnesia present. He has been breast-fed and has been (D) posttraumatic vomiting nursing well. On examination, you also note (E) a small linear skull fracture some blood in his nose. He is not jaundiced; a rectal examination and guaic test of the stool 41. A 4-year-old previously healthy but unimmu- confirms that blood is present. His examination nized boy presents with sudden onset of high is otherwise normal. He is on no medications. fever, inspiratory stridor, and refusal to drink. Which of the following is the most likely Of the following causes of inspiratory stridor, diagnosis? which best fits this clinical scenario? (A) child abuse (A) epiglottitis (B) vitamin K deficiency (B) vascular ring (C) breast milk allergy (C) croup (D) sepsis (D) foreign body aspiration (E) liver disease (E) laryngeal tumor Questions 45 and 46 Questions 42 and 43 A 6-year-old Caucasian female has breast enlarge- A 15-month-old African American male, who is oth- ment (Tanner stage II) and coarse curly pubic hair. erwise healthy, is found to have a hemoglobin level She is not yet menstruating. She is otherwise of 8 g/dL on routine screening. The mean corpus- healthy and has normal growth parameters. There cular volume (MCV) is decreased. His lead screen is are no signs of virilization and her abdominal (c) ketabton.com: The Digital Library

Questions: 39–51 95

examination reveals no masses. Examination of the (C) patent ductus arteriosus vaginal area shows signs of estrogenization. (D) aortic stenosis (E) pulmonary stenosis 45. Which of the following is the most likely pre- liminary working diagnosis? 49. A 4-year-old child was brought in for evalua- (A) precocious puberty tion of sleep problems. He cried and screamed (B) premature thelarche within an hour of falling asleep. He seemed disoriented and confused; he did not seem (C) premature pubarche aware of his parents’ presence. They were (D) normal development unable to arouse him to comfort him. This (E) precocious menarche resolved spontaneously, and he had no recol- lection of the event the next morning. You 46. On laboratory evaluation, you find elevated informed the parents that he was most likely levels of follicle-stimulating hormone (FSH), experiencing which of the following? luteinizing hormone (LH), and pubertal levels of estradiol. The bone age is advanced beyond (A) nightmares the height and chronologic age. The most likely (B) night terrors cause is which of the following? (C) somnambulism (A) idiopathic (D) somniloquy (B) central nervous system (CNS) tumor (E) narcolepsy (C) ovarian tumor 50. A 3-year-old boy was bitten while teasing a (D) functional neighborhood cat. On examination, there are (E) congenital adrenal hyperplasia two puncture wounds on the right hand and some superficial scratch marks. There is ery- 47. A 10-year-old boy is brought in with a chief thema, warmth, and induration around the complaint of multiple colds. On further ques- puncture sites. Which of the following organ- tioning, you elicit a history of chronic, clear isms most likely caused the infection? nasal discharge with no seasonal variation. Other symptoms include sneezing, itching of (A) Pasturella multicoda the nose and eyes, as well as tearing and occa- (B) Bartonella henselae sional eye redness. Some relief is obtained with (C) Eikenella corrodens an over-the-counter cold medicine containing (D) Peptostreptococcus species antihistamine and a decongestant. His history (E) alpha Streptococci suggests which of the following? (A) nasal foreign body 51. A 5-week-old bottle-fed boy presents with per- (B) immunologic deficiency sistent and worsening projectile vomiting, poor weight gain, and hypochloremic metabolic (C) rhinitis medicamentosa alkalosis. Of the following diagnostic modali- (D) chronic sinusitis ties, which would most likely reveal the diag- (E) allergic rhinitis nosis?

48. A full-term newborn develops cyanosis a few (A) ultrasound of abdomen hours after birth. Oxygen administration does (B) barium enema not improve color or oxygen saturations. (C) evaluation of stool for ova and parasites Which of the following is the most likely diag- (D) testing well water for presence of nosis? nitrites (A) atrial septal defect (E) serum thyroxine (B) ventricular septal defect (c) ketabton.com: The Digital Library

96 3: Pediatrics

52. Your next patient is a 4-month-old infant who 54. A 10-year-old boy presents with a 1-day history is returning to have her ear checked. You diag- of fever, cough, and chest pain. He has not been nosed her with otitis media 2 weeks ago, and eating and has been listless. He does not have she has taken 10 days of amoxicillin. She is feel- any previous history of health problems. On ing well, and her mother’s only concern is that physical examination, his temperature is 40°C, she has developed a diaper rash over the last 3 and he is tachypneic. He looks ill. He has rales days. The mother has been using emollient on his left posterior lower lung fields. You creams on it which have not helped. On phys- order a chest x-ray (Figure 3-3). Which of the ical examination, there are no abnormal find- following organisms is most likely responsible ings except for the rash (Figure 3-2). Which of for his pneumonia? the following is the most likely diagnosis?

FIG. 3-2 Also see color insert. (Courtesy of Neil S. Prose.)

(A) allergic dermatitis (B) bullous impetigo (C) Candida dermatitis (D) irritant dermatitis (E) seborrheic dermatitis

53. A 7-year-old boy presents with a rash. His mother states that he was well until 3 days ago when he developed fever and malaise. The next day, the rash started as papules on the trunk, which rapidly changed to vesicles. The lesions have spread all over the body. On physical examination, he has no fever and seems well. You note numerous vesicles all over the body, some of which have crusted over. Which of the following is the most likely diagnosis? (A) chicken pox (B) Kawasaki disease (C) measles (D) rubella (E) staphylococcal scalded skin syndrome FIG. 3-3 (c) ketabton.com: The Digital Library

Questions: 52–59 97

(A) Haemophilus influenzae 57. A 9-year-old boy presents with a several-day his- (B) Mycoplasma pneumoniae tory of progressive arm and leg weakness. He (C) Pneumocystis carinii has been well except for an upper respiratory infection 2 weeks ago. The patient is alert and ori- (D) Staphylococcus aureus ented. On repeated examination, the heart rate (E) Streptococcus pneumoniae varies between 60 and 140 beats/min, and the blood pressure (BP) varies between 90/60 and 55. An 18-month-old girl is brought to the hospital 140/90 mmHg. Respirations are shallow, with a with a history of 6 days of bloody diarrhea. rate of 50/min. There is symmetric weakness of She has been drinking well but has not been the face and all four extremities. Deep tendon wetting her diaper. She has been irritable. On reflexes are absent. Sensation is intact. Which of physical examination, she has periorbital the following is the most likely diagnosis? edema. She appears pale and is tachycardic. Her CBC shows a hemoglobin of 6 g/dL and a (A) polymyositis platelet count of 100,000/mm3. Her blood urea (B) myasthenia gravis nitrogen (BUN) is 50 mg/dL and creatinine is (C) transverse myelitis 5.5 mg/dL. Her urinalysis shows gross hema- (D) Guillain-Barré syndrome turia. Which of the following is the most likely (E) viral encephalitis causative organism for her clinical problem? (A) E. coli 0157:H7 58. An 18-month-old boy is brought to the clinic for (B) group A Streptococci a checkup. As part of his routine care, a serum (C) group B Streptococci (GBS) lead level is obtained. It is 25 g/mL. Which of the following is the most appropriate next step (D) S. aureus in his management? (E) the cause of this illness is not known (A) chelation with CaEDTA 56. A 1-year-old child with ALL in remission for 3 (B) chelation with succimer months is in the office for a health maintenance (C) investigation of his home for lead visit. He is due for multiple vaccinations hazards including hepatitis B vaccine, inactivated polio (D) reassurance that this level is not a problem vaccine (IPV), varicella vaccine H. influenzae B (E) repeating the level in 6 months vaccine (Hib), and pneumococcal vaccine (PCV). You remember that some of these vac- 59. An infant is due for her first dose of polio vac- cines are live attenuated viruses and are con- cine. Her parents have heard that there are two traindicated in immunocompromised patients. different types of vaccine for polio. They want Which vaccine will you not give to this patient? to know why their daughter needs to get (A) PCV another shot rather than just taking the oral (B) varicella vaccine form of the vaccine. You tell the parents the (C) hepatitis B vaccine major advantage of the injectable vaccine is which of the following? (D) Hib (E) IPV (A) lower cost (B) increased mucosal immunity (C) better efficacy (D) avoidance of vaccine-associated paralytic poliomyelitis (E) boosting her immunity through secondary transmission (c) ketabton.com: The Digital Library

98 3: Pediatrics

60. A 7-year-old child is scheduled for an elective (A) obtaining a chest x-ray tonsillectomy. The most important instruction (B) obtaining an electrocardiogram (ECG) to the parents should be to make sure that the (C) arterial blood gas determinations child does which of the following? (D) inserting an oropharyngeal airway (A) avoids contact with other children (E) administration of naloxone (B) discontinues antibiotics 72 hour before surgery 64. A 4-month-old baby is in for a well-child check (C) avoids aspirin and antihistamines for and routine immunizations. The baby had a 2 weeks before surgery fever of 39°C the day he received his 2-month (D) does not drink from siblings’ cups immunizations. The parents have read about the vaccine on the Internet and express their (E) eats iron-laden foods for 3 weeks before concerns. Which of the following is an absolute surgery contraindication to giving the diphtheria and tetanus toxoids and acellular pertussis (DTaP)? 61. Routine examination of an otherwise healthy kindergarten child with a history of asthma (A) history of fever >38°C after previous reveals a BP of 140/90 mmHg. Which of the fol- vaccination lowing is the most likely cause of the hyper- (B) history of local reaction after previous tension? vaccination (redness, soreness, swelling) (A) theophylline toxicity (C) family history of seizures (B) chronic lung disease (D) encephalopathy within 7 days of admin- (C) renal disease istration of previous dose of vaccine (D) coarctation of the aorta (E) current antibiotic therapy (E) obesity 65. A 16-year-old male is brought to the emergency department with a crush injury due to a farm 62. A 13-year-old girl presents with lethargy, fever, accident. His immunization status is unknown. severe headache, and a stiff neck. On exami- The wound is heavily contaminated with soil, nation, a unilateral fixed, dilated pupil and and you are concerned about tetanus. Which of papilledema are noted. Which of the follow- the following is the most appropriate manage- ing is the most appropriate initial step in man- ment step? aging this patient? (A) administer a Tdap vaccination (A) administration of IV cefotaxime (B) administer a Td vaccine only (B) administration of IV mannitol (C) administer Tdap and tetanus immune (C) CT of the head globulin (TIG) (D) intubation and hyperventilation (D) administer TIG only (E) performance of a lumbar puncture (E) await immunization records 63. A newborn infant requires repeated resuscita- 66. Children with sickle cell anemia are at tion in the delivery room because of failure to increased risk of developing overwhelming breathe and cyanosis. During spells of crying, infection with certain microorganisms. Which which appear to alleviate the cyanosis, his of the following is the most reasonable step to breath and heart sounds are normal, as is direct prevent such infection? laryngoscopy. Vigorous respiratory movements appear ineffectual. Immediate management of (A) periodic injections of gamma globulin this infant consists of which of the following? (B) injection of VZIG after exposure to varicella (C) withholding live virus vaccines (c) ketabton.com: The Digital Library

Questions: 60–72 99

(D) prophylactic administration of oral (A) congenital biliary atresia penicillin daily (B) isoimmune hemolytic disease (E) early use of amoxicillin at home for (C) Crigler-Najjar syndrome episodes of fever (D) breast milk jaundice (E) breast-feeding jaundice 67. Varicella vaccination is a live virus vaccine. It is generally not recommended in immunocom- 70. A 7-month-old baby presents with a history of promised patients. Which of the following is an constipation for 1 month. He has one hard stool exception to this rule? every week. He has been well otherwise. His (A) children on high doses of corticosteroids physical examination is normal. Which of the (B) leukemia in inducton therapy following is the most likely cause of his prob- (C) lymphoma lem? (D) congenital T-cell abnormalities (A) hypothyroidism (E) leukemia in remission for >1 year and a (B) lead poisoning normal lymphocyte count (C) functional constipation (D) Hirschsprung disease 68. A 1-year-old patient is in the office for a health (E) hypocalcemia maintenance visit and is ready for immuniza- tions. The child has a mild upper respiratory Questions 71 and 72 infection and a low-grade fever. The mother does not want the child to receive vaccine A 10-year-old boy presents with a 3- to 4-day history because she has been told that the vaccine of left ear pain. He is afebrile; he has had no symp- could make the illness worse. You tell her the toms of cold or cough. He has been swimming daily. only true contraindication to vaccination is On physical examination, there is pain on moving which of the following? the pinna and the tragus. There is erythema and swelling of the ear canal; the tympanic membrane is (A) if the child has a skin rash obscured by thick white discharge. (B) if there is an immunosuppressed adult in the household 71. Which of the following is the most likely diag- (C) if the child has hypersensitivity to a nosis? vaccine component (D) if a pregnant woman is in the household (A) otitis externa (E) if the mother is breast-feeding (B) furunculosis (C) otitis media with effusion 69. A 4-day-old infant presents with yellow dis- (D) mastoiditis coloration of the skin and sclera. The baby was (E) foreign body in the ear born at term by a normal vaginal delivery. Pregnancy was uncomplicated; there were no 72. Which of the following is the most likely organ- risk factors for sepsis and no history of mater- ism involved in this case? nal alcohol or drug use. The baby is breast-fed (A) S. aureus and has been nursing every 2 hours, about 10 minutes at each breast. The bilirubin level is (B) Proteus mirabilis 15 mg/dL (all unconjugated), the hematocrit is (C) Candida 45%, and the Coombs test is negative. Which of (D) Pseudomonas aeruginosa the following is the most likely diagnosis? (E) Streptococci (c) ketabton.com: The Digital Library

100 3: Pediatrics

73. A specific pattern of abnormalities has been not seem to bother the child. On examining his identified among infants born to mothers who gait, his knees point forward and his feet turn consume moderate-to-large amounts of alco- inward. Which of the following is the most hol during their pregnancies. Which of the fol- likely cause of this condition? lowing abnormalities is characteristic of these (A) adducted great toe infants? (B) femoral anteversion (A) cataracts (C) Legg-Calvé-Perthes disease (B) developmental dysplasia of the hip (D) medial tibial torsion (C) gonadal dysgenesis (E) metatarsus adductus (D) neural tube defects (E) mental retardation 77. A 13-year-old girl presents with parental con- cerns of poor posture. She has not had any back 74. A 4-year-old girl presents to the emergency pain. On examination, she has unequal shoul- department with fever and a petechial rash. A der height, asymmetric flank creases, and a for- sepsis workup is performed, and IV antibiotics ward-bending test that shows rib asymmetry. are administered. Gram-negative diplococci are The physical examination is otherwise normal. identified in the CSF. Which of the following is Which of the following is the most likely cause true of this condition? of her condition? (A) Antibiotic prophylaxis of fellow daycare (A) congenital scoliosis attendees is not necessary. (B) leg length inequality (B) The most common neurologic residual (C) idiopathic scoliosis is seizures. (D) postural roundback (C) The presence of meningitis decreases (E) Scheuermann kyphosis the survival rate. (D) Shock is the usual cause of death. 78. After 10 days of nasal congestion and rhinor- (E) Vancomycin administered intravenously rhea, a 3-month-old infant develops a severe is the treatment of choice. hacking cough during which he repeatedly turns dusky and appears to choke on or to 75. A 3-year-old boy suddenly begins choking and vomit profuse thick, clear nasopharyngeal coughing while eating peanuts. On physical mucus. For 7 days, the coughing continues examination he is coughing frequently. He has unabated. On physical examination, he is inspiratory stridor and mild intercostal and afebrile and his lungs are clear. His chest x-ray suprasternal retractions. Initial management is normal. His WBC count is 24,000/mm3, with should include which of the following? 15% polymorphonuclear cells, 82% lympho- cytes, and 3% monocytes. Which of the fol- (A) back blows lowing antibiotics should be used to treat this (B) abdominal thrusts patient? (C) blind finger-sweeps of the hypopharynx (A) amoxicillin (D) permitting him to clear the foreign body by coughing (B) amoxicillin-clavulanic acid (E) emergency tracheostomy (C) erythromycin (D) tetracycline 76. During a well-child visit, the grandmother of (E) no antibiotics are necessary an 18-month-old patient is concerned because the child’s feet turn inward. She first noticed this when her grandson began to walk. It does (c) ketabton.com: The Digital Library

Questions: 73–85 101

79. A 16-year-old girl presents with a history of (A) rapid streptococcal antigen test primary amenorrhea. On examination, short (B) heterophil titer stature and a short neck with a low posterior (C) Epstein-Barr virus (EBV) titer hairline are noted. Chromosomal analysis most (D) chest x-ray likely would reveal which of the following? (E) bone marrow examination (A) fragile X (B) trisomy 18 83. A 2-year-old child is brought to the emergency (C) trisomy 21 department with sudden onset of unrespon- (D) 45,XO siveness, miosis, bradycardia, and muscle fas- ciculations. These findings are most suggestive (E) XXY of poisoning with which of the following? 80. A beekeeper’s previously healthy 6-month-old (A) acetaminophen son develops gradual onset of lethargy, poor (B) organophosphates feeding, constipation, and generalized weakness. (C) salicylates On taking a history, you determine that the child (D) tricyclic antidepressants has recently been placed on a homemade for- mula consisting of evaporated milk, water, and (E) vitamin A honey. Which of the following is the most likely explanation for this symptom complex? 84. A 2-year-old boy presents with refusal to use his right arm for 1 day. He is otherwise well. (A) sodium intoxication His mother states she pulled upward on his (B) Hirschsprung disease arm the previous evening to keep him from (C) hypothyroidism tripping down the stairs. Which of the follow- (D) spinal cord tumor ing is the most likely diagnosis? (E) botulism (A) Colles fracture (B) fractured clavicle 81. A 12-month-old patient has allergies to multi- (C) greenstick fracture of the humerus ple foods. The child’s mother has eliminated (D) rotator cuff injury the foods from the diet and wants to know if these allergies will be lifelong. You tell her that (E) subluxation of the radial head some allergies do get better if the food is elim- inated for 1–2 years. In which of the following 85. A 4-year-old child presents with an enlarged is the allergy most likely to resolve, with elim- submandibular node that is 4 cm in diameter, ination of the food from the diet? nontender, and not fluctuant. The node has been enlarged for about 4 weeks, and there is (A) peanuts no history of fever or contact with any person (B) milk who was ill. A CBC is normal, and a Mantoux (C) nuts test with 5 tuberculin units of PPD shows 6 (D) fish mm of induration. Which of the following is the most likely diagnosis? (E) shellfish (A) cat-scratch fever 82. A 4-year-old child manifests symptoms of (B) acute pyogenic lymphadenitis fever, sore throat, and swollen lymph nodes. (C) acute lymphoblastic leukemia The spleen tip is palpable. Throat culture and (D) tuberculous lymphadenitis rapid slide (Monospot) test results are nega- tive. The next logical diagnostic procedure (E) atypical mycobacteria lymphadenitis would involve which of the following? (c) ketabton.com: The Digital Library

102 3: Pediatrics

86. A 4-year-old child with grade III vesicoureteral (A) cytomegalovirus (CMV) reflux has recurrent UTIs despite adequate (B) HSV antibiotic prophylaxis. Which of the following (C) GBS is the most appropriate next step in the treat- (D) T. gondii ment of this patient? (E) T. pallidum (A) IV antibiotic treatment for 2 weeks (B) repeat renal scan 90. A 7-month-old patient presents with a history (C) renal arteriogram of 3 days of fever to 104°F, which resolved the (D) antireflux surgery same day that an exanthem erupted. The exan- them is prominent on the neck and trunk. It is (E) addition of vitamin C (ascorbic acid) to macular, with discrete lesions 3–5 mm in diam- the treatment regimen eter. Which of the following is the most likely diagnosis? 87. A 2-week-old Caucasian male presents with constipation since birth. He was born full term (A) erythema infectiosum via a normal vaginal delivery. He did not pass (B) measles meconium till his 3rd day of life, after he was (C) roseola infantum given a glycerin suppository. He has since (D) rubella stooled every 3–4 days, only with the help of a suppository. The stools are pellet like. He has (E) scarlet fever had increasing abdominal distention. On rectal examination, tone appears normal and the 91. A 3-year-old male presents after having a tonic- ampulla contains no stool. Which of the fol- clonic seizure lasting about 1 minute. On exam- lowing is the most likely cause? ination, the child now has no nuerologic abnormalities. He has a temperature of 40.3°C (A) cystic fibrosis and has an obvious otitis media on the left but (B) Hirschprung disease no other abnormalities on physical examina- (C) anal stenosis tion. You correctly counsel the family with (D) functional constipation which one of the following statements? (E) hypothyroidism (A) The child will need hospitalization, a lumbar puncture, and antibiotics. 88. Which of the following is the most appropriate (B) An EEG and CNS imaging must be evaluative procedure for an otherwise normal done. 7-day-old boy with perineal hypospadias? (C) Anticonvulsants must be stated and (A) renal ultrasonography continued for 6 months. (B) serum creatinine determination (D) There is a slight increase in risk for (C) cystography development of epilepsy. (D) circumcision (E) The child must be monitored carefully for long-term neurologic damage. (E) intravenous pyelography (IVP) Questions 92 through 94 89. A 2-week-old infant presents with hepatosplenomegaly and a thick, purulent, Children with sickle cell disease are at risk for cer- bloody nasal discharge. Coppery, oval, macu- tain conditions with characteristic presentation. lopapular skin lesions are present in an acral Match the clinical scenario with the syndrome. distribution. The neurologic examination is normal, including head circumference. Which (A) acute chest syndrome of the following is the most likely cause of this (B) acute splenic sequestration congenital infection? (C) aplastic crisis (c) ketabton.com: The Digital Library

Questions: 86–99 103

(D) hand-foot syndrome (D) E. coli (E) Salmonella osteomyelitis (E) group A Streptococcus (F) vasoocclusive crisis 97. A few weeks after a presumed viral respiratory 92. A 12-month old patient presents with a 3-day infection, a 4-year-old girl presents with bruis- history of lethargy and fever. He has also had ing and petechiae. Bone marrow examination rhinorrhea and a cough. On physical examina- reveals increased numbers of megakaryocytes tion, he is pale, tachycardic, and has a left upper but is otherwise normal. Hb is 13.5 g/100 mL. quadrant mass. His hemoglobin is 4 g/dL, Platelet count is 30,000/mm3. Which of the fol- platelet count is 100,000, and WBC is 15,000 lowing would be appropriate for this child at with 50% segmented neutrophils. His reticulo- this time? cyte count is 15%. (A) daily prednisone (B) a transfusion of packed RBCs and 93. An 8-month-old patient with sickle cell disease platelets presents with a 2-day history of painful, swollen fingers and toes. (C) IV gamma globulin (D) splenectomy 94. A 5-year-old boy with sickle cell anemia pres- (E) no specific therapy ents with a 2-day history of increasing pain in both legs. He has had multiple admissions for 98. A 4-month-old child presents with a 2-day his- similar episodes of pain. On examination, he tory of vomiting and intermittent irritability. has no swelling of the legs. There is diffuse ten- On examination, “currant jelly” stool is noted derness along both legs. There is no joint in the diaper, and a sausage-shaped mass is involvement, and there is full range of move- palpated in the right upper quadrant of the ment. abdomen. Which of the following conditions is most likely to cause this? 95. A 1-day-old infant who received silver nitrate (A) appendicitis eye drops in the delivery room is suffering from bilateral purulent conjunctival discharge. (B) diaphragmatic hernia Which of the following is the most likely cause (C) giardiasis of this child’s condition? (D) intussusception (E) rotavirus gastroenteritis (A) N. gonorrhoeae infection (B) herpes simplex infection 99. During a routine yearly checkup, a 10-year-old (C) nasolacrimal duct obstruction boy is found to have 2+ proteinuria on urinal- (D) chemical irritation ysis. Which of the following would be the most (E) Pseudomonas infection appropriate diagnostic test? (A) electrolytes, BUN, and serum creatinine 96. A 2-year-old girl presents with fever of 39.3°C and irritability. She has had an upper respira- (B) antistreptococcal antibodies tory tract infection for 4 days. On examination, (C) IVP the right ear is bulging and has poor move- (D) renal ultrasound ment on insufflation. Which of the following (E) a repeat urinalysis organisms is most likely responsible for these findings? (A) S. pneumoniae (B) S. aureus (C) M. pneumoniae (c) ketabton.com: The Digital Library

104 3: Pediatrics

100. An 18-month-old boy has received 5 days of playing vigorously. The pain does not radiate. The amoxicillin for otitis media. He continues to pain and dyspnea resolve with rest. She does not have fever, and on physical examination, the have palpitations or any lightheadedness associated right tympanic membrane is bulging with with the pain. She does not have pain or dyspnea at purulent fluid behind it. Which of the follow- other times. There is no history of early cardiac ing is the best antibiotic to use? deaths or unexplained deaths of young people in her family. Her physical examination is normal, (A) amoxicillin-clavulanic acid except for a grade 2/6 systolic vibratory murmur (B) dicloxacillin heard at the left lower sternal border. (C) cephalexin (D) erythromycin 103. Which of the following is the most likely cause (E) penicillin of her symptoms? (A) angina Questions 101 and 102 (B) asthma A 13-year-old boy presents for evaluation of short (C) costochondritis stature. His growth chart from ages 2 through 12 (D) esophagitis years is shown in Figure 3-4. His growth in the first (E) mitral valve prolapse 2 years of life was typically at the 25th percentile. He has been healthy, has a good appetite, and is 104. The same patient now complains of palpita- doing well in school. He lives with his parents and tions and dizziness with the chest pain. Which is an only child. His parents’ heights are both at the of the following tests should be ordered for 50th percentile. His father states that he grew sev- this patient? eral inches after he completed high school. A com- plete physical examination is normal. His Tanner (A) chest x-ray stage is I. (B) echocardiogram (C) pulmonary function tests 101. Which of the following is the most likely cause (D) 24-hour Holter monitoring of this patient’s short stature? (E) cardiac enzymes (A) constitutional delay (B) deprivational dwarfism 105. A 10-year-old boy presents with red discol- (C) familial short stature oration of the urine since the morning. He is healthy and otherwise asymptomatic. He (D) growth hormone deficiency denies dysuria, frequency, urgency, flank, or (E) hypothyroidism abdominal pain. His BP is normal. His exami- nation is within normal limits including 102. Which of the following tests is the most appro- abdomen and genitourinary system. There is priate next step in the care of this patient? no rash or edema. His urine is pink in color; (A) bone age urinalysis is negative for hemoglobin or pro- (B) cranial imaging tein. No white cells, red cells, or bacteria are noted. Which of the following is the most (C) growth hormone stimulation appropriate next step? (D) thyroid function tests (E) no tests are necessary (A) obtain a recent dietary and drug history (B) obtain a urine culture Questions 103 and 104 (C) test for myoglobin in the urine A 12-year-old girl presents with chest pain when (D) obtain a renal ultrasound she plays basketball. The pain is substernal, is asso- (E) obtain antistreptococcal antibodies ciated with dyspnea, and occurs after she has been (c) ketabton.com: The Digital Library

BOYS: 2 TO 18 YEARS PHYSICAL GROWTH NCHS PERCENTILE 3456789101112131415161718 77 195 76 75 190 74 95 73 185 90 72 75 71 180

70 50 69 175 68 25 67 170 10 66 5 65 165 250 64 63 160 110 240 62 61 155 S 105 230 60 T A 59 150 T 100 220 58 U R 57 145 95 210 E 95 56 200 55 140 90 54 190 90 53 135 85 52 180 51 130 80 50 170 75 49 125 75 160 48 47 120 70 150 46 50 115 65 45 140 44 25 43 110 60 130 42 10 41 105 55 120 5 40 100 50 110 39 38 95 45 100 37 36 90 90 40 35 W E 34 80 85 I 35 33 G H 32 70 80 T 30 31 60 30 75 25 29 50 in cm 20 40 15 30 AGE ( YEARS) kg lb 3456789101112131415161718 FIG. 3-4

105 (c) ketabton.com: The Digital Library

106 3: Pediatrics

106. A 12-year-old girl has had a sore throat over 2 with 60% neutrophils, 35% lymphocytes, and days. She now has a fever of 39.5°C and has dif- 5% monocytes. His hemoglobin is 12.0 g/dL ficulty opening her mouth, swallowing, or and platelet count is 500,000/ mm3. Which of speaking. Her throat can be visualized with the following is the most likely diagnosis? difficulty, the right tonsil is significantly more (A) erythema infectiosum (fifth disease) enlarged than the left, and the uvula is dis- placed to the left side. Which of the following (B) Kawasaki disease is the most likely diagnosis? (C) rubella (D) rubeola (measles) (A) retropharyngeal abscess (E) rheumatic fever (B) acute uvulitis (C) peritonsillar abscess 108. An athletic 12-year-old boy complains of left (D) acute pharyngitis knee pain when he runs and plays sports. The (E) lateral pharyngeal abscess pain resolves when he rests. He has otherwise been well. His physical examination is normal, 107. An 18-month-old boy presents with a history of except for swelling and increased prominence fever to 39.0°C for 5 days. He has also been over the left tibial tubercle. A radiograph of the irritable and has not been drinking well. left knee is normal. Which of the following is Associated symptoms include red eyes, a rash, the most likely diagnosis? and some trouble walking. On physical exam- (A) Legg-Calvé-Perthes disease ination, he has a temperature of 39.5°C. He has bilateral bulbar conjunctivitis, a strawberry (B) Osgood-Schlatter disease tongue, an inflamed oral pharynx, edema of (C) patellar subluxation the hands and feet, a morbilliform rash, and (D) popliteal cyst cervical lymphadenopathy. He is very irrita- (E) slipped capital femoral epiphysis ble. His CBC shows a WBC of 15,000/mm3 (c) ketabton.com: The Digital Library

Answers and Explanations

1. (B) Generally, the interpretation of tuberculin immunoprophylaxis (HBIG), preferably within skin test (TST) is the same regardless of BCG 12 hours of life. The immunization series should status. Induration >5 mm is considered posi- be completed by 6 months of life. The child tive in children in close contact with known or should have serology testing 1–3 months after suspected cases of tuberculosis disease or chil- completion of the series. Testing for anti-HBs dren suspected to have tuberculosis disease. will establish if additional vaccine doses are Induration >10 mm is considered positive in needed; testing for HBsAg will identify infants children at greater risk of disseminated disease who are chronically infected. Mothers whose (age <4 years; other medical conditions such as HBsAg status is unknown should be tested as lymphoma, diabetes, chronic renal failure, or soon as possible; the first vaccine dose should malnutrition) or children at greater risk of expo- be given within 12 hours of birth. If the woman sure to tuberculosis disease (born in, or parents is found to be positive, HBIG should be given as born in high-prevalence regions, travel to these soon as possible, not later than 7 days of age. regions, exposure to adults at high risk, such as (American Academy of Pediatrics, 2006, pp. 351–352) HIV infected, homeless, or drug abusers). Induration >15 mm is positive in children >4 3. (A) Pityriasis rosea is a papulosquamous eruption years without any risk factors. Radiographic consisting of multiple oval-shaped scaling lesions evaluation of all children with positive TST is which are truncal in distribution. This eruption recommended. Latent tuberculosis infection is resembles the papulosquamous eruption of sec- defined as an infection in a person with a posi- ondary syphilis, although the rash of secondary tive TST, no physical findings of the disease, syphilis often involves the palms and soles. The and a chest radiograph that is either normal or etiology of pityriasis rosea is unknown, but is felt reveals only granulomas or calcifications in the to be viral. It is a self-limiting illness lasting sev- lungs or regional lymph nodes. Children with eral weeks to a few months, and there is no ade- latent tuberculosis infection should receive pro- quate treatment other than symptomatic phylaxis, usually 9 months of INH. Those with treatment of pruritus, when necessary. (McMillan symptoms, signs, and/or radiographic mani- et al., 2006, pp. 83–84) festations are said to have tuberculosis disease. There is no benefit to repeating the test in 3–6 4. (E) Children with chicken pox may be infectious months, and it will delay treatment. Sputum for 1 or 2 days before the appearance of the rash. cultures are difficult to obtain in younger chil- Once skin lesions have crusted, the patient is dren. Gastric aspirate specimens obtained with no longer infectious. Susceptible individuals can a nasogastric tube are preferred. Culture mate- contract chicken pox from patients with zoster. rial should be obtained in children with evi- In the cases of both chicken pox and zoster, dence of the disease in order to obtain transmission is thought to occur by the respira- information on drug susceptibility and resist- tory route rather than by direct contact. The ance patterns. (American Academy of Pediatrics, 2006, virus can travel long distances in the air and pp. 678–697) remain viable. Transmission from one hospital patient to other susceptible hospitalized patients 2. (C) Transmission of perinatal hepatitis B virus has been reported to occur through air vents. (HBV) infection can be prevented in 95% of VZIG should be given within 3 or 4 days of infants born to HBsAg-positive mothers by exposure to varicella-susceptible individuals who early active (immunization) and passive are immunocompromised. (American Academy of

107 (c) ketabton.com: The Digital Library

108 3: Pediatrics

Pediatrics, 2006, pp. 715–719; McMillan et al., 2006, blue-filtered light. Treatment consists of fre- pp. 1263–1267) quent applications of topical antibiotic ointment until the epithelium is healed. The use of a patch 5. (A) Prone sleeping is a major risk factor for does not accelerate healing, and if improperly sudden infant death syndrome (SIDS). Since applied, may abrade the cornea. Referral to an the 1992 American Academy of Pediatrics ophthalmologist should be considered if there (AAP) recommendation that infants be placed are significant changes in vision, or signs of to sleep on their backs, the frequency of prone deeper or more penetrating injury which often sleeping has decreased from 70 to 20%, and the result in papillary abnormalities. (Behrman et al., SIDS rate has decreased by >40%. Side sleeping 2006, p. 2129) has a slightly higher SIDS risk than supine but is still safer than the prone position. Other risk 8. (C) The AAP recommends that children should factors include maternal smoking, soft bed- face the rear of the vehicle until they are at least ding, overheating, younger maternal age, pre- 20 lbs and 1 year of age to reduce the risk of cer- maturity, low birth weight, and male gender. vical spine injury in the event of a crash. Infants Rates among African Americans and Native who weigh 20 lbs before 1 year of age should Americans are two to three times the national ride rear facing in a convertible seat or infant average. The issue of bed sharing or cosleeping seat approved for higher weights until 1 year of is controversial. There are reports of overlying age. A car seat should never be placed in the by adults leading to suffocation, especially front passenger seat. (American Academy of Pediatrics, when the adult uses drugs or alcohol. Bed shar- 1996, pp. 761–762) ing with multiple family members may be haz- ardous; there is increased risk of overlying, 9. (C) Though infection must be considered as an entrapment, rolling into prone position, and etiology, acute trauma is more likely in this sce- use of soft sleeping surfaces. Some studies nario. This case represents the classic picture of show that infants have more arousals and less the shaken baby syndrome which produces slow-wave sleep during bed sharing; however, intracranial trauma without obvious external there is no epidemiologic evidence that bed shar- findings. This infant is critically ill and lacks ing is protective. (American Academy of Pediatrics, preceding illness or constitutional symptoms. 2000b, pp. 650–656) The tense fontanels reflect increased intracra- nial pressure. A cranial CT scan may show dif- 6. (A) Children, especially those in daycare, com- fuse edema or a localized lesion, such as a monly are infected with the hepatitis A virus. subdural hemorrhage. Metabolic causes of Unlike adults, children most often are asymp- seizures do not cause increased intracranial tomatic. Frequently, outbreaks of hepatitis A pressure. Acetaminophen toxicity does not cause in a daycare center are not recognized until a CNS symptoms. (Kliegman et al., 2004, pp. 518–521; daycare worker or parent of an attendee Rudolph and Rudolph, 2003, pp. 463–469) becomes ill. Immunization against hepatitis A virus is now routinely recommended because 10. (B) The insect order Hymenoptera includes of this. After exposure both immune globulin ants, bees, and wasps. Their venom usually and vaccine should be given to the unvacci- only causes a local reaction. About 1–4% of the nated individual. (American Academy of Pediatrics, population is sensitized to the venom and at 2006, pp. 326–335) risk for immediate hypersensitivity reactions. Reactions may include urticaria, angioedema, 7. (A) Superficial corneal injuries expose under- wheezing, or hypotension. Severe reactions lying layers causing pain, photophobia, tear- should be treated with IV fluids, oxygen, and ing, and decreased vision. Irrigation is epinephrine. Although the child responded recommended only if a foreign body is sus- well to diphenhydramine, because there was a pected. Abrasions are detected by instilling flu- systemic reaction, it is advisable to carry an orescein dye and inspecting the cornea using Epi-pen Jr at all times. Only children with (c) ketabton.com: The Digital Library

Answers: 5–14 109

life-threatening systemic reactions need to be and it spontaneously stops in at least 15% of referred for desensitization. Testing IgE or skin- affected children every year. Psychological fac- prick test with Hymenoptera venom is not pre- tors are often involved in secondary enuresis. A dictive of future systemic reactions. (Behrman et al., careful history should be obtained to rule out 2006, p. 2177) such organic factors as UTI (dysuria, frequency, urgency). Children with diabetes insipidus or 11. (D) Children frequently insert foreign bodies diabetes mellitus have polydipsia and polyuria. into the nose. Initial symptoms are local Urinalysis should be considered to rule out an obstruction, sneezing, and pain. Subsequently, organic cause. In diabetes mellitus, urinalysis there is swelling and infection leading to a may reveal glycosuria and ketonuria. A urine- purulent, malodorous, and often bloody dis- specific gravity of >1.015 makes diabetes charge. The infection clears after removal of insipidus unlikely. (Behrman et al., 2006, pp. 74–75) the foreign body. Nasal polyps cause obstruc- tion of the nasal passages, hyponasal speech, 13. (E) Active treatment should be avoided in chil- and mouth breathing; gray, grape-like masses dren under age 6 years, as nocturnal enuresis is can be visualized on nasal examination. An common. Parents should be reassured that the upper respiratory infection is usually sug- condition is self-limited. Fluid intake 1 hour gested by a careful history. Initial symptoms before sleep should be restricted. Simple behav- include a scratchy throat, followed by devel- ioral reinforcement, such as a star or sticker chart opment of thin nasal discharge and sneez- to record dry nights, may be helpful. Punitive or ing. Myalgia, low-grade fever, headache, humiliating measures should be discouraged. malaise, and decreased appetite may be pres- Bladder-stretching exercises and encouraging ent. By the 2nd or 3rd day, the discharge children to hold urine for longer periods during becomes thicker and more purulent. Cough is the day are usually not helpful. Pharmacologic common. Symptoms usually resolve by 7–10 therapy is not curative. DDAVP is a synthetic days. Adolescents with sinusitis may have clas- analog of antidiuretic hormone. It reduces urine sic symptoms of headache and sinus tender- production overnight. Hyponatremia has been ness. In children, cough and nasal discharge are reported with use of this drug. If used, it should common; the cough is worse when supine. If only be for a limited time. Imipramine is a tri- upper respiratory infection symptoms persist cyclic antidepressant which was used more often without improvement for >10 days, sinusitis in the past. It is effective in 30–60% of children, should be considered. A more acute form may but side effects include anxiety, insomnia, and occur, with a shorter duration and more severe dry mouth. There is a poisoning risk, especially symptoms such as fever >39°C, purulent nasal for younger children. Conditioning therapy may discharge, headache, and eye swelling. Children be considered in children older than 6 years. with allergic rhinitis present with sneezing, Success rates range from 30 to 60%. It involves clear watery, rhinorrhea, and itching of the nose, the use of an alarm attached to electrodes in the palate, pharynx, and eyes. Itching, redness, and underwear, which sounds when the child voids. tearing of the eyes may be present. This occurs Consistent use of the device is often helpful; it is in response to exposure to an allergen such as more effective in older, more motivated children. pollen, mold spores, and animal or mite anti- A common complaint is that the alarm wakes gens. (Behrman et al., 2006, pp. 1357, 1386–1389) up other family members but not the affected child. (Behrman et al., 2006, pp. 74–75) 12. (B) Enuresis may be primary (75%) where noc- turnal control was never achieved; secondary 14. (D) Gynecomastia is the enlargement of male enuresis (25%) is when the child was dry at breast tissue and occurs in approximately one- night for at least a few months. Nocturnal third of adolescent males during early- to mid- enuresis is more common in boys, and family puberty. It usually resolves spontaneously and history is positive in at least 50%. This may requires no further evaluation beyond a careful affect as much as 20% of children at age 5 years, history and physical examination. Features (c) ketabton.com: The Digital Library

110 3: Pediatrics

include: breast tissue <4 cm in diameter and 16. (B) resembling female breast budding, and puber- tal development between Tanner stage II and IV. 17. (B) Pubertal development signs precede gyneco- mastia by at least 6 months. It may be more 18. (B) noticeable in obese boys. A drug and medication history should be 19. (D) obtained; these include estrogens, androgens, human chorionic gonadotropin (hCG), cardio- Explanations 16 through 19 vascular drugs (reserpine, methyldopa, digi- talis), cytotoxic agents (busulfan, vincristine), Urine for urinalysis and culture must be prop- antituberculosis drugs (INH), psychoactive erly obtained. Catheterization is the most reli- drugs (tricyclic antidepressants, diazepam), able method of the choices offered. Suprapubic ketoconazole, spironolactone, cimetidine, and tap is considered the “gold-standard” but is not phenytoin. Illegal drugs include marijuana, always technically feasible, especially in an out- heroin, methadone, amphetamines, as well as patient office setting. A midstream, clean catch alcohol. specimen would be acceptable in an older, If there is evidence of precocious puberty, toilet-trained child. “Bagged” specimens are not hypogonadism or macrogynecomastia (breast recommended because of possible skin or fecal tissue >5 cm diameter), laboratory testing contamination of the specimen. Similarly, should be done including dehydroepiandros- obtaining a sample from a diaper or potty terone sulfate (DHAS), FSH, and LH, hCG, would be unacceptable. Urinalysis includes dip- estradiol, and testosterone. Thyroid-stimulating stick method and microscopic examination. hormone (TSH) may be obtained to rule out Leukocyte esterase (an enzyme in WBC) and hyperthyroidism. Boys with Klinefelter syn- nitrites suggest probable infection. Microscopic drome have hypogonadism (testes <3 cm in analysis of unspun urine for WBC (>10/high- diameter), delayed pubertal development, power field) or bacteria is also predictive of and gynecomastia. Laboratory tests reveal infection. RBCs are often present in a UTI. increased FSH and LH, and decreased testos- The patient is vomiting and dehydrated; terone; the diagnosis is confirmed by chromo- this may indicate possible pyelonephritis. The some analysis. If DHAS, hCG, or estradiol lev- most appropriate course would be IV hydration els are increased, an MRI of the head to and empiric treatment with antibiotics (ceftriax- exclude a CNS tumor and ultrasound of one) while awaiting cultures. Children with abdomen and testes to rule out an adrenal, pyelonephritis are at increased risk of renal scar- liver, or testicular tumor should be consid- ring, especially younger children, and should be treated early. E. coli is the most common organ- ered. (Mahony, 1990, pp. 1389–1404) ism cultured; others include Proteus, Klebsiella, S. 15. (A) Wilms tumor is a malignant embryonal saprophyticus, and Enterococcus. The occurrence neoplasm of the kidney. It is the second most of a UTI in a girl under age 3–5 years and in a common solid tumor of childhood. Girls are boy of any age may be a marker for an underly- affected more frequently than boys (2:1). The ing congenital anatomic abnormality, in particu- incidence of Wilms tumor peaks at 1–3 years of lar, vesicourethral reflux. Radiologic investiga- age. The classic presentation is a painless tion with renal ultrasound and VCUG is recom- abdominal mass that is usually hard, smooth, mended. (Behrman et al., 2006, pp. 1785–1789) and unilateral. Hematuria occurs in 12–25% of children with Wilms tumor, and hypertension 20. (E) Because many childhood viral illnesses has been reported in up to 60% of patients. have seasonal presentations, the etiologic agent Aniridia or hemihypertrophy may be observed may be suspected on the basis of clinical and seasonal presentation. Yearly winter outbreaks in patients with Wilms tumor. (McMillan et al., 2006, pp. 1775–1777) of bronchiolitis and pneumonia are associated (c) ketabton.com: The Digital Library

Answers: 15–29 111

with respiratory syncytial virus. Summer out- defects, are the most common heart defects breaks of gastroenteritis are associated with among children with Down syndrome. (Rudolph enterovirus, while winter outbreaks are asso- and Rudolph, 2003, pp. 2087–2090) ciated with rotavirus. Although adenovirus can cause diarrhea, it more commonly causes respi- 25. (C) Marfan syndrome is a genetic disorder of ratory symptoms. Human herpes virus type 6 connective tissue. It is transmitted in an auto- is the etiologic agent in roseola infantum. somal dominant manner. Patients have tall (McMillan et al., 2006, pp. 1395–1401, 1225–1229) stature and skeletal disproportion, where the arm span exceeds the height. Other important 21. (B) Penicillin remains the drug of choice for treat- clinical features include subluxation of the ment of streptococcal pharyngitis. Amoxicillin, ocular lens which occurs in 50–80% of patients. macrolides, and cephalosporins are acceptable Progressive dilatation of the aortic root and alternatives. (McMillan et al., 2006, pp. 1490–1491, ascending aorta can lead to dissection or rup- 1223–1230) ture. (Rudolph and Rudolph, 2003, pp. 762–764)

22. (C) Scarlet fever is caused by toxins made by 26. (E) Marfan syndrome is associated with mitral group A Streptococci. It is usually seen in valve prolapse and aortic root dilatation. patients with strep throat. The rash is papular (Rudolph and Rudolph, 2003, p. 764) and described as sandpaper like. Sometimes it is easier to feel it than to see it. An allergic rash 27. (A) SLE is an autoimmune disorder that affects would be urticarial. More than 80% of patients multiple organs. The diagnosis is based on the with EBV infection develop a maculopapular presence of four or more major criteria. These rash if given amoxicillin. This patient’s clinical include malar rash, oral ulcers, arthritis, and course is not typical for EBV which presents photosensitivity. This patient has all of these more gradually, and patients often have poste- symptoms, as well as the systemic symptoms rior cervical adenopathy and splenomegaly. often seen at presentation. The disorder is pre- Patients with serum sickness often have dominately a disease of women and, in the urticarial rashes, sometimes progressing to pediatric population, is a disease of adoles- angioedema. They may also have arthritis, cence. Chronic renal disease is an important myalgias, and lymphadenopathy. The rash in and common cause of morbidity and mortality enteroviral infections is typically macular. among patients with SLE. (Rudolph and Rudolph, (Rudolph and Rudolph, 2003, p. 996, 1225) 2003, pp. 847–851)

23. (D) Patients with Turner syndrome have a 45,X 28. (E) Organic acids produced by bacterial fer- karyotype. The classic physical features are mentation lower the pH of dental plaque caus- illustrated in this case. Patients have short ing demineralization and caries of the adjacent stature, a webbed neck, ptosis, triangular faces, tooth. Nursing bottle caries is a pattern of caries prominent brow, hypertelorism, low-set ears, involving the upper and lower incisors. It and pectus excavatum. (Rudolph and Rudolph, occurs because of prolonged contact of the tooth 2003, pp. 2087–2090) to a sugar-containing liquid (juice or milk). This is more likely to occur with overnight exposure 24. (C) Turner syndrome is associated with coarc- or with use of a bottle. A similar pattern of caries tation of the aorta and aortic stenosis. Williams may rarely occur with breast-fed babies who syndrome is associated specifically with feed through the night. Cup feeding or drinks supravalvular aortic stenosis. In Noonan syn- given during mealtimes are less likely to cause drome, the cardiac defect most often is pul- prolonged contact to the teeth. (Behrman et al., monary valvular stenosis or an atrial septal 2006, pp. 1209–1212) defect. Marfan syndrome is associated with mitral valve prolapse and aortic root dilatation. 29. (E) A tender, boggy mass on the scalp is most Septal defects, primarily endocardial cushion likely to be a kerion. This is an inflammatory (c) ketabton.com: The Digital Library

112 3: Pediatrics

form of tinea capitis. Tinea capitis is more infected animals. Psittacosis is obtained from common in African American children and is exposure to bird feces. (American Academy of the most common cause of alopecia in children. Pediatrics, 2006, pp. 223–224) In the United States, a majority of cases are caused by Trichophyton tonsurans; Microsporum 32. (E) Erythema infectiosum is a common child- canis may also be involved. Tinea capitis may hood viral exanthem caused by parvovirus B19. present with diffuse scaling, a black dot form, It was the fifth in a classification system of child- and as the inflammatory form—kerion. The hood exanthems; the others were rubella, recommended treatment is 8–12 weeks of oral measles, scarlet fever, atypical scarlet fever, and griseofulvin. Shampooing with selenium sul- roseola infantum. The rash classically presents fide shampoo is a helpful adjunctive therapy early with flushed cheeks or a “slapped cheek” which decreases spore shedding. Topical ther- appearance. It is followed by development of a apy alone is ineffective. Incision and drainage macular erythematous rash on trunk and or antibiotic therapy are not recommended in extremities, which then shows central clearing, treatment of tinea capitis. (Behrman et al., 2006, developing a lacy, reticulated appearance. The pp. 2230–2235) infection is often not clinically apparent. Adult and older adolescents, especially females, may 30. (B) Coxsackie A16 is the major cause of hand, develop arthropathy. The symptoms are usu- foot, and mouth disease. This is a summer ally self-limited. Parvovirus B19 is clinically sig- enteroviral illness presenting with classic nificant in people with hemolytic anemias lesions of the hand, feet, and mouth. Herpetic because it may induce a transient aplastic crisis. gingivostomatitis is the most common cause Immunocompromised individuals are also at of stomatitis in children aged 1–3 years. There risk for chronic infections accompanied by is often a high fever, fetor oris, refusal to eat, anemia, neutropenia, and thrombocytopenia. It and irritability. The lesions are initially vesicu- may also induce fetal demise in case of primary lar, and soon form ulcers ranging from 2 to 10 infection of pregnant women. (Behrman et al., 2006, mm in diameter. The tongue, cheek, and gums pp. 1049–1050) are usually involved, and there may be sub- maxillary lymphadenitis. Aphthous ulcerations 33. (B) Centers for Disease Control and Prevention (canker sores) are painful ulcerations, which (CDC) criteria require fever of at least 5 days’ present as erythematous, indurated papules duration for a clinical diagnosis of Kawasaki that erode to form circumscribed necrotic ulcers disease. According to these criteria, patients with gray fibrinous exudates and erythema- also must have at least four of five other find- tous halo. They are 2–10 mm in diameter, heal ings, including bilateral conjunctival infection, spontaneously, and often recur. Behçet syn- one or more changes of the oral mucous mem- drome is a multisystem disorder characterized branes (e.g., pharyngeal erythema; dry, fissured, by recurrent oral and genital ulceration, iritis or and erythematous lips; and strawberry tongue), uveitis, as well as other cutaneous, arthritic, one or more changes of the extremities (e.g., neurologic, vascular, and gastrointestinal (GI) erythema, edema, and desquamation), rash, manifestations. It is rare in children. Traumatic and cervical lymphadenopathy. Kawasaki dis- oral ulcers may be seen in chronic cheek biters ease occurs most commonly during the first 2 but do not involve extremities. (Behrman et al., years of life. Thrombocytosis, rather than 2006, pp. 1042–1048, 1204–1216) thrombocytopenia, is an almost invariable fea- ture late in the course of illness. The most 31. (C) Kawasaki disease is an acute vasculitis of common serious complication of Kawasaki dis- unknown etiology. Humans contract brucel- ease is coronary artery aneurysm formation losis by direct contact with infected animals which can result in thrombosis, aneurysmal or by drinking unpasteurized milk. Babesiosis rupture, or other cardiac effects. (McMillan et al., is transmitted by ticks. Leptospirosis is 2006, pp. 1015–1020) obtained from exposure to the urine of (c) ketabton.com: The Digital Library

Answers: 30–37 113

34. (B) Orbital (also referred to as postseptal) cel- milk contains adequate vitamin C as long as lulitis is a medical emergency. It is a bacterial the mother is not deficient. (Behrman et al., 2006, infection of the orbit. It must be distinguished pp. 186–189) from periorbital (also referred to as preseptal) cellulitis by the presence of proptosis or limi- 36. (D) Testicular torsion is the most common tations of extraocular movements. When cause of testicular pain in boys 12 years and orbital cellulitis is suspected, cultures of blood older and is uncommon in those under 10 and CSF should be obtained, appropriate years. It may be sometimes related to trauma or antibiotics should be administered intra- injury but may occur spontaneously. If not venously, an ophthalmologist should be con- diagnosed early, loss of blood flow to the testi- sulted, and CT films should be obtained to cle may result in permanent loss of testicular delineate the extent of the infectious process. function. Torsion of the testicular appendix Both retinoblastoma and battered child syn- usually occurs between the ages of 2 and 11 drome may present with lid edema. Typically, years. The testicular appendix is a vestigial these children are afebrile and nontoxic in stalk at the upper pole of the testis. Torsion appearance. Hyphema is hemorrhage into the results in pain and swelling of the scrotum, but anterior chamber of the eye and is caused by the onset of pain is more gradual. There is a 3- trauma. Twenty percent of patients with neu- to 5-mm indurated, tender mass at the upper roblastoma present with eye symptoms from pole of the testis. It may sometimes be visible as metastasis. Proptosis is one of the possible pre- a “blue-dot.” Testicular scan may be helpful sentations and can be of relatively acute onset. when this cannot be clinically differentiated In general, other systemic symptoms are pres- from testicular torsion. A hydrocele is a pain- ent and have developed more gradually. less collection of fluid in the tunica vaginalis. (McMillan et al., 2006, pp. 1475, 813–814) Transillumination confirms that the mass is filled with fluid. Hydroceles are present in 35. (D) Babies who are exclusively breast-fed for 1–2% of male newborns and usually resolve prolonged periods of time are at risk for devel- by age 1 year. In older boys, a communicating oping rickets. Dark-skinned infants are at high hydrocele may be associated with an inguinal risk, especially during winter months when they hernia. Inguinal hernias usually appear as a receive inadequate sunlight. Supplementation bulge in the inguinal area extending into the with vitamin D is recommended in children scrotum. Hernias are painless, and are more who are at high risk, as well as pregnant and lac- noticeable during crying or straining. They are tating mothers. Clinical features include cran- painful only when strangulated or incarcer- iotabes, a thinning of the outer table of the skull. ated. Epididymitis is an acute inflammation of This may also occur in osteogenesis imperfecta. the epididymis, and is more common in sexu- Enlargement of the costochondral junctions ally active adolescents. Urinalysis shows (rachitic rosary) may be seen in rickets, scurvy, pyuria and the etiology may be gonococcus or and chondrodystrophy. Other features may chlamydia but is often undetermined. (Behrman include delayed primary teeth, enamel defects, et al., 2006, pp. 1818–1819) and caries. There may be thickening of the wrists and ankles; bending of the femur, tibia, and 37. (B) C. trachomatis is considered an unusual fibula result in bowlegs or knock-knees. cause of otitis media at any age. N. gonorrhoeae Greenstick fractures of long bones may causes conjunctivitis in the newborn. Syphilis occur without symptoms. Diagnosis is based and toxoplasmosis cause congenital infections. on history of inadequate vitamin D intake and E. coli is one of the neonatal pathogens that also clinical features. Diagnosis may be confirmed causes otitis media in neonates. The symptoms by x-rays and chemistry; serum calcium is low of otitis media in newborns are often similar to or normal, serum phosphorus is low, serum those of sepsis; they are subtle and nonspe- alkaline phosphatase is elevated, and serum cific and may include poor feeding, lethargy, 25-hydroxycholecalciferol is decreased. Breast vomiting, or diarrhea. Once the diagnosis is (c) ketabton.com: The Digital Library

114 3: Pediatrics

established, the initial therapy should be simi- are relatively common immediate conse- lar to that for neonatal sepsis, such as par- quences of head trauma. They are usually tran- enteral ampicillin and cefotaxime. Under ideal sient and are not highly correlated with a risk circumstances, the results of cultures obtained of subsequent posttraumatic seizures. (Rudolph by tympanocentesis may then allow further and Rudolph, 2003, pp. 2260–2261) treatment with a more specific antibiotic of low toxicity. Older infants may respond well to oral 41. (A) Croup and epiglottitis have similar pre- therapy but require frequent observation. sentations but need to be distinguished imme- (Feigin and Cherry, 1998, pp. 908–909) diately. Croup usually results from a viral infection of the larynx and epiglottitis from a 38. (A) Proptosis and limitation of extraocular bacterial (H. influenzae type B) infection of the motility distinguish orbital cellulitis from peri- epiglottis. Children with epiglottitis tend to be orbital cellulitis. Fever, lid swelling, redness of toxic in appearance. Croup involves the airway, the eye, and leukocytosis generally are pres- and epiglottitis involves the airway and the ent in either condition. Orbital cellulitis (infec- digestive tract. Children with croup usually tion within the orbit) may follow directly from will swallow and drink. Children with epiglot- a wound near the orbit or may result from bac- titis most often will refuse to drink and may teremia, but the most common source involves even drool as a result of their refusal to swallow extension from the paranasal sinuses. The saliva. Patients with foreign bodies in their organisms most frequently implicated as upper airways do not typically have fever. pathogens are H. influenzae, S. aureus, group A Patients with vascular rings and laryngeal beta-hemolytic Streptococci, and S. pneumoniae. tumors have more gradual onset of symptoms. The risk of complication is great, with exten- (McMillan et al., 2006, pp. 2721, 694–695, 1508) sion resulting in cavernous sinus thrombosis, meningitis, or brain abscess. Prompt hospital- 42. (D) Iron deficiency is the most common cause ization and parenteral antibiotic therapy are of microcytic anemia. In children it is often indicated. (McMillan et al., 2006, pp. 813–814) related to excessive consumption of cow’s milk, which is low in iron content, and inadequate 39. (A) H. influenzae, S. pneumoniae, and Moraxella consumption of iron-rich foods. Allergy to catarrhalis are the most common bacterial cow’s milk may also cause occult GI blood pathogens in otitis media of children. Amoxicillin losses. In thalassemia major, there is usually is still the initial drug to use in uncomplicated physical evidence of chronic anemia with signs otitis media because of its good coverage, except of bone marrow expansion (frontal bossing) for beta-lactamase-positive organisms, and its and severe anemia often requiring transfusions. excellent safety profile. The other drugs (except Lead poisoning may cause microcytic anemias; for erythromycin) are acceptable second-line it may also be associated with iron-deficiency medications. (McMillan et al., 2006, pp. 14972–1500) anemia, which enhances lead absorption and, therefore, should always be excluded. Anemia 40. (B) Late posttraumatic epilepsy is diagnosed of chronic disease (renal disease) may be micro- when a seizure occurs for the first time more cytic or normocytic and should be excluded by than 1 week after a head injury. Factors that history and examination. (Behrman et al., 2006, correlate with an increased risk of developing pp. 1614–1616) posttraumatic epilepsy include presence of a depressed skull fracture, acute intracranial 43. (E) If iron deficiency is strongly suspected, it is hemorrhage, cerebral contusion, or uncon- reasonable to treat empirically with 3–6 sciousness lasting more than 24 hours. Because mg/kg/day of elemental iron. An increase in the risk of a subsequent seizure is approxi- hemoglobin of 1 g/dL within 2–4 weeks con- mately 75%, acute and chronic treatment with firms the diagnosis. If laboratory confirmation anticonvulsants is indicated. Loss of con- is necessary because the child is at low risk for sciousness, retrograde amnesia, and vomiting iron deficiency, confirmatory iron studies may (c) ketabton.com: The Digital Library

Answers: 38–48 115

be obtained. The serum iron is low, the total 46. (A) The laboratory and radiologic studies indi- iron binding capacity high, and the ferritin is cate a form of central precocious puberty. It is low. A reticulocyte count is helpful in hemolytic most likely to be idiopathic; however, imaging anemias where it is elevated. Bone marrow of the head as well as a careful neurologic and aspirate in iron deficiency is necessary if bone visual examination is recommended to exclude marrow infiltration is suspected (leukemia), but a CNS lesion (tumor, trauma, hamartoma, and is overinvasive in this situation. Hemoglobin so forth). Estradiol secreting ovarian tumors electrophoresis may be done if thalassemia or and functional ovarian cysts may cause periph- sickle cell anemia is likely. (Behrman et al., 2006, eral precocious puberty; however, the levels of pp. 1614–1616) FSH and LH are prepubertal. Congenital adre- nal hyperplasia results in signs of virilization; 44. (B) Neonates are routinely given intramuscular these include excessive hirsutism, deepening vitamin K at the time of birth. This is done to voice, acne, clitoromegaly, and muscle devel- prevent the transient deficiency of vitamin- opment. (Behrman et al., 2006, pp. 1863–1870; Miller K-dependent factors, which occurs because of and Styne, 1999) absence of bacterial intestinal flora which syn- thesize vitamin K. Hemorrhagic disease in the 47. (E) The symptoms are suggestive of perennial newborn because of vitamin K deficiency may allergic rhinitis. Causative agents are usually result in GI, nasal, subgaleal, and intracranial those to which the child is exposed year round, bleeding, or bleeding after circumcision. The such as house dust, mold spores, or pet dan- prothrombin time (PT), partial thromboplas- ders. Seasonal allergic rhinitis is attributable to tin time (PTT), and bleeding time are pro- sensitization to pollens of trees, grasses, and longed. These all correct after administration weeds. Nasal foreign bodies usually result in a of vitamin K. Child abuse should always be foul smelling, unilateral purulent, and occa- considered with unusual bleeding, but the his- sionally blood tinged, discharge. Recurrent tory reveals the etiology in this case. Babies are infections may rarely be attributable to more likely to be allergic to formula than breast immunologic deficiencies. Recurrent pneumo- milk; however, it occurs rarely and may present nias are the most common complaint. Rhinitis with bloody stools. It does not, however, cause medicamentosa occurs secondary to excessive epistaxis. Neonatal sepsis may result in dis- use of vasoconstrictor nose drops or sprays, seminated intravascular coagulation and bleed- resulting in rebound nasal obstruction. ing; the infant is usually ill appearing, with Sinusitis is suggested by a bilateral purulent associated acidosis or shock. Liver disease may nasal discharge, often accompanied by fever, cause factor deficiencies and should be cough, headache, and sometimes sinus tender- excluded if there is no response to vitamin K. ness. (Behrman et al., 2006, pp. 754–760) (Behrman et al., 2006, pp. 606–660) 48. (E) Cyanosis in newborn infants is associated 45. (A) Precocious puberty has been redefined for with major right-to-left shunts. Total anomalous girls as the presence of either pubic hair or pulmonary venous return results in a right-to- breast development before age 6 for African left shunting through an interatrial communi- Americans and age 7 for Caucasians. Premature cation, usually a patent foramen ovale. Patent thelarche is the isolated development of ductus arteriosus and atrial septal defect, when breasts, with no other secondary sexual devel- unaccompanied by other cardiovascular abnor- opment. Premature pubarche is the isolated malities, cause left-to-right shunts that do not development of pubic or axillary hair (sexual produce cyanosis. Coarctation of the aorta does hair). Precocious menarche is a rare form of not typically cause symptoms in the newborn. incomplete precocious puberty with cyclic Hypoplastic left heart presents with signs of menstruation but no other secondary sexual failure in the newborn period. (Rudolph and characteristics. (Kaplowitz and Oberfield, 1999, pp. Rudolph, 2003, pp. 1774, 1800–1833) 936–941; Behrman et al., 2006, pp. 1863–1870) (c) ketabton.com: The Digital Library

116 3: Pediatrics

49. (B) Parasomnias or disorders of arousal It is common in infants, especially when they include nightmares, night terrors, sleepwalk- have been on antibiotics. In bullous impetigo, ing (somnambulism), and sleep talking (som- the skin is initially erythematous and then niloquy). This pattern suggests night terrors bullae develop. Allergic dermatitis and irritant and is most common between ages 2 and 6 dermatitis are the most prominent on the years. Nightmares occur at any age but peak convex areas and are intensely red. In sebor- between ages 3 and 5 years; they occur later in rheic dermatitis, children tend to have the rash the night during rapid eye movement (REM) on the scalp, neck, and face also. It is scaly and sleep. The child usually remembers the dream more prominent in the intertriginous areas. vividly, is upset on waking, but can be com- (Hurwitz, 1993, pp. 34–38) forted by the parent. Sleepwalking occurs, as do night terrors, during non-REM sleep. It is 53. (A) This is a typical presentation of chicken most common between ages 4 and 8 years; pox. A prodrome of fever and malaise is fol- safety of the child is the main concern. Sleep lowed by the rapid eruption of papules that talking is not specific to any stage of sleep and turn to vesicles and crust over. The rash in may occur in association with nightmares and measles, rubella, and Kawasaki disease are night terrors. Narcolepsy is a rare disorder macular or maculopapular. In staphylococcal characterized by excessive daytime sleepiness. scalded skin syndrome, a diffuse, tender ery- Confirmation requires referral to a sleep labo- throderma develops. (Behrman et al., 2006, p. 1058) ratory. (Behrman et al., 2006, p. 78) 54. (E) This is a “round pneumonia,” most com- 50. (A) P. multicoda and S. aureus are organisms monly caused by S. pneumoniae. Onset of this commonly associated with cat bites. The cat’s disease is relatively acute. H. influenzae type B sharp teeth and claws predispose the victim to is an uncommon cause of systemic infections puncture wounds. Wound infections are more because of routine immunization. Mycoplasma common in cat bites than dog bites. E. corrodens, is the most common cause of community- Peptostreptococcus species, and alpha Streptococci acquired pneumonia in this age group. Patients are more common with human bites. B. henselae typically have a more gradual onset of symp- causes cat-scratch disease, which presents with toms. Pneumocystis does not cause pneumonia subacute lymphadenitis. (Kliegman et al., 2004, in otherwise healthy children. S. aureus can pp. 973–975, 866–867) cause pneumonia in healthy children, but it is not as common as S. pneumoniae or Mycoplasma. 51. (A) The case presented is classic of pyloric (McMillan et al., 2006, pp. 1407–1411) stenosis. This results from hypertrophy and hyperplasia of smooth muscle in the stomach, 55. (A) The child most likely has hemolytic-uremic causing a narrowed, even, obstructed outlet. syndrome. This illness is most common in chil- Persistent projectile vomiting causes ongoing dren under 2 years old. They present with a pro- losses of calories and electrolytes, resulting in dromal illness, bloody diarrhea, and then a growth failure and hypochloremic metabolic sudden onset of lethargy and pallor when the alkalosis. Hyponatremia and hypokalemia may hemolytic anemia occurs. Coincident with this is also be associated. Often, the diagnosis can be the development of acute renal failure, often with made by physical examination alone. However, low urine output. E. coli 0157:H7 is the most if an olive-shaped mass is not palpated, an common organism in the United States. Group A abdominal ultrasound may confirm the diag- Streptococci are associated with poststreptococ- nosis. (Behrman et al., 2006, pp. 1229–1231) cal acute glomerulonephritis. Thrombocytopenia and anemia are not seen in this disease. (Rudolph 52. (C) Diaper dermatitis is a very common prob- and Rudolph, 2003, pp. 1586–1587) lem in infants. The infant’s rash is due to Candida. Candida dermatitis is red, without 56. (B) Varicella vaccine is a live vaccine. All others bullae, and has satellite lesions at the margins. are killed vaccines. Diphtheria and tetanus are (c) ketabton.com: The Digital Library

Answers: 49–62 117

both toxoids, the others are from killed the OPV vaccine. Although it is contraindicated microorganisms. (American Academy of Pediatrics, in children or their contacts with immunodefi- 2006, pp. 10, 71–84) ciencies, they are at high risk in case of inad- vertent exposure, because there is feco-oral 57. (D) Progressive, symmetric motor weakness, transmission of the virus for 4–6 weeks after areflexia, and autonomic instability, with mild the vaccine is given. The advantages of the OPV or absent sensory signs, are typical features of include lower cost, fewer “shots,” and boosting Guillain-Barré syndrome. Frequently, there is a herd immunity. The OPV vaccine is still recom- history of infection (often respiratory) in the mended for global polio eradication. (American several weeks preceding clinical onset of the Academy of Pediatrics, 2006, pp. 1–10, 542–547) syndrome. Supportive evidence for the diag- nosis includes elevation of CSF protein con- 60. (C) Aspirin and antihistamines have been centration with a mild (10 or fewer cells/mL) shown to adversely affect platelet aggregation, mononuclear pleocytosis, and slowing of nerve leading to increased bleeding time. Moreover, conduction velocities. In polymyositis, deep this effect may persist for 7–10 days after dis- tendon reflexes would be intact. Myasthenia continuing these medications. When possible, gravis is characterized by weakness aggravated children undergoing surgery should not be by repetitive movement. In transverse myelitis, receiving aspirin or antihistamines. The use of sensation would also be lost. Viral encephalitis antibiotics would not be a contraindication to is characterized by mental status abnormali- elective surgery. Children undergoing elective ties. (McMillan et al., 2006, pp. 2311–2312) surgery should be free of respiratory infection. It is prudent to counsel the parents in ways to 58. (C) The CDC has identified lead poisoning as minimize infection, but avoiding social contacts one of the most common and preventable and shared eating utensils would likely have childhood health problems in the United States. little effect in the case described in the ques- Recent data indicate that undesirable behav- tion. A child should be free of anemia before ioral and cognitive deficits can occur at levels elective surgery, but eating iron-rich foods previously thought to be “safe.” Screening all would not significantly elevate Hb in a short children aged 6–72 months, by questionnaire or period of time. (Champion et al., 1976, p. 335, 1559) blood-lead level, is suggested. Children at greatest risk for lead poisoning include young 61. (C) The most common causes of hypertension inner-city children who live in housing con- in young children are renal in origin. Polycystic structed before 1960; children living near lead kidney disease, congenital vascular anomalies, processing smelters, battery recycling plants, or tumors, and infections all are causes. Urologic other industries that release lead; or children evaluation is imperative for the child described with siblings or playmates diagnosed with lead in the question. Theophylline toxicity that is poisoning. Eliminating the lead source is the severe enough to elevate BP significantly cornerstone of treatment. Chelation therapy would be unlikely in the absence of jitteriness, generally is reserved for those children with nausea, or tachycardia. Chronic lung disease blood-lead levels greater than 45 μg/dL. (Centers would not elevate the systemic BP in an other- for Disease Control and Prevention (CDC), 2005, pp. 51–65; wise healthy child. Coarctation of the aorta is a Committee on Drugs, 1995, pp. 155–160) less common cause of hypertension in this age group. BPs taken on all extremities would be 59. (D) In 1997, the expanded use of IPV in the helpful in the diagnosis. (Rudolph and Rudolph, United States began. Before this, the risk of 2003, pp. 1879–1882) vaccine-associated paralytic polio was 1 case per 2.4 million doses of oral polio vaccine 62. (D) This case most likely represents an adoles- (OPV); the rate after the first dose was 1 per cent with meningitis who has developed 750,000 doses, including vaccine recipient and increased intracranial pressure. Intubation and contact cases. This is the main disadvantage of hyperventilation is indicated immediately. (c) ketabton.com: The Digital Library

118 3: Pediatrics

Hyperventilation is the most appropriate temperature of >40.5°C within 48 hours; col- immediate, nonsurgical treatment of intracra- lapse or shock-like state (hypotonichypore- nial hypertension. By hyperventilating this sponsive episode) within 48 hours; seizures

patient, you will decrease the PCO2, resulting within 3 days; persistent inconsolable crying in vasoconstriction in the CNS. Decreasing the lasting 3 hours within 48 hours. Moderate or

PCO2 5–10 mmHg will decrease intracranial severe illness with or without a fever may be pressure 25–30%. Administering antibiotics, considered a precaution; however, a mild ill- preferably after blood cultures are obtained, is ness, such as an upper respiratory infection, is appropriate. Obtaining a CT scan of the head not a contraindication. Low-grade fevers and may reveal intracranial lesions which require local reactions are common and are not con- additional therapy. Mannitol given intra- traindications; nor are antibiotic therapy or a venously also is a highly effective means for family history of seizures. (American Academy of lowering intracranial pressure. Mannitol does Pediatrics, 2006, pp. 505–511) not cross the blood-brain barrier. It remains in the capillaries and creates an 65. (C) In patients 11–18 years of age with a clean, osmotic gradient, causing fluid to shift from minor wound, Tdap vaccine is required if the intracellular spaces to the vasculature, thereby patient has had <3 tetanus vaccinations or if the decreasing intracranial pressure. Although a immunization status is unknown. In contami- lumbar puncture may be necessary eventually, nated wounds (dirt, feces, soil, and saliva), punc- it is contraindicated as initial management ture wound, avulsions, and wounds resulting because of the possibility of brain stem hernia- from missiles, crushing, burns and frostbite, TIG tion. (McMillan et al., 2006, p. 1085) and Tdap should be given in the 11–18 years age group. Waiting for immunization records is 63. (D) The presence of a congenital membranous not appropriate in an emergency situation. or bony septum between the nose and pharynx (American Academy of Pediatrics, 2006, pp. 650–651) is called choanal atresia. Most newborns are obligatory nose breathers and breathe effec- 66. (D) Children with sickle cell disease develop tively only through their noses. Therefore, if functional asplenia, presumably from repeated choanal obstruction is unilateral, breathing dif- splenic infarction. This results in vulnerability to ficulty may not occur until the first respiratory bacteremia and overwhelming infection, espe- infection. On the other hand, those newborns cially with encapsulated bacteria. The organ- with bilateral atresia who are also obligatory ism most commonly involved is S. pneumoniae. nose breathers will make vigorous attempts to Daily prophylactic oral penicillin is indicated inspire with sucking in of their lips, or may for young children. Because of the risk of bac- promptly become apneic and cyanotic, requir- teremia, these patients need careful medical ing resuscitation. Those who are able to mouth evaluation when they develop fever. There are breathe may have difficulty when feeding or no data to support the use of gamma globulin manifest persistent mouth breathing and in these children. They are not at higher risk cyanosis that is relieved by crying. Treatment for complications from live virus vaccines or consists of surgical correction. (Rudolph and from varicella. (Rudolph and Rudolph, 2003, p. 1561) Rudolph, 2003, p. 202, 1260) 67. (E) Varicella vaccine should not be routinely 64. (D) Absolute contraindications to use of given to children with T-lymphocyte deficiency, DTP/DTaP include history of anaphylactic reac- including lymphoma, leukemia, neoplasms tion to the vaccine or history of encephalopathy. affecting the bone marrow or lymphatic sys- The following are precautions. These circum- tems, and congenital T-cell abnormalities. One stances should be carefully reviewed, and if vac- exception is children with ALL, in continuous cine benefits outweigh the risks, the vaccine remission for at least 1 year, with a lymphocyte should be given. Precautions include any of count >700/μL and a platelet count >100 × the following after a prior dose of DTP/DTaP: 103/μL. Another exception is HIV-infected (c) ketabton.com: The Digital Library

Answers: 63–72 119

children in CDC class 1. These children are at infants, this is often exaggerated, with levels increased risk of morbidity from varicella and >13 mg/dL. This occurs in 10–25% of breast- herpes zoster; therefore, the benefits outweigh fed infants, as opposed to 4–7% of formula- the risks in these limited circumstances. fed infants. It is known as “breast-feeding” Children receiving high doses of steroids jaundice. “Breast-milk” jaundice, develops should not receive the vaccine. This is defined after the first week of life, peaking between as >2 mg/kg/day of prednisone or its equiva- the 2nd and 3rd week to 10–20 mg/dL. The lent, or 20 mg/day, if their weight is >10 kg, for cause of this has not been established. (Kliegman 14 days or more. The vaccine may be given a et al., 2004, pp. 364–366) month after discontinuation of the therapy. (American Academy of Pediatrics, 2006, pp. 723–725) 70. (C) Hypocalcemia is not a cause of constipation. On the contrary, it increases irritability of nerve 68. (C) Mild illness is not a contraindication to vac- cells and may result in diarrhea. Hypothyroidism, cination, nor is breast-feeding. Such live vaccines lead poisoning, and Hirschsprung disease all as MMR and varicella are not recommended for may be associated with constipation. Congenital pregnant women, but may be safely given to hypothyroidism and Hirschsprung disease their children. They are also not recommended in (a congenital disorder characterized by regional many immunocompromised children; however, absence of ganglion cells from the myenteric it is recommended that household contacts plexus of the colon) present at birth. Lead poi- receive the vaccinations to decrease the patient’s soning is more common after the child becomes chances of exposure. It is not necessary to restart mobile. Functional constipation is the most a vaccine series if there has been a long gap common cause of constipation at this age. It is between immunizations. (American Academy of usually due to dietary factors. (Behrman et al., 2006, Pediatrics, 2006, pp. 1–8) pp. 1198–1202)

69. (E) The most common causes of neonatal 71. (A) Otitis externa is an infection of the external cholestasis are extrahepatic biliary atresia and ear canal. Predisposing factors include excessive idiopathic neonatal hepatitis. Infants usually wetness (swimming), dryness (lack of protec- develop icterus by 2–6 weeks of age. They have tive cerumen), trauma (foreign body), and other conjugated hyperbilirubinemia, dark urine, and skin pathology (eczema). It is characterized by acholic stools. At time of presentation, there is pain, accentuated by moving the pinna and usually hepatomegaly, as well as pruritis, especially the tragus; edema and inflammation splenomegaly, and ascites. The use of RhoGAM of the canal; and discharge. Furuncles usually (anti-D gamma globulin) has reduced the inci- cause a localized swelling or papule in the hair- dence of Rh sensitization and resulting jaun- bearing part of the canal. If the tympanic mem- dice. ABO incompatibility can also cause a brane is not visualized, otitis media is hard to milder form of isoimmune hemolytic disease differentiate from otitis externa. Otitis media and jaundice. It is more common in infants with does not cause pain with movement of the blood types A and B born to mothers with blood pinna. In severe otitis externa, the periauricular type O. Anemia is usually present; direct edema may push the pinna forward; this may Coombs test is weakly positive; and indirect be confused with mastoiditis. In mastoiditis, Coombs test is positive. Crigler-Najjar syn- however, the postauricular fold is usually oblit- drome is autosomal recessive; there is usually erated. There is often a history of otitis media marked hyperbilirubinemia (2–40 mg/dL) in and hearing loss and tenderness over the mas- an otherwise asymptomatic infant. The high toid antrum. (Behrman et al., 2006, pp. 2136–2137) levels result in kernicterus. Physiologic jaun- dice is the most common cause of unconjugated 72. (D) P. aeruginosa is the most common agent hyperbilirubinemia; it is characterized by peak involved in external otitis. The other organ- bilirubin level of <13 mg/dL on day of life 3–5, isms listed may also be isolated. (Behrman et al., decrease to normal within 2 weeks. In breast-fed 2006, pp. 2136–2137) (c) ketabton.com: The Digital Library

120 3: Pediatrics

73. (E) The characteristics of fetal alcohol syn- obstruction unrelieved by other maneuvers. drome include: (A) persistent deficient growth (McMillan et al., 2006, pp. 693–700) affecting weight, height, and head circumfer- ence and beginning in utero; (B) such facial 76. (D) Adducted great toe, metatarsus adductus, abnormalities as micrognathia, short palpebral medial tibial torsion, and femoral anteversion fissures, and a thin upper lip; (C) cardiac can result in intoeing. In most cases, this is a abnormalities, commonly septal defects; (D) benign condition that requires only observa- minor limb abnormalities with some restric- tion. In this child, because the child’s knees are tion of mobility and some alteration in palmar straight, the rotational deformity is below this crease patterns; and (E) mental deficiency joint. In metatarsus adductus, the forefoot is ranging from mild to severe. There is a decided adducted as compared to the hindfoot. relationship between the extent of abnormali- Idiopathic avascular juvenile necrosis of the ties and the degree of mental retardation. femoral head, or Legg-Calvé-Perthes disease, Affected infants may present with hypo- most commonly is seen in 4- to 8-year-old boys. glycemia and alcohol withdrawal symptoms Loss of hip medial rotation is an early sign. which may last for 48–72 hours. Immediate (Staheli, 1998, pp. 30–35) management of these infants consists of cor- rection of the hypoglycemia. Ongoing moni- 77. (C) Idiopathic scoliosis is the most common toring of the child’s development is essential. back deformity in children. The incidence Prevention by restriction of alcohol consump- peaks in early adolescence and is much more tion during pregnancy is advised. (Rudolph and common in girls. Screening for scoliosis should Rudolph, 2003, pp. 775–776) be part of every well check and sports physical in children at Tanner (II–V) stages. Congenital 74. (D) Meningococcemia is a fulminant systemic scoliosis is caused by failure of formation or rapidly progressing infection that results in fusion of the ossific nuclei of the vertebrae. It shock and is followed by death in 20% of can present at any age, depending on the afflicted children. The presence of meningitis degree of curvature, and is much less common has been shown to increase the survival rate to than idiopathic scoliosis. Patients with leg approximately 95%. Sensorineural deafness is length inequality present with a limp. Patients the most common residual following bacterial with Scheuermann kyphosis usually present meningitis. Penicillin, ampicillin, or a third-gen- with back pain and have a sharp kyphotic eration cephalosporin would be an appropriate angulation with forward bending. Postural antibiotic to choose for treatment. Vancomycin’s roundback is an exaggerated kyphotic appear- spectrum of activity is limited to gram-positive ance often seen in adolescents. (Rudolph and organisms. (McMillan et al., 2006, pp. 1082–1087) Rudolph, 2003, pp. 2440–2442)

75. (D) In the management of foreign body aspira- 78. (C) Whooping cough, or pertussis, tends to tion, it is generally felt that if the victim can have a prolonged course, with a 2-week pro- speak, breathe, or cough, all interventions are drome of undifferentiated upper respiratory unnecessary and potentially dangerous. When infection followed by approximately 2 weeks of intervention is required, the first maneuver is a a paroxysmal, machine gun-like cough and series of abdominal thrusts (for children >1 year nasopharyngeal mucus that is strangling, thick, of age) or back blows (for children 1 year of age and clear. The typical whoop, a stridorous or younger). When obstruction persists, foreign inspiratory gasp at the end of each paroxysm, bodies sometimes can be removed from the oral is often absent in infants younger than 6 cavity or pharynx if they can be seen, but blind months. Posttussive vomiting more likely will finger sweeps of the hypopharynx are not rec- be found in this age group. The diagnosis ommended. Emergency tracheostomy, prefer- should be suspected in children with a parox- ably performed by an experienced clinician, is ysmal, harsh cough and absolute lymphocyto- employed only in cases of critical airway sis. Specific inquiry into the history of a severe (c) ketabton.com: The Digital Library

Answers: 73–84 121

or long-lasting cough in adult caretakers 82. (C) Infectious mononucleosis may affect chil- should be sought. The diagnosis is made by dren of all ages. The rapid slide (Monospot) test culture of the organism (Bordetella pertussis) or response is positive in approximately 90% of by an immunofluorescent study of throat swab infected persons; however, younger children material. Erythromycin is the drug of choice. It with mononucleosis may have a negative result. has little effect on the illness course after parox- Moreover, many younger children have poor ysms are established and is used primarily to antibody response to the heterophil titer test. limit spread of infection to others. (American The specific serodiagnostic test for EBV, the Academy of Pediatrics, 2006, pp. 498–502) agent responsible for most cases of infectious mononucleosis, confirms the diagnosis. A 79. (D) Turner syndrome (usually 45,XO karyotype) repeat throat culture, even if positive for beta- occurs in 1 of 3000 live births. The hallmark of hemolytic Streptococcus, may be of only partial this genetic disease is gonadal dysgenesis. value, because both infectious mononucleosis Although sexual maturation usually does not and streptococcal pharyngitis may be present occur, a girl with Turner syndrome occasionally simultaneously. Bone marrow examinations will have menstrual periods but rarely will be potentially are painful and contribute little to fertile. Treatment may include estrogen replace- the correct diagnosis. (McMillan et al., 2006, p. 1243) ment or growth hormone usage. Psychosocial support is extremely important. (McMillan et al., 83. (B) Organophosphate poisoning is a leading 2006, pp. 2635–2636) cause of fatal ingestions of nonpharmaceutical compounds. Common components of insecti- 80. (E) The child described in the question seems cides, organophosphates are readily absorbed to be afflicted with infant botulism. Clostridium across skin and mucous membranes. They bind botulinum spores are commonly found in irreversibly to cholinesterase, which results in honey, and the toxin responsible for the symp- prolongation of the effects of acetylcholine, cen- toms described is produced in the infant’s GI trally and peripherally. Symptoms include tract. Therefore, children younger than 1 year muscle fasciculations, paralysis (nicotinic should not be fed honey. Hypernatremic dehy- effect) and miosis, salivation, diarrhea, brady- dration may show some similarities to infant cardia, lacrimation (muscarinic effect) and botulism, but the skin and mucous membranes obtundation, seizures, or apnea (central effect). are characteristically dry. Serum sodium level Acetaminophen ingestion can present with is often greater than 160. Hirschsprung dis- vomiting and then later signs of liver failure if ease would explain constipation but not the it is severe enough. Patients with salicylate other findings listed. Congenital hypothy- overdose present with hypoglycemia, respira- roidism shows a more insidious onset, with tory alkalosis followed by metabolic acidosis, prolonged constipation and weakness. In hypokalemia, and mental status changes. addition, developmental retardation would Tricyclic antidepressants poisoning causes likely be present. (Rudolph and Rudolph, 2003, arrhythmias, mental status changes, and anti- pp. 917–918) cholinergic symptoms. Patients with acute vitamin A toxicity have mental status changes, 81. (B) Cow’s milk allergy may occur in 2–3% of nausea, and vomiting. (Ellenhorn, 1997, pp. 184, infants and toddlers. After elimination from 210–219, 622, 1021, 1615) the diet, by age 3, 85% no longer have symp- toms on food challenge. Older children and 84. (E) Nursemaid’s elbow, or subluxation of the adults may also lose sensitivity to an offending radial head, occurs in children following lon- food when it is eliminated from the diet for 1–2 gitudinal traction on a pronated extended years. The exceptions are IgE-mediated aller- elbow. When attempting to restrain a child, an gies to peanuts, nuts, fish, or shellfish. (Behrman uninformed caretaker may jerk on a child’s et al., 2006, pp. 789–792) upper extremity. The result is a painful subluxed elbow that is easily reduced by simultaneous (c) ketabton.com: The Digital Library

122 3: Pediatrics

flexion and supination of the forearm. (McMillan 87. (B) Hirschprung disease or congenital agan- et al., 2006, p. 2493) glionic megacolon is the most common cause of lower intestinal obstruction in neonates. 85. (E) Nontuberculous lymphadenitis (atypical Incidence is 1/5000 live births; males are four mycobacteria) is characterized by nontender times as likely to be affected. Initial presenta- lymphadenitis. Affected persons are usually tion is with delayed passage of meconium. afebrile, and the CBC is usually normal. Unlike Failure to pass stools leads to dilation of prox- tuberculous lymphadenitis, a history of contact imal bowel and abdominal distention. with a tuberculous individual is lacking, and Diagnosis is by barium enema which reveals a the reaction to 5 tuberculin units of PPD is small-caliber rectum with transition in the rec- almost always less than 10 mm of induration. tosigmoid to a dilated, obstructed proximal Cat-scratch fever is characterized by tender, fluc- colon. Diagnosis is confirmed by biopsy. tuating nodes and low-grade fever. Acute lym- Meconium plugs and meconium ileus are often phadenitis is characterized by tender nodes that found in cystic fibrosis, which should be may fluctuate. The WBC count is often elevated, excluded. Anal stenosis may be diagnosed by and there is frequently a shift to the left on the rectal examination or endoscopy. Functional differential. In addition, the sufferer is often constipation usually presents in children older febrile. Acute lymphoblastic leukemia may than 2 years. There may be some abdominal present as lymphadenitis, but the CBC is usually distention; anal tone is usually normal, and the abnormal, with blasts present on the peripheral rectal ampulla is often full of stool. Congenital smear. (Feigin and Cherry, 1998, pp. 220–230, hypothyroidism may present with constipa- 1085–1086, 1354–1356) tion; neonatal screening tests usually allow early diagnosis. Other features include feed- 86. (D) Vesicoureteral reflux is the most common ing difficulties, prolonged jaundice, sluggish- anatomic abnormality associated with recur- ness, a large abdomen with umbilical hernia, rent UTI in children. Many cases of reflux are subnormal body temperature, myxedema, and the result of an inadequate length of submu- developmental delay. (Behrman et al., 2006, cosal ureter immediately proximal to its open- pp. 2039–2040; Kliegman et al., 2004, pp. 576–578) ing into the bladder lumen, a condition that sometimes requires surgical correction. 88. (A) Children with more severe cases of However, in other children, reflux often seems hypospadias have an increased incidence of to result from the direct effects of infection on concomitant urinary tract anomalies and ureteral tone and peristalsis. Thus, many chil- require careful evaluation. Ultrasonography is dren may outgrow mild degrees of reflux if they a safe and noninvasive procedure that is sensi- are maintained on prophylactic antibiotics. tive in the diagnosis of neonatal urinary tract Moderate-to-severe degrees of reflux frequently pathology. It is a better choice than either IVP or require surgery. Failure of adequate antibiotic cystography which use contrast media and treatment to prevent infection is also a prime radiation. Serum creatinine determinations are indication for surgery. Repeating an IVP or per- a measure of renal function and are unnecessary forming a renal arteriogram on an already diag- in an otherwise healthy child. Circumcision is nosed case would not be useful, although a not indicated in children with hypospadias. In radionuclide scan may be very helpful to fact, it may be contraindicated in cases of determine the present degree of reflux with a second- or third-degree hypospadias, in which minimum of radiation exposure. Vitamin C, the prepuce can be used to construct an absent although reportedly useful in acidifying the distal segment of urethra. (Rudolph and Rudolph, urine to help prevent infection, does not 2003, p. 1739) enhance adequate antibiotic prophylaxis. IV antibiotics would be necessary only if oral 89. (E) Transplacental passage of T. pallidum causes antibiotics were not successful in eradicating widespread disease in the fetus. Organs most infection. (Rudolph and Rudolph, 2003, pp. 1671–1672) severely affected include brain, bone, liver, and (c) ketabton.com: The Digital Library

Answers: 85–96 123

lung. Hepatosplenomegaly, rare in neonates with the first few years of life, before the spleen GBS or HSV infections, occurs in 90% of neonates autoinfarcts. This often occurs following an with congenital syphilis. Mucocutaneous lesions acute febrile illness. Blood pools in the spleen, produce a persistent, purulent, often bloody nasal which becomes enlarged; signs of circulatory discharge which is termed snuffles. This nasal collapse may develop. In an aplastic crisis, a discharge is highly infectious. Skin rash is very low reticulocyte count would be expected, uncommon in CMV and GBS infections. The and there is no such splenic enlargement. acral distribution described is characteristic of (Behrman et al., 2006, pp. 1624–1628) congenital syphilis. Congenital toxoplasmosis characteristically presents with neurologic abnor- 93. (D) Acute sickle dactylitis or hand-foot syn- malities. The classic triad includes hydro- drome is often the earliest clinical syndrome cephalus, chorioretinitis, and diffuse intracranial seen in children with sickle cell anemia. There calcifications. (McMillan et al., 2006, pp. 523–527, 2633) is painful, symmetrical swelling of the hands and feet. Roentgenograms usually reveal bony 90. (C) Roseola infantum, or sixth disease, is a destruction only in the later phase, 1–2 weeks common acute illness of young children. later. (Behrman et al., 2006, pp. 1624–1628) Human herpesvirus 6 is the most common eti- ologic agent. The rash of erythema infectiosum 94. (F) Acute painful episodes are the most presents initially on the face. It is intensely red common manifestation of sickle cell disease. with a “slapped-cheek” appearance. Rubella Most patients experience some pain nearly and measles are not commonly seen because of daily. In younger children, this usually involves routine vaccination. The rash of scarlet fever is the extremities. In older children, head, chest, on the trunk and is described as sandpaper abdomen, and back pain may occur. (Behrman like. (Feigin and Cherry, 1998, pp. 1789–1791) et al., 2006, pp. 1624–1628)

91. (D) Simple febrile seizures are common in chil- 95. (D) Silver nitrate, the traditional prophylactic dren between ages 6 months and 5 years. They treatment for the neonate’s eyes, is very effec- are usually brief, with bilateral clonic or tonic- tive against gonorrheal ophthalmitis. These clonic movement. They have a 30% likelihood of drops are not effective against the most recurrence. The investigation should include a common cause of neonatal conjunctivitis, C. search for the cause of the fever, usually a viral trachomatis, which requires erythromycin or infection, UTI, or following immunization. A tetracycline for eradication. Silver nitrate is lumbar puncture must be performed if there is commonly associated with the sterile purulent suspicion of an intracranial infection and when discharge of chemical conjunctivitis. HSV can features of the seizure suggest a focal or lateral- cause neonatal conjunctivitis, but its occurrence ized seizure. In these situations, EEG and imag- on the first day of life would be unusual. ing may also be considered. However, in most Pseudomonas conjunctivitis generally is hospital children with uncomplicated febrile seizures, acquired, occurring in ill infants receiving these procedures are unnecessary. There is an mechanical ventilation. Nasolacrimal duct increased risk for developing epilepsy in later obstruction is a transient structural anomaly life, as high as 7% in a study with mean follow- that may be accompanied by persistent tearing up of 18 years. When risk factors are present, the and occasional purulent discharge. The condi- incidence of epilepsy rises to 49%. Risk factors tion is usually unilateral and clears within 6–9 include prior neurologic abnormality, prolonged months after birth. Rarely, duct probing or sur- seizures (>30 minutes), focal or lateralized gery is necessary because of persistent stenosis seizure, and repeated seizure within 24 hours. beyond 1 year of age or for repeated infections. (Kliegman et al., 2004, pp. 687–688) (McMillan et al., 2006, p. 813, 578)

92. (B) This is the classic presentation of splenic 96. (A) Nontypeable H. influenzae, S. pneumoniae, and sequestration which occurs in these patients in M. catarrhalis are the most common bacterial (c) ketabton.com: The Digital Library

124 3: Pediatrics

pathogens in otitis media of children. S. aureus, should be done. BP should also be checked. E. coli, and group A Streptococci each account When proteinuria is intermittent and not for 2% or less of all cases of otitis media in chil- accompanied by hematuria, chronic renal dis- dren beyond the neonatal period. Mycoplasma ease is unusual. A child who has proteinuria on is thought to be an uncommon cause of otitis a single specimen, thus, will need repeated uri- media. (McMillan et al., 2006, pp. 1497–1500) nalyses. If isolated proteinuria is present in three consecutive urinalyses testing for ortho- 97. (E) Most cases of idiopathic thrombocytopenic static proteinuria should be considered. If this purpura (ITP) in children are preceded by viral test is negative, qualitative measurement of infections and, in contrast to adults, the great proteinuria can confirm the diagnosis (>4 majority of children recover spontaneously. mg/m2/h) and further evaluation may be con- Although not all patients require therapy, most sidered. If proteinuria remains intermittent, authorities suggest treating when the platelet most physicians do not perform invasive pro- count is less than 20,000. Standard treatment cedures such as a biopsy. Proteinuria alone is has been oral prednisone. Recently, IV gamma unlikely to be an indicator of UTI or structural globulin has been shown to be effective. kidney disease; thus, an IVP and renal ultra- However, this agent is expensive and less con- sound are unlikely to be helpful. Likewise, with venient than oral prednisone. The child isolated and intermittent proteinuria as the described in the question might reasonably be only abnormality, electrolytes, BUN, and crea- treated with either agent. Because the child’s tinine levels are highly unlikely to be abnormal. Hb is 13.3 g/100 mL, there is no indication for (Kliegman et al., 2004, pp. 413–420) transfusion of RBCs. Although the platelet count is very low, platelet transfusions are short 100. (A) Patients with persistent otitis media after lived and generally are indicated only in the 5 days of amoxicillin likely have resistant presence of serious bleeding, as, for example, S. pneumoniae, or a beta-lactamase-positive M. from the GI tract. Thus, platelet transfusions are catarrhalis or H. influenzae. Cephalexin, erythro- not indicated in this patient. Splenectomy is mycin, dicloxacillin, and penicillin would not reserved for the very rare child who does not cover the beta-lactamase-positive organisms. respond to conservative therapy or who Some would recommend that the patient would develops chronic ITP. (Rudolph and Rudolph, 2003, benefit from a higher dose of the amoxicillin pp. 1556–1557; Kliegman et al., 2004, pp. 917–918) component in the combination antibiotic to pro- vide better coverage for resistant S. pneumoniae. 98. (D) Intussusception, or telescoping of the (Behrman et al., 2006, pp. 1950–1959) bowel into a more distal section of bowel, is the most common cause of intestinal obstruction in 101. (A) Constitutional delay is a normal pattern of infants aged 3–12 months. The case presented growth, characterized by a relatively late represents the classic presentation. Giardiasis pubertal growth spurt. It is recognized most presents less acutely and would not be associ- commonly in boys. Patients typically show a ated with a mass. In gastroenteritis, frequent moderate degree of short stature in early to loose stools without blood would be the major middle childhood. The growth pattern is often symptom. Diaphragmatic hernia occurs in similar to one or both parents. Final adult newborns, and the major symptom is respira- height is within the expected genetic potential. tory distress. Although appendicitis can occur The children are otherwise well. Familial short in infants, it is very unusual. (McMillan et al., stature is also a normal growth pattern in a 2006, pp. 1938–1940) short but otherwise normal family. One or both parents are typically 1–2 standard deviations 99. (E) Many healthy children have intermittent below mean height for adults. The growth pat- proteinuria. Transient proteinuria may occur tern parallels the normal growth curve at a per- with fever, strenuous exercise, and cold expo- centile consistent with genetic potential. sure. Testing for hematuria and RBCs and casts Deprivational dwarfism is due to psychosocial (c) ketabton.com: The Digital Library

Answers: 97–106 125

factors. It typically presents at a younger age, patients fail the therapeutic trial. (Kliegman et al., and weight is affected more than the height so 2004, pp. 186–196) that these children are not proportionately small. Hypothyroidism can affect growth but 105. (A) There are many drugs and foods that cause would cause a decrease in growth velocity red urine. These include azo dyes, beets, black- when it occurred. The patient’s height curve berries, ibuprofen, methyldopa, red food color, would flatten out, instead of paralleling the rifampin, phenolphthalein, pyridium, sul- normal curves. Short stature from growth hor- fasalazine, and many others. Dark brown or mone deficiency typically presents by 3 years. black urine can be associated with alanine, cas- (Kliegman et al., 2004, pp. 1096–1097) cara, resocinol, and thymol. If the diet or drug history as well as dipstick are negative, por- 102. (E) Given that this is a classic case of constitu- phyrinuria should be considered. A positive tional growth delay, no diagnostic studies are reagent strip (dipstick) indicates hemoglobin or indicated. Close monitoring of growth would myoglobin. Negative dipstick and normal uri- be indicated. A bone age, if performed, would nalysis makes renal pathology unlikely. be less than chronologic age, demonstrating Because the urinalysis is negative, and the the growth potential for the patient. Cranial patient is asymptomatic, urine culture is not imaging would be indicated if the patient had indicated at this time. Test for antistreptococcal evidence for onset of secondary hypopitu- antibodies should be done if poststreptococcal itarism. (Kliegman et al., 2004, pp. 1096–1097) glomerulonephritis is suspected; it is charac- terized by proteinuria, hematuria, edema, and 103. (B) Chest pain in adolescents is a common hypertension. (Kliegman et al., 2004, pp. 425–429) problem. It is rarely associated with serious ill- ness. In this patient, the onset with exercise, 106. (C) This is a classic presentation for a periton- resolution with rest, and a family history of sillar abscess. Development of a peritonsillar asthma, exercise-induced asthma is the most abscess is usually preceded by acute pharyn- likely cause. Angina is a rare cause of chest gotonsillitis, followed by development of pain in adolescents, and with a normal cardiac severe throat pain and trismus. It is usually examination and no family history of cardiac caused by group A hemolytic Streptococci or disease, this is unlikely. Costochondritis is a oral anaerobes in preadolescent or adolescent common cause of chest pain but typically has patients. Speech is often with a “hot potato” an insidious onset and does not resolve with voice. The affected tonsil is enlarged causing rest. Esophagitis is a common cause of chest the uvula to be pushed to the other side. pain but is typically impacted by eating, not Antibiotics (penicillin) and incision and exercise. Mitral valve prolapse can cause chest drainage are usually required. Retropharyngeal pain, although most pediatric patients with abscess is usually a complication of bacterial mitral valve prolapse are asymptomatic. On pharyngitis in younger children under age 3–4 examination, they often have a systolic click. years. It is caused by infection and further sup- (Kliegman et al., 2004, pp. 148–162) puration of nodes in the retropharyngeal area. Symptoms include high fever, difficulty swal- 104. (E) In a patient with symptoms and signs con- lowing, feeding refusal, hyperextension of the sistent with exercise-induced asthma, a thera- head, and drooling. There is bulging of the pos- peutic trial of inhaled albuterol is the first line terior pharyngeal wall; diagnosis can be con- of therapy and diagnosis. If there is evidence of firmed by widening of the retropharyngeal cardiac disease on history or physical exami- space on x-ray. With lateral pharyngeal abscess nation, then one should proceed with the indi- there is bulging of the lateral pharyngeal wall. cated tests. Pulmonary function tests could be Acute uvulitis is caused by group A hemolytic used to confirm the diagnosis and are used in Streptococci and H. influenzae type B, often in cases in which the diagnosis is uncertain or if association with tonsillitis and uvulitis. (Behrman et al., 2006, p. 1395) (c) ketabton.com: The Digital Library

126 3: Pediatrics

107. (B) Kawasaki disease is an acute febrile illness rash, and nodules. Diagnosis is made accord- of unknown etiology that typically affects ing to the Jones criteria. (Kliegman et al., 2004, young children, usually those under 5 years of pp. 1010–1011) age. There are six clinical criteria used for diag- nosing this disease. The presence of 5 days or 108. (B) Osgood-Schlatter results from microfrac- more of fever, in addition to four of the five tures and inflammation of the tibial tubercle additional criteria, establishes the diagnosis. where the patellar tendon inserts. It is most The five additional criteria are bilateral bulbar commonly seen in young adolescents who are nonexudative conjunctivitis, rash, hand and involved in athletics. Legg-Calvé-Perthes dis- foot changes (edema followed by desquama- ease is idiopathic avascular necrosis of the cap- tion), oral changes such as strawberry tongue ital femoral epiphysis and presents between and erythema, and cervical lymphadenopathy. the ages of 2 and 12 with a painless limp. Erythema infectiosum presents with a pro- Patellar subluxation is usually due to a con- drome of malaise and myalgia and then with genital deficiency within the patellofemoral local erythema of the cheeks (slapped cheeks). joint. On examination, these patients have ten- Rubella and rubeola are unusual because of the derness over the inferior surface of the patella MMR vaccination. Rubeola presents with the and terminal subluxation of the patella when three “Cs”—cough, coryza, and conjunctivitis— the knee is fully extended. Popliteal cysts are followed by the oral inflammation and the usually asymptomatic and present with a fluid- pathognomonic Koplik spots, rash, and fever. filled mass in the popliteal fossa. The symp- Rubella is typically a mild disease character- toms of slipped capital femoral epiphysis are ized by low-grade fever and a maculopapular variable but typically involve hip pain and rash. Rheumatic fever is also unusual. It tends limp. On examination, patients have limitation to present in children over 3 years of age after of motion in the hip. It is most common in an infection with group A Streptococci, with obese adolescents. (Behrman et al., 2006, p. 2272, 2311) transient migratory arthritis, carditis, chorea, (c) ketabton.com: The Digital Library

BIBLIOGRAPHY

American Academy of Pediatrics. Committee on Feigin RD, Cherry JD. Textbook of Pediatric Infectious Injury and Poison Prevention. Selection and using Diseases, 4th ed. Philadelphia, PA: W.B. Saunders, the most appropriate car safety seats for growing 1998. children: guidelines for counseling parents. Hurwitz S. Clinical Pediatric Dermatology: A Textbook of Pediatrics 1996;97(5):761–762. Skin Disorders of Childhood and Adolescence, 2nd ed. American Academy of Pediatrics. Report of the Philadelphia, PA: W.B. Saunders, 1993. Committee on Infectious Diseases, 27th ed. Evanston, Kaplowitz PB, Oberfield SE. Re-examination of the IL: American Academy of Pediatrics, 2006. age limit for defining when puberty is precocious American Academy of Pediatrics. Task force on infant in girls in the United States: implications for eval- sleep position and sudden infant death syndrome. uation and treatment. Pediatrics 1999;104:936–941. Changing concepts of sudden infant death impli- Kliegman RM, Greenbaum L, Lye P. Practical cations for infant sleeping environment syndrome Strategies in Pediatric Diagnosis and Therapy. and sleep position. Pediatrics 2000b; 105(3):650–656. Philadelphia, PA: Elsevier, 2004. Behrman RE, Kliegman RM, Jenson HB. Nelson Mahony CP, ed. Adolescent gynecomastia. Pediatr Textbook of Pediatrics, 17th ed. Philadelphia, PA: Clin North Am 1990;37(6):1389–1404. W.B. Saunders, 2006. McMillan JA, DeAngelis CD, Feigin RD, et al. Oski’s Centers for Disease Control and Prevention (CDC). Pediatrics: Principles and Practice, 4th ed. Preventing Lead Poisoning in Young Children. Atlanta, Philadelphia, PA: JB Lippincott, 2006. GA, October 2005. Miller WL, Styne DM. Female puberty and its disorders. Champion LAA, Schwartz AD, Luddy RE, et al. The In: Yen SSC, Jaffe RB, Barbieri R, eds. Reproductive effects of four commonly used drugs on platelet Endocrinology: Physiology, Pathophysiology, and function. J Pediatr 1976;89:653–656. Clinical Management, 4th ed. Philadelphia, PA: W.B. Committee on Drugs. Treatment guidelines for lead Saunders, 1999. exposure in children. Pediatrics 1995;96:155–160. Rudolph CD, Rudolph AM. Pediatrics, 21st ed. New Ellenhorn MJ. Ellenhorn’s Medical Toxicology: Diagnosis York, NY: McGraw-Hill, 2003. and Treatment of Human Poisoning, 2nd ed. Staheli LT. Fundamentals of Pediatric Orthopedics, 2nd Baltimore, MD: Williams & Wilkins, 1997. ed. Philadelphia, PA: Lippincott Raven, 1998.

127 (c) ketabton.com: The Digital Library

This page intentionally left blank (c) ketabton.com: The Digital Library

CHAPTER 4 Preventive Medicine Stephen K. Liu, MD, MPH

Questions

1. A previously healthy male postal worker com- (A) green monkeys plains of fever, headache, myalgia, and cough (B) mosquitoes of the species Aedes aegypti for the past 3 days. He reports that several of his (C) contaminated vaccine coworkers have also been ill with similar com- (D) polluted water sources plaints. His leukocyte count is normal with a relative lymphopenia. A chest x-ray shows only (E) poorly cooked food enlarged hilar shadows. Which of the following is the most likely cause of this infection? 4. A 20-year-old student asks to have his vacci- nations updated. You recommend that he be (A) Influenza A virus vaccinated for typhoid fever under which of (B) Bacillus anthracis the following circumstances? (C) Francisella tularensis (A) Natural disasters destroy the local water (D) Yersinia pestis and sewage systems. (E) Clostridium botulinum (B) He takes a rural vacation in the south- western United States. 2. As a health officer, you have identified blind- (C) He travels in countries with endemic ness among the elderly in your state as a cause typhoid. of falls and resultant inability to perform activ- (D) He eats organic foods fertilized with ities of daily living. You try to prevent this by raw cow manure. improving access to which of the following? (E) He lives in a community in which carri- (A) residence in nursing homes ers are found. (B) corneal surgery for near vision (C) cataract surgery (D) treatment of glaucoma (E) refractive correction

3. You participate in the global effort to eradicate poliomyelitis. A poliomyelitis outbreak has been identified in a community in Africa. You advise your outbreak investigation team that they must focus on transmission from which one of the following sources?

129

Copyright © 2008 by The McGraw-Hill Companies, Inc. Click here for terms of use. (c) ketabton.com: The Digital Library

130 4: Preventive Medicine

5. A 50-year-old slaughterhouse worker com- (A) annual sigmoidoscopy plains of intermittent episodes of fever, malaise, (B) routine immunization weakness, and weight loss over the past several (C) mammography months. Several of his coworkers have experi- (D) prostate-specific antigen (PSA) testing enced flu-like symptoms since last winter. Which of the following statements about his (E) isolation of disease contacts illness is correct? 8. Food-borne illness has been a recurrent prob- (A) Each Brucella species produces its own lem in your community over the past year. As distinct human illness. a result of this, you ask that the health depart- (B) People who recover from brucellosis are ment’s registered sanitarian pay particular more likely to become reinfected when attention to which of the following during his they are re-exposed to Brucella than they inspection of restaurants? were before their initial infection. (A) unhygienic food-handling methods (C) Person-to-person transmission of brucel- (B) improper storage of rodenticides losis is common. (C) inadequate cooking (D) People with brucellosis generally have a chronic relapsing infection. (D) the use of unlabeled products (E) Humans are the primary reservoir for (E) use of old utensils Brucella. 9. It is 2007. You are planning allocation of health 6. You are asked by a company predominantly department resources to meet the needs of a employing women to design an educational typical U.S. community over the next 10 years. program to reduce morbidity and mortality You decide to allocate funding for each popu- due to cardiovascular disease. Which of the fol- lation group based on anticipated percent of lowing statements should you include in this population growth in each age group. The program to describe women’s risk of cardio- largest proportional increase in funding will vascular disease? go toward which age group? (A) Men have fewer heart attacks than (A) 0- through 10-year olds women. (B) 15- through 34-year olds (B) The underlying cause of heart disease in (C) 35- through 45-year olds women is now well understood. (D) 65- through 74-year olds (C) The gender difference in vascular dis- (E) those 75 years of age and older ease is greater in cerebral, aortic, and peripheral vessels than it is in the coro- 10. It is reported that an alarming number of frac- nary arteries. tures are occurring among the elderly in your (D) Postmenopausal hormone replacement community. You are contacted by a local radio therapy (HRT) is beneficial in reducing station for an interview on this subject. You the risk of cardiovascular disease. explain that which of the following is the most (E) In women, cardiovascular disease is likely cause of the high number of fractures? more likely to present as angina than (A) Alzheimer’s disease in men. (B) osteoporosis (C) obesity 7. The health commissioner asks you to propose (D) Parkinson’s disease a primary prevention program for your com- munity. Which of the following should you (E) deteriorating eyesight recommend? (c) ketabton.com: The Digital Library

Questions: 5–16 131

11. You counsel a 47-year-old smoker who says (D) Suggest that you provide her with a that she does not intend to quit smoking restriction stating that she is not to work because she is not worried about the health at unprotected heights. risks of smoking. All of her relatives smoke, (E) Write a note to the employer requesting and none have developed lung cancer. You reassignment of the patient to a secretar- point out that the Framingham studies have ial position. indicated that disease of other organ systems is also associated with smoking, and her relatives 14. A study finds that the incidence of asthma in have had such disease. The Framingham study your community is higher than expected, and found smoking to be associated with disease of that 40% of the homes in your community are which of the following? heated with forced air. Which of the following (A) skeletal system best describes this study design? (B) spleen (A) ecological study (C) cerebrovascular system (B) cross-sectional study (D) thyroid (C) cohort study (E) auditory system (D) case-control study (E) prospective study 12. Many patients you see in your practice live in homes built prior to 1977. In compliance with 15. A particular community is found to have high the Centers for Disease Control and Prevention rates of dental caries. The health commissioner is (CDC) guidelines, at which of the following attempting to institute a program to prevent fur- ages will you start the relevant routine lead ther dental caries in his community. Which of screening of the children? the following is the most cost-effective method of (A) birth preventing dental caries within a community? (B) 3 months of age (A) community fluoridation of the drinking (C) 12 months of age water (D) entry to preschool (B) promoting dental hygiene (E) entry to first grade (C) instituting a universal sealant program in school children 13. A young woman who works full time doing (D) instituting dental hygienist visits to manual work in a factory, but who also has local schools secretarial skills, is making her first prenatal (E) instituting semiannual topical applica- visit to your office. Her work involves climbing tion of fluoride by a professional high ladders, with the risk of falling. Which of the following is the best recommendation 16. A 1-year-old child in a large nursery school regarding this patient’s job? develops fever, irritability, confusion, a possible (A) Advise the patient not to work. stiff neck, and a petechial rash, over the course of several hours. Which of the following agents (B) Recommend that the patient seek alter- is recommended for others in the nursery native work. school who have had contact with the child, to (C) Recommend no change in employment control a possible outbreak? but suggest avoiding risk. (A) rifampin, ciprofloxacin or ceftriaxone (B) gamma globulin (C) group A meningococcal vaccine (D) group C meningococcal vaccine (E) quadrivalent meningococcal vaccine (c) ketabton.com: The Digital Library

132 4: Preventive Medicine

17. A 16-year old male becomes ill with fever, (A) lack of government resources chills, headache, myalgia, and arthralgia 6 days (B) failure to triage casualties after hunting prairie dogs with his cousin at his (C) failure of management uncle’s farm. Examination reveals extremely (D) lack of telecommunications large, painful, and tender inguinal lymph nodes and a couple of small papules around his (E) lack of volunteer helpers ankles. Which of the following is the most likely diagnosis? 21. Repeated thunderstorms have resulted in over- flow of municipal wastewater treatment plants (A) influenza C by floodwaters. Volunteer sandbaggers down- (B) bubonic plague stream from the plants developed health con- (C) tularemia cerns when the overflow became evident. As a (D) cat-scratch fever public health officer you are asked to evaluate the situation. Which of the following is your (E) West Nile virus most important recommendation to workers in order to prevent illness from exposure to the 18. Schedules for the routine immunization of contaminated water? young children are developed jointly by the Advisory Commission on Immunization (A) Do careful hand washing before meals Practices (a federal commission) and the and cigarette smoke breaks. American Academy of Pediatrics. Which of the (B) Boil all water that is to be used for following vaccines is recommended for rou- drinking or cooking. tine vaccination for all children in the United (C) Offer hepatitis A vaccinations to all States? work crews prior to the next disaster. (A) anthrax vaccine (D) Start prophylaxis with a broad-spectrum (B) rabies vaccine antibiotic for all workers who do not (C) Haemophilus influenzae b (Hib) vaccine have a contraindication. (D) hepatitis A vaccine (E) Offer oral polio vaccine to all eligible workers. (E) typhoid vaccine 22. A father you are treating for hyperlipidemia 19. A health insurance company decides to market brings his 23-month-old son into your clinic its services to a population that will not incur for a routine checkup. He reports that he and high charges. The use of health services in the his wife are separated and that he is uncertain United States is most strongly associated with if the child has received appropriate medical which of the following characteristics? care. The child has no known medical prob- (A) age lems and is not a member of any high-risk pop- (B) sex ulation. The child is new to your clinic, but the (C) race father produces an immunization record which states the child has received the following vac- (D) education cines: diphtheria, tetanus, and acellular per- (E) income tussis at 2, 5, and 7 months; hepatitis B vaccine at birth, 2 months, and 7 months; H. influenzae 20. A hurricane devastates your community. You type b at 2 and 5 months; inactivated poliovirus decide to devote your attention to the aspect of at 2, 5, and 7 months; and a measles, mumps, disaster response that is most likely to be inad- and rubella vaccine at 12 months. You tell the equate in coping with the disaster. Which of father that the child has received some of the rec- the following is most likely to hamper disaster ommended immunizations late, but that the response? child is adequately protected. The infant should (c) ketabton.com: The Digital Library

Questions: 17–27 133

receive varicella immunization in addition to smear at last year’s examination and also the which of the following vaccinations? year prior to that. You recommend that her next Pap smear be performed when? (A) hepatitis A vaccine now, and again in 6 months (A) now (B) pneumococcus vaccine now, and again (B) in 1 year in 6 months (C) in 2 years (C) oral polio vaccine now (D) in 3 years (D) Diptheria, Tetanus, acellular Pertussis (E) in 4 years (DTaP) vaccine now (E) oral typhoid vaccination now Questions 26 and 27

23. A worker complains of paresthesias, numb- A 30-year-old asymptomatic male presents to your ness, and tingling that started distally in the office because his father just had a heart attack. He lower extremities but that is starting to affect is concerned that he may have inherited his father’s his hands. He is developing muscle weakness. condition because a cholesterol level test done at his He feels tired, and has a headache and com- work site last year was 220 mg/dL. You review his plains of memory deficit. Electromyographic history and find that he smokes 25 cigarettes a day, abnormalities suggest axonal degeneration and eats mostly at fast food restaurants, sits at a desk demyelination. Exposure to which of the fol- job, and has no regular moderate intensity physical lowing is the most suspect cause of this clinical activities. His blood pressure is 130/85 mmHg and picture? his body mass index (BMI) is 26.

(A) lead 26. Which of the following is the best first recom- (B) benzene mendation? (C) zinc (A) electrocardiography (ECG) (D) carbon disulfide (B) ECG and an exercise treadmill test (E) vibration (ETT) (C) a diet for weight loss 24. A 20-year-old asymptomatic college student (D) commencement of a daily exercise presents to your clinic for contraception. She routine states that she has been sexually active for 2 years with one partner, and that they usually (E) antihypertensive medication use condoms. Along with a Papanicolaou (Pap) smear and pelvic examination, which of the 27. Which of the following is the most effective following should you also recommend? step to successfully help the patient quit smoking? (A) self-breast examination (A) smoking cessation counseling sessions (B) screening for chlamydia (B) nicotine patches (C) screening for syphilis (C) bupropion hydrochloride (D) thyroid-stimulating hormone (TSH) test (D) physician recommendation to stop (E) screening for gonorrhea smoking 25. You have performed annual examinations on a (E) identifying social supports to help in his young mother for the last 3 years. She and her cessation attempts husband are considering having another child in 5 years, and she would like to restart oral contraceptive pills. You review her medical record and find that she had a normal Pap (c) ketabton.com: The Digital Library

134 4: Preventive Medicine

28. Jehovah’s Witnesses usually refuse to consider Questions 31 through 35 a recommended blood transfusion. A 5-year- old child with hemophilia and dangerously You are examining the relationship between hyper- low hemoglobin levels needs such a transfu- tension and myocardial infarction (MI) in your com- sion, but the Jehovah’s Witness parents refuse munity. In order to do so, you send a questionnaire to accept the recommendation. How are prin- to the whole population in your community (1000 ciples of autonomy best resolved? persons). All 1000 persons responded. The results obtained from that questionnaire are presented (A) The hospital’s representatives respect below in Table 4-1. the parents’ right to consent and sup- port their refusal. TABLE 4-1 ALL 1000 MEMBERS OF POPULATION “A”: (B) The physician explains the need, in sim- RESPONSES TO A QUESTIONNAIRE ple words, to the child who refuses the History of MI transfusion. (C) The hospital requests a local court to History of hypertension Present Absent exercise the rights of the child through Present 15 185 the appointment of a surrogate Absent 5 795 guardian. (D) The local court gives the physician the right to decide if the transfusion is 31. What is the reported prevalence of hyperten- warranted. sion in the population per 1000? (E) The hospital Chaplain is usually given (A) 150 the final authority in these matters. (B) 185 (C) 200 29. Age-adjusted cancer death rates for men and (D) 220 women reveal an increasing rate for which cancer over the last 20 years? (E) 250 (A) prostate cancer 32. What is the prevalence of MI per 1000 hyper- (B) lung cancer in women tensive persons in the total population? Author should (C) breast cancer double- (A) 10 check (D) colorectal cancer answer (B) 15 choices? (E) stomach cancer (C) 19 (D) 75 30. You have decided to survey the population to establish a health risk profile for the popula- (E) 81 tion. Due to budget constraints, you must gather data using a stratified random sample. 33. What is the prevalence of MI per 1000 persons This approach is correctly described as which in the community? of the following? (A) 5 (A) based on selecting individuals from a (B) 10 list at predetermined intervals (every ith (C) 15 individual) (D) 20 (B) random sampling of separate segments (E) 25 of a population (C) randomly grouping the population 34. You randomly select a sample of 100 question- (D) sampling a cluster of individuals naires in order to determine how representative (E) removing outliers and sampling from your results would have been compared to the remainder of group whole population if you had originally only (c) ketabton.com: The Digital Library

Questions: 28–38 135

sampled 100 individuals from your commu- (A) history of cough variant asthma nity. The results obtained are shown below in (B) silicosis Table 4-2. (C) chronic obstructive pulmonary disease (COPD) TABLE 4-2 SAMPLE OF 100 MEMBERS OF POPULATION “A”: (D) bronchitis FINDINGS ON CLINICAL EXAMINATION (E) siderosis History of MI

History of hypertension Present Absent 37. The causative organism of cholera, Vibrio cholerae, was first isolated by Koch in 1883. Present 4 36 Absent 1 59 There have been seven pandemics of cholera, with the most recent subsiding only in the 1980s. With humans as the usual reservoir, the Which of the following statistical tests could organism spreads as man travels. Which of best be used to determine whether there is a the following is the most likely mode of significant increase in the history of MI among spread? those persons who have hypertension in com- (A) contaminated fomites parison with those without hypertension? (B) specific strains of mosquitoes (A) t-test, single-tailed (C) food cleaned in contaminated water (B) t-test, two-tailed (D) person-to-person transmission via inhala- (C) test of variance tion of droplet nuclei in crowded places (D) P value (E) the phlebotomine fly (sand fly) (E) chi-square test 38. A 4-year-old child presents to his pediatrician 35. If we compare the population sample exam- with a complaint of a mild rash and fevers. His ined with the whole population that responded travel history is positive for a camping trip 2 to the questionnaire, which of the following weeks prior. The parents do not recall a tick statements accurately describes the available bite on the child and do not remember if there information? were ticks in the area. No other members of the family complain of similar symptoms and (A) The sample group confirms the findings the child does not attend day care. Physical of the questionnaire. examination is positive for a temperature of (B) As expected, there is a higher “real” 100.9°F with an erythematous rash noted over incidence of hypertension than reported. the child’s trunk. Which of the following factors (C) As expected, there is a higher “real” would significantly increase the possibility that incidence of MI than reported. the child has Lyme disease? (D) There is a statistical test that could be (A) The camping trip occurred in an area applied to assess the significance of the endemic for the tick Ixodes dentatus. differences. (B) The camping trip occurred in Texas. (E) The data as presented are not really ade- (C) The camping trip occurred in an area quate for further statistical examination. with a high indigenous population of lizards. 36. A 45-year-old male living in a homeless shel- (D) The camping trip occurred in an area ter is exposed to TB from a fellow resident. He endemic for the white-footed mouse. is found to have a 15 mm of induration on the Mantoux skin test. Which of the following (E) The camping trip occurred in an area risk factors will increase the man’s likelihood which had recently been sprayed with of developing active TB subsequent to his pesticides. infection? (c) ketabton.com: The Digital Library

136 4: Preventive Medicine

39. For which of the following circumstances 42. Several members of a group of young adults would you consider initiating chelation, such camping, cooking, and traveling together in as with CaEDTA, to treat workers for work- the developing world develop fever, malaise, place exposure to lead and other heavy metals? nausea, and vomiting, and have dark urine. Two have yellow sclera. How could this best (A) as prophylaxis for all employees have been prevented? exposed to metal dust and fumes (B) only after waiting for toxic effects of (A) avoiding eating local foods heavy metal exposure to resolve on their (B) washing hands before eating own (C) taking prophylactic Pepto Bismol (C) only for patients with symptomatic (D) getting vaccinations disease (E) cooking all foods thoroughly and drink- (D) when metal is being absorbed through ing boiled water the gastrointestinal (GI) tract (E) when patients remain in a workplace Questions 43 through 47 where exposure occurs 43. A group of male workers between the ages of Questions 40 and 41 20 and 39 years are being screened for lung disease by spirometry. Nine subjects are exam- Six hundred asymptomatic men with prostatic nod- ined. Their forced expiratory volume in 1

ules are given a PSA test to screen for prostate can- second (FEV1) divided by forced vital capacity cer. With a cutoff of 5 ng/mL, the PSA results are (FEV1/FVC%) results are 80, 76, 73, 61, 64, 79, positive in 200 cases. Of these, 100 persons are con- 64, 64, and 78. What is the mean? firmed on biopsy and follow-up testing to have the (A) 61 disease; however, 50 of the individuals who had negative test results are also shown to have the dis- (B) 64 ease based on biopsy. You have been asked to screen (C) 71 another similar group of 50 men using the PSA test. (D) 73 (E) 76 40. What proportion of persons having prostatic cancers will you correctly identify in the new 44. What is the modal reading? group? (A) 61 (A) 16.7% (B) 64 (B) 22.2% (C) 71 (C) 66.7% (D) 73 (D) 77.8% (E) 76 (E) 87.5% 45. What is the median value? 41. What proportion of persons with no prostatic cancer will you correctly identify in the new (A) 61 group? (B) 64 (C) 71 (A) 16.7% (D) 73 (B) 22.2% (E) 76 (C) 66.7% (D) 77.8% 46. What is the range? (E) 87.5% (A) 17 (B) 18 (c) ketabton.com: The Digital Library

Questions: 39–52 137

(C) 19 49. What is the odds ratio (OR) in your study? (D) 20 (A) 3 (E) 21 (B) 6 (C) 9 47. The variance of the set of values is 58.75. What is the standard deviation (SD)? (D) 12 (E) 20 (A) 5.8 (B) 6.5 50. Using the same table, what is the relative risk? (C) 7.7 (A) 3 (D) 8.5 (B) 6 (E) 9.0 (C) 9 48. The crude death rate in Sweden was 0.010 per (D) 12 year, while in Costa Rica it was 0.008 per year. (E) cannot be calculated All age-specific death rates, except those for the oldest-age category, were higher in Costa 51. Which of the following is most true concerning Rica than in Sweden. From these data, one can ORs and relative risk calculations, as pertaining correctly infer which of the following? to this study? (A) The difference is too small for any (A) The overall size of the series is too large deductions to be made. to estimate relative risk. (B) It is healthier to live in Sweden than in (B) If the number of controls were increased, Costa Rica. the two ratios would be similar. (C) There is less cardiovascular disease in (C) The number of controls is twice as many Costa Rica than in Sweden. as the cases. (D) A greater proportion of the Swedish (D) The OR is not appropriate for this series. population is in the older-age categories. (E) The disease is rare. (E) There is unexplained progressive deteri- oration of health indicators in Costa 52. An investigator in a community hospital Rica relative to those of Sweden. decides to examine all patients for a problem with alcoholism to determine prevalence in the Questions 49 through 51 community. In addition to recall bias, which of the following is the most obvious error in A large segment of the population in your commu- selecting all patients admitted to a community nity smokes. In order to convince your patient popu- hospital? lation that it would be beneficial to quit smoking in order to prevent lung cancer, you conduct a case- (A) observer bias control study of cigarette smoking and lung cancer. (B) selection bias The results from that study can be found in Table 4-3. (C) detection bias (D) interpretive bias

TABLE 4-3 CIGARETTE SMOKING AND LUNG CANCER (E) calculation bias

Lung cancer

Cigarette smoking Cases Controls Totals

Yes 75 25 100 No 25 75 100 Totals 100 100 200 (c) ketabton.com: The Digital Library

138 4: Preventive Medicine

DIRECTIONS (Questions 53 through 109): Each 57. A local health department employs popula- set of matching questions in this section consists tion-based planning in meeting the needs of its of a list of lettered options followed by several community. numbered items. For each question, select the ONE best lettered answer that is most closely asso- 58. A Health Maintenance Organization (HMO) ciated with it. Each lettered answer may be selected employs institutional planning to identify its once, more than once, or not at all. goals and objectives.

59. A maternal and child health care system Questions 53 through 56 employs program-based planning to address Workers in certain occupations are exposed to dis- its goals. eases for which animals are the reservoir. These workers may then become a source of infection to Questions 60 through 64 others. For each of the occupations listed, choose the For each of the diseases listed, select the arthropod infectious disease the workers are most likely to vector responsible for its transmission. acquire and transmit. (A) A. aegypti (A) anthrax (B) Anopheles species (B) brucellosis (C) Pediculus humanus corporis (C) Lyme disease (D) Dermacentor andersoni (D) murine (endemic) typhus (E) Sarcoptes scabiei (E) salmonellosis 60. Epidemic typhus 53. Butcher 61. Malaria 54. Warehouse worker 62. Dengue fever 55. Livestock worker 63. Colorado tick fever 56. Park ranger 64. Yellow fever Questions 57 through 59

The following questions identify health care plan- Questions 65 through 69 ning methods used by various organizations. The For each of the regulatory issues identified, select answer options are strategies that may correspond the corresponding regulatory agency. to these methods. Select the strategy that best corre- sponds to each health care planning method. (A) Food and Drug Administration (FDA) (B) United States Department of Agriculture (A) identifying what the market for services (USDA) are and estimating future demands (C) Environmental Protection Agency (EPA) (B) identifying mechanisms for carrying out established goals within specific (D) United Nations Food and Agriculture program areas Organization (FAO) (C) identifying financial resources to meet (E) World Health Organization (WHO) community needs 65. Which regulatory agency has the authority to (D) identifying barriers to growth and the control the use of pesticides in the United resources needed to overcome them States? (c) ketabton.com: The Digital Library

Questions: 53–83 139

66. A shipment of produce is identified as having 74. Used in transformers because they withstand a higher than allowable level of pesticide. high temperatures Which of the following regulatory agencies retains the authority to remove the produce 75. Used as insecticides and responsible for more from the market? deaths on a worldwide basis than any other group of insecticides 67. Catfish caught in a freshwater stream in Florida has been identified as exceeding the action 76. Highly toxic and dangerous as a solvent; used level for polychlorinated biphenyls in fish in the manufacture of rubber, dyes, and lubri- (PCBs). Which regulatory agency is responsible cating oils for defining these action levels? 77. Used in the production of resins, plasticizers, 68. Which regulatory agency enforces standards and solvents in the sale of meat? 78. Used in the manufacture of textiles and mate- 69. Which of the following institutions is involved rials; often found in manufactured homes in mounting international control programs for the eradication of communicable disease? Questions 79 through 83

Questions 70 through 73 For each of the following scenarios, select the gas exposure responsible for the signs and symptoms. For each organism causing food-related illness, (A) carbon monoxide choose the corresponding average incubation period. (B) methane (A) under 4 hours (C) hydrogen sulfide (B) 8–24 hours (D) ozone (C) 12–36 hours (E) sulfur dioxide (D) 12 hours to 6 days (E) 1–3 weeks 79. A patient working with an electric arc noted a pungent odor, and now has signs of asthma or 70. Clostridium perfringens early pulmonary edema.

71. C. botulinum 80. A sewer worker has acute onset of nausea, headache, and shortness of breath, and has 72. Staphylococcus aureus anosmia.

73. Salmonella 81. A man has been pulled unconscious from a mine. No odors are noted, but an experienced Questions 74 through 78 miner says there was coal damp in the mine.

Several groups of organic compounds are associated 82. A garage worker turns on the ventilation with serious toxic effects when used as described. system in the winter time while testing a motor, For each use, select the potentially toxic compounds and now complains of a headache and vertigo. employed. (A) nitrosamines 83. A worker drilling for oil experiences acute tear- (B) epoxy compounds ing, mucous membrane irritation, and onset of a cough while repairing machinery. (C) PCBs (D) formaldehydes (E) organophosphorus compounds (c) ketabton.com: The Digital Library

140 4: Preventive Medicine

Questions 84 through 88 Questions 94 through 100

For the screening tests listed below, select the For each of the infectious diseases of childhood, screening schedule that is appropriate for women select the appropriate incubation period. (as per the U.S. Preventive Services Task Force (A) 1–6 days [USPSTF], The Guide to Clinical Preventive Services, 2006) (B) 7–8 days (C) 8–10 days (A) do not routinely screen (D) 10–21 days (B) yearly over age 50 (E) 30–50 days (C) at first prenatal visit (F) 120–180 days (D) every 1–2 years at age 40 and older (E) every 1–2 years at age 50 and older 94. Diphtheria (F) every 3 years following an initial exami- nation, but not after age 65 95. Chicken pox (G) every 3 years at age 50 and older 96. Infectious mononucleosis 84. Mammography with or without clinical breast examination 97. Mumps

85. Cervical cytology (Pap smear) 98. Pertussis

86. Fecal occult blood testing (FOBT) 99. Tetanus

87. Screening for hepatitis B 100. Rubella

88. Palpation, ultrasound, or serologic testing of the Questions 101 through 109 abdomen to screen for cancer of the pancreas For each of the conditions listed, select the organism associated with it. Questions 89 through 93 (A) coagulase-positive S. aureus For each of the clinical indications, choose an option (B) beta-hemolytic Streptococcus for use of immune globulin (IG). (C) respiratory syncytial virus (RSV) (A) indicated (D) Mycoplasma pneumoniae (B) not proven effective (E) Haemophilus pertussis (C) not routinely indicated (F) Helicobacter pylori (D) contraindicated (G) Escherichia coli (E) compulsory (H) Rickettsia prowazekii (I) Giardia lamblia 89. Hepatitis A prophylaxis (J) C. perfringens 90. Hepatitis B prophylaxis 101. A male student returns from traveling to a 91. Hepatitis C prophylaxis developing country, with a complaint of eruc- tation, abdominal cramps, and diarrhea for the 92. Measles prophylaxis past 2 weeks, and has lost 10 lbs.

93. Rubella prophylaxis (c) ketabton.com: The Digital Library

Questions: 84–114 141

102. A 4-year-old child in late summer fell 1 week (C) 2 ago, and now has a crusty, mildly erythema- (D) 100 tous wound with regional lymphadenitis. (E) 5000

103. A parent of a 4-year-old child who goes to nurs- 111. How many people would have screened as ery school develops a chronic cough and low- false negative? grade fever. (A) 4995 104. Chronic diarrhea in a homosexual male (B) 98 (C) 2 105. Nursery epidemics of watery diarrhea (D) 100 (E) 5000 106. Furunculosis 112. Multiple disease outcomes associated with 107. A 50-year-old man who reports drinking two or smoking can be assessed with which type of three alcoholic beverages per day complains study? of chronic, vague, intermittent dyspepsia. (A) cross-sectional study 108. A 2-year-old child is pulling on her earlobe and (B) randomized, controlled trial has a temperature of 39°C and a bulging (C) cohort study eardrum. (D) case-control study (E) case series 109. Approximately 12 hours after a banquet, most of those who ate stew report abdominal cramps 113. You are reviewing a cohort (follow-up) study to and diarrhea which subsided after 1–2 days. determine whether dietary fiber reduces the risk of colon cancer. In the study, the population DIRECTIONS (Questions 110 through 134): Each at risk at the beginning of the follow-up period of the numbered items or incomplete statements should consist of which of the following? in this section is followed by answers or by com- pletions of the statement. Select the ONE lettered (A) persons who all have diagnosed disease answer or completion that is BEST in each case. (B) persons with diverse exposure levels and disease Questions 110 and 111 (C) persons of comparable age, gender, and race When making recommendations to a state general (D) persons with homogeneous disease assembly against routine premarital screening, the probability State Health Commissioner used the following data (E) persons who are susceptible but free of to arrive at his conclusions. The state had a young disease adult population of 100,000. Their actual prevalence of human immunodeficiency virus (HIV) infection 114. Among the observational study designs, the was 1 per 1000. The best screening test available had prospective cohort design offers which of the a sensitivity of 98% and a specificity of 95%. following advantages? 110. How many people in this population would (A) It provides a relatively quick answer. have screened false positive? (B) It allows one to take advantage of existing outcome data. (A) 4995 (C) It is easy to assemble a comparison group. (B) 98 (D) It allows one to measure incidence. (E) It is relatively cheap. (c) ketabton.com: The Digital Library

142 4: Preventive Medicine

115. On January 15, 2005, a health survey was per- 118. What is the exposure odds among cases? formed in an elementary school of a developing (A) 0.250 country. All the school children were examined for conjunctivitis, and 2% of the children were (B) 0.500 diagnosed with the disease. From these data, (C) 2.000 one can determine which of the following? (D) 4.000 (A) the incidence of conjunctivitis (E) 8.000 (B) the prevalence of conjunctivitis 119. What is the exposure odds among controls? (C) both incidence and prevalence of conjunctivitis (A) 3.973 (D) neither the incidence nor prevalence of (B) 1.682 conjunctivitis (C) 0.184 (E) the attributable risk (D) 0.053 (E) 0.009 116. A new screening test for prostate cancer becomes available. You assess whether this will 120. What is the OR? be useful to your practice by reviewing the operating characteristics of the test. The test’s (A) 0.053 ability to correctly classify diseased persons as (B) 0.11 having disease is called what? (C) 2.00 (D) 2.201 (A) specificity (E) 4.716 (B) sensitivity (C) positive predictive value 121. A 3-year-old child recovers from a severe (D) negative predictive value episode of bloody diarrhea, hemolysis, and (E) reproducibility uremia. The child’s case is linked to other cases across the country by statistical association with 117. The laboratory test has a sensitivity of 85% and consumption of hamburgers obtained from a a specificity of 70%. You want to have 1000 per- nationwide supplier of ground beef. Which of sons take the test. To estimate the positive pre- the following is the best method for preventing dictive value of the test for this population, this illness in the general population? you need to know which of the following? (A) cooking ground beef to be well done, (A) disease incidence and thoroughly washing fruits and (B) disease prevalence vegetables (C) negative predictive value (B) regulations enforcing worker hygiene in (D) the cut off values for the test the workplace (E) the latest period of the disease (C) a testing program for enteric disease in livestock Questions 118 through 120 (D) regulations enforcing sanitary condi- tions in slaughterhouses Doing a retrospective chart review of unmatched (E) a ban on imported meats and produce cases and controls, you calculate an OR to make an initial assessment of whether women who have had induced abortions are more likely to develop breast 122. A study finds that the relative risk of stomach cancer. Twenty of 100 women with breast cancer cancer after the consumption of a new sugar reported a history of induced abortion. Ten of 200 substitute is 3.5 with a 95% confidence interval women without breast cancer reported a history of of 1.1–5.3. Which of the following best induced abortion. describes the true relative risk? (c) ketabton.com: The Digital Library

Questions: 115–126 143

(A) If repeated samples are taken from the Which of the following is the best next step in population, 95% of the time the relative management? risk will fall between 1.1 and 5.3. (A) Begin three-drug antituberculosis (B) If repeated samples are taken from the therapy. population and confidence intervals are (B) Educate the patient on the symptoms of found for each sample, 95% of the confi- tuberculosis and repeat the chest x-ray dence intervals will include the true in 1 month. population relative risk. (C) Isolate her from her family and other (C) If repeated samples are taken from the close contacts. population and confidence intervals are found for each sample, 95% of the confi- (D) Immunize the patient with bacillus dence intervals will fall somewhere Calmette-Guérin (BCG) vaccine. between 1.1 and 5.3. (E) Begin isoniazid, 300 mg daily. (D) If repeated samples are taken from the population, 95% of the time the true rel- 125. You see a young mother whose child is in need ative risk for the population will fall of medical care. She has an income below the between 1.1 and 5.3. federal poverty level. She is eligible to receive cash payments under the Supplemental (E) Ninety-five percent of individuals from Security Income Program (SSI). As she has no a particular sample will have a relative health insurance, you ask a social worker to risk between 1.1 and 5.3. help her enroll in a program funded with state and federal tax dollars, which provides com- 123. You provide health services for employers in plete medical care for her child. You ask that she your community. The employers explain to you be enrolled in which of the following programs? that they appreciate efforts to contain costs because their insurance premiums are experi- (A) Medicaid ence rated. Which of the following describes (B) Medicare why employers are concerned? (C) Blue Cross/Blue Shield (A) An experienced management team has (D) The Robert Wood Johnson Foundation set new rates for the employers. (E) Women, Infants, and Children program (B) The employers pay the cost of service as (WIC) out of pocket expenses. (C) Everyone in the community pays the 126. A young child finds a bat lying on the floor of same rate for insurance based on com- his room. The child picks it up to show it to his munity charges and overheads. mother, and it bites him on the hand. The bat (D) The employer pays a large deductible then escapes, flying out of an open window. premium. Which of the following diseases poses the most serious threat to the child’s health? (E) An employer’s rates reflect past utiliza- tion by the employer. (A) rabies (B) lacrosse encephalitis 124. You assume care of a 28-year-old nursing (C) distemper home aide, who recently had a positive tuber- (D) tularemia culin skin test (TST). In the past, her tests have always been read as negative; this year, (E) tetanus she developed a 20 × 25 mm induration. She feels well and has no cough. A baseline white blood cell count and liver function test is normal, and a recent HIV antibody test is neg- ative. You order a chest x-ray, which is normal. (c) ketabton.com: The Digital Library

144 4: Preventive Medicine

127. An outbreak of influenza occurs across the (C) Recalculate the rates using 5-year United States in the early winter. Of individuals aggregate data. contracting influenza, a large proportion had (D) Calculate an age-standardized death received vaccinations earlier in the fall. Which rate for each country. of the following is the most likely explanation? (E) Compare the mortality rates for cities of (A) The vaccine was manufactured equal size in each country. improperly. (B) The vaccine used did not contain anti- 130. An HMO has its annual medical directors, gen specific to the outbreak strain. meeting, and new treatments are discussed. A recent study has just clearly demonstrated that (C) A systemic storage problem with a a new drug can lower blood pressure signifi- major shipper damaged the vaccine. cantly, although monitoring tests for renal func- (D) Due to unusually cold weather, people tion are required. The directors consider adding were more susceptible. the treatment to their list of approved treat- (E) The virus was especially virulent. ments but feel handicapped. Which of the fol- lowing is one of the real disadvantages of 128. A family is scheduled to move into a home that making treatment decisions based on clinical is 15 years old. Its water supply is a well, and outcomes alone? sewage is discharged to an on-site septic system. In preparation for the move, they (A) Clinical tests cannot reliably tell which obtain a series of water tests from the well. All treatment works best. of the results show the presence of coliform (B) Clinical trials ignore the difference in bacteria. Which of the following is the impli- costs between treatments. cation of these data? (C) Results of clinical trials cannot possibly be applied to real-world situations. (A) They are at risk of acquiring a coliform bacterial infection. (D) Effects noted in the study population will not show up in the general (B) The well water has been mixed with population. untreated surface or groundwater. (E) The advantages of improvement in clini- (C) Nothing—this is a common finding in cal outcomes are always too significant the country. to ignore. (D) The groundwater is extensively contam- inated and the house is unlivable. 131. The USPSTF made recommendations for (E) They can live in the house but must seek screening mammography and clinical breast medical care at the first sign of illness. examination every 1–2 years for women aged 40 years and older. Which of the following is 129. The mortality rates from two countries are the best explanation for why routine screening being compared. Despite vast differences in mammography was not recommended for wealth, level of public services, birth rates, and women in the general population under 40 education, the difference in crude mortality years of age? rate is the opposite of what is expected: The more developed country has the higher crude (A) Screening mammography in women mortality rate. Which of the following should less than 40 is not as sensitive as in you do to better understand this difference? those over 40. (B) Screening mammography in women (A) Verify a difference in health status of less than 40 is more difficult due to residents by doing a health survey. tissue density. (B) Examine the causes of death to deter- (C) Women under the age of 40 are still mine the reason for the difference. likely to have high estrogen levels. (c) ketabton.com: The Digital Library

Questions: 127–135 145

(D) Breast self-examination in the younger 133. There is public alarm over the possible food- group is more sensitive in detecting borne transmission of bovine spongiform cancers than mammography. encephalitis in your community. You decide to (E) The benefit of detecting cancers in the institute an active surveillance system to gather younger age group was outweighed by information on possible cases. In order to do the risks screening caused in that age this you should do which of the following? group. (A) Collect information by gathering volun- tary data reports from health care 132. In order to allocate health care resources in providers, laboratories, and others. your community, you compare the health (B) Conduct a case-control study of individ- status of subpopulations by comparing infant uals in your community with and with- mortality rates. Which of the following most out the disease. accurately compares the infant mortality rates for children born to White mothers and for chil- (C) Organize the systematic calling of dren born to Black or African-American moth- pathologists and neurologists from sur- ers in 2004 in the United States? rounding areas in an attempt to identify cases. (A) The infant mortality rate for children (D) Monitor disease in animal flocks. born to Black or African-American (E) Collect billing reports to identify from mothers was one-third the infant where the cattle were purchased. mortality rate for children born to White mothers. 134. The National Center for Health Statistics col- (B) The infant mortality rate for children lects information on chronic disease risk factors born to Black or African-American moth- such as obesity, treatment for blood pressure, ers was one-half the infant mortality rate exercise, and alcohol use. These data are best for children born to White mothers. captured using which of the following tools? (C) The infant mortality rate for children born to Black or African-American (A) health survey mothers was between one and two (B) hospital records times the infant mortality rate for (C) registries children born to White mothers. (D) physician-based reports (D) The infant mortality rate for children born to African-American mothers was 135. A study that has been stratified for age finds a between two and three times the infant statistically significant association between mortality rate for children born to alcohol use and socioeconomic status (SES). In White mothers. reviewing the data, the investigators find that (E) The infant mortality rate for children the relationship between alcohol and SES is born to Black or African-American greatest for those in the 40- to 50-year age mothers was six times the mortality rate group. In this scenario, age plays which of the for children born to White mothers. following roles in the relationship between alcohol use and SES? (A) bias (B) confounder (C) effect modifier (D) chance (E) distractor (c) ketabton.com: The Digital Library

Answers and Explanations

1. (A) Onset of influenza usually is abrupt, with 4. (C) Vaccination against typhoid fever (caused fever, chills, fatigue, headache, myalgias, by Salmonella typhi) is less effective than antibi- malaise, anorexia, scratchy throat, and nonpro- otic treatment, with only 65–75% effective- ductive cough. Fever appears early and may ness. Transmission is via fecal-oral route, with reach 40°C (104°F). Myalgias affecting the back humans as the reservoir. Most cases (62%) are and legs, and retrobulbar headache are worse contracted as a result of overseas travel to cer- with high fever. There is a normal leukocyte tain areas of the world where the incidence count with relative lymphopenia. Chest x-ray remains high. It is in such endemic areas that may show enlarged hilar shadows. Fever lasts vaccine is still advised. There has been no a few days. Upper and then lower respiratory indication that immunization after earth- symptoms become more prominent, and cough quakes or other cataclysmic disasters is either may persist for weeks. Respiratory anthrax is necessary or effective. (Wallace and Doebbeling, exceedingly rare. Initial symptoms may resem- 1998, p. 238) ble influenza, but rhinorrhea is rare. X-rays gen- erally show patchy lung infiltrate and 5. (D) Human illnesses caused by the various mediastinal widening (due to enlarged hemor- species of Brucella organisms tend to be quite rhagic lymph nodes). Early treatment with similar. All tend to be chronic, relapsing infec- antibiotics such as ciprofloxacin sometimes pre- tions. Reinfection rarely occurs in recovered vents death. (Centers for Disease Control and patients, with immunity to subsequent Brucella Prevention, 2007; Wallace and Doebbeling, 1998, p. 108) infections in 90% of people. Infection is rarely airborne but can be encountered in laboratory 2. (C) There is a markedly higher frequency of and abattoir (slaughterhouse) workers. Person- impaired vision of the elderly in nursing homes to-person spread is rare. The primary reservoir than in the community. While it is not frequently is in animals. The mode of infection is princi- recorded as a reason for nursing home admis- pally from direct contact with infected animals sion, people with impaired vision experience or by the ingestion of infected unpasteurized many other limitations. Cataract is the leading milk. (Wallace and Doebbeling, 1998, pp. 359–360) cause of blindness in both communities. Maximizing visual acuity by appropriate refrac- 6. (E) Researchers have reported significant dis- tion obviously improves mobility and ability to parities between men and women in heart dis- function but is not effective if the patient has ease. An excess risk is documented in Western cataracts. (Wallace and Doebbeling, 1998, pp. 1031–1033) society through studies such as the Framingham study and studies in Finland. 3. (D) The poliovirus is excreted in stools and There appears to be relative protection from pharyngeal secretions. Transmission occurs estrogens among younger women. However, mainly by the fecal-oral route, particularly the Women’s Health Initiative demonstrated where sanitation and personal hygiene are an increase in risk for heart disease in women poor, as in developing countries. The WHO, using exogenous postmenopausal HRT. with the assistance of Rotary International, is Cardiac disease is more likely to present as progressing toward global eradication. There is angina in women. Older women carry more no known reservoir for poliovirus except cholesterol as high-density lipoprotein (HDL) humans. By 2000, the Americas were polio free. than low-density lipoprotein (LDL) compared (Wallace and Doebbeling, 1998, pp. 123–125) to younger women. The gender difference in

146 (c) ketabton.com: The Digital Library

Answers: 1–13 147

vascular disease is less apparent in the aorta, important for this age group. (Health, United cerebral, and peripheral arteries than in the States, 2006; Wallace and Doebbeling, 1998, p. 1059) coronary arteries. In Eastern Europe, cardio- vascular disease is increasing rapidly in 10. (B) There are many intrinsic factors that lead to women, while in the United States, the age- falls in the elderly. Iatrogenic causes from med- specific increase in cardiovascular disease is ication prescribed commonly impair stability of greater among women than men. (Lang and gait. However, it is osteoporosis, particularly in Hensrud, 2004; Wallace and Doebbeling, 1998, p. 940) the elderly female that results in the excess frac- tures, usually of the femur. A low BMI is asso- 7. (B) Examples of primary prevention include ciated with a higher risk of osteoporosis and routine immunization of individuals at risk for therefore, fractures. Although the elderly are infectious disease, or healthy diet and exercise more likely to have Alzheimer’s and Parkinson’s for persons at risk for diabetes; presympto- diseases, and poor eyesight, these are not as matic and clinical diseases are not present and important as osteoporosis in fractures among are being avoided. Screening, such as by using the elderly. (Wallace and Doebbeling, 1998, p. 1063) Pap smears, colonoscopy, or mammography, typifies secondary prevention; disease is rec- 11. (C) The Framingham study found no associa- ognized earlier than it otherwise would be, tion between smoking and disease of the bones, making improved management of the disease spleen, thyroid, and auditory system. A number possible. Efforts to reduce the consequences of of studies have shown definite increased rate of existing recognized disease, such as isolation of diseases of the cerebrovascular system due to disease contacts, are termed tertiary prevention. smoking. There may also be some increase in (Wallace and Doebbeling, 1998, p. 895) noise-induced hearing loss and some increased risk of fracture of the femur among smokers. 8. (A) The major hazards associated with food- (U.S. Preventive Services Task Force, 2006) borne illnesses are of biological origin. Although all of the phases of the food prepa- 12. (C) Lead-based paint was banned from use in ration process may present opportunities for residential homes in 1977, and children exposed contamination, the major problem is related to to environments built before that time should be food handling rather than to the quality of the screened for lead. The age at which children food itself. Poor personal hygiene and most frequently ingest the largest amounts of improper holding temperatures are the leading lead is during the crawling and walking stage, factors resulting in contamination with bacter- which is also the oral-anal stage of develop- ial and viral pathogens. Commonly reported ment that occurs between the ages of 6 months pathogens include Salmonella, S. aureus, Shigella, and 2 years. Until children are mobile, they are C. perfringens, E. coli, hepatitis A, and Norovirus unlikely to come into contact with objects that (previously described as Norwalk-like virus). might have been coated with lead-based paint. (Centers for Disease Control and Prevention, 2007; Current guidelines recommend screening Wallace and Doebbeling, 1998, p. 726) beginning at 1 year of age in high-risk popula- tions. Lead levels generally peak at about 2 9. (E) Data from the National Center for Health years of age. After 2 years, children normally Statistics indicate that the older age groups will have less tendency to put unusual objects and continue to grow more rapidly than the total soiled fingers in their mouths. (Lane and Kemper, population. The population age 65–74 years 2001, pp. 78–82www.acpm.org) will increase from 6 to 10% of the total popula- tion between 2005 and 2030. The proportion of 13. (B) The situation described in the question calls the population age 75 years and older is grow- for clinical judgment. Restrictions should be ing at the fastest rate and by 2040 this group specific and should pertain to what the patient will exceed the population 65–74 years of age. may actually be asked to do. A company may Disability and quality-of-life issues are very not interdict a woman from performing a job (c) ketabton.com: The Digital Library

148 4: Preventive Medicine

just because she is pregnant. You should offer Semiannual topical application of fluoride by a your best advice to a pregnant woman con- professional, although effective, will be an cerning her health and the health of the fetus. expensive program to institute especially in Advice should be given to patients and not to populations that do not have dental coverage. their employers. The vast majority of women (Wallace and Doebbeling, 1998, pp. 1096–1102) can continue to do their jobs without restrictions. Telling a patient to simply avoid risk is imprac- 16. (A) Rifampin, ciprofloxacin, and ceftriaxone tical, difficult to interpret, and cannot be applied have been recommended for the chemopro- effectively. Advise and offer to write task-spe- phylaxis of close contacts of cases of meningo- cific restrictions, such as, “not to climb ladders coccal meningitis. Gamma globulin has not been and not to lift, push, or pull with more than 20 lbs shown to be effective for the prevention of of force,” or “not to work at unprotected heights meningococcal disease. Most outbreaks are such as on high ladders.” The patient is well caused by strains of groups A, B, C, Y, and W-135. advised to discuss her needs with her employer. Most outbreaks in children less than 1 year old The physician should not communicate directly are caused by group B (for which there is no vac- with the employer unless that is requested by the cine). A vaccine for groups A, C, Y, and W-135 is patient. (Wallace and Doebbeling, 1998, p. 696) available but is used only when the source has been isolated and typed, which is rarely the case 14. (A) Ecological studies compare groups not indi- when prophylaxis must be started immedi- viduals. The unit of observation in the above ately. The immunogenicity of vaccines for study was a community not individuals. A groups A and C is poor, especially in children. cross-sectional study design would have looked Meningococcal vaccine has been recommended at the incidence of asthma in those who also for high-risk groups, including military troops owned the particular home heating system at a and students living in dormitories. The search particular point in time. A cohort study would continues for a widely applicable vaccine. have taken disease-free individuals in the com- (Wallace and Doebbeling, 1998, pp. 206–208; MMWR, munity who also owned the home heating June 30, 2000; Centers for Disease Control and Prevention. system and followed them over time to see who Prevention and control of meningococcal disease. MMWR, developed disease. A case-control approach May 27, 2005. 54(RR07); 1–21) would have taken individuals with and without asthma and identified how many from each 17. (B) Bubonic plague is uncommon in the United group also had the particular home heating States. It is almost always contracted through a system. (Wallace and Doebbeling, 1998, pp. 18–19) bite by an infected flea. A small local papule or vesicle, and sometimes a local ulceration, may 15. (A) Community fluoridation is a very cost- occur at the site of the Y. pestis infecting bite. effective method of preventing dental caries; Mucous membranes and broken skin also can in fact, it is actually a cost-saving practice. be entry sites. After an incubation period of Implementation requires no additional effort on 2–6 days, illness is manifested by fever, chills, the part of the community in terms of health care headache, myalgia, and arthralgia. Eventually, utilization and reimbursement. Promoting there is painful enlargement (the bubo) of dental hygiene in and of itself has not been lymph nodes draining the inoculation site. found to be effective in preventing dental caries. (Wallace and Doebbeling, 1998, p. 312) Therefore, regular visits to dental hygienists or having dental hygienists visit schools will not 18. (C) Hib vaccine is recommended for routine by itself prevent dental caries. Sealants are very immunization of children. Introduction of the effective in protecting against dental carries in Hib conjugate vaccine in the late 1980s was fol- populations which have deep pits and grooves lowed by a spectacular decrease in the inci- on the dental surfaces. It is not as effective on dence of H. influenzae meningitis. Anthrax smooth dental surfaces. Therefore, it is cost- vaccine is used to vaccinate military troops in effective but only in a subset of the population. selected overseas deployment. Rabies vaccine (c) ketabton.com: The Digital Library

Answers: 14–28 149

is used for postexposure prophylaxis when 23. (D) Carbon disulfide causes sensorimotor children are bitten by potentially rabid mam- peripheral neuropathy. Central nervous system mals. Hepatitis A vaccine is used for children at (CNS) effects and optic neuritis are also special risk of such infection. Typhoid vaccine is common. Long-term exposure may increase not routinely recommended for use in the United cardiovascular risk. Lower sperm counts and States but may be indicated for travelers to areas more abnormal spermatozoa are observed than in the developing world where typhoid fever is in unexposed subjects. (Wallace and Doebbeling, endemic. (Wallace and Doebbeling, 1998, p. 117) 1998, p. 535)

19. (A) In 1990, there were 3 million persons in the 24. (B) The USPSTF strongly recommends that cli- United States aged 65 or more. In 2001, there nicians routinely screen all sexually active were 35 million, and by 2030 there may be 71 women aged 25 years and younger, and other million. For the population as a whole, usage of asymptomatic women at increased risk for infec- health care is high at the time of birth and, to an tion, for chlamydial infection. (U.S. Preventive even greater degree, in the period prior to Services Task Force, 2006) death. (Health, United States, 2006, p. 109; Wallace and Doebbeling, 1998, p. 1066) 25. (D) After two normal Pap smears, women with normal risk can be screened every 3 years. The 20. (C) Since the 1950s, the single most important USPSTF has found no evidence that annual failure in coping with disasters has been poor screening provides better outcomes than management by those in charge. There has been screening every 3 years. The majority of cervi- no significant problem with resources needed or cal cancers in the United States occur in women with triage of casualties. Neither has there been who have never been screened, those who did evidence of poor response to the need for volun- not receive appropriate follow-up after an teer helpers. Telecommunications have improved abnormal Pap smear, or those who have not continuously, especially with satellite commu- been screened within the past 5 years. (U.S. nications available. (Wallace and Doebbeling, 1998, Preventive Services Task Force, 2006) pp. 1170–1173) 26. (D) The USPSTF does not recommend ECG or 21. (A) Public health sanitation measures in emer- ETT in asymptomatic patients. A BMI greater gency conditions should reduce fecal contamina- than 27 is associated with increased mortality. tion of food and water supplies. Communicable The U.S. Surgeon General recommends a pro- disease such as typhoid fever, cholera, bacillary gram of moderate exercise most days of the and amoebic dysentery, hepatitis, polio, schisto- week. (U.S. Preventive Services Task Force, 2006) somiasis, various helminthes infestations, and viral gastroenteritis can be transmitted through 27. (C) Whereas all of the interventions have a pos- contact with human feces. Avoiding putting itive effect on smoking cessation, the bupropion fecal material in the mouth is key to avoiding hydrochloride has the greatest effect at 6-month infection. (Wallace and Doebbeling, 1998, p. 1171) follow-up. (Hunt et al., 1997, pp. 1195–1202; U.S. Preventive Services Task Force, 2006) 22. (D) The fourth DTaP must be at least 6 months later than the third dose. Hepatitis A vaccine is 28. (C) The law has generally recognized that the recommended in some high-risk communities, parents do not necessarily exercise autonomy but it is not recommended for all children. appropriately on behalf of their child. It also Pneumococcus vaccine for children not previ- recognizes that children cannot exercise auton- ously immunized by 23 months should receive omy for themselves, and that children should two doses 8 weeks apart. Oral polio vaccine not be required to make complicated decisions should not be used when inactivated poliovirus about their own care. For children, and for is available. (Centers for Disease Control and Prevention, adults who are unable to make medical deci- 2007) sions, a surrogate decision maker is normally (c) ketabton.com: The Digital Library

150 4: Preventive Medicine

named by a court of appropriate jurisdiction. 33. (D) In the study described in the question, the (Wallace and Doebbeling, 1998, pp. 35–36) prevalence rate is the total number of cases occurring in the community, divided by the 29. (B) The age-adjusted lung cancer mortality rate number of members in the community, multi- rose rapidly following adoption of smoking by plied by 1000. This is a point prevalence rate: women. This now has stabilized as the leading the number of individuals who have had an MI cause of cancer death in women. (National Center at the time the questionnaire is administered. for Health Statistics, Centers for Disease Control and Thus, the prevalence is (20/1000) × 1000 = 20. Prevention, Health, United States, 2006) (Gordis, 2004, p. 33)

30. (B) Sometimes it is possible to identify sub- 34. (E) The chi-square test is the most appropriate sta- groups or strata of a population. Randomly tistical test to determine whether series of fre- sampling these segments, called strata, may quencies or proportions are significantly different reduce sampling error. Systematic sampling, from each other. It is designed to describe with a selecting every “ith” individual, is not neces- single number how much the frequencies in each sarily random, depending on how lists are con- cell of a box of paired readings differ from the fre- structed. Random groups would presumably quency we would expect if there were no rela- be based on random selection of individuals, tionship between the observed readings. If the and little benefit would derive from studying observed readings are similar to expected read- such groups. Random groups produce more ings, the chi-square will be a small number. If hazards statistically than randomized strata. there is a greater difference, the chi-square will be Clusters are somewhat different; they may be larger. The mathematical equation is: small groups of the population occurring in specific areas, such as families, villages, or ()0 − E 2 c 2 = ∑ wards. The characteristics of clusters are not E necessarily those of the population, but more those of location. Cluster sampling may be (Feinstein, 2002, p. 246; Wallace and Doebbeling, 1998, useful but does not have the same outcome as p. 25) a stratified sample. (Gordis, 2004, pp. 210–211; Greenberg et al., 2001, p. 97) 35. (E) In a questionnaire relating to history, patients’ knowledge as to whether or not they 31. (C) Prevalence is the number of existing cases have hypertension or have had an MI might be of a disease occurring in the total population at sufficiently accurate to allow further statistical a given period of time. In this study, the preva- analysis. The data provided in this question- lence rate is calculated based on reported fig- naire are not adequate for more detailed sta- ures (15 + 185 = 200 persons per 1000 persons tistical analysis. The questionnaire responses surveyed report having hypertension). The (Table 4-1) rely on memory recall, which at actual figure, as demonstrated by the investi- best is questionable, for comparison with an gation of the sample, is likely to be higher. actual examination (Table 4-2). Criteria for (Greenberg et al., 2001, pp. 15–27) establishing a diagnosis of MI and, if possible, actual blood pressure readings, as well as a 32. (D) In the tables that accompany the question, definition of hypertension, are required. The the number of hypertensive persons in the crux of epidemiologic analysis is a detailed community is 200. The number of patients criterion for establishing a diagnosis. With this with hypertension who also report a history of additional information, relevant statistical MI is 15. The prevalence is therefore 15 in a tests could be applied. In the absence of this population of 200, which may be translated as information, any further statistical analysis is a rate of 75 per 1000. (Greenberg et al., 2001, likely to lead to misleading results. (Feinstein, pp. 15–27) 2002, pp. 2–3) (c) ketabton.com: The Digital Library

Answers: 29–41 151

36. (B) Silicosis is considered a risk factor for the humans. Texas is not one of the endemic areas development of active TB disease for those who for Lyme disease. Lizards are actually consid- have contracted the infection. HIV infection, ered to be a zooprophylactic host because they malnutrition, IV drug use, alcoholism, leukemia are an incompetent reservoir of B. burgdorferi, and other cancers, poorly controlled diabetes, and they are also a preferential feeding source severe kidney disease, immune system sup- for immature stages of the tick. Pesticide spray- pression caused by medications such as drugs ing is actually protective against tick infestation to prevent rejection of a transplanted organ, and therefore will reduce the chances of tick and living in crowded indoor conditions, are exposure. (Wallace and Doebbeling, 1998, pp. 327–332) also considered risk factors for progression to active disease. Cough variant asthma and bron- 39. (C) Chelating drugs are given as treatment for chitis are not included in this list. COPD can symptomatic poisoning by lead and other occur as a consequence of chronic TB disease heavy metals. They should not be given pro- and is not considered one of the risk factors. phylactically, since the agents themselves have Siderosis results from accumulation of nonfi- some possible toxic side effects. These toxic brogenic iron oxide particles in the lung and is effects may add to those already caused by not considered to predispose to TB. (Wallace and ingestion of the metals and may actually Doebbeling, 1998, p. 209) increase absorption of the metal. For these rea- sons, advice should be given to workers to seek 37. (C) Humans are the usual reservoir of V. employment away from exposure to the cholerae. It tolerates exposure and drying poorly. offending agent while therapy continues. It survives longest in water, especially if the Removal from exposure is essential if levels water is at temperatures of 18–23°C (60–70°F). are high. (Rom, 1998, pp. 989–990) It does not spread on infected clothing (fomites). Direct person-to-person transmission probably does not occur. Contaminated water is the main Disease Disease source of infection (e.g., frequent exposure to present absent Totals polluted surface water through bathing, food Test positive 100(TP) 100(FP) 200 preparation, and utensil washing). Although Test negative 50(FN) 350(TN) 400 flies may transport small numbers of vibrios Totals 150 450 600 from excreta to food, lack of multiplication makes it unlikely that flies play an important part in transmission. Mosquitoes are not vec- tors. (Wallace and Doebbeling, 1998, pp. 240–243) 40. (C) Several concepts are important in deter- mining the sensitivity of tests. The percentage 38. (D) The principal risk factor for Lyme disease sensitivity is the percentage of individuals with in the United States is residence in an area with the disease (true positives [TP]) detected by high infestation rates of infected ticks. In the the tests. False negatives (FN) are those who coastal northeastern United States, the white- have the disease, but who were not detected by footed mouse is the most competent vertebrate the test. The percentage sensitivity is calculated reservoir of B. burgdorferi, the organism respon- as: TP/(TP + FN) × 100. In the example given in sible for Lyme disease, with infection rates of the question, the calculation is 100/(100 + 50) × up to 80%. Therefore, residence in an area 100 = 66.7%. (Greenberg et al., 2001, pp. 78–79) where there is a large population of white- footed mice will also increase the likelihood of 41. (D) The percentage specificity of a test is the exposure to Lyme disease. The tick species I. percentage of those persons without the disease dentatus has maintained cryptic cycles of B. (true negatives [TN]) who were correctly burgdorferi, but it is not considered to be a labeled by the test as not diseased. The false public health risk because it rarely feeds on positives (FP) are those who were incorrectly (c) ketabton.com: The Digital Library

152 4: Preventive Medicine

labeled by the test as having the disease. The contains an odd number of readings. Although specificity is thus expressed as TN/(TN + FP) not very frequently used as a statistic, the × 100. In the example given in the question, median has the advantage of not being affected the calculation is 350/(350 + 100) × 100 = 77.8%. by extreme observations. For example, if the (Greenberg et al., 2001, pp. 78–79) lowest reading of those in the question had been 55 instead of 61, the median would be 42. (D) Hepatitis A was first isolated in 1973. Since unchanged. (Feinstein, 2002, pp. 31–34) then, it has been demonstrated to be conveyed from person to person chiefly by the fecal-oral 46. (C) The range is the difference between the route. Humans appear to be the only natural highest and lowest readings in a group of host. Outbreaks attributed to food and water observations. The lowest reading is subtracted supplies are frequently reported. There were from the highest. Only one of these two read- an estimated 61,000 new infections in the ings is included in the range. If both were to be United States (particularly in the West) in 2003, included, the range would be 20, as there are 20 continuing a trend downward. It does not cause possible values included in the limits of the chronic infection. Vaccination is effective and data presented in the question. For simplicity, available for persons at risk, including travelers. the first method is generally accepted. One Examples of where outbreaks occur include: disadvantage of the concept of range is that it among travelers, among young adults clustered may increase as the number of observations together, and in day care environments. (Centers increases. (Feinstein, 2002, p. 60) for Disease Control and Prevention, 2007; Wallace and Doebbeling, 1998, pp. 175–178) 47. (C) The variance is the sum of the squares of the difference of each reading from the mean. This 43. (C) The mean, or average, is the total of all is a mathematical way of eliminating the readings divided by the number of readings, or opposing effects of negative and positive (∑x)/n. In the example described in the ques- values. But the variance has a result that is of a tion, the total of all readings equals 639; the different order than the original observations. number of readings is 9. Therefore, the mean is The variance of 58.75 seems to have little rela- 639 divided by 9, or 71. (Feinstein, 2002, pp. 31–34) tionship to a series of readings from 61 to 80. In order to determine a value that is the same 44. (B) The modal reading is the most frequently order as the observations, the SD is calculated occurring observation in a group of data. (On as the square root of the variance. This is math- a graph of frequency of different observations, ematically represented as: Au: this would be where the peak is.) This reading (Needs a generally is not very useful for statistical cal- bar over Vx()⊇=− Vx ()Σ ( x x )2 one of the culations, and when the series is small, no SD(x ) = x’s) modal reading may even occur. When drawn in n − 1 graph form, the determination of unimodal or bimodal distribution may be a helpful con- Without using a calculator, it can be seen that cept from which conclusions may be drawn. the nearest correct answer (correct to 1 deci- The distribution in the example that accom- mal place) is 7.7. (LaDou, 2003, pp. 819, 822–825) panies the question is unimodal. (Feinstein, 2002, pp. 31–34) 48. (D) In a crude death rate, all deaths are in the numerator, and the total midyear population in 45. (D) The median is the observation that lies in the denominator. In age-specific death rates, the middle of the series, if the observations are the calculation is done using data from specific tabulated in numerical order. Half of the obser- age intervals. Small differences in crude death vations are lower in numerical value than the rates may enable specific deductions to be median, and the remainder are higher. Clearly, made. A higher death rate in the older popula- this value is easily identified if the series tion may indicate that a more fragile population (c) ketabton.com: The Digital Library

Answers: 42–56 153

has successfully survived to that age. Of the Explanations 53 through 56 statements listed, considering that age-specific death rates were greater in all age groups Note that each illness listed may be contracted except the elderly, only the fact that a greater by a person working with animals, animal proportion of the population in Sweden is in products, and the associated rodents. The fol- the older age groups could account for the dif- lowing answers refer to the most likely infections: ference in crude mortality rate. (Wallace and 53. (A) Doebbeling, 1998, p. 47) Cutaneous anthrax is associated with a characteristic skin lesion, which becomes 49. (C) The OR is used when the denominator data infected by the introduction of the bacillus are missing. It is calculated using the following through the skin. Occasionally, an infected car- formula, comparing smokers and nonsmokers: cass is not identified before butchering. A minor wound on the butcher may become infected. Human anthrax is secondary to the ad× disease in animals, primarily mammals. In × bc humans, the disease has three major clinical forms: cutaneous, inhalational, and GI. (Wallace In this case, the mathematics is relatively and Doebbeling, 1998, p. 357) simple: 75 × 75/25 × 25. By reduction, this is × × 3 3/1 1. (Greenberg et al., 2001, p. 134) 54. (D) Rodents frequently infest granaries, despite strenuous efforts to control them. Murine 50. (E) A true risk ratio cannot be calculated from (endemic) typhus has its principal reservoir in a case-control study. (Gordis, 2004, pp. 135–136, rodents and is conveyed to humans by flea 178–179) bites. In the United States, murine or endemic typhus occurs mainly in the Southeastern and 51. (B) For ease of calculation, to illustrate the con- Gulf states. It is also common in parts of South cept, the numbers in the question were kept America, Africa, and Southeast Asia. The dis- simple. If the numbers of controls were ease generally follows a less severe course than increased, the difference would be smaller. The epidemic typhus. (Wallace and Doebbeling, 1998, ORs are a reasonable estimate when denomi- p. 306) nator data are missing. Relative risk is deter- mined when denominator data are available. 55. (B) Brucella infections in humans follow a Few case-control experiments have denomi- varied and sometimes chronic or recurrent nator data. (Gordis, 2004, pp. 184–185) course. Chronic disease is rare in appropriately treated patients. Human infections generally 52. (B) Bias is a systematic error that may be intro- occur through one of three routes: ingestion, duced, generally unwittingly, into an investi- direct contact, or inhalation. Cattle sheds gation. Selection bias is due to systemic become infected after abortion, a manifestation differences between those selected and not of the disease, or occasionally after normal par- selected for a study. When selecting all admis- turition. Worldwide, ingestion of unpasteur- sions to a community hospital, alcoholics may ized dairy products is the primary source of be overrepresented among hospital patients infection. (Wallace and Doebbeling, 1998, p. 359) and not reflect the community base from which the patients were admitted. Further, the inves- 56. (C) Lyme disease was first comprehensively tigator has not chosen a control sample but described in Connecticut in 1977. It is found in only patients already in the hospital, which temperate North America and Eurasia. It may further invalidates any outcome. While other be identified some time after the initial illness, biases may creep into this study, the selection which may be a vague collection of symptoms. bias is the major problem with the study. The vector is a tick found on the white-footed (Greenberg et al., 2001, pp. 144–151) mouse, which is the reservoir. Persons who (c) ketabton.com: The Digital Library

154 4: Preventive Medicine

spend a significant amount of time in affected 63. (D) wooded areas are at risk for Lyme disease. (Wallace and Doebbeling, 1998, p. 327) 64. (A)

57. (A) Local health departments make plans for Explanations 60 through 64 the entire population in the jurisdiction for which they have responsibility, whether it is a Epidemic typhus (classical typhus fever, or single county, city, or multicounty health louse-borne typhus) has disappeared from department. Their planning is population most areas of the world but might reappear in based. This involves estimating health require- conditions of famine, war, or other disasters. ments, matching them with existing resources, There are small areas where it is endemic. The and outlining a health strategy based on the responsible organism, a rickettsia, is conveyed from case to case by the human body louse, P. deficit or surplus demonstrated. (Wallace and Doebbeling, 1998, pp. 1155–1158) humanus corporis. Malaria, in its various forms (Plasmodium falciparum, Plasmodium vivax, 58. (B) A distinction is frequently made between Plasmodium ovale, and Plasmodium malariae), is what the patient wants (which would be used spread from human to human by females of the if available and money was no object), needs various Anopheles group of mosquitoes. (services determined by professionals to be Dengue fever has a worldwide distribution in appropriate), and demands (services that are tropical and subtropical areas. In addition to actually used in the current market situation). producing the classical fever with severe myal- Planning for an HMO is based on that segment gia (breakbone fever), it can also cause a hem- of the population or “market” for which the orrhagic fever. The causative agent, a group B HMO is responsible. The population may or arbovirus with four distinct serogroups, is may not live in contiguous areas. The planning virus-conveyed from case to case by the A. is designed to identify goals and objectives in aegypti mosquito. Colorado tick fever occurs institutional terms: What is the market for the mainly in mountainous areas of the United services the organization provides, and what is States within the range of its vector, D. andersoni. the estimate of future demands? The population The highest incidence is in May and June. need (as opposed to the population demand) is Several hundred cases are recorded annually, rarely a concern of institutional planners. but it is likely that the actual incidence is much (Wallace and Doebbeling, 1998, pp. 1155–1158) higher. Avoidance of tick bites is the principal control measure. Yellow fever, the prototypical 59. (D) Program planning concerns itself with nei- viral hemorrhagic fever, is African in origin but ther the fiscal need for a particular service nor its has spread to and remains endemic in equato- marketability. By definition, “the program” (e.g., rial regions of Central and South America. The maternal and child health care) will be devel- vector, A. aegypti, has also spread worldwide, oped and provided as directed by the state or but surprisingly, cases have not been reported local government. The planning is directed to in India and Southeast Asia. The illness varies carry out program goals for a targeted (select) in severity from a mild, nonspecific fever to a population (pregnant women and their infants). more severe condition with hemorrhagic, This type of planning is necessary to implement hepatic, and renal manifestations. (Wallace and government or private foundation-sponsored Doebbeling, 1998, p. 294, 295, 300, 305) programs. (Wallace and Doebbeling, 1998, pp. 1155–1158) 65. (C) 60. (C) 66. (A) 61. (B) 67. (C) 62. (A) (c) ketabton.com: The Digital Library

Answers: 57–75 155

68. (B) help in establishing C. botulinum as the cause of illness, since certain strains of S. aureus cause 69. (E) food-borne disease by the production of entero- toxin. As no time is required after ingestion for Explanations 65 through 69 the growth of colonies in the infected host, and the toxin affects the vagus nerve in the stom- In the United States, the organization of food ach, the incubation period is under 4 hours. C. and water control is complex. Among the fed- perfringens, formerly known as Clostridium eral control agencies, the EPA is the most welchii, causes food-borne disease, after 8–24 recent, and in many ways the most active and hours when enterotoxin is released when C. powerful. This agency has now set up an elab- perfringens passes from stomach to intestine. orate system of regulation and control of the Meat prepared in bulk for consumption at a use of pesticides (which until 1970 was the banquet or in an institution is a possible source. responsibility of the USDA) and has banned Spores that survive incomplete cooking may the marketing of chlorphenothane (DDT) for start reproducing during cooling and may per- use in the home. The FDA has authority to sist if subsequent rewarming is not completed remove food from the market if it contains pes- to a temperature above 60°C (140°F) required to ticides (e.g., PCBs in fish) in excess of the action kill the organisms. Salmonella has an incuba- levels set by the EPA. The FDA also retains the tion time of 12–36 hours. It may also survive in authority to remove from the market any food meat and other products if cooking is inade- with inappropriate additives, that contains sub- quate and heat does not penetrate below the stances harmful to human health, that is stored surface of the food. The organisms multiply in in unsanitary conditions, that has decom- the gut of the infected host, and low infective posed, or that is not fit for consumption. The doses may therefore have longer incubation USDA enforces wholesomeness standards that periods. (Wallace and Doebbeling, 1998, p. 724) it sets for the production and sale of meat. International control is assisted by the WHO. 74. (C) PCBs were extensively used in the manu- This agency has mounted control programs for facture of electrical transformers until produc- the eradication of communicable disease with tion was halted in the mid-1970s. The first sign conspicuous success in the case of smallpox. It of chronic exposure to these substances is the also publishes the International Statistical appearance of an acne-like eruption with Classification of Disease (ICD-9). (Department of inflammatory pustules. Other effects are eye Health and Human Services Organizational Chart, 2007; irritation and GI disturbance. The substances LaDou, 2003, p. 557) are persistent, and more than 25% of the pop- ulation in the United States was discovered to 70. (B) have residues of greater than one part per mil- lion (ppm) in adipose tissue. Dietary exposure 71. (D) of the general population has been alleged to occur through milk, eggs, cheese, meat, and 72. (A) fish. (Rom, 1998, pp. 1205–1217)

73. (C) 75. (E) Organophosphorus compounds have been widely used since the 1950s as insecticides, Explanations 70 through 73 both in national pest control programs and Knowing the incubation period (average and domestically. They have been responsible for range) of a pathogen can be important in deter- many deaths on a worldwide basis, despite mining the source of infection in food-borne the lives initially saved by control of mosqui- disease. Knowing what food was eaten on the toes and malaria. From the point of view of day of an attack of food poisoning may not the environmental toxicologist, it was perhaps (c) ketabton.com: The Digital Library

156 4: Preventive Medicine

fortuitous that many pests began to develop 80. (C) Hydrogen sulfide is a colorless gas that resistance to the substances fairly early in the rapidly paralyzes the nasal receptors, and is use of these compounds. More recently, con- found in sewers. Hydrogen sulfide produces cern for environmental control has further nausea, headache, and shortness of breath. limited their use; studies have attributed car- Because it paralyzes the nasal receptors at a cinogenic properties to several of these pesti- concentration of 150 ppm and cannot be cides. (Rom, 1998, pp. 1159–1163) smelled shortly after exposure, it is highly dangerous, with instant death from a concen- 76. (A) Nitrosamines are highly toxic and dan- tration as low as 1000 ppm. (Wallace and gerous to handle when used as solvents. Toxic Doebbeling, 1998, p. 577) amounts may be absorbed without warning because danger signals such as specific odor 81. (B) Methane is a colorless, odorless, flammable or irritant effects are lacking. The manufac- gas sometimes encountered in mines and wells. ture of rubber, dyes, lubricating oils, explo- Methane (coal damp) is a frequent cause of sives, insecticides, and fungicides, as well as death in inadequately ventilated mines and the electrical industry, all have associations wells. It acts as an asphyxiant as well as being with these substances. Nitrosamines have explosive. Miners used to take caged animals, animal carcinogenic properties and have been especially birds, with them. The birds suc- transmitted transplacentally in rats. (Rom, 1998, cumbed to the asphyxiants (methane and pp. 1227–1238) carbon dioxide) sooner than humans. (Rom, 1998, p. 652) 77. (B) Epoxy compounds are used in the produc- tion of resins. They cause irritation of the skin 82. (A) Carbon monoxide is an odorless, color- and mucosa and have caused acute pulmonary less, tasteless gas produced by partial com- edema. (Rom, 1998, p. 499) bustion of tobacco and fuels. The amounts produced by cigarette smoking are not 78. (D) Formaldehyde commercial solutions con- insignificant. The gas combines with hemo- tain up to 15% methanol. Formaldehyde has globin preferentially to form carboxyhemo- numerous industrial applications including, globin, which makes the patient appear pink. use as a base for urea formaldehyde resins. The This diminishes the oxygen-carrying capacity gas is an irritant to the eyes and upper respira- of the blood. The resulting anoxia is the major tory mucosa. The odors of products manufac- hazard. The onset of symptoms is insidious. tured with formaldehyde products have been Individuals exposed to carbon monoxide may responsible for actions against manufacturers not voluntarily take the action necessary to of tightly built manufactured homes. (Rom, 1998, remove themselves from the toxic fumes. (Rom, pp. 1115–1127) 1998, pp. 1505–1507)

79. (D) Ozone is a colorless, pungent gas occurring 83. (E) Sulfur dioxide is a colorless, pungent gas naturally in the stratosphere, which can be pro- encountered in drilling for oil, paper produc- duced by electric arcs. Ozone is generated by tion, treatment of fruit, and other processes. electrical storms and UV light and electric arcs Sulfur dioxide is an irritant gas. It causes tear- and some forms of fuel combustion. In the strat- ing, mucous membrane irritation, cough, and osphere, it is protective by blocking solar radia- eventually pulmonary edema. In asthmatics it tion. At 10 ppm, it can cause pulmonary edema provokes bronchospasm at low doses. Like and tracheal pain and is believed to cause other irritant gases, in large quantities, it will asthma. Based on animal tests and observations damage alveoli and capillary endothelial cells. of gases trapped during inversions, an action (Rom, 1998, pp. 631–639) level of 10 ppm has been set for work-place exposure. (Wallace and Doebbeling, 1998, p. 577) 84. (D) The USPSTF recommends that women aged 40 and older have screening mammography (c) ketabton.com: The Digital Library

Answers: 76–100 157

with or without clinical breast examination, 92. (A) every 1–2 years. (U.S. Preventive Services Task Force, 2006) 93. (C)

85. (F) The USPSTF strongly recommends screen- Explanations 89 through 93 ing for cervical cancer (Pap smear) at least every 3 years in women who have been sexu- IG given before exposure or within 14 days of ally active or are 21 years old, and have a exposure is 75–85% effective in preventing cervix. It recommends against screening after a symptomatic illness from hepatitis A. IG is pro- normal hysterectomy, or after age 65, if recent duced from the plasma of normal adults and Pap smears were normal and the woman is not does not contain sufficient antibody to prevent at high risk. (Note: Although evidence is still hepatitis B infection. Hepatitis B immune glob- considered insufficient, screening for human ulin (HBIG) is prepared from plasma known to papilloma virus [HPV] infection shows prom- contain high antibody titers for hepatitis B sur- ise as a part of cervical cancer prevention. face antigen (HBsAg) and is specific for hepa- Vaccination against HPV is expected to greatly titis B. Given immediately postexposure, and decrease the incidence of cervical cancer in the again 1 month later, it has a combined efficacy of about 75% in the prevention of hepatitis B. future.) (U.S. Preventive Services Task Force, 2006) Postexposure IG has not been found effective in 86. (E) The USPSTF strongly recommends that the prevention of hepatitis C infection; on the men and women aged 50 or older be screened other hand, treatment of early hepatitis C infec- for colorectal cancer. Various methods exist for tion is possible, and thus it is important to mon- screening and the optimal interval for screen- itor exposed individuals to determine whether ing varies based on the method of screening. infection occurs. IG administered to individu- Current options for screening average risk indi- als exposed to measles infection who are sus- viduals include FOBT every 1–2 years, sigmoi- ceptible to the disease has been shown to be doscopy every 5 years, colonoscopy every 10 effective if given within 6 days of exposure. years, and other tests. Higher-risk individuals, Recent use of IG is a contraindication to immu- those with a first-degree relative with a diag- nization with rubella vaccine. IG is not very nosis of colon cancer before age 60, should effective at preventing in utero infection with receive screening at an earlier age and at rubella, and infants with congenital rubella syndrome have been born to women given IG shorter intervals. (U.S. Preventive Services Task Force, 2006) shortly after exposure. IG is not routinely indi- cated, as it is indicated only if abortion is not 87. (C) The USPSTF strongly recommends screen- elected. (U.S. Preventive Services Task Force, 2006; ing pregnant women for hepatitis B at their Wallace and Doebbeling, 1998, p. 67, 184, 91, 97) first prenatal visit, but recommends against 94. (A) screening the general population. (U.S. Preventive Services Task Force, 2006) 95. (D) 88. (A) The USPSTF recommends against routine screening for pancreatic cancer in asymptomatic 96. (E) adults. (U.S. Preventive Services Task Force, 2006) 97. (D) 89. (A) 98. (C) 90. (B) 99. (B) 91. (B) 100. (D) (c) ketabton.com: The Digital Library

158 4: Preventive Medicine

Explanations 94 through 100 wounds well and removal of crust. Mycoplasma infections are particularly common in families The incubation period for diphtheria is 1–7 with younger children. They are frequently days. The incubation period for chicken pox is imported to the family by school-aged chil- 10–21 days, average 14. Infectious mononucle- dren, leading to a low-grade fever and persist- osis, caused by the Epstein-Barr virus, has an ing tracheobronchitis in the parents, or more estimated incubation period of 30–50 days. The acutely, an atypical pneumonia. G. lamblia is incubation period following infection by the found in up to 20% of homosexual males, and mumps virus is usually 16–18 days but, like may cause chronic diarrhea, although in these chicken pox, may vary from 14 to 25 days. patients it tends to be asymptomatic. E. coli Pertussis has a shorter incubation period, usu- was first reported as a cause of watery diarrhea ally 7–10 days, with a variation of 4–21 days. in nurseries in the 1940s. Although nursery epi- The usual period from contamination with demics with enteropathogenic serotypes had tetanus spores to clinical symptoms is generally decreased in recent years in the United States, 6–8 days. For rubella, the incubation period is the increase of infant-child day care centers has from 14 to 21 days, but usually ranges from 16 resulted in their relatively frequent occurrence. to 18 days. (Wallace and Doebbeling, 1998, p. 105, 117, Furunculosis is most frequently caused by 190, 93, 98, 103, 95) coagulase-positive staphylococcal infections. The public health significance of this largely 101. (I) relates to the hazards of skin infections in food handlers and subsequent staphylococcal toxin 102. (B) in the food, leading to staphylococcal intoxica- tion food-borne disease. H. pylori has been asso- 103. (D) ciated with gastric ulcers, but not with duodenal ulcers. Otitis media, whether acute or 104. (I) with effusion, commonly results from viral infection, such as by RSV. Various other organ- 105. (G) isms may be responsible including Streptococcus pneumoniae, H. influenzae, and others. C. per- 106. (A) fringens, with rare exceptions, is transmitted in a meat dish prepared in bulk. Under propi- 107. (F) tious circumstances for the organism, especially on cooling of the food, bacterial multiplication 108. (C) can be very rapid. Symptoms begin to occur in the affected population in about 12 hours. 109. (J) Epidemic typhus is a rickettsial illness. Man is the host and long-term reservoir. The vectors Explanations 101 through 109 are body lice (P. humanus corporis). The rick- Giardiasis may cause cramping and a chronic ettsia are not present in human excretions and diarrheal syndrome, with malabsorption and cannot be transmitted by person-to-person con- weight loss. Its distribution is worldwide, par- tact. (Wallace and Doebbeling, 1998, p. 252, 201, ticularly where hygienic standards are not 173, 252, 243, 265, 994, 116, 266, 305) high. It also occurs sporadically in high-risk individuals. Streptococcal pyoderma, includ- 110. (A) ing erysipelas and impetigo, has been demon- 111. (C) strated to precede acute glomerulonephritis. Even when appropriate antibiotics are given Explanations 110 and 111 in adequate dosage and duration for these con- ditions, renal damage may still result. Prevention The following table was constructed using the thus consists of wound care, including cleaning data in the question. There would be 4995 FPs, (c) ketabton.com: The Digital Library

Answers: 101–119 159

98 TPs, and 2 FNs. (Greenberg et al., 2001, pp. 7–8, study designs (cross-sectional, case-control). 78–79) (Gordis, 2004, pp. 149–157)

115. (B) The health survey was conducted in a Disease defined population on a particular date. With

Present Absent Totals this information, the prevalence of conjunc- tivitis can be determined, but not the incidence. Test Measuring incidence requires information on Positive 98 4995 5093 Negative 2 94,905 94,905 new occurrences of the disease from a follow- Totals 100 99,900 100,000 up study. (Gordis, 2004, pp. 33–37)

116. (B) By definition, sensitivity is the probability 112. (C) In a randomized, controlled trial, the of testing positive if the disease is truly present. determination of treatment group assignment It indicates the percentage of persons with the is left to chance. The procedure maximizes the disease of interest who have positive test probability that the two groups are similar in results. Positive predictive value estimates the important background characteristics. Thus, it probability of disease in those who have posi- avoids self-selection of study subjects to dif- tive test results. It indicates the percentage of ferent exposure groups. In a cross-sectional persons with positive test results who actually study design, it is generally difficult to ascer- have the disease of interest. (Wallace and tain the antecendent-consequent aspects of the Doebbeling, 1998, pp. 907–908) hypothesized relationship. In other words, since the exposure and outcome are measured 117. (B) In order to calculate predictive value, it is at a given point in time, it is difficult to deter- necessary to know prevalence. To illustrate this, mine which came first. A cohort study classifies consider the extremes if prevalence is zero, pos- study subjects by exposure status and follows itive predictive value will be zero, and negative them forward in time to determine develop- predictive value will be 100%! (Wallace and ment of disease. More than one disease can be Doebbeling, 1998, pp. 907–908) targeted as outcomes of interest. A case-control study defines cases and controls and retro- 118. (A) The problem can be represented by the fol- spectively assesses the frequency of exposure. lowing table: Multiple exposures can be assessed in connec- tion with a specific disease. (Gordis, 2004, pp. 115–128, 159–173, 149–157, 173–174) Breast cancer

113. (E) The design of a cohort study requires a Abortion Yes No follow-up of a group of subjects who are sus- Ye s 2 0 1 0 ceptible but free of the disease of interest at the No 80 190 Total 100 200 beginning of the study period. (Gordis, 2004, pp. 149–157)

114. (D) In a prospective cohort design, a group of The odds of exposure among cases is calcu- subjects at risk of developing disease are fol- lated by dividing the probability that a case lowed over a specified period of time. This was exposed by the probability that a case was design permits the direct calculation of inci- not exposed: (20/100)/(80/100) = 20/80 = 0.25. dence by dividing the number of subjects who (Greenberg et al., 2001, pp. 204–205) developed the disease of interest during the follow-up period by the population or person- 119. (D) The odds of exposure among controls are time at risk. In contrast, incidence cannot be calculated by dividing the probability that a calculated directly from the other observational control was exposed by the probability that a (c) ketabton.com: The Digital Library

160 4: Preventive Medicine

control was not exposed: (10/200)/(190/200) = of the intervals will include the unknown pop- 10/190 = 0.053. (Greenberg et al., 2001, pp. 204–205) ulation parameter. (Feinstein, 2002, pp. 195–197)

120. (E) The OR for exposure is calculated by divid- 123. (E) The insurance company charges premi- ing the odds of exposure for cases by the odds ums to the employer based on “experience of exposure for controls: OR = (20/80)/(10/190) rating,” which means charges are based on = 4.716. (Note that an unmatched case-control past utilization of service. To control premi- study such as this example would be flawed ums in the future, control utilization now. by confounding [age] and bias [recall bias]. Community rating would be different, in that Patients with breast cancer are more likely to risk would be spread across a broader com- remember and report that they had certain munity. Community rating makes insurance exposures compared with those without breast more affordable for individuals or groups that cancer.) (Greenberg et al., 2001, pp. 204–205) use a lot of health care, but removes incentive to control costs. (Rom, 1998, p. 84) 121. (A) The illness described is consistent with hemolytic uremic syndrome associated with E. 124. (E) The patient has a positive reaction to puri- coli 0157:H7 infection. E. coli 0157:H7 is the fied protein derivative (PPD). This indicates most common strain found of the enterohem- tuberculosis infection, but not necessarily clin- orrhagic E. coli (EHEC) group. Although its ical disease. Health care workers have a risk of most common reservoir is thought to be in acquiring tuberculosis that is 2–10 times the cattle, it has been found in other livestock. The risk for the general public. Since conversion is usual mode of exposure is contamination of recent (she had a negative test last year), the beef. The problem is compounded significantly risk of progressing to disease is relatively large when beef is ground and mixed in bulk. Testing compared to the risk of hepatotoxicity from and elimination programs do not appear sen- preventive treatment with isoniazid. Such pre- sitive enough to eliminate exposure, although ventive treatment is 65–80% effective in pre- active research in the area continues. In addi- venting progression to active disease. Note that tion, there are many other outbreaks associ- new assays (ELISPOT, QuantiFERON) per- ated with fresh vegetables, bean sprouts, and formed on a blood sample or oral swab are unpasteurized juices. It is hypothesized that becoming available, and that they are more these are due to contamination with human or sensitive and specific for exposure to animal waste. Since the organism is killed by Mycobacterium tuberculosis than is the TST. heating, thorough cooking of ground beef (Centers for Disease Control and Prevention, 2007; Rom, products, avoidance of contamination of fresh 1998, p. 802) foods with raw meat, and washing of produce intended to be served fresh is the most practi- 125. (A) Medicaid and Medicare began under fed- cal intervention. Currently, this remains the eral legislation in 1966. Medicaid is a state-run most practical advice to give the public. (Wallace program funded with federal and state tax dol- and Doebbeling, 1998, p. 726) lars. It was established to provide medical serv- ices to the poor, with special consideration to 122. (B) A confidence interval gives an estimated pregnant women and small children. Although range of values which is likely to include an eligibility varies greatly from state to state, per- unknown population parameter, in this case sons who receive SSI welfare payments are cat- the relative risk of stomach cancer with con- egorically eligible. Medicare is a federal program, sumption of the sugar substitute. The estimated completely financed and run by the federal gov- range is calculated from a given set of sample ernment. Its purpose is to provide medical care data. If independent samples are taken repeat- to citizens over 65. Blue Cross/Blue Shield is a edly from the same population, and a confi- private insurance that must be purchased; the dence interval calculated for each sample, then Robert Wood Johnson Foundation is a large a certain percentage (termed the confidence level) foundation that provides grants; and the WIC (c) ketabton.com: The Digital Library

Answers: 120–131 161

program is a food supplement program for down the outside of the well casing and into needy mothers and children. (Wallace and the water source. (Wallace and Doebbeling, 1998, Doebbeling, 1998, p. 1128) p. 745)

126. (A) Transmission of rabies virus by a bat bite 129. (D) Crude mortality rates for a population are has been well documented, and in many areas calculated using total deaths divided by total of the country, bats are known to harbor the population. Differences in the age compositon rabies virus. There are also well-documented of the population can make an enormous dif- cases of human rabies due to viral strains found ference in crude mortality rates, since the eld- in bats, but without a good history of being erly have a much higher mortality rate. bitten. Therefore, presumptive treatment is rec- Adjusting for age computes a hypothetical ommended in the case of a bite (and in some “adjusted” rate for each country based on their cases for a possible bite) when the bat cannot be own age-specific mortality rates, but using recovered and tested. Lacrosse encephalitis, standard population. The technique is called distemper, and tularemia are not known to be age adjusting, or standardization. It is used in transmitted by bats. Tetanus is unlikely because other rate studies as well. For instance, the a child this age has probably been immunized. crude mortality rates for MI would be different (Wallace and Doebbeling, 1998, p. 349, 350) for a largely middle-aged population than for an older population. Age adjusting allows com- 127. (B) Manufacturing, storage, and shipping prob- parison. (Greenberg et al., 2001, pp. 51–53) lems do occur. However, they have not been related to nationwide failures, although a major 130. (B) Although clinical trials can determine manufacturing problem in 2004 resulted in a which treatment works best, they cannot deter- major reduction in supply of vaccine. It is more mine the cost of this success. Unfortunately, likely that the vaccine in use is not completely they do not take into account costs and ultimate protective against the prevalent viral strain. benefits, which would be the role of a cost- The WHO monitors influenza outbreaks benefit study. If properly constructed, with worldwide. Based on strains present in out- study and control groups randomly drawn breaks, and especially in the Pacific Rim, WHO from the general population, the results should makes a recommendation every spring regard- be applicable to the general population. In ing the antigens to use for the fall immuniza- addition, they usually determine what works tion campaign. Manufacturing then begins. best by measurement of a clinical endpoint. In Although their track record is good in this many cases, the treatment may achieve its regard, antigen changes in influenza may objective, but the change in parameters may reach the United States undetected or ahead of not be significant. It is worthy of note that the schedule. (Wallace and Doebbeling, 1998, p. 110) USPSTF is now taking cost-effectiveness into account when making recommendations. (U.S. 128. (B) Testing for the presence of coliform bacte- Preventive Services Task Force, 2006) ria is a standard bacterial test of water quality. Since coliform bacteria are ubiquitous in 131. (E) It is true that mammography for detection nature, their presence means only that the of breast cancer in older women is technically water has been contaminated with water from easier and probably more sensitive than in the ground surface or the septic system. It is younger women. However, the burden of cost only an indicator. Coliform bacteria are not the and mortality resulting from screening women principal agents of disease. It is the other agents less than 40 was the real reason that screening introduced into the water by the contamina- them was not recommended. For first mam- tion that create the risk. The problem should be mograms of women aged 40–49, cancers were resolved before using the well water. A diagnosed at half the rate of women aged 50–59 common cause of contamination is poor well years old, yet twice as many follow-up diag- construction, resulting in surface water moving nostic tests were performed. Simply put, the (c) ketabton.com: The Digital Library

162 4: Preventive Medicine

cost in terms of dollars, mortality from testing, for blood pressure, alcohol use, and exercise. and mortality from radiation exposure for Review of hospital records may capture some women under 40 were not considered to be of this information, but is not an efficient worth the benefit. (U.S. Preventive Services Task method of capturing information on multiple Force, 2006) risk factors for chronic disease. Registries are primarily used to track clinical information 132. (D) In 2004, the infant mortality rate for chil- regarding particular diseases or conditions. dren born to White mothers was 5.7 infant They do not tend to capture a large body of deaths/1000 live births. The infant mortality information on behavioral risk factors. rate for children born to Black or African- Physician-based reports are usually generated American mothers was 13.8 infant deaths/1000 as a consequence of state or federal mandates live births. (Health, US, 2006) regarding the reporting of communicable dis- ease, rather than behavioral risk factors for 133. (C) Active surveillance is used in urgent situa- chronic disease. (Wallace and Doebbeling, 1998, p. 12) tions, such as active and ongoing epidemics. Health agencies contact those data sources 135. (C) An effect modifier changes the relationship most likely to have current information regard- between a risk factor and an outcome. In this ing cases. The collection of data which has been example, the overall relationship between alco- voluntarily submitted is referred to as passive hol and SES has not changed, but its effects are surveillance. Case-control studies are a form greatest in a particular age group, and thus age of investigation and represent a “next step” is modifying the effect of the association after surveillance. These studies seek to identify between alcohol and SES. In this example, the further information regarding the health prob- study was stratified based on age, which would lem by studying individuals with the disease. control for confounding effect based on age. Surveillance systems on the other hand are Bias is seen when there is a systematic error in used primarily to identify whether or not a the manner in which age is distributed between problem exists and how it is changing through the risk factor and outcome categories. Bias time. Monitoring disease in animal population occurs when there is a systematic error in the or in other specific populations that are higher design or conduct of a study. There is not or earlier risk is referred to as sentinel data col- enough information provided to determine lection or surveillance. Identifying where cattle whether bias may be present. Because the find- were purchased may help to identify the extent ings have been stratified and have been found of disease spread in animals, but does not to be statistically significant at each stratum, describe active surveillance of human cases. there is less of a possibility that chance is play- (Wallace and Doebbeling, 1998, pp. 8–16) ing a role in the relationship between age and the exposure or outcome. Distractor is not a 134. (A) The Behavioral Risk Factor Surveillance term commonly used to describe a variable’s System is a health survey that uses telephone relationship to an outcome. (Wallace and interviews to collect information about chronic Doebbeling, 1998, pp. 24–25) disease risk factors such as obesity, treatment (c) ketabton.com: The Digital Library

BIBLIOGRAPHY

Agency for Toxic Substances and Disease Registry. Gordis L. Epidemiology, 3rd ed. Philadelphia, PA: W.B. Lead Toxicity. Case Studies in Environmental Medicine. Saunders, 2004. Atlanta, GA: U.S. Department of Health and Greenberg RS, Daniels SR, Flanders WD, et al. Medical Human Services, 2000. Available at: http://www. Epidemiology, 3rd ed. New York, NY: McGraw-Hill, atsdr.cdc.gov/HEC/CSEM/lead/index.html. 2001. Centers for Disease Control and Prevention. Anthrax, Health, United States, 2006. U.S. Department of 2007. Available at: www.cdc.gov/. Health and Human Services, Centers for Disease Centers for Disease Control and Prevention. Control and Prevention, National Center for Tuberculosis, 2007. Available at: http://www. Health Statistics. Available at: http://www.cdc. cdc.gov/nchstp/tb/default.htm. gov/nchs/data/hus/hus06.pdf. Centers for Disease Control and Prevention. Hunt RD, Sachs DPL, Glover ED, et al. A comparison Influenza—Clinical Description and Diagnosis, of sustained release bupropion and placebo for 2007. Available at: http://www.cdc.gov/flu/ smoking cessation. N Engl J Med 1997;337(17): professionals/diagnosis/. 1195–1202. Centers for Disease Control and Prevention. National LaDou J. Current Occupational & Environmental Center for Health Statistics. Health, United States, Medicine, 3rd ed. New York, NY: McGraw-Hill, 2006. Table 29: age-adjusted death rates for selected 2003. causes of death, according to sex, race, and Lane WG, Kemper AR. American College of Hispanic origin. Available at: http://www.cdc.gov/ Preventive Medicine. Practice Policy Statement. nchs/data/hus/hus06.pdf. Screening for elevated blood lead levels in chil- Centers for Disease Control and Prevention. National dren. Am J Prev Med 2001;20(1):78–82. Available at: Center for Infectious Diseases: Viral Hepatitis http://www.acpm.org/polstmt_blood.pdf. Surveillance. Disease burden from Hepatitis A, B, Lang RS, Hensrud DD. Clinical Preventive Medicine, and C in the United States, 2007. Available at: 2nd ed. Chicago, IL: AMA Press, 2004. http://www.cdc.gov/ncidod/diseases/hepatitis/ Last JM. Public Health and Human Ecology, 3rd ed. resource/PDFs/disease_burden.pdf. Stamford, CT: Appleton & Lange, 1998. Centers for Disease Control and Prevention. McCunney RJ. A Practical Approach to Occupational Norovirus, 2007. Available at: http://www.cdc. and Environmental Medicine, 3rd ed. Philadelphia, gov/ncidod/dvrd/revb/gastro/norovirus.htm. PA: Lippincott Williams & Wilkins, 2003. Centers for Disease Control and Prevention. Rom WN, ed. Environmental and Occupational Prevention and control of meningococcal disease. Medicine, 3rd ed. Philadelphia, PA: Lippincott- MMWR 2000;49:RR07, 1. Available at: www.cdc. Raven, 1998. gov/mmwr. U.S. Census Bureau, Statistical Abstract of the United Centers for Disease Control and Prevention. States: 2007 (126th Edition) Washington, DC, 2005. Recommended childhood and adolescent immu- Available at: http://www.census.gov/compendia/ nization schedule—United States, 2007. Available at: statab/. http://www.cdc.gov/nip/recs/child-schedule.htm. U.S. Preventive Services Task Force. The Guide to Dawson-Saunders EK. Basic and Clinical Biostatistics, Clinical Preventive Services, 2006. Agency for 3rd ed. New York, NY: McGraw-Hill, 2000. Healthcare Research and Quality, US Department Department of Health and Human Services of Health and Human Services. Available at: Organizational Chart, 2007. Available at: http:// www.preventiveservices.ahrq.gov. www.hhs.gov/about/orgchart.html. Wallace RB, Doebbeling BN, eds. Maxcy-Rosenau-Last Feinstein AR. Principles of Medical Statistics. Textbook of Public Health & Preventive Medicine, 14th Washington, DC: Chapman & Hall, 2002. ed. Stamford, CT: Appleton & Lange, 1998.

163 (c) ketabton.com: The Digital Library

This page intentionally left blank (c) ketabton.com: The Digital Library

CHAPTER 5 Psychiatry Jon A. Lehrmann, MD

Questions

Questions 1 through 3 (C) haldoperidol (D) dantrolene A 28-year-old woman with a 7-year history of chronic undifferentiated schizophrenia is hospitalized for an (E) fluoxetine exacerbation of her schizophrenia, with an increase in auditory hallucinations. She has also developed the Questions 4 and 5 delusion that she is controlled by aliens from Mars. A 35-year-old man with no previous psychiatric his- She has always been very sensitive to the extrapyra- tory is referred by his family physician for psychi- midal side effects (EPS) of antipsychotic medications. atric evaluation. The family physician has been fol- lowing the man for mild hyperlipidemia, which is 1. Which of the following is an extrapyramidal currently being successfully treated with dietary side effect of antipsychotic medications most changes alone. The man reports that he has been hap- likely to be seen in the first few days of treat- pily married for 10 years and has two children. He ment with typical neuroleptic antipsychotics? has been working as an accountant and has generally (A) pill-rolling tremor of the hands enjoyed his job. About 2 months ago, with no obvi- (B) severe restlessness of the arms and legs ous precipitating event, he says, the man began to feel very blue, with a drop in his desire to play with (C) involuntary lip smacking his children and to compete in his usual volleyball (D) muscle spasm in the neck league games. He reports that he has lost most of his (E) masked facies libido and is having difficulty sleeping, with early- morning wakening. He reports that his appetite has 2. This woman develops cogwheel rigidity and a become very poor over the past 2 months, and he pill-rolling tremor. Which of the following is estimates that he has lost 10 lbs over that time. the most appropriate treatment choice? (A) benztropine 4. Which of the following is the most likely (B) propranolol diagnosis? (C) haldoperidol (A) Alzheimer’s disease (D) dantrolene (B) schizophrenia (E) fluoxetine (C) major depressive disorder (D) metastatic cancer to the brain 3. She developed an acute restlessness in her legs (E) systemic lupus erythematosus and arms which her psychiatrist diagnosed as akathisia. Which of the following is the pre- ferred treatment option? (A) benztropine (B) propranolol

165

Copyright © 2008 by The McGraw-Hill Companies, Inc. Click here for terms of use. (c) ketabton.com: The Digital Library

166 5: Psychiatry

5. Of the following, which would be the most 9. A 55-year-old man requests “some kind of pain appropriate initial treatment for this patient? medication that really works!” to relieve the “extreme” pain in his foot. He walks with a (A) electroconvulsive therapy (ECT) cane. He angrily claims that his previous (B) clozapine employer did not care about what happened to (C) diazepam him in an accident 1 year earlier in which his (D) fluoxetine foot was struck by an iron rod. No fracture was (E) buspirone found. He claims his doctor said he had a “severe contusion” and then states “the doctor DIRECTIONS (Questions 6 through 10): For each didn’t know anything.” numbered item, select the ONE best lettered option that is most closely associated with it. Each 10. A 40-year-old woman is brought to the emer- lettered option may be selected once, more than gency department after she had frantically once, or not at all. called the paramedics because she thought she would die. She was experiencing sharp chest pain, shortness of breath, racing heartbeat, and Identify the most likely diagnosis with the case cold, sweaty chills. Cardiac assessment proved descriptions below. negative for myocardial ischemia. (A) body dysmorphic disorder (B) conversion disorder DIRECTIONS (Questions 11 through 69): Each of the numbered items or incomplete statements in (C) factitious disorder this section is followed by answers or by comple- (D) hypochondriasis tions of the statement. Select the ONE lettered (E) malingering answer or completion that is BEST in each case. (F) pain disorder (G) panic disorder Questions 11 and 12 (H) somatization disorder A 70-year-old female with chronic paranoid schizo- 6. Despite repeated efforts to reassure a 40-year- phrenia presents to the ER acutely confused with old woman that the stomach pain she is expe- visual hallucinations. Her skin is warm and dry and riencing is not cancerous, she continues to her heart rate is 110 beats per minute. Her group worry and fears that she will die. home nurse tells you that the patient had been com- plaining of having a dry mouth and having diffi- 7. A 23-year-old violinist reports to his neurolo- culty initiating urination this past week. gist that he thinks he has had a stroke. He is unable to feel anything with his left fingers and 11. Which of the following is the most likely cause is barely able to hold down the violin strings for this presentation? with this same hand because of “paralysis.” (A) psychotic exacerbation of schizophrenia The numbness he describes reaches to his wrist only, and he “even feels a band” around the (B) urinary tract infection wrist delineating the sensitive from the insen- (C) Alzheimer dementia sitive areas. (D) anticholinergic delirium (E) myocardial infection 8. A 35-year-old woman complains that she has been to multiple doctors, none of whom have 12. Which of the following is the treatment of been able to effectively treat or even diagnose choice for this patient’s acute condition? the cause of her chronic stomach pain and diar- rhea, repeated problems swallowing, headache, (A) bethanechol and recurrent back pain. The symptoms have (B) haldoperidol been present on and off for most of her adult life. (C) physostigmine (c) ketabton.com: The Digital Library

Questions: 5–18 167

(D) bromocriptine 15. Given the information above, which of the fol- (E) dantrolene lowing is the most likely diagnosis? (A) depersonalization disorder 13. A 25-year-old Caucasian man reports that he (B) posttraumatic stress disorder (PTSD) has got in trouble with the law as a result of his rubbing up against a woman he did not know (C) dissociative amnesia in an elevator at work. (D) dissociative fugue Recurrent, intense sexually arousing fan- (E) major depression tasies, sexual urges, or behaviors involving touching and rubbing against a nonconsenting 16. Of the following treatments, which is the most person, such as a stranger in an elevator, which appropriate to help this patient regain her occur for at least 6 months and which cause memory? significant impairment in an individual’s func- (A) electroshock therapy tioning are best characterized as which of the following? (B) individual psychotherapy (C) chlordiazepoxide (A) exhibitionism (D) sertraline (B) fetishism (E) aripiprazole (C) frotteurism (D) pedophilia Questions 17 through 20 (E) sexual masochism A 44-year-old man presents with fears that his math- ematical abilities have been slowly sucked out of his 14. A patient reports that, on his way to the hospi- brain for the last 4 years. He believes an “alien force tal, he saw a man feeding two squirrels in the disguised as a human being” is responsible. To avoid park. He says that this means his future will be contacting this being, he has isolated himself in a decided in 2 weeks. This man, he believes, is room in a boarding house. His wife divorced him deliberately out to alarm him (the patient). One and left with their children. After 10 years teaching of the squirrels is scheming with the man; the math at a local high school, he resigned about 3 years other is innocent and trusting. Which of the ago. He supports himself by “collecting cans.” His following terms best describes what this man is affect is blunted. His appearance is disheveled, experiencing? unshaven, and unwashed. (A) illusions (B) hallucinations 17. Which of the following is the most likely (C) delusions diagnosis? (D) loosened associations (A) paranoid schizophrenia (E) neologisms (B) alcohol abuse and dependence (C) major depression with psychotic features Questions 15 and 16 (D) Alzheimer’s disease A young mother seeks psychiatric help because she (E) Huntington’s disease is unable to remember events surrounding her 3-year-old son’s death after being struck by a car 18. Which of the following factors in this case 2 months ago. She is worried that maybe she did would favor a more positive prognosis? something that put him at risk. Except for this brief (A) presence of negative symptoms time period, she is able to recall other events both before and after the tragedy. (B) late onset (C) being male (D) being divorced (E) experience in teaching mathematics (c) ketabton.com: The Digital Library

168 5: Psychiatry

19. Which of the following hypotheses is the lead- (C) Send her home telling her parents this is ing hypothesis to explain the patient’s psy- normal teen behavior. chotic symptoms? (D) Start a prenatal vitamin. (A) serotonin hypothesis (E) Admit her for inpatient psychiatric (B) biogenic amine hypothesis hospitalization. (C) acetylcholine hypothesis Questions 23 through 25 (D) dopamine hypothesis (E) gamma aminobutyric acid (GABA) A 70-year-old man is brought to his primary care hypothesis doctor by the man’s son. According to his son, who had not seen his father for about a year, the father 20. Considering the information learned thus far, seemed to have some personality changes. He was which of the following would be an appropri- no longer interested in his hobbies and seemed apa- ate first-line treatment for this patient? thetic. He seemed to forget easily, and he repeatedly asked the same already answered questions. On at (A) thiamine least two occasions, the father wandered out of the (B) olanzapine house and was found by neighbors, who thought he (C) amitriptyline was confused. (D) lithium carbonate (E) fluoxetine 23. Which of the following is the most common cause of dementia in a 70-year-old man? Questions 21 and 22 (A) Alzheimer’s disease A 17-year-old girl is brought in by her parents (B) Pick’s disease because of their concerns about her weight loss. She (C) Parkinson’s disease is petite, and a normal weight for someone her (D) vascular dementia height is 100 lbs, but she weighs 78 lbs. She reports (E) subcortical dementia menstrual irregularity. The patient believes she is obese. She does not believe she has a problem. Her 24. Considering the information learned thus far, mother reports that she found laxatives on her which of the following medications would be daughter’s nightstand, and heard her vomiting in the most appropriate treatment here? the bathroom yesterday after dinner. (A) donepezil 21. Which of the following is the most likely (B) fluoxetine diagnosis? (C) aspirin (A) bulimia nervosa (D) amitriptyline (B) pregnancy (E) ginkgo biloba (C) anorexia nervosa 25. If this man had Pick’s disease, where would (D) no diagnosis, normal presentation the preponderance of pathology be found? (E) obesity (A) cerebellum 22. Which of the following is the best initial (B) caudate nucleus treatment? (C) hippocampus (A) Have her return in 2 weeks for cognitive (D) frontotemporal areas behavioral therapy. (E) parietotemporal areas (B) Initiate lithium as it often causes weight gain. (c) ketabton.com: The Digital Library

Questions: 19–31 169

Questions 26 through 28 29. A 70-year-old woman was brought to the emer- gency department following her involvement A 25-year-old male graduate student presents at a in a minor car accident. She had sustained no university hospital emergency department com- injuries but was very upset and was, therefore, plaining of a sudden onset of a pounding in his referred to a psychiatrist. After speaking at chest, a feeling of choking, and shortness of breath. length about her part in the accident and shar- He reports that things somehow suddenly seem ing her reactions, she still remained tremulous, unreal, and he is afraid he is dying of a heart attack. anxious, and tearful. She has no history of An electrocardiogram (ECG) shows normal sinus addiction. You decide to use an anxiolytic to rhythm and no abnormalities. help her. Which of the following is the best choice? 26. Which of the following is the most likely diagnosis? (A) diazepam (B) clorazepate (A) myocardial infarction (C) lorazepam (B) panic attack (D) buspirone (C) hypochondriasis (E) temazepam (D) multiple sclerosis (E) generalized anxiety disorder 30. Which of the following delusions would most likely be observed in a psychotically depressed 27. If this patient continues to experience similar person? episodes over the next 2 months, with sub- stantial apprehension about the episodes, (A) “My mind’s eye is perfused with a which of the following drugs would be most radiance of the gods.” appropriate for the treatment of this patient’s (B) “I’ve been targeted by the FBI.” condition? (C) “My body is rotting inside out.” (A) paroxetine (D) “I have been hand-picked to be the (B) buspirone world’s leader; I am awaiting the signal to bring the masses together.” (C) ziprasidone (E) “All I need to do is clutch the book to (D) propranolol myself and all the knowledge pours (E) haloperidol into me.”

28. You encounter this patient 1 year after the onset 31. A fourth-year medical student on an emer- of his symptoms. He has not been very com- gency medicine clerkship is fascinated by the pliant with his medications, or appointments. number of personality disordered patients who Now he reports he rarely leaves his home and come to the emergency room on weekends. avoids places and social situations where he Which of the following personality disorders feels he cannot readily escape. When he does belongs to cluster A (odd, eccentric) in the go places, he reports having an escape plan DSM-IV-TR (Diagnostic and Statistical Manual arranged. Which of the following best describes of Mental Disorders, 4th ed., Text Revision) this condition? classification of personality disorders? (A) social phobia (A) schizoid personality disorder (B) claustrophobia (B) borderline personality disorder (C) arachnophobia (C) antisocial personality disorder (D) agoraphobia (D) avoidant personality disorder (E) simple phobia (E) obsessive-compulsive personality disorder (c) ketabton.com: The Digital Library

170 5: Psychiatry

32. A 5-year-old girl had been doing well at home Questions 35 and 36 and with her playmates until she started kindergarten 1 month ago. She cries whenever You are called to the ICU to see a 65-year-old female her mother and father go to drop her off at who is 2 days status post hip surgery. The medical school, and she complains of fearing that she team is concerned because the patient is having will never see them again. She complains of visual hallucinations and did not sleep last night. nightmares and refuses to go to school due to She scored 21/30 on a Mini-Mental Status severe stomachaches. Which of the following is Examination (MMSE) with 5 points off for disorien- the most likely diagnosis? tation and 2 off for both concentration and short- term memory. The patient is restrained because she (A) generalized anxiety disorder pulled out her IV last night. Records indicate she (B) separation anxiety disorder has no previous psychiatric history and that her (C) social phobia cognitive functioning presurgery was normal. (D) specific phobia 35. Which of the following is the most likely axis I (E) obsessive-compulsive disorder diagnosis? 33. A family brings in their 7-year-old boy because (A) bipolar disorder manic phase he has not been fitting in at school. The mother (B) schizophrenia, undifferentiated says he has not made any friends and tends to (C) dementia NOS (not otherwise specified) play mostly by himself. He exhibits repetitive (D) delirium NOS hand flapping. When you interview him, he does not provide any eye contact, and the (E) psychosis NOS mother says this is typical for him. His speech seems age appropriate and per history has 36. Which of the following is not a predisposing developed normally. This past year his IQ was factor for the above condition? tested to be normal. Which of the following is (A) cancer the most likely diagnosis? (B) advanced age (A) autism (C) female gender (B) Asperger disorder (D) dementia (C) Rett disorder (E) alcohol dependence (D) attention deficit hyperactivity disorder (ADHD) Questions 37 and 38 (E) borderline personality disorder A 56-year-old man with a dual diagnosis of schizo- phrenia and alcohol dependency was arrested for 34. An 8-year-old boy suffers from compulsive “driving under the influence.” After 2 days in jail, eating, obesity, small stature, hypogonadism, he was noted to be acting agitated and “crazy,” and mental retardation. Which of the following claiming that the guards were going to kill him that is his most likely diagnosis? night. When questioned about his profuse sweating, (A) phenylketonuria he claimed that he had just come in from his job working in the heat and humidity. He was dis- (B) Prader-Willi syndrome tracted by various noises, claiming that these were (C) Down syndrome made by people watching him and waiting for him. (D) fragile X syndrome He appeared to watch things that seemed to be (E) Cri-du-chat syndrome moving on the walls. On further examination, it was (E) disorientation noted that he was very tremulous, unable to follow an object with his eyes only, and quite ataxic. (c) ketabton.com: The Digital Library

Questions: 32–42 171

37. The working diagnosis at this point is most Questions 41 and 42 likely which of the following? A 35-year-old woman comes into the ER after cut- (A) acute exacerbation of schizophrenia ting her wrists for the 10th time. She did this after (B) alcohol withdrawal delirium her boyfriend of 2 weeks left her yesterday. She (C) alcohol-induced persisting dementia reports a history of unstable interpersonal relation- (D) alcohol intoxication ships, chronic feelings of emptiness, impulsive sex- ual relationships, and problems with her sense of (E) malingering identity. 38. Which of the following benzodiazepines can 41. Which of the following is the most likely axis II reliably be given intramuscularly to this patient diagnosis? to help control his symptoms? (A) histrionic personality disorder (A) diazepam (B) borderline personality disorder (B) chlordiazepoxide (C) antisocial personality disorder (C) lorazepam (D) dependent personality disorder (D) alprazolam (E) avoidant personality disorder (E) clonazepam 42. The one psychotherapeutic technique to avoid 39. A 25-year-old woman presents with a recur- would be: rent depression, and says she has taken many antidepressant medications from several dif- (A) therapeutic physical holding sessions to ferent classes of drugs in the past. Which of help the patient feel in control the following atypical antidepressants is a (B) clear roles and responsibilities of patient presynaptic alpha-2 receptor noradrenergic and therapist are established antagonist and a 5-HT2 and 5-HT3 receptor (C) therapist conveys empathic validation serotonergic antagonist? (D) flexibility (A) bupropion (E) patient and therapist mutually develop (B) trazodone a hierarchy of priorities (C) nefazodone (D) venlafaxine Questions 43 through 45 (E) mirtazapine A 58-year-old woman with a history of chronic paranoid schizophrenia, who has been continu- 40. A 60-year-old man disappears from his home ously treated with antipsychotics for the past 20 and travels 100 miles to a small town, where he years, lives in a community-based residential facil- opens a small grocery store using a different ity. She has recently suffered an increase in auditory name. Six weeks later, he awakens in some agi- hallucinations, and her haloperidol dose has been tation, uses his original name, and asks to increased from 2.5 to 10 mg/day. Four days later, know where he is. He wishes to return to his she is brought by a visiting nurse to the emergency home. Which of the following is the most likely room, where she presents with confusion, marked diagnosis? flexor and extensor rigidity in her legs and arms, (A) dissociative amnesia and a temperature of 103.5°F. Her blood pressure is 160/120 mmHg, her pulse is 120/min and irregular. (B) dissociative fugue (C) dissociative identity disorder (D) depersonalization disorder (E) dissociative disorder not otherwise specified (c) ketabton.com: The Digital Library

172 5: Psychiatry

43. Which of the following is the most likely diag- Which of the following is the most appropriate nosis that best describes this woman’s current step in management? condition? (A) Advise her to take an anti-inflammatory (A) neuroleptic malignant syndrome (NMS) analgesic. (B) metabolic syndrome (B) Advise her to rest and call again in (C) extrapyramidal symptoms 8 hour if the pain has not subsided. (D) malingering (C) Consult immediately with her (E) alcohol withdrawal ophthalmologist. (D) Plan to evaluate her eye at her next 44. Which of the following is the most important psychiatric appointment in 2 weeks. laboratory test to evaluate the possibility of the (E) Decrease imipramine to 125 mg/day. diagnosis? 48. A 40-year old man has been treated for chronic (A) serum creatine phosphokinase (CPK) paranoid schizophrenia for many years with a level typical neuroleptic. To decrease his risk for tar- (B) serum sodium level dive dyskinesia, his psychiatrist wants to (C) serum potassium level change his medication to an atypical antipsy- (D) serum glucose level chotic. Which of the following atypical antipsy- (E) serum calcium level chotics is limited in its use by the risk of agranulocytosis, which occurs in 1–2% of all 45. Which of the following is most likely to be an patients treated? effective treatment for this condition? (A) aripiprazole (A) intramuscular haloperidol (B) clozapine (B) oral bromocriptine (C) risperidone (C) intramuscular lorazepam (D) quetiapine (D) intramuscular benztropine (E) ziprasidone (E) oral propranolol Questions 49 and 50 46. Narcolepsy is a primary sleep disorder with A 45-year-old homeless schizophrenic patient pres- all of the following symptoms except which of ents to you with suicidal ideation. You interview the following? him and find out he is a divorced Roman Catholic. (A) sleep paralysis He recently lost his job after being caught a second (B) sleep attacks with sleep onset REM time drinking on the job. He had attempted suicide (rapid eye movement) impulsively 5 years previous by overdosing. He is (C) cataplexy not currently psychotic. He bought a handgun and ammunition recently and has been thinking about (D) hypnagogic hallucinations shooting himself in the head. He has gotten as close (E) apnea to acting on it as having loaded the gun and held it up to his head this morning. Someone walking by 47. A 37-year-old woman telephones to alert her stopped him and convinced him to come and see psychiatrist that she has developed a severe you. He is ambivalent about seeking help. pain in her right eye that has persisted for about 5 hours. She has no history of migraine 49. Which of the following is not associated with headaches. The psychiatrist is treating her with an increased suicide risk? 150 mg imipramine for major depression. She denies any recent injury or infection in this eye. (A) Roman Catholic religion She wears corrective lenses for nearsightedness. (B) male (c) ketabton.com: The Digital Library

Questions: 43–55 173

(C) divorced (D) Refer for supportive group psychotherapy. (D) previous suicide attempt (E) Refer to a pain management clinic. (E) schizophrenia 53. Which of the following most appropriately 50. Which of the following is the most appropriate describes this woman’s disorder? immediate treatment recommendation? (A) more frequently seen in women than in (A) Start him on antidepressant medication. men (B) Send him home to live with his brother (B) 20–25% prevalence rate in a general and ask the brother to keep the gun. medical practice (C) Increase his antipsychotic medication. (C) course of disorder usually of short dura- (D) Prescribe a benzodiazepine to calm him tion (2–3 months) down. (D) associated with elevated erythrocyte (E) Admit him to the inpatient psychiatric sedimentation rate (ESR) unit. (E) absence of secondary gain is a favorable prognostic indicator Questions 51 through 53 54. During the course of psychotherapy with a A 45-year-old woman, seen by her medical internist, woman who has a severe phobia of cars, it is has been experiencing fears that she may have a discovered that her first sexual experience, serious illness. She complains that after eating she which was a humiliating one for her, took place experiences “a lot of gas” and abdominal pain, fol- in an automobile. What is the defense mecha- lowed by diarrhea on occasion. Her heart at times nism illustrated by this phobia called? seems to be beating rapidly, and she feels faint at times, has chest “discomfort,” and wonders if she is (A) acting out having a heart attack. Multiple tests have identified (B) reaction formation only a mild irritable bowel syndrome. The woman’s (C) displacement fears are not allayed by this. She makes repeated (D) sublimation calls to be seen by her doctors as well as seeking (E) repression consultation from other specialists. She insists that “there’s something there” and believes the doctors 55. A 65-year-old man is referred for a psychiatric are not taking her seriously. evaluation by his primary care doctor. The doctor has noted that his patient seems less 51. Which of the following is the most likely concerned about his personal hygiene, his diagnosis? clothes are mismatched, and he is no longer (A) factitious disorder getting to his doctor’s appointments on time. In (B) major depression addition, the patient seems depressed, cries, and “no longer enjoys a good joke.” Which of (C) reaction psychosis the following is the most therapeutic opening (D) hypochondriasis question in interviewing this man? (E) pain disorder (A) “Tell me about your depression.” 52. Which of the following is the most effective (B) “Why are you crying?” long-term management of this patient? (C) “Tell me what’s been happening that brings you here.” (A) Transfer her care to a psychiatrist. (D) “Your doctor tells me you don’t match (B) Prescribe alprazolam. your clothes anymore—why not?” (C) Establish regular follow-up visits with (E) “Your doctor says you’re depressed. How regularly scheduled physical about an antidepressant to help you?” examinations. (c) ketabton.com: The Digital Library

174 5: Psychiatry

56. A 30-year-old woman complains that she has 59. A 78-year-old woman is seen by a psychiatrist had sleep disturbances since the start of her for depression. She is fairly cooperative in depression 2 months ago. Which of the follow- responding to questions. She admits to feeling ing is an accurate description of typical sleep blue; she “catnaps” throughout the day and is abnormalities in depression? up at night; and her appetite is very poor. She thinks of death frequently but denies feeling (A) Sleep latency (the period of time suicidal. There is no past psychiatric history. On between going to bed and falling asleep) the MMSE, she obtains a score of 14. Her is shortened. depressive symptoms have been present for (B) REM latency (the period of time from “several days.” Which of the following is the onset of sleep to the first REM highly suggested by the findings? period) is shortened. (C) Wakefulness is decreased. (A) impaired cognitive functioning (D) The arousal threshold is increased. (B) psychosis not otherwise specified (E) Stage 3 and stage 4 sleep are increased. (C) bipolar disorder—manic (D) dysthymia Questions 57 and 58 (E) changes secondary to normal aging

A 25-year-old man presents in the emergency depart- Questions 60 and 61 ment for a 2-week problem of worsening urinary hes- itancy. He has had problems getting his urine stream 60. A 40-year-old man has been unsuccessfully started and has noted a decrease in the force of the treated for depression with two different med- stream. Now it seems to just “dribble out.” He denies ications for the past 3 months. He has a number any pain or burning, any medical problems, and any of medical problems, and he recently was hos- exposure to sexually transmitted diseases. For pitalized after threatening suicide. His psychi- approximately 1 month, he has been taking thiori- atrist is considering the use of ECT for the dazine, 200 mg bid, and benztropine, 2 mg qid, and patient. Which of the following is a relative “sometimes one or two benztropine” prn. contraindication to ECT? (A) hypertension 57. Given the above information, which of the fol- lowing is the most likely cause of this man’s (B) history of seizures problem? (C) clinically significant space-occupying cerebral lesion (A) anticholinergic side effects to the thiori- (D) degenerative joint disease of the spine dazine and benztropine (E) suicidality (B) urethral stricture (C) breakthrough of a psychotic delusion 61. The patient has consented to ECT, and the pre- that he cannot urinate ECT workup has been completed. Which of (D) injury from a perverse sexual practice the following medications could routinely be he is not admitting to continued through a course of ECT? (E) infection of the urethra (A) lithium 58. Which of the following is a safer choice of med- (B) divalproex ication for this man? (C) bupropion (D) clonazepam (A) amitriptyline (E) risperdal (B) risperidone (C) chlorpromazine (D) mesoridazine (E) imipramine (c) ketabton.com: The Digital Library

Questions: 56–68 175

Questions 62 and 63 (A) lithium (B) bupropion A 40-year-old woman with no previous psychiatric history seeks help from her internist for a sleep (C) alprazolam problem. Initially, she is able to fall asleep but then (D) propranolol sleeps fitfully, and finally around 4:00 a.m. decides (E) perphenazine to stay up. She averages approximately 3–4 hours of sleep per night, and this has been occurring for the 66. A middle-aged man with depression requests last 3 weeks. She finds herself quite tired and “blue” help for his symptoms of low self-esteem and during the day but is unable to nap. Mornings are feelings that “life is bad no matter what you “the worst” for her, but she feels better toward the do.” He prefers to use no medication and end of the day. There has been a 15-lb weight loss expresses the desire to not be in therapy “for because “I’m just not hungry.” She denies any phys- years.” There is no previous psychiatric treat- ical problems except for constipation. As a grade- ment. Which of the following therapies would school teacher, she feels extremely stressed but sees be the most helpful? no way out and no way to improve the situation. At (A) psychoanalysis times, suicide seems like a possible option, and she admits to spending long hours brooding on how to (B) behavioral therapy do it. A physical examination is unremarkable. (C) cognitive psychotherapy (D) supportive psychotherapy 62. Which of the following is the most likely (E) group psychotherapy diagnosis? (A) borderline personality disorder 67. A 35-year-old woman is seen by her primary care physician for a physical examination. She (B) major depression tells him she has a twin brother who has bipo- (C) dysthymia lar disorder and has been worried that she will (D) Alzheimer’s disease develop it. Which of the following would be (E) generalized anxiety disorder most helpful for her to hear? (A) “You’re past the age when bipolar disor- 63. Which of the following is the most appropriate der develops, so don’t worry about it.” treatment choice? (B) “There is no clear evidence that a bipo- (A) olanzapine lar disorder is genetically determined.” (B) paroxetine (C) “The concordance rate for bipolar disor- (C) alprazolam der for dizygotic twins is 19%.” (D) tranylcypromine (D) “The concordance rate for bipolar disor- (E) ECT der for dizygotic twins is 79%.” (E) “Prophylactic treatment with lithium is 64. Which of the following is considered a negative advisable.” symptom of schizophrenia? 68. Of the following, which is considered a cortical (A) anhedonia dementia? (B) loose associations (C) delusions of thought insertion (A) Huntington’s disease (D) incoherence (B) Pick’s disease (E) stereotypic gestures (C) Parkinson’s disease (D) occult hydrocephalus (normal pressure) 65. Which of the following medications may be (E) none of the above appropriate for treating children with atten- tion deficit disorder? (c) ketabton.com: The Digital Library

176 5: Psychiatry

69. A 42-year-old married woman reports being 71. An 11-year-old girl has become uncharacteris- raped in an elevator 1 year ago. Her arms were tically and markedly withdrawn in the past 8 fractured in the assault but have healed nicely. months, staying in her room so that she can Still she reports difficulty sleeping, having “talk to the ghosts in the attic.” nightmares of the attack several times per week ever since the assault. She avoids using the ele- 72. An 11-year-old girl has become markedly with- vator and does not want to talk about the inci- drawn in the past 8 months and has com- dent with anyone. She has been unable to plained of persisting abdominal pain and return to work. Her husband feels she has been constipation, for which no organic cause has hypervigilant and irritable, and has been resis- been found. tive to going out socially. She has a depressed mood. Her husband is encouraging her to seek 73. A 5-year-old boy is reported by his kindergarten disability. teacher to be easily distracted, impulsive, in need Which of the following is the most likely of continual supervision, but not hyperactive. diagnosis? 74. A 3-year-old boy spends hours rocking in a (A) major depression chair or spinning the blades of a toy windmill; (B) adjustment disorder with anxious and his parents say he never cries when he falls. depressed features (C) acute stress disorder Questions 75 through 79 (D) malingering For each of the case vignettes below, identify the (E) PTSD diagnosis that best describes the situation. DIRECTIONS (Questions 70 through 111): Each (A) major depressive disorder, recurrent set of matching questions in this section consists (B) bipolar I disorder of a list of lettered options followed by several (C) bipolar II disorder numbered items. For each item, select the ONE (D) cyclothymia best lettered option that is most closely associated with it. Each lettered option may be selected once, (E) dysthymic disorder more than once, or not at all. (F) mood disorder due to a general medical condition (G) substance-induced mood disorder Questions 70 through 74

For each clinical description that follows, select the 75. A 35-year-old man comes into the hospital with diagnosis with which it is most likely to be associated. deep sadness and suicidal thoughts. He reports having experienced similar episodes in the (A) childhood depression past; he says he has also experienced two peri- (B) childhood schizophrenia ods of extremely elated moods, during which (C) conduct disorder he talked nonstop, went without sleep, and (D) ADHD believed he was God. (E) infantile autism 76. A 45-year-old woman comes to your office saying that she has once again got into a deep 70. A 9-year-old boy has had persisting difficul- funk, losing sleep and weight, and feeling she ties in language and interpersonal relationships is worthless. She reports this is the third time in since the age of 2 years, and, although he can her life that she has experienced such episodes, barely read, he is able to perform arithmetic but she has never had periods of abnormally calculations at the fifth-grade level. elevated moods. (c) ketabton.com: The Digital Library

Questions: 69–83 177

77. A 24-year-old cocaine addict with no previous covered all electrical appliances and outlets psychiatric history, reports that he had been with duct tape claiming that “electromagnetic bingeing on cocaine for 2 months earlier in the waves are disturbing the microchip in my year. He stopped using cocaine 2 months ago, brain.” He acknowledges that he once took a and he has been very sad and tearful for the medication for the same problem with the entire period since he stopped his cocaine use. “microchip,” but then he became unbearably restless, could not sit still, and felt that he was 78. A 32-year-old woman reports that she has been “crawling” inside. in a low-grade, chronically depressed mood for more days than not for the past 3 years. She 81. A 36-year-old man complains that he cannot reports that she has never had a period of sleep at night. “I just can’t settle down; my severely depressed mood, but the low-grade mind is constantly working.” He describes feeling of sadness has not gone away for more being able to complete a lot of work, being than 2 or 3 days at a time over the past 3 years. highly energized, and having a lot of fun. He is afraid that without sleep he will “crash.” Once 79. A 53-year-old man reports that he is currently before, he took a medication for depression. in one of his “up” periods. He states that he He requests medication at this time. feels very good, and he is quite talkative, but he reports that he is having no trouble keeping 82. A 65-year-old woman with a history of cardiac his job, and he is sleeping well. He has never problems complains that she has lost her had any periods of elevated mood that have appetite, she cries frequently, and has lost inter- substantially interfered with his working, and est in her grandchildren, her gardening, and her he has never needed hospitalization for his craft making. She thinks maybe it is time to die. “up” periods. However, he has suffered two periods of severe depression, each requiring 83. A 50-year-old woman with a long history of hospitalization for suicidal thoughts, the most taking trifluoperazine is noted to have repeti- recent having occurred 3 years ago. tive chewing motions, and periodically pro- trudes her tongue. Her arms and shoulders Questions 80 through 83 seem to jerk fairly often, and there is a peculiar twisting movement in her right hand. She tried For each patient’s psychiatric symptoms, select the several of the “newer” medications that are not most appropriate medication. Presume no medical supposed to cause the movement problems, problems other than those mentioned. but then her auditory hallucinations started (A) amitriptyline again. (B) clozapine Questions 84 through 88 (C) divalproex (D) fluoxetine Match the antidepressants below with the effect (E) hypericum perforatum described. (F) olanzapine (A) amitriptyline (G) lorazepam (B) nefazodone (H) propranolol (C) citalopram (I) temazepam (D) phenelzine (J) thiothixene (E) duloxetine

80. A 23-year-old university student has lost inter- est in his master’s degree program. His two friends who bring him to the psychiatrist say he has been isolating himself in his room and has (c) ketabton.com: The Digital Library

178 5: Psychiatry

84. Primarily a selective serotonin reuptake 93. A 45-year-old woman reports a 6-month his- inhibitor (SSRI) tory of free-floating anxiety, which makes her feel tense most of the time on most days.

85. Both a SSRI and serotonin type 2 (5-HT2) recep- tor blockade 94. A 7-year-old boy presents with his mother who reports he cannot pay attention and is having 86. Strong sedation, strong serotonin effect, and severe problems at school because of this and norepinephrine effect his hyperactivity.

87. Little sedation, strong serotonin, and norepi- Questions 95 through 99 nephrine effect Identify the following personality disorders with 88. Monoamine oxidase (MAO) inhibitory effect the symptoms listed below. (A) antisocial Questions 89 through 94 (B) avoidant For each clinical vignette below, select the pharma- (C) borderline cotherapeutic agent which is most likely to be help- (D) dependent ful to the patient. (E) histrionic (A) buspirone (F) narcissistic (B) sertraline (G) obsessive-compulsive (C) risperidone (H) paranoid (D) lithium carbonate (I) schizoid (E) acamprosate (J) schizotypal (F) dextroamphetamine 95. Is quick to perceive a slight as an attack or 89. A 35-year-old man has a history of alternating assault on one’s character episodes of major depression and severe mania, several of which have required hospi- 96. Seems to not care what others think or feel; is talization. aloof

90. A 40-year-old man has a history of severe alco- 97. Preoccupied with feelings of superstitiousness; hol dependence and is very interested in what- has a sixth sense; seems odd ever help he can get to avoid relapsing in his alcohol use. 98. Chronic feelings of emptiness, fear of aban- donment; unstable self-image 91. A 21-year-old male college student reports a 1- month period of intensely depressed mood, 99. Unwilling to take personal risks; perceives self with marked anxiety, sleep disturbance, and as inept, unappealing, inferior weight loss. He has never before experienced an episode of a mood disorder.

92. A 41-year-old woman has a long history of chronic paranoid schizophrenia. She experi- ences auditory and visual hallucinations and has had difficulty maintaining a job. (c) ketabton.com: The Digital Library

Questions: 84–111 179

Questions 100 through 104 105. An SSRI with a half-life of 4–6 days

Identify the following defense mechanisms with the 106. Need to avoid tyramine with this medication descriptions below. (A) acting out Questions 107 through 110 (B) altruism Match the medication with the potential blood (C) displacement dyscrasia side effect it can be associated with. (D) intellectualization (A) leukocytosis (E) passive-aggressive behavior (B) thrombocytopenia (F) projection (C) agranulocytosis (G) rationalization (D) megaloblastic anemia (H) reaction formation (E) lymphocytosis (I) sublimation (J) suppression 107. Valproate

100. Becoming angry with one’s spouse at a party 108. Lithium and deciding to wait until a more appropriate time to express it 109. Clozaril

101. Becoming angry with one’s spouse at a party 110. Carbamazepine and calling him or her a name in front of everyone 111. A 77-year-old male presents, accompanied by his wife to your clinic. She reports that the 102. Working as a nurse in a pediatric ward because patient has been having visual hallucinations of one would like to have children but cannot little people in his room on and off for the past 3 months. Six months ago he developed a 103. Being afraid of one’s rage and anger and pre- tremor and gait disturbance. Over this past senting as unusually meek and mild year he has become more forgetful, and has had episodes of confusion. 104. Accusing another of being angry and jealous The most likely diagnosis is: when the feelings belong to oneself (A) Alzheimer dementia Questions 105 and 106 (B) vascular dementia (C) Parkinsion’s disease with dementia Match the antidepressant with the side effect or (D) Lewy body dementia characteristic described. (E) Frontotemporal dementia (A) phenelzine (B) venlafaxine (C) trazodone (D) fluoxetine (E) mirtazapine (F) nortriptyline (G) escitalopram (c) ketabton.com: The Digital Library

Answers and Explanations

1. (D) patients often report decreased libido, sleep disturbance, and appetite disturbance. It would 2. (A) be important to rule out organic causes for this man’s disorder, such as hypothyroidism or the 3. (B) relatively rare phenomenon of metastatic cancer to the brain in a 35-year-old. Given the Explanations 1 through 3 patient’s age and recent onset of symptoms, Alzheimer’s disease is unlikely. Schizophrenia Typical neuroleptic antipsychotic medications generally has a gradual onset and is accompa- frequently cause unpleasant side effects, which nied by psychotic symptoms such as halluci- occur at various times during treatment. The nations and delusions. (Kaplan and Sadock, 2003, extrapyramidal side effect most likely to occur in pp. 542–544, 556–557) the first few days of treatment is an acute dys- tonia, such as a muscle spasm in the neck. A 5. (D) The most appropriate initial therapy for pill-rolling tremor of the hands and masked this man would be antidepressant medication facies are signs of Parkinsonian EPS, which tend with a medication such as fluoxetine, an SSRI to have their onset several weeks after treatment with a relatively benign side effect profile in is begun; whereas severe restlessness of the arms most patients. SSRIs produce antidepressant and legs is caused by an unpleasant sensation response rates approaching 70%. The addition called “akathisia,” which also tends to have its of psychotherapy to help the patient get onset several weeks after treatment is begun. through his depression would likely be helpful. Involuntary lip smacking is a sign of tardive Although ECT is the most dependably effective dyskinesia, a sometimes irreversible motor syn- treatment for major depression, it has signifi- drome that tends to occur after months or years cant side effects, such as short-term memory of treatment with typical antipsychotics. loss. The absence of severe suicidal tendencies Anticholinergic medications such as ben- in this patient means that ECT should be ztropine are effective in treating dystonias and reserved for possible later use, should two dif- pill-rolling tremors. Propranolol is effective in ferent antidepressant medications fail. treating akathisia. Haldoperidol would worsen Clozapine, diazepam, and buspirone do little EPS. Dantrolene uncouples muscle contrac- by themselves to alleviate a depressive mood. tions and is used occasionally in severe NMS. (Kaplan and Sadock, 2003, pp. 560–570) L-Dopa would decrease the Parkinsonian EPS, but would worsen the psychosis and therefore 6. (D) is not used. Fluoxetine is an SSRI. (Kaplan and Sadock, 2003, pp. 1057–1061) 7. (B)

4. (C) The essential feature of a major depressive 8. (H) disorder is the development of a major depres- sive episode without a history of mania or 9. (E) hypomania. The hallmarks of a major depres- sive episode are a subjective sense of dyspho- 10. (G) ria and a loss of interest in previously enjoyed activities, also called anhedonia. Depressed

180 (c) ketabton.com: The Digital Library

Answers: 1–13 181

Explanations 6 through 10 the deliberate production of signs and symp- toms of illness in order to assume the sick role. Complaints involving both psychological and (Kaplan and Sadock, 2003, pp. 651–653, 647–651, medical conditions are difficult to diagnose and 643–647, 599–602, 897–898) treat. At times, medically identifiable causes are present, but the psychological factors con- 11. (D) Acute confusion with visual hallucina- tributing to the discomfort complicate the diag- tions would be characteristic of a delirium. nosis and treatment and lead to frustration in Warm and dry skin, tachycardia, dry mouth, both physician and patient. At other times, no constipation, and urinary retention are anti- identifiable cause for pain or other physical cholinergic side effects. Many antipsychotics symptoms can be found; nevertheless, the are anticholinergic, and when Parkinsonian patient still has the symptoms. Questions arise: EPS present, anticholinergic medicines are Is the patient lying? Is there some deep psy- often added to reverse these side effects. These chological problem? Could there be a medical anticholinergic effects can be additive and can disorder in the early stages of development cause delirium. (Kaplan and Sadock, 2003, p. 878, that gives rise to physical symptoms but no 946, 981) clear physical signs to make the diagnosis? One group of psychiatric disorders addresses some 12. (C) Severe anticholinergic reactions, such as of these issues—the somatoform disorders. delirium, should be treated with intramuscu- Somatization disorder is an axis I psychiatric lar or intravenous injection of physostigmine, disorder where a patient complains of multiple 1–2 mg IV (1 mg every 2 minutes) or IM every somatic complaints involving multiple organ 30–60 minutes. The first dose should be systems, but which cannot be explained by repeated in 15–20 minutes if no improvement physical and/or laboratory findings. Conver- is seen. Such peripheral anticholinergic side sion disorder is another axis I disorder where effects as urinary retention can be treated there is a disturbance of bodily function that with bethanechol. Dantrolene and bromocrip- does not conform to anatomic or neurologic tine, not effective in the treatment of anti- concepts, and it is due to psychological factors. cholinergic reactions, are two drugs that have Body dysmorphic disorder is a pervasive sub- been tried in the treatment of NMS, a rare but jective feeling/belief that some aspect of the extremely dangerous neuroleptic-induced patient’s appearance is ugly or deformed. disorder. Haldol, having anticholinergic Hypochondriasis is a persistent belief in the effects, is contraindicated. (Kaplan and Sadock, presence of one or two serious physical dis- 2003, p. 1014, 1060) eases despite medical assurance that one does not have the disease/illness. Pain disorder 13. (C) Frotteurism is the term given to obtaining (somatoform pain disorder) is a disorder with sexual gratification by rubbing. It is usually the the presence of severe, distressing and persist- only source of sexual gratification for the man ent pain which cannot be explained adequately involved, who rubs his penis against the but- by evidence of a physiologic process or physi- tocks or other bodily part of a fully clothed cal disorder. woman to achieve orgasm. Exhibitionism is the Panic disorder is a kind of anxiety disor- exposure of one’s genitals to an unsuspecting der. Discrete periods of extreme sympathetic person. Fetishism is the term for intense sexual nervous system symptoms occur, including fantasies and behaviors involving the use of tachycardia, sweating, shortness of breath, nonliving objects, such as female undergar- and others, during which time a person expe- ments. Pedophilia is the term for intense sexual riences extreme fear. Malingering is the delib- urges toward children 13 years of age or erate manufacture of false or exaggerated younger, and sexual masochism is the term for symptoms for financial gain or to avoid an sexual arousal involving the real act of being unpleasant situation such as jail time or humiliated, beaten, bound, or otherwise made military duty. In a factitious disorder, there is to suffer. (Kaplan and Sadock, 2003, p. 721) (c) ketabton.com: The Digital Library

182 5: Psychiatry

14. (C) Delusions are false ideas that cannot be cor- psychosis), then treatment would include rected by reasoning and that are not based on appropriate drugs or ECT. In some cases, there reality. Psychotic patients often experience is a role for hypnosis or sodium amobarbital ideas or delusions of reference and misinterpret interview to help recall. Benzodiazepines may incidents or events in the outside world as be helpful to reduce anxiety. Integration having direct personal reference to themselves. through psychotherapy of the events of the Delusions may occur in a variety of psychiatric traumatic episode into one’s conscious state is disorders, including schizophrenia, paranoia, important for recovery. (Kaplan and Sadock, 2003, mania, depression, and organic brain syn- pp. 676–679) dromes. The bizarre nature of the delusion described in the question is more characteristic 17. (A) The preoccupation with a rather well- of schizophrenia than of other types of psychi- developed delusional system and later age at atric ailments. Illusions are sensory misper- onset suggest paranoid schizophrenia. A case ceptions that occasionally may be experienced can be made for undifferentiated schizophrenia even by normal individuals. Psychotic persons because of the apparent disorganization in per- may report hallucinations, which are sensory sonal habits and the flattening of affect. There experiences that cannot be substantiated by is no history of alcohol abuse and dependence normal observers. Loosened associations and to support the diagnosis. The long period of neologisms are patterns of speech often noted symptoms, bizarreness of paranoid delusion, in psychotic individuals. (Kaplan and Sadock, 2003, and decline in functioning are more character- pp. 283–284) istic of schizophrenia. The time course of a major depression is much shorter. Usually, in 15. (C) Dissociative amnesia is loss of ability to major depression there is not the profound recall information occurring within a certain decline in functioning. No symptoms of time period, usually related to a severely stress- memory impairment or loss of cognitive func- ful event as occurred with this woman. In tioning have occurred that would suggest depersonalization disorder, a person feels Alzheimer or Huntington dementias. In addi- detached from his or her own body or mental tion, in Huntington dementia, one would processes and feels as if he or she is standing expect a prominent movement disorder as seen apart and acting as observer. In PTSD, there is in subcortical dementia. (Kaplan and Sadock, 2003, a persistent re-experiencing of the traumatic pp. 471–504, 331–333, 333–334) event in a variety of ways, persistent avoid- ance of stimuli associated with that event, and 18. (B) Features that weigh toward a good progno- persistent symptoms of arousal. It is possible sis in schizophrenia include late onset, obvious for the arousal to be a part of this larger symp- precipitating factors, good premorbid func- tom complex. In dissociative fugue, the person tioning, mood disorder symptoms, being mar- forgets his identity, travels, and may even ried, a family history of mood disorders, good establish a new identity. Symptoms of major support systems, and having primarily posi- depression involve mood, inability to experi- tive symptoms. (Kaplan and Sadock, 2003, p. 485) ence pleasure, appetite and sleep disturbance, fatigue, and other disturbances of affect and 19. (D) The dopamine hypothesis of schizophrenia mood. (Kaplan and Sadock, 2003, pp. 676–679) grew from the observations that medications that block dopamine receptors have antipsy- 16. (B) Individual therapy, which would include chotic activity and medications that stimulate exploring the events recalled surrounding the dopamine receptors (amphetamines) can induce incident, reactions to the child’s death, feelings psychosis. Serotonin abnormalities have been about motherhood, as well as other issues, is implicated in mood and anxiety disorders. The considered the most effective treatment for biogenic amine hypothesis of mood disorders dissociative amnesia. If other symptoms indi- was based on the finding that tricyclic and cate another disorder (e.g., major depression, MAOI drugs are effective in alleviating the (c) ketabton.com: The Digital Library

Answers: 14–26 183

symptoms of depression. The GABAergic implicated in anorexia nervosa, but the etiology system has been implicated in anxiety disorders has yet to be clearly established. Most com- because benzodiazepines which are GABAergic monly, the course consists of a single episode have antianxiety effects. Acetylcholine abnor- followed by remission. Some patients may malities have been associated with dementia. suffer a series of relapses and remissions. (Kaplan and Sadock, 2003, pp. 97–106) Mortality rates have been estimated to be as high as 20%. (Kaplan and Sadock, 2003, pp. 739–746) 20. (B) The treatment of choice for patients with a schizophrenic disorder is an antipsychotic 22. (E) Generally speaking, anorexic patients who drug. Of the medications listed, only olanzap- are 20% or more below their expected weight ine is an antipsychotic drug. Olanzapine causes for their height are recommended for inpatient relatively fewer EPS than traditional neurolep- programs. Inpatient programs for anorexia ner- tics, such as haloperidol, and seems to be sub- vosa usually use a combination of a behavioral stantially less likely to cause tardive dyskinesia management approach, individual psychother- than typical neuroleptics. Olanzapine blocks apy, family education and therapy, and in some

both dopamine D2 receptors and serotonin 5- cases psychotropic medications. (Kaplan and HT2 receptors. Its use is often associated with Sadock, 2003, p. 744) some weight gain by patients. (Kaplan and Sadock, 2003, p. 498) 23. (A) Alzheimer’s disease is the most common form of dementia. Of persons with dementia, 21. (C) Anorexia nervosa is an eating disorder that 50–60% will have Alzheimer’s. Vascular demen- predominantly affects women in their teens tia is the second most common, accounting for and in early adulthood. It is defined as refusal about 15–30% of dementias. (Kaplan and Sadock, to maintain a minimal normal weight, at least 2003, pp. 328–329) 85% of that weight considered normal for that person’s age and height, and a morbid preoc- 24. (A) Donepezil is a cholinesterase inhibitor used cupation with feeling obese. Common strate- for the treatment of mild-to-moderate impair- gies to lose weight include avoidance of all fats ment in Alzheimer’s disease. Fluoxetine is an and carbohydrates, self-induced vomiting, SSRI antidepressant. Aspirin would decrease obsessive physical activity, and abuse of laxa- clotting. Amitriptyline is a tricyclic antidepres- tives or diuretics or both. Despite apparent sant and with its anticholinergic properties, it aversion to gaining weight, anorectics fre- would worsen cognition. Ginkgo is an herbal quently take very special care in preparation medicine. (Kaplan and Sadock, 2003, p. 1041) and consumption of food and may delight in preparing gourmet feasts for others. Menstrual 25. (D) Pathologic changes will be seen in the fron- irregularity and amenorrhea are also com- totemporal cortex in patients with Pick’s dis- monly reported but are not essential factors in ease. Alzheimer’s disease, also a cortical making the diagnosis. It is not yet clear dementia like Pick’s, has pathologic changes whether such menstrual problems are simply in the parietotemporal areas. (Kaplan and Sadock, secondary to starvation or whether they reflect 2003, pp. 331–333) a more pervasive endocrine dysfunction. Perhaps the most striking clinical feature of 26. (B) Panic attacks most frequently have their this disorder is the misperception of body onset sometime between the late teens and image. Regardless of the method of confronta- early thirties. Most patients describe their ini- tion, including use of mirrors or photographs, tial panic attack as coming out of the blue. They sufferers see themselves as overweight. The frequently suffer from palpitations, sweating, patient often refuses to agree that there is any trembling, shortness of breath, a feeling of problem whatsoever. Numerous factors, choking, chest pain, nausea, light headedness, including developmental, family, endocrine, derealization (feelings of unreality) or deper- and gastrointestinal disturbances have been sonalization (being detached from oneself), fear (c) ketabton.com: The Digital Library

184 5: Psychiatry

of going crazy, fear of dying, numbness, and situations in which embarrassment can occur. hot flashes or chills. Individuals who suffer Specific (simple) phobias are among the most from recurrent, unexpected panic attacks with common mental disorders and involve an at least 1 month of persistent concern about excessive fear of an object or specific situation the attacks or their implications, or a significant which produces an avoidance of the object or change of behavior related to the attacks suffer situation. (Stern and Herman, 2000, p. 123) from panic disorder. If a patient with panic dis- order avoids situations in which panic attacks 29. (C) Lorazepam, because of its relatively short have occurred or endures the situations, such half-life, intermediate rate of onset, and absence as shopping in a mall, with marked distress or of active metabolites, would be an appropriate anxiety about having a panic attack, the patient medication for this elderly woman. Diazepam is said to suffer from panic disorder with ago- and clorazepate both have long half-lives as raphobia. This patient’s negative ECG makes well as active metabolites. These properties myocardial infarction an unlikely diagnosis. lead to more severe side effects in older Hypochondriasis is the term given to a long- patients, such as prolonged sedation, respira- term preoccupation with fears of having a seri- tory depression, confusional states, and dis- ous disease based on the misinterpretation of orientation. Temazepam, although it has no bodily symptoms; it does not generally present active metabolites and a relatively short half- in the sudden manner in which panic attack life, is very sedating and is used to promote presents. Multiple sclerosis usually causes sleep. Buspirone, a nonbenzodiazepine, is an many neurologic disturbances which vary effective antianxiety agent, but it may take up across time. Generalized anxiety disorder is to a week to exert its effect. The woman in this not typified by the sudden onset of severe anx- case needs medication that will help her iety characteristic of a panic attack. (Kaplan and quickly. (Kaplan and Sadock, 2003, pp. 1022–1029, Sadock, 2003, pp. 599–602, 651–653) 1031–1033)

27. (A) If the patient continues to suffer panic 30. (C) Patients with severe psychotic depression attacks for a substantial period, he is said to be will often have delusions that are mood con- suffering from panic disorder. SSRIs, such as gruent and reflect the depth of their despair paroxetine, are frequently effective and gener- and self-abhorrence. Patients with mania are ally safe drugs for the treatment of panic more likely to have delusions that are mood disorder. Despite its usefulness in treating gen- congruent that would reflect their grandiosity, eralized anxiety disorder, buspirone seems to paranoid feelings, inflated self-esteem, and be ineffective in treating panic. Beta-blockers feelings of having special powers. (Kaplan and such as propranolol may block symptoms of Sadock, 2003, pp. 542–544) palpitations or tremor, but are generally not as effective against panic attacks as are SSRIs. 31. (A) Schizoid personality disorder is character- Antipsychotic medications such as ziprasidone ized by a pattern of pervasive social detach- and haloperidol are not appropriate medica- ment with a narrow range of emotional tions for the first-line treatment of uncompli- expression. Schizoid persons seem to be fairly cated panic disorder. (Kaplan and Sadock, 2003, content with a lack of intimacy and are pp. 1126–1127) considered odd by persons around them. This contrasts with the picture of individuals with 28. (D) Agoraphobia is most often associated with avoidant personality disorder, who long for panic disorder. It involves a severe fear of and social interaction, but feel inadequate and avoidance of entering social situations where riddled with self-doubt in social situations. it is difficult to get help or escape from. Like individuals with obsessive-compulsive Arachnophobia is the phobia of spiders, claus- disorder, people with avoidant personality trophobia is a phobia of closed spaces. Social disorder are categorized as having a cluster C phobia is an excessive, persisting fear of social (anxious, fearful) personality disorder; whereas, (c) ketabton.com: The Digital Library

Answers: 27–37 185

those with borderline personality disorder or 35. (D) The hallmark of delirium is fluctuation in antisocial personality disorder have problems level of consciousness. Periods of lucency may characterized by engaging in impulsive behav- be interspersed with periods of clouding iors and are classified as having a cluster B and unresponsiveness. Impaired judgment, (dramatic, impulsive) personality disorder. impaired memory, and disorientation are seen (Kaplan and Sadock, 2003, pp. 800–821) in both delirium and dementia. Disordered thought is seen in both and tends to be disor- 32. (B) Separation anxiety disorder is the name ganized in delirium and impoverished in given to the problem of a child’s developmen- dementia. Another distinguishing feature is tally excessive anxiety concerning separation that the onset of delirium usually occurs from home and parents. Children who are within hours or days, whereas, the onset of more globally anxious may suffer from gener- dementia may be insidious throughout a alized anxiety disorder, while children who period of weeks to months. (Kaplan and Sadock, avoid playmates may suffer from social phobia. 2003, pp. 327–328) Children who have an unrealistic fear of, for example, dogs, have a specific phobia; whereas, 36. (C) Advanced age is a major risk factor for the those who suffer from intrusive anxious development of delirium. Other predisposing thoughts that lead them to spend much time risk factors include preexisting brain damage, a performing rituals such as hand washing suffer history of previous delirium, alcohol depend- from obsessive-compulsive disorder. (Kaplan and ence, diabetes, cancer, dementia, sensory impair- Sadock, 2003, pp. 1259–1265) ment, and malnutrition. Male gender is also an independent risk factor. (Kaplan and Sadock, 2003, 33. (B) Children with Asperger disorder show severe pp. 323–324) sustained impairment in social interaction, and restricted repetitive patterns of behavior. Unlike 37. (B) According to the case study, this man can be autistic disorder, in Asperger disorder language presumed to have been without alcohol for at and cognitive development are not delayed. In least the 2 days he has been in jail, and, therefore, Rett disorder, a child shows deterioration of the emerging symptoms would be most attrib- developmental milestones, head circumference, utable to withdrawal and delirium. Withdrawal and overall growth. In Rett disorder, verbal abil- symptoms include autonomic hyperactivity, ities are usually lost completely. (Kaplan and Sadock, hand tremor, hallucinations, illusions, and agi- 2003, pp. 1218–1219) tation. Added to these are the signs of delirium, disorientation, irritability, agitation, disorgan- 34. (B) All of the listed syndromes cause mental ized thought, inability to focus and concen- retardation, but Prader-Willi syndrome is also trate, and development of symptoms over a typified by compulsive eating and obesity, short time. A diagnosis of exacerbation of schiz- hypogonadism, and small stature. It is thought ophrenia does not adequately explain these to be due to a small deletion on chromosome symptoms and their development shortly after 15. Phenylketonuria is an autosomal-recessive the withdrawal of alcohol. The signs and symp- trait which can cause mental retardation in chil- toms are not consistent with dementia, in dren who do not eat a low phenylalanine diet. which one would expect a longer period for Down syndrome, or trisomy 21, causes mental symptom development and a clouding of con- retardation and distinctive facies. Fragile X syn- sciousness. Although malingering might be drome causes mental retardation, short stature, considered, especially for an incarcerated indi- and postpubertal macroorchidism. Cri-du-chat vidual, the history of recent alcohol intoxication syndrome, caused by a deletion on chromosome with abrupt withdrawal makes alcohol with- 5, is characterized by severe mental retardation, drawal delirium a more consistent diagnosis. microcephaly, and a cat-like cry in infants due to (American Psychiatric Association (APA), 2000, pp. laryngeal abnormalities. (Kaplan and Sadock, 2003, 136–147, 739; Kaplan and Sadock, 2003, pp. 319–328) pp. 1165–1167) (c) ketabton.com: The Digital Library

186 5: Psychiatry

38. (C) Diazepam, chlordiazepoxide, and lorazepam 42. (A) Physically holding a patient would be a can all be given parenterally, but only lorazepam boundary violation. Conveying empathic val- is reliably absorbed after intramuscular injec- idation, having clear roles and responsibilities, tion. Alprazolam and clonazepam are not avail- being flexible, and developing a hierarchy of able in parenteral form in the United States. priorities are all common features of recom- (Kaplan and Sadock, 2003, pp. 403–406) mended psychotherapy for a patient with bor- derline personality disorder. (Kaplan and Sadock, 39. (E) Mirtazapine is a tetracyclic antidepressant 2003, pp. 808–810) that has both presynaptic alpha-2 receptor

noradrenergic antagonist properties and 5-HT2 43. (A) NMS is a rare complication of neuroleptic and 5-HT3 receptor serotonergic antagonist therapy which involves symptoms of severe properties, which, taken together, result in a muscle rigidity, elevated temperature, and two net increase in both noradrenergic and sero- or more of the following: diaphoresis, dyspha- tonergic neurotransmission. Bupropion may gia, tremor, incontinence, changes in level of exert its antidepressant effects by acting on the consciousness, tachycardia, mutism, leukocy- noradrenergic system, but its method of action tosis, elevated CPK, or labile blood pressure. is currently being investigated. Trazodone and EPS would not explain the extensor rigidity, nefazodone are combined serotonin reuptake elevated temperature, and blood pressure.

inhibitors and 5-HT2 serotonin antagonists. Metabolic syndrome refers to another neu- Venlafaxine is a combined norepinephrine and roleptc side effect where lipid levels are ele- serotonin reuptake inhibitor. (Kaplan and Sadock, vated and adult onset diabetes can develop. 2003, pp. 1030, 1075, 1080, 1123–1124, 1135–1136) Malingering would not easily explain the ele- vated vital signs. Though alcohol withdrawal 40. (B) Dissociative fugue is classically typified by would be in the differential with the elevated a person’s suddenly and unexpectedly travel- vital signs, the pipe-like rigidity could not be ing away from his or her home, assuming a explained. (Stern and Herman, 2000, p. 220) new identity, and eventually recovering from the fugue or flight, unable to recall the events 44. (A) Patients with NMS typically demonstrate that took place during the episode. Dissociative tachycardia, labile blood pressure, severe amnesia is a more generalized term given to an muscle rigidity, and severe fever. Serum CPK inability to recall significant personal infor- levels, which can be elevated to more than 100 mation. Dissociative identity disorder is the times normal levels as a result of muscle current diagnostic term for what was classi- damage, are the most consistently noted cally called multiple personality disorder. abnormalities in patients with NMS. Although Depersonalization disorder is an alteration of the white blood cell count may be elevated, experience in which an individual feels like an and blood levels of calcium, iron, and magne- outside observer of his or her body or mental sium may be decreased in patients with the processes. Dissociative disorder not otherwise syndrome, serum CPK is the most important specified is a name given to other dissociative laboratory study to obtain in a patient with illnesses not specifically listed in DSM-IV-TR, suspected NMS. (Bernstein et al., 1997, pp. 130–133; such as dissociative states occurring in indi- Kaplan and Sadock, 2003, pp. 993–994) viduals subjected to brainwashing or indoctri- nation while held captive by terrorists. (Kaplan 45. (B) The most effective treatments for the and Sadock, 2003, pp. 679–685) extremely serious, potentially fatal complica- tion of antipsychotic treatment called NMS 41. (B) Recurrent suicidal behaviors, affective are oral bromocriptine, a dopaminergic ago- instability, unstable interpersonal relationships, nist, and intravenous or oral dantrolene, a chronic feelings of emptiness, and rejection sen- skeletal muscle relaxant. A further increase in sitivity are all traits of borderline personality the patient’s haloperidol dose would likely disorder. (Kaplan and Sadock, 2003, pp. 808–810) worsen her NMS. While the anticholinergic (c) ketabton.com: The Digital Library

Answers: 38–53 187

effects benztropine may alleviate some of the and Protestants. Being divorced, being male, neuroleptic-induced muscular dystonia asso- and having a previous suicide attempt all ciated with the syndrome, and lorazepam may increase the risk for suicide. Up to 10% of schiz- help relax the muscle rigidity, they are not ophrenics die from suicide. (Kaplan and Sadock, likely to be life saving, and propranolol is not 2003, pp. 913–915) an effective medication in the treatment of NMS. (Bernstein et al., 1997, pp. 130–134; Kaplan and 50. (E) Most suicides among psychiatric patients Sadock, 2003, pp. 993–994) are felt to be preventable, as supported by the evidence that inadequate assessment or treat- 46. (E) Narcolepsy is a rare dyssomnia (incidence ment is often associated with suicide. When to of .07%) defined by the following four symp- hospitalize patients with suicidal ideation is toms: sleep paralysis occurs upon falling asleep the most important clinical decision to be or waking, sleep attacks with sleep onset REM made. The absence of a strong support system, which are brief (10–15 minutes) and occur in a history of past suicide attempt and impul- inappropriate situations (sleep attacks are effec- sivity, and having a suicidal plan with intent tively treated with stimulants), cataplexy which would be indications for hospitalization. (Kaplan is a condition that involves sudden transient and Sadock, 2003, p. 920) bilateral weakness or paralysis, and hypna- gogic hallucinations. Apnea is the cessation of 51. (D) nasobuccal breathing for more than 10 seconds and is found in obstructive sleep apnea, central 52. (C) sleep apnea, and mixed sleep apnea. (Stern and Herman, 2000, pp. 175–176) 53. (E)

47. (C) The onset of severe, persistent eye pain is Explanations 51 through 53 always a cause for concern. In a patient med- icated with a drug with anticholinergic side Hypochondriasis is a somatoform disorder in effects, such as imipramine, there is a potential which misperceptions or distortions of somatic for the development of narrow-angle glau- signs and symptoms lead to preoccupation with coma. A delay in the diagnosis and treatment of fears of having a serious illness. In factitious this will lead to irreparable harm to the eye. In disorders, one deliberately manufactures signs this case, the psychiatrist would act immedi- and symptoms to enter the sick role. The pre- ately to facilitate appropriate evaluation and occupation with fear of serious illness is not treatment which would best be provided by part of factitious disorder. Major depression is characterized by symptoms of depression: sleep her ophthalmologist. (Bernstein et al., 1997, p. 170, 174; Kaplan and Sadock, 2003, p. 1014) disturbance, appetite disturbance, and so forth. It may be complicated by hypochondriasis. In 48. (B) Aripiprazole, clozapine, risperidone, queti- the case study, no supporting evidence for apine, and ziprasidone are all atypical antipsy- major depression (for which she would have chotic medications. The use of clozapine, been evaluated) is provided. This woman’s however, is limited because of the risk of poten- symptoms as described are not of a psychotic tially fatal agranulocytosis in patients taking it. level; thus, reactive psychosis would be inap- Because the agranulocytosis is reversible, mon- propriate. In pain disorder, pain in a specific itoring the blood count of patients on clozapine body site is the predominant focus, unlike the is recommended, usually starting on a weekly predominance of fear seen in hypochondriasis. Care of these patients is best managed suppor- basis at the beginning of treatment. (Kaplan and Sadock, 2003, pp. 1104–1113) tively by developing a therapeutic alliance with them. Anticipating their needs by establishing 49. (A) Historically, suicide rates among Roman regular office visits and physical examinations Catholics have been lower than among Jews with them will help allay fears as well as reassure (c) ketabton.com: The Digital Library

188 5: Psychiatry

them of one’s concern for them, and that if an 57. (A) occult condition becomes evident it will be diag- nosed early. Certainly, regular consultation with 58. (B) other specialists is in order to manage these patients. Although the course of hypochondria- Explanations 57 and 58 sis tends to be chronic, there are indications that factor in for a good outcome. One of these is the Given the temporal relationship in the start of absence of secondary gain. This disorder is seen two anticholinergic drugs and the onset of the equally in both men and women. The preva- urinary hesitancy in an otherwise healthy young lence in a general medical practice is approxi- male, it would be reasonable to conclude that the mately 4–6%. There is no relationship between drugs are causing the problem. Certainly, a rapid assessment regarding the possibility of other hypochondriasis and increased ESR. (Kaplan and Sadock, 2003, pp. 651–653) causes (e.g., infection, trauma, stricture) is important. Careful, attentive listening for any 54. (C) In the defense mechanism of displacement, hint of psychotic delusion involving urination is an emotion is severed from its original con- important to screen for. The manner in which the nection with a person or event and attached to patient describes his symptoms is invaluable in a substitute person or object. With its origin facilitating diagnosis. Also remember that a real thus disguised, the emotion may be more safely medical condition can be described in bizarre, expressed. In the example described in the distorted terms, making assessment more diffi- question, anxiety associated with sexual feel- cult and complicated. Drug-induced urinary ings is displaced on to the setting in which they hesitancy may be treated by discontinuing the causative medications. In addition, bethanechol, occurred, with resulting phobic anxiety. (Kaplan and Sadock, 2003, p. 198) 10–30 mg three to four times each day, may be administered. Bethanechol acts by stimulating 55. (C) In starting an interview, generally an open- the parasympathetic nervous system. The tone ended question will allow the patient the free- of the detrusor urinae muscle increases, pro- dom to tell his or her story. Choice A assumes ducing a contraction strong enough to initiate the person is depressed; the patient may object micturition and emptying of the bladder. Giving if he or she feels already diagnosed and hasn’t benztropine, an anticholinergic, would only had the opportunity to talk. Choices B and D heighten the problem. Unfortunately, some may be seen as critical and do not directly patients understand that benztropine is “for the address the issue of what brings this patient to side effect” of their antipsychotic medication the psychiatrist. The patient may experience but do not understand the difference between these as unempathetic. Choice E is premature. the extrapyramidal effect and the anticholiner- As the consulting psychiatrist, you must per- gic effects. Increased thioridazine would also form a thorough evaluation to determine the increase the urinary problem. Calling in a urol- nature and treatment of this patient’s problem. ogist would be indicated if the initial treatment (Kaplan and Sadock, 2003, pp. 1–15) failed to work or if the emergency department physician were not able to “get beyond” an 56. (B) One of the earliest findings in biological extremely distorted, disorganized presentation psychiatry was the abnormal sleep pattern of by the patient. If the bladder were extremely dis- depressed patients. Electroencephalographic tended and the patient very uncomfortable, monitoring of sleep divides sleep into REM insertion of a urinary catheter would be a rea- and non-REM sleep. Sleep latency is generally sonable course of action. In the patient described, prolonged in depression, while REM latency the bladder is not distended. is shortened. General wakefulness is increased, Of the medications listed, risperidone is a with a decreased arousal threshold. There tends reasonable and safe choice because its anti- to be a reduction in stage 3 and stage 4 sleep in cholinergic effects are low compared to the other drugs listed, and much lower than thioridazine. depression. (Kaplan and Sadock, 2003, p. 539, 759) (c) ketabton.com: The Digital Library

Answers: 54–64 189

Chlorpromazine and mesoridazine have sub- and clonazepam would typically be withdrawn stantial anticholinergic effects. Amitriptyline due to their anticonvulsant effect. If a benzo- and imipramine have substantial anticholin- diazepine is required a short-acting medica- ergic effects, and, in addition, they are tricyclic tion should be used. Bupropion has been antidepressants, not antipsychotics. (Kaplan and associated with late appearing seizures. (Kaplan Sadock, 2003, pp. 982, 497–500) and Sadock, 2003, p. 1140)

59. (A) One of the most significant findings here is 62. (B) This woman’s symptoms meet the criteria that the woman, cooperative with the examina- for a major depressive episode. She has had a tion, has the score of 14 on the MMSE. A score of depressed (blue) mood for at least a 2-week 25–30 indicates no cognitive impairment, 20–25 period, a significant weight loss, insomnia, suggests possible mild impairment, and less fatigue and loss of energy, and thoughts of sui- than 20 is very strongly suggestive of cognitive cide. Because her symptoms seem to be limited impairment. This degree of change on the to 3 weeks, dysthymic disorder would most MMSE is not a normal sign of aging. In addition, likely not be considered. There are no indica- there are no signs of psychosis or mania. Even if tions for an organic mental disorder that would there were, in this woman with no previous psy- suggest Alzheimer’s disease. Generalized anx- chiatric history, one would not likely consider iety disorder is characterized by excessive anx- psychosis not otherwise specified or mania. The iety and worry for about 6 months. For a time frame for dysthymia is not met by the diagnosis of borderline personality disorder, “several days” length described here. (Kaplan and patterns of instability in relationships, self- Sadock, 2003, p. 1320, 321) image, affect, and impulsivity would have been present in early adulthood. None of that is 60. (C) ECT can be a life-saving tool in the treat- described here. (American Psychiatric Association ment of depression, particularly in individuals (APA), 2000, pp. 349–356, 472–476; Kaplan and Sadock, who are very suicidal, because of its relatively 2003, pp. 542–544) quick onset of action. It is a relative con- traindication to give patients with a clinically 63. (B) For a first, relatively acute episode of major significant space-occupying cerebral lesion depression, a tricyclic or SSRI is usually con- ECT because of the risk of brain stem hernia- sidered a first-choice drug. The SSRIs, consid- tion. ECT can be performed on patients with ered as effective as the tricyclics, are often space-occupying lesions rarely, but the benefit favored by clinicians because of their greater needs to outweigh the risk, and it should be safety profiles. Olanzapine is an example of performed by experts. However, although an antipsychotic drug. Alprazolam is a ben- hypertension and cardiovascular disease put zodiazepine that does have some anxiolytic patients at a higher risk for complications from value in depression. Its addictive potential ECT, they are not absolute contraindications to does not make it a drug of choice for depres- its use. With the use of muscle relaxants as part sion. Tranylcypromine is an effective MAO of the electroconvulsive technique, patients inhibitor antidepressant selected for use after with degenerative joint disease of the spine can a depression has failed to respond to the tri- generally safely receive ECT. Seizures actually cyclics and SSRIs. ECT is also used after other would typically decrease in frequency with the treatments have failed. In very severe, debili- application of ECT. (Kaplan and Sadock, 2003, tating depressions, however, a clinician may p. 1140) choose ECT as a first treatment. (Kaplan and Sadock, 2003, pp. 565–570) 61. (E) Risperdal and high potency neuroleptics slightly decrease the seizure threshold and 64. (A) Negative symptoms of schizophrenia reflect would enhance the seizure and can typically be the absence or deficiency of a mental function continued through a course of ECT. Lithium can that is normally present. Anhedonia, or the result in increased postictal delirium, divalproex inability to experience pleasure, is an example (c) ketabton.com: The Digital Library

190 5: Psychiatry

of such. Positive symptoms of schizophrenia disorder, or saying that no genetic link has been reflect aberrance or distortion of mental func- determined is very misleading and clinically tions. Loose associations, delusions of thought, incorrect. (Kaplan and Sadock, 2003, pp. 123–125) insertion, incoherence, and stereotypic gestures are all examples of these distortions. (Kaplan and 68. (B) Pick’s disease is considered a cortical demen- Sadock, 2003, pp. 490–491) tia, with the preponderance of pathologic find- ings found in the frontotemporal area. Aphasia, 65. (B) The antidepressant bupropion has been apraxia, and agnosia are signs sometimes seen in found effective for treating some cases of atten- these patients. Huntington’s and Parkinson’s tion deficit disorder and offers help to those diseases are caused by pathologic changes in children not responsive to the usual treatment the basal ganglia. Pathologic changes are seen in with stimulants (methylphenidate, pemoline). the ventricles in occult hydrocephalus. Signs The remaining choices have not been found seen in subcortical dementia more characteristi- useful in treating this condition. They are cally involve motor disorders: rigidity, tics, gait lithium, a mood stabilizer; alprazolam, a ben- difficulties, and incoordination. (Kaplan and Sadock, zodiazepine anxiolytic; propranolol, a beta- 2003, pp. 331–334, 1323) blocker; and perphenazine, an antipsychotic. (Kaplan and Sadock, 2003, pp. 1225–1226) 69. (E) The lifetime prevalence of PTSD is approxi- mately 8%. For PTSD to be diagnosed, the 66. (C) Cognitive psychotherapy would be helpful trauma has to be where serious injury or death to this man to see and understand how cogni- were threatened or involved, and the trauma- tive distortions about himself, others, and the tized individual experienced a sense of help- future bring about his depressive feelings. lessness, fear, or horror, and has at least one Psychoanalysis, a process lasting several years reliving symptom (nightmares of the trauma, with a weekly commitment of three to four ses- recurrent intrusive thoughts of the event, intense sions, would require this person to be willing to psychological stress or physiologic reactivity to explore and work through issues and conflicts internal or external cues that symbolize or that have their source in childhood. Behavioral resemble an aspect of the trauma, or flashbacks), therapy has as its goal the disruption of inap- two or more symptoms of increased arousal (dif- propriate behaviors with the substitute of more ficulty falling to or staying asleep, irritability, appropriate behaviors. It is intended for the difficulty concentrating, hypervigilance, and treatment of phobias and various psychoso- exaggerated startle response), and three or more matic disorders (e.g., migraine, hypertension). avoidance symptoms (efforts to avoid thoughts, Supportive psychotherapy could also be of feelings, or conversations about the trauma, some value. This is used frequently in conjunc- efforts to avoid people, things or places that tion with medication. Group therapy may be of remind one of the trauma, inability to remember some value after this patient has had the oppor- an important aspect of the trauma, diminished tunity to work in a one-to-one situation in which participation in activities, feeling detached or understandings about himself have developed. estranged from others, restricted range of affect, Proper preparation is essential before entering and/or sense of foreshortened future). The group therapy. (Kaplan and Sadock, 2003, pp. 956–960) symptoms have to be recurring for at least a month. (Kaplan and Sadock, 2003, pp. 626–627) 67. (C) There is strong evidence for a genetic predisposition to bipolar disorder. Some of the 70. (E) evidence comes from twin studies. The con- cordance rate for monozygotic twin is 79%, but 71. (B) for dizygotic twin it is 19%. Advising the patient that she is past the age when bipolar 72. (A) disorders develop, using lithium to prevent the (c) ketabton.com: The Digital Library

Answers: 65–79 191

73. (D) 78. (E)

74. (E) 79. (C)

Explanations 70 through 74 Explanations 75 through 79

Infantile autism, called a pervasive develop- The criteria for mood disorders depend on the mental disorder in DSM-IV, typically is diag- presence or absence and duration of depres- nosed when children do not demonstrate the sive and hypomanic or manic symptoms as acquisition of communication skills. Ability to well as on their severity, and also on the pres- form interpersonal relationships also is grossly ence or absence of a causative general medical impaired. Other behavioral manifestations of condition or the ingestion of substances. Major infantile autism include unusual repetitive man- depressive disorder, recurrent, is marked by nerisms (e.g., spinning), marked anxiety during the lifetime occurrence of two or more major environmental changes, and high pain threshold. depressive episodes without intervening hypo- As to be expected, school performance is poor, manic or manic episodes. A major depressive though autistic children may display isolated episode is a severe depression which has lasted areas (islands) of normal or superior intellectual at least 2 weeks. Bipolar I disorder is charac- functioning. Behavioral manipulation is useful in terized by a history of at least one full-blown trying to contain the behavior of autistic chil- manic episode, during which the patient’s dren. Unlike infantile autism, childhood schizo- mood has been abnormally and persistently phrenia usually develops later in childhood and elevated, expansive or irritable for at least follows an intermittent course. Deterioration in 1 week with marked impairment in occupa- social or school functioning is a characteristic tional functioning. Bipolar II disorder, on the presenting feature, along with hallucinations, other hand, is marked by a history of at least delusions, and other manifestations of psychosis. one major depressive episode and at least one Phenothiazine drugs offer effective treatment. hypomanic episode, during which a patient’s Symptoms and signs of depression in children mood has been elevated, but not to the extent are similar to those in adults. However, children of causing marked impairment in social or may not be able to recognize depressed feelings. occupational functioning. A patient with bipo- Persistence of puzzling physical problems in lar II disorder may not, by definition, have had association with apathetic, withdrawn behavior a full-blown manic episode. Cyclothymia is is a common presentation. The use of antide- marked by periods of hypomanic symptoms pressants is controversial; family and individual alternating with depressive symptoms that do counseling often can be quite helpful. ADHD not meet the criteria for a major depressive once was called hyperactivity and minimal episode. Dysthymic disorder is marked by a brain dysfunction. Characteristic signs include persistent, low-grade depression occurring impulsivity, distractibility, inattention in school, more days than not for at least 2 years. A mood and (usually but not universally) hyperactiv- disorder due to a general medical condition is ity. A variety of pharmacologic agents, includ- a prominent and persistent disturbance in ing imipramine, dextroamphetamine, and mood that is judged to be the direct physio- methylphenidate (Ritalin), have been recom- logic effect of a general medical condition, such mended for treatment of ADHD. (American as hyperthyroidism. A substance-induced Psychiatric Association (APA), 2000, pp. 70–75, 85–93; mood disorder is a prominent and persistent Kaplan and Sadock, 2003, pp. 1208–1231) disturbance in mood that is judged to be due to the direct effects of a substance, but which con- 75. (B) tinues beyond the usual period of intoxication or withdrawal from a substance. (American 76. (A) Psychiatric Association (APA), 2000, pp. 345–428)

77. (G) (c) ketabton.com: The Digital Library

192 5: Psychiatry

80. (F) This man’s symptoms are psychotic in Understanding the site of action, neurotrans- nature and somewhat bizarre. In addition, the mitter(s) involved, and side effects characteris- information strongly suggests that he is very tic of these classes is helpful in selecting an sensitive to the extrapyramidal effects of the antidepressant for a particular patient. SSRIs traditional antipsychotics. Olanzapine, an atyp- that are comparable in their antidepressant ical antipsychotic with very few extrapyrami- effects to the older tricyclics but significantly dal effects, would be a good choice here. (Kaplan safer when taken in larger doses, as in suicidal and Sadock, 2003, pp. 1105–1106) overdose, are frequently used as the first choice in the treatment of depression. An example here 81. (C) A hypomanic state is described here. This is is citalopram. Drugs that both inhibit serotonin

seen in bipolar I and bipolar II disorders. A treat- reuptake and block 5-HT2 receptors are charac- ment of choice is the mood stabilizer dival- teristic of the triazolopyridines. The overall proex. If psychotic symptoms were present, the effect of these actions is believed to decrease addition of an antipsychotic would be indi- both depression and anxiety in patients. There cated. (Kaplan and Sadock, 2003, pp. 570–572, are two drugs in this class: trazodone and nefa- 1131–1132) zodone. Strong sedation caused by histaminer- gic and anticholinergic activity is seen in the 82. (D) This woman is exhibiting signs of severe older antidepressants—the tricyclics. These also depression. Because of her cardiac condition, have both serotonin and norepinephrine effects avoiding an antidepressant with negative car- that are important in decreasing depression. diac effect is important; therefore, amitripty- Amitriptyline is the drug example listed here. line would be eliminated. Fluoxetine, an SSRI, Drugs demonstrating little sedation and signif- would be an appropriate choice. (Kaplan and icant serotonin, norepinephrine, and dopamine Sadock, 2003, pp. 565–566) effects are more characteristic of the serotonin- norepinephrine reuptake inhibitors. They are 83. (B) Signs of tardive dyskinesia are evident in effective in managing depression because there this woman. She also was tried on several is no antihistaminergic activity and little seda- “newer” medications, one of which may have tion is seen. Duloxetine and Venlafaxine are been olanzapine. This would have to be deter- examples. MAOIs increase the concentrations of mined. Assuming this is so, a good choice is serotonin, norepinephrine, and dopamine by clozapine, which does not contribute to the inhibiting their degradation. The MAOIs, development of tardive dyskinesia. (Kaplan and although effective as antidepressants, are used Sadock, 2003, pp. 497–499, 1106–1107) relatively infrequently because of the potential development of a hypertensive crisis induced 84. (C) by consuming tyramine-containing foods while on the MAOI. An example here is phenelzine. 85. (B) (Kaplan and Sadock, 2003, pp. 1093–1104, 1076–1081, 1123–1130, 1135–1136) 86. (A) 89. (D) 87. (E) 90. (E) 88. (D) 91. (B) Explanations 84 through 88 92. (C) The drugs listed in this question are examples of the various classes of antidepressants. 93. (A) These classes include the tricyclics, the SSRIs, the MAOIs, the triazolopyridines, and the 94. (F) serotonin-norepinephrine reuptake inhibitors. (c) ketabton.com: The Digital Library

Answers: 80–106 193

Explanations 89 through 94 All have problems with interpersonal relation- ships. (Kaplan and Sadock, 2003, pp. 800–821) Lithium carbonate is an effective treatment for manic and depressive episodes due to 100. (J) bipolar I disorder, as well as for the prophy- laxis of manic and depressive episodes in 101. (A) patients with bipolar I disorder. It can impede the release of thyroid hormone from the thy- 102. (I) roid, and it can reduce the ability of the kid- neys to concentrate urine; its use requires 103. (H) regular blood levels to ensure the avoidance of toxic blood levels that could cause tremor, 104. (F) dysarthria, ataxia, or death. Acamprosate (Campral) is gabanergic and decreases the Explanations 100 through 104 craving for alcohol in patients with alcohol dependence. Sertraline (Zoloft) is a SSRI Defense mechanisms provide a means for deal- which is effective in the treatment of major ing with anxiety and affect. The mechanisms depressive disorder, as well as in the treat- chosen range from the very narcissistic and ment of panic disorder, obsessive-compulsive immature to mature. In suppression, a person disorder, and PTSD. Risperidone is an atypi- makes a conscious decision to put the conflict cal antipsychotic that is effective in the treatment aside until it can be dealt with more appropri- of psychotic conditions, including schizophre- ately. On the other hand, in acting out, there is nia, and is associated with fewer serious long- little or no attempt to contain the affect, and it term side effects, such as tardive dyskinesia, than is directly expressed, as in name calling. traditional neuroleptics. Risperidone is a potent Sublimation provides a channel for the indi-

blocker of both 5-HT2 serotonin receptors and rect expression of a need or affect. Its use is D2 dopamine receptors. Buspirone, a nonaddic- positive and socially acceptable. In reaction for- tive azapirone, acts as an agonist or partial mation, the person acts as if the strong need or

agonist of 5-HT1 serotonin receptors, and is effec- affect did not exist and acts out the opposing tive in the treatment of generalized anxiety dis- feeling. In projection, unacceptable feelings and order. Dextroamphetamine is a stimulant and is thoughts are denied as part of the self and an effective treatment for ADHD. (Kaplan and instead are “put on” the other person. (Kaplan Sadock, 2003, pp. 412–413, 1067–1069, 1102, 1085–1089, and Sadock, 2003, pp. 207–208) 1031–1033, 1111, 1308) 105. (D) 95. (H) 106. (A) 96. (I) Explanations 105 and 106 97. (J) Fluoxetine has the longest half-life of the current 98. (C) SSRIs (escitalopram’s half-life is shorter—less than 24 hours), phenelzine is an MAOI and 99. (B) foods rich in tyramine can induce a hypertensive crisis. Venlafaxine can induce hypertension, Explanations 95 through 99 especially at higher doses. Trazodone can rarely induce priapism (a painful sustained erection). Persons with personality disorders are rigidly Nortriptyline is a tricyclic antidepressant, and at bound to the use of patterns of defense and high doses, it can cause arrhythmias. (Kaplan and various traits that distinguish the disorders. Sadock, 2003, pp. 1094, 1078, 1136, 1124) (c) ketabton.com: The Digital Library

194 5: Psychiatry

107. (B) 111. (D) Though Alzheimer’s dementia is the most common form, it is characterized by a gradual 108. (A) and progressive course typically starting with a loss of short-term memory. Vascular demen- 109. (C) tia is known for its stepwise progression. Frontotemporal dementia (Pick’s) usually pres- 110. (C) ents insidiously with behavior and personality changes. Lewy body dementia shares features Explanations 107 through 110 with Alzheimer’s dementia and dementia asso- ciated with Parkinson’s disease, but is more Valproate can be associated with thrombocy- progressive than the dementia associated with topenia and platelet dysfunction especially at Parkinson’s disease, and tends to exhibit cogni- high doses. Leukocytosis is a common benign tive decline soon after the Parkinson’s symptoms effect of lithium. Clozaril can cause agranulo- present. Visual hallucinations and fluctuating cytosis in 1–2% of patients. Agranulocytosis cognition are also common with this type of can be an idiosyncratic adverse event dementia. (Stern and Herman, 2000, pp. 52–54) with carbamazepine. (Kaplan and Sadock, 2003, p. 1133, 1069) (c) ketabton.com: The Digital Library

BIBLIOGRAPHY

American Psychiatric Association (APA). Diagnostic Kaplan HI, Sadock BJ. Synopsis of Psychiatry: and Statistical Manual of Mental Disorders, 4th ed. Text Behavioral Sciences/Clinical Psychiatry, 9th ed. Revision. Washington, DC: American Psychiatric Baltimore, MD: Williams & Wilkins, 2003. Association, 2000. Stern TA, Herman JB. Massachusetts General Hospital Bernstein CA, Ladds BJ, Maloney AS, et al. On Call Psychiatry Update and Board Preparation, 2nd ed. Psychiatry. Philadelphia, PA: W.B. Saunders, 1997. New York, NY: McGraw-Hill, 2000.

195 (c) ketabton.com: The Digital Library

This page intentionally left blank (c) ketabton.com: The Digital Library

CHAPTER 6 Surgery Kelli R. Brown, MD and Travis Webb, MD

Questions

1. A 32-year-old, previously healthy man is a Questions 2 and 3 victim of a drive-by shooting, sustaining a gun- shot wound to the left lower extremity. The A 16-year-old girl with a history of ulcerative colitis entrance wound is located over the medial managed with steroid therapy presents to the emer- aspect of the calf, with an exit wound over the gency department with a 36-hour history of nausea, anterior pretibial region. Neurovascular exam- crampy abdominal pain, and severe bloody diar- ination of the extremity is normal. There is rhea. On examination, the patient is febrile and pale, associated soft-tissue injury from the blast with a blood pressure of 90/60 mmHg and heart effect and a severely comminuted tibial fracture rate of 130 beats/min. Her abdomen is distended demonstrated on radiographs. Appropriate and diffusely tender. A complete blood count (CBC) management of this injury includes which of demonstrates a leukocytosis with a left shift. The the following? patient receives IV fluid resuscitation and nasogas- tric (NG) tube decompression. (A) local wound irrigation, closure of the soft-tissue defect, closed reduction, and 2. Further therapeutic interventions should immobilization in a long-leg cast include which one of the following? (B) local wound irrigation with antibiotic (A) 6-mercaptopurine solution, closed reduction, and immobi- lization in a long-leg cast, with contin- (B) azathioprine ued local wound care through an ante- (C) opioid antidiarrheals rior cast window (D) colonoscopic decompression (C) tetanus prophylaxis, intravenous (IV) (E) high-dose IV steroids and broad-spectrum antibiotics, and operative wound irriga- antibiotics tion and debridement, with application of an external fixation device 3. After 48 hours, there is no clinical improve- (D) tetanus prophylaxis, IV antibiotics, oper- ment. Which of the following is the most ative wound irrigation with closure of appropriate next step in management? the soft-tissue defect, closed reduction, (A) colonoscopic decompression and immobilization in a long-leg cast (B) cyclosporine (E) tetanus prophylaxis, IV antibiotics, long- (C) abdominal colectomy and ileostomy leg splint for immobilization, and opera- and Hartmann’s procedure tive intervention during elective surgical (D) proctocolectomy with ileal pouch-anal schedule anastomosis (E) abdominal colectomy with ileorectal anastomosis

197

Copyright © 2008 by The McGraw-Hill Companies, Inc. Click here for terms of use. (c) ketabton.com: The Digital Library

198 6: Surgery

Questions 4 and 5 6. A 2-year-old child presents with a 2-day history of painless rectal bleeding. On examination, the A term infant is born at a small community hospital child is pale with tachycardia. The abdomen is by cesarean section for failure to progress. The nondistended and nontender. There is dark blood infant is noted to have the following abnormality at on rectal examination. The child has the follow- birth (see Figure 6-1). ing imaging study (see Figure 6-2). Which of the following is the most appropriate management?

FIG. 6-1

4. Which of the following is the most likely diagnosis? FIG. 6-2 (A) umbilical hernia (B) omphalitis (A) surgical exploration (C) omphalocele (B) aggressive resuscitation followed by (D) gastroschisis surgical exploration (E) traumatic evisceration (C) colonoscopy (D) acid suppression therapy 5. Which of the following is the most appropriate (E) IV steroids initial management? (A) IV antibiotics alone Questions 7 and 8 (B) emergency surgery for reduction A 55-year-old-woman presents to the physician’s (C) monitor for spontaneous closure, with office for evaluation of mammographic findings on surgical intervention for persistent fas- a screening mammogram. She denies any breast cial defect masses, nipple discharge, pain, or skin changes. (D) IV fluids, IV antibiotics, warm occlusive Past history is pertinent for insulin-dependent dia- dressing, and transfer to a center with a betes. Family history is positive for postmenopausal pediatric surgeon breast cancer in her mother. She has a normal breast (E) elective umbilical exploration examination and no axillary adenopathy. A medio- lateral oblique (MLO) view of the right breast is shown Figure 6-3. (c) ketabton.com: The Digital Library

Questions: 4–8 199

(D) lumpectomy, radiation therapy, and sentinel lymph node (SLN) biopsy (E) total mastectomy

Questions 9 and 10

A 51-year-old woman presents to the physician’s office with a 2-month history of a right breast blood- tinged nipple discharge. Past history is unremarkable. Family history is positive for postmenopausal breast cancer in a maternal grandmother. Examination reveals no palpable masses or regional adenopathy, but a serous discharge is easily elicited from a single duct in the right breast. Bilateral mammograms show no abnormalities. Cytology from the discharge was not diagnostic. A ductogram was ordered, and the results are shown in Figure 6-4.

FIG. 6-3

7. Which of the following is the most likely diagnosis? (A) milk of calcium (B) lobular carcinoma in situ (LCIS) with or without an invasive component (C) ductal carcinoma in situ (DCIS) with or without an invasive component (D) involuting fibroadenoma (E) phyllodes tumor

8. Which of the following is the most appropriate next step in management? (A) observation, with repeat mammogram in 6–12 months FIG. 6-4 (B) ultrasound (C) biopsy (c) ketabton.com: The Digital Library

200 6: Surgery

9. Which of the following is the most likely diagnosis? (A) invasive carcinoma (B) intraductal carcinoma (C) intraductal papilloma (D) fibrocystic disease (E) duct ectasia

10. Which of the following is the most appropriate next step in management? (A) collection of discharge for repeat cyto- logic analysis (B) observation, with repeat examination and imaging studies in 3–6 months (C) modified radical mastectomy (D) central lumpectomy (including removal of the nipple/areolar complex) (E) terminal duct excision (microdochectomy)

Questions 11 and 12

An 85-year-old man presents to the emergency room FIG. 6-5a with an acute onset of midepigastric pain, nausea, vomiting, and hiccups starting 2 days ago. He is unable to keep any food down. Past history is perti- nent for a long-standing hiatal hernia, hypertension, and diet-controlled diabetes. Examination reveals vital signs of pulse rate 82/min, BP 100/52 mmHg, respiratory rate 16/min, and temperature 97.2°F. The patient is in no acute distress, but has epigastric tenderness without guarding. Laboratory analysis revealed a hematocrit of 46 and a normal white blood cell (WBC) count. A chest x-ray is shown in Figure 6-5a. A fluoroscopically guided NG tube was placed using contrast, and his stomach was decom- pressed. After adequate fluid and electrolyte resusci- tation, an upper gastrointestinal (UGI) contrast study was obtained and is shown in Figure 6-5b.

FIG. 6-5b (c) ketabton.com: The Digital Library

Questions: 9–17 201

11. Which of the following is the most likely 14. The diagnosis is best confirmed by which of the diagnosis? following? (A) sliding hiatal hernia (A) abdominal ultrasound (B) hernia of Bochdalek (posterorlateral (B) careful clinical examination with palpa- congenital diaphragmatic hernia) tion of an epigastric mass (C) hernia of Morgagni (parasternal congen- (C) UGI contrast study ital diaphragmatic hernia) (D) surgical exploration (D) paraesophageal hernia (E) endoscopy (E) eventration of the diaphragm (central diaphragm) 15. Electrolytes and a urinalysis are evaluated. Which of the following laboratory findings are 12. Which of the following is the most appropriate most likely to be seen in this patient? next step in management? (A) Na 145, K 3.0, Cl 110, CO2 17, urine pH 8.0 (A) laparotomy or laparoscopy and opera- (B) Na 130, K 3.0, Cl 80, CO2 36, urine pH 4.0 tive repair (C) Na 135, K 4.0, Cl 104, CO2 23, urine pH 7.0 (B) continued NG tube decompression and (D) Na 140, K 5.2, Cl 100, CO2 16, urine pH 4.0 initiation of total parenteral nutrition (E) Na 132, K 3.2, Cl 96, CO 25, urine pH 7.0 (TPN) 2 (C) thoracotomy or thoracoscopy and oper- 16. Which of the following is the most appropriate ative repair next step in management of this infant? (D) endotracheal intubation and initiation of ventilatory support (A) immediate surgical exploration (E) upper endoscopy (B) send the child home with an oral electrolyte rehydration solution Questions 13 through 16 (C) change the infant’s formula and feeding regimen A 5-week-old infant presents with a 1-week history (D) IV fluid resuscitation, followed by of progressive nonbilious emesis, associated with a surgical intervention 24-hour history of decreased urine output. The (E) initiate therapy with a prokinetic agent infant continues to be active and eager to feed. On examination, the infant has a sunken fontanelle and Questions 17 and 18 decreased skin turgor. The abdomen is scaphoid, and with a test feed, there is a visible peristaltic A 40-year-old previously healthy man presents with wave in the epigastrium. sudden onset of severe abdominal pain that radiates from the right loin (flank) to groin. This pain is asso- 13. Which of the following is the most likely ciated with nausea, sweating, and urinary urgency. diagnosis? He is distressed and restless, but an abdominal (A) viral gastroenteritis examination is normal. (B) gastroesophageal reflux 17. Which of the following is the most likely (C) urinary tract sepsis diagnosis? (D) pyloric stenosis (A) torsion of the right testicle (E) milk protein allergy (B) pyelonephritis (C) appendicitis (D) right ureteral calculus (E) acute urinary retention (c) ketabton.com: The Digital Library

202 6: Surgery

18. Which of the following is the most appropriate chest wall movement, and decreased air entry next step in management? over the left lung field. Which of the following is the most appropriate next step in the initial (A) insertion of a urethral catheter evaluation and management of this patient? (B) IV fluid hydration, IV analgesics, and nonenhanced computed tomography (A) fluid resuscitation with 2 L of isotonic (CT) scan crystalloid (C) IV fluid hydration, IV analgesics, and (B) needle decompression of the left chest, arrangements for lithotripsy followed by insertion of a chest tube (D) cystoscopy and retrograde pyelogram (C) portable chest x-ray (E) urine culture, followed by initiation of (D) immediate intubation and assisted antibiotic therapy ventilation (E) emergency department thoracotomy 19. A 25-year-old woman was involved in a motor vehicle crash and sustained a significant Questions 21 through 23 closed-head injury, a pulmonary contusion, A 4-year-old previously healthy girl presents to the and a pelvic fracture. She is unresponsive and emergency department with a 24-hour history of is ventilated in the intensive care unit (ICU). rectal bleeding and dizziness. She has no other gas- Which of the following is the best initial trointestinal symptoms. On examination, she approach to the management of this patient’s appears pale. Her heart rate is 140 beats/min, and nutritional needs? she has a 20 mmHg postural drop in systolic blood (A) insertion of a subclavian venous pressure. The child’s abdomen is nondistended and catheter and initiation of central IV nontender, and fresh blood and clots are in the rectal hyperalimentation vault on rectal examination. (B) wait for extubation and improvement of neurologic status, allowing institution of 21. Which of the following is the most likely an oral caloric intake diagnosis? (C) early institution of NG or nasojejunal (A) a bleeding Meckel’s diverticulum tube feeding with an elemental (B) juvenile rectal polyp formulation (C) hemorrhoids (D) wait for resolution of the associated gas- (D) an anal fissure trointestinal ileus, followed by delayed initiation of NG tube feeding with a (E) intussusception complex hypercaloric formulation 22. Which of the following is the most appropriate (E) peripheral IV hyperalimentation diagnostic study to order for this patient? 20. A 26-year-old previously healthy man was (A) colonoscopy pinned under a crane at a construction site. After (B) barium enema a prolonged extrication, he was brought to the (C) technetium scan emergency department, immobilized on a back (D) UGI contrast study with small-bowel board and receiving 100% oxygen by mask. He follow-through is alert and complaining of chest pain with res- (E) laparoscopy piratory effort. On examination, he is found to have an oxygen saturation of 90% by pulse 23. Definitive management of this child should oximetry, shallow respirations at a respiratory include which of the following? rate of 35/min, heart rate of 120 beats/min, and a blood pressure of 85/60 mmHg. The trachea is (A) immediate exploratory laparotomy deviated to the right. There is tenderness and (B) IV fluid resuscitation, transfusion with crepitation over the left chest wall, asymmetric blood products as indicated, followed (c) ketabton.com: The Digital Library

Questions: 18–28 203

by a laparotomy with Meckel’s divertic- 25. Which of the following is the most likely ulectomy and ileal resection diagnosis? (C) IV fluid resuscitation, followed by a (A) ulcerative colitis colonoscopic polypectomy (B) appendicitis (D) hemorrhoidectomy (C) Crohn’s disease (E) stool softeners and topical steroids (D) irritable bowel syndrome 24. A 29-year-old nonhelmeted motorcycle driver (E) lactose intolerance is involved in a single vehicular crash, result- ing in a significant closed-head injury. He is 26. Which of the following is the best diagnostic intubated in the field and transported to a test to confirm the diagnosis? level 1 trauma center. On arrival, he is oxy- (A) repeat CT scan with delayed imaging genating well with assisted ventilation and (B) ultrasonography has a normal blood pressure and moderate (C) sigmoidoscopy tachycardia. His Glasgow Coma Score is 7, and his pupils are equal and sluggishly reac- (D) colonoscopy tive. After stabilization in the emergency (E) small-bowel radiography department, the patient undergoes a CT scan of the head that demonstrates a small amount 27. Initial management should include which of of subarachnoid blood and a right frontal lobe the following? contusion with edema with no midline shift. (A) antibiotics and IV fluids CT scan of the abdomen is normal. The (B) lactose-free diet patient is transferred to the ICU. The optimal initial management of this patient’s intracra- (C) antispasmodics nial pressure (ICP) would be which of the (D) nutritional supplementation and systemic following? steroids (E) laparotomy (A) craniotomy (B) fluid restriction, hyperventilation, and 28. A 13-year-old boy is brought to the emergency osmotic diuresis department at midnight with a 4-hour history (C) fluid restriction, hyperventilation, and of right scrotal pain that was sudden in onset ventriculostomy and associated with nausea and one episode of (D) hyperventilation and IV steroids vomiting. On examination, he is in obvious dis- (E) normovolemia, normocarbia, sedation, tress. He has mild right lower abdominal ten- and ventriculostomy derness, and high-riding, tender right testes. CBC and urinalysis are normal. Which of the Questions 25 through 27 following is the most appropriate next step in management? A 55-year-old woman presents with a 6-month his- tory of weight loss, abdominal cramps, and inter- (A) admit the patient to the hospital and mittent nonbloody diarrhea. On examination, her place him on bed rest abdomen is mildly distended and there is a palpa- (B) analgesics and a scrotal support ble mass in the right lower quadrant. Stool cultures (C) antibiotic therapy yield normal fecal flora. CT scan with oral contrast (D) schedule a testicular isotope scan demonstrates an inflammatory mass in the right (E) urgent surgical exploration lower quadrant, with thickening of the terminal ileum and ileocecal valve. (c) ketabton.com: The Digital Library

204 6: Surgery

29. A 70-year-old man presents with back pain and (C) observation in the ICU only if carotid increasing difficulty with initiating a urinary angiogram is normal stream. On rectal examination, he is found to (D) observation in the ICU only if carotid have a hard, irregularly enlarged prostate. He angiogram, contrast esophagram, and has an elevated prostate-specific antigen (PSA), bronchoscopy are normal and osteoblastic lesions in the vertebral column (E) neck exploration and bones of the pelvis. A needle biopsy of the prostate shows well-differentiated adenocarci- 32. A 45-year-old man is brought to the emergency noma. Which of the following is the treatment department after being involved in an auto- of choice? mobile crash. He is alert and oriented, with a (A) radical prostatectomy normal neurologic examination. His respira- (B) transurethral prostatectomy tory rate is 20/min, with clear lungs, pulse rate of 120/min, and blood pressure of 80/40 (C) cytotoxic chemotherapy mmHg. On examination, he is noted to have a (D) hormonal manipulation distended abdomen, with decreased bowel (E) radiotherapy sounds, and a fracture of the right ankle. IV access is established, and the patient receives a 30. A 25-year-old previously healthy man is rapid infusion of 2 L of saline, without changes scheduled for elective inguinal hernia repair to pulse rate or blood pressure. Which of the under general anesthesia. After induction of following is the most appropriate next step in anesthesia and initial inguinal incision, the his management? patient develops tachycardia, muscle rigidity, fever of 38.5°C, and elevated end-tidal carbon (A) abdominal CT scan dioxide. Which of the following is the most (B) insertion of a Swan—Ganz catheter likely diagnosis? (C) exploratory laparotomy (A) pneumonia (D) focused abdominal sonography for (B) atelectasis trauma (FAST) (C) urinary tract infection (E) diagnostic peritoneal lavage (D) myocardial infarction DIRECTIONS (Questions 33 through 51): Each set (E) malignant hyperthermia of matching questions in this section consists of a list of lettered options followed by several num- 31. A previously healthy 19-year-old man presents bered items. For each item, select the ONE best let- to the emergency department with a penetrat- tered option that is most closely associated with it. ing wound to the right neck. There were Each lettered option may be selected once, more reports of bleeding at the scene. The patient is than once, or not at all. talking, complaining of pain at the injury site and pain with swallowing. On examination, he has a normal respiratory rate, clear air entry on Questions 33 and 34 auscultation, blood pressure of 120/70 mmHg, and heart rate of 95 beats/min. There is a pen- Select the most likely diagnosis for each of the etrating right neck wound in zone 2 (between patients with polyuria. the clavicle and the lower part of the mandible), (A) central diabetes insipidus (DI) with a surrounding hematoma. On probing, (B) nephrogenic DI there is violation of the platysma. Which of the (C) water intoxication following is the best next step in the manage- ment of this patient? (D) solute overload (E) diabetes mellitus (A) intubation and observation in the ICU (B) admission to the ICU for close observa- tion without intubation (c) ketabton.com: The Digital Library

Questions: 29–39 205

33. A 25-year-old man was admitted to the ICU with decreased appetite. She denied dysuria. Her severe head injury with a basal skull fracture. last menstrual period was 2 weeks earlier. On Eighteen hours after the injury, he developed examination, she was febrile (temperature polyuria. Urine osmolality was 150 mOsm/L and 38.2°C), and was found to have localized ten- serum osmolality was 350 mOsm/L. IV fluids derness in the right lower quadrant with guard- were stopped, and 1 hour later urine output and ing. Rectal examination was normal. Laboratory urine osmolality remained unchanged. Five units examination demonstrated mild leukocytosis. of vasopressin were administered intravenously, and urine osmolality increased to 300 mOsm/L. 36. A 40-year-old man with a history of alcohol abuse presents after an episode of binge drinking. He is 34. A 70-year-old man was admitted to the ICU complaining of epigastric pain, radiating to the with severe pancreatitis. During his ICU course, back, associated with nausea and vomiting. On he underwent several CT scans with IV contrast examination, he has marked tenderness in the and was also treated with an aminoglycoside epigastrium, with guarding, decreased bowel for a urinary tract infection. The patient required sounds, and moderate abdominal distention. a prolonged course of TPN, and developed Laboratory findings include leukocytosis and Candida sepsis treated with amphotericin. He increased serum amylase and lipase. Abdominal subsequently developed polyuria with urine roentgenograms demonstrate several dilated osmolality of 250 mOsm/L and serum osmolal- bowel loops in the upper abdomen. ity of 350 mOsm/L. After receiving 5 units of vasopressin intravenously, there is no change in 37. A 65-year-old man presents with a 4-day history urine osmolality or urine output. of worsening lower abdominal pain and con- stipation. On examination, he is febrile (38.5°C) Questions 35 through 39 and has lower abdominal tenderness that is For each patient with abdominal pain, select the most intense in the midline and left lower quad- most likely diagnosis. rant associated with a palpable fullness. Laboratory findings demonstrate a moderate (A) gastroenteritis leukocytosis and abdominal roentgenograms (B) regional enteritis show an ileus pattern. (C) acute appendicitis (D) perforated peptic ulcer 38. A 30-year-old man presents with sudden onset of severe epigastric pain 6 hours ago. Examination (E) sigmoid diverticulitis reveals a low-grade fever, tender abdomen (F) acute pancreatitis throughout, with rigidity of the abdominal mus- (G) acute cholecystitis culature. Abdominal roentgenograms show (H) superior mesenteric artery embolism pneumoperitoneum. (I) ruptured abdominal aortic aneurysm (J) ruptured ovarian cyst 39. A 40-year-old woman presents to the emer- (K) cecal volvulus gency room with a 3-day history of worsening abdominal pain, with nausea and vomiting. 35. A 21-year-old previously healthy woman pres- Examination reveals a low-grade fever and ents with abdominal pain of 48-hour duration. abdominal tenderness in the right upper quad- The pain was initially periumbilical and on pro- rant with guarding, especially during inspira- gression became localized in the right lower tion. Laboratory findings include a mild quadrant. The woman had nausea and a leukocytosis and a slightly elevated bilirubin. (c) ketabton.com: The Digital Library

206 6: Surgery

Questions 40 through 46 and is painless. On examination, a nontender, firm, 2.5-cm mass is noted slightly below and For each patient with a neck mass, select the most posterior to the angle of the mandible on the likely diagnosis. right. (A) thyroid carcinoma (B) cystic hygroma 45. A 35-year-old woman presents to the physi- cian’s office for evaluation of a left neck mass (C) acute suppurative lymphadenitis discovered 1 month ago on a routine physical (D) thyroglossal duct cyst examination. On examination, the mass meas- (E) lipoma ures 2 cm and is located anterolateral to the (F) carotid artery aneurysm larynx and trachea. It is nontender and moves (G) mixed parotid tumor (pleomorphic with swallowing. Past history is pertinent for a adenoma) 15 pack-year smoking history and occasional (H) laryngeal carcinoma alcohol intake. (I) parathyroid adenoma 46. A 55-year-old man presents to the physician’s (J) branchial cleft cyst office with complaints of hoarseness and left (K) tuberculosis neck fullness for the past month. On examina- tion, a firm, movable, left submandibular mass 40. A 3-year-old boy presents to the physician’s is noted. Past history is pertinent for a 30 pack- office with an asymptomatic neck mass located year smoking history with occasional alcohol in the midline, just below the level of the thy- intake. roid cartilage. The mass moves with deglutition and on protrusion of the tongue. Questions 47 through 51

41. A 45-year-old man presents to the physician’s For each patient with jaundice, select the one most office for evaluation of a posterior neck mass. likely diagnosis. The mass has been present for years, but (A) hepatitis A has slowly enlarged over the last 2 years. (B) hemolysis Examination reveals a subcutaneous mass that is soft, nontender, and movable. (C) choledocholithiasis (D) biliary stricture 42. A 6-year-old boy presents to the emergency (E) choledochal cyst department with a cough, sore throat, and (F) pancreatic carcinoma malaise of 4 days’ duration. Examination (G) liver metastases reveals a temperature of 101.5°F, erythematous (H) cirrhosis pharynx, and a tender right neck mass with (I) pancreatitis overlying erythema. 47. A 50-year-old man presents to the emergency 43. An 18-month-old girl is brought to the physi- department for increasing abdominal disten- cian’s office for evaluation of left neck mass. tion and jaundice over the last 4–6 weeks. Examination reveals a 2-cm soft, nontender, Examination reveals mild jaundice, spider fluctuant mass in the left lateral neck. This is angiomas, and ascites. Enlarged veins are noted located at the anterior border of the ster- around the umbilicus. nomastoid, midway between the mastoid and clavicle. 48. A 75-year-old man is brought to the emergency department by his family for evaluation of 44. A 50-year-old woman presents to the physi- jaundice. He complains of pruritus of 2 weeks’ cian’s office for evaluation of a right neck duration and a recent 10-lb weight loss. On mass. The mass has been present for 3 years (c) ketabton.com: The Digital Library

Questions: 40–52 207

examination, he is deeply jaundiced and has a abuse, and diabetes. Examination reveals a nontender, globular mass in the right upper mildly jaundiced patient with vital signs of quadrant of the abdomen that moves with res- temperature 100°F, pulse rate 95/min, and piration. BP 110/60 mmHg. Cardiac examination is unremarkable, lung examination reveals 49. A 75-year-old woman is brought to the emer- decreased breath sounds at the bases bilater- gency department from the nursing home for ally, and abdominal examination reveals full- jaundice and mental confusion. The nursing ness in the epigastrium with tenderness and home notes state that she has become less voluntary guarding. responsive and has developed jaundice over the last 2 weeks. Past history is pertinent for DIRECTIONS (Questions 52 through 113): Each hypertension, diabetes, and prior colon resec- of the numbered items or incomplete statements tion for cancer at age 55. Examination reveals in this section is followed by answers or by com- mild jaundice with vital signs of temperature pletions of the statement. Select the ONE lettered 101.5°F, pulse rate 110/min, and BP 100/60 answer or completion that is BEST in each case. mmHg. She does not respond to verbal com- mands, but withdraws to pain. Abdominal 52. A 56-year-old woman presents to the physi- examination reveals tenderness in the epigas- cian’s office with complaints of a new left trium and right upper quadrant. breast mass. She denies any pain, nipple dis- charge, or skin dimpling. She has a prior his- 50. A 65-year-old man presents to the physician’s tory of breast cysts 5 years ago, treated by office with complaints of abdominal discomfort aspiration at that time. Her last mammogram and jaundice for the past 3 weeks. Past history was at age 53. Past history is pertinent for a 30 is pertinent for 30 pack-year smoking history, pack-year smoking history, prior total abdom- occasional alcohol intake, and a 5.5-mm ulcer- inal hysterectomy-bilateral salpingo-oophorec- 1 ating melanoma removed from his back 2 /2 tomy (TAH-BSO) at age 54 for leiomyomas, years ago. Examination reveals a mildly jaun- and current use of hormone replacement ther- diced patient with normal vital signs and a apy (HRT). Family history is negative for breast slightly distended abdomen with mild right disease. Examination reveals a firm, well- upper quadrant tenderness and significant defined, mobile, 1.5-cm nodule in the upper hepatomegaly. outer quadrant of the left breast without any regional lymphadenopathy. Which of the fol- 51. A 54-year-old man presents to the emergency lowing is the most appropriate next step in department on transfer from another hospi- management? tal at the request of the family. He was admit- (A) fine-needle aspiration (FNA) biopsy ted to the outside hospital 2 weeks ago with (B) discontinuation of HRT and re- abdominal pain, nausea, vomiting, and fever. examination in 4–6 weeks He was treated with antibiotics, NG tube (C) breast imaging decompression, and TPN without significant improvement. He developed jaundice 2 days (D) open surgical biopsy ago. His past history is pertinent for a 40 (E) core needle biopsy pack-year smoking history, chronic alcohol (c) ketabton.com: The Digital Library

208 6: Surgery

53. A 56-year-old woman presents to the clinic for Questions 55 and 56 routine health screening. Her concern is the development of breast cancer. She has no cur- A 35-year-old woman presents to the clinic for a dis- rent breast-related complaints. Past history is cussion on breast cancer risk. Her family history is pertinent for fibrocystic changes with atypical pertinent for a grandmother who died of breast can- ductal hyperplasia and a single fibroadenoma, cer at age 53, a mother who died of premenopausal both diagnosed by open biopsy 5 years ago. breast cancer, and one of three sisters with breast can- She smokes one pack per day and drinks one cer diagnosed at age 42. The sister with breast cancer can of beer daily. Family history is positive for underwent genetic testing and was found to have a breast cancer in her mother, diagnosed at the BRCA1 mutation. Subsequently, the 35-year-old age of 85. Current medications include a cho- woman underwent genetic testing and was found to lesterol-lowering agent, an antihypertensive, and be a carrier of the same deleterious BRCA1 mutation. HRT, which she has taken for 5 years. Physical examination is unremarkable. Mammograms 55. Which of the following ranges represents the show dense breasts, decreasing the accuracy of lifetime risk for breast cancer that should be the study, but no suspicious findings were noted. quoted for this patient? Which of the following is the most common risk (A) 0–30% factor in evaluating women for breast cancer? (B) 10–40% (A) fibrocystic changes with atypical ductal (C) 20–50% hyperplasia (D) 50–80% (B) alcohol consumption (E) 70–100% (C) positive family history (D) HRT 56. For this patient, which of the following strate- (E) age gies represent an accepted management option for her high-risk status? 54. A 42-year-old woman returns to the clinic fol- (A) yearly self-breast examinations lowing an uneventful biopsy for a well- (B) semiannual mammography defined, mobile mass. The pathology report (C) bilateral breast irradiation describes the mass as a fibroadenoma, but LCIS is identified in the breast parenchyma adjacent (D) prophylactic unilateral mastectomy to the fibroadenoma and extending to the (E) chemoprevention with tamoxifen margin of resection. She has no current ill- nesses, is on no medications, and her family Questions 57 and 58 history is negative for breast cancer. Breast A 65-year-old woman presents to the physician’s imaging studies show fatty breasts with no office for a second opinion on the management abnormal findings except for the fibroade- options for recently diagnosed breast cancer. She noma. Which of the following is the most presents with a 2.5-cm mass in the upper outer appropriate management option? quadrant of the left breast associated with a palpa- (A) re-excision of the biopsy cavity to gain ble axillary node suspicious for metastatic disease. negative margins of resection The remainder of her examination is normal. (B) ipsilateral mastectomy Mammography demonstrates the cancer and shows (C) contralateral breast biopsy no other suspicious lesions in either breast. Chest x-ray, bone scan, and blood test panel, including (D) observation including examinations and liver function tests, are normal. Family history is mammography positive for breast cancer diagnosed in her sister at (E) bilateral total mastectomies age 65. Past history is unremarkable. The first physi- cian recommended modified radical mastectomy. (c) ketabton.com: The Digital Library

Questions: 53–58 209

57. Which of the following is the most appropriate (C) No, because the success of SLN biopsy management option for locoregional control in patients over age 60 is decreased. yielding results equally effective as mastectomy? (D) No, because SLN biopsy is contraindi- (A) radical mastectomy cated when a palpable axillary node is (B) lumpectomy, irradiation, and axillary suspicious for metastatic disease. node dissection (E) No, because SLN biopsy is contraindi- (C) lumpectomy and axillary node dissection cated for tumors greater than 2 cm. (D) irradiation of the breast and axilla Questions 59 and 60 (E) quadrantectomy, irradiation, and axillary node dissection A 65-year-old woman presents to the physician’s office for evaluation of an abnormal screening 58. The patient has read about SLN biopsy. She mammogram. She denies any breast masses, nip- avidly wants to avoid the risk of lymphedema ple discharge, pain, or skin changes. Past history that her sister must endure. She asks the ques- is pertinent for hypertension. Family history is tion “Am I a candidate for a SLN biopsy positive for postmenopausal breast cancer in a sis- instead of a complete axillary dissection?” ter. She has a normal breast examination and no Which of the following is the most appropriate axillary adenopathy. The remainder of her exami- answer to her question? nation is unremarkable. An MLO view of the right (A) Yes, and if the SLN if positive, then a breast is shown in Figure 6-6a along with a mag- complete axillary dissection should be nification view of the craniocaudal (CC) film performed. (Figure 6-6b). (B) Yes, and if the SLN is negative, then an axillary dissection can be avoided.

FIG. 6-6a FIG. 6-6b (c) ketabton.com: The Digital Library

210 6: Surgery

59. Which of the following is the most likely diagnosis? (A) milk of calcium (B) LCIS with or without an invasive component (C) DCIS with or without an invasive component (D) involuting fibroadenoma (E) phyllodes tumor

60. Which of the following is the most appropriate next step in management? (A) observation, with repeat mammogram in 6 months (B) observation, with repeat mammogram on an annual basis (C) biopsy (D) lumpectomy, radiation therapy, and SLN biopsy (E) total mastectomy

Questions 61 and 62

An 83-year-old woman presents to a mammo- graphic facility for a screening mammogram. The technician notices a mass in the lateral right breast. FIG. 6-7 The patient denies any breast pain, nipple dis- charge, skin changes, or breast trauma. A right 61. Which of the following is the most likely breast CC view is shown in Figure 6-7. diagnosis? (A) papilloma (B) invasive carcinoma (C) cystosarcoma phyllodes (D) DCIS (E) fat necrosis

62. Which of the following is the most appropriate next step in management? (A) incisional biopsy (B) needle biopsy (C) lumpectomy, axillary dissection, and irradiation (D) total mastectomy (E) modified radical mastectomy (c) ketabton.com: The Digital Library

Questions: 59–65 211

Questions 63 and 64 (D) endoscopy (E) CT scan A 65-year-old woman presents to the physician’s office with a 6-month history of epigastric discom- 65. A 55-year-old man presents to the physician’s fort, poor appetite, and 10-lb weight loss. Past his- office for his yearly physical examination. He tory is pertinent for hypertension, diabetes, a 30 is asymptomatic. Past history is pertinent for pack-year smoking history, and occasional alcohol hypertension. Family history is positive for intake. Examination is unremarkable except for mild breast cancer in his mother at age 70 and epigastric tenderness to deep palpation. An abdom- colon cancer in his father at age 65. His exam- inal ultrasound reveals cholelithiasis, and one view ination is unremarkable except for guiac of a UGI x-ray series is shown in Figure 6-8. positive stool. Barium enema shows a sig- moid colon polyp. Colonoscopy confirms a 63. Which of the following is the most likely 3-cm pedunculated polyp in the sigmoid diagnosis? colon, and snare polypectomy is performed. (A) cholecystoenteric fistula Pathologic examination reveals an adenoma- (B) duodenal ulcer tous polyp with a focus of invasive carcinoma (C) gastric ulcer in the head, with a 4-mm resection margin and no tumor noted in the stalk. Which of the (D) gastric diverticulum following is the most appropriate next step in (E) duodenal diverticulum management? 64. Which of the following is the most appropriate (A) CT scan next step in management? (B) magnetic resonance imaging (MRI) scan (C) surgical resection of sigmoid (A) H2 blockers with re-evaluation by UGI in 6 months (D) observation (B) vagotomy and pyloroplasty (E) regular use of nonsteroidal anti- (C) total gastrectomy inflammatory drugs (NSAIDs)

FIG. 6-8 (Reproduced, with permission, from Zinner MJ. Maingot’s Abdominal Operations, 10th ed., Vol. 2. Stamford, CT: Appleton & Lange, 1997.) (c) ketabton.com: The Digital Library

212 6: Surgery

Questions 66 and 67 fibrillation, and chronic constipation. Examination reveals a temperature of 98.6°F, pulse rate 90/min A 55-year-old man presents to the emergency and irregularly irregular, and BP 160/90 mmHg. department with left lower quadrant abdominal Heart examination reveals irregularly irregular pain. The pain has been present for 1 week, but has rhythm with no murmurs; lung examination increased in intensity over the last 2 days associated reveals few bibasilar rales; and abdominal exami- with nausea, constipation, and dysuria. Past history nation reveals a distended, tympanic abdomen is unremarkable. Examination reveals a tempera- with mild tenderness and no rebound tenderness. ture of 101°F, pulse rate of 95/min, BP of 130/70 Plain abdominal x-rays reveal dilated loops of mmHg, and normal heart and lung examinations. bowel, and a barium enema is obtained and shown Abdominal examination reveals fullness and marked in Figure 6-9. tenderness in the left lower quadrant, with voluntary guarding and decreased bowel sounds. Laboratory tests reveal a WBC count of 18,000 with a left shift and 20–50 WBCs in the urinalysis. A CT scan of the abdomen reveals a thickened sigmoid colon with pericolonic inflammation. He is admitted to the hos- pital for treatment.

66. Which of the following is the most likely diagnosis? (A) colon cancer with contained perforation (B) ischemic colitis (C) pseudomembranous colitis (D) diverticulitis (E) pyelonephritis

67. Which of the following is the most appropriate management of this patient? (A) NPO, IV fluids, and IV antibiotics for gram-negative and anaerobic coverage (B) NPO, IV fluid hydration, followed by immediate sigmoid colon resection (C) NPO, IV fluids, and anticoagulation (D) NPO, IV fluids, evaluation of stool for Clostridium difficile toxin, and either FIG. 6-9 (Reproduced, with permission, from Zinner MJ. Maingot’s metronidazole or vancomycin antibiotic Abdominal Operations, 10th ed., Vol. 2. Stamford, CT: Appleton & therapy Lange, 1997.) (E) NPO, IV fluids, initiation of bowel preparation for elective sigmoid colon resection during the current 68. Which of the following is the most likely hospitalization diagnosis? Questions 68 and 69 (A) ischemic colitis with stricture (B) diverticulitis with obstruction A 75-year-old woman is brought to the emergency department from a nursing home for abdominal (C) cecal volvulus pain, distention, and obstipation over the last 2 days. (D) sigmoid volvulus Past history is pertinent for stroke, diabetes, atrial (E) colon cancer with obstruction (c) ketabton.com: The Digital Library

Questions: 66–73 213

69. Which of the following is the most appropriate (C) lymphoma next step in management following NG tube (D) ischemia with stricture decompression and resuscitation? (E) Crohn’s colitis with stricture (A) urgent sigmoid resection (B) nonoperative reduction by proctoscopy 71. Which of the following is the most appropriate and rectal tube therapy following colonoscopy? (C) proximal colostomy (A) proximal colostomy with mucous fistula (D) urgent operative detorsion (B) radiation therapy (E) nonoperative reduction by passage of (C) chemotherapy well-lubricated rectal tube (D) surgical resection and primary anastomosis Questions 70 and 71 (E) surgical bypass (colocolostomy) A 65-year-old man presents to the physician’s office Questions 72 and 73 for his yearly physical examination. His only com- plaint relates to early fatigue while playing golf. A 54-year-old woman presents to her physician for an Past history is pertinent for mild hypertension. opinion regarding additional therapy following cura- Examination is unremarkable except for trace tive resection of recently diagnosed colon cancer. She hematest-positive stool. Blood tests are normal underwent uncomplicated sigmoid resection for inva- except for a hematocrit of 32. A UGI series is per- sive colon cancer 4 weeks ago. The pathology revealed formed and is normal. A barium enema is per- carcinoma invading into, but not through, the muscu- formed, and one view is shown in Figure 6-10. laris propria, with one of eight positive mesenteric nodes. There was no evidence of liver metastases at the time of operation. Preoperative chest x-ray and CT scan of the abdomen showed no evidence of distant disease. Preoperative carcinoembryonic antigen (CEA) level was normal. Past history is positive for diabetes and mild hypertension. Examination is unre- markable except for a healing abdominal incision.

72. Which of the following is the correct stage of this patient’s colon cancer? (A) stage 0 (B) stage I (C) stage II (D) stage III (E) stage IV

73. Which of the following is the most appropriate recommendation regarding adjuvant therapy? FIG. 6-10 (Reproduced, with permission, from Zinner MJ. Maingot’s Abdominal Operations, 10th ed., Vol. 2. Stamford, CT: Appleton & (A) no therapy indicated Lange, 1997.) (B) 5-fluorouracil chemotherapy (C) 5-fluorouracil chemotherapy with 70. Which of the following is the most likely leucovorin diagnosis? (D) doxorubicin (Adriamycin) chemotherapy (A) diverticular disease (E) Adriamycin chemotherapy with (B) colon cancer methotrexate and cytoxan (c) ketabton.com: The Digital Library

214 6: Surgery

Questions 74 and 75 (C) colonoscopy (D) endoscopic dilation of the stricture A 62-year-old woman presents to the physician’s office with complaints of constipation. She has had (E) NPO, IV fluids, and antibiotics constipation for the last 6 months, which has wors- ened over the last month, associated with mild bloat- Questions 76 and 77 ing. She noted that her stool has become “pencil thin” A 45-year-old man presents to the physician’s office in the last month, with occasional blood, but she con- for evaluation of a skin lesion on his abdomen. He tinues to have bowel movements daily. Past history is states that the lesion has been present for 1 year, but unremarkable. Examination reveals normal vital has recently enlarged over the last 2 months. The signs and heart and lung examination. Abdominal mass is nontender, and he is otherwise asympto- examination reveals mild fullness, especially in the matic. Past history is unremarkable. Examination lower quadrants. Rectal examination shows no rectal reveals a 3-cm, pigmented, irregular skin lesion masses, but the stool is hematest positive. A barium x- located in the left lower quadrant of the abdomen, ray is obtained, and one view is shown in Figure 6-11. as shown in Figure 6-12. Heart, lung, and abdomi- nal examination are normal. There are no palpable cervical, axillary, or inguinal lymph nodes. Chest x-ray and liver function tests are normal.

FIG. 6-11 (Reproduced, with permission, from Zinner MJ. Maingot’s Abdominal Operations, 10th ed., Vol. 2. Stamford, CT: Appleton & Lange, 1997.)

74. Which of the following is the most likely diagnosis? FIG. 6-12 (Reproduced, with permission, from Hurwitz RM, Hood (A) Crohn’s disease AF.Pathology of the Skin: Atlas of Clinical-Pathological Correlation. (B) ischemia with stricture Stamford, CT: Appleton & Lange, 1998:432.) (C) rectal carcinoma (D) sigmoid volvulus 76. Which of the following is the most likely diagnosis? (E) diverticulitis with colovesical fistula (A) squamous cell carcinoma 75. Which of the following is the most appropriate (B) basal cell carcinoma next step in management? (C) Merkel cell carcinoma (A) proctoscopy and passage of a rectal tube (D) melanoma (B) proctoscopy and biopsy (E) keratoacanthoma (c) ketabton.com: The Digital Library

Questions: 74–80 215

77. Which of the following is the most appropriate next step in management? (A) wide excision with 2 cm margin (B) wide excision with 2 cm margin and SLN mapping (C) shave biopsy (D) excisional biopsy with 1–2 mm margins (E) Mohs’ surgical excision

78. A 75-year-old woman is admitted to the hospi- tal from a nursing home for abdominal pain and pneumonia. She was noted to be short of breath with increasing cough for 2 days before admission. Treatment, consisting of supple- mental oxygen, IV antibiotics, and pulmonary toilet, is instituted, with improvement within 2 days. On the third hospital day, her abdominal pain worsens. Examination reveals a mildly distended abdomen with bowel sounds but no signs of peritonitis. Remainder of examination reveals a tender bulge in the medial left thigh below the inguinal ligament. Gentle pressure FIG. 6-13 (Reproduced, with permission, from Niederhuber JE. Fundamentals of Surgery. Stamford, CT: Appleton & Lange, causes more pain but does not change the size 1998.) or shape of the bulge. Abdominal films show a nonspecific bowel gas pattern. Laboratory 79. Which of the following is the most likely analysis shows a WBC of 13,000, decreased diagnosis? from 18,000 at the time of admission. Which of the following is the most likely diagnosis? (A) small cell lung cancer (B) tuberculosis (A) incarcerated direct inguinal hernia (C) nonsmall cell lung cancer (B) lymph node with abscess (D) hamartoma (C) femoral artery aneurysm (E) abscess (D) incarcerated indirect inguinal hernia (E) incarcerated femoral hernia 80. Which of the following is the most appropriate next diagnostic test? Questions 79 and 80 (A) percutaneous needle biopsy A 65-year-old woman presents to the physician’s (B) CT scan office for her yearly physical examination. She has no (C) pulmonary function tests complaints except for a recent 10-lb weight loss. Past history is pertinent for a 40 pack-year smoking his- (D) mediastinoscopy tory, hypertension, asthma, and hypothyroidism. (E) bronchoscopy Examination reveals a thin woman with normal vital signs and unremarkable heart and abdominal exam- inations. Lung examination reveals mild wheezing and a few bibasilar rales. A chest x-ray is obtained and is shown in Figure 6-13. A chest x-ray obtained 3 years ago was normal. Yearly laboratory tests includ- ing a CBC, electrolytes, and lipid panels are normal. (c) ketabton.com: The Digital Library

216 6: Surgery

Questions 81 and 82 (D) CT scan (E) abdominal x-rays A 75-year-old man is brought to the emergency department for severe pain in the left flank and back Questions 83 through 85 of 1 hour duration. He has a prior history of a myocardial infarction and coronary artery bypass A 65-year-old man presents to the emergency grafting 8 years ago. On examination, he is found to department with sudden onset of pain and weak- have a BP of 80/50 mmHg, pulse rate of 110/min, ness of the left lower extremity of 2-hour duration. respiratory rate of 15/min, and a pulsatile, tender Past history reveals chronic atrial fibrillation follow- abdominal mass. He has had two large-bore IV lines ing a myocardial infarction 12 months ago. On placed by the paramedics. He is alert and oriented, examination, he is found to have a cool, pale left and gives consent for surgery. lower extremity with decreased strength and absent popliteal and pedal pulses. The opposite leg has a 81. Which of the following is the most appropriate normal appearance with palpable pulses. next step in management of this patient? (A) immediate consultation with cardiology 83. Which of the following is the most appropriate to assess cardiac risk for surgery, fol- first step in management of this patient? lowed by transfer to the operating room (A) echocardiography (B) resuscitation in the emergency depart- (B) anticoagulation with heparin ment with IV fluids, transfer to radiol- (C) anticoagulation with warfarin ogy for a CT scan to assess for the loca- (D) arteriography tion and degree of rupture, followed by transfer to the operating room (E) alkalinization of the urine with IV sodium bicarbonate (C) resuscitation in the emergency depart- ment with IV fluids to achieve a systolic 84. Which of the following is the treatment of BP greater than 100, followed by trans- choice for this patient? fer to the operating room (D) immediate transfer to the operating (A) r-TPA (tissue plasminogen activator) room with concomitant resuscitation infusion following anticoagulation and laparotomy (B) administration of vasodilators (E) resuscitation in the emergency depart- (C) four-compartment fasciotomy ment with IV fluids, transfer to radiol- (D) thromboembolectomy ogy for immediate aortic angiogram for (E) anticoagulation and close observation assessment of the location of the rup- ture, followed by transfer to the 85. Following successful treatment for an embo- operating room lus to the left femoral artery with no evidence of a reperfusion injury, which of the following 82. On postoperative day 3, the patient develops long-term treatments would most likely dark-colored diarrhea but remains normoten- decrease the chance of recurrent embolus? sive, on full mechanical ventilation, and is awake. Laboratory analysis reveals normal (A) anticoagulation electrolytes, blood urea nitrogen (BUN), and (B) exercise program creatinine; hematocrit of 30; and WBC of 15,000. (C) coronary artery bypass grafting Which is the most appropriate next step in (D) aortofemoral bypass grafting management? (E) placement of a vena cava filter (A) stool for C. difficile toxin test and institu- tion of metronidazole (B) sigmoidoscopy (C) air contrast barium enema (c) ketabton.com: The Digital Library

Questions: 81–91 217

Questions 86 and 87 (C) low-dose irradiation during infancy (D) chronicity of the nodule A 65-year-old man presents to the emergency department with an abrupt onset of excruciating (E) past history of Hashimoto’s disease chest pain 1 hour ago. The pain is localized to the anterior chest, but radiates to the back and neck. 89. Which of the following is the most appropriate On examination, the patient is afebrile, with a BP next step in her management? of 210/110 mmHg, pulse rate of 95/min, and a res- (A) ultrasound of the neck piratory rate of 12/min. He appears pale and (B) thyroid scintiscan sweaty. Unequal carotid, radial, and femoral (C) MRI of the neck pulses are noted. An electrocardiogram (ECG) shows nonspecific ST-T segment changes. Chest (D) CT scan of the neck and chest x-ray shows a slightly widened mediastinum and (E) FNA of the nodule normal lung fields. Questions 90 and 91 86. Which of the following is the preferred modal- A 45-year-old man presents to the physician’s office ity in establishing the diagnosis? complaining of dysphagia and retrosternal pressure (A) transcutaneous echocardiography and pain of 2-year duration. The symptoms have (B) transesophageal echocardiography worsened over the last 3 months. He has a 30 pack- (C) CT scan year smoking history and drinks beer on weekends. Vital signs include a BP of 150/90 mmHg, pulse rate (D) coronary angiography of 90/min, and respiratory rate of 12/min, with a (E) aortography normal temperature. Examination reveals a thin man with a normal heart, lung, and abdomen exam- 87. Which of the following is the first step in ination. An esophagogram reveals a 6-cm, smooth, management of this patient? concave defect in the midesophagus with sharp bor- (A) treatment with thrombolytic agents ders. Esophagoscopy reveals intact overlying (B) systemic anticoagulation mucosa and a mobile tumor. (C) control of hypertension 90. Which of the following is the most likely (D) placement of an intra-aortic balloon pump diagnosis? (E) immediate operation (A) esophageal carcinoma Questions 88 and 89 (B) bronchogenic carcinoma with invasion of the esophagus A 39-year-old woman presents to the physician’s (C) benign esophageal polyp office for evaluation of a palpable nodule in the (D) leiomyoma neck of 2 years’ duration. Her past history is perti- (E) lymphoma nent for Hashimoto’s disease diagnosed 5 years ago, for which she takes thyroid hormone. She has a 91. Which of the following is the most appropriate history of low-dose chest irradiation for an enlarged next step? thymus gland during infancy. On examination, a 2.5-cm nodule is palpable in the left lobe of the (A) repeat esophagoscopy with biopsy thyroid and is firm and nontender. (B) thoracotomy with extramucosal resection (C) thoracotomy with esophageal resection 88. Which of the following portions of her history (D) radiation therapy increases the risk for thyroid cancer? (E) chemotherapy (A) age group of 20–40 years (B) female gender (c) ketabton.com: The Digital Library

218 6: Surgery

Questions 92 and 93 show normal electrolytes and a hematocrit of 30. A chest x-ray is unremarkable. Volume A 49-year-old woman presents to her physician resuscitation, gastric lavage, and NG tube with dysphagia, regurgitation of undigested food decompression are initiated. Which of the fol- eaten hours earlier, and coughing over the last 6 lowing is the most appropriate diagnostic months. She was hospitalized 1 month ago for aspi- test? ration pneumonia and successfully treated with antibiotics. Examination reveals a thin-appearing (A) barium esophagogram woman with normal vital signs and unremarkable (B) water-soluble contrast esophagogram chest, heart, and abdominal examination. A UGI (C) esophagoscopy contrast study is performed and reveals a pharyn- (D) CT scan goesophageal (Zenker’s) diverticulum. (E) angiogram 92. Which of the following statements is true Questions 95 and 96 regarding Zenker’s diverticula? (A) Cervical dysphagia is related to the size A 68-year-old man presents to the physician’s office of the diverticulum. complaining of progressive dysphagia over the last 3 months associated with mild chest discomfort. He (B) Pharyngoesophageal diverticula are of reports a 15-lb weight loss, a 30 pack-year smoking the pulsion type. history, and occasional alcohol intake. The physical (C) Pharyngoesophageal diverticula are true examination, including vital signs, is unremarkable. diverticula. A chest x-ray was normal, and a barium esopha- (D) Pharyngoesophageal diverticula are gogram shows an irregular filling defect in the congenital in origin. distal third of the esophagus with distortion and (E) Upper esophageal sphincter function is narrowing of the lumen. usually normal. 95. Which of the following is the most likely 93. Which of the following is the most important diagnosis? aspect of treatment? (A) esophagitis with stricture (A) resection of the diverticulum (B) esophageal carcinoma (B) cricopharyngeal muscle myotomy (C) lung carcinoma with invasion into the

(C) H2 blockers esophagus (D) elevation of the head of the bed (D) lymphoma (E) diverticulopexy (E) achalasia

94. A 55-year-old man presents to the emergency 96. Which of the following is the most appropriate department at 5 a.m. complaining of vomit- next step in management? ing blood. After binge drinking last night, the (A) CT scan patient began to vomit repeatedly. After a number of episodes, the patient noted blood in (B) esophagoscopy the vomitus, followed by a melanotic stool 5 (C) MRI scan hours later. His past history is pertinent for (D) surgical resection ethanol abuse and a 40 pack-year smoking his- (E) bronchoscopy tory. Vital signs reveal a BP of 100/60 mmHg, pulse rate of 95/min, respiratory rate of Questions 97 through 99 12/min, and temperature of 97°F. Examination reveals a thin man with normal chest, cardiac, A 30-year-old man presents to the emergency and abdominal findings. Rectal examination department with sudden onset of severe epigastric reveals heme-positive stool. Laboratory data pain and vomiting 3 hours ago. He reports a 6-month (c) ketabton.com: The Digital Library

Questions: 92–101 219

history of chronic epigastric pain occurring nearly examination reveals normal vital signs, mild epi- every day and relieved by antacids. On examination, gastric pain with deep palpation, and mildly heme- he appears sweaty and avoids movement. Vital positive stool. An evaluation for recurrence of a gas- signs reveal a temperature of 100°F, BP of 100/60 tric ulcer is recommended. mmHg, pulse rate of 110/min, and respiratory rate of 12/min. The remainder of his examination reveals 100. Which of the following tests is the most reliable diminished bowel sounds and a markedly tender method for diagnosing a gastric ulcer? and rigid abdomen. A chest x-ray and abdominal (A) UGI barium x-rays films reveal pneumoperitoneum. (B) fiberoptic upper endoscopy 97. Which of the following is the most likely (C) CT scan diagnosis? (D) endoscopic ultrasound (A) small-bowel obstruction (E) MRI (B) dead bowel 101. In this patient, a benign gastric ulcer was (C) perforated colon carcinoma found, and he was placed on a proton-pump (D) perforated duodenal ulcer inhibitor and triple antibiotics for Helicobacter (E) perforated gastric ulcer pylori. He returns to the physician’s office 3 months later with similar complaints and, on 98. Which of the following is the most appropriate re-evaluation, the gastric ulcer was found to next diagnostic test? persist. Which of the following is the most appropriate next step in management? (A) CT scan (B) UGI water-soluble contrast study (A) a second trial of proton-pump inhibitors (C) lower GI water-soluble contrast study with triple antibiotics and re-evaluation in 2 months (D) abdominal ultrasound (B) a trial of H blockers with triple antibi- (E) none of the above 2 otics and re-evaluation in 2 months 99. Which of the following is the most appropriate (C) a trial of sucralfate and re-evaluation in next step in management? 2 months (D) surgical management (A) immediate laparotomy (E) a trial of prostaglandins and (B) nonoperative management with NG re-evaluation in 2 months decompression and antibiotics (C) fluid resuscitation Questions 102 through 104 (D) administration of H blockers 2 A 65-year-old man presents to the physician’s office (E) placement of a central venous line for his yearly examination. His past history is perti- nent for a 40 pack-year smoking history and colon Questions 100 and 101 cancer 3 years ago for which he underwent a sigmoid A 55-year-old man presents to the physician’s office colectomy. The most recent colonoscopic follow-up complaining of upper abdominal pain of 2 months’ 3 months ago was negative. His physical examina- duration. The pain is described as gnawing, local- tion is normal. Laboratory results show a normal ized to the upper midline, and associated with nau- CBC and electrolytes, markedly elevated cholesterol, sea. The pain is exacerbated by food, and there is an and a CEA of 12 compared to values of less than 5 associated 20-lb weight loss over 2 months. His obtained every 6 months since colectomy. A repeat past history is pertinent for a 30 pack-year smoking CEA 4 weeks later was 15, and liver function tests history, occasional alcohol intake, and a prior his- revealed a minimally elevated alkaline phosphatase, tory of a benign gastric ulcer 5 years ago. Physical with normal transaminases and bilirubin. (c) ketabton.com: The Digital Library

220 6: Surgery

102. Which of the following is the most appropriate 105. Which of the following is the most likely next diagnostic test in this patient? diagnosis? (A) positron emission tomography (PET) scan (A) metastatic carcinoma (B) radionuclide liver scan (B) infectious parotitis (C) ultrasound (C) pleomorphic adenoma of the parotid (D) CT scan (D) Hodgkin’s disease (E) MRI scan (E) reactive cervical lymphatic hyperplasia

103. The imaging studies demonstrate three lesions 106. Which of the following is the best next step in in the right hepatic lobe suspicious for metasta- the management of this patient? tic disease, each measuring 3–4 cm in diameter. (A) antibiotics There was no evidence of extrahepatic disease. Which of the following is the most appropriate (B) excisional biopsy next step in management? (C) observation with re-evaluation in 2–4 weeks (A) systemic chemotherapy (D) superficial parotidectomy (B) intra-arterial chemotherapy through the (E) chest x-ray hepatic artery (C) surgical resection Questions 107 and 108 (D) radiation therapy to the liver (E) repeat imaging studies in 3 months to A 10-month-old infant presents to the emergency determine the growth rate of the disease department with a 24-hour history of low-grade fever and anorexia. The parents report several 104. In your discussion with the patient regarding episodes in which the child has been suddenly the risks and benefits of the different manage- inconsolable and crying, followed by periods of ment options listed above, which of the fol- lethargy. He has had nonbilious vomiting and sev- lowing values should you quote regarding the eral loose stools. On examination, the infant is pale expected 5-year survival rate following cura- and mildly dehydrated. His abdomen is soft and tive surgical resection? nondistended, with fullness to palpation in the right upper quadrant. The child passed another stool in (A) 5–10% the emergency department (see Figure 6-14). (B) 15–20% (C) 25–35% (D) 40–50% (E) 60–70%

Questions 105 and 106

A previously healthy 45-year-old man presents with a 9-month history of a slow-growing, painless right neck mass. He is a nonsmoker and has no signifi- cant past medical history. On examination, there is a nontender, discrete, 3-cm mass over the angle of the right mandible. Facial muscle function and sensa- tion are normal. An oropharyngeal examination is normal. FIG. 6-14 (c) ketabton.com: The Digital Library

Questions: 102–111 221

107. Which of the following is the most likely 110. A 23-year-old man presents to the emergency diagnosis? department with a soft-tissue injury to the left lower extremity. The injury was sustained 8 (A) gastroenteritis hours earlier in a motorcycle accident on a (B) intussusception gravel road. On examination, the patient has a (C) midgut volvulus 7-cm deep laceration to the calf, with visible (D) Meckel’s diverticulum road debris. He had full tetanus immunization (E) juvenile rectal polyp as a child and a tetanus booster immunization at age 15. Appropriate management of this 108. Which of the following is the most appropriate injury would include which of the following? next step in the diagnostic evaluation and man- (A) irrigation and debridement of the wound agement of this patient? (B) irrigation and debridement of the (A) proctoscopy wound; tetanus toxoid and tetanus (B) oral rehydration and stool cultures immune globulin (C) IV fluid rehydration and a hydrostatic (C) irrigation and debridement of the barium enema wound; tetanus toxoid (D) technetium scan (D) irrigation and debridement of the (E) IV fluid rehydration, NG decompres- wound; IV antibiotics sion, and a UGI contrast study (E) tetanus toxoid and IV antibiotics

109. A 65-year-old diabetic man presents to the 111. A 6-year-old boy presents to the emergency emergency department with a history of a pen- department with a painful, markedly swollen etrating wound to his buttock by a wooden elbow. While ice-skating, he fell with his arm stump while working in his garden 24 hours outstretched. Radiographs of the elbow earlier. On examination, he is febrile, the tissue demonstrate a displaced, supracondylar frac- around the wound is violaceous in color, and ture of the humerus. On examination, there is several bullae and crepitus are noted in the pain on passive flexion at the wrist and a buttock. The drainage from the wound is foul decreased radial pulse, with diminished capil- smelling, watery, and grayish in appearance. lary refill in the hand. Which of the following The optimal treatment for this patient would is the most appropriate management of this include which of the following? injury? (A) high-dose IV penicillin G and broad- (A) admission to hospital for close observa- spectrum antibiotics tion, with immobilization of the elbow (B) high-dose IV penicillin G, broad- at 90° of flexion spectrum antibiotics, and local wound (B) closed reduction with percutaneous care with unroofing of bullae and pinning under general anesthesia culture of wound drainage (C) open reduction and pinning under (C) high-dose IV penicillin G, broad- general anesthesia spectrum antibiotics, with surgical (D) open reduction with pinning, and debridement only if and when there is exploration of the brachial artery no improvement with antibiotics (E) open reduction with pinning, explo- (D) radical surgical debridement ration of the brachial artery, and decom- (E) high-dose IV penicillin G, broad- pression fasciotomy of the forearm fas- spectrum antibiotics, radical surgical cial compartments debridement, and hyperbaric oxygen therapy (c) ketabton.com: The Digital Library

222 6: Surgery

Questions 112 and 113 114. A 10-day-old infant presenting with bilious vomiting, paucity of gas on plain radiographs, A 35-year-old woman is involved in a motor vehicle and duodenal obstruction on UGI contrast crash, sustaining a severe pelvic fracture, with dis- study (Figures 6-15 and 6-16) ruption of the pelvic ring. In the trauma resuscitation room, she is confused and tachypneic, with a blood pressure of 90 mmHg systolic and a heart rate of 130/min. Laboratory investigations include serum electrolyte analysis, revealing a sodium of 139, a chloride of 103, and a bicarbonate of 14 meq/L.

112. This patient demonstrates which of the following? (A) nonanion gap metabolic acidosis (B) anion gap metabolic acidosis (C) metabolic alkalosis (D) respiratory acidosis (E) normal serum electrolytes

113. Which of the following is the most appropriate management of this acid-based derangement? (A) administration of sodium bicarbonate to correct the base deficit (B) restoration of blood volume with aggressive IV fluid resuscitation (C) IV hydrochloric acid (D) intubation and hyperventilation (E) this patient has no acid-based abnormality FIG. 6-15

DIRECTIONS (Questions 114 through 116): For each numbered item, select the ONE best lettered option that is most closely associated with it. Each lettered option may be selected once, more than once, or not at all. For each newborn with vomiting and illustrated radiographs, select the most likely diagnosis. (A) congenital hypertrophic pyloric stenosis (B) annular pancreas (C) duodenal atresia (D) midgut volvulus (E) intussusception (F) imperforate anus (G) Meckel’s diverticulum (H) meconium ileus (I) Hirschsprung’s disease (J) jejunal atresia FIG. 6-16 (c) ketabton.com: The Digital Library

Questions: 112–116 223

115. A neonate with bile-stained vomiting, abdom- inal distention, dilated loops of bowel on plain radiographs, and a small-caliber colon on con- trast enema (Figure 6-17)

FIG. 6-18

DIRECTIONS (Questions 117 through 136): Each of the numbered items or incomplete statements in this section is followed by answers or by com- pletions of the statement. Select the ONE lettered FIG. 6-17 answer that is BEST in each case.

Questions 117 and 118 116. A 1-day-old infant with Down syndrome, feed- ing intolerance, bilious vomiting, and a double A 28-year-old man with a past history of bilateral bubble on plain radiographs (Figure 6-18) orchiopexy for cryptorchidism presents with a pain- less, unilateral right scrotal enlargement. On exami- nation, there is a palpable right testicular mass and enlarged inguinal nodes. Scrotal ultrasonography demonstrates heterogeneity of the testis, with an associated hydrocele. A CT scan of the abdomen and pelvis demonstrated right-sided retroperitoneal adenopathy. CT scan of the chest is normal. (c) ketabton.com: The Digital Library

224 6: Surgery

117. Which of the following would help confirm 119. Which of the following is the most likely the diagnosis? diagnosis? (A) transscrotal needle biopsy (A) noncommunicating hydrocele (B) transscrotal aspiration of the hydrocele (B) inguinal adenitis for cytology (C) reducible inguinal hernia (C) radical orchiectomy through an inguinal (D) incarcerated inguinal hernia incision (E) undescended testes (D) transscrotal exploration and orchiectomy (E) laparotomy with pelvic and retroperi- 120. Which of the following is the most appropriate toneal node dissection management at this time? (A) antibiotics 118. Staging workup and surgery reveal a semi- (B) reassurance to the parents that the abnor- noma of the testicle, with positive inguinal and mality will resolve without intervention retroperitoneal nodes. Therapeutic manage- ment for this patient is which of the following? (C) referral to the emergency department for immediate surgical consultation (A) external beam radiotherapy (D) referral for elective surgical repair (B) multidrug combination chemotherapy (E) scrotal support (C) combination radiotherapy and multidrug chemotherapy 121. Several weeks later, the child presents to the (D) clinical surveillance emergency department with a 4-hour history of (E) laparotomy with pelvic and retroperi- irritability. He has had one episode of nonbil- toneal node dissection ious vomiting and has refused to breast-feed. In the emergency department, the infant appears Questions 119 through 121 inconsolable. He is afebrile, and his abdomen is mildly distended but soft. On removal of his In a 6-month-old previously healthy male infant, an diaper, the same abnormality is documented abnormality is revealed during a routine diaper (see Figure 6-19). Which of the following is the change, as illustrated in Figure 6-19. The parents most appropriate management at this time? have noted this finding on and off on several occa- sions over the last month. On each occasion, the child (A) urgent surgical exploration has been feeding well, and is content and playful. (B) systemic antibiotics (C) elective surgical repair (D) sedation with manual reduction and arrangements for elective surgical repair (E) sedation with manual reduction, admis- sion, rehydration, and surgical repair within 24–48 hours

Questions 122 and 123

A 70-year-old man with a 50 pack-year history of smoking presents with a 6-week history of intermit- tent, painless, gross hematuria and urinary fre- quency. There are no masses palpable on abdominal examination, and rectal examination is normal. FIG. 6-19 Urinalysis confirms the presence of hematuria, and urine culture is negative. (c) ketabton.com: The Digital Library

Questions: 117–126 225

122. Which of the following is the most appropriate (A) radioisotope scanning initial diagnostic evaluation of this patient? (B) radioisotope scanning with preimaging (A) plain abdominal radiographs and an phenobarbital administration intravenous pyelogram (IVP) (C) abdominal ultrasound (B) voiding cystourethrogram (D) CT scan of the abdomen (C) cystourethroscopy (E) MRI scan of the abdomen (D) abdominal ultrasound (E) urine for cytology Questions 125 and 126 During diagnostic evaluation, a 14-year-old girl 123. The initial diagnostic evaluation does not reveal with menorrhagia, frequent nosebleeds, and iron- any abnormalities. Which of the following is deficiency anemia is found to have a low platelet the best next step in the diagnostic workup? count with a normal coagulation profile. Bone mar- (A) an abdominal CT scan row biopsy reveals abundant megakaryocytes. On (B) cystourethroscopy and urinary cytology abdominal examination, no organomegaly is noted. (C) a transrectal ultrasound 125. Which of the following is the most appropriate (D) exploratory laparoscopy initial therapy for this patient? (E) re-evaluation in 2–4 weeks, with repeat urinalysis and urine culture (A) splenectomy (B) platelet transfusion when peripheral 124. A 7-week-old, breast-fed, term infant presents platelet count drops below 50,000/mL with increasing jaundice, abdominal distention, (C) systemic steroids and abnormal stools (Figure 6-20). Liver function (D) chemotherapy tests demonstrate a conjugated hyperbiliru- (E) expectant, with intervention only if the binemia, mildly elevated transaminases, and patient develops significant clinical an elevated gamma-glutamyl transpeptidase. bleeding TORCH (congenital infection complex, includ- ing toxoplasmosis, rubella, cytomegalovirus, 126. The patient has a satisfactory response to the and hepatitis) serology and screening for initial therapeutic intervention, but over 6–12 inborn errors of metabolism are negative. As months’ time, the response is less dramatic and part of the diagnostic evaluation, the most sen- shorter in duration. There are signs and symp- sitive imaging study in this clinical setting toms of increasing side effects from therapy. would be which of the following? The next step in management should be to rec- ommend which of the following? (A) partial splenectomy (B) splenectomy (C) increase in steroid dose and frequency (D) bone marrow transplant (E) plasmapheresis

FIG. 6-20 (c) ketabton.com: The Digital Library

226 6: Surgery

127. A 50-year-old man is admitted to the hospital (A) anti-inflammatory medication and with a UGI bleed from acute erosive gastritis, application of ice secondary to chronic nonsteroidal anti-inflam- (B) elastic wrist support, analgesics, and matory use. His hematocrit is 28%. With fluid restricted activity for 1–2 weeks resuscitation, his blood pressure normalizes, but (C) presumptive diagnosis of a scaphoid he has a persistent hyperdynamic precordium, fracture, with application of a wrist tachycardia, and flow murmur on auscultation. splint, and repeat x-rays in 10–14 days He complains of shortness of breath on ambu- (D) presumptive diagnosis of a scaphoid lation. An ECG shows depressed ST-T segments. fracture, with application of a short-arm Which of the following is the next appropriate cast including the thumb step in management? (E) presumptive diagnosis of a scaphoid (A) initiation of iron supplementation therapy fracture, application of a short-arm cast (B) supplemental oxygen including the thumb, and removal of the (C) continued IV fluid resuscitation cast, with repeat x-rays in 10–14 days (D) initiation of a calcium channel blocker 130. A previously healthy 45-year-old woman is (E) blood transfusion involved in a motor vehicle crash, sustaining multiple rib fractures, a complex duodenal 128. A previously healthy 28-year-old woman devel- injury, and a fractured pelvis. She is ventilated ops significant postpartum hemorrhage, with a in the ICU. Because of a persistent high-output rapid drop in hematocrit to 18%. Despite duodenal fistula, the patient has required pro- aggressive IV fluid resuscitation, the patient has longed parenteral alimentation. During her a persistent tachycardia, labile systolic blood ICU course, the patient develops diarrhea, pressure, and poor urine output. Ongoing mental depression, alopecia, and perioral and resuscitation includes emergency transfusion periorbital dermatitis. Administration of which with 2 units of O-negative packed red blood of the following trace elements are most likely cells. During transfusion of the second unit, the to reverse these complications? patient develops chills, fever, vomiting, and hypertension. These symptoms are most likely (A) iodine the result of which of the following? (B) zinc (A) a febrile nonhemolytic transfusion (C) selenium reaction (D) silicon (B) an anaphylactic transfusion reaction (E) tin (C) ABO incompatibility with acute hemolytic transfusion reaction Questions 131 and 132 (D) delayed hemolytic transfusion reaction A 70-year-old man is admitted to the ICU after repair (E) acute bacterial infection transmitted in of an abdominal aortic aneurysm. He has a prior the blood product history of hypertension and mild congestive heart fail- ure, which were adequately controlled with digoxin 129. A 22-year-old professional basketball player falls and diuretics. To facilitate perioperative management, on his outstretched hand during a scrimmage a Swan-Ganz (multilumen pulmonary artery) catheter game. He has mild swelling at the wrist and ten- was inserted in the operating room. During the first derness to palpation in the anatomic snuffbox. few hours postoperatively, the patient is noted to have No fracture is visible on multiple radiographs of a blood pressure of 140/70 mmHg, heart rate of the wrist and hand. Which of the following is the 110/min, flat neck veins, a pulmonary arterial wedge most appropriate management of this patient? pressure of 9 mmHg, and poor urine output. (c) ketabton.com: The Digital Library

Questions: 127–136 227

131. Which of the following is the most appropriate 134. A 40-year-old alcoholic is brought to the emer- next step in management of this patient? gency department with frostbite to both lower extremities. His core body temperature is 36°C. (A) IV furosemide Which of the following is the most appropriate (B) a bolus of IV crystalloid initial treatment for the patient’s thermal (C) a dopamine infusion injury? (D) a nitroprusside infusion (A) sympathectomy without any delay (E) IV digoxin administration (B) debridement of devitalized tissues 132. Several hours after this intervention, the patient (C) slow rewarming at room temperature is reassessed. The blood pressure is 150/85 (D) slow rewarming with dry heat mmHg, heart rate is 90/min, neck veins are (E) rapid rewarming in warm water distended, and the pulmonary arterial wedge pressure is 17 mmHg. Urine output is still low Questions 135 and 136 in volume. At this point, management should be which of the following? A 50-year-old woman with a history of essential hypertension presents to the emergency department (A) IV furosemide with sudden onset of a severe headache, nausea and (B) a bolus of IV crystalloid vomiting, and photophobia. On examination, her BP (C) a dopamine infusion is 160/100 mmHg. She is mildly confused and has (D) a nitroprusside infusion nuchal rigidity, without focal neurologic signs. (E) IV digoxin administration 135. Which of the following is the most likely diagnosis? 133. A 19-year-old previously healthy man is an unbelted driver of a motor vehicle involved in (A) meningitis a front-end collision. On arrival in the emer- (B) ruptured cerebral aneurysm gency department, the patient is noted to have (C) hemorrhagic stroke stridor, with marked respiratory distress, and (D) ischemic cerebrovascular accident an oxygen saturation of 88% despite 100% oxygen by mask. He has obvious extensive (E) transient ischemic attack facial injuries, a flail chest, and poor chest 136. Once the diagnosis has been confirmed, which expansion. Bag-mask-valve ventilation is inef- of the following is the next most important step fective. Which of the following is the most in patient management? appropriate next step in management? (A) admission to the ICU, close monitoring, (A) orotracheal intubation and aggressive treatment of hypertension (B) nasotracheal intubation (B) urgent surgical intervention with (C) cricothyroidotomy aneurysm clipping (D) tracheostomy (C) admission to the ICU, close monitoring, (E) placement of bilateral chest tubes and IV antibiotics (D) serial lumbar punctures to drain cerebrospinal fluid (CSF) (E) anticoagulation and antiplatelet therapy (c) ketabton.com: The Digital Library

Answers and Explanations

1. (C) This is an open fracture, and management indicated. Azathioprine and 6-mercaptopurine constitutes an orthopedic emergency. Tetanus are immunosuppressive agents that may be prophylaxis is indicated because the soft- beneficial in the treatment of steroid refractory tissue injury is a tetanus-prone wound. An colitis, but they are not indicated in the man- open fracture is associated with a high risk of agement of an acute toxic exacerbation. Opioid osteomyelitis. Systemic antibiotics should be antidiarrheals are contraindicated, because started in the emergency department and they may increase colonic distention and continued postoperatively. Optimal local increase the risk of perforation. Colonoscopy wound irrigation and debridement is may also cause increased colonic distention achieved under general anesthesia. This frac- with perforation. Urgent surgery in a patient ture is severely comminuted and most likely with toxic megacolon should consist of abdom- unstable. Fracture stabilization can be accom- inal colectomy, Hartmann’s procedure (closure plished with internal fixation or application of the rectal stump), and ileostomy. Ileal pouch- of an external fixation device. The soft-tissue anal anastomosis is a lengthy procedure, and is defect associated with an open fracture considered only for elective reconstruction. should not be closed primarily. It may require When performed in a systemically ill patient further debridement. With aggressive local undergoing emergency colectomy of an wound care, delayed closure may be possible unprepped colon, there are increased risks of if the wound remains clean. Local wound irri- anastomotic complications. Ileorectal anasto- gation and debridement may be limited by mosis is no longer appropriate for the manage- patient discomfort. The addition of antibi- ment of ulcerative colitis because of the retained otics to the irrigation solution is of no addi- diseased rectal mucosa, with concomitant risk tional benefit. Closed reduction would not be of malignancy. (Townsend et al., 2004, p. 1431; Way possible in this patient because the fracture is and Doherty, 2006, pp. 722–729) severely comminuted. Furthermore, a long- leg cast will not provide adequate immobi- 4. (C) lization of the unstable fracture fragments. (Townsend et al., 2004, pp. 557–561) 5. (D)

2. (E) Explanations 4 and 5

3. (C) This infant has an omphalocele. This is a result of failure of the abdominal muscles to close in Explanations 2 and 3 the midline at the umbilical cord. The abdomi- nal wall defect is therefore midline, with vis- This patient presents with an acute exacerba- cera enclosed in a sac composed of amniotic tion of ulcerative colitis with systemic toxicity. membranes. Children with omphalocele may Toxic megacolon is potentially life threatening have other associated anomalies. Infants with and requires aggressive fluid resuscitation, abdominal wall defects are at risk for fluid and bowel rest, and systemic antibiotics. High-dose temperature loss, and infection. Therefore, the steroids are initiated to treat the colonic inflam- initial management consists of measures to mation. If there is no clinical improvement after decompress the gastrointestinal tract, fluid 48 hours of medical therapy, urgent surgery is resuscitation, IV antibiotics, and placing the

228 (c) ketabton.com: The Digital Library

Answers: 1–12 229

viscera in a warm, moist occlusive dressing. An Mammographic follow-up is appropriate. A umbilical hernia results when the umbilical ring repeat study at 6 months would be consid- does not close, with viscera enclosed in a sac ered if no prior films are available in order to covered by peritoneum and skin. Many of these assess stability of a new mammographic may spontaneously close on their own. finding. Though ultrasound is often per- Therefore, surgical intervention is restricted to formed for solid masses, the amount of cal- those children with a persistent fascial defect. cification in the mass would lead to artifact, Omphalitis results from bacterial infection at making interpretation difficult. Biopsy would the base of the cord and is well treated with not be suggested based on the characteristic antibiotics to cover skin organisms. Gastroschisis mammographic findings. Other surgical pro- is a congenital evisceration, located to the right cedures would not be indicated. (Donegan and of the umbilical cord, and thought to be related Spratt, 2002, pp. 71–73, 294) to obliteration of one of the umbilical veins prior to birth. As with omphalocele, the child needs 9. (C) NG decompression, IV fluids and antibiotics, and a warm moist occlusive dressing. In gas- 10. (E) troschisis, the bowel may be at risk of mechanical or vascular compromise, thus urgent surgical Explanations 9 and 10 intervention is required. (Brunicardi et al., 2005, pp. 1502–1503; Greenfield et al., 2005, pp. 1878–1881; The most common cause of a nipple discharge Townsend et al., 2004, pp. 2116–2117) is an intraductal papilloma. Galactography can often demonstrate a filling defect in the termi- 6. (B) Meckel’s diverticulum typically presents in nal ducts as is demonstrated in the figure. Duct young children with painless rectal bleeding. ectasia may also cause a discharge, but the dis- The etiology of the bleeding is from ulceration charge is usually thick and pasty. Carcinoma is of the adjacent ileum that is bathed by the acid an uncommon cause of nipple discharge. The produced from the ectopic gastric mucosa in next step in management would be excision of the diverticulum. Bleeding may result in hypo- the terminal duct containing the papilloma. volemia, requiring aggressive resuscitation Repeat cytology would not be useful because a before any surgical intervention. Diagnosis is negative result does not preclude biopsy. made by technetium scan, with the child pre- Observation would not be prudent, because a diagnosis of carcinoma must be excluded, and treated with H2 blocks to increase the sensitivity. Colonoscopy is reserved for those children the papilloma may enlarge, requiring a more with a negative Meckel’s scan. Acid suppres- extensive operation at a later date. A more sion therapy is not indicated. At surgical explo- extensive operation, such as a central lumpec- ration, the diverticulum and adjacent ileum are tomy or mastectomy, would be unnecessary because the lesion is benign. (Donegan and Spratt, removed. (Brunicardi et al., 2005, pp. 1495–1496; 2002, pp. 95–97, 318–319; Greenfield et al., 2005, pp. Greenfield et al., 2005, pp. 1917–1919; Townsend et al., 1260–1261) 2004, pp. 1366–1368)

7. (D) 11. (D)

8. (A) 12. (A)

Explanations 7 and 8 Explanations 11 and 12

The mammographic appearance of popcorn- The most likely diagnosis based on the chest x- like, coarse calcifications in the breast is char- ray and UGI contrast study is a paraesophageal acteristic of an involuting, or degenerating hernia involving the gastric antrum and proximal fibroadenoma in a postmenopausal woman. duodenum. Although some contrast is seen in (c) ketabton.com: The Digital Library

230 6: Surgery

the transverse colon from that used during flu- associated with colicky abdominal pain and oroscopic placement of the NG tube, most of diarrhea. The pathognomonic sign on clinical the contrast in the UGI study does not leave the examination is a palpable “olive” in the epi- stomach and proximal duodenum, suggesting gastrium or right upper quadrant. Abdominal some degree of obstruction. A sliding hiatal ultrasound is operator dependent, but with hernia involves movement of the gastroe- expertise in interpretation of the study, the sophageal junction into the mediastinum above thickened elongated pyloric channel can be the diaphragmatic hiatus. The figure shows the demonstrated. A UGI contrast study may show gastroesophageal junction below the diaphrag- the classic “shouldering” of the pyloric muscle, matic hiatus. Hernias of Bochdalek and with a “string sign”; this also requires expertise Morgagni are congenital diaphragmatic her- in performing the examination, and other nias involving the posterorlateral and paraster- causes of pyloric outlet obstruction, such as nal locations, respectively. Eventration is an pylorospasm may be misinterpreted as a posi- attenuation of the central portion of the tive study. Surgical exploration should be diaphragm with resultant upward displace- reserved for those patients in whom the diag- ment. The most appropriate treatment is nosis has been confirmed and only after the laparotomy or laparoscopy with reduction of infant has received fluid resuscitation. Infants the hernia and repair. Thoracotomy or thora- with gastric outlet obstruction develop a coscopy would not allow easy access to the hypochloremic, hypokalemic metabolic alkalo- abdominal contents and is associated with sis. This is secondary to the loss of chloride in increased morbidity. Delaying surgery may the gastric contents, and the renal reabsorption result in strangulation and ischemia. Because of sodium in exchange for potassium and the patient did not demonstrate any respiratory hydrogen. Carbonic anhydrase converts car- compromise, endotracheal intubation was not bonic acid to hydrogen and bicarbonate ions, indicated. Endoscopy is useful to evaluate for allowing hydrogen to be excreted in the urine, mucosal erosions in the case of gastrointestinal with retention of the bicarbonate. Hence, with bleeding. (Townsend et al., 2004, pp. 1164–1166) the metabolic alkalosis, there is a “paradoxical aciduria.” Hypernatremic, hyperchloremic, 13. (D) hypokalemic metabolic acidosis develops in infants with diarrhea. Infants with gastroe- 14. (B) sophageal reflux do not usually develop signif- icant electrolyte derangements. Infants with 15. (B) pyloric stenosis will usually require a period of fluid resuscitation to correct hypovolemia as 16. (D) well as electrolyte and acid-base abnormalities. This is followed by a pyloromyotomy. Infants Explanations 13 through 16 with vomiting and diarrhea from viral gas- troenteritis are often successfully managed with Infants with pyloric stenosis usually present oral rehydration. Prokinetic agents have been after the third week of life with symptoms of used in the management of gastroesophageal progressive pyloric outlet obstruction second- reflux. Soy formulas or elemental formulas are ary to increasing hypertrophy of the pyloric recommended for the infant with a milk protein muscle. There are often clinical signs of dehy- allergy. (Brunicardi et al., 2005, pp. 1486–1487; dration, but the infant usually appears well and Greenfield et al., 2005, pp. 1914–1915; Townsend et al., is eager to feed. Viral gastroenteritis and urinary 2004, pp. 2107–2108) tract sepsis may be associated with signs of such systemic illness as lethargy, poor feeding, and, 17. (D) in some cases, fever. Gastroesophageal reflux more typically presents with a history of regur- 18. (B) gitation since birth. Milk protein allergy is often (c) ketabton.com: The Digital Library

Answers: 13–23 231

Explanations 17 and 18 20. (B) This patient has a left tension pneumotho- rax, a diagnosis established based on symp- The clinical signs and symptoms of a ureteral toms and clinical examination. The patient is calculus are secondary to sudden obstruction of hypoxic, with respiratory distress, and demon- a hollow viscus, with visceral referred pain from strates deviation of the mediastinum to the con- loin to groin. The pain is severe and colicky in tralateral side. Hypotension is from the nature, with ureteral peristalsis against the mediastinal shift that compromises venous obstruction. This is often associated with reflex return and not from hypovolemia. Therefore, vomiting mediated by visceral stretch and pain aggressive fluid resuscitation is not indicated. fibers. Typically, the patient is restless and cannot A chest x-ray is unnecessary and will delay find a position of comfort. Urinary urgency and definitive life-saving intervention. The patient hematuria are common. Torsion of the testes pro- requires urgent decompression with a large- duces sudden scrotal pain, and may have asso- bore needle in the second intercostal space ciated vague lower abdominal pain and anteriorly, followed by insertion of a chest tube. vomiting. Pyelonephritis is associated with flank Although assisted ventilation can improve oxy- pain and costovertebral angle tenderness that is genation, positive pressure may increase the progressive in severity and constant in nature. pneumothorax if initiated before adequate Appendicitis will present with vague perium- decompression. (Townsend et al., 2004, p. 508; Way bilical pain, migrating to the right lower quad- and Doherty, 2006, pp. 223–224) rant with the development of peritonitis. In the latter stages, the patient will lie quietly, as move- 21. (A) ment exacerbates the pain from peritoneal irri- tation. By increasing hydration and adequate 22. (C) analgesia, most patients will pass ureteral stones spontaneously. An imaging study should be 23. (B) obtained in all patients presenting with symp- toms of urinary calculi. Nonenhanced CT scan Explanations 21 through 23 will identify the location of the stone, size, and number of stones. This information assists with Hemorrhage associated with a Meckel’s diver- planning further management options, including ticulum classically presents with painless rectal referral for lithotripsy or cystoscopy and retro- bleeding in the absence of other gastrointesti- grade ureteroscopy. (Brunicardi et al., 2005, pp. nal symptoms. The amount of hemorrhage 1546–1547; Greenfield et al., 2005, pp. 2078–2079; may be enough to result in hypovolemia, with Townsend et al., 2004, p. 2299) pallor, tachycardia, and postural hypotension. Abdominal examination is usually normal. 19. (C) During the early catabolic phase after Diagnosis is confirmed by technetium scan, injury, nutritional support is essential in the with the isotope concentrated in the gastric management of the multiply injured patient. mucosa of the diverticulum. Initial manage- Elemental tube feeding can be initiated via the ment should include IV fluid resuscitation and NG route, or via the nasojejunal route if there transfusion as needed, before laparotomy and is delayed gastric emptying. Enteral nutrition diverticulectomy with resection of the adja- will aid in maintaining the integrity of the gas- cent ileum. Rectal polyps, hemorrhoids, and trointestinal mucosal barrier; thereby, reduc- anal fissures may be associated with rectal ing the risk of bacterial translocation and bleeding. The bleeding is usually small in sepsis. The enteral route is less expensive than amount and often temporally related to defe- parenteral nutrition and does not subject the cation, typically on the surface of the stool or patient to the complications associated with an after defecation. Colonoscopy and proctoscopy indwelling central venous catheter. (Townsend et are useful adjuncts to diagnosis. Bleeding asso- al., 2004, pp. 170–172, 628–629; Way and Doherty, 2006, ciated with intussusception is described as pp. 153–158) “currant jelly” and is secondary to mucosal ischemia of the lead point. These children are (c) ketabton.com: The Digital Library

232 6: Surgery

most commonly between 2 months and 2 years diarrhea. It most commonly involves the ter- of age, and often have a prodromal viral illness. minal ileum and right colon. Because eating They present with colicky abdominal pain and can exacerbate symptoms, oral intake may be dehydration. Management includes hydrostatic decreased, contributing to the associated reduction. (Brunicardi et al., 2005, pp. 1493–1495; weight loss. Transmural inflammation leads Greenfield et al., 2005, pp. 1917–1919; Townsend et al., to bowel wall thickening, and with adjacent 2004, pp. 1366–1368, 2112–2113) mesenteric inflammation, the patient may develop a palpable mass. It may be difficult to 24. (E) The guiding principle of management of differentiate Crohn’s disease from ulcerative closed-head injury is to maintain cerebral perfu- colitis on the basis of history and clinical exam- sion and oxygenation; thereby, preventing sec- ination. However, ulcerative colitis is a ondary brain insult. Cerebral perfusion pressure mucosal disease that is limited to the colon (CPP) is dependent on systemic blood pressure, and nearly always involves the rectum. circulating blood volume, and ICP (i.e., CPP = Diarrhea is usually bloody, and hemorrhage mean BP − ICP). Normal CPP requires an ade- may be significant enough to require transfu- quate circulating blood volume with mainte- sion therapy. Complicated appendicitis may nance of normovolemia. Hypercarbia should be present with a right lower quadrant mass and avoided because it leads to cerebral vasodilata- diarrhea if there is perforation with abscess tion and increased ICP. Early insertion of a ven- formation. The history is that of an acute ill- triculostomy is beneficial to permit controlled ness in a previously well patient. Irritable drainage of CSF as required to maintain a normal bowel syndrome is associated with intermit- ICP. Fluid restriction and hyperventilation tent crampy abdominal pain, and diarrhea should be avoided in the early stages of man- alternating with constipation. There is no agement of a closed-head injury. Autoregulation inflammatory process, and weight loss is not a of cerebral blood flow is disrupted in the early clinical feature. Evaluation of the small bowel phases after head injury. Aggressive hyperventi- is best accomplished with contrast radiogra- lation with resultant cerebral vasoconstriction phy, such as a small-bowel follow-through may precariously compromise the perfusion to study or enteroclysis. Radiographic abnor- the injured brain and to the surrounding nonin- malities of small-bowel Crohn’s disease are jured brain. In patients with deteriorating neuro- often distinctive and can demonstrate compli- logic status and/or evidence of increasing ICP cations such as strictures and fistulae. CT scan- that is not well controlled with a ventriculostomy, ning does not assist in confirming the osmotic diuretics and moderate hyperventilation diagnosis, but is helpful in detecting such com- may be useful adjuncts to therapy. The use of plications as abscess. Ultrasonography has lim- steroids in the management of closed-head injury ited value. Sigmoidoscopy may not be useful, is not indicated. Craniotomy is indicated for because Crohn’s disease commonly spares the increased ICP attributed to a mass with a midline rectum and may be worse on the right side of shift. (Way and Doherty, 2006, pp. 863–866) the colon. Colonoscopy may be helpful when the colon is involved and when intubation of the 25. (C) ileocecal valve can be achieved; however, the disease may be limited to the small bowel 26. (E) resulting in a nondiagnostic examination. The principle of initial management of Crohn’s dis- 27. (D) ease is relief of symptoms, nutritional therapy, and suppression of the inflammatory process. Explanations 25 through 27 Nutritional supplementation may require TPN in conjunction with bowel rest. Acute exacer- Crohn’s disease is a chronic inflammatory dis- bations of the disease are initially treated with ease of the gastrointestinal tract that presents systemic steroids. The use of antispasmodics with intermittent crampy abdominal pain and may be effective in the treatment of irritable (c) ketabton.com: The Digital Library

Answers: 24–31 233

bowel syndrome. In Crohn’s disease, however, with bone pain that is not well palliated with antispasmodics may lead to an ileus or toxic hormonal manipulation, radiation therapy may bowel dilatation. Surgery in Crohn’s disease is be useful. (Brunicardi et al., 2005, pp. 1535–1538; indicated for the management of complica- Greenfield et al., 2005, pp. 2088–2091) tions, including fistula or abscess formation, stricture with obstruction, and perforation. 30. (E) Malignant hyperthermia may occur after (Townsend et al., 2004, pp. 1342–1349; Way and administration of certain inhalation agents for Doherty, 2006, pp. 668–672) induction of general anesthesia or with suc- cinylcholine for muscle relaxation. This is a 28. (E) Testicular torsion presents with acute onset of result of a genetic defect in calcium release from scrotal pain, reflex vomiting, referred abdominal the sarcoplasm of skeletal muscle. It often pain, and an elevated tender testis. If there is a occurs within 30 minutes of induction, and in high index of suspicion based on history and addition to fever, tachycardia, and muscle clinical examination, the patient should undergo rigidity, there is a metabolic acidosis and hyper- an urgent surgical exploration. Delay in defini- kalemia. The treatment is administration of tive therapy increases the risk of testicular loss dantrolene to block calcium release from the secondary to ischemia. Isotope scan may demon- sarcoplasm and insulin/bicarbonate/dextrose strate absence of testicular blood flow in torsion, infusion to treat the hyperkalemia. Diagnosis is and increased flow in orchitis or epididymitis. confirmed by muscle biopsy. Pneumonia is an Although useful in the differential diagnosis, infective, inflammatory process; is not associ- these nuclear medicine studies may not be read- ated with muscle rigidity; and is not likely to ily available, and definitive therapy should not have a rapid progressive onset after induction be delayed awaiting imaging. Orchitis and epi- of anesthesia in a previously healthy patient. didymitis present with a more insidious clinical Atelectasis is unlikely under general anesthe- course associated with the progression of the sia, particularly in patients receiving positive inflammatory process. There may be a concomi- pressure ventilation. Although pyelonephritis tant urinary tract infection, and therapy includes may be associated with fever, it is not associ- analgesics and antibiotics. (Brunicardi et al., 2005, ated with muscle rigidity or metabolic acidosis pp. 1555–1556; Greenfield et al., 2005, p. 1954; Townsend and would be unlikely to become symptomatic et al., 2004, pp. 2297–2298) after induction in an otherwise healthy patient. Myocardial infarction may be associated with 29. (D) This elderly patient has metastatic adeno- tachyarrhythmias but would not account for carcinoma of the prostate, and management is, the muscle rigidity, fever, or metabolic acidosis therefore, aimed at tumor control for palliation and, in the absence of risk factors, would be of symptoms. This is achieved with hormonal very unlikely in this healthy patient. (Brunicardi manipulation, either by orchiectomy or exoge- et al., 2005, p. 357; Townsend et al., 2004, pp. 233, nous estrogen therapy. Radical prostatectomy 304–305) is indicated only for patients in whom the malignancy is confined to the prostate gland. 31. (E) The anterior triangle of the neck is divided Transurethral prostatectomy is used to treat into three zones: zone I at the base of the neck benign prostatic hypertrophy, and is not con- and thoracic inlet, zone II in the midbody of the sidered adequate surgical therapy for prostatic neck, and zone III above the angle of the malignancy. Cytotoxic chemotherapy may be mandible. Zone II, the most common area useful as an adjunct to radical surgical excision injured with penetrating trauma, encompasses of localized disease. Chemotherapy is not indi- the carotid artery, jugular vein, larynx, trachea, cated for the treatment of metastatic disease. and esophagus. Patients with penetrating Radiation therapy has been used for the man- injuries to the neck that violate the platysma agement of localized disease, and there is some should be admitted to the hospital for further evidence that it affords equivalent survival evaluation. This patient has a penetrating when compared to surgical excision. In patients injury through the platysma, in zone II of the (c) ketabton.com: The Digital Library

234 6: Surgery

anterior triangle. He has signs of significant 33. (A) injury (i.e., external bleeding at the scene, odynophagia, and a neck hematoma on exam- 34. (B) ination). This patient should undergo surgical exploration, without prior diagnostic studies. Explanations 33 and 34 Observation in the ICU, with or without intu- bation, is not appropriate in a patient with DI is a disorder due to impaired renal conserva- obvious clinical signs of injury. Furthermore, tion of water. DI presents with polyuria and extensive preoperative imaging studies are not dilute urine in the presence of an elevated serum necessary for zone II injuries because surgical osmolality. This is either secondary to impaired exposure of vital structures in this area of the production of antidiuretic hormone (ADH) from neck is easily achieved. All patients with clini- the posterior pituitary (central DI), or refractori- cal signs of injury should undergo surgical ness of the distal renal tubules to ADH (nephro- exploration. However, there is controversy genic DI). Central DI may complicate closed-head with respect to the management of patients injury, and is considered a poor prognostic sign. without clinical signs of injury. There are two These patients will respond to exogenous IV approaches: (a) mandatory surgical explo- vasopressin, with resultant increase in urine ration; or (b) selective observation with or with- osmolality and decrease in urine volume. Nephrogenic DI may be congenital, familial, or out imaging studies. (Townsend et al., 2004, pp. 498–502; Way and Doherty, 2006, pp. 219–221) acquired. Acquired nephrogenic DI may occur in the setting of repeated renal tubular insults 32. (C) This patient has a distended abdomen, with such as sepsis, IV contrast, and nephrotoxic drug decreased bowel sounds, in the presence of therapy. With administration of vasopressin, shock that is unresponsive to aggressive fluid these patients will have no change in urine osmo- resuscitation. Intra-abdominal hemorrhage lality or urine volume because the renal tubules from solid visceral injury (hepatic, splenic, or are unresponsive. DI must be differentiated from renal) is the most likely etiology. The patient other causes of polyuria. Water intoxication should undergo an urgent exploratory laparo- results from ingestion of a large volume of fluid, tomy and damage control (packing) for con- with resultant dilutional hyponatremia. If the trol of the bleeding, in conjunction with patient has a normal diluting capacity, there will ongoing resuscitation with infusion of IV fluids be polyuria, with a proportionally low serum and blood products. Although hypotension can and urine osmolality. Prolonged fluid restriction result from a cervical cord injury, it is unlikely will result in appropriate rise in urine osmolality. in this case, in the presence of a documented Osmotic diuresis may occur from solute over- normal neurologic examination. A Swan-Ganz load when the renal tubules are unable to reab- catheter is not indicated in the initial evaluation sorb adequate quantities of filtered solutes. This and management of a patient presenting in is associated with administration of mannitol or, hypovolemic shock from blunt trauma. in the presence of glycosuria, from diabetes mel- Abdominal CT scan is indicated only for eval- litus. (Fauci et al., 2001, pp. 2004–2009; Townsend et al., uation of blunt abdominal trauma in patients 2004, pp. 81–84) who are hemodynamically stable. FAST and diagnostic peritoneal lavage may be indicated 35. (C) in the evaluation of patients with hypotension in which the source of bleeding is unclear. In 36. (F) this patient, however, the presence of a dis- tended abdomen suggests hemoperitoneum, 37. (E) and therefore, FAST and lavage are not neces- 38. (D) sary. (Townsend et al., 2004, pp. 513–514, 528; Way and Doherty, 2006, pp. 227–231) 39. (G) (c) ketabton.com: The Digital Library

Answers: 32–43 235

Explanations 35 through 39 serum lipase and amylase. Pain from a perfo- rated ulcer, however, is sudden in onset and Acute appendicitis initially presents with peri- may be associated with shoulder-tip pain from umbilical pain secondary to obstruction of the diaphragmatic irritation. About 75% of patients appendiceal lumen. This is mediated through with perforated duodenal ulcers have pneu- visceral pain fibers, and because the appendix moperitoneum on chest and abdominal radi- is from the embryologic midgut, the pain is ographs. Acute cholecystitis will usually referred to the umbilicus. With obstruction of a commence after a large meal and initially pres- hollow viscus, there may be associated nausea. ents as colicky epigastric pain, progressing to As the inflammatory process progresses to pain localized in the right upper abdomen involve the visceral and parietal peritoneal sur- when transmural inflammation of the gall- faces, the pain becomes localized directly over bladder wall produces peritoneal irritation. the appendix in the right lower quadrant. Fever Acute mesenteric occlusion presents with sudden and leukocytosis are nonspecific signs of an onset of severe but poorly localized periumbilical inflammatory process. Gastroenteritis may be abdominal pain, associated with acidosis. There associated with nausea, anorexia, and low- may be elevation of serum amylase and lipase. A grade fever. Periumbilical pain is colicky and ruptured abdominal aortic aneurysm will present secondary to increased peristalsis. Localized with sudden onset of midabdominal pain, back pain and signs of peritoneal irritation are pain, and hemodynamic instability. (Townsend uncommon. A ruptured right ovarian cyst may et al., 2004, pp. 1385–1387, 1651–1652, 1418–1420, 1284, mimic appendicitis. Patients may exhibit right 1611–1612) lower abdominal peritoneal irritation. However, the onset of pain is usually sudden, and the 40. (D) A thyroglossal duct cyst represents the rem- pain is initially felt in the right lower quad- nants of the thyroglossal duct tract left over rant. These patients do not have anorexia or from descent of the thyroid gland from the other gastrointestinal symptoms. The clinical foramen cecum. It is located in the midline and picture of regional enteritis (Crohn’s disease) is moves superiorly as the tongue protrudes one of a chronic illness, often associated with because the tract communicates with the fora- weight loss, intermittent cramps, and diarrhea. men cecum. (Brunicardi et al., 2005, pp. 1474–1476; Fever, tenderness, and a palpable right lower Greenfield et al., 2005, pp. 1842–1845; Townsend et al., quadrant inflammatory mass may result from 2004, p. 2103) complications of ileal involvement. Sigmoid diverticulitis is more common in older patients, 41. (E) Lipomas present as soft, subcutaneous often with a prodromal history of irregular masses that arise in all areas of the body. They bowel habits. There may be left lower quadrant are treated by simple excision. (Brunicardi et al., pain and tenderness, with a palpable left-sided 2005, p. 439) inflammatory mass. A cecal volvulus presents with sudden onset of colicky abdominal pain 42. (C) Acute suppurative lymphadenitis is related and signs and symptoms of a bowel obstruc- to bacterial pathogens and most often accom- tion, including bilious emesis and abdominal panies an infectious illness, such as an upper distention. Alcohol-related acute pancreatitis respiratory tract infection. The nodes enlarge presents with pain referred to the epigastrium, rapidly, are tender, and demonstrate overlying with radiation to the back mediated through erythema of the skin. (Brunicardi et al., 2005, pp. the celiac ganglia. The patient may develop 1474–1476; Greenfield et al., 2005, p. 1845; Townsend et abdominal distention secondary to the associ- al., 2004, p. 2104) ated paralytic ileus. Hyperamylasemia and an elevated serum lipase, in this clinical setting, 43. (J) The location of this mass and its cystic are suggestive of pancreatitis. Perforated peptic nature are typical for a second branchial cleft ulcer and acute cholecystitis may also present remnant. Surgical excision is recommended, with epigastric pain, and elevations of both including the associated tract, which traverses (c) ketabton.com: The Digital Library

236 6: Surgery

between the carotid bifurcation and ends at the and sepsis. The sepsis may manifest as fever, tonsillar fossa. (Brunicardi et al., 2005, pp. 1474–1476; hypotension, and altered mental status. (Townsend Greenfield et al., 2005, pp. 1840–1842; Townsend et al., et al., 2004, pp. 1615–1617) 2004, pp. 2102–2103) 50. (G) Liver dysfunction resulting in jaundice may 44. (G) Most tumors of the salivary glands present be a manifestation of advanced metastatic dis- in the parotid, the most common of which is the ease to the liver. In a patient with a known malig- pleomorphic adenoma. These occur most fre- nancy at high risk for metastases (i.e., deeply quently in the fifth decade and present as a soli- invasive and ulcerative melanoma), metastatic tary painless mass in the superficial lobe of the disease must be considered in the evaluation of parotid gland. Surgical treatment is complete the jaundiced patient. Hepatomegaly found excision with negative margins. (Brunicardi et al., on abdominal examination is supportive of 2005, pp. 534–540; Greenfield et al., 2005, pp. 651–652; advanced metastatic disease. (Townsend et al., Townsend et al., 2004, pp. 852–854) 2004, pp. 1602–1604)

45. (A) The location of this neck mass and its move- 51. (I) Pancreatitis may be the cause of jaundice ment with swallowing is strongly suggestive of by different mechanisms, resulting in com- a thyroid mass. The most common type of thy- pression of the common bile duct (CBD). Acute roid cancer is papillary carcinoma, which has an pancreatitis may cause edema of the head with excellent prognosis under the age of 40. Needle resultant compression of the CBD; pancreatitis biopsy should be performed as a diagnostic test, may lead to a pseudocyst in the head with com- followed by operation. (Brunicardi et al., 2005, pp. pression of the duct; and chronic pancreatitis 1413–1421; Greenfield et al., 2005, pp. 1298–1301; may lead to dense scarring around the duct Townsend et al., 2004, pp. 961–965) with a resultant stricture. The clinical scenario of an alcohol abuser with acute abdominal 46. (H) Laryngeal carcinoma is the most common pain; nausea; vomiting; jaundice; and a tender, malignancy of the upper aerodigestive tract in palpable epigastric mass is most consistent the United States. Presenting symptoms include with acute pancreatitis with CBD obstruction hoarseness of the voice and, for supraglottic attributable to a pseudocyst. (Way and Doherty, lesions, early metastatic disease to the neck. 2006, pp. 602–607) Risk factors include exposure to tobacco and alcohol. (Brunicardi et al., 2005, pp. 526–530; Greenfield 52. (C) A new mass in a woman of this age group et al., 2005, pp. 652–655; Townsend et al., 2004, pp. must be fully evaluated to rule out carcinoma. 844–846) Though biopsy by fine needle, core needle, or open biopsy will be required to determine the 47. (H) Chronic liver disease, such as cirrhosis, may etiology of the mass, they should not be per- be a cause of jaundice. Clinical features such as formed until all necessary breast imaging stud- spider angiomas, ascites, and varices suggest ies have been performed. Imaging should cirrhosis. (Townsend et al., 2004, pp. 1602–1604) include diagnostic mammography and possi- bly sonography to determine whether the mass 48. (F) Pancreatic carcinoma occurs in the head of the is solid or cystic. An appropriate type of biopsy gland in 75% of cases. About 75% of the patients can be chosen (i.e., FNA for cystic lesions or core with carcinoma in the head of the pancreas pres- biopsy for solid lesions). Imaging should not be ent with obstructive jaundice, weight loss, and, delayed, although the patient should discon- in the presence of advanced disease, abdominal tinue HRT until the etiology of the mass is deter- pain. (Townsend et al., 2004, pp. 1602–1604, 1668–1669) mined. (Greenfield et al., 2005, pp. 1261–1265)

49. (C) Common duct stones (choledocholithiasis) 53. (E) A number of breast cancer risk factors have may be the cause of acute bile duct obstruction been identified. These include family history, without warning, resulting in jaundice, pain, benign breast disease with atypical epithelial (c) ketabton.com: The Digital Library

Answers: 44–58 237

hyperplasia, alcohol consumption, and HRT. options for known mutation carriers, or other The most common and clinically useful risk high-risk women, include intensive surveil- factors are age and female gender. In most lance with monthly self-breast examinations, patients (70%) with breast cancer, however, annual or semiannual clinical breast examina- no other major risk factor can be identified. tions, annual mammography starting 10 years (Brunicardi et al., 2005, pp. 466–472; Greenfield et al., prior to the earliest age of onset of breast cancer 2005, pp. 1267–1268; Townsend et al., 2004, pp. in a family member, chemoprevention with 881–885) tamoxifen, and prophylactic mastectomy. The data supporting these options in BRCA-muta- 54. (D) LCIS is considered a marker for suscepti- tion carriers, however, are limited to small bility of the breasts for malignant change. series. Evidence for imaging of these high-risk Lobular and ductal elements of both breasts women using MRI as a screening modality is are at risk. The risk for development of cancer promising. (Brunicardi et al., 2005, pp. 466–470; is approximately 1% per year, equally divided Donegan and Spratt, 2002, pp. 253, 260–262; Greenfield between the two breasts. Therefore, re-exci- et al., 2005, p. 1267; Townsend et al., 2004, pp. 884–887) sion to gain negative margins is not indicated, nor is ipsilateral mastectomy. There is no 57. (B) known added significance to bilateral LCIS; therefore, contralateral breast biopsy is not 58. (D) useful for determining appropriate manage- ment. Both breasts should be treated the same. Explanations 57 and 58 For patients without any other identifiable risk factors, close observation with frequent Prospective randomized trials have addressed physician examinations, monthly breast self- the issue of locoregional control of breast cancer. examination, and yearly mammography may Conservative management consisting of lumpec- be the best option. However, some patients tomy (with negative margins), irradiation, and may prefer bilateral total mastectomies as a axillary node dissection is equally effective as personal preference or for situations in which modified radical mastectomy in patients with they cannot adhere to close follow-up recom- clinical stage I or II breast carcinoma. Although mendations or whose risk for cancer is deemed radical mastectomy yields equivalent survival to modified radical mastectomy, it is disfiguring too high. (Brunicardi et al., 2005, pp. 472–473; Donegan and Spratt, 2002, pp. 350–351; Greenfield et al., and disabling and is rarely employed for 2005, p. 1269) treatment of primary operable breast cancer. Lumpectomy without irradiation leads to unac- 55. (D) ceptably high local recurrence rates. Irradiation without surgery is not as effective as surgery in 56. (E) the presence of gross disease. Quadrantectomy removes excessive tissue and leads to an inferior Explanations 55 and 56 cosmetic result as compared to lumpectomy. If quadrantectomy is thought to be required to Deleterious mutations in the BRCA1 and gain negative margins, mastectomy with recon- BRCA2 genes are associated with markedly struction should be entertained. The use of SLN elevated lifetime risks of breast cancer. The risk biopsy to stage the axilla in breast cancer is gain- initially quoted was determined from families ing wider acceptance as experience with the pro- with high penetrance or expression of the dis- cedure grows. Trials are ongoing that will ease, in the 80–90% range. A subsequent study, answer numerous questions regarding the accu- based on a larger population of gene mutation racy and efficacy of SLN biopsy for breast carriers, demonstrated a lower risk of 56%. cancer. There are contraindications to SLN Therefore, the risk generally quoted for mutation biopsy that include palpable adenopathy that carriers is in the range of 50–80%. Management is suspicious for metastatic disease, locally (c) ketabton.com: The Digital Library

238 6: Surgery

advanced disease or tumor size >5 cm, multi- possessing ill-defined borders. Other features centric disease, pregnant or lactating patient, and suggestive of malignancy include clustered use of preoperative chemotherapy. Therefore, in microcalcifications, asymmetric density, archi- this patient with palpable suspicious adenopa- tectural distortion, and skin or nipple retrac- thy, SLN biopsy would be contraindicated. tion. DCIS usually presents as calcifications Although age may influence success of SLN without a definite mass. Cystosarcoma phyl- biopsy, increasing age does not preclude the pro- lodes presents much like a fibroadenoma as a cedure. (Brunicardi et al., 2005, pp. 481–491; Greenfield well-defined mass with smooth margins. et al., 2005, pp. 1275–1280) Papillomas are small and not usually palpable or noted by mammography. Fat necrosis can 59. (C) mimic the mammographic features of malig- nancy and is often associated with trauma. 60. (C) However, only when an oil cyst is noted, can an unequivocal diagnosis of fat necrosis be made. Explanations 59 and 60 The next step is needle biopsy, by FNA, or core The mammographic appearance is most consis- biopsy so a histologic diagnosis can be made tent with DCIS with or without an invasive and options for management (i.e., modified component. The calcifications of DCIS are clus- radical mastectomy or lumpectomy, axillary tered, pleomorphic, irregular, and linear, and dissection, and irradiation) discussed. Needle may be associated with a mass representing an biopsy is less invasive and less expensive than invasive carcinoma. LCIS does not usually pres- incisional biopsy and, therefore, is preferred. ent with calcifications. Cystosarcoma phyllodes Excisional biopsy (not listed) is acceptable, but presents as a mass, often indistinguishable from often requires a two-step procedure, the first to a fibroadenoma. An involuting fibroadenoma establish the diagnosis and the second for often presents as a mass with large, coarse definitive treatment. A needle biopsy, especially calcifications, indicating its benignity. Milk of a core biopsy that renders a histologic diagnosis, calcium will often layer differently in the CC can often avoid the necessity of a second surgi- and mediolateral projections, suggesting fluid cal procedure. (Brunicardi et al., 2005, pp. 481–490; with calcium. The next step in management is to Donegan and Spratt, 2002, pp. 289–293; Greenfield et al., obtain a biopsy, preferably a core needle biopsy, 2005, pp. 1263–1265) for histologic confirmation and to evaluate for invasive disease. Observation is not recom- 63. (C) mended because these calcifications appear malignant. All other choices (i.e., lumpectomy or 64. (D) total mastectomy) are not indicated until a diag- nosis by biopsy is obtained. (Brunicardi et al., 2005, Explanations 63 and 64 pp. 477, 481, 487–488; Donegan and Spratt, 2002, pp. The symptoms of gastric cancer are nonspe- 293–298; Greenfield et al., 2005, pp. 1271–1273) cific and may mimic those of such benign con- ditions as benign gastric ulcer. Pain, nausea, 61. (B) anorexia, and weight loss are common non- specific symptoms. A UGI series shows a gas- 62. (B) tric ulcer that has characteristics of malignancy, including an intraluminal crater with nodular Explanations 61 and 62 margins. A gastric diverticulum would extend A new mass in an older woman must be as a protrusion beyond the gastric lumen. The evaluated for malignancy. In this case, the duodenum is not well visualized in the x-ray, mammogram has characteristics of malignancy, making the diagnosis of duodenal disease dif- which include a density or mass that is stellate ficult. A fistula would be suggested by contrast or spiculated, irregular in size or shape, and filling of the gallbladder and biliary tree. Given (c) ketabton.com: The Digital Library

Answers: 59–69 239

the x-ray findings suggestive of malignancy, the cancer, should be noted on the CT scan. There next step would be to obtain a tissue diagnosis is no history of antecedent antibiotic therapy to for confirmation by endoscopy. Once malig- suggest the diagnosis of pseudomembranous nancy is confirmed, a CT scan would be helpful colitis. Though WBCs were present in the uri- to evaluate for liver metastasis and extent of dis- nalysis, a diagnosis of pyelonephritis cannot ease. Operative intervention may be determined be made on this basis alone, because peri- at that time, usually a subtotal or total gastrec- colonic inflammation may be responsible for tomy. Vagotomy and pyloroplasty would not the WBCs. CT scanning is very accurate in be appropriate for gastric cancer. Medical ther- diagnosing diverticulitis, so there is no need

apy with H2 blockers may improve the patient’s for any additional test. Barium enema and symptoms but should not delay endoscopy and colonoscopy should not be performed in biopsy. (Way and Doherty, 2006, pp. 531–534) patients with suspected acute diverticulitis. The increased intraluminal pressure from either of 65. (D) The prevalence of adenomas without symp- these examinations may lead to free rupture of toms in patients over the age of 50 ranges a contained abscess or phlegmon, leading to between 20 and 40%. Screening studies suggest emergency surgery. However, either examina- that 30% of patients without symptoms over tion, or both, should be performed after com- the age of 50 who undergo colonoscopy for plete resolution of diverticulitis (e.g., in 6 weeks’ evaluation of positive fecal occult blood have a time) to evaluate for extent of disease, compli- polyp detected. Polyps greater than 2 cm have cations, and carcinoma. IVP and angiography a high potential for malignant degeneration. are not indicated for diverticulitis. The appro- The polyp should be completely removed, priate management in this patient with his first preferably by snare polypectomy. Most studies episode of diverticulitis is medical manage- indicate that polypectomy is adequate for ment with IV antibiotics for gram-negative polyps with carcinoma in the head if the margin and anaerobic bacteria. Colon resection, either of resection is 2 mm or greater, especially if the immediate or elective, should not be under- stalk is not invaded. Following adequate taken unless the patient’s condition deterio- polypectomy, observation would be indicated rates or recurs. Bowel preparation cannot be with postpolypectomy colonoscopic surveil- performed safely in patients with acute diver- lance. If cancer is present at or near the margin, ticulitis. Anticoagulation has no role in ther- then colon resection is indicated. Because the apy. Metronidazole or vancomycin therapy incidence of residual cancer and metastatic dis- would be appropriate for pseudomembranous ease is very low after successful polypectomy, colitis, but not for diverticulitis. (Townsend et al., scanning by CT or MRI is not indicated. The 2004, pp. 1418–1420) data on NSAID use are promising but insuffi- cient to support a clinical recommendation. 68. (D) (Townsend et al., 2004, pp. 1448–1451) 69. (B) 66. (D) Explanations 68 and 69 67. (A) The diagnosis of sigmoid volvulus is based on Explanations 66 and 67 the history, examination, and radiographs. Acute onset of abdominal pain, distention, and The gradual onset of left lower quadrant pain obstipation is suggestive of volvulus. Barium over a number of days with left lower quadrant enema is diagnostic of sigmoid volvulus showing abdominal tenderness and CT scan showing the characteristic tapering to a “bird’s beak” sigmoid colon inflammatory changes is most pointing to the site of obstruction. Cecal volvu- consistent with diverticulitis. A contained per- lus would show complete filling of the left foration, either due to the diverticulitis or colon colon. Stricture as a result of ischemic colitis (c) ketabton.com: The Digital Library

240 6: Surgery

would show a long, narrowed segment of colon. and diarrhea, and barium x-rays would show Diverticulitis would be suggested by a different thickened bowel wall, mucosal ulcerations, and clinical presentation including fever, sepsis, and cobblestone appearance. The treatment of colon pain localized to the left lower quadrant. cancer in this patient would be surgical resection Obstruction from colon cancer would show an and anastomosis. Colostomy may be appropri- irregular narrowing of the colon segment rather ate in selected patients with obstruction in an than a smooth tapering. In patients who have no unstable patient in whom resection is not feasi- signs of bowel wall ischemia (e.g., rebound ten- ble. Surgical bypass would be appropriate only derness, sepsis, and so forth), nonoperative for palliative therapy of unresectable disease. reduction should be attempted and would be Radiation therapy or chemotherapy without expected to be successful in 70–80% of patients. surgical resection and staging is not recom- The most widely used method of reduction is mended. (Townsend et al., 2004, pp. 1457–1460) proctoscopy and rectal tube placement under direct vision. Blind passage of a rectal tube may 72. (D) lead to perforation and is contraindicated. Operation is indicated if nonoperative reduc- 73. (C) tion is unsuccessful, with operative reduction preferred, followed by delayed resection and Explanations 72 and 73 primary anastomosis rather than sigmoid resec- tion. Operative reduction by detorsion alone is The stage of colon cancer is based on the depth unacceptable because of the high recurrence rate of invasion, nodal involvement, and distant and is, therefore, combined with sigmoidopexy metastases. Stage 0 represents carcinoma in situ, or sigmoidostomy. Proximal colostomy alone is stage I invasion of the submucosa or muscularis contraindicated, because strangulation of the propria without node involvement, stage II sigmoid or recurrent volvulus is not prevented. invasion through the muscularis propria or (Townsend et al., 2004, pp. 1422–1424) directly invading other organs without nodal involvement, stage III any depth of invasion 70. (B) with nodal metastasis, and stage IV any depth of invasion or nodal status with distant metas- 71. (D) tases. Adjuvant therapy has been shown to be beneficial in patients with stage III disease in Explanations 70 and 71 randomized studies. The recommended regi- men is 5-fluorouracil-based chemotherapy with The clinical features of colon cancer are variable leucovorin, rather than 5-fluorouracil alone. depending on the location. On the right, Adriamycin therapy, either alone or with other fatigue, as a manifestation of anemia, may be agents, has not been shown to be beneficial in the predominant symptom; whereas, obstruc- patients with colon cancer. No adjuvant ther- tive complaints may predominate for lesions on apy would be indicated for patients with stage the left. In the figure, an annular or “apple 0, I, or II disease, although some patients with core” lesion is noted, consistent with carci- stage II disease manifesting poor prognostic noma. Radiography of diverticular disease indicators may be candidates for adjuvant would show numerous protrusions from the therapy. (Townsend et al., 2004, pp. 1460–1462) lumen, usually localized to the sigmoid colon. Lymphoma may occur in the colon, but this 74. (C) site of disease is rare, and widespread disease can be documented in most cases. Ischemia 75. (B) usually occurs at the splenic flexure, and the resultant stricture would produce a longer Explanations 74 and 75 segment of narrowing than that usually seen with carcinoma. Patients with Crohn’s disease The history of decreasing caliber of the stool would manifest symptoms of abdominal pain with evidence of bleeding is highly suggestive (c) ketabton.com: The Digital Library

Answers: 70–80 241

of rectal carcinoma. The barium x-ray shows a margin of resection and determination of lymph near-obstructing lesion of the rectum with an node management would depend on the depth “apple core” appearance of cancer. Diverticulitis of invasion of the melanoma measured in mil- does not occur in the rectum due to the lack of limeters. Therefore, wide excision is not recom- diverticular disease at this site. Ischemia usu- mended until the depth of invasion of the lesion ally does not involve the rectum due to its more is determined by excisional biopsy with narrow abundant blood supply than the colon. The margins. Mohs’ surgery should be considered barium enema findings of sigmoid volvulus for nonmelanoma tumors but is not recom- would show a smooth, tapering, so-called mended for melanoma. (Way and Doherty, 2006, pp. “bird’s beak” at the rectosigmoid junction, 1343–1346) rather than an irregular lesion in the midrec- tum as shown in the figure. Crohn’s disease 78. (E) The diagnosis of an incarcerated hernia would be expected to show fistulas, either by must be considered in the differential diagno- examination or radiography. The next step in sis of a patient with abdominal symptoms and management is to confirm the diagnosis by a nonreducible inguinal bulge. Femoral hernia proctoscopy and biopsy. Colonoscopy may not presents as a bulge below the inguinal liga- be feasible given the degree of the stricture, ment medial to the femoral artery. Direct or and endoscopic dilation is not routine and indirect inguinal hernias would present above would not be recommended. Experience with the inguinal ligament. An aneurysm of the placement of stents is accruing, but endoscopic femoral artery should be pulsatile. If there is placement of stents is not yet widely available. concern for an aneurysm, a Doppler ultrasound Because this process does not represent a examination may be diagnostic. A lymph node volvulus, placement of a rectal tube is not with abscess may present as a tender, nonre- required. Administration of antibiotics would ducible mass, but should be accompanied by be considered in the perioperative period, but additional adenopathy and a source of the not before making a histologic diagnosis. infection. (Townsend et al., 2004, p. 1211) (Townsend et al., 2004, pp. 1462–1463) 79. (C) 76. (D) 80. (B) 77. (D) Explanations 79 and 80 Explanations 76 and 77 The finding of a new, irregular lesion in a patient A multicolored brown or black pigmented lesion with a long smoking history must be consid- with irregular borders should raise the concern ered a lung carcinoma and should be managed of melanoma. Squamous cell carcinoma usually accordingly. Nonsmall cell carcinoma is the most presents as an erythematous papular nodule. common lung neoplasm. Small cell carcinomas The most common type of basal carcinoma pres- usually grow rapidly and disseminate widely by ents as an ulcerative, well-circumscribed nodule, the time of diagnosis. Tuberculosis would present although occasionally it may be pigmented and with systemic symptoms and apical disease on confused with melanoma. Merkel cell carci- chest x-ray. Likewise, a lung abscess would be noma appears as red to purple papular nod- accompanied by systemic symptoms and may ules. Keratoacanthoma is a well-circumscribed show air-fluid levels in the abscess cavity. A keratotic lesion that may regress without treat- hamartoma presents as an extremely slowly ment. The most appropriate next step is to per- growing nodule that may contain popcorn cal- form an excisional biopsy with narrow margins cifications. The most appropriate test following to confirm the diagnosis and determine depth suspicious findings on a chest x-ray would be a of invasion. Shave biopsy would yield a patho- CT scan to evaluate further the nodule, evaluate logic diagnosis, but would not allow appropri- the lymph node status, and triage subsequent ate staging and is never recommended. The diagnostic tests. If enlarged mediastinal nodes (c) ketabton.com: The Digital Library

242 6: Surgery

are seen, then mediastinoscopy may be indi- fibrillation is strongly suggestive of a cardiac cated. Bronchoscopy may be helpful to assess for source of the emboli. The first step in manage- endobronchial lesions and to obtain tissue for ment is immediate heparinization to prevent diagnosis. Pulmonary function tests are neces- propagation of the clot and maintain patency sary once a decision is made to consider oper- of collaterals. The cornerstone of treatment is ation. Percutaneous needle biopsy may be thromboembolectomy. Thrombolytic therapy is required to obtain tissue once CT scanning is reserved for treatment of irretrievable clots in performed. (Brunicardi et al., 2005, pp. 556, 560–561; small vessels. Fasciotomy, alkalinization of the Greenfield et al., 2005, pp. 1369–1371; Townsend et al., urine, and mannitol diuresis are adjuncts to 2004, pp. 1775, 1783–1784) treatment, particularly if there is a delay in oper- ation, increasing the risk of a reperfusion injury. 81. (D) Anticoagulation has been shown to reduce the rate of recurrent embolism. (Brunicardi et al., 2005, 82. (B) pp. 759–762)

Explanations 81 and 82 86. (B)

The clinical manifestations of ruptured abdom- 87. (C) inal aneurysm are back, flank, or abdominal pain; hypotension; and a pulsatile abdominal Explanations 86 and 87 mass. The treatment should be immediate trans- fer to the operating room for laparotomy. The diagnosis of an aortic dissection is strongly Ultrasound can be done quickly in the ER to suggested by the history of an abrupt onset of confirm the diagnosis. If the patient is hemody- excruciating pain in the chest and back with vari- namically stable, and the diagnosis is in doubt, able radiation patterns, and a hypertensive, ill- a CT scan can be performed, but is generally appearing patient. A chest x-ray showing a not needed. Angiogram is not indicated unless widened mediastinum may be noted, but the endovascular repair is being considered, and radiograph may be normal. The differential diag- then it should be performed in the operating nosis of an acute myocardial infarction must be room. Following successful treatment, diarrhea entertained and an ECG performed. Though aor- may suggest ischemic colitis, and urgent sig- tography has historically been the definitive diag- moidoscopy is indicated. A delay in diagnosis of nostic procedure and may be required in some bowel ischemia caused by numerous other diag- patients, transesophageal echocardiography has nostic tests may increase the mortality rate. If become the preferred diagnostic modality. It can bowel ischemia is found, immediate colon resec- be performed in the emergency department, thus tion should be undertaken. (Brunicardi et al., 2005, obviating the need to move an extremely ill pp. 741–742, 738–739; Greenfield et al., 2005, pp. patient. CT scan may also be helpful in estab- 1714–1716, 1728–1729) lishing the diagnosis. Immediate drug therapy to control hypertension is mandatory, followed 83. (B) by definitive therapy, depending on the type of dissection. Involvement with the ascending aorta 84. (D) mandates immediate surgical repair. Dissections involving only the descending aorta can be man- 85. (A) aged medically, initially. Thrombolytic therapy and anticoagulation are not indicated and may Explanations 83 through 85 precipitate exsanguination. (Brunicardi et al., 2005, pp. 704–706; Greenfield et al., 2005, pp. 1691–1697) The diagnosis of arterial embolism is suggested when the patient presents with an acute onset of 88. (C) severe pain, pallor, pulselessness, paresthesia, and paralysis (five P’s). The presence of atrial 89. (E) (c) ketabton.com: The Digital Library

Answers: 81–96 243

Explanations 88 and 89 underlying neuromotor abnormality exists, which is responsible for increased pharyngeal Factors that increase the risk for carcinoma pressure. The most important aspect of treat- include exposure to low-dose irradiation, age ment is a cricopharyngeal muscle myotomy, under 20 or over 60, male gender (especially which can be combined with resection or diver- over age 40), and recent onset. An increased inci- ticulopexy. (Townsend et al., 2004, pp. 1106–1107) dence of thyroid carcinoma in patients with Hashimoto’s thyroiditis has not been substanti- 94. (C) Mallory—Weiss tear involves acute UGI ated. For diagnosis, FNA is highly accurate and hemorrhage that occurs after retching or vom- has become the preferred diagnostic modality. iting and accounts for 5–15% of patients with FNA is the most important diagnostic test for UGI bleeding. The patient is often an alco- selecting patients for operation, supplanting all holic who vomits after binge drinking. other diagnostic tests. (Brunicardi et al., 2005, pp. Hematemesis follows vomiting without blood. 1413–1415; Greenfield et al., 2005, pp. 1300–1301) After resuscitation, esophagoscopy is required to determine the source of bleeding and may 90. (D) be helpful in nonoperative management. Contrast studies are not helpful, and the use of 91. (B) barium would coat the stomach and preclude a diagnostic endoscopic examination. Most Explanations 90 and 91 patients with Mallory—Weiss tears stop Leiomyomas are the most common benign bleeding spontaneously. Angiography may be tumors of the esophagus. They are intramural, helpful in selected patients who continue to occur between 20 and 50 years of age, and may bleed and in whom the site of hemorrhage be symptomatic when over 5 cm. Symptoms cannot be determined endoscopically. (Greenfield may include dysphagia and retrosternal pres- et al., 2005, p. 1061) sure and pain. Esophagogram shows charac- teristic features of a smooth concave defect 95. (B) with sharp borders. Esophagoscopy is indi- cated to rule out carcinoma. These tumors are 96. (B) mobile, with intact overlying mucosa. Biopsy should not be performed so that subsequent Explanations 95 and 96 extramural resection can be performed without Progressive dysphagia in an older adult warrants complication. Excision is recommended for evaluation, especially with associated symptoms symptomatic leiomyomas or those greater than of weight loss, chest pain, or hematemesis. A 5 cm. (Townsend et al., 2004, pp. 1116–1117) barium esophagogram is the first study that should be obtained. The typical carcinoma 92. (B) demonstrates an irregular, rigid narrowing of the esophageal wall with distortion of the lumen. 93. (B) Achalasia demonstrates a narrow, tapering bird’s beak appearance of the distal esophagus. Explanations 92 and 93 Development of an esophageal stricture causes Pharyngoesophageal (Zenker’s) diverticulum is slowly progressive dysphagia, usually after a the most common esophageal diverticulum and long history of gastroesophageal reflux disease typically occurs in the 30–50 age group and, (GERD). Esophagoscopy and biopsy are manda- therefore, is believed to be acquired. Its symp- tory for evaluation of esophageal stenosis and toms include cervical dysphagia, regurgitation of yield a diagnosis of carcinoma in 95% of patients undigested food, and recurrent aspiration. It is with malignant strictures. CT scanning is the stan- categorized as a pulsion type, creating protrusion dard technique for staging, once the diagnosis of mucosa, resulting in a false diverticulum. An has been made. Bronchoscopy is helpful in (c) ketabton.com: The Digital Library

244 6: Surgery

patients with upper and middle third carcino- ultrasound may be helpful in staging gastric mas to exclude invasion of the trachea or bronchi cancer, but are not routinely used with benign before esophagectomy. (Townsend et al., 2004, pp. disease. The failure to respond to 12 weeks of 1119–1120) medical management is an indication for sur- gical therapy to avoid potential complications 97. (D) and to exclude malignancy, despite biopsies obtained by endoscopy that show benign dis- 98. (E) ease. (Greenfield et al., 2005, p. 733)

99. (C) 102. (D)

Explanations 97 through 99 103. (C)

Perforated duodenal ulcer usually presents as 104. (C) a sudden onset of acute abdominal pain. Examination usually reveals severe abdominal Explanations 102 through 104 tenderness with rigidity of the abdominal mus- culature (i.e., an acute abdomen). With a prior In a patient who has undergone surgical resec- history of abdominal pain relieved by antacids, tion for colon cancer, elevated CEA, and liver a chronic ulcer that has now perforated is function tests must be followed by an evalua- strongly suggested. Perforated colon cancer tion for metastatic disease, including the pos- occurs in an older age group, as well as gastric sibility of extrahepatic disease. The CT scan is ulcer. Following plain radiographs that show the most useful examination to evaluate both pneumoperitoneum, no additional diagnostic intra- and extrahepatic disease. Various CT tests are required and serve only to delay treat- scans have been advocated for liver tumors, ment. The treatment is laparotomy and either including dynamic and portography scans. patch closure of the perforation or definitive PET scans may detect occult extrahepatic dis- operation, the latter being preferred, depending ease and studies are underway to define the on operative findings. However, the patient role of this modality in metastatic colon must receive fluid resuscitation before laparo- cancer. MRI shows promise as a useful exam- tomy to avoid hypotension and its conse- ination and can be useful to characterize quences. Although nonoperative management lesions of uncertain significance. Radionuclide for contained perforations has been suggested liver scans have been supplanted by more by some authors, an acute abdomen is an indi- accurate scans. Surgical resection, if possible, cation for operative management. (Greenfield is the treatment of choice for metastatic col- et al., 2005, pp. 731–732) orectal cancer to the liver. Chemotherapy is reserved for patients who are not surgical can- 100. (B) didates or refuse surgical treatment. Radiation therapy is not usually used in these patients. 101. (D) Observation and repeat imaging delays the treatment for patients who may be resectable. Explanations 100 and 101 The expected 5-year survival has been shown in multiple studies to be greater than 20%, Gastric ulcers present with symptoms of usually in the range of 25 and 35%. (Townsend abdominal pain, aggravated by food, and asso- et al., 2004, pp. 1554–1557) ciated with nausea, vomiting, anorexia, and weight loss. The two principal means of diag- 105. (C) nosing a gastric ulcer are UGI radiographs and fiberoptic endoscopy, the latter being the 106. (D) most reliable method. CT scan and endoscopic (c) ketabton.com: The Digital Library

Answers: 97–110 245

Explanations 105 and 106 bowel distention and partial obstruction. There may be a palpable, right-sided, “sausage- The anatomic location of the mass suggests a shaped” mass, but in many patients, the parotid origin, and the lengthy history and abdominal examination is entirely normal. The absence of symptoms and signs of inflamma- classic “currant jelly” stool (see Figure 6-14) is a tion are consistent with a neoplasm of the late sign and is a result of ischemia and mucosal parotid. The most common salivary gland neo- sloughing of the lead point. After the child has plasm is a benign pleomorphic adenoma. received IV fluid resuscitation, the management Metastatic carcinoma from a head and neck is hydrostatic reduction, either by contrast primary tumor may first present as a neck enema or air enema. Intussusception may occur mass, usually along the anterior or posterior during the clinical course of viral gastroenteri- cervical lymph node chain, and often in a tis. Bloody stools are more commonly associ- patient with such risk factors as a history of ated with bacterial gastrointestinal infections, smoking. Infectious parotitis may occur in the with characteristically loose, mucousy stools, elderly or diabetic patient, usually presenting and blood mixed with fecal material. Diagnosis with a shorter history, with symptoms and is aided by obtaining stool cultures. A midgut signs of inflammation. Hodgkin’s disease can volvulus can be associated with passage of a present as a painless neck mass involving the “currant jelly” stool secondary to small-bowel anterior or supraclavicular lymph nodes. ischemia. However, these children usually pres- Reactive cervical lymphatic hyperplasia is asso- ent with bilious vomiting. Diagnosis may be ciated with an inflammatory or infectious focus confirmed with a UGI contrast study. Rectal in the head and neck. The optimal manage- bleeding from a Meckel’s diverticulum is typi- ment for a pleomorphic adenoma in the lateral cally painless, without other associated gas- lobe of the parotid is a superficial parotidec- trointestinal symptoms. Technetium scan is tomy. IV antibiotics are not indicated in the useful for diagnosis. Bleeding from a juvenile absence of an inflammatory or infectious rectal polyp is usually small in amount and process. Although an excisional biopsy may be often occurs during normal stool passage. The indicated for a mass arising from cervical children are clinically well, without other gas- lymph nodes, enucleation of a neoplastic trointestinal symptoms. These polyps may be parotid mass is insufficient and associated with seen on proctoscopy. (Brunicardi et al., 2005, pp. an increased incidence of local recurrence. 1493–1494; Greenfield et al., 2005, pp. 1915–1917; Observation and re-evaluation are inappropri- Townsend et al., 2004, pp. 2112–2113) ate in this patient. A chest x-ray would be indi- cated in the evaluation of a patient with 109. (E) This patient presents with a rapidly progres- suspected Hodgkin’s disease. (Brunicardi et al., sive, necrotizing soft-tissue infection. The skin 2005, pp. 536–540; Greenfield et al., 2005, pp. 651–653; edema, purple hue, bullae, water drainage, and Townsend et al., 2004, pp. 852–854) crepitus are classic findings in clostridial infec- tions. Although culture of the wound drainage 107. (B) may be confirmatory, the diagnosis should be suspected on a clinical basis. Antibiotics alone are 108. (C) insufficient therapy. The mainstay of therapy is radical surgical debridement of devitalized tis- Explanations 107 and 108 sues, in conjunction with high-dose IV antibi- Intussusception most commonly occurs between otics. Hyperbaric oxygen therapy may facilitate 2 months and 2 years of age, often associated recovery. (Townsend et al., 2004, pp. 264–266; Way and with a prodromal viral illness. Children will pres- Doherty, 2006, pp. 108–110) ent with intermittent episodes of abdominal colic, secondary to peristaltic waves of the ileum 110. (C) All traumatic soft-tissue injuries should be against the partially obstructing ileocolic lesion. managed with aggressive local wound care. Reflex nonbilious vomiting is secondary to Because this injury is greater than 6hours old, (c) ketabton.com: The Digital Library

246 6: Surgery

contaminated, and greater than 1 cm in depth, should be aimed at improving tissue perfusion it is a tetanus-prone wound. Therefore, in addi- with aggressive IV fluid resuscitation. Metabolic tion, this patient should receive tetanus toxoid, alkalosis is associated with serum bicarbonate because it has been more than 5 years since his greater than 26 meq/L. Respiratory acidosis is last immunization. He had full immunization related to primary carbon dioxide retention from as a child and, therefore, does not require addi- decreased alveolar ventilation. Administration tional passive immunization with tetanus of sodium bicarbonate is indicated only in immune globulin. Prophylactic antibiotics are severe acidosis (pH <7.2), and in patients with controversial in the absence of an established evidence of myocardial instability or arrhyth- wound infection. (Townsend et al., 2004, p. 261, 264; mias. Hydrochloric acid is indicated only in Way and Doherty, 2006, pp. 112–113) life-threatening metabolic alkalosis that is not chloride responsive. Respiratory acidosis with 111. (E) This child has a displaced supracondylar alveolar hypoventilation may be corrected with fracture associated with vascular compromise assisted ventilation. (Townsend et al., 2004, pp. of the forearm from associated brachial artery 77–81; Way and Doherty, 2006, pp. 135–138) compression, distortion, or vessel injury. Decreased perfusion below the fracture in con- 114. (D) junction with pain on passive wrist flexion are signs of a developing forearm compartment 115. (J) syndrome. Management should include oper- ative exploration of the brachial artery, open 116. (C) reduction and pinning of the fracture, and fore- arm compartment fasciotomy to limit progres- Explanations 114 through 116 sion of muscular ischemia. Immobilization of the elbow at 90° is suitable only for undisplaced Pyloric stenosis presents with nonbilious vom- fractures. For displaced fractures without neu- iting and gastric distention. An annular pancreas rovascular compromise, closed reduction and does not result in obstruction, except when it pinning may be adequate, but if adequate is associated with an underlying duodenal reduction cannot be achieved, open reduction abnormality. Duodenal atresia is associated with Down syndrome. It results in early onset may be required. (Brunicardi et al., 2005, p. 1696; Greenfield et al., 2005, p. 2030; Townsend et al., 2004, pp. of bilious vomiting from complete duodenal 552–557) obstruction distal to the ampulla. There is a “double bubble” sign on plain abdominal radi- 112. (B) ographs from air in the stomach and proximal duodenum. Midgut volvulus is a life-threaten- 113. (B) ing complication of malrotation. It presents with acute onset of bilious vomiting, usually in Explanations 112 and 113 infants in the first year of life. There is a paucity of gas on plain radiographs, with evidence of This patient is acidotic, with a low serum bicar- duodenal obstruction. UGI contrast study will bonate (bicarbonate depletion defined as a confirm the abnormal position of the duodenal- serum bicarbonate less than 22 meq/L). She jejunal junction and may demonstrate a has hypovolemic shock from trauma and acute corkscrew of the duodenum from volvulus. blood loss, resulting in decreased tissue perfu- Intussusception is uncommon in newborns. sion and lactic acidosis. The resultant elevated Bilious vomiting is unusual at the outset, but anion gap (139 − [103 + 14] = 22 meq/L with a may develop if the intussusception has been normal anion gap of 8–16 meq/L) is from the present for a significant time. Imperforate anus increased lactic acid. The tachypnea may be can be excluded by clinical examination. If caused by the respiratory compensation with unrecognized, the infant will develop a clinical

decreased PCO2. Correction of the acidosis picture of a distal bowel obstruction, with (c) ketabton.com: The Digital Library

Answers: 111–121 247

dilated small and large bowel. Meckel’s diver- radiosensitive, and radiotherapy is the primary ticulum can present with obstruction second- adjuvant therapy for local control. Chemotherapy ary to volvulus around a Meckel’s band, with is indicated in patients with nodal spread or a distal small-bowel obstruction. Contrast disseminated disease. Radiotherapy is restricted enema will demonstrate a normal-caliber to therapy for localized disease only. (Brunicardi decompressed colon, with proximal dilated et al., 2005, p. 1538; Greenfield et al., 2005, pp. small bowel. Meconium ileus is associated 2091–2093; Townsend et al., 2004, pp. 2311–2316) with cystic fibrosis. Obstruction occurs from inspissated meconium in the terminal ileum. 119. (C) Plain radiographs may demonstrate a “soap bubble” pattern in the right lower quadrant, 120. (D) with a decompressed colon on contrast enema. Hirsch-sprung’s disease presents with a distal 121. (E) bowel obstruction and delayed passage of meconium. A contrast enema may demon- Explanations 119 through 121 strate a transition zone, with a narrow distal aganglionic segment, and proximal colonic This patient has an inguinoscrotal mass from dilatation. Jejunal atresia is a result of an an indirect inguinal hernia. His initial presen- intrauterine vascular accident. Infants present tation is one of a reducible inguinal hernia. with bile-stained vomiting and abdominal dis- Repair is indicated because of the risk of incar- tention early after birth. The colon is unused, ceration. He should be referred for early elec- and characteristically, on contrast enema, it is tive surgery. The second presentation several abnormally small in caliber (microcolon). weeks later is at the time of incarceration of (Brunicardi et al., 2005, pp. 1486–1499; Greenfield et al., the hernia. This has resulted in pain, irritability, 2005, pp. 1884–1914; Townsend et al., 2004, pp. and reflex vomiting. Prolonged incarceration 2107–2116) increases the risk of bowel ischemia. The appropriate management is sedation with 117. (C) manual reduction, admission with observation in hospital, and surgical repair within 24–48 118. (C) hours. Delaying repair after an initial episode of incarceration increases the risk of further Explanations 117 and 118 episodes of incarceration, with potential bowel or testicular compromise. Failure to reduce an Cryptorchidism increases the risk of develop- incarcerated hernia successfully mandates ing a testicular malignancy. This patient has a urgent surgical intervention. Testicular torsion is solid testicular mass which should be pre- uncommon in this age group and presents with sumed to be secondary to a testicular malignancy. a tender, high-riding testicle. When suspected, Optimal surgical management is inguinal urgent surgical exploration is indicated. exploration, control of the spermatic cord, Inguinal adenitis may be the result of an biopsy of the mass, and radical orchiectomy inflammatory focus in the diaper area, with with high ligation of the cord, if tumor is con- resultant adenopathy, and secondary infection firmed. Transscrotal aspiration, exploration, or of the inguinal nodes with a gram-positive needle biopsy is contraindicated because of risk organism. The infant is usually febrile, with a of tumor spillage, and risk of altering the lym- tender inguinal mass. Therapy includes sys- phatic drainage of the scrotum. Laparotomy temic antibiotics. An undescended testicle may and retroperitoneal node dissection is not indi- present as an inguinal mass, with an empty cated until after confirmation of the diagnosis hemiscrotum. It is usually asymptomatic. and excision of the primary tumor. This patient Management is elective orchiopexy at approx- has seminoma with disease spread to ipsilateral imately 1 year of age. A noncommunicating regional lymph nodes. The tumor is very hydrocele presents as an asymptomatic, fluctuant (c) ketabton.com: The Digital Library

248 6: Surgery

scrotal mass that transilluminates. Surgical inter- 124. (B) This child presents with progressive vention is not required, because most will resolve cholestatic jaundice, as indicated by the elevated spontaneously by 1 year of age. (Brunicardi et al., conjugated hyperbilirubinemia. Metabolic 2005, pp. 1504–1506; Greenfield et al., 2005, pp. screening for inborn errors of metabolism and 1882–1884; Townsend et al., 2004, pp. 2117–2119) serologic evaluation for intrauterine infections are important to exclude these causes of intra- 122. (A) hepatic cholestasis. Figure 6-20 depicts an acholic stool (absence of bile pigments), which 123. (B) is usually indicative of complete biliary tract obstruction. In this clinical setting, biliary atre- Explanations 122 and 123 sia is the most probable diagnosis. The most sensitive imaging study is radioisotope scan- Patients with gross hematuria require aggres- ning. Preimaging phenobarbital increases the sive diagnostic evaluation. A careful, planned diagnostic yield by stimulating hepatic micro- approach will yield the cause in the majority of somal enzymes. Abdominal ultrasound may patients. Painless hematuria is often the first show absence of the gallbladder, but this study sign of a urinary tract malignancy. After con- is operator dependent and does not evaluate firmation of hematuria, and exclusion of infec- hepatocyte function and bile excretory pattern. tion, all patients should have plain radiographs CT or MRI scans may demonstrate hepatic and IVP. This is the optimal initial diagnostic parenchymal changes (e.g., extensive cirrhosis) approach to aid in distinguishing between and the presence or absence of bile duct dilata- upper tract (renal) pathology and lower tract tion, but do not evaluate and differentiate (lower ureteric and bladder) pathology. Further abnormalities of hepatocyte function or bile diagnostic evaluation will be guided by these excretory pattern. (Greenfield et al., 2005, pp. noninvasive studies. A voiding cystourethro- 1929–1934; Townsend et al., 2004, pp. 2123–2124) gram is invasive. It is a limited examination of bladder function and anatomy, and although 125. (C) advanced invasive bladder tumors may be demonstrated as a filling defect, it is not sensi- 126. (B) tive for lower stages of bladder neoplasms. Cystourethroscopy is invasive and is, there- Explanations 125 and 126 fore, not the initial examination in the evaluation of hematuria. It is indicated in the evaluation of This patient has idiopathic thrombocytopenic gross hematuria in patients with a normal IVP. purpura (ITP), a disease characterized by a low It is the optimal tool for evaluation of potential platelet count, normal coagulation profile, bladder pathology. An abdominal ultrasound increased megakaryocytes, and a normal-sized or CT scan is indicated in patients with a sus- spleen. Patients with ITP will often demon- pected renal mass, either by clinical examina- strate excessive bleeding in response to a minor tion or demonstrated on IVP. Urine for cytology injury. Circulating antiplatelet antibodies coat is useful for screening of patients with sus- normal platelets, which are then sequestered by pected urinary tract malignancy, but it is falsely the spleen, with resultant platelet destruction. negative in approximately 20% of patients and The majority of patients respond to initial ther- should not be used as the only diagnostic eval- apy with systemic steroids. Splenectomy is uation. A transrectal ultrasound may be helpful indicated in patients who become steroid in evaluating the extent of invasion of a bladder dependent with significant side effects or in or prostatic neoplasm. Abdominal CT scan is a patients requiring increasing doses of steroids superior imaging study for this purpose. to maintain a satisfactory platelet count. The (Brunicardi et al., 2005, p. 1524; Townsend et al., 2004, entire spleen must be excised, including any pp. 2305–2308) accessory spleens found at surgery. Residual splenic parenchyma would result in persistent (c) ketabton.com: The Digital Library

Answers: 122–130 249

platelet sequestration. Splenectomy is not indi- the transfusion and antipyretics. If further cated in the initial management of ITP. Platelet transfusion is required, further reactions can transfusion is rarely required. Spontaneous be prevented by filtration of blood products bleeding is unusual unless the platelet counts for leukocyte reduction. Anaphylactic transfu- drop below 20,000/L. When this occurs, if the sion reactions are rare. Patients develop patient is not responsive to steroids, platelet urticaria, flushing, hypotension, and bron- transfusion and urgent splenectomy is indi- chospasm. O blood type is characterized by the cated. Antineoplastic chemotherapy is not used absence of ABO antigens on the red blood cell in the management of ITP. Expectant manage- surface. Therefore, type O blood is universally ment is associated with significant risk, as the accepted as the donor type for transfusion ther- most life-threatening complication of ITP is apy, making an acute hemolytic transfusion spontaneous intracerebral hemorrhage. Bone reaction from ABO incompatability impossi- marrow transplant is not indicated. ITP is a ble. Delayed hemolytic reactions usually occur disease of peripheral platelet destruction, with 1–3 weeks after a first transfusion and are man- normal or increased platelet production. ifested by an unexplained drop in hematocrit, (Brunicardi et al., 2005, pp. 1305–1306; Greenfield et al., associated with unconjugated hyperbiliru- 2005, pp. 1233–1235; Townsend et al., 2004, pp. binemia. Acute bacterial infection transmitted 1682–1685) through blood products is extremely rare and has been reported only in association with 127. (E) This patient has symptomatic anemia. The platelet concentrates stored at room tempera- decreased oxygen-carrying capacity has resulted ture. (Townsend et al., 2004, p. 130) in decreased tissue perfusion. The heart attempts to compensate with increased con- 129. (E) Any patient with this history and point tractility and heart rate, in an attempt to tenderness in the anatomic snuffbox must improve cardiac output and oxygen delivery. In be assumed to have a scaphoid fracture. this patient, however, this is inadequate and Undisplaced fractures may be difficult to visu- has also placed excess metabolic demands on alize on initial radiographs, even when multi- the myocardium with signs of ischemia. These ple views are obtained. The appropriate changes can be ameliorated with a blood trans- management is full immobilization of the fusion. Iron supplementation is indicated in scaphoid, which is achieved only with a cast the treatment of chronic iron-deficiency that extends to include the thumb. X-rays anemia. Restoration of iron stores and a normal should be repeated in 10–14 days, and if the red cell mass usually takes several months. fracture is confirmed, immobilization should Therefore, it is not appropriate in a patient with be continued. Avascular necrosis is a common symptomatic anemia. Supplemental oxygen complication. Minor wrist injury with liga- will not improve oxygen delivery in a patient mentous sprain may be adequately treated with limited oxygen-carrying capacity and with anti-inflammatory medication, applica- compensatory maximum oxygen extraction at tion of ice, an elastic wrist support, and the tissue level. IV fluid resuscitation will restricted activity. However, these are not ade- increase circulating blood volume, resulting in quate therapy for a suspected scaphoid frac- hemodilution and decreased red cell concen- ture. Furthermore, a wrist splint does not tration. Calcium channel blockade is indicated provide adequate immobilization of the for management of myocardial ischemia from scaphoid. (Townsend et al., 2004, p. 2218; Way and primary coronary or myocardial pathology. Doherty, 2006, p. 1127) (Townsend et al., 2004, pp. 125–130) 130. (B) Symptoms of zinc deficiency include diar- 128. (A) A febrile nonhemolytic transfusion reaction rhea, depression, alopecia, and perioral and is usually caused by an interaction between periorbital dermatitis. Patients at greater risk recipient antibodies and leukocytes in the trans- for developing this syndrome include those fused blood. Treatment is discontinuation of with high gastrointestinal fluid losses, patients (c) ketabton.com: The Digital Library

250 6: Surgery

with multisystem trauma, and patients on pro- oropharyngeal secretions. Nasotracheal intuba- longed parenteral nutrition. The symptoms tion is contraindicated in this setting. A defini- resolve with zinc supplementation. Iodine defi- tive tracheostomy is more time consuming than ciency results in hypothyroidism. Deficiency a cricothyroidotomy and requires specific sur- syndromes for selenium, silicon, and tin have gical expertise. Stabilization of the airway is the not been described. (Townsend et al., 2004, p. 156; first resuscitation priority, before placement of Way and Doherty, 2006, pp. 147–148) chest tubes to relieve potential pneumothoraces. (Townsend et al., 2004, pp. 490–491, 502; Way and 131. (B) Doherty, 2006, pp. 209–213)

132. (D) 134. (E) Frostbite is produced by formation of ice crystals in the tissue, with cessation of tissue Explanations 131 and 132 perfusion. Appropriate initial treatment is rapid rewarming in warm water, to minimize further In the initial postoperative period, the patient tissue damage. Dry heat can cause further tissue has a low pulmonary artery wedge pressure damage. With reperfusion, there is continued and poor urine output. Renal perfusion is com- progression of tissue injury because of progres- promised by hypovolemia, with subsequent sive microcirculatory thrombosis. Therefore, inadequate preload and decreased cardiac nonviable tissue should be allowed to demar- output. The appropriate therapeutic interven- cate over several weeks, with delayed debride- tion at this time is further IV fluid resuscitation. ment. A sympathectomy is not indicated Diuretics are contraindicated in the patient acutely, because the vasculature in frozen tissue with hypovolemia and are unlikely to improve is already maximally dilated. (Townsend et al., urine output in the face of inadequate renal 2004, p. 2216; Way and Doherty, 2006, pp. 257–259) perfusion. A dopamine infusion or digoxin may improve cardiac contractility but will not 135. (B) result in improvement in cardiac output unless there is adequate preload. In a hypovolemic 136. (B) patient, nitroprusside will result in a signifi- cant drop in blood pressure. After receiving a Explanations 135 and 136 fluid bolus, the patient develops distended neck veins and an elevated pulmonary wedge Ruptured cerebral aneurysms often occur in pressure, indicating biventricular dysfunction the setting of hypertension. The severe headache, with increased left ventricular end-diastolic nausea and vomiting, photophobia, and nuchal pressure, and increased left ventricular end- rigidity are the result of meningeal irritation systolic volume. Cardiac output is low, and from subarachnoid blood. Subarachnoid hem- urine output has not improved. In a patient orrhage is visualized on CT scan, with defini- with a history of hypertension, this clinical pic- tive diagnosis of the aneurysm and its location ture is often caused by increased afterload. by cerebral angiography. Early surgical clip- Afterload reduction can be obtained with a ping is the current neurosurgical approach nitroprusside infusion. (Townsend et al., 2004, pp. because of the significant risk of rebleeding in 310–316, 617–622) the first 24 hours after initial presentation. Hydrocephalus may occur as a late complica- 133. (C) This patient has an obstructed airway from tion of subarachnoid hemorrhage and require maxillofacial trauma. The patient is stridorous, serial lumbar puncture to drain CSF and con- hypoxic, and cannot be ventilated with bag and trol ICP. A hemorrhagic stroke can occur in mask. Immediate cricothyroidotomy is lifesav- association with malignant hypertension and ing. In the presence of severe facial trauma, oro- may have concurrent subarachnoid hemor- tracheal intubation is likely to be difficult because rhage. Focal neurologic signs are usually pres- of distortion of landmarks and excessive ent. Meningitis will produce similar signs of (c) ketabton.com: The Digital Library

Answers: 131–136 251

meningeal irritation, but usually with other sys- Focal neurologic signs are usually present. temic signs of infection and a clinical prodrome Evaluation of a possible cause includes suggesting an infectious etiology. Lumbar Doppler examination of the carotid arteries. puncture is diagnostic, and if a bacterial Management includes anticoagulation and source is suspected, systemic antibiotics are antiplatelet therapy. EEG measures brain elec- initiated pending culture of CSF. Ischemic trical activity and is indicated in the diagnos- cerebrovascular accidents and transient tic evaluation of seizures. (Brunicardi et al., 2005, ischemic attacks are not associated with sub- pp. 1629–1631; Greenfield et al., 2005, pp. 2054–2058; arachnoid hemorrhage and, hence, do not Townsend et al., 2004, pp. 2135–2139) present with signs of meningeal irritation. (c) ketabton.com: The Digital Library

BIBLIOGRAPHY

Brunicardi FC, Andersen DK, Billiar TR, et al., eds. Townsend CM Jr, Beauchamp RD, Evers BM, et al., Schwartz’s Principles of Surgery, 8th ed. New York, eds. Sabiston Textbook of Surgery: The Biologic Basis of NY: McGraw-Hill, 2005. Modern Surgical Practice, 17th ed. Philadelphia, PA: Donegan WL, Spratt JS, eds. Cancer of the Breast, 5th W.B. Saunders, 2004. ed. Philadelphia, PA: W.B. Saunders, 2002. Way LW, Doherty GM, eds. Current Surgical Diagnosis Fauci AS, Braunwald E, Isselbacher KJ, et al., eds. and Treatment, 124th ed. New York, NY: McGraw- Harrison’s Principles of Internal Medicine, 15th ed. Hill, 2006. New York, NY: McGraw-Hill, 2001. Zinner MJ. Maingot’s Abdominal Operations, 10th ed., Greenfield LJ, Mulholland M, Lillemoe KD, et al., Vols. I and II. Stamford, CT: Appleton & Lange, 1997. eds. Surgery: Scientific Principles and Practice, 4th ed. Philadelphia, PA: Lippincott-Raven, 2005.

252 (c) ketabton.com: The Digital Library

CHAPTER 7 Practice Test 1 Questions

Questions 1 through 3 Questions 5 and 6

Match the antidepressant with the side effect or 5. In a patient with diabetes, which of the follow- characteristic described. ing is the ideal low-density lipoprotein (LDL) cholesterol? (A) phenelzine (B) venlafaxine (A) >100 (C) trazodone (B) >130 (D) fluoxetine (C) <130 (E) mirtazapine (D) <100 (F) nortriptyline (E) >160 (G) escitalopram 6. Which of the following medications is most 1. Can cause hypertension at higher doses, but is useful in a patient with diabetes who is hyper- not a monoamine oxidase inhibitor (MAOI). tensive and has evidence for microalbuminuria? (A) metoprolol 2. Can cause priapism. (B) enalapril (C) clonidine 3. Can cause prolonged QT on electrocardiogram (ECG) (especially with high doses). (D) nifedipine (E) reserpine 4. A 22-year-old complains of vaginal spotting that began 37 days after her last menstrual 7. Labor is induced with a dilute IV oxytocin infu- period, and lasted 3 days. She denies any sion at 39 weeks’ gestation in a 29-year-old abdominal pain and states that her bleeding woman whose pregnancy was uncomplicated. has now stopped completely. She had a positive Labor and delivery occur without complica- home urine pregnancy test and you ordered a tions. Which one of the following is a con- serum hCG (human chorionic gonadotropin) traindication to the use of oxytocin? concentration earlier today, which returned as (A) oligohydramnios 432 mIU/mL. What should be the next step in (B) nonreactive nonstress test (NST) your management of this patient? (C) prior stillbirth at 41 weeks’ gestation (A) uterine curettage (D) prior low transverse cesarean section (B) transvaginal ultrasound of the pelvis (E) prior classical (vertical) cesarean section (C) diagnostic laparoscopy (D) hCG concentration in 24 hours (E) hCG concentration in 48 hours

253

Copyright © 2008 by The McGraw-Hill Companies, Inc. Click here for terms of use. (c) ketabton.com: The Digital Library

254 7: Practice Test 1

8. Which of the following groups of disorders is (C) sigmoid colon resection with most common as a cause of syncope? Hartmann’s procedure (A) psychiatric (D) abdominoperineal resection and perma- nent colostomy (B) arrhythmias (E) total abdominal colectomy and ileorectal (C) neurally mediated anastomosis (D) medication related (E) gastrointestinal 12. A 2-year-old boy is brought to the emergency department with a barky cough for 1 day. He Questions 9 through 11 has had congestion and rhinorrhea for the 2 A 55-year-old man with a history of diverticulosis days prior to this. On physical examination, he ° presents with a 2-week history of dysuria, urgency, has a temperature of 38.8 C and his respiratory and pneumaturia. Furthermore, he has a 3-month rate is 28. He has suprasternal retractions and history of vague, intermittent, left lower quadrant stridor at rest. Which is the best initial therapy? abdominal pain, and irregular bowel habits. On (A) albuterol via nebulizer examination, he has fullness on palpation in the left (B) racemic epinephrine via nebulizer lower quadrant, and urine culture demonstrates a (C) steroids via nebulizer polymicrobial infection. A computed tomography (CT) scan demonstrates an inflammatory mass in (D) antibiotics via nebulizer the left pelvis. (E) endotracheal intubation

9. Which of the following investigations is most 13. You are the sideline doctor at the high school likely to aid in diagnosis? soccer game. A 17-year-old girl suffers a head injury during the game. She collided with (A) voiding cystourethrogram another player while attempting to “head” the (B) CT scan with contrast ball and fell to the ground. The coach wants to (C) air contrast barium enema know if he can get his star player back into the (D) colonoscopy game. There was no loss of consciousness but (E) laparoscopy she was somewhat confused and disoriented for 10 minutes after the injury. Her neurologic 10. After confirming the diagnosis, the most appro- examination is completely normal. When can priate initial steps in the management of this she return to play? patient include which one of the following? (A) after 6 months of being symptom free (A) broad-spectrum antibiotics and insertion (B) after a week of no physical activity of an indwelling bladder catheter (C) only if her examination is normal in (B) outpatient oral antibiotics and stool 45 minutes softeners (D) after 20 minutes and being asymptomatic (C) clear fluids, magnesium citrate, and oral (E) immediately antibiotics (D) urgent exploratory laparotomy Questions 14 through 16 (E) proximal defunctioning colostomy A 22-year-old college student is evaluated by a psychiatrist for the onset of paranoid delusions, 11. Which of the following is the optimal definitive command auditory hallucinations, and an over- surgical management for this patient? whelming sense that his thoughts are being drawn (A) a proximal defunctioning loop colostomy out of his brain by “negative electromagnetic waves.” There is no prior psychiatric history. His (B) sigmoid colon resection with primary parents, who accompany him, report that 2 months anastomosis ago he had moved back home with them, isolated (c) ketabton.com: The Digital Library

Questions: 8–20 255

himself in his room, and became increasingly care- (A) ABO incompatability less about his personal appearance. He was (B) biliary atresia described as a quiet but friendly person with sev- (C) breast-feeding jaundice eral good friends. In both high school and college (D) neonatal hepatitis (except for the last 2 or 3 months), he was a good student and seemed to enjoy his studies as well as (E) Crigler-Najjar syndrome participating in soccer and track. It seemed to his parents that he was worried about graduation and 18. During an elective laparoscopic cholecystec- fears of finding a job. The patient denied the use of tomy, the anesthesiologist reports that the illicit drugs. It was noted that a paternal uncle had patient has developed a sudden drop in sys- a diagnosis of schizophrenia. tolic blood pressure, arterial desaturation, and an increase in ventilatory pressure. Which of 14. Which of the following is the best initial drug the following is the most appropriate step in to treat this man? management? (A) buspirone (A) an IV fluid bolus (B) bupropion (B) decompression of the pneumoperitoneum (C) fluvoxamine (C) insertion of a chest tube (D) clomipramine (D) re-evaluating the position of the endo- tracheal tube and obtaining a portable (E) quetiapine chest x-ray 15. The sense that thoughts were being drawn out (E) aborting the procedure and converting of his brain by “negative electromagnetic to an open cholecystectomy waves” is an example of which of the following? 19. A 22-year-old nulligravid woman had monthly (A) projection menses without pain until approximately 18 (B) loose association months ago. In the past 18 months, she has noted (C) delusion progressively more painful menses for which (D) hallucination she misses 2 days of work each month. She has no urinary or gastrointestinal symptoms. She (E) heightened sensitivity to environmental has never had sexual intercourse and has had no stimuli gynecologic surgery. The pain is partially relieved by nonsteroidal anti-inflammatory medication. 16. Which of the following supports the possibility Which of the following is the most likely cause of of a good prognosis for this patient? her secondary dysmenorrhea? (A) uncle with schizophrenia (A) endometriosis (B) history of good achievement in school (B) pelvic infection and adhesions (C) being single (C) pelvic congestion syndrome (D) insidious onset (D) cervical stenosis (E) no clear precipitating factors (E) irritable bowel syndrome 17. A 2-day-old breast-fed infant comes to your 20. Which of the following is a side effect or toxic office for her first checkup. Her birth weight effect of cisplatin? was 6 lb 12 oz. Today her weight is 6 lb 4 oz. Her physical examination is unremarkable (A) renal toxicity except that she is icteric. Her total bilirubin is 12 (B) leukopenia mg/dL with a conjugated component of 0.6 (C) atopic dermatitis mg/dL. The most likely reason for her jaundice (D) hemorrhagic cystitis is which of the following? (E) decreased visual acuity (c) ketabton.com: The Digital Library

256 7: Practice Test 1

21. A 4-year-old boy comes into the clinic for a sick 24. A 25-year-old welder is found to have diffuse visit. His mother is worried that he might have small pulmonary opacities on a routine chest sinusitis. Which of the following histories is x-ray. He has no respiratory complaints and is most suggestive of bacterial sinusitis? found to have normal ventilatory function on lung function tests. Which of the following (A) clear rhinorrhea for 3 days metals are most likely responsible for his (B) fever and headache for 5 days “benign pneumoconiosis?” (C) hoarseness and barky cough for 2 days (A) lead (D) mucopurulent rhinorrhea for 12 days (B) copper (E) unilateral nasal discharge and foul breath for 6 weeks (C) cadmium (D) iron 22. A 3-year-old boy is brought to the emergency (E) silver room with sudden onset of respiratory distress, wheezing, and cough. He has had no recent 25. A 23-year-old woman has a positive urine illnesses but did have bronchiolitis as an infant. ligase chain reaction (LCR) test for chlamydia. On physical examination, he has mild intercostal She is 8 weeks pregnant. Which of the following retractions and is anxious. There is decreased is the appropriate next step in diagnosis and aeration on the right side with inspiratory and treatment? expiratory wheezes on the right side only. (A) Repeat the LCR test. Which of the following would be the most likely diagnosis? (B) Prescribe a 7-day course of doxycycline. (C) Prescribe a 7-day course of amoxicillin (A) acute exacerbation of asthma or erythromycin, or a single dose of (B) foreign body aspiration azithromycin. (C) laryngotracheobronchitis (D) Order a culture to confirm diagnosis and (D) pneumonia determine sensitivity of the organism (E) recurrent bronchiolitis before starting treatment. (E) Order a polymerase chain reaction (PCR) 23. Which of the following statements about pacli- test to confirm the diagnosis before taxel (Taxol) is true? starting treatment. (A) It is extracted from the roots of the west- 26. A Puerto Rican woman, treated for depression, ern yew. is reported to be pulling out her hair. In evalu- (B) Myelosuppression and neuropathy are ating her, you note that her eyelashes are miss- the major dose-limiting effects. ing and her head hair is very thin, with patches (C) It acts by preventing microtubule of no hair evident. She repetitively pulls on her assembly. hair, but at least for the moment appears not to (D) It is used to treat advanced cervical pull any out. She reports that she has engaged cancer. in this behavior for almost a year. Which of the (E) Nausea and vomiting are uncommon following is the most likely diagnosis? side effects. (A) catatonic schizophrenia—excited form (B) trichotillomania (C) ghost sickness (D) mal de ojo (E) autism (c) ketabton.com: The Digital Library

Questions: 21–32 257

Questions 27 through 29 30. A 24-year-old G2P2 woman engages in unpro- tected intercourse on the 14th day of her men- A 52-year-old schoolteacher presents to the emer- strual cycle. Two days later, she requests gency department for evaluation of a tender, swollen, emergency contraception. Which of the fol- red left thigh. She just returned from spring break, lowing is effective contraception? during which she drove for 9 hours one way to visit her parents. She is a smoker whose only medications (A) one triphasic oral contraceptive tablet are estrogen and progesterone. Her vital signs, daily for 21 days starting on cycle day 16 including respiratory rate, are normal. Other than (B) methotrexate, 50 mg/m2 intramuscularly her leg, her examination is unremarkable. (IM) (C) diethylstilbestrol (DES), 5 mg orally one 27. Which of the following is the best initial step in time her evaluation? (D) danazol 200 mg orally taken once (A) ventilation-perfusion (VQ) scan (E) levonorgestrel 0.75 mg orally every (B) pulmonary angiography 12 hours for two doses (C) duplex scanning of her left leg 31. A thin, White 53-year-old woman who had her (D) venogram last menstrual period 2 years ago complains of (E) pulse oximetry hot flashes for 1 year, vaginal dryness, and sleep loss from night sweats. You offer hor- 28. After finding that she has thrombosis of the mone replacement with combination estrogen- left femoral vein, she is admitted for IV progesterone therapy, explaining that it will do heparin treatment. After 5 days of therapy, which of the following? her platelets are noted to be 45,000. Admission platelets were 375,000. You make the diagno- (A) increase her risk of endometrial carcinoma sis of heparin-induced thrombocytopenia (B) increase her risk of breast cancer (HIT). Which of the following is true about (C) increase orgasmic ability this syndrome? (D) increase bone mineral density (A) It usually occurs within hours after (E) increase her risk of colon cancer beginning heparin. (B) Risk usually is dose-dependent and 32. A 25-year-old nullipara consults you because increases with higher doses. she stopped menstruating 4 months ago. Her (C) It usually produces marked thrombocy- menses were monthly until 1 year ago, when topenia with counts less than 20,000. they became less frequent. She also complains of hot flashes, vaginal dryness, and discomfort (D) There is an IgA antibody that binds to a with intercourse. The most appropriate first complex of heparin and platelet factor 4. step in her evaluation is measurement of which (E) Risk is higher with use of porcine rather of the following? than bovine heparin. (A) serum thyroid-stimulating hormone 29. Which of the following is the most common (TSH) concentration complication? (B) serum prolactin concentration (A) bleeding (C) hCG concentration (B) arterial thrombosis (D) serum estradiol-17 concentration (C) deep venous thrombosis (DVT) (E) serum testosterone concentration (D) stroke (E) skin necrosis (c) ketabton.com: The Digital Library

258 7: Practice Test 1

33. An 80-year-old female resident of a nursing (A) Withholding information about progno- home presents to the emergency department sis and treatment of side effects so as with a history of sudden onset of crampy not to burden the patient. abdominal pain, vomiting, and abdominal dis- (B) Asking the patient how much she or he tention. Her last bowel movement was 4 days would like to know about the illness. before presentation, and she has not passed (C) Trusting that families and friends will flatus since the onset of her symptoms. On provide the emotional support the examination, the patient is mildly dehydrated patient needs. and afebrile, with a markedly distended but (D) Administering analgesic medications on nontender abdomen. Barium enema is shown a strict schedule so the patient will get in Figure 7-1. The patient receives IV resusci- the medication regularly. tation and nasogastric tube decompression. Which of the following is the most appropriate (E) Not referring for group therapy, which next step in the management of this patient? is of little value for this population. 35. An outbreak of cholera occurs in a village in South America. The source of the outbreak is thought to be a well used by the majority of the villagers. A public health worker is sent to deter- mine the microbiological quality of the water source. Which of the following organisms is most often used as a measure of fecal contamination? (A) coliform bacteria (B) parvoviruses (C) protozoa (D) helminths (E) amoebas

36. A 69-year-old male is recovering from a femoropopliteal bypass 5 days prior. He remains on heparin while he is being converted to coumadin. On morning rounds he is found to have a cold leg, and his graft has no pulse in it. His thigh has a large hematoma. His morn- FIG. 7-1 (Reproduced, with permission, from Zinner MJ. Maingot’s Abdominal Operations, 10th ed., Vol. 2. Stamford, CT: Appleton & ing laboratory findings include a prothrombin Lange, 1997.) time (PT) of 15, with international normalized ratio (INR) of 1.3 and a partial thromboplastin (A) urgent laparotomy time (PTT) of 90. His hematocrit (HCT) is 28, (B) admission to the hospital and serial down from 32, and his platelet count is 50,000. abdominal examinations Which of the following is the most appropriate initial action prior to taking the patient to the (C) placement of a rectal decompression operating room (OR) for graft thrombectomy? tube in the emergency department (D) rigid proctoscopy and directed place- (A) Hold heparin for 2 hours, increase ment of a colonic decompression tube coumadin. (E) colonic enemas (B) Turn heparin down, recheck PTT in 1 hour. 34. Which of the following would be the most (C) Stop all heparin, stop coumadin, start helpful in the management of the terminally ill argatroban, and send platelet factor patient? 4 for assay. (c) ketabton.com: The Digital Library

Questions: 33–41 259

(D) Stop all heparin, start low molecular 39. A 15-year-old girl comes to the clinic with a weight heparin (LMWH), and send complaint of headaches for the past several platelet factor 4 for assay. months. The headaches have become more (E) Continue heparin at current level, hold intense and frequent over time. They usually coumadin until after surgery. occur in the morning and are associated with nausea and vomiting. Her mother has noted 37. A private foundation wants certain protected decreased activity and worsening in her school health information such as names, addresses, performance over that same time. Her neuro- and antibiotics which will be used to set up a logic examination is normal. Which of these is new Tuberculosis Surveillance System. This is the most likely cause for her symptoms? being done in an effort to track and monitor (A) tension headache cases of multidrug resistance TB which seems (B) posterior fossa brain tumor to be on the rise in a particular community. Which of the following best describes the situ- (C) cluster headaches ation, according to the Health Insurance (D) migraine headache Portability and Accountability Act (HIPAA)? (E) depression (A) This will be possible because only dei- 40. An outbreak of fever, shock, and pulmonary dentified information is being used for edema without a known etiology is occurring the surveillance system. in your community in the southwestern United (B) This will be possible because private States. The Centers for Disease Control (CDC) foundations are not subject to privacy sends a team of experts to the area for the pur- rules governed by HIPAA. pose of finding the cause and stopping the out- (C) This will not be possible because private break. This is best done by using which of the foundations are only allowed to retrieve following? private health information under strict conditions which have not been met. (A) active surveillance (D) This will not be possible because private (B) passive surveillance foundations are not allowed to collect (C) sentinel surveillance private health information under any (D) case-control study circumstances. (E) mandatory case reporting (E) This will not be possible because tuber- culosis information can only be collected 41. A 7-year-old boy has had staring episodes for by the government. the past several months. They were first noted by his teacher, but now the parents have 38. A 37-year-old obese (body mass index [BMI] = noticed them as well. The boy suddenly stops 32 kg/m2) woman who smokes 10 cigarettes what he is doing and seems to stare vacantly daily requests oral contraception. She has a his- and move his lips. These last a few seconds tory of migraine headaches relieved with med- and he returns to what he was previously ication, cholelithiasis, adult onset diabetes doing. Which of the following is the most likely mellitus, and 10-week size uterine fibroids. explanation for these episodes? Which of the following is an absolute con- (A) absence seizures traindication to oral contraceptives? (B) attention deficit disorder (A) women over age 35 who use tobacco (C) breath holding spells products (D) cardiac dysrhythmia leading to syncope (B) migraine headaches (E) attention-seeking behavior (C) cholelithiasis (D) diabetes mellitus (E) uterine fibroids (c) ketabton.com: The Digital Library

260 7: Practice Test 1

42. A 56-year-old woman with no medical prob- 44. A 68-year-old woman with chronic obstructive lems complains of leakage of small amounts pulmonary disease (COPD) is admitted in of urine before she can make it to the bath- January with right middle lobe pneumonia. room. She denies leakage with coughing or She lives at home with her husband, has two sneezing, but has the sensation that she needs cats and two birds. Which of the following is to empty her bladder frequently. Her most the most likely cause of her pneumonia? severe episodes of leakage occur when she gets (A) Staphylococcus aureus home from shopping or while doing dishes. She had three vaginal deliveries of children (B) Chlamydia psittaci ranging in weight from 7 to 8 lbs. She is not (C) influenza taking hormone replacement therapy in any (D) Haemophilus influenzae form. Which of the following is most likely to (E) Streptococcus pneumoniae be causing her symptoms? (A) detrusor overactivity 45. A 2-year-old girl comes to the office with limp- ing and difficulty walking for 1 day. She com- (B) neurogenic bladder plains of pain in her right leg. On examination, (C) nephrolithiasis she is afebrile and has no point tenderness or (D) genuine stress incontinence swelling in her lower extremities. There is full (E) intrinsic urethral sphincter deficiency range of motion in all joints except pain with abduction of her right hip. Which of the fol- 43. A 24-year-old gravida 1 woman received mag- lowing is the most likely diagnosis? nesium sulfate for preeclampsia and oxytocin (A) slipped capital femoral epiphysis to induce labor at 39 weeks’ gestation. Labor and delivery were uneventful until 1 hour post- (B) aseptic necrosis of the femoral head partum when she had severe postpartum hem- (C) toxic synovitis orrhage that required vigorous fundal massage (D) septic arthritis of the hip and ergotamine to control the bleeding. (E) osteomyelitis Although she wished to breast-feed her son, she was unable to do so because she never pro- 46. A 62-year-old male with a past history of duced breast milk. Six months later, you see COPD, gastroesophageal reflux disease (GERD), her because she has yet to menstruate since the coronary artery disease (CAD), and a 40 pack- delivery. Which of the following is the most year history of smoking is brought to the emer- likely diagnosis? gency room with hemoptysis. He states that he (A) polycystic ovary syndrome has coughed up approximately one cup of blood at home, but he is able to maintain his (B) premature ovarian failure airway. He continues to cough up blood in the (C) endometrial sclerosis (Asherman’s emergency room. What is the most appropriate syndrome) initial therapy? (D) cervical stenosis (A) chest x-ray (E) postpartum pituitary failure (Sheehan’s syndrome) (B) emergent intubation (C) emergent thoracotomy (D) bronchoscopy (E) angiography with bronchial artery embolization (c) ketabton.com: The Digital Library

Answers and Explanations

1. (B) 5. (D)

2. (C) 6. (B)

3. (F) Explanations 5 and 6

Explanations 1 through 3 Fasting lipids measure an important risk factor for vascular disease, and ideally LDL values Fluoxetine has the longest half-life of the cur- are less than 100 in a patient with diabetes. rent selective serotonin reuptake inhibitors Renal disease is a common end-organ compli- (SSRIs) (escitalopram’s half-life is shorter—less cation of diabetes that can lead to microalbu- than 24 hours), phenelzine is an MAOI and minuria and renal insufficiency. Angiotensin- foods rich in tyramine can induce a hyperten- converting enzyme (ACE) inhibitors (enalapril sive crisis. Venlafaxine can induce hypertension, is an ACE inhibitor) have been shown to delay especially at higher doses. Trazodone can rarely progression from microalbuminuria to overt induce priapism (a painful sustained erection). nephropathy and subsequent renal failure in Nortriptyline is a tricyclic antidepressant, and patients with diabetes. (Kaspar et al., pp. 2164–2156, at high doses, it can cause arrhythmias. (Kaplan 2167) and Sadock, p. 1094, 1078, 1136, 1124) 7. (E) Oxytocin is contraindicated in women with 4. (E) In normal intrauterine pregnancy, the serum a prior classical cesarean section (vertical inci- hCG concentration should rise by at least 66% sion in the uterine fundus). The risk of uterine every 48 hours and often at least doubles. The rupture is as high as 5–10%. Oligohydramnios, hCG concentration will only rarely rise that fast nonreactive NST, and a prior stillbirth are in the case of ectopic pregnancy. In this patient potential indications for oxytocin induction of who otherwise appears completely stable, labor to prevent a possible adverse fetal out- repeating the hCG level in 48 hours would be the come. While oxytocin for labor induction is not best option. A normal intrauterine pregnancy contraindicated, there is an uncertain increased will usually be visualized by transvaginal ultra- risk in dehiscence of the uterine scar. The total sound when the hCG level approaches dose of oxytocin and the labor should be mon- 1500–2000 mIU/mL. Performing the ultrasound itored closely. (Scott et al., pp. 410–412) now at this low hCG level will likely not localize the pregnancy. Curettage should absolutely not 8. (C) Neurally mediated syncope (also called be performed until there is clear evidence that vasovagal or neurocardiogenic) is most common this is an abnormal intrauterine pregnancy or as a cause of syncope. Studies from the 1980s when hCG levels are not rising appropriately showed that approximately 24% of syncope is and ectopic pregnancy needs to be ruled out. neurally mediated and 34% had an unknown Laparoscopy may be useful in this patient in the cause. More recently, through increased use of future, but first attempts should be made to local- tilt table testing, about 50–66% of the cases pre- ize the pregnancy with hCG levels and ultra- viously classified as unknown are now thought sound. If her hCG level exceeds 2000 mIU/mL to be neurally mediated. Medications cause and the ultrasound is normal, laparoscopy approximately 3%, psychiatric disorders 2%, would be the next step. (Scott et al., pp. 92–94) and arrhythmias 14%. (Kasper, pp. 126–128)

261 (c) ketabton.com: The Digital Library

262 7: Practice Test 1

9. (B) rectum. The sigmoid colon is the most common location for symptomatic diverticular disease 10. (A) resulting in a colovesical fistula. A total abdom- inal colectomy is therefore rarely indicated. 11. (B) (Brunicardi et al., pp. 1083–1084; Greenfield et al., pp. 1130–1135; Townsend et al., pp. 1417–1422) Explanations 9 through 11 12. (B) Croup (acute laryngotracheobronchitis) is This patient with diverticular disease has the most common cause of acute stridor in chil- developed a colovesical fistula from divertic- dren. It is more common during late fall and ulitis, with localized sigmoid perforation into winter. There is typically an upper respiratory the adjacent bladder. Patients will present with infection (URI) before stridor develops. Many signs and symptoms of a urinary tract infec- children will have a barky cough and mild res- tion, with air in the urinary stream and multi- piratory distress. Most episodes of croups will ple fecal organisms on urine culture. CT scan is be self-limited but some require therapy. The most useful in the diagnostic evaluation, and acute treatment relies on inhaled racemic epi- will often demonstrate air within the bladder. nephrine to reduce airway edema and stridor. An air contrast barium enema may show evi- Therapy with dexamethasone following racemic dence of diverticular disease and is useful to epinephrine is indicated to prevent recurrence of exclude other pathology. The diagnostic yield stridor. (Behrman, Kliegman, and Jenson, pp. 1405–1408) of cystography and colonoscopy are approxi- mately 20%. Diagnostic laparoscopy is not indi- 13. (D) Head injuries are common in high school cated. This patient should be managed with sports such as football and soccer. Guidelines broad-spectrum antibiotics and bladder drainage. for return to play are based on severity of head When the acute inflammatory process has injury. The athlete here has a minor head injury resolved, definitive surgical therapy can be per- (grade I) based on the Colorado Medical formed electively. These patients will tolerate a Society Guidelines. She may return to play after gentle bowel prep and can then undergo a sig- 20 minutes as long as she is asymptomatic and moid colon resection and primary anastomosis. has a normal neurologic evaluation. More severe Outpatient management is not optimal, and or repeated minor injuries call for longer dura- oral antibiotics are insufficient therapy. Clear tion of exclusion from participation. (Behrman, fluids, magnesium citrate, and oral antibiotics Kliegman, and Jenson, p. 2313) are components of a surgical bowel prep. This should only be undertaken after treatment of 14. (E) This young man has psychotic symptoms. the urinary tract sepsis and a period of bowel Of the medications listed, quetiapine is the only rest. A colovesical fistula is not a surgical emer- antipsychotic. This is a benzothiazepine with high gency. Management of the complications of affinity for serotonin types 2 and 6 (5-HT2 and diverticular disease by proximal defunction- 5-HT6), moderate affinity for dopamine type 2 ing colostomy is reserved for patients who are (D2) receptors, and lower for dopamine type 1 profoundly ill and require diversion of the fecal (D1) and lower still for dopamine type 4 (D4). It stream to control continued intra-abdominal has been found effective for various psychotic and pelvic sepsis. This is not definitive therapy disorders and is fairly new among the antipsy- for a colovesical fistula, because it leaves a chotic agents. Buspirone is a nonbenzodi- column of stool above the fistula, with contin- azepine anxiolytic agent. Bupropion is an ued fecal contamination of the urinary tract. A antidepressant drug recently experiencing resection with Hartmann’s procedure has the renewed use in smoking cessation programs. disadvantage of requiring a second major Fluvoxamine, an antidepressant (SSRI), and laparotomy to re-establish intestinal continuity. clomipramine, an antidepressant, have been of An abdominoperineal resection is not required particular value in the treatment of obsessive- for the management of diverticular disease. compulsive disorder (OCD). (Kaplan and Sadock, Diverticular disease does not involve the pp. 497–498, 1029–1032, 1127) (c) ketabton.com: The Digital Library

Answers: 9–20 263

15. (C) The sense that thoughts are drawn out of 18. (C) This patient has developed a tension pneu- one’s brain by “negative electromagnetic mothorax from dissection of carbon dioxide into waves” is a delusion, a false belief that cannot be the pleural space, either via an unrecognized changed by logical argument. Other responses defect in the diaphragm or via the retroperi- noted here may also be found in this young toneal and retropleural spaces. Treatment is man’s disorder. Projection is attributing to immediate decompression with a chest tube. another person or thing one’s own impulses or Hypotension associated with a tension pneu- feelings. In loose associations, thoughts shift mothorax is secondary to decreased venous without any apparent connection to one return from mediastinal shift. IV fluids are another. Hallucinations are sensory experi- indicated only if there is associated hypov- ences not associated with any real external olemia. Release of the pneumoperitoneum stimuli. Heightened sensitivity to environ- may prevent further carbon dioxide accumu- mental stimuli, although occurring in most lation, but chest tube decompression is still people at some time in their lives, has gone required. A tension pneumothorax is a clinical beyond what is reasonable in this young man. diagnosis and does not require a chest x-ray (Kaplan and Sadock, p. 283) for confirmation. Once the pneumothorax is adequately decompressed and the patient is 16. (B) A history of good achievement in school stabilized, the laparoscopic procedure may suggests good premorbid functioning, which in continue. (Townsend et al., pp. 452–454, 508) turn supports the possibility of a good prog- nostic outcome. All the other items weigh 19. (A) Primary dysmenorrhea begins shortly toward poor outcome when one is assessing after menarche and is not associated with prognosis. One needs to factor in all the various pelvic pathology, but rather is mediated by indicators, positive and negative, that point to prostaglandins. It generally responds well to the eventual outcome. This is critical in pro- nonsteroidal anti-inflammatory medications. viding hope for the patient as well as for Secondary dysmenorrhea arises later and is making reasonable future plans. (Kaplan and usually associated with some form of pelvic Sadock, p. 485) pathology. The onset of pain remote from menarche and the progressive nature of the 17. (C) Breast-feeding jaundice is related to the pain indicate secondary dysmenorrhea, most sequence in production of milk in mothers probably associated with endometriosis. who are breast-feeding. Normally, milk pro- Pelvic infection and adhesions are unlikely in duction will not occur until about 4 days after a woman who has never been sexually active. delivery. Until that time the intake by the new- Cervical cryosurgery, conization, or other cer- born will consist of colostrum that may not be vical procedures increase the risk of cervical adequate for hydration and to allow removal stenosis. Dysmenorrhea occurs because of the of bilirubin via stooling. Mild dehydration increased risk of retrograde menstruation and and weight loss may occur and the jaundice obstruction to outflow. The pain with pelvic that occurs is secondary to enterohepatic cir- congestion syndrome, essentially pelvic culation of the bilirubin. This is not a patho- venous varicosities, is more constant and usu- logic condition and will resolve once the milk ally described as a pressure or fullness. supply increases. Infants should be followed Although symptoms of irritable bowel syn- to assure adequate intake is occurring and drome may be aggravated around the time of jaundice is resolving. ABO incompatibility menses, the absence of any gastrointestinal will result in higher concentrations of biliru- symptoms make this an unlikely diagnosis. bin. An infant with neonatal hepatitis will (Speroff and Fitz, p. 1108) appear ill and biliary atresia and Crigler- Najjar will present with conjugated hyper- 20. (A) Dose-related and cumulative renal insuffi- bilirubinemia outside of the first week of life. ciency is the major dose-limiting toxicity of cis- (Rudolph & Rudolph, pp. 165–166) platin. The main route of excretion of cisplatin (c) ketabton.com: The Digital Library

264 7: Practice Test 1

is through the kidneys, and good hydration 24. (D) Iron siderosis is often referred to as a may help minimize potential tubular damage. benign pneumoconiosis because only limited Other toxic reactions include myelosuppres- clinical effects are seen with iron inhalation. sion and neurotoxicity. Dermatitis, cystitis, and This term is used to label those pneumoconiosis decreased visual acuity are not toxic effects of which generally have a favorable prognosis cisplatin. Cisplatin is commonly used to treat because they are caused by relatively inert ovarian cancer. (Kasper et al., p. 475, 555) (i.e., nonfibrogenic) dusts, in contrast to those that may evolve toward debilitating pul- 21. (D) Bacterial sinusitis is a common complica- monary fibrosis. Findings are most often lim- tion of a viral URI. Five to ten percent of URIs ited to radiologic findings such as those develop into bacterial sinusitis. If mucopu- mentioned above. But workers are most often rulent discharge occurs for more than 7 days, exposed to more than just iron dust, and com- bacterial sinusitis must be considered. Clear binations of iron with other substances such as rhinorrhea may indicate allergic rhinitis. A asbestos or radon can cause significant lung barky cough is more indicative of croup. A damage. Iron siderosis is most commonly seen patient with unilateral discharge may have a in welders, foundry workers, or grinders. nasal foreign body. (Rudolph & Rudolph, pp. (Wallace and Doebbeling, p. 499) 1263–1264) 25. (C) The LCR test is highly sensitive and spe- 22. (B) The sudden onset of respiratory distress cific, and a confirmatory test is not needed. In can be due to numerous causes. The history is a nonpregnant woman or man, a 7-day course of utmost importance in assisting in the diag- of doxycycline or a single dose of azithromycin nosis. The sudden onset in a previously healthy would be effective treatment. Tetracyclines are toddler is most likely not infectious. The pres- contraindicated in pregnancy, however, due to ence of unilateral wheezing on the right side is effects on fetal bone and cartilage. For a preg- suspicious for a bronchial foreign body. An nant woman, one would prescribe a course of asthma exacerbation would occur in a child amoxicillin, 500 mg by mouth three times per with a previous history and typically has bilat- day for 7 days, or erythromycin, 500 mg by eral wheezing. Laryngotracheobronchitis (croup) mouth four times per day for 7 days, or results in stridor and has a preceding viral ill- azithromycin, 1 g by mouth as a single dose, ness. While pneumonia and bronchiolitis are and maintain abstinence for 7 days. Repeat test- infectious diseases and will not generally pres- ing at least 3 weeks after treatment. (CDC ent with acute onset of severe symptoms. MMWR 2006: Vol. 55, pp. 38-40; USPSTF, pp. 1–80) (Behrman, Kliegman, and Jenson, pp. 1410–1411) 26. (B) Trichotillomania is an impulse control dis- 23. (B) Taxol is frequently used in the treatment of order characterized by recurrent compulsive advanced epithelial ovarian cancers. Originally, pulling out of one’s hair from any area of the it was derived from the bark of the western body. The symptoms described here give no yew tree, but can now be chemically synthe- support for a diagnosis of catatonic (nor any) sized. It is a mitotic spindle inhibitor, but acts schizophrenia. Ghost sickness and mal de ojo in a manner different from other such inhibitors. are culture-bound syndromes. Ghost sickness is Taxol promotes assembly of microtubules and seen in members of American Indian tribes and stabilizes them, thus preventing depolymer- is characterized by a preoccupation with death ization and cell duplication. Myelosuppression and the deceased. Associated symptoms and peripheral neuropathy are the major toxic include bad dreams, weakness, hallucinations, effects that limit the utility of taxol, and other feelings of futility, and others. Mal de ojo (evil typical side effects of chemotherapy, such as eye) is seen in Mediterranean cultures. Persons alopecia, nausea, and vomiting are also seen. with this disorder experience disturbed sleep, (Ropper and Brown, p. 1040) crying without apparent cause, diarrhea, and (c) ketabton.com: The Digital Library

Answers: 21–31 265

vomiting. It is usually seen in children. Autism 30. (E) Levonorgestrel 0.75 mg every 12 hours for is seen in children and is characterized by a two doses will prevent 75–89% of pregnancies profoundly abnormal and impaired ability to when taken within 72 hours of unprotected interact socially with others. (Kaplan and Sadock, intercourse. This is marketed under the name pp. 790–792) “plan B.” Effective postcoital contraception requires inhibition of the preovulatory luteiniz- 27. (C) DVT is variable in its presentation. A ing hormone (LH) surge and subsequent ovu- patient may have pain, discoloration, and a lation. A triphasic oral contraceptive taken palpable cord, but these classic findings are daily for 21 days will not prevent ovulation low in sensitivity and specificity. Unilateral leg when begun on cycle day 16. Methotrexate edema may be the only finding and is the most causes necrosis of actively dividing cells (e.g., sensitive indicator of DVT. Duplex scanning is trophoblasts) and will not suppress ovulation. the most sensitive and specific of methods, Moreover, there is no clinical trial to demon- especially for DVT above the knee, with a sen- strate its effectiveness as a postcoital contra- sitivity of 95%, but is less sensitive for clots ceptive. DES may be an effective postcoital below the knee. Impedance plethysmography contraceptive, but it has not been evaluated for is also an option, but has a high rate of false this and will cause significant nausea and vom- positives. Venography is the most definitive iting. Likewise, danazol has never been tested technique for detection, but is uncomfortable, as a postcoital contraceptive. (Scott et al., p. 555) costly, technically difficult, and may even induce thrombophlebitis. Although she is at 31. (B) The Women’s Health Initiative (WHI) is a risk for pulmonary embolus (PE), her respira- National Institute of Health (NIH) funded ran- tory rate is normal, and there is nothing to sug- domized, prospective, controlled, double- gest PE, so VQ scan, pulmonary angiography, blind clinical trial of combination estrogen + and pulse oximetry are not crucial. (Kasper et al., progestin, estrogen only (in women who had pp. 1491–1492) a prior hysterectomy), and no HRT. After 5.5 years, the investigators stopped the combina- 28. (B) tion HRT arm of the study because of a 26% increase in breast cancer (38/10,000 vs. 29. (C) 30/10,000 women). About 1 year later, the estrogen only arm was stopped because of an Explanations 28 and 29 increase in breast cancer in the treated group. HIT has been increasing in incidence and is an Though the increase in both arms was not sta- important immunologic drug reaction because tistically significant, the treatment arms were of its potential complications. Evidence sug- stopped because the frequency exceeded gests that its risk is dose-dependent. The high- the predicted acceptable risk established by est risk occurs when heparin is given at full the study’s Data Safety Monitoring Board doses, and risk is lower with intermediate-dose before the study began. Primarily as a conse- heparin (7500 U twice daily). It usually occurs quence of the WHI results, the Food and Drug 5–15 days into therapy, and thrombocytopenia Administration (FDA) has established recom- is moderate (counts range from 25,000 to mendations for the use of HRT: (1) indicated 100,000). Typically, an IgG antibody (very for relief of menopausal symptoms (hot rarely IgM, but not IgA) binds to heparin. Risk flashes, vaginal dryness), (2) the lowest effec- of HIT is higher with bovine rather than tive dose, (3) the shortest possible duration of porcine heparin. Serious thrombotic complica- treatment, (4) use of alternatives if the indica- tions can occur, both arterial and venous, with tion is other than symptomatic relief, such as venous four times as common. Bleeding is not osteoporosis prevention. The addition of a a common complication. Stroke and skin necro- progestin does not seem to reduce this risk, sis can occur, but less commonly. (Kasper et al., and may actually increase the risk further. p. 675, 689, 692) Despite serious concerns about methodological (c) ketabton.com: The Digital Library

266 7: Practice Test 1

problems in these studies (small number of amenorrhea is accompanied by signs of andro- women with breast cancer, selection bias, gen excess (e.g., hirsutism, deepening of the ascertainment bias, and so forth), women voice, clitoral enlargement, and so forth). should be informed of this potential risk, but (Speroff and Fritz, pp. 404–409) also informed of the benefits of HRT. Although estrogen given alone increases the 33. (D) This patient presents with signs and symp- risk of endometrial adenocarcinoma three- to toms of a sigmoid volvulus. Most commonly, eightfold, estrogen replacement given with this occurs in the elderly debilitated patient with adequate progesterone supplementation actu- a history of chronic constipation, resulting in ally decreases the likelihood of developing the development of a long, redundant, mobile endometrial carcinoma. At least 70 mg of sigmoid colon. Patients present with a clinical pic- medroxyprogesterone acetate (Provera, Cycrin) ture of an acute colonic obstruction. Abdominal monthly is necessary to prevent an increased radiographs will show a large dilated loop of risk of endometrial cancer. Culture and atti- colon originating in the left lower abdomen, tude, plus the availability of a suitable sexual and extending to the right upper abdomen. partner, is more responsible than estrogen defi- Management includes decompression with a ciency for the decline in sexual activity and rigid proctoscope, which will usually result in response after menopause. Estrogen replace- detorsion of the bowel. Placement of a sigmoid ment will stabilize bone mineral density but decompression tube is necessary to prevent will not increase it. Evidence is increasing that recurrence. Urgent laparotomy is indicated in estrogen replacement decreases the risk of patients with fever, abdominal tenderness, and colon cancer. (Speroff and Fritz, pp. 689–777) metabolic acidosis, suggesting colonic ischemia. Patients admitted to the hospital who do not 32. (C) The most common cause of secondary undergo urgent decompression are at risk of amenorrhea in reproductive-age women is developing colonic ischemia from the closed- pregnancy. Therefore, a pregnancy test should loop obstruction. A rectal decompression tube is be the first step in the evaluation of secondary insufficient for decompression because the tip is amenorrhea. Her complaint of hot flashes and located distal to the obstruction. Colonic enemas vaginal dryness suggest estrogen deficiency as are ineffective in the face of a complete obstruc- a cause of her amenorrhea. Estrogen deficiency tion and may, in fact, further increase colonic may occur as a result of hypothalamic-pituitary distention. (Townsend et al., pp. 1422–1424; Way and failure and the loss of follicle stimulating hor- Doherty, pp. 721–722) mone (FSH) and LH secretion leading to a diminished ovarian estrogen secretion. The 34. (D) Patients with terminal illnesses have spe- presence of hot flashes suggests that the loss of cific needs and present particular problems for estrogen is the result of ovarian failure, because in-hospital and outpatient management. hot flashes typically occur as a result of (a) pres- Families are often fearful, confused, and ence of estrogen, followed by loss of estrogen unclear as to how to relate to these patients. and (b) increased FSH and LH concentrations. House officers are not immune to this prob- Hypothyroidism and hyperprolactinemia can lem. Analgesics for pain relief are generally also cause secondary amenorrhea. Serum estra- more efficacious when administered on a reg- diol concentrations are less useful in assessing ular schedule, allowing the patient to refuse. the cause of amenorrhea than measurement of The inappropriate use of prn regimens stems FSH. A decreased estradiol concentration from a prevalent but unsubstantiated fear that occurs with either hypothalamic-pituitary fail- patients will become addicted to the medica- ure or ovarian failure. A decreased serum FSH tion. In fact, studies have shown that fewer concentration indicates hypothalamic-pituitary than 1% of patients treated with pain medica- failure, whereas, an elevated FSH concentra- tion become addicted. Patients confronting tion indicates ovarian failure. A serum testos- death are acutely socially sensitive, and mis- terone concentration is appropriate only if the communications or evasive discussions are (c) ketabton.com: The Digital Library

Answers: 32–39 267

interpreted with the most negative outlook. suspicion is high, then a serotonin-release assay Patients generally prefer honest information, can be done to confirm the diagnosis. especially if they have to make decisions about Coumadin should not be started until after the subjecting themselves to often painful procedures platelet count recovers to >100,000. (Brunicardi and treatments. Checking with the patient on et al., p. 69; Townsend et al., p. 633) how much information he or she would like about the illness helps clarify the direction the 37. (C) Private foundations are regulated by physician will follow to effectively address the HIPAA privacy rules. They are allowed to patient’s information needs. Studies of groups collect health information provided that (1) of terminally ill patients with metastatic carci- they have obtained a release of information noma described group therapy as an excep- from the patients, (2) the personal health tionally effective mode of treatment. It offers a information is deidentified (the information close support system based on a common bond which is planned for use in the surveillance and allows the patients to learn from each other system has not been deidentified), (3) the how to cope and how to help their loved ones personal health information is contained in a cope with the stresses of terminal illness. (Kaplan limited data set governed by a data-use and Sadock, pp. 1338–1347) agreement (this is not mentioned in the vignette), (4) release of the personal health 35. (A) Determination of whether coliform bacteria information is in accord with the privacy are present has been used as a measure of fecal rule’s provisions for disclosure for research contamination since the early twentieth cen- without authorization (this has not been clearly tury. Coliform bacteria, whether fecal or nonfe- stated in the vignette). (HIPAA Privacy Rule and cal, should not be present in significant Public Health Guidance from CDC and the U.S. DHHS, numbers in any potable water supply. (Wallace http://www.cdc.gov/mmwr/preview/mmwrhtml/m2e and Doebbeling, pp. 745–746) 411a1.htm, 2004)

36. (C) This patient has the signs of HIT. In HIT, the 38. (A) Oral contraceptives are absolutely con- platelet count begins to fall 5–7 days after traindicated in women over 35 who smoke heparin has been started, but if the patient has because there is an additive risk of smoking been previously exposed to heparin, it can fall and estrogen for heart attack and stroke. The as early as 1–2 days after exposure. The throm- other choices are all relative contraindications bocytopenia is not usually severe. HIT should that require clinical judgment, thorough dis- be suspected if the platelet count falls to less cussion with the patient about contraceptive than 100,000, or if it fall to less than 50% of options, and informed consent. The discussion baseline. HIT is also characterized by a high and consent must be documented in the med- incidence of thrombosis that can be arterial, ical record. In this author’s opinion, the patient venous, or graft related. HIT is most common should be counseled to choose a nonhormonal with unfractionated heparin, but can occur contraceptive method or at least a hormonal with LMWH. If HIT is suspected, the appro- contraceptive that does not contain estrogen priate first treatment is to withdraw all sources (e.g., progestin-only oral contraceptive, Depot of heparin, including heparin-bonded catheters medroxyprogesterone acetate, or an intrauter- and heparin-locked IVs. In addition, another ine device). Transdermal and transvaginal con- source of anticoagulation needs to be started, to traception likely have the same heart attack prevent further thrombosis. The alternative and stroke risk as a combination oral contra- sources of anticoagulation available in the ceptive. (Speroff and Fritz, p. 906) United States include argatroban and lepirudin. Arixtra is an injectable form which also may be 39. (B) Headaches in adolescence are common and used. The initial assay for HIT is an assay for can have varying causes. The triad of platelet factor 4. If this is positive, the diagnosis headache, nausea, and vomiting in the morn- is confirmed. If it is negative, and the clinical ing are concerning for a brain tumor in the (c) ketabton.com: The Digital Library

268 7: Practice Test 1

posterior fossa. These tumors cause impair- deficiency, a weakness of the urinary sphincter ment in drainage of cerebrospinal fluid (CSF) mechanism itself. Urge incontinence is charac- with a resultant increase in intracranial pres- terized by leakage of small to large amounts of sure when lying down. The intracranial pressure urine typically associated with some provoca- subsides with upright posture. This patient tion, such as getting near a bathroom or hear- needs evaluation with a cranial MRI or CT ing running water. Detrusor overactivity is scan. The other causes of headache have his- defined as involuntary bladder contractions torical features distinct from this scenario. not under central brain control. Neurogenic (Rudolph, pp. 474–475, 2709–2710) bladder occurs in cases of advanced diabetes or multiple sclerosis where bladder reflexes 40. (D) Surveillance systems focus on descriptive become abnormal. Nephrolithiasis does not information that is analyzed to detect distribu- cause asymptomatic incontinence. (Scott et al., pp. tions and trends. Surveillance systems are used 845–866) to detect whether or not a problem exists, and how it is changing. Case-control studies are 43. (E) Postpartum amenorrhea in a woman with used once a problem has in fact been detected, postpartum hemorrhage, especially if the preg- and help assess causation and how the problem nancy was complicated by hypertension, is may be controlled and prevented. (Wallace and most likely the result of Sheehan’s syndrome. Doebbeling, pp. 8–9) This is a disorder caused by pituitary necrosis and resulting in gonadotropin deficiency. More 41. (A) The patient described here has a classic his- severe forms will include deficiency of other tory for absence (petit mal) seizures. These are pituitary trophic hormones, including TSH generalized seizures that occur in childhood and adrenocorticotropic hormone (ACTH). The and have a typical electroencephalogram (EEG) clinical clue that reinforces this diagnosis is pattern during seizures. This type of seizure the inability to lactate. This is presumed to be the commonly occurs between 4 and 12 years of result of necrosis of pituitary lactotropes and age and consists of short periods of epilepti- severely decreased pituitary prolactin secre- form activity that results in the clinical find- tion. Endometrial sclerosis or cervical stenosis ings. The typical history is of staring episodes would be more likely if the history included a that occur at increasing intervals over time. postpartum dilation and curettage (D&C) to Treatment with antiepileptic medication is control the hemorrhage. Although there is no required to eliminate or reduce seizure fre- clinical information to exclude polycystic ovary quency. Outcome varies with some patient’s syndrome or premature ovarian failure, these seizures resolving over time while others will are unlikely and, for premature ovarian fail- progress to generalized tonic-clonic seizures ure, would be coincidental, not causally related and lifelong epilepsy. (Behrman, Kliegman, and to the obstetric hemorrhage. (Cunningham et al., Jenson, 1997) pp. 825–826)

42. (A) Urinary incontinence is a common prob- 44. (E) S. pneumoniae is the most common bacterial lem, affecting 15–50% of adult women at some pathogen in community-acquired pneumonia. point in their lives. There are many causes, but This is the case no matter if the patient is young these can be subdivided into two major cate- or old or immunocompromised. If the pneu- gories: stress incontinence and urge inconti- monia goes undiagnosed and the patient does nence. Stress incontinence is marked by leakage not improve, you certainly would consider C. of small amounts of urine when intra-abdominal psittaci given the birds in her home, but this is pressure is increased, such as laughing, cough- a rare cause of pneumonia. Also, H. influenzae ing, or sneezing. This can be caused by ure- is more common in patients with COPD but thral hypermobility due to pelvic floor damage still not as common as S. pneumoniae. (Kaspar et al., during childbirth or by intrinsic sphincter pp. 1530–1534) (c) ketabton.com: The Digital Library

Answers: 40–46 269

45. (C) Toxic synovitis (transient synovitis of the amount of bleeding. In all patients, oxygen hip) is a common nonspecific inflammation of should be administered, and two large-bore IVs the hip joint of unknown etiology. It is a self- should be placed. In patients with significant, limited condition that resolves within a week. persistent but nonmassive bleeding, the airway It frequently follows a bacterial or viral infec- is generally able to be maintained without tion. Complaints include hip pain and a limp. emergent intubation. The first diagnostic test is Physical examination reveals discomfort at the generally a CXR. This can localize the prob- hip with abduction. The other diagnoses listed lem, and may be followed by either chest CT or here will also present with hip pain. A slipped bronchoscopy. In cases of massive hemoptysis, capital femoral epiphysis occurs later in life the patient generally cannot safely maintain an and is generally seen in overweight children. airway, and emergent intubation is necessary. Aseptic necrosis of the femoral head will result Most cases of massive hemoptysis arise from in a painless limp and limitation of hip move- the bronchial arterial tree, and bronchial ment. Osteomyelitis and septic arthritis are angiography with embolization is frequently infectious and generally present with fever and the best therapeutic option. Thoracotomy with other signs of infection. (Rudolph, pp. 2436–2737) definitive treatment of the causative lesion is usually accomplished after embolization con- 46. (A) Significant hemoptysis has many causes, trols the bleeding. (Brunicardi et al., pp. 581–582; but the initial therapy can be decided depend- Townsend et al., pp. 1806–1807) ing on the ability to maintain an airway, and the (c) ketabton.com: The Digital Library

BIBLIOGRAPHY

Behrman RE, Kliegman RM, Jenson HB. Nelson Kasper DL, Braunwald E, Fauci A, et al. Harrison’s Textbook of Pediatrics, 17th ed. Philadelphia, PA: Principles of Internal Medicine, 16th ed. New York, W.B. Saunders, 2006. NY: McGraw-Hill, 2005. Brunicardi FC, Andersen DK, Billiar TR, et al., eds. Ropper and Brown, p. 1040 Schwartz’s Principles of Surgery, 8th ed. New York, Rudolph CD, Rudolph AM. Pediatrics, 21st ed. New NY: McGraw-Hill, 2005. York, NY: McGraw-Hill, 2003. Centers for Disease Control and Prevention. Scott JR, Gibbs RS, Karlan BY, et al., eds. Danforth’s MMWR Morbidity and Mortality Weekly Report. Obstetrics and Gynecology, 9th ed. Philadelphia, PA: Recommendations and Reports. Sexually Transmitted Lippincott Williams & Wilkins, 2003. Diseases Guidelines, Vol. 55, No. RR-11, August 4, Speroff L, Fritz MA. Clinical Gynecologic Endocrinology 2006, pp. 38–40. and Infertility, 7th ed. Philadelphia, PA: Lippincott Cunningham FG, Leveno KJ, Bloom SL, et al. Williams Williams & Wilkins, 2005. Obstetrics, 22nd ed. New York, NY: McGraw Hill, Townsend CM Jr, Beauchamp RD, Evers BM, et al., 2005. eds. Sabiston Textbook of Surgery: The Biologic Basis of Greenfield LJ, Mulholland M, Lillemoe KD, et al., Modern Surgical Practice, 17th ed. Philadelphia, PA: eds. Surgery: Scientific Principles and Practice, 4th W.B. Saunders, 2004. ed. Philadelphia, PA: Lippincott-Raven, 2005. USPSTF, The Guide to Clinical Preventive Services, 2006. HIPAA Privacy Rule and Public Health Guidance from Wallace RB, Doebbeling BN, eds. Maxcy-Rosenau-Last CDC and the U.S. DHHS, http://www.cdc.gov/mmwr/ Textbook of Public Health & Preventive Medicine, 14th preview/mmwrhtml/m2e 411a1.htm, 2004. ed. Stamford, CT: Appleton & Lange, 1998. Kaplan HI, Sadock BJ. Synopsis of Psychiatry: Way LW, Doherty GM, eds. Current Surgical Diagnosis Behavioral Sciences/Clinical Psychiatry, 9th ed. and Treatment, 124th ed. New York, NY: McGraw- Baltimore, MD: Williams & Wilkins, 2003. Hill, 2006.

270 (c) ketabton.com: The Digital Library

Subject List: Practice Test 1

Question Number and Subject 23. Obstetrics-gynecology 24. Preventure Medicine 1. Psychiatry 25. Obstetrics-gynecology 2. Psychiatry 26. Psychiatry 3. Psychiatry 27. Internal Medicine 4. Obstetrics-gynecology 28. Internal Medicine 5. Internal Medicine 29. Internal Medicine 6. Internal Medicine 30. Obstetrics-gynecology 7. Obstetrics-gynecology 31. Obstetrics-gynecology 8. Internal Medicine 32. Obstetrics-gynecology 9. Surgery 33. Surgery 10. Surgery 34. Psychiatry 11. Surgery 35. Preventive Medicine 12. Pediatrics 36. Surgery 13. Pediatrics 37. Preventive Medicine 14. Psychiatry 38. Obstetrics-gynecology 15. Psychiatry 39. Pediatrics 16. Psychiatry 40. Preventive Medicine 17. Pediatrics 41. Pediatrics 18. Surgery 42. Obstetrics-gynecology 19. Obstetrics-gynecology 43. Obstetrics-gynecology 20. Internal Medicine 44. Internal Medicine 21. Pediatrics 45. Pediatrics 22. Pediatrics 46. Surgery

271 (c) ketabton.com: The Digital Library

This page intentionally left blank (c) ketabton.com: The Digital Library

CHAPTER 8 Practice Test 2 Questions

1. A 2-week-old infant is brought to the office for 3. You are seeing a 2-week-old with trisomy 21 in her first checkup. Her birth weight was 7 lb 3 the office. The parents are asking for which oz. Her weight is now 7 lb 4 oz. The mother is medical problems their child will need to be breast-feeding and is concerned that she does evaluated or screened. You recall that children not have enough milk. Which of the following with trisomy 21 are at risk for all of the follow- items would best demonstrate she is producing ing conditions except which one? enough milk? (A) acute myelogenous leukemia (A) The infant is feeding eagerly four times (B) atlantoaxial instability per day. (C) coarctation of the aorta (B) The infant is nursing for 2 minutes each (D) early onset dementia feeding. (E) hypothyroidism (C) The infant has three wet diapers per day. (D) The baby has regained birth weight by 4. Meningitis caused by type B H. influenzae is 2 weeks of life. prevalent in a developing country. You advo- (E) The mother is not experiencing milk let cate for health policy development to address down. this issue. Which of the following would be the most effective policy in reducing the 2. A 35-year-old morbidly obese woman with number of cases? newly diagnosed type 2 diabetes is interested (A) voluntary vaccination of children prior in gastric bypass to help her lose weight. Which to school entry with Hib vaccine one of these factors would preclude her from being considered a good candidate for surgery? (B) mandated reporting of cases of meningi- tis caused by type B H. influenzae to (A) body mass index (BMI) of 38 local health authorities (B) hypertension (HTN) (C) mandated vaccination of all children (C) age prior to entry into day care or school (D) diabetes with Hib vaccine (E) inability to follow a diet (D) active surveillance of cases of meningitis caused by type B H. influenzae (E) passive surveillance of cases of meningi- tis caused by type B H. influenzae

273

Copyright © 2008 by The McGraw-Hill Companies, Inc. Click here for terms of use. (c) ketabton.com: The Digital Library

274 8: Practice Test 2

5. A 72-year-old male with a history of coronary (C) The stillbirth rate is significantly artery disease, HTN, diabetes mellitus (DM), increased. and renal insufficiency presents to the emer- (D) Evidence of congenital syphilis is usually gency room complaining of right arm and leg present at birth. weakness which started an hour prior to pres- (E) The false-negative rate of serologic tests entation, but has resolved by the time he is for syphilis is increased in human examined. On examination, the patient is neu- immunodeficiency virus (HIV)-infected rologically intact. What is the most appropriate women. next study? (A) head computed tomography (CT) 8. A 1-year-old girl stopped breathing after catch- (B) head magnetic resonance imaging (MRI) ing her hand in the bathroom door. The parents report she accidentally closed the door on her (C) chest x-ray (CXR) fingers, and then became pale, limp, fell to the (D) carotid duplex ultrasound ground, and then began breathing. She cried (E) carotid angiogram for a few minutes afterward, the parents placed ice on her hand and brought her to the office. 6. A 53-year-old diabetic man who is dialysis The child has a minor contusion of her third dependent presents to his physician complain- and fourth digits on the right hand. Her exam- ing of some redness on his foot that began after ination is otherwise normal. Your most likely he cut his toenails. On physical examination, cause for the episode is which of the following? the patient has a small wound on the great toe with surrounding cellulitic changes. His toe is (A) head trauma secondary to child abuse tender to touch and a small amount of cloudy (B) generalized seizure drainage is expressed from the wound. He has (C) breath holding spell no palpable pedal pulses, but his foot is warm (D) hypoglycemia and swollen. The most appropriate therapy (E) syncope due to cardiac dysrhythmia includes which of the following? (A) Prescribe oral ciprofloxacin and see 9. A 52-year-old woman presents with a calcium patient in 2 weeks. level of 11.5 mg/dL found on routine labora- (B) Give vancomycin with his dialysis until tory tests done at her yearly physical examina- cellulitis resolves. tion. She has no other symptoms, and denies any recent dietary changes. Her physical exam- (C) Prescribe oral lanazolid, tell patient to ination is unremarkable. The most appropri- elevate foot, and see patient in 1 week. ate next step is which of the following? (D) Admit to the hospital for IV antibiotics and surgical drainage. (A) Repeat her calcium level next year at (E) Admit patient for toe amputation. her annual physical. (B) Reassure patient that a calcium level of 7. An infant is born to a mother treated late in 11.5 mg/dL is normal. pregnancy for primary syphilis. After birth, the (C) Repeat her calcium level and obtain a infant is evaluated for evidence of congenital parathyroid hormone (PTH) level. syphilis. Which of the following statements is (D) Obtain a CT scan of her neck. true? (E) Obtain positron emission tomography (A) Mean infant birth weight is increased (PET) scan to look for bony metastases. because of fetal hydrops. (B) Congenital syphilis is more common 10. A 25-year-old man is hospitalized after weeks when the mother had late syphilis of worsening psychosis. He is begun on the versus primary syphilis. antipsychotic drug olanzapine. Five days later, he develops a fever of 39.8°C. In addition, he (c) ketabton.com: The Digital Library

Questions: 5–16 275

has become delirious, is lying stiffly in his bed, 13. You are seeing a 6-month-old child that has and is uncommunicative. His family reports been exclusively breast-fed. The child also eats he had been physically well at the time of rice cereal and pureed fruits. The mother wants admission. The admission physical examina- to know if the child needs supplemental vita- tion confirmed this. Which of the following is mins. You recommend which of the following? of most immediate concern at this point? (A) vitamin D (A) worsening psychotic state (B) iron (B) pseudoparkinsonism reaction (C) calcium (C) ruptured cerebral aneurysm (D) vitamin C (D) unsuspected opioid dependence (E) no additional supplements are needed (E) neuroleptic malignant syndrome (NMS) DIRECTIONS (Questions 14 through 16): Each set Questions 11 and 12 of matching questions in this section consists of a list of lettered options followed by several num- A 23-year-old male presents to the emergency bered items. For each question, select the ONE department after sustaining a single gunshot to the best lettered answer that is most closely associated right chest. He is awake and alert and complaining with it. Each lettered answer may be selected once, of significant abdominal pain and chest pain. He more than once, or not at all. has a pulse of 110, blood pressure (BP) of 90/60, respirations of 24. On examination, his breath sounds are decreased on the right. His chest is dull Questions 14 through 16 to percussion on the right. His abdomen is slightly distended and diffusely tender. His pulse oximeter For each situation below, select the category of pre- shows 88% on 6 L oxygen by face mask. vention it represents. (A) primary prevention 11. What is the next appropriate step in this (B) secondary prevention patient’s management? (C) tertiary prevention (A) immediate intubation (D) selective prevention (B) pericardiocentesis (E) community prevention (C) resuscitative thoracotomy (F) preventive therapy (D) placement of two large-bore IVs (E) placement of right chest tube 14. A community hospital conducts a screening for hypercholesterolemia by setting up stations and 12. The patient is now responding to his resusci- testing blood samples. Persons with elevated tation and has a pulse of 105, BP of 110/65, res- cholesterol are referred for dietary counseling, pirations of 18, and a pulse oximeter of 94%. exercise programs, and possible treatment. Which of the following is the most appropriate next step? 15. State officials conduct a program providing public service messages outlining the dangers (A) CT scan of the abdomen of smoking and urging adolescents not to start. (B) FAST (focused abdominal sonography The program is a success, in that surveys indi- for trauma) examination of the abdomen cate fewer adolescents begin smoking. (C) exploratory laparotomy (D) local wound exploration 16. A patient has severe respiratory disease as a (E) diagnostic peritoneal lavage result of years of smoking. He successfully quits smoking and improves his respiratory function. (c) ketabton.com: The Digital Library

276 8: Practice Test 2

DIRECTIONS (Questions 17 through 21): For each 20. A 2-month-old infant who is growing and item, select the ONE best lettered option that is most developing normally is noted to have a closely associated with it. Each lettered heading murmur at a checkup. The physical examina- may be selected once, more than once, or not at all. tion is entirely normal, except for a 2/6 sys- tolic ejection murmur heard over the entire For each of the following patients with possible con- chest, especially in the axilla and back. genital heart disease, select the most likely diagnosis. (A) aortic stenosis 21. An 18-month-old child who is growing and (B) atrial septal defect developing normally has a murmur noted at a (C) atrioventricular (AV) canal defect checkup. The physical examination is entirely normal, except for a 2/6 holosystolic murmur (D) carotid bruit at the left sternal border. (E) coarctation of the aorta (F) mitral stenosis 22. A 30-year-old male patient presents to your office (G) mitral valve prolapse as a new patient. His past medical history is sig- (H) peripheral pulmonic stenosis nificant only for having a splenectomy 5 years (I) pulmonic stenosis ago after a motor vehicle accident. What vacci- (J) Still’s murmur nations should you make sure that the patient has received based on his past medical history? (K) transposition of the great arteries (L) tetralogy of Fallot (A) hepatitis A vaccine series (M) truncus arteriosus (B) hepatitis B vaccine series (N) venous hum (C) tetanus vaccine (O) ventricular septal defect (D) meningococcal and pneumococcal vaccinations 17. A newborn full-term infant develops cyanosis (E) measles, mumps, and rubella vaccination shortly after birth. On examination, the infant is in no distress, is well perfused, has a normal 23. A 55-year-old woman is referred for psychi- cardiac examination, including no obvious atric consultation for depression. She is 60 days murmurs, and has no organomegaly. The pulse post bone marrow transplant. Her husband oximeter is 77%. The electrocardiogram (ECG) says that she “just cries,” barely says anything, is normal, and CXR reveals a normal heart size sleeps very restlessly if at all, and has lost inter- with a narrow mediastinum and normal pul- est in him and their children, as well as in her monary vascularity. housework and hobbies. The oncologist who referred her wants you to “give her something 18. A 4-year-old boy with normal growth, devel- for her depression.” Which of the following is opment, and physical activity is noted to have the next appropriate step? a murmur at a checkup. On examination, he (A) prescribe sertraline has no abnormal findings except for a 2/6 (B) prescribe lorazepam musical systolic ejection murmur at the lower sternal border, which is softer when he sits up. (C) interview the woman (D) have a family conference 19. A 5-year-old boy with normal growth, devel- (E) electroconvulsive therapy (ECT) opment, and physical activity is in the clinic for a checkup. His physical examination is Questions 24 through 27 totally normal, except for a widely split second A 37-year-old woman is being evaluated for possible heart sound and a 2/6 systolic ejection murmur multiple sclerosis (MS). She began noting paresthe- at the upper left sternal border that does not sias in both legs intermittently several months ago radiate. and now has symptoms of urinary incontinence. (c) ketabton.com: The Digital Library

Questions: 17–30 277

24. Which of the following physical findings is and have been greater than 200 for his entire characteristic of MS? hospital stay. You are called to the bedside to evaluate a large amount of salmon colored (A) memory loss fluid that has spontaneously drained from his (B) constipation midline wound. What is the next appropriate (C) internuclear ophthalmoplegia step in this patient’s management? (D) intention tremor (A) open wound at the bedside (E) bilateral upgoing toes (B) ultrasound-guided drainage of fluid collection 25. Which of the following is the best initial step in establishing the diagnosis? (C) CT scan of the abdomen (D) immediate esophagogastroduo- (A) brain CT scan denoscopy (EGD) (B) brain MRI (E) immediate exploration of wound in the (C) electroencephalogram (EEG) operating room (OR) (D) visual evoked potentials (E) auditory evoked potentials Questions 29 through 31 A 65-year-old male has been admitted for a colonic 26. Which of the following is true about MS? obstruction. His workup has demonstrated an (A) Males are affected twice as often as obstructing colon cancer in the sigmoid colon. After females. a full preoperative evaluation, he undergoes a sig- (B) Typical age of onset is in the fifth or moid colon resection with primary anastomosis. On sixth decade of life. postoperative day 4, he is noted to have significant (C) Eighty percent of patients have the pain and swelling in his left lower extremity. relapsing-remitting type MS. 29. What is the next appropriate step in this (D) Fifty percent of patients have primary patient’s management? progressive MS. (E) Optic neuritis typically is bilateral. (A) MRI of the left leg (B) duplex scan of the left leg 27. Which of the following medications is useful to (C) CT scan of the chest hasten clinical recovery with an acute relapse (D) transthoracic echocardiography of MS? (E) plain x-ray of the left leg (A) interferon beta-1b (B) cyclosporin 30. Which of the following would be the most (C) methotrexate appropriate treatment if the suspected diagnosis is confirmed? (D) corticosteroids (E) penicillins (A) ASA (aspirin) 325 mg PR daily (B) ibuprofen 600 mg PO tid 28. A 65-year-old diabetic male underwent abdom- (C) amiodarone 600 mg IV daily inal exploration for a bleeding duodenal ulcer. (D) enoxaparin 1 mg/kg SQ bid On postoperative day 3, he feels well and is (E) heparin 5000 units SQ tid hungry. He is afebrile and has a pulse of 90. His blood glucoses have been poorly controlled (c) ketabton.com: The Digital Library

278 8: Practice Test 2

31. Three days after beginning appropriate ther- (D) The risk of placenta previa is increased apy, the patient now complains of significant in this woman. chest pain and shortness of breath. His pulse (E) Perinatal mortality is increased slightly oximeter is 85% and his BP is 120/60. Of the in hypertensive pregnant women. following, which is the most appropriate next step in his management? Questions 35 through 37

(A) immediate cardioversion A 7-year-old girl is brought to your office by her (B) immediate laparotomy mother because she is having problems in school (C) CT scan of the chest and at home. She is having temper tantrums, and is (D) MRI of the chest frequently oppositional. Her teachers noticed she (E) duplex scan of the lower extremities was behind the other children last year and placed her in special education classes. Neuropsychiatric 32. Which of the following is a side effect of testing confirmed a below average intelligence quo- methotrexate? tient (IQ) with a full scale score of 66.

(A) congestive heart failure 35. Which of the following syndromes/disorders is (B) peripheral neuropathy unlikely to be the cause of this child’s mental (C) bone marrow depression retardation? (D) hemorrhagic cystitis (A) Down syndrome (E) conduction hearing loss (B) fragile X syndrome (C) fetal alcohol syndrome 33. A patient had an uncomplicated cesarean deliv- (D) Turner’s syndrome ery 6 days previously. She has been on a regi- men of ampicillin, gentamicin, and clindamycin (E) cri-du-chat syndrome for 5 days for postpartum fevers, but still has temperature spikes of 39.4°C (103°F). Physical 36. You obtain further historical information from examination is normal. Which of the following her mother. She was hypotonic at birth. She is the most likely cause of her fever? has simian creases on her hands and is short is stature. You notice she has slanted palpebral (A) pelvic abscess fissures and a flat wide nasal bridge. She had a (B) septic pelvic thrombophlebitis congenital heart defect repaired when she was (C) 3 years old. What is the most likely cause of her (D) pyelonephritis condition? (E) breast engorgement (A) Down syndrome (B) fragile X syndrome 34. A 19-year-old primipara at 38 weeks’ gesta- (C) fetal alcohol syndrome tion has a BP of 150/106 mmHg, hyperreflexia, (D) Klinefelter’s syndrome and 3+ proteinuria. Which of the following statements is correct? (E) cri-du-chat syndrome

(A) The risk of eclampsia increases with 37. Which of the following is the most widely increasing systolic BP. accepted and useful test for the assessment of (B) Delivery of the infant is the treatment intelligence in children? for this woman. (A) Stanford-Binet Test (C) A beta-blocker is contraindicated in (B) Metropolitan Achievement Test pregnancy. (C) Wechsler Intelligence Scale for Children (WISC) (c) ketabton.com: The Digital Library

Questions: 31–41 279

(D) Bellevue-Wechsler Scale (D) IV fluids, admittance to the hospital, (E) Vineland Social Maturity Scale discontinuation of cephalosporin, and institution of metronidazole Questions 38 and 39 (E) IV fluids in the emergency department and discharge to home on an oral A 68-year-old man comes to the emergency depart- cephalosporin with an appointment to ment with complaints of watery diarrhea, crampy return for an air contrast barium enema abdominal pain with distention, and low-grade fever of 3 days’ duration. He underwent coronary Questions 40 and 41 artery bypass surgery 10 days previously and was discharged on postoperative day 5 on an oral first- A 22-year-old man is brought to the emergency room generation cephalosporin for wound cellulitis that of a general hospital by police. They report that he has since resolved. Physical examination reveals a disturbed services at a church by walking in and temperature of 100°F, pulse rate 95/min, BP 110/60 announcing that he was Jesus Christ. The patient’s mmHg, respiratory rate 12/min, and a mildly dis- mother comes to the emergency room soon after the tended abdomen with mild left lower quadrant patient’s arrival and reports that her son had an pain with guarding, no rebound tenderness, and episode of severe depression 2 years ago, but that he heme-positive stool on rectal examination. Pertinent has never exhibited symptoms like his current ones. laboratory tests reveal an elevated blood urea nitrogen When the patient is examined, he talks rapidly and (BUN) and creatinine, and white blood count incessantly, insisting that he is the Messiah. (WBC) of 25,000. 40. Which of the following is this patient’s most 38. Which of the following tests is most likely to likely diagnosis? yield a definitive diagnosis? (A) bipolar I disorder (A) chest and abdominal x-rays (B) bipolar II disorder (B) stool for ova and parasites (C) paranoid schizophrenia (C) stool for Clostridium difficile toxins (D) hyperthyroidism (D) stool for enterotoxigenic Escherichia coli (E) hypothyroidism (E) CT scan of the abdomen 41. Which of the following medications is most 39. Which of the following is the most appropriate likely to be effective in the treatment of this next step in management of this patient? patient? (A) intravenous (IV) fluids in the emergency (A) valproic acid department and discharge to home on (B) fluoxetine an oral cephalosporin with instructions (C) clonazepam to return for increasing symptoms or (D) bupropion fever greater than 101.5°F (E) lamotrigine (B) IV fluids, admittance to the hospital, and institution of third-generation cephalosporin antibiotics (C) IV fluids, admittance to the hospital, discontinuation of cephalosporin, and colonoscopy (c) ketabton.com: The Digital Library

280 8: Practice Test 2

42. You are seeing a 68-year-old male patient in your his stomach. An abdominal film is ordered and office who has just been found to have a is shown in Figure 8-1. Which of the following mesothelioma. He worked for many years in the is the most likely diagnosis for this infant? shipbuilding industry and had extended peri- ods of unprotected asbestos exposure. He also has a significant history of tobacco use and has smoked about a pack a day for the past 50 years. He asks you if his mesothelioma was caused by his smoking or his asbestos exposure. Which of the following is the best description of what is known about the association between smoking, asbestos exposure, and mesotheliomas? (A) Smoking and asbestos exposure are synergistic—together they significantly increase the risk of mesothelioma. (B) Smoking can increase one’s risk of mesothelioma regardless of asbestos exposure. (C) Asbestos exposure increases the risk of mesothelioma and there is no increased risk with smoking. (D) Smoking is slightly protective against the damaging effects of asbestos. (E) The amount of asbestos exposure is FIG. 8-1 needed to determine a patient’s risk of developing mesothelioma as there is a (A) adrenogenital syndrome dose-response relationship with (B) pyloric stenosis asbestos and the development of mesothelioma. (C) duodenal atresia (D) esophageal atresia with distal 43. A 36-year-old alcoholic patient has cirrhosis tracheoesophageal atresia and pancreatic insufficiency due to recurrent (E) cystic fibrosis pancreatitis. He complains of night blindness, decreased ability to taste food, and dry skin 45. A 78-year-old female presents to your office with hyperpigmentation. These complaints with a 2-year history of a cognitive decline suggest deficiency of which of the following? which includes an impairment in short-term memory. Her husband brought her in because (A) copper he is very worried about her. This past week (B) zinc she became lost coming home from the neigh- (C) selenium borhood grocery store. Which of the following (D) chromium is the most likely diagnosis? (E) manganese (A) vascular dementia (B) alcohol-related dementia 44. A 1-day-old infant has persistent vomiting. He has not had any stools. He was born at (C) Alzheimer’s dementia term and has not had any respiratory diffi- (D) delirium NOS (not otherwise specified) culties. A nasogastric tube is easily placed into (E) Pick’s disease (c) ketabton.com: The Digital Library

Questions: 42–46 281

46. After a 12-hour first stage, a 45-minute second (C) The first stage of labor is prolonged in stage, and a 10-minute third stage of labor, a this patient. healthy primigravid woman delivers a 7.9-lb (D) The second stage begins with complete infant male with Apgar scores of 9 at 1 minute cervical dilatation and ends with the and 10 at 5 minutes. Which of the following delivery of the placenta. statements about labor is correct? (E) The third stage of labor begins after (A) Labor is defined by the presence of reg- delivery of the placenta and ends when ular uterine contractions at an interval the episiotomy is repaired. of 5 minutes or less. (B) The first stage begins with the onset of regular uterine contractions and ends with full dilatation of the cervix. (c) ketabton.com: The Digital Library

Answers and Explanations

1. (D) Breast-feeding can be anxiety-provoking entire population at risk and may allow for the for mothers. The adequacy of feeding is a continued spread of disease. Mandated report- common question. Breast-fed infants will gen- ing and active surveillance will help to inform erally eat every 2–4 hours and nurse for 5–10 health officials regarding changes in the trends minutes. Mothers should begin to feel milk let of disease, but will not directly help in con- down and if this is not occurring investigation taining or managing disease. (Wallace and into adequacy of supply should be undertaken. Doebbeling, pp. 12–13, 116–117) Infants who are well hydrated should have 6–8 wet diapers per day. The best indication of ade- 5. (D) This patient has suffered from a transient quate supply here is the demonstrated weight ischemic attack (TIA). The most common cause gain. (Behrman, Kliegman, and Jenson, pp. 158–161) of TIA is carotid atherosclerotic disease, and the most appropriate first step in diagnosing 2. (E) Bariatric surgery is becoming more and more this is a carotid duplex scan. Carotid athero- common. The preoprerative selection of patients sclerosis can be quantified by duplex ultra- eligible for this type of surgery is critical. sound, and carotid endarterectomy can be done Requirements for eligibility include a BMI >35 without further imaging in most cases. A head with comorbid conditions such as HTN or dia- CT or MRI would be helpful if the patient still betes, BMI >40 without comorbid conditions, had residual symptoms, but would not likely psychiatrically stable, motivated, ability to be useful in this case. A carotid angiogram is understand the operation, and failed attempts at not generally necessary and should not be the dieting. However, the patient needs to be able to first diagnostic test. (Brunicardi et al., pp.785–787; follow a strict eating plan after the surgery in Townsend et al., pp. 1942–1945) order to be successful at weight loss. (Brunicardi et al., pp. 997–1000; Townsend et al., pp. 357–361) 6. (D) This patient has an infected wound on his great toe. He has compromised ability to fight 3. (C) Children with trisomy 21 (Down syn- infection because his diabetes decreases the drome) are at greater risk than the general pop- ability of his WBCs to phagocytose bacteria. ulation for development of numerous health Therefore, the patient needs to be admitted to problems throughout life. Hypothyroidism and the hospital for close observation. The pres- congenital heart disease are more common ence of cloudy drainage from a small wound than the others. The heart disease is typically an may indicate a deeper infection, and the AV canal. Atlantoaxial instability can result in wound should be explored in the operating spinal cord injury with minor neck trauma. room. Much of the time, if the infection is This condition needs to be ruled out prior to caught early, the toe can be salvaged, as long as participation in competitive sporting activities. the joint is not involved. Most diabetic foot (Rudolph, pp. 732–733) infections have mixed bacteria present, there- fore broad-spectrum antibiotics that cover both 4. (C) Infection with hemophilus type B is com- gram-positive, gram–negative, and anaerobic municable and may cause meningitis. The most bacteria are necessary until culture results are effective measure that can be employed to available. (Greenfield et al., p. 91–93) reduce the incidence of these infections is the use of mandated universal vaccination. 7. (C) Reported rates of stillbirth in mothers with Voluntary vaccination will not capture the syphilis were 14–34%, a dramatic increase as

282 (c) ketabton.com: The Digital Library

Answers: 1–12 283

compared to the 0.7% in the general popula- Symptoms typically include high fever, delir- tion. Mean birth weights are decreased signif- ium, and rigidity, and they usually develop icantly in infected mothers. This is the result of within a week of starting a new drug or raising both preterm labor and intrauterine growth the dosage of the currently prescribed drug. restriction. Women with primary or secondary Once NMS is suspected, the antipsychotic drug syphilis are more likely to transmit infection should be stopped and emergency supportive to their fetus than are those with late or latent measures begun. Mortality from NMS has been syphilis. Most infants with early congenital reported to be as high as 20%. (Kaplan and Sadock, syphilis do not develop evidence of active dis- pp. 993–994) ease for 10–14 days after birth. Despite the pos- sibility of immune suppression in HIV-infected 11. (E) women, serologic tests for syphilis are accu- rate in mother and infant. (Cunningham et al., pp. 12. (C) 1302–1305) Explanations 11 and 12 8. (C) Breath holding spells can present as both cyanotic and pallid form. In both of these The initial evaluation of the trauma patient con- instances they occur in children between 2 and sists of a rapid primary survey, aimed at identi- 5 years of age. The pallid form is usually initi- fying and treating immediately life-threatening ated by a painful experience. There is no asso- problems. The primary survey is conducted ciated EEG abnormality or cardiac dysrhythmia according to the mnemonic ABCDE: Airway, associated with these spells. The parents Breathing, Circulation, Disability, and Exposure. require reassurance that this is not life threat- Once the airway has been assessed and is found ening and the child will outgrow this tendency. to be patent, breathing should be assessed. The In this child there is no evidence of head injury physical examination of the chest should iden- and the injury is consistent with the story and tify life-threatening situations that require immediate attention such as a tension pneu- child abuse is very unlikely. (Behrman, Kliegman, and Jenson, p. 2010) mothorax, massive hemothorax, open pneu- mothorax, flail chest, and cardiac tamponade. 9. (C) This patient likely suffers from asympto- Tube thoracostomy is the most common matic primary hyperparathyroidism. The procedure performed in the management of symptoms of hyperparathyroidism are often thoracic trauma. In fact, 85% of the patients subtle, and include muscle weakness, anorexia, sustaining chest injuries will require only clin- nausea, constipation, polyuria, polydipsia, bony ical observation or tube thoracostomy. pain, and renal calculi (stones, bones, groans, An emergent thoracotomy is indicated moans). Often patients don’t realize they have after chest trauma in the following situations: symptoms until specifically questioned. The (1) cardiac arrest (resuscitative thoracotomy), best first step in the workup of hyperparathy- (2) massive hemothorax (greater than 1500 roidism after a careful history and physical is to mL of blood through the chest tube acutely or repeat the calcium level, and check the parathy- greater than 200–300 mL/h after initial roid hormone level. If both are elevated, then the drainage), (3) penetrating injuries of the ante- patient likely has hyperparathyroidism. These rior chest with cardiac tamponade, (4) large patients often also have low serum phosphate open wounds of the thoracic cage, (5) major levels as well as a hyperchloremic metabolic aci- thoracic vascular injuries in the presence of dosis due to the effect of PTH on the kidney’s hemodynamic instability, (6) major tracheo- bronchial injuries, and (7) evidence of excretion of bicarbonate. (Brunicardi et al., pp. 1434–1438; Townsend et al., pp. 987–989) esophageal perforation. Patients who present with thoracoabdom- 10. (E) NMS is a rare, potentially fatal complication inal gunshot wounds require assessment and of the use of antipsychotic medication. management of both the chest and abdomen. (c) ketabton.com: The Digital Library

284 8: Practice Test 2

Based on the high frequency of intra-abdominal 17. (K) organ injury after gunshot wounds, mandatory abdominal exploration remains the standard 18. (J) form of management. Patients who present with evidence of peritonitis require an immedi- 19. (B) ate laparotomy. (Townsend et al., pp. 490–492, 505–506, 508–509, 512) 20. (H)

13. (A) Breast-feeding supplies all required nutri- 21. (O) ents over the first few months of life. However vitamin D stores may be inadequate or depleted Explanations 17 through 21 in mothers and therefore supplementation in breast-fed infant is required to prevent rickets. Transposition of the great arteries is the most The American Academy of Pediatrics (AAP) common cause of cyanotic heart disease in the currently recommends all breast-fed infants neonate. Cyanosis usually appears in the first receive vitamin D supplementation. Vitamin D day of life. Cardiac findings are often otherwise is produced with exposure to sunlight. Due to normal, although patients may have a single S . The classic CXR shows a normal heart size concerns about long-term problems with skin 2 cancer, infants may be receiving inadequate with a narrow mediastinum (egg on a string). exposure to produce enough vitamin D and Other causes of cyanotic congenital heart dis- therefore require supplementation. Formula- ease are tetralogy of Fallot and truncus arterio- fed infant do not require vitamin D supple- sus. The CXR in truncus arteriosus shows increased pulmonary flow and in tetralogy of mentation. (AAP 111(4):908–910) Fallot decreased pulmonary flow. Murmurs are 14. (B) This activity is secondary prevention since commonly heard in children with normal it identifies individuals with a risk factor for a hearts. This is particularly true in young chil- disease and attempts to prevent the disease. It dren with relatively thin chest walls where the is important to note that the disease itself is not normal flow murmurs arising from the pul- yet present, only the risk factor for its develop- monary and aortic roots are easily heard on the ment. In this case, the risk factor for develop- anterior chest. To label a murmur as normal, ment of atherosclerotic vessel disease was high patients should be free of any cardiac symp- cholesterol, and the subjects of the intervention toms. The rest of the cardiovascular examina- tion should also be normal. One of the typical did not yet have any symptoms. (USPSTF) flow murmurs in young children is the Still’s 15. (A) Primary prevention interventions prevent murmur. The murmur is musical or vibratory. It the development of risk factors or of disease. is a systolic ejection murmur and is usually For instance, a program that reduces consump- heard best at the left lower sternal border. Atrial tion of saturated fats in order to reduce hyper- septal defect is one of the common congenital cholesterolemia (risk factor), and thus the risk of heart lesions with a left-to-right shunt. Patients developing coronary artery disease (disease) are usually asymptomatic, and the problem is and a myocardial infarction (MI) (disease) is usually diagnosed when the murmur is detected. Murmurs are due to increased flow primary prevention. (Lang and Hensrud, pp. 3–4) across normal valves. The systolic murmur is 16. (C) Tertiary prevention is the correction of a dis- due to increased flow across the right ventricu- eased state and prevention of complications that lar outflow tract. The widely split second heart result from that state. In this case, the patient sound is due to the increased right ventricular already had developed disease from smoking. diastolic volume and a prolonged ejection time Cessation of tobacco use is simply treating the across the pulmonary valve. Peripheral pul- disease. In this sense, most medical therapy is monic stenosis is an ejection murmur heard best at the upper sternal border with transmission to tertiary prevention. (Lang and Hensrud, pp. 3–4) the axilla and the back. It is caused by the sharp (c) ketabton.com: The Digital Library

Answers: 13–28 285

angle between the main pulmonary artery and Explanations 24 through 27 its branches. It is heard in infants only and should disappear by 6 months of age. Bilateral internuclear ophthalmoplegia is very Ventricular septal defects are the most common characteristic of MS. Careful physical examina- congenital heart lesions. The most important tion reveals evidence of multiple lesions, such characteristic about the murmur is that it begins as afferent pupillary defect and bilateral upgo- ing toes. Memory loss may occur later in the early in systole, often obscuring S1. Depending on the size of the defect, the murmur will be course of illness. Autonomic injury may pro- longer or shorter. AV canal detected after the duce constipation, but this is not specific for newborn period can present with a similar MS. Intention tremor is a manifestation of cere- sounding murmur. It is much less common than bellar involvement, but again not specific for a ventricular septal defect but would be a strong MS. Periventricular plaques on brain MRI are possibility if the patient had Down syndrome. characteristic and found in over 90% of patients (Kliegman et al., pp. 66–67, 136–138) with known MS. MRI is much more sensitive than brain CT scan and should be the first step. 22. (D) Patients who are asplenic (either from sur- Cerebrospinal fluid (CSF) is abnormal in 95% of gical removal or functional asplenia from MS patients. Increases in IgG and oligoclonal chronic medical conditions) are at higher risk of IgG bands are specific and suggest increased developing infections from encapsulated risk of disseminated disease and should also be organisms such as Neisseria meningitidis and done. Visual and auditory evoked potentials are Streptococcus pneumoniae species. In addition to abnormal in demyelinated tracts and serve as routine vaccinations appropriate for all additional supporting evidence of MS. EEG is patients, patients who are asplenic should not helpful. Relapsing-remitting MS is the type receive meningococcal and pneumococcal vac- present in 80% of patients; primary progressive cinations. If an elective splenectomy is planned, MS affects approximately 20%. Symptoms typ- pneumococcal vaccination should be given ically begin in the second or third decade of life. MS affects females approximately twice as often 2 weeks prior to surgery. (Recommended Adult Immunization Schedule. CDC web site: http://www.cdc. as males. Optic neuritis typically is unilateral. gov/nip/recs/adult-schedule-11x17.pdf) Corticosteroids often hasten clinical recovery during acute relapses of MS. Interferons and 23. (C) Interviewing the patient is critical to deter- immune globulin are medications useful to mining her needs. Ideally, this would be with reduce the rate of clinical relapse. There is no her alone. Patients frequently will provide clear evidence that cyclosporin or methotrexate important information about themselves. They are effective. (Kaspar et al., pp. 2461–2469) may not if a family member is present. All other options listed are premature. Prescribing sertra- 28. (E) Wound dehiscence is a partial or total dis- line, lorazepam, or ECT require a thorough eval- ruption of any or all of an operative wound. uation of the patient’s history, symptoms, Dehiscence is rare in young healthy patients, psychosocial situation, and so forth before deter- but in those greater than 60 years of age, the mining a course of action. A family conference incidence is about 5%. Risk factors include DM, may be of real value, but should follow interview uremia, immunosuppression, jaundice, sepsis, hypoalbuminemia, cancer, obesity, and those of the patient. (Kaplan and Sadock, pp. 229–233) receiving corticosteroids. The three prevailing 24. (C) local risks for dehiscence are inadequate clo- sure, increased intra-abdominal pressure, and 25. (B) impaired wound healing. When layers of the abdominal closure dehisce, then viscera may 26. (C) protrude through the wound which is called evisceration. Most wounds dehisce or eviscer- 27. (D) ate between postoperative days 5 and 8 when the wounds are at their weakest. The classic (c) ketabton.com: The Digital Library

286 8: Practice Test 2

first sign of a dehiscence is the drainage of a include megaloblastic anemia, diarrhea, stom- large amount of salmon-colored fluid from the atitis, vomiting, vasculitis, and pulmonary fibro- wound. Wound dehiscence is best managed by sis. Myocardial damage leading to heart failure, prompt reclosure of the incision. Further open- peripheral neuropathy, hemorrhagic cystitis, and ing the wound at the bedside could lead to hearing loss are not side effects of methotrexate, evisceration which has a higher morbidity and but may occur with other chemotherapeutic mortality associated with it. (Way, pp. 22–23) agents. (Kasper et al., p. 471, 2013)

29. (B) 33. (B) Septic pelvic thrombophlebitis should be suspected in women with postpartum fever that 30. (D) fails to respond to broad-spectrum antibiotics, and whose physical examination is normal. CT 31. (C) or MRI of the pelvis may disclose pelvic vein phlebitis. The clinical impression that anticoag- Explanations 29 through 31 ulant doses of heparin result in rapid deferves- cence is not supported by convincing clinical Deep venous thrombosis (DVT) is a common data. Infectious causes of postpartum fever usu- complication noted in surgical patients. ally respond to broad- spectrum antibiotics Virchow’s triad including vascular injury, hyper- within 48–72 hours. A pelvic abscess is usually coagulability, and venous stasis are the basis for easy to palpate on bimanual examination and the pathophysiology of the disease. Most cases may be anterior or posterior to the uterus. are multifactorial, but risk factors include Subinvolution of the uterus, increased uterine trauma, older age, cancer, immobilization, and tenderness, and increased postpartum bleeding antiphospholipid antibodies. Diagnosis cannot with a foul lochia are the signs of postpartum be made solely on clinical signs and symptoms. endometritis. Pyelonephritis should also have Unilateral leg swelling associated with com- responded to antibiotics within 48–72 hours. plaints of a dull ache or pain in the calf is the Breast engorgement may cause fever within most common presentation. Low-grade fever 24–48 hours postpartum, but the fever is usually and tachycardia may also occur. The test of less severe and resolves without antibiotics. choice is duplex ultrasound of the venous system (Cunningham et al., pp. 718–719) to evaluate for presence of thrombosis indicated by venous dilation and incompressibility during 34. (B) The triad of HTN, hyperreflexia, and pro- probe pressure. Treatment for DVT is aimed at teinuria in pregnancy establishes the diagnosis decreasing the incidence of complications asso- of preeclampsia. The etiology is still unknown, ciated with DVT. Full anticoagulation with and the only accepted treatment at term preg- heparin or a low molecular weight heparin is nancy is delivery of the infant. The risk of indicated. Transition to warfarin with an over- eclampsia is not proportional to the increase in lapping time period is recommended for 3–6 systolic or diastolic pressure, and convulsions months for a first episode of uncomplicated have been reported to occur at the time of DVT. Pulmonary embolism (PE) typically pres- normal BP readings. Beta-blockers are not con- ents with sudden deterioration of pulmonary traindicated in pregnancy, although angiotensin- function 3 or more days after an inciting event converting enzyme (ACE) inhibitors are such as immobilization, trauma, or surgery. contraindicated. Beta-blocking drugs effec- Evaluation should include a CXR and PE proto- tively lower BP, and are most effectively used col chest CT if the CXR does not suggest an alter- when the pregnant woman is early in the third native diagnosis for the sudden decline in trimester and prolongation of pregnancy is pulmonary function. (Way, pp. 192, 850–852) desired. The risk of abruptio placenta, as opposed to the risk of placenta previa, is 32. (C) Bone marrow depression is the most serious increased in hypertensive pregnant women. toxic effect of methotrexate. Other toxic effects Perinatal mortality is significantly increased in (c) ketabton.com: The Digital Library

Answers: 29–39 287

pregnant women with HTN and proteinuria. In the results of an IQ examination, when corre- one study, it was increased threefold in women lated with other information from the person’s whose diastolic BPs were 105 torr or higher. history or present status, are valuable data. In (Cunningham et al., pp. 762–808) 1896, the French clinician Binet began a project to develop an objective measure by which to 35. (D) Down syndrome, fragile X syndrome, fetal quantify individual differences in mental abil- alcohol syndrome, and cri-du-chat syndrome ities (intelligence). From 1905 to 1908, Binet and all are associated with mental retardation. Simon introduced a series of standardized IQ Turner’s syndrome is not. Persons with tests in which correct responses to items that Turner’s syndrome (XO) have a female habitus, differed progressively in level of difficulty were but because of the lack of the second X chro- correlated with a child’s chronologic age. In mosome, there is an agenesis or dysgenesis of 1916, Terman, at Stanford University, revised the gonads. No secondary sexual characteris- the Binet-Simon Intelligence Scale. The tics develop without treatment. Short stature, Stanford-Binet Test was quickly adopted in the webbed neck, and low posterior hairline are United States for assessing the intelligence of common. Later medical management is neces- children. Wechsler, as chief psychologist at sary to assist them with their infertility and Bellevue Hospital in New York, introduced the absence of secondary sexual characteristics. next major development in the history of intel- (Kaplan and Sadock, pp. 733, 1161–1178) ligence testing. Before Wechsler, IQ testing was primarily used with children 15 years old or 36. (A) Down syndrome results from trisomy 21 younger. Wechsler developed scales that com- and occurs in 1/1000 live births. Clinical fea- pared the performance of a child with the per- tures include hypotonia, upward slanted palpe- formance of his or her own agemates on the bral fissures, midface depression, flat, wide same test items. The Bellevue-Wechsler Scale nasal bridge, simian creases, short stature, and was introduced in 1939. In 1949, this scale was frequent congenital heart problems. Fragile X used as the model for a revised test specifically syndrome is a recessive syndrome that results for children (ages 6–16 years), the WISC. from a mutation of the X chromosome, and Restandardization of the Bellevue-Wechsler occurs in 1/1000 male births and 1/3000 female Scale led to the introduction of the Wechsler births. It is the second most common cause for Adult Intelligence Scale (WAIS) in 1958, with mental retardation. The traditional phenotype norms for ages 16–75 years. The WISC and includes a long head and ears, short stature, WAIS are currently the most widely accepted hyperextensible joints and postpubertal macro- tests of intelligence in use today. The orchidism. Fetal alcohol syndrome results in Metropolitan Achievement Test and the mental retardation and a typical phenotype of Vineland Social Maturity Scale are contempo- hypertelorism, microcephaly, short palpebral rary standardized tests of school readiness and fissures, inner epicanthal folds, and a short grade-level achievement and acquisition of cul- turned up nose. Cri-du-chat (cat’s cry) syn- ture-based social skills, respectively. (Kaplan and drome results from a lack of chromosome #5. Sadock, pp. 1156–1160) These children are severely retarded and often have microcephaly, low set ears, micrognathia, 38. (C) and hypertelorism and make a characteristic cat-like cry caused by laryngeal abnormalities. 39. (D) (Kaplan and Sadock, pp. 1161–1178) Explanations 38 and 39 37. (C) Psychological assessment techniques are, like most tools, useful for a diversity of pur- Antibiotic-associated colitis often occurs in eld- poses. Their worth partially depends on the erly persons, after exposure to antibiotics, and is training, competence, and ethical values of the caused by enterotoxins produced by C. difficile. tester. In the hands of a competent clinician, Whereas abdominal x-rays and CT scanning can (c) ketabton.com: The Digital Library

288 8: Practice Test 2

demonstrate abnormal findings, detection of valproic acid and lithium. Lamotrigine is an toxin in the stool will yield a definitive diag- anticonvulsant that is most effective in treating nosis. The appropriate treatment of patients bipolar depression in bipolar disorder. (Kaplan with dehydration and abdominal symptoms and Sadock, pp. 570–572) would be IV fluids and admittance to the hos- pital for monitoring of their disease, discontin- 42. (C) Individuals exposed to asbestos are at risk uation of the antibiotics that precipitated the for a number of diseases, most notably asbesto- disease, and institution of metronidazole. sis, lung cancer, and mesothelioma. While smok- Barium enema should be avoided because it ing and asbestos exposure are synergistic with may precipitate complications. Whereas proc- respect to the development of lung cancer, there toscopy may be helpful, colonoscopy may pre- is no known association between smoking and cipitate perforation and should be avoided. the development of mesothelioma. There is also (Brunicardi et al., p. 1100; Townsend et al., pp. 1428–1429) no known dose-response relationship between asbestos and mesothelioma. Individuals with 40. (A) This patient seems to be suffering from a prolonged exposure to asbestos have a similar full-blown manic episode, coming after a pre- risk of mesothelioma as those with brief expo- vious depressive episode. Bipolar I disorder is sures. (Wallace and Doebbeling, pp. 468–469) characterized by at least one full-blown manic episode. Bipolar II disorder is characterized by 43. (B) Several disease states predispose to zinc periods of major depression which alternate deficiency, including cirrhosis, alcoholism, poor with hypomanic periods which do not reach nutrition, and pancreatic insufficiency. Zinc the level of psychosis seen in this patient. This deficiency can cause growth retardation, patient’s prominent mood-related symptoms hypogonadism, anorexia, impaired sense of make the diagnosis of paranoid schizophrenia taste and smell, diarrhea, alopecia, dermatitis, unlikely as an explanation for his psychotic hyperpigmentation, and poor wound healing. symptoms. Although both hypothyroidism and Night blindness due to combined zinc and hyperthyroidism can cause mood disorders, and vitamin A deficiencies improves only if both should not be ruled out in patients presenting substances are replenished. Zinc deficiency in with mood disorders, particularly when accom- cirrhosis is associated with hepatic coma. panied by other symptoms of thyroid disease Adult copper deficiency is also rare and is asso- (such as weight gain, fatigue, and cold intolerance ciated with Keshan disease, a fatal cardiomy- in hypothyroidism, and tachycardia and heat opathy occurring in children. Chromium intolerance in hyperthyroidism), they are less deficiency causes impaired growth and abnor- frequent as a cause of manic symptoms than is mal lipid, glucose, and protein metabolism. bipolar I disorder. (Kaplan and Sadock, pp. 542–544) Hypomagnesemia is associated with weight loss, dermatitis, change in hair color, and 41. (A) The anticonvulsant valproic acid is well hypocholesterolemia. (Kasper et al., pp. 409–412) established as an effective treatment for the manic phase of bipolar I disorder, and is used 44. (C) Duodenal atresia presents with vomiting on about as frequently as lithium carbonate for the first day of life. The diagnosis is suspected that purpose. Fluoxetine and bupropion are when the abdominal film shows the “double antidepressants that can be effective in the bubble” sign shown in Figure 8-1. Surgical treatment of the depressed phase of individu- repair is usually successful, and prognosis is als with bipolar I disorder, but one runs the excellent. Pyloric stenosis typically manifests risk of exacerbating a manic episode by using after the second to third week of life, and the antidepressants in such patients. Clonazepam, vomiting grows progressively worse as the a benzodiazepine, is a useful adjunctive med- pyloric muscle hypertrophies. The obstruction is ication in the treatment of bipolar I disorder, rarely present at birth. Adrenogenital syndrome but is not well established as a first-line treat- presents with vomiting at birth or shortly there- ment of mania, as are medications such as after. Electrolyte abnormalities are a major issue. (c) ketabton.com: The Digital Library

Answers: 40–46 289

Esophageal atresia is excluded by passing a HIV screening, neuroimaging, EEG, and phys- nasogastric tube into the stomach. Cystic fibro- ical examination (looking for neurologic find- sis does not present with vomiting unless meco- ings characteristic of vascular dementias) with nium ileus is present at birth. (Brunicardi et al., pp. Mini Mental Status Examination are parts of a 1487–1488; Greenfield et al., pp. 1887–1890; Townsend routine dementia workup. (Kaplan and Sadock, pp. et al., pp. 2104–2108, 2111–2112) 322, 329–344)

45. (C) Of all the patients with dementia, 50–60% 46. (B) Classically, there are three stages of labor. have the most common type of dementia— The first stage begins with the onset of regular Alzheimer’s dementia. The second most uterine contractions that cause cervical dila- common type is vascular dementia. Dementia is tion and ends with complete dilatation of the differentiated from delirium in that it involves cervix. The interval between contractions can a more chronic and insidious cognitive decline, vary from woman to woman at the onset of and the patient is usually alert and the cognitive labor. The first stage can be divided into a latent changes are more stable over time. Pick’s dis- phase and an active phase. In a primigravida, ease is another form of dementia which the length of the latent and active phases aver- involves a preponderance of frontotemporal age 8.6 and 4.9 hours, respectively. Latent labor atrophy and involves personality changes more is considered prolonged at 20 hours for nulli- frequently along with cognitive decline. parous women and 14 hours for parous A dementia workup is initiated to make women. Both times are shorter in women who sure the appropriate treatments are initiated have had a child. The second stage begins with for the form of dementia which presents. Some complete dilatation of the cervix and ends with

forms of dementia can be resolved (e.g., B12 defi- delivery of the infant. The third stage begins ciency leading to cognitive decline), and one after delivery of the infant and ends with deliv- does not want to miss such an opportunity to ery of the placenta. Subsequent events such as correct this. RPR (rapid plasma reagin, test for myometrial contractions, inspection of the syphilis), liver enzymes, thyroid-stimulating cervix and vagina, and repair of an episiotomy hormone (TSH), complete blood count (CBC), or laceration are not recognized as a stage of urinalysis (UA), renal function, blood and urine labor. (Cunningham et al., pp. 151–161) drug screens, erythrocyte sedimentation rate, (c) ketabton.com: The Digital Library

BIBLIOGRAPHY

American Academy of Pediatrics (AAP). Committee Kliegman RM, Greenbaum L, Lye P. Practical on Injury and Poison Prevention. Selection and Strategies in Pediatric Diagnosis and Therapy. using the most appropriate car safety seats for Philadelphia, PA: Elsevier, 2004. growing children: guidelines for counseling par- Lang AE, Lozano AM. Parkinson’s disease. Part 1. ents. Pediatrics 1996;97, 111(4):908–910. [Review] N Engl J Med 1998;339(15):1044–1053. Behrman RE, Kliegman RM, Jenson HB. Nelson Recommended Adult Immunization Schedule, October Textbook of Pediatrics, 17th ed. Philadelphia, PA: 2006–September 2007. CDC web site: http://www. W.B. Saunders, 2006. cdc.gov/nip/recs/adult-schedule-11x17.pdf Brunicardi FC, Andersen DK, Billiar TR, et al., eds. Rudolph CD, Rudolph AM. Pediatrics, 21st ed. New Schwartz’s Principles of Surgery, 8th ed. New York, York, NY: McGraw-Hill, 2003. NY: McGraw-Hill, 2005. Townsend CM Jr, Beauchamp RD, Evers BM, et al., Cunningham FG, Leveno KJ, Bloom SL, et al. Williams eds. Sabiston Textbook of Surgery: The Biologic Basis of Obstetrics, 22nd ed. New York, NY: McGraw Hill, Modern Surgical Practice, 17th ed. Philadelphia, PA: 2005. W.B. Saunders, 2004. Greenfield LJ, Mulholland M, Lillemoe KD, et al., USPSTF, The Guide to Clinical Preventive Services, 2006 eds. Surgery: Scientific Principles and Practice, 4th Wallace RB, Doebbeling BN, eds. Maxcy-Rosenau-Last ed. Philadelphia, PA: Lippincott-Raven, 2005. Textbook of Public Health & Preventive Medicine, 14th Kaplan HI, Sadock BJ. Synopsis of Psychiatry: ed. Stamford, CT: Appleton & Lange, 1998. Behavioral Sciences/Clinical Psychiatry, 9th ed. Way LW, Doherty GM, eds. Current Surgical Diagnosis Baltimore, MD: Williams & Wilkins, 2003. and Treatment, 124th ed. New York, NY: McGraw- Kasper DL, Braunwald E, Fauci A, et al. Harrison’s Hill, 2006. Principles of Internal Medicine, 16th ed. New York, NY: McGraw-Hill, 2005.

290 (c) ketabton.com: The Digital Library

Subject List: Practice Test 2

Question Number and Subject 23. Psychiatry 24. Internal Medicine 1. Pediatrics 25. Internal Medicine 2. Surgery 26. Internal Medicine 3. Pediatrics 27. Internal Medicine 4. Preventive Medicine 28. Surgery 5. Surgery 29. Surgery 6. Surgery 30. Surgery 7. Obstetrics-gynecology 31. Surgery 8. Pediatrics 32. Internal Medicine 9. Surgery 33. Obstetrics-gynecology 10. Psychiatry 34. Obstetrics-gynecology 11. Surgery 35. Psychiatry 12. Surgery 36. Psychiatry 13. Pediatrics 37. Psychiatry 14. Preventive Medicine 38. Surgery 15. Preventive Medicine 39. Surgery 16. Preventive Medicine 40. Psychiatry 17. Pediatrics 41. Psychiatry 18. Pediatrics 42. Preventive Medicine 19. Pediatrics 43. Internal Medicine 20. Pediatrics 44. Surgery 21. Pediatrics 45. Psychiatry 22. Preventive Medicine 46. Obstetrics-gynecology

291 (c) ketabton.com: The Digital Library

This page intentionally left blank (c) ketabton.com: The Digital Library

CHAPTER 9 Practice Test 3 Questions

Questions 1 through 4 (D) atelectasis (E) a pelvic abscess A 60-year-old previously healthy woman is scheduled for an elective colon resection for a nonobstructing sig- 3. Appropriate evaluation and management moid colon carcinoma. The patient began a clear liq- should include which of the following? uid diet 2 days preoperatively. Oral antibiotics were started the day before surgery, in addition to magne- (A) empiric antibiotic therapy sium citrate. The patient vomited after the first dose of (B) blood, urine, and sputum cultures, fol- magnesium citrate and was not able to tolerate a sec- lowed by empiric antibiotic therapy ond oral dose. At surgery, the patient received a third- (C) pulmonary toilet with incentive spirom- generation cephalosporin intravenously on induction etry and chest physiotherapy of anesthesia. During the colonic anastomosis, there (D) removal of the urinary catheter was gastrointestinal spillage. (E) computed tomography (CT) scan of the 1. This operative procedure would be classified as abdomen, with percutaneous abscess drainage (A) class I clean (B) class II clean-contaminated 4. By the 6th postoperative day, the patient has (C) class II clean-contaminated with inade- normal bowel function and is tolerating a full quate bowel preparation and gastroin- oral diet. However, she develops a spiking fever testinal spillage pattern and complains of increasing incisional (D) class III contaminated discomfort. On examination, there is a small amount of drainage on the wound dressing, and (E) class IV dirty the incision is erythematous and tender. Appropriate evaluation and management at this 2. Thirty-six hours after surgery, the patient has a point should include which of the following? persistent low-grade fever and mild tachyp- nea. On examination of the chest, there is (A) blood, urine, and sputum cultures; chest decreased air entry at the lung bases. The x-ray and abdominal CT scan abdominal dressing is clean and dry, and urine (B) discharging the patient on oral antibiotics output via catheter is clear and adequate in (C) culture of the wound drainage, and ini- volume. Which of the following is the most tiation of IV antibiotics likely cause of the patient’s fever? (D) opening the incision, culture of the (A) a wound infection wound drainage, and initiating local (B) a urinary tract infection wound care (C) pneumonia (E) opening the incision, culture of the wound drainage, and initiating IV antibiotics

293

Copyright © 2008 by The McGraw-Hill Companies, Inc. Click here for terms of use. (c) ketabton.com: The Digital Library

294 9: Practice Test 3

5. A 53-year-old male just received notice from ago. It has not been bothering him at all, so he the blood bank that his most recent blood dona- did not mention it. Which of the following is tion tested positive for HIV. He is from South the most likely cause of this lesion? Dakota, is well known by you, is believed to be trustworthy, and gives blood annually. He reports that for the past 20 years he has been in a mutually monogamous relationship with his wife, and that they have never used intra- venous drugs or transfusions. The blood bank uses an enzyme-linked immunosorbent assay (ELISA) to test for HIV.

Which of the following would be the most cor- rect about this patient? (A) He is at very low risk of having HIV. He should abstain from sex or practice safe sex while awaiting the results of a repeat ELISA. FIG. 9-1 Also see color insert. (Courtesy of Neil S. Prose.) (B) He is at very low risk of having HIV. He should abstain from sex or practice safe sex while awaiting the results of a (A) condylomata acuminata infection confirmatory test such as a Western (B) herpes simplex infection Blot test. (C) staphylococcal infection (C) Based on the specificity of the ELISA (D) trauma test, he has approximately a 13% chance (E) primary syphilis infection of being infected with HIV. He should abstain from sex or practice safe sex 7. Your hospital is about to launch a new breast while awaiting the results of a repeat cancer awareness program. In preparation for ELISA. determining the effectiveness of the initiative, (D) Based on the specificity of the ELISA you are asked to determine the current rate of test, he has approximately a 13% chance mammographies among women admitted to of being infected with HIV. He should your hospital. What type of study would be abstain from sex or practice safe sex most appropriate for this purpose? while awaiting the results of a confirma- tory test such as a Western Blot test. (A) a case-control study (E) He should immediately be offered post- (B) a cohort study exposure prophylaxis, which would be (C) a cross-sectional study initiated while awaiting results from (D) a randomized trial confirmatory testing. (E) a clinical trial

6. A 17-year-old adolescent comes to the clinic 8. You have been treating an otherwise healthy for a sports physical. He has no concerns about 35-year-old male assembly line worker for sci- his health. He is a senior and is looking forward atica, secondary to lumbar disk herniation at to playing baseball. He states that he is sexually L4–L5. He is improving, several modified duty active and has had three partners in the last 6 jobs are available. Which of the following months. They usually use condoms for birth would be least hazardous for him? control. His physical examination is normal except for the lesion shown in Figure 9-1. On further questioning, he first noted it 2 weeks (c) ketabton.com: The Digital Library

Questions: 5–9 295

(A) assembly of 20 5-lb parts per hour while 9. A 3-year-old boy was hypotonic at birth. He standing at a table and repeatedly lean- also has moderate short stature and the physi- ing across the table to tighten bolts with cal features shown in Figure 9-2. a wrench (B) inspection of dense 15-lb metal parts while standing or sitting at a table, and 10 times per hour carrying the part 10 ft at waist level to another table (C) assembly of three 1-lb parts per minute while continuously seated at the assem- bly line, and doing job rotation to other similar positions two times per shift (D) picking up 10-lb boxes of parts from the floor without squatting and transferring them to a waist-level conveyor once per minute (E) keeping him off work until he can work without restrictions

DIRECTIONS (Questions 9 and 10): For each num- bered item, select the ONE best lettered option that is most closely associated with it. Each lettered option may be selected once, more than once, or not at all.

For each of the following children with birth defects, select the most likely diagnosis. (A) Beckwith-Wiedemann syndrome (B) Cornelia de Lange’s syndrome (C) defects associated with maternal use of FIG. 9-2 (Reproduced, with permission, from Rudolph AM. Rudolph’s coumadin derivatives Pediatrics, 20th ed. Stamford, CT: Appleton & Lange, 1995) (D) fetal alcohol syndrome (E) fetal hydantoin syndrome (F) Noonan’s syndrome (G) Prader-Willi syndrome (H) Williams syndrome (c) ketabton.com: The Digital Library

296 9: Practice Test 3

10. A 12-month-old boy has a history of failure to Questions 11 and 12 thrive and developmental delay. He also has a cleft palate, a ventricular septal defect, and A 33-year-old man presents to the emergency room hearing loss. He has the facial features shown complaining of left-sided chest pressure and short- in Figure 9-3. ness of breath that began about an hour ago. He is otherwise healthy with no significant past history and a recent normal physical examination. His cho- lesterol was 168 with a high-density lipoprotein (HDL) of 75. He has no family history of coronary artery disease. His electrocardiogram (ECG) is shown in Figure 9-4.

11. What is the abnormality on the ECG? (A) acute pericarditis (B) acute anteriolateral wall myocardial infarction (MI) (C) acute inferior wall MI (D) congestive heart failure (E) atrial fibrillation

12. Which of the following is the most likely cause of these findings? (A) cocaine use FIG. 9-3 (Reproduced, with permission, from Rudolph AM. Rudolph’s (B) coronary artery disease Pediatrics, 20th ed. Stamford, CT: Appleton & Lange, 1995.) (C) viral pericarditis

FIG. 9-4 (c) ketabton.com: The Digital Library

Questions: 10–20 297

(D) pulmonary embolism (A) axis I (E) congestive heart failure (B) axis II (C) axis III DIRECTIONS (Questions 13 through 19): For each (D) axis IV numbered item, select the ONE best lettered (E) axis V option that is most closely associated with it. Each lettered option may be selected once, more than 13. Mild mental retardation once, or not at all. 14. Ejection from group home Questions 13 through 19 15. Chronic paranoid schizophrenia A 24-year-old man is brought to a general hospital emergency room by local police. They explain that 16. Borderline personality disorder the man is well known to them as a “troublemaker.” The police report that he suffers from mild mental 17. Asthma retardation, and that he has been very upset for the past month because of the recent death of his mother, 18. Type II diabetes mellitus who he had visited every week while he lived in a group home. They report that the young man is also 19. Recent death of mother mentally ill and that his behavior has recently deteri- orated, resulting in his eviction from the group home Questions 20 and 21 1 day earlier. The physician on call obtains the patient’s past medical record from the file room, and A 16-year-old boy comes to the clinic for a sports learns that the patient has a history of asthma and physical. You have followed him for many years type II diabetes mellitus, for both of which he is seen and he has always been healthy. He continues to be in a hospital clinic. The records also indicate that the healthy and has no concerns. On physical examina- patient has a history of impulsive and self-destruc- tion, the only abnormality that you discover is an tive behavior, and that he was diagnosed at age 18 as irregular heart rhythm. You order a rhythm strip having borderline personality disorder. At age 21, the which is shown in Figure 9-5. records indicate, the patient began to experience auditory hallucinations and to believe that the FBI 20. Which of the following is the most likely was out to get him. Since then, he has carried the diagnosis? diagnosis of chronic paranoid schizophrenia. (A) atrial premature beats In the Diagnostic and Statistical Manual of (B) sick sinus syndrome Mental Disorder, 4th Edition, Text Revision (DSM-IV- TR), a multiaxial diagnostic system is used. Using the (C) sinoatrial block above case, match the axis below to the diagnostic (D) sinus arrhythmia features listed. (E) supraventricular tachycardia

FIG. 9-5 (Reproduced, with permission, from Rudolph C, et al. Rudolph’s Pediatrics, 21st ed. New York, NY: McGraw-Hill, 2003.) (c) ketabton.com: The Digital Library

298 9: Practice Test 3

21. After identifying the particular rhythm, you (D) This is likely his physiologic nadir. arrange for further testing. Which of the fol- (E) Transfusion should be considered for lowing diagnostic tests would be the most any hemoglobin less than 10 g/dL. appropriate initial step in caring for this patient? (A) chest x-ray 25. A study finds that the incidence of asthma in your community is higher than what is (B) echocardiogram reported for the state, and that 40% of the (C) ECG homes in your community are heated with (D) repeat rhythm strip in 1 week forced air. The study design described here is (E) no further diagnostic tests are necessary best described as which of the following? (A) an ecologic study 22. A child is able to use a pincer grasp (thumb–first digit) to pick up a raisin. He can stand with (B) a cross-sectional study support. He says “mama” and “dada.” He is (C) a cohort study shy around strangers. You correctly note that he (D) a case-control study is developing normally. How old is he? (E) a randomized clinical study (A) 6 months 26. In reviewing your clinical practice, you notice (B) 9 months that almost one quarter of the hospital inpa- (C) 12 months tients you have seen in the past 3 months have (D) 15 months carried the diagnosis of congestive heart fail- (E) 18 months ure. Which of the following best describes about how common this disorder is among 23. A 2-month-old infant is brought to the emer- patients currently in the hospital? gency department after having a seizure at (A) incidence home. The baby has not been ill, and there is no history of trauma. On examination, the baby is (B) prevalence afebrile and is very lethargic. You are con- (C) positive predictive value cerned about the possibility of child abuse. (D) negative predictive value What physical finding would suggest that the (E) relative risk child had injuries from violent shaking? (A) Kernig’s sign 27. A 6-year-old boy has a purpuric and petechial rash over the buttocks and lower extremities. He (B) neck tenderness appears to be well and is afebrile, but has swelling (C) overlapping cranial sutures of the right knee. His CBC is normal. Which of the (D) positive rooting reflex following is the most likely diagnosis? (E) retinal hemorrhages (A) idiopathic thrombocytopenic purpura (ITP) 24. A 9-week-old infant has a hemoglobin of 9.2 g/dL. The rest of his complete blood count (B) systemic lupus erythematosus (SLE) (CBC) is normal. He has not had any health (C) meningococcemia problems and was born at term without com- (D) Henoch-Schönlein (anaphylactoid) plications. Which of the following statements is purpura (HSP) true about anemia in infants at this age? (E) juvenile rheumatoid arthritis (A) Breast-fed infants commonly develop iron deficiency at this age. 28. A 41-year-old woman requests information about cervical cytology (Pap smear). In your (B) A congenital hemoglobinopathy is likely. counseling you tell her each of the following (C) Lead toxicity is a common cause of statements. Which one of the following state- anemia at this age. ments is correct? (c) ketabton.com: The Digital Library

Questions: 21–33 299

(A) The first Pap smear should be done at has a culture of her cervix that is positive for age 18 or when the woman becomes group B Streptococcus (GBS). Her first child was sexually active, whichever is first. delivered at 35 weeks after spontaneous rup- (B) A Pap smear should be done annually ture of the amniotic membranes and sponta- in all women. neous onset of labor. During her first labor, she (C) A CIN I (cervical intraepithelial neopla- had several recorded temperatures of 101.6°F. sia, mild dysplasia) Pap result will Which of the following is the most appropriate progress to cervical cancer in more than next step in management? 50% of women. (A) Prescribe oral amoxicillin for 1 week and (D) Cigarette smoking reduces the risk of repeat the cervical culture in 4 weeks. cervical cancer. (B) Repeat the cervical culture at 36 weeks’ (E) Liquid-based cervical cytology (Thin- gestation and administer amoxicillin if it Prep, Autocyte) yield have a higher is positive. true-positive rate than cytology inter- (C) Administer IV penicillin when she goes preted from a microscopic slide. into labor. (D) Perform a cesarean section to prevent 29. A 6-week-old male infant presents with a 5-day neonatal infection. history of progressively worsening vomiting. (E) Do nothing, because the attack rate to He has not had a stool in 2 days. On physical the infant is very low. examination, he is dehydrated. Serum elec- trolytes are: 32. During a routine yearly checkup, an 8-year-old boy is found to have 2+ proteinuria on urinal- + + − −1 Na K Cl HCO3 ysis. Which of the following is the most appro- 136 3.0 88 36 priate diagnostic test to order? (A) blood urea nitrogen (BUN) and serum Which of the following is the most likely creatinine diagnosis? (B) urine culture (A) congenital adrenal hyperplasia (C) intravenous pyelogram (IVP) (B) gastroesophageal reflux (D) a repeat urinalysis (C) pyloric stenosis (E) renal sonogram (D) renal tubular acidosis 33. A 30-year-old employee has had audiometric test- (E) viral gastroenteritis ing done as a part of the company’s Hearing Conservation Program. You notice that on the 30. The mother of a 2-year-old boy asks you how right there is a 10 dB hearing loss at both 3000 Hz tall you think he will be. His father is an NBA and 4000 Hz. On the left, there is a 5 dB loss at 3000 player and his mother is wondering if the child Hz and a 10 dB loss at 4000 Hz. Which of the fol- might follow in his footsteps. Currently his lowing is the most appropriate course of action? height is 37.5 in. You tell the mother that her son will most likely be how tall? (A) Return this person to work with ear plugs. (A) about 5 ft tall (B) Wash out an accumulation of cerumen. (B) about 7 ft tall (C) Refer to an ear, nose, and throat special- ist to rule out acoustic neuroma. (C) about 6 ft tall (D) Assume a recent middle ear infection (D) 5 ft 3 in. tall and recheck in 2 weeks. (E) there is no way to tell (E) Alert the primary care physician to the possibility that an ototoxin is inducing 31. At her first prenatal visit at 8 weeks’ gestation, hearing loss. a woman who is pregnant for the second time (c) ketabton.com: The Digital Library

300 9: Practice Test 3

34. A young farm worker who was working with 36. Her urine culture is positive the next day. pesticides is brought to the emergency room Which of the following organisms is the most with headache, vomiting, salivation, diarrhea, likely cause of her infection? muscle fasciculations, difficulty walking, and (A) Escherichia coli difficulty speaking. His clothing has been removed, he has been washed, and he has been (B) Klebsiella given activated charcoal. What is the most (C) nontypeable Haemophilus influenzae effective remaining treatment for this case of (D) Pseudomonas aeruginosa pesticide poisoning? (E) Staphylococcus aureus (A) epinephrine 37. You see her in the office 3 weeks later after the (B) chelation infection has been treated. All of her symptoms (C) a spironolactone have resolved, and she has a normal physical (D) atropine examination. Which of the following is the most (E) supportive therapy appropriate next step in her management? (A) Order a renal ultrasound. Questions 35 through 37 (B) Order a voiding cystourethrogram. An 8-month-old girl is brought to the clinic with a (C) Order a renal ultrasound and a voiding 3-day history of fever, vomiting, and irritability. She cystourethrogram. has never had a similar problem. On physical exam- (D) Order a nuclear medicine renal scan. ination, her temperature is 40.0°C. She appears (E) No tests are necessary. moderately dehydrated. She vomits twice during the visit. Her initial workup shows a WBC of 20,000 Questions 38 and 39 with 70% neutrophils. Her catheterized urinalysis shows a specific gravity of 1.030, 1+ protein, and 38. A 27-year-old woman presents with cauliflower- trace blood. The microscopic examination reveals like lesions of the external genitalia, vagina, and 0–5 red blood cells per high-power field, numerous cervix. They vary in size and are discrete and WBCs per high-power field, and moderate bacteria. nontender (see Figure 9-6). Which of the fol- A urine culture is sent. lowing is the most likely cause of the lesions?

35. Which of the following is the most appropriate next step in her management? (A) Prescribe trimethoprim-sulfamethoxazole and see her tomorrow in the office. (B) Give IM ceftriaxone and see her tomor- row in the office. (C) Admit her to the hospital for IV hydra- tion and IV antibiotics. (D) Wait for the urine culture and treat her if it is positive. (E) Start oral rehydration in the office and prescribe amoxicillin.

FIG. 9-6 (c) ketabton.com: The Digital Library

Questions: 34–46 301

(A) herpes simplex (C) condoms (B) gonorrhea (D) tubal ligation (C) Hemophilus ducreyi (E) vasectomy (D) human papillomavirus (HPV) (E) Chlamydia trachomatis Questions 42 through 46 For each of the following patients with a respiratory 39. A 33-year-old woman presents to your office illness, select the most likely pathogen. with the vulvar lesions shown in Figure 9-6. Which one of the following statements is (A) adenovirus correct? (B) Bordetella pertussis (A) A biopsy or excision of the lesions (C) C. trachomatis should be done to establish the (D) coxsackievirus diagnosis. (E) Epstein-Barr virus (B) Type-specific HPV nucleic acid testing (F) H. influenzae should be done to confirm the (G) herpes simplex 1 diagnosis. (H) Moraxella catarrhalis (C) The presence of these lesions increases (I) Mycoplasma pneumoniae the risk of cervical cancer. (J) parainfluenza virus (D) Successful medical or surgical therapy (K) respiratory syncytial virus (RSV) reduces the risk of future transmission. (L) rotavirus (E) The lesions most commonly occur as a (M) S. aureus result of sexual transmission. (N) Streptococcus agalactiae (group B) 40. A 72-year-old patient presents at your office (O) Streptococcus pneumoniae uncertain if he can afford your services. He does (P) Streptococcus pyogenes (group A) have a Medicare card. Which of the following is the most applicable to Medicare Part B? 42. A 3-month-old infant with wheezing and his- tory of 3 days of cough, congestion, and (A) It is a joint federal state program. rhinorrhea (B) It categorizes diagnoses and conditions into Diagnosis Related Groups (DRGs). 43. A 9-year-old with gradual onset of fever, (C) It reimburses based on the Resource malaise, and worsening cough over 5 days and Based Relative Value Scales (RBRVSs). rales in the area of the right upper lobe (D) It corresponds to Title 19 of the Social Security Act. 44. A 3-month-old infant with history of eye (E) It provides a minimum set of services to drainage at 2 weeks of age who now has insid- the needy. ious onset of staccato cough and is afebrile, with bilateral rales on examination 41. A 33-year-old woman with two children requests your advice about contraceptive 45. A 3-month-old with history of 2 weeks of con- choices. She and her partner desire no more gestion and rhinorrhea, who now has worsen- children. Both are healthy, nonsmoking adults. ing paroxysms of coughing She is especially interested in the method with the lowest failure rate. Which one of the fol- 46. A 2-year-old with 2 days of rhinorrhea and lowing has the lowest failure rate as used? cough, now with hoarseness and a barky cough (A) oral contraceptives (B) intrauterine device (IUD) (c) ketabton.com: The Digital Library

Answers and Explanations

1. (D) Surgical procedures are classified into four Nosocomial pneumonia is a significant categories, based on the risk of wound infec- problem in the surgical patient but does not tion. Elective colon surgery with an adequate usually present in the first few days after elec- bowel preparation, and no spillage of gas- tive surgery. Patients with inadequate pain trointestinal contents, is classified as a clean- control, poor pulmonary toilet, and prolonged contaminated (class II) procedure. In this preoperative hospitalization are at greater risk. patient, there was an incomplete, and hence, Intra-abdominal abscess may develop as a inadequate bowel prep, with resultant soilage complication after colonic surgery, either as a at surgery, converting this to a class III con- consequence of intraoperative fecal contamina- taminated procedure. (Townsend et al., pp. 258–259, tion, or as a result of an anastomotic leak. 301–303) Established pelvic infection does not usually present before the fifth to seventh postopera- 2. (D) tive day. Management options include percuta- neous CT-guided drainage. (Townsend et al., pp. 3. (C) 267–268, 299–303, 305–307; Way and Doherty, p. 34)

4. (D) 5. (A) The diagnosis of HIV infection depends on the demonstration of antibodies to HIV and/or Explanations 2 through 4 the direct detection of HIV or one of its com- ponents. The standard screening test for HIV is Evaluation of postoperative fever should begin an ELISA, which detects antibodies to HIV. with a thorough clinical history and examina- Antibodies to HIV generally become detectable tion of the patient. Empiric cultures and antibi- in the circulation 4–12 weeks after infection. otic therapy are not indicated and are not The sensitivity of the ELISA test approaches cost-effective. Therapy should be targeted at 100% in persons with clinical AIDS. The speci- clearly identified foci. Atelectasis is the most ficity of the ELISA test is generally above 99.5%. common cause of postoperative fever in the This means that among 1000 normal persons, first 24–48 hours. It is managed with aggres- up to 5 may be expected to be falsely identified sive pulmonary toilet, chest physiotherapy, as being seropositive for HIV. In a study of a ambulation, and adequate pain control. group of volunteer blood donors, for example, Postoperative wound infection usually pres- only 13% of ELISA-positive individuals truly ents between the 5th and 10th postoperative had HIV infection. (The positive predictive days, with fever, increased incisional discom- value of the test was 13%.) The individual fort, wound erythema, and purulent drainage. described in this question is from an even Management is directed at opening the wound lower risk group, for which the positive pre- for adequate drainage and local wound care. dictive value of the ELISA test would be even Therapy with antibiotics is not usually required lower, meaning that an even larger proportion of if adequate drainage is achieved. A urinary positives would be false-positives. Factors asso- tract infection may develop postoperatively, ciated with false-positive ELISA tests include but usually does not present before the third to large exposure to many antigens, as in receiving fifth postoperative day. There is an increased multiple transfusions, and having autoantibod- risk in patients with indwelling urinary ies, hepatic disease, or recent influenza vacci- catheters for greater than 5 days. nation. Positive results thus must be confirmed.

302 (c) ketabton.com: The Digital Library

Answers: 1–12 303

A person initially testing positive with an improve tolerance and outcome in low back ELISA test should first have the ELISA test pain. Carrying weight close to the waist and repeated. If it is still positive, confirmatory test- without bending moderately increases pressure ing should be done with a test such as a on intervertebral disks. Extended forward reach, Western Blot. A negative Western Blot allows bending at the waist, and sitting markedly one to conclude with certainty that the ELISA increase pressure on the lumbar intervertebral reactivity was a false positive. Clinicians disks. (Wallace and Doebbeling, pp. 652–657) should have these tests performed only at qual- ified laboratories. (Wallace and Doebbeling, p. 157; 9. (G) These are the classic history and physical CDC Revised Recommendations for HIV Testing for findings seen in Prader-Willi syndrome. Typical Adults, Adolescents, and Pregnant Women in Health- features are obesity, almond-shaped eyes, and Care Settings. Available at: http://www.cdc.gov/ mmwr/ narrow bifrontal diameter. Patients also have preview/mmwrhtml/rr5514a1.htm) severe neonatal hypotonia, hypoplastic genitalia, small hands and feet, polyphagia, and mental 6. (E) The primary syphilitic lesion or chancre retardation. It is inherited sporadically, and many starts as a papule and over several weeks affected individuals have an abnormality of chro- develops into a small plaque with an erosive mosome 15. (Rudolph et al., p.737, 2022, 2303) surface. It occurs at the site of penetration of the treponemes and is most commonly found on 10. (D) This child has fetal alcohol syndrome. The the genitalia. It is not painful. Herpes infection facial features shown include narrow palpebral lesions are typically grouped vesicles on an fissures; a short nose with a broad, low bridge; erythematous base. Lesions from trauma and midfacial hypoplasia; and a long philtrum with staphylococcal infections would be painful. a narrow vermilion border. Associated anom- Lesions from condylomata acuminata are soft, alies are cleft palate; cardiac malformations flesh-colored, and nodular. (Hurwitz, pp. 302–305) (especially atrial and ventricular defects); hear- ing loss; and joint, skin, and skeletal abnormal- 7. (C) A cross-sectional study is a survey, and is ities. There are also varying degrees of growth appropriate to answer this question. Cross- failure and developmental delay. The severity of sectional studies often produce biased results. the defects seems to be proportional to the Subjects selected for survey may not be repre- amount of alcohol consumed during the preg- sentative of the desired population, or only an nancy. (Rudolph et al., pp. 775–778) unrepresentative subset of those selected may actually respond to the survey (selection bias). 11. (B) The ST-T segment elevation in the anterior

Subjects, interviewers, or test instruments may and lateral leads (V2–V6, I, AVL) in this ECG is systematically record information in an unrep- indicative of an acute anterolateral wall MI. resentative manner (information bias). The effect Acute pericarditis causing ST-segment eleva- of an extraneous variable may be mixed with the tion would cause ST-T elevation in all of the effect of the studied exposure on the studied leads. An acute inferior wall MI would show disease (confounding). Trials and cohort studies changes in the inferior leads (II, III, AVF) and generally refer to prospective studies. A case- not ST-T elevation in the anterolateral leads. control study generally identifies a group of Pulmonary embolism and congestive heart fail- individuals with an uncommon disease and ure would not typically cause focal ST-T ele- compares the historical profiles of those “cases” vation. (Kasper, pp. 1311–1319) with those of disease-free individuals who oth- erwise are generally comparable. (Gordis, pp. 12. (A) This patient is young and has no risk factors 173–174; Greenberg and Daniels, pp. 144–150) for coronary artery disease. Specifically, his cho- lesterol profile is excellent, and he has no family 8. (B) Early return to appropriate work is associ- history of coronary disease. Cocaine would be ated with improved functional outcome. most likely in this patient as the cause of the Opportunities to change posture frequently MI. Cocaine abuse is associated with a variety of (c) ketabton.com: The Digital Library

304 9: Practice Test 3

life-threatening cardiac complications including the diagnosis of atrial premature beats that sudden death, myocarditis, dilated cardiomy- would have premature P waves that were opathy, and acute MI resulting from coronary shaped differently. Patients with sick sinus syn- spasm and/or thrombosis with or without drome usually have episodes of tachycardia underlying stenosis. Nitrates, calcium channel and episodes of sinus arrest. In sinoatrial block, blockers, and benzodiazepines have been used the sinoatrial pacemaker is discharging, but to treat cocaine-induced cardiotoxicities. Beta- occasionally the impulse does not depolarize blockers should be avoided. (Kasper, p. 1410) the atria. The strip shows pauses that approx- imate the normal P-P interval. This patient does 13. (B) not have a tachycardic rhythm, and patients with supraventricular tachycardia usually have 14. (D) symptoms of palpitations or a sensation of their heart racing. (Rudolph et al., pp. 1853–1854) 15. (A) 21. (E) Because this is a normal variant, no further 16. (B) diagnostic testing is necessary. (Rudolph et al., p. 1853) 17. (C) 22. (B) The pincer grasp is attained in the second 6 18. (C) months of life. Ninety percent of children can do this by age 10 months. The milestone of object 19. (D) constancy (the understanding that an object con- tinues to exist even if the child does not see it) Explanations 13 through 19 occurs around 9 months. The infant then recog- nizes the difference between people he knows The multiaxial diagnostic system of DSM-IV- well and strangers, which leads to the develop- TR is used to code information about several ment of stranger anxiety. Ninety percent of facets of a patient’s condition. Clinical psychi- infants can stand with support by 10 months. atric disorders, other than personality disorders Between 8 and 10 months, babbling becomes and mental retardation, are coded on axis I. more complex, with “mama” and “dada” used Chronic paranoid schizophrenia is an example nonspecifically. (Behrman et al., pp. 43–44) of such a clinical disorder. Personality disor- ders (such as borderline personality disorder) 23. (E) Retinal hemorrhages in young children, and mental retardation are coded on axis II. when seen with other evidence of trauma, General medical conditions, such as asthma and especially when the history is not consistent type II diabetes mellitus, that are potentially with the physical examination, are pathogno- relevant to the understanding or management monic of child abuse. Kernig’s sign is a sign of of an individual’s mental disorder are coded meningeal irritation and is seen with meningi- on axis III. Psychosocial and environmental tis. The rooting reflex is normal in infants. problems, such as the recent death of this Overlapping cranial sutures can be a normal patient’s mother and his ejection from his group finding shortly after a vaginal birth. They also home, are coded on axis IV, and a global assess- can occur when the infant has microcephaly. ment of an individual’s functioning on a scale Neck tenderness could be due to abuse or acci- from 1 to 100 is coded on axis V. (APA, pp. 27–35) dental trauma and of itself does not suggest abuse. (Rudolph et al., p. 2176, 2238, 2415) 20. (D) Sinus arrhythmia is a normal variant that is common in pediatric patients. It usually varies 24. (D) At birth, the average hemoglobin is 17 g/L with the respiratory cycle, slowing with expi- because of the relatively hypoxic intrauterine ration. On the strip, each normal QRS wave is environment. After birth, the PaO2 rises, and preceded by a normal P wave. This excludes erythropoiesis decreases. Red cell life span is (c) ketabton.com: The Digital Library

Answers: 13–28 305

shorter at this time as well. These two processes and occasionally arthritis, nephritis, abdominal lead to the physiologic nadir in hemoglobin pain due to gastrointestinal bleeding, or other that occurs around 2 months of age. Iron defi- vasculitic complications. Remember, HSP is ciency is the most common anemia in child- characterized by nonthrombocytopenic pur- hood but is not typically seen until after 6–12 pura. A normal platelet count rules out ITP. months. Lead toxicity is not seen until children SLE, more commonly afflicting females, is not can put objects in their mouths. The patient often encountered in children. Children with could have a congenital hemolytic anemia with meningococcemia are acutely ill and febrile. a normal hemoglobin, but this is much less Petechiae are not a feature of juvenile rheuma- likely. Transfusion is not indicated in this toid arthritis. (Behrman et al., pp. 677–678) patient unless there are symptoms or signs of cardiac failure. (Kliegman et al., pp. 803–817) 28. (E) Liquid-based cytology diagnosed 36% to more than 200% more cases of low-grade squa- 25. (A) Ecologic studies compare groups not indi- mous intraepithelial lesion (LSIL) than the con- viduals. The unit of observation in the above ventional slide technique of Pap smears. study was a community not individuals. A cross- However, the degree of improved sensitivity sectional study design would have looked at the with liquid-based Pap smears and the difference incidence of asthma in those who also owned a in specificity from slide-based cytology is uncer- particular home heating system at a particular tain. With liquid-based cytology the remaining point in time. A cohort study would have taken liquid (commonly retained 2–4 weeks) can be disease-free individuals in the community who assayed for HPV serotypes in women with atyp- also owned the home heating system and fol- ical squamous cells of undetermined significance lowed them over time to see who developed dis- (ASCUS) to ascertain the presence of high-risk ease. A case-control approach would have taken serotypes for cervical dysplasia and cancer. individuals with and without asthma and iden- Women with LSIL or high-grade squamous tified how many from each group also had the intraepithelial lesion (HSIL) Pap smears are pre- particular home heating system. A randomized sumed to have exposure to high-risk strains of clinical trial would have randomly selected cer- HPV. In a review of the literature since 1950, 57% tain families to live in homes with the two dif- of patients with CIN I underwent spontaneous ferent heating systems and would have regression whereas only 1% of women with CIN monitored outcomes after a set period of time. I progressed to invasive cervical cancer. In pro- (Wallace and Doebbeling, pp. 18–19; Gordis, pp. 204–205) grams with organized and systematic screening for cervical cancer, there was little advantage of 26. (B) Prevalence is the number of affected per- annual Pap smears over a Pap smear every 2–3 sons in a population divided by the total years. The American College of Obstetricians number of persons in the population at that and Gynecologists currently recommend annual time. Incidence is the number of new cases of Pap smears until age 30. Women over 30 who a disease that occur during a specified period have had three consecutive negative Pap smears of time within a population at risk. Both require may then be screened every 2–3 years. More fre- knowledge of population numbers. Calculation quent screening should be continued for women of prevalence requires knowledge of the total with certain risk factors for cervical dysplasia or number of individuals in the population from cancer: women with HIV, immunosuppressed which the cases arose. Calculation of incidence women, and women exposed to diethylstilbe- requires knowledge of the total number of indi- strol (DES) in utero. ACOG now recommends viduals in the population at risk at the begin- that screening with cervical cytology begins at ning of the study. (Gordis, pp. 35–37) age 21 or three years after the onset of sexual activity. While cigarette smoking appears to 27. (D) HSP, the most common vasculitis of child- decrease the risk of endometrial cancer, the risk hood, is characterized by petechiae or pur- of cervical cancer is increased in these women. pura of the buttocks and lower extremities (ACOG Practice Bulletin) (c) ketabton.com: The Digital Library

306 9: Practice Test 3

29. (C) Children with one of the salt-losing forms of 32. (D) Many healthy children have intermittent congenital adrenal hyperplasia have aldosterone proteinuria that can be quite substantial and can deficiency. In the absence of this hormone, the be exaggerated by vigorous exercise. In those kidney does not retain sodium normally, and in whom it is intermittent and not accompanied there is inappropriate retention of potassium by hematuria, chronic renal disease is unusual. and hydrogen ions, resulting in hyponatremia, A child who has proteinuria on a single speci- hyperkalemia, and metabolic acidosis. The men, thus, will need repeated urinalyses to characteristic serum electrolyte aberrations establish the intermittent nature of the finding. observed in patients with hypertrophic pyloric Careful examination of the sediment to look for stenosis include hypokalemia, hypochloremia, red blood cells and casts should also be per- and metabolic alkalosis. Children with renal formed. BP should be determined at each visit. tubular acidosis often present with hyper- Some clinicians also suggest protein determina- chloremic metabolic acidosis and hypokalemia. tions on 12-hour specimens, collected while Patients with gastroesophageal reflux usually these children are active and again while they present with vomiting over several weeks. have been resting for 12 hours, thus establishing Patients with viral gastroenteritis usually have the link to exercise. If proteinuria remains inter- vomiting and diarrhea. (Kliegman et al., pp. 301–317) mittent, most physicians do not perform inva- sive procedures such as a biopsy. Proteinuria 30. (C) Children usually reach 50% of their alone is unlikely to be an indicator of urinary expected adult height by the age of 2 years. tract infection or structural kidney disease; thus, Additionally, at 2 years of age, children reach an IVP and urine culture are unlikely to be help- approximately 20% of their expected adult ful. Likewise, with isolated and intermittent pro- weight and 85% of their expected adult head teinuria as the only abnormality, BUN and circumference. (Behrman et al., p. 43) creatinine levels are highly unlikely to be abnormal. (Rudolph et al., pp. 1664–1665) 31. (C) As many as 15–40% of women have GBS in their lower genital tract. The overall attack rate for 33. (A) This pattern of loss is typical for very mild early onset neonatal sepsis is 1–2 per 1000 births. noise-induced hearing loss. The employee In women whose lower genital tracts are known would, if exposed to loud noise on the job to be colonized, the attack rate is 1–2%. Because and in the OSHA (Occupational Safety and of the low attack rate and the high rate of recolo- Health Administration)-mandated Hearing nization after antibiotic treatment, antibiotic Conservation Program, be fitted with and pro- therapy of the initial positive culture is neither vided with hearing protection, trained in hear- beneficial nor cost-effective. For the same reason, ing conservation, and returned to work. a repeat culture and treatment of a positive cul- Obstruction with cerumen would decrease ture at 36 weeks is not justified. There is no evi- hearing more profoundly and at all frequen- dence that a cesarean section reduces the cies. An acoustic neuroma generally produces probability of neonatal GBS sepsis. The woman a rapidly progressive (and soon more pro- who is likely to benefit the most from intrapartum found) hearing loss that is unilateral and may antibiotic prophylaxis is one who has a known be associated with new onset of tinnitus. positive culture and one or more of the following Middle ear infection may cause some tempo- risk factors (Creasy and Resnick, pp. 754–757): rary loss at all frequencies, and generally can be revealed by history and otoscopic examination. • Preterm labor (37 weeks) Ototoxins generally cause loss in a pattern sim- • Preterm, premature rupture of the membranes ilar to that of presbycusis, in which most loss (37 weeks) occurs at the highest frequencies. (LaDou, pp. • Prolonged rupture of the membranes 112–118; McCunney, pp. 381–393; Rom, pp. 1345–1355) (18 hours) • Sibling affected by symptomatic GBS 34. (D) This man has been poisoned with an infection organophosphate insecticide such as parathion. • Maternal fever during labor (c) ketabton.com: The Digital Library

Answers: 29–41 307

Organophosphates are the most common cause hepatitis B and C. Annual Pap smears are cru- of insecticide poisonings. Symptoms may cial, because several strains of HPV are associ- develop during or up to about 12 hours after ated with a high risk of developing cervical exposure. Respiratory, dermal, and gastroin- dysplasia or neoplasia (HPV types 16/18, 45, testinal absorption routes are all possible. and 46). Venereal warts are usually caused by Organophosphates lead to nicotinic, mus- strains of HPV yielding a low to intermediate carinic, and central nervous system overstim- risk of cervical dysplasia or neoplasia, but ulation. DUMBELS (diarrhea, urination, miosis, women can be infected with more than one bronchospasm, emesis, lacrimation, and sali- strain of HPV. (CDC, No. 55, pp. 94–100) vation) is a mnemonic for the muscarinic (cholinergic) signs seen with organophosphate 39. (E) The woman has a classic picture of condy- poisoning. Atropine is an anticholinergic agent. loma acuminata. There is no persuasive evi- (LaDou, pp. 186–187; Rom, pp. 1157–1163) dence that transmission occurs in any manner other than sexual. This is caused by one or 35. (C) more strains of HPV. A biopsy or excision is not necessary to establish the diagnosis. 36. (A) However, there are several circumstances when a biopsy should be done: the diagnosis is 37. (C) uncertain (e.g., similar lesions in older women may be verrucous carcinoma), the lesions do Explanations 35 through 37 not respond or worsen with standard therapy, the patient is immunocompromised, or the The patient seems to have pyelonephritis. warts are pigmented, indurated, fixed, and Treatment is directed toward treating the most ulcerated. No data support the use of type- commonly acquired community organism, E. specific HPV nucleic acid testing. There is no coli, and ensuring good hydration. Many pedi- evidence that the presence of vulvar condylo- atric textbooks also recommend initial treat- mata is associated with an increased risk of cer- ment in the hospital for all patients less than 1 vical cancer. However, certain strains of HPV year of age. In this patient, because she is vom- are associated with cervical dysplasia or carci- iting and has signs of dehydration, initial man- noma (types 16, 18, 31, 33, and 35). It is uncer- agement with IV fluids and antibiotics is tain if medical or surgical therapy to reduce indicated. Aggressive therapy for pyelonephri- the viral DNA load affects future transmission. tis is especially critical for younger patients (CDC, No. 55, pp. 94–100) because their risk for renal scarring is the great- est. Pediatric patients with pyelonephritis are 40. (C) Medicare (Title 18 of the Social Security relatively likely to have an anatomic abnor- Act) covers people aged 65 and older, and the mality or vesicoureteral reflux. A renal ultra- permanently and totally disabled persons, and sound and voiding cystourethrogram are done people with end-stage renal disease. It is a fed- to check for these. A renal scan could also be eral program divided into part A (paying for used to detect anatomic abnormalities but is hospital services according to DRGs) and part more expensive and exposes the patient to B (paying physician services according to radiation, which ultrasound does not. (Kliegman RBRVSs). Medicaid (Title 19 of the Social et al., pp. 403–411) Security Act), covers people who are needy by providing a minimum set of services. It is a 38. (D) This is a classic case of condyloma (vene- federal-state program with benefits that vary real warts). Condyloma acuminatum is caused from state to state. (Wallace and Doebbeling, p. 1128) by the HPV. Because this is a sexually trans- mitted disease (STD), women with condyloma 41. (E) Two statistics are given to express failure should be screened for other STDs, such as rates of a contraceptive method: the failure gonorrhea, Chlamydia, syphilis, and possibly rate with perfect use and the failure rate with (c) ketabton.com: The Digital Library

308 9: Practice Test 3

typical use (both rates after 1 year of use). The Adenovirus can present with wheezing but latter is the most relevant because nonsteril- more typically also includes conjunctivitis and ization methods are not always used, or are pharyngitis. M. pneumoniae is the most common used incorrectly. Reported failure rates with cause of pneumonia in school-aged children. typical use are: The typical course is a gradual onset of fever, cough, and malaise. S. pneumoniae can cause • Oral contraceptives 7.6% pneumonia in this age group but typically pres- • IUD 0.1% ents more acutely. C. trachomatis is the most • Condoms 13.9% common cause for conjunctivitis in the first few • Female sterilization 0.05% months of life. Infants acquire it at birth from • Male sterilization 0.15% their mothers. In some infants, it progresses to The best answer is male sterilization because of pneumonitis with the classic “staccato” cough. the widely reported failure rates with female Adenovirus can cause a respiratory infec- sterilization. This variation depends on such tion as well as conjunctivitis, but the two factors as experience of the surgeon, type of symptoms occur concurrently. Pertussis pres- tubal sterilization procedure, age of the woman ents with an insidious prodrome over several (fertility declines with age), frequency of coitus weeks, which is difficult to differentiate from and others. The Mirena (levonorgestrel impreg- other upper respiratory infections, except that nated) IUD has a lower failure rate than the it lasts longer. The second phase is the parox- Copper T-380 (ParaGard), 0.1% vs. 0.8%. (Speroff ysmal phase, with the episodic cough that can and Fritz, p. 829) be quite severe. Although pertussis vaccine is in widespread use, the disease is relatively 42. (K) common. The reservoir of infection is typi- cally adults whose immunity has waned. 43. (I) Many pathogens cause respiratory infections in children. In this patient, the course is too 44. (C) long for a typical viral infection and not acute enough for bacterial pneumonia, such as that 45. (B) caused by GBS. Croup is a common respira- tory infection that occurs in the autumn. The 46. (J) typical course is a few days of cough, rhinor- rhea, and low-grade fever, followed by the Explanations 42 through 46 onset of paroxysms of barky or seal-like cough. It is typically worse at night and The most common cause of lower respiratory resolves in 2 days. The most common cause is infections in infants is RSV. It commonly pres- parainfluenza virus. (Kleigman et al., pp. 43–69) ents during the winter months with rhinorrhea and wheezing. Infants typically are afebrile. (c) ketabton.com: The Digital Library

BIBLIOGRAPHY

ACOG Committee on Practice Bulletins. ACOG Kliegman RM, Greenbaum L, Lye P. Practical Practice Bulletin Number 45: Cervical cytology Strategies in Pediatric Diagnosis and Therapy. screening. Obstet Gynecol 2003;102:417–27. Philadelphia, PA: Elsevier, 2004. American Psychiatric Association (APA). Diagnostic LaDou J. Current Occupational & Environmental and Statistical Manual of Mental Disorders, 4th ed. Text Medicine, 3rd ed. New York, NY: McGraw-Hill, Revision. Washington, DC: American Psychiatric 2003. Association, 2000. McCunney RJ. A Practical Approach to Occupational Behrman RE, Kliegman RM, Jenson HB. Nelson and Environmental Medicine, 3rd ed. Philadelphia, Textbook of Pediatrics, 17th ed. Philadelphia, PA: PA: Lippincott Williams & Wilkins, 2003. W.B. Saunders, 2006. Rom WN, ed. Environmental and Occupational CDC Revised Recommendations for HIV Testing for Medicine, 3rd ed. Philadelphia, PA: Lippincott- Adults, Adolescents, and Pregnant Women in Raven, 1998. Health-Care Settings, September 2006. Available at: Rudolph CD, Rudolph AM. Pediatrics, 21st ed. New http://www.cdc.gov/mmwr/preview/mmwrhtml/ York, NY: McGraw-Hill, 2003. rr5514a1.htm Speroff L, Fritz MA. Clinical Gynecologic Endocrinology Creasy RK, Resnik R, Iams JD. Maternal–Fetal Medicine: and Infertility, 7th ed. Philadelphia, PA: Lippincott Principles and Practice, 5th ed. Philadelphia, PA: W.B. Williams & Wilkins, 2005. Saunders, 2004. Townsend CM Jr, Beauchamp RD, Evers BM, et al., Gordis L. Epidemiology, 3rd ed. Philadelphia, PA: W.B. eds. Sabiston Textbook of Surgery: The Biologic Basis of Saunders, 2004. Modern Surgical Practice, 17th ed. Philadelphia, PA: Greenberg RS, Daniels SR, Flanders WD, et al. Medical W.B. Saunders, 2004. Epidemiology, 3rd ed. New York, NY: McGraw-Hill, Wallace RB, Doebbeling BN, eds. Maxcy-Rosenau-Last 2001. Textbook of Public Health & Preventive Medicine, 14th Hurwitz S. Clinical Pediatric Dermatology: A Textbook of ed. Stamford, CT: Appleton & Lange, 1998. Skin Disorders of Childhood and Adolescence, 2nd ed. Way LW, Doherty GM, eds. Current Surgical Diagnosis Philadelphia, PA: W.B. Saunders, 1993. and Treatment, 124th ed. New York, NY: McGraw- Kasper DL, Braunwald E, Fauci A, et al. Harrison’s Hill, 2006. Principles of Internal Medicine, 16th ed. New York, NY: McGraw-Hill, 2005.

309 (c) ketabton.com: The Digital Library

Subject List: Practice Test 3

Question Number and Subject 23. Pediatrics 24. Pediatrics 1. Surgery 25. Preventive Medicine 2. Surgery 26. Preventive Medicine 3. Surgery 27. Pediatrics 4. Surgery 28. Obstetrics-gynecology 5. Preventive Medicine 29. Pediatrics 6. Pediatrics 30. Pediatrics 7. Preventive Medicine 31. Obstetrics-gynecology 8. Preventive Medicine 32. Pediatrics 9. Pediatrics 33. Preventive Medicine 10. Pediatrics 34. Preventive Medicine 11. Internal Medicine 35. Pediatrics 12. Internal Medicine 36. Pediatrics 13. Psychiatry 37. Pediatrics 14. Psychiatry 38. Obstetrics-gynecology 15. Psychiatry 39. Obstetrics-gynecology 16. Psychiatry 40. Preventive Medicine 17. Psychiatry 41. Obstetrics-gynecology 18. Psychiatry 42. Pediatrics 19. Psychiatry 43. Pediatrics 20. Pediatrics 44. Pediatrics 21. Pediatrics 45. Pediatrics 22. Pediatrics 46. Pediatrics

310 (c) ketabton.com: The Digital Library

CHAPTER 10 Practice Test 4 Questions

1. A 19-year-old G1 whose last menses began 3. An infant recently had a sweat chloride test 9 weeks ago comes to the emergency depart- because her private physician was concerned ment because of heavy vaginal bleeding and about her having cystic fibrosis. The test was lower abdominal cramping for 3 hours. Her BP positive. She has been referred to you for fur- is 146/96 mmHg, and her pulse rate is 84 min- ther diagnostic testing and counseling. She is utes. A lower abdominal mass is palpable the first child of a married couple. No one in halfway between her symphysis and umbili- either family has ever been diagnosed with this cus. The cervix is closed, and there is active disease. Which of the following should you bleeding through the cervical os. On bimanual include in your initial session when advising examination, the uterus is approximately 16 this patient’s family? weeks’ size. Fetal heart tones could not be (A) The sweat chloride test is a screening heard, and a fetus is not seen by abdominal and test, but further tests should be done transvaginal ultrasonography. Her serum to confirm the diagnosis. human chorionic gonadotropin (hCG) concen- tration is 80,000 mIU/mL, significantly higher (B) Pulmonary complications are severe and than normal for a 9-week pregnancy. Which of serious but fortunately are uncommon. the following is the most likely diagnosis? (C) Glucose intolerance is common in the first year of life. (A) multiple gestation (D) Current therapy does not provide a cure (B) tubal pregnancy for this disease but can prolong life (C) blighted ovum considerably. (D) singleton pregnancy in a myomatous (E) The disease is clearly genetic but does uterus not follow a definite autosomal or reces- (E) hydatidiform mole sive pattern.

2. You are asked to evaluate a group of individu- 4. A 12-month-old infant, previously in good als who all abruptly developed profuse vomit- health, develops an illness characterized by ing about 3 hours after eating fried rice at a 3 days of fever followed by defervescence and restaurant. All individuals are afebrile and later the development of a maculopapular rash on went on to develop watery diarrhea that resolved the trunk. No other signs or symptoms are a day later. The most likely organism responsible present. Which of the following is the most for this illness is which of these? likely diagnosis? (A) Shigella (A) erythema infectiosum (fifth disease) (B) Bacillus cereus (B) Kawasaki disease (C) Clostridium perfringes (C) roseola infantum (exanthem subitum) (D) Escherichia coli (D) rubeola (measles) (E) Salmonella (E) scarlet fever

311

Copyright © 2008 by The McGraw-Hill Companies, Inc. Click here for terms of use. (c) ketabton.com: The Digital Library

312 10: Practice Test 4

Questions 5 and 6 and uncooperative. She complained bitterly to the nurses about their insensitivity and rudeness. Her A premenopausal woman complains of a thick, white, husband tried to reassure the nurses that “really, she malodorous discharge. The pH is less than 4.5. There is not like this at all,” but then he left when his wife is no amine odor when potassium hydroxide is told him to “get out!” The nursing staff requested a added to the discharge on a slide. There are no clue psychiatric consultation to medicate this “unreason- cells on wet smear. The microscopic appearance of able, uncontrolled woman.” the discharge is shown in Figure 10-1. 7. This patient’s behavior is most likely (A) indicative of borderline personality disorder (B) the result of (PMS) (C) regression and acting out (D) due to metastasis to the brain, giving rise to behavioral changes (E) due to major depression with agitation

8. The nurses’ anger is best described as an exam- ple of which of the following?

FIG. 10-1 (A) acting out (B) transference (C) countertransference 5. Which of the following is the most likely cause of the discharge? (D) reaction formation (E) splitting (A) Gardnerella vaginalis (B) Trichomonas vaginalis 9. Which of the following is the major goal of a (C) Chlamydia trachomatis psychiatrist evaluating this woman? (D) Candida albicans (A) determine the need for restraints (E) Treponema pallidum (B) determine the need to transfer her to another hospital 6. Which of the following is the most appropriate (C) set firm limits to her shouting and treatment for that organism? complaining (A) oral metronidazole (D) help her identify and talk about her (B) oral fluconazole fears (C) IM ceftriaxone (E) get specific information about specific (D) oral azithromycin nurses in order to file a grievance (E) oral clindamycin Questions 10 through 13

Questions 7 through 9 A 22-year-old man presents to the physician’s office A 38-year-old married woman, the mother of two with the complaint of a new nodule in the neck children and a medical social worker, is hospitalized noted 1 month ago. He is the oldest of five children with a diagnosis of breast cancer. She is anticipating ranging in age from 8 to 22. His family history is surgery and then possible radiation and chemother- pertinent for thyroid carcinoma, hypertension in his apy. Since arriving, she has been demanding, hostile, paternal grandparents, and an unknown type of tumor in the abdomen of his father, now deceased. (c) ketabton.com: The Digital Library

Questions: 5–16 313

On examination, he has a normal appearance 14. Based on recent U.S. Census Bureau estimates, and body habitus. He is afebrile, with a BP of approximately what percentage of Americans 145/90 mmHg and pulse rate of 70/min. Neck were without health insurance in 2005? examination reveals a firm, 2-cm thyroid nodule. (A) 10% Fine-needle aspiration of the nodule reveals thyroid carcinoma. (B) 15% (C) 20% 10. Which of the following is the most likely type (D) 25% of thyroid carcinoma? (E) 30% (A) papillary (B) follicular Questions 15 and 16 (C) Hürthle cell A 45-year-old man is involved in a motor vehicle (D) medullary accident after drinking heavily. He is brought to a (E) anaplastic general hospital, where he is treated for a fractured left femur. About 6 hours after being admitted to the 11. Screening for which one of the following dis- hospital, he begins to demonstrate tremulousness, eases must be undertaken before treatment of which progresses to a pronounced tremor. He goes the thyroid carcinoma? on to develop full-blown alcohol withdrawal delirium (delirium tremens) on the third day after admission. (A) an adrenocorticotropic hormone He becomes disoriented and diaphoretic, with a (ACTH)-producing tumor pulse rate of 120/min and BP of 220/140 mmHg. He (B) Zollinger-Ellison syndrome reports seeing terrifying bugs in his room. (C) pituitary adenoma (D) pheochromocytoma 15. In addition to tremulousness and the clinical picture of delirium tremens outlined above, (E) ganglioneuroma which of the following is a frequent complica- tion of alcohol withdrawal? 12. Which of the following is the minimal treat- ment for the patient’s thyroid carcinoma? (A) generalized seizures (B) persistent stupor (A) ipsilateral subtotal lobectomy (C) hyperglycemia (B) ipsilateral lobectomy and isthmectomy (D) hypernatremia (C) subtotal thyroidectomy and isthmectomy (E) hypermagnesemia (D) total thyroidectomy (E) total thyroidectomy and parathyroidec- 16. Which of the following medications can help tomy prevent alcohol-induced persisting amnestic disorder (Wernicke’s encephalopathy followed 13. Screening for abnormalities in which pro- by Korsakoff’s syndrome) in patients such as tooncogene should be offered to his siblings? this one? (A) myc (A) IV glucose (B) ret (B) IV thiamine (C) ras (C) IV lorazapam (D) myb (D) IV diazepam (E) erbB (E) IV haloperidol (c) ketabton.com: The Digital Library

314 10: Practice Test 4

17. Which of the following classes of medications (C) hormone replacement with estrogen causes chronic cough? plus progestin (A) angiotensin-converting enzyme (ACE) (D) glucocorticoid replacement inhibitors (E) creation of a vagina (B) calcium channel blockers (C) fluoroquinolones 21. A 2-week-old infant is brought to the emer- gency department with vomiting for 1 day. On (D) HMG (3-hydroxy-3-methyl-glutary) physical examination, the baby is very lethargic, coreductase inhibitors is poorly perfused, and appears dehydrated. (E) cephalosporins His electrolytes are sodium, 115; potassium, 6.0; and carbon dioxide, 15. His glucose is 40. Which Questions 18 through 20 of the following is the most likely diagnosis? A 19-year-old woman has never menstruated, (A) congenital adrenal hyperplasia although breast growth began about the age of (B) duodenal atresia 11 years. Pubic hair first appeared 6–12 months (C) gastroenteritis later. There are no affected family members. Her BP (D) pyloric stenosis and pulse rate are 106/68 mmHg and 68/min, respectively. Her breasts and pubic hair are devel- (E) tracheoesophageal fistula oped to Tanner stage 5. The vagina is very short and there is no cervix seen by speculum examination. A 22. A 70-year-old male asks you about his risk for uterus cannot be palpated, despite an adequate having an abdominal aortic aneurysm (AAA). bimanual examination. He is concerned because his father passed away from a ruptured AAA when he was in his sixties. 18. Which of the following is the most appropri- He has smoked about a pack and a half a day for ate next step in evaluating the cause of her most of his life but otherwise does not have a amenorrhea? history of heart disease, elevated cholesterol, or hypertension. What is the best screening test to (A) pelvic ultrasound order to evaluate the patient for a AAA? (B) computed tomography (CT) scan of her pituitary (A) an abdominal ultrasound (C) laparoscopy (B) an abdominal CT scan (D) serum estradiol (C) a close physical examination looking for a pulsatile mass in the abdominal region (E) karyotype (D) an aortic angiogram 19. Which of the following is the most likely (E) no specific screening for AAA is indi- diagnosis? cated at this time

(A) anorexia nervosa 23. The parents of an infant born at term are con- (B) gonadal dysgenesis cerned about the possibility of their child (C) Müllerian agenesis having developmental dysplasia of the hip (D) testicular feminization (DDH). Which of the following historical fac- (E) 17α-hydroxylase deficiency tors would make this diagnosis more likely? (A) The baby is a boy. 20. Which of the following is the best treatment (B) The baby is delivered in the breech for this woman’s condition? position. (A) surgical removal of her gonads (C) The baby is delivered vaginally. (B) hypertension with estrogen alone (D) The baby is their third child. (E) The family is Black. (c) ketabton.com: The Digital Library

Questions: 17–29 315

24. A 15-year-old girl comes to the emergency 27. The man wants to know if he can be treated department 4 hours after ingesting aspirin. nonpharmacologically. Successful therapeutic After obtaining the history, you estimate that intervention would most likely be accom- she has ingested approximately 150 mg/kg of plished by which of the following? aspirin. Her physical examination is entirely (A) electroconvulsive therapy (ECT) normal. Which of the following treatments would be most useful in her management? (B) behavioral therapy (C) hypnosis (A) induced emesis (D) short-term dynamic psychotherapy (B) administration of N-acetylcysteine (E) transcranial magnetic stimulation (TMS) (C) administration of a cathartic (D) administration of IV fluid to increase 28. A 2-year-old toddler refuses to move his left urine output arm and is holding it flexed at the elbow, with (E) no medical intervention is necessary the forearm pronated. His father had been swinging him by the forearms; there is no other Questions 25 through 27 history of trauma. Which of the following is the most likely diagnosis? A 41-year-old man reports that he washes his hands 50 times a day. In the evening, he will check the (A) fractured clavicle doors, windows, and stove at least a dozen times (B) Salter type IV fracture of the distal before retiring for the night. He is fearful of the humerus number 3; for example, he will not write out a check (C) subluxation of the radial head with a 3 in the number, and he will not stop his car (D) torus fracture of the distal radius if the odometer number ends in 3. (E) contusion of the ulnar nerve 25. Which of the following is the most likely diagnosis? Questions 29 and 30 (A) paranoid disorder A 25-year-old woman presents to the physician’s (B) paranoid schizophrenia office with a 2-week history of right breast pain. The pain is localized to the nipple areolar complex and (C) schizotypal personality disorder surrounding skin and is associated with redness (D) obsessive-compulsive disorder (OCD) and swelling. The pain does not change with the (E) presenile dementia menstrual cycle. On examination, vital signs are normal, and there is induration, erythema, and ten- 26. Of the following drugs, which is recom- derness involving the nipple areolar complex of the mended as a first choice to control this man’s breast, with a subareolar mass. Axillary examination symptoms? reveals slightly tender ipsilateral nodes. Ultrasound examination suggests the presence of a subareolar (A) fluvoxamine complex fluid collection. (B) alprazolam (C) buspirone 29. Which of the following is the most likely (D) zolpidem diagnosis? (E) lithium (A) inflammatory carcinoma of the breast (B) periductal mastitis with abscess (C) Paget’s disease of the nipple (D) Mondor’s disease (thrombophlebitis) (E) intraductal papilloma (c) ketabton.com: The Digital Library

316 10: Practice Test 4

30. Which of the following is the most appropriate (A) the level of systolic BP next step in management? (B) the level of diastolic BP (A) aspiration of the fluid and antibiotics (C) abnormalities of renal function (B) central lumpectomy, axillary node (D) future compliance dissection, and radiation therapy (E) heart size (C) incision and drainage (D) antibiotics alone with re-evaluation in 34. The number needed to treat (NNT) is increas- 1 week ingly being used to gauge the clinical effect of (E) excisional biopsy an intervention from clinical studies. Which of the following is a true statement regarding the use of the NNT? 31. A 14-year-old boy comes to the clinic for a checkup. His only concern is that he has devel- (A) A NNT >50 suggests that an interven- oped acne. On physical examination, he has tion is highly effective. moderate facial acne and is Tanner stage 3. (B) The NNT is calculated by taking the Which of the following is the best next step in his reciprocal of the absolute risk reduction management? for a treatment. (A) Advise him to wash his face vigorously (C) The NNT cannot be adapted to assess four times per day. the adverse effects of a therapy. (B) Advise him that it should resolve as he (D) The NNT can be used to compare differ- continues to progress through puberty. ent outcomes from the same therapies. (C) Ask him to decrease his intake of choco- (E) The NNT is calculated by taking the late, starches, and oily or fried foods. reciprocal of the relative risk reduction (D) Reassure him that no treatment is neces- for a treatment. sary unless he develops cystic acne. (E) Treat him with topical benzoyl peroxide 35. A 64-year-old woman is brought to your office and tretinoin. by her husband for evaluation of forgetfulness. She also has some word-finding problems and some paranoid delusions about the next-door Questions 32 and 33 neighbor. Which of the following is most spe- A 52-year-old patient is seen in a walk-in medical clinic cific for Alzheimer’s disease? complaining of a headache. On examination, his BP is (A) normal score on the Beck Depression noted to be 210/140 mmHg. His retina shows hemor- Inventory rhages and papilledema. Electrocardiogram shows left (B) poor score on the Folstein Mini-Mental ventricular hypertrophy (LVH). Blood urea nitrogen Status Examination (BUN) and creatinine are both elevated at 56 and 2.8. (C) mild cerebral atrophy on a brain CT scan 32. On what basis can the diagnosis of malignant (D) rapid, resting tremor hypertension be made? (E) decreased bilateral parietal lobe activity on positron emission tomography (PET) scan (A) the level of systolic BP (B) the level of diastolic BP 36. You are seeing a 63-year-old male patient in (C) headache your office with 3 days of explosive watery (D) funduscopic changes diarrhea. He also reports diffuse abdominal (E) heart size pain, fevers, loss of appetite, and nausea. Of note, he was recently discharged from a hospi- 33. Which of the following provides the best infor- tal where he spent a week being treated for mation regarding the prognosis of the malig- pneumonia with antibiotics. What is the most nant hypertension? likely organism responsible for his diarrhea? (c) ketabton.com: The Digital Library

Questions: 30–41 317

(A) Salmonella history is negative for hirsutism or oligomenorrhea. (B) Shigella Her BP is normal. She has no galactorrhea, and her (C) Clostridium difficile pelvic examination is normal except for a male pubic hair pattern. Her vagina contains rugae, and abundant (D) Staphylococcus aureus clear mucus is present in the cervical canal. (E) E. coli 39. Which of the following is the most appropriate 37. After a motor vehicle accident, your 49-year- initial diagnostic test? old male patient presents complaining of pain in his left lower leg. On examination, you note (A) a pelvic ultrasound bruising over his left calf and 1+ edema in his (B) measurement of serum 17- lower left leg. Compression ultrasonography hydroxyprogesterone immediately is ordered and is negative for (C) an ACTH stimulation test deep vein thrombosis (DVT). Which of the fol- (D) a dexamethasone suppression test lowing is the best next step in management? (E) measurement of serum follicle-stimulating (A) no follow-up needed hormone (FSH) (B) venography (C) CT scan of left leg 40. The patient’s serum 17-hydroxyprogesterone level is normal. In addition, her serum prolactin (D) referral to orthopedics concentration is 13 ng/mL (normal, <20). Which (E) repeat ultrasonography in 1 week of the following is the most likely diagnosis?

38. A 55-year-old woman presents to your clinic (A) polycystic ovary syndrome for follow-up. At her last visit with you, her (B) attenuated 21-hydroxylase deficiency blood pressure was 162/101 and you insti- (C) pituitary adenoma tuted therapeutic lifestyle changes and asked (D) Sertoli-Leydig cell tumor her to return for recheck of her blood pres- (E) adrenal adenoma sure. She has made the changes, reduced sodium, lost 3 lbs, and began following the 41. A hospital administrator is increasingly con- DASH (Dietary Approaches to Stop cerned by the rising number of patients who Hypertension) diet. On examination, you find have been diagnosed with methicillin-resistant her blood pressure 165/105 with a pulse of 88 staphylococcal aureus (MRSA) infections. You and note some increased vascular tortuosity have been asked to develop a program to and A-V nicking present on her opthalmo- reduce the occurrence of MRSA infections in scopic examination. What is the most appro- hospitalized patients. What is the most effective priate therapy to initiate? intervention to reduce the spread of MRSA (A) amlodipine/benazepril combination within the hospital? (B) continued lifestyle changes (A) Restrict the use of broad-spectrum (C) lisinopril antibiotics to only critically ill patients. (D) lisinopril/hydrochlorothiazide combination (B) Culturing all patients on admission to (E) atenolol ensure that they are not colonized with MRSA. Questions 39 and 40 (C) Treating all patients who are found to have MRSA with vancomycin. A 27-year-old woman complains of progressive facial (D) Ensuring that all health care providers hirsutism and menstrual intervals that vary from 26 to wash their hands before and after con- 90 days. Her hirsutism and oligomenorrhea began tact with patients. shortly after menarche at the age of 13 years. She takes no medications with androgenic effects. Her family (E) Isolating patients known to have MRSA in private rooms. (c) ketabton.com: The Digital Library

318 10: Practice Test 4

42. A 60-year-old man presents for follow-up to history significant for type II diabetes mellitus your clinic. He is 5 ft 10 in. tall and weighs and hypertension. Her medications include 255 lbs. You have had difficulty controlling his metformin, lisinopril, and oral contraceptive blood pressure. He currently takes a thiazide pills. She had been on all of her medications for diuretic, calcium channel blocker, and ACE over a year. She has never had this happen inhibitor. His blood pressure continues to be before and denies any new exposures to foods elevated at 155/95 with a pulse of 82. He does or consumer products. On examination, you admit to having headaches and daytime sleepi- note a comfortable lady in mild distress, breath- ness. Which of the following is the most likely ing easily. She does not have a fever. You note identifiable cause of resistant hypertension in asymmetric nontender left-sided lower facial this patient? and lip swelling. There is no rash. Her exami- nation is otherwise unremarkable. What is the (A) pheochromocytoma most likely cause of her swelling? (B) sleep apnea (C) hyperthyroidism (A) parotid gland inflammation (D) Cushing’s syndrome (B) angioedema (E) hyperthyroidism (C) DVT (D) sinus infection 43. A 26-year-old G1P0 woman presents to you for (E) siladenitis her first prenatal visit at 4 weeks’ gestation. The height and weight of the patient are meas- 45. What is the most likely cause? ured, and you calculate her body mass index (A) Streptococcus pneumoniae (BMI) to be 31. Which of the following recom- mendations concerning target weight gain for (B) anaerobic bacteria her pregnancy is most appropriate? (C) viral (mumps, influenza, para influenza) (D) lisinopril (A) You should limit weight gain to about 15 lbs. (E) oral contraceptive pills (B) You should limit weight gain to about 40 lbs. 46. You are evaluating a study describing the use of chest CT scans to detect early lung cancers. (C) You should roughly maintain your The study discovers that individuals who have prepregnancy weight. lung cancers detected at an early stage with (D) You should attempt to lose a moderate CT scans lived longer than those who did not amount of weight during the course of receive screening. The authors conclude that your pregnancy. screening with CT scans can increase survival (E) You should not worry about weight in lung cancer. This is an example of which gain or loss. kind of bias? (A) selection bias Questions 44 and 45 (B) lead-time bias 44. A 45-year-old lady presents to the emergency (C) enrollment bias department with the complaint of painless (D) recall bias facial swelling. It began yesterday and had (E) observational bias gradually worsened. She has a past medical (c) ketabton.com: The Digital Library

Answers and Explanations

1. (E) Hypertension before the third trimester, uter- 3. (D) Glucose intolerance is very common in ine size greater than dates, and the absence of a patients with cystic fibrosis, and about 40% of fetus with a detectable heartbeat after 6 post- adolescent and older patients will develop clin- menstrual weeks strongly suggests a diagnosis of ical diabetes mellitus. The diabetes associated gestational trophoblastic disease, most likely a with cystic fibrosis resembles maturity-onset hydatidiform mole. A multiple pregnancy is not diabetes more than juvenile diabetes mellitus, in a consideration because gestational sacs, each that ketoacidosis is rare, and satisfactory clini- containing a fetus, with cardiac activity in each cal control usually can be achieved with low- sac, would be seen by transvaginal ultrasonog- dose insulin. The sweat test is too labor raphy after 6 postmenstrual weeks. Although the intensive to be practical as a mass screening test absence of an intrauterine pregnancy is ultrasonic for the general population. However, because of evidence of a tubal pregnancy, the ectopic gesta- its accuracy, it is the diagnostic test of choice. tional sac is usually seen when the hCG concen- Pulmonary complications are serious and tration is 10,000 mIU/mL or higher. The presence also, unfortunately, very common. Cystic fibro- of a significantly enlarged uterus does not favor sis clearly is an autosomal-recessive disorder. a diagnosis of tubal pregnancy. Serum hCG con- Treatment programs aimed at symptomatic centrations are lower than expected in the case of control of pulmonary, gastrointestinal, and blighted ovum, and the uterus tends to be smaller other manifestations can significantly prolong than the expected gestational age. The elevated life, although they certainly do not amount to a hCG concentration, the hypertension, and cure. (Rudolph et al., pp. 1967–1979, 2114) the absence of a fetus with a heartbeat all rule against the diagnosis of a normal intrauterine 4. (C) Roseola infantum (also known as exan- pregnancy in a woman with a myomatous thema subitum) is a common disease of child- uterus. Management of this patient is evacuation hood. It is seen predominantly in infants of the uterus by suction curettage and serial hCG between the ages of 6 and 24 months and measurements for 6–12 months to be certain that typically causes temperatures as high as this woman does not subsequently develop chori- 38.9–40.5°C (102–105°F) for 3–5 days without ocarcinoma. (Scott et al., pp. 1019–1030) accompanying symptoms. Physical examina- tion is frequently normal but may reveal 2. (B) The most likely organism responsible for mild pharyngeal injection, suboccipital lym- this illness is B. cereus. Fried rice is the leading phadenopathy, or a bulging anterior fontanel. cause of B. cereus food poisoning in the United Defervescence then occurs, followed closely by States. B. cereus is frequently found in uncooked the development of a truncal maculopapular rice and heat-resistant spores may survive cook- rash, which fades after 2–3 days. It is most often ing. Symptoms characteristic for B. cereus- caused by human herpes virus-6. It is the induced food poisoning are the development of appearance of a rash after the cessation of fever abrupt, intense vomiting within 1–6 hours after that differentiates roseola infantum from other ingestion followed by watery diarrhea with an infectious exanthems. Measles is characterized incubation period of 6–24 hours. The illness is by a generalized red, blotchy rash in a febrile usually self-limited and not severe which likely individual with cough, coryza, conjunctivitis, leads to underreporting. (CDC web site, MMWR, and photophobia. The typical fine red papular 1994. Available at: http://www.cdc.gov/mmwr/preview/ rash of scarlet fever usually appears at the height mmwrhtml/00025744.htm) of fever. Kawasaki disease is characterized by

319 (c) ketabton.com: The Digital Library

320 10: Practice Test 4

conjunctivitis, oral lesions, cervical adenitis, and PMS, though possible, require further inves- fever for 5 days, palmar or plantar erythema, tigation, and from the information given are not and desquamation from the fingers or toes. The well-supported diagnoses. Sometimes clinicians, accompanying red rash is nonspecific. The rash in their own angry countertransference to such in erythema infectiosum is a bright red rash on patients, may “act out” by labeling the patient the cheeks (slapped cheek), followed by a retic- with these, since both terms can have a negative ular rash on the extremities. (Rudolph et al., pp. connotation. The nurses’ anger in this case is an 1029, 1221–1225) example of countertransference, which are feel- ings and attitudes of the therapist (medical per- 5. (D) Candida vaginitis is the only local infection sonnel) toward the patient that may arise from in which the vaginal pH is less than 4.5. the past or be a reaction to the patient’s transfer- Figure 10-1 shows the branching hyphae diag- ence feelings or attitudes toward the therapist. In nostic of candidal infection. The absence of clue this case, for example, if the countertransference cells and no amine odor excludes bacterial vagi- anger led to angry demands to medicate or to nosis due to G. vaginalis. Trichomonas vaginitis restrain this patient, we would also have an is due to a unicellular, flagellated organism that example of “acting out.” Although the consult- would demonstrate flagellar motion on a saline ing psychiatrist must be cognizant of the reason wet smear. Chlamydia and syphilis present for referral “to medicate,” he or she must quickly with no diagnostic vaginal discharge. (CDC determine what the real and most therapeutic MMWR 2006: Vol. 55, pp. 54–56) goal is. Given the case scenario, the psychiatrist needs to first determine why this woman is 6. (B) Either oral or vaginal antifungal agents are acting the way she is. One would presume that usually effective in eradication of Candida species once this woman is encouraged to talk about from the vagina. Metronidazole is used to treat her angry feelings, she will be able to get to the trichomoniasis or bacterial vaginosis. Ceftriaxone underlying fears, which might include fear of is the current treatment of choice for gonorrhea. mutilation, fear for her children, fear of losing Oral azithromycin is recommended as cother- her job or her husband’s love, and fear of losing apy with ceftriaxone to eradicate concurrent life itself. Appropriate medications may be indi- Chlamydia infection, present in up to 50% of cated to help this woman deal with her fears women with a positive gonococcal culture and anxieties. (Stoudemire, 1998b, pp. 3–35, 70–77) (even if the ligase chain reaction [LCR] is nega- tive for chlamydia). Oral or vaginal clindamycin 10. (D) is effective therapy for bacterial vaginosis, giving cure rates comparable to metronidazole. 11. (D) (CDC MMWR 2006: Vol. 55, pp. 54–56) 12. (D) 7. (C) 13. (B) 8. (C) Explanations 10 through 13 9. (D) Medullary carcinoma of the thyroid (MCT) Explanations 7 through 9 appears in three clinical settings: sporadic, as a component of multiple endocrine neoplasia, A common response to frightening news and a type IIa or type IIb (MEN IIa or MEN IIb). A possible life-threatening illness is regression—a family history of thyroid carcinoma with or return to an earlier form of behavior that is more without pheochromocytoma is invariably pres- childlike. It is an attempt to flee overwhelming ent as part of the MEN IIa syndrome. Patients anxiety and fear. It is a seeking to be cared for with MEN IIb have a marfanoid habitus and and protected. Borderline personality disorder characteristic facies with ganglioneuromas. (c) ketabton.com: The Digital Library

Answers: 5–18 321

Patients with MEN syndrome must be evalu- dependence who is at risk for the syndrome is ated for possible pheochromocytomas before to be given IV glucose, the IV glucose should treatment of the thyroid carcinoma. Total thy- include 100 mg of thiamine to each liter of the roidectomy is essential and represents the mini- glucose solution, or the condition may be wors- mal treatment in patients with MEN IIa disease. ened. Oral thiamine should be given after IV Total parathyroidectomy results in hypoparathy- thiamine is started. Lorazepam, diazepam, and roidism and should be avoided. Genetic screen- haloperidol will not prevent the progression ing for ret protooncogene mutations is highly of Wernicke-Korsakoff syndrome. (Kaplan and sensitive and specific. Prophylactic thyroidec- Sadock, p. 406) tomy is recommended for patients with proven mutations consistent with MEN IIa. (Brunicardi 17. (A) Postnasal drip syndromes (such as allergic et al., pp. 1423–1425; Greenfield et al., pp. 1303–1304; rhinitis and chronic sinusitis), asthma, gastroe- Townsend et al., pp. 971–972, 1075–1076) sophageal reflux, and chronic bronchitis (usu- ally in patients with a long smoking history) 14. (B) Recent estimates published by the U.S. are common causes of chronic cough. ACE Census Bureau report that in 2005, 44.8 million inhibitors may cause chronic coughing. With people or 15.3% of the population were without discontinuation of the drug, coughing should health insurance. In the United States, most improve or resolve within 4 weeks. (Kaspar et al., individuals receive health care through their pp. 205–206) employers. Due to rising health care premi- ums, smaller employers are less able to pro- 18. (E) A karyotype of peripheral lymphocytes is vide health care benefits to their employees. In the most appropriate option of the listed addition, increased reliance on part-time and choices. It will be 46,XX in this patient. The contract workers (who are usually not given normal height, normal breast, and pubic hair health care benefits) increases the percentage of development, absent cervix and uterus, and a individuals without health insurance. Young negative family history point to a diagnosis of adults (18–24 years old) are the age group least vaginal and Müllerian agenesis. In general, a likely to have health insurance with close to karyotype is a useful early step in the evalua- 31% not having coverage. (2005. U.S. Census tion of primary amenorrhea because 45,X Bureau, August 2006) gonadal dysgenesis is the most common cause of primary amenorrhea, and because several causes 15. (A) Generalized seizures frequently occur start- of primary amenorrhea are associated with the ing 12–24 hours after a patient with alcohol presence of a Y some. Apchromoproximately 30% dependence stops drinking. Although patients of women with a Y chromosome will develop a in alcohol withdrawal can have periods of gonadal malignancy by the age of 30 years. A lethargy, they generally alternate with periods pelvic ultrasound is unnecessary if the pelvic of hyperexcitability in patients with delirium examination is reliable in revealing the absence tremens. Patients with long-term severe alcohol of a cervix and uterus, but should be done if abuse are more likely to suffer from hypo- there is any doubt about the pelvic findings. glycemia, hyponatremia, and hypomagne- Breast development signifies the presence of semia, all of which can result in seizures, than estrogen, which means that the hypothalamic- from hyperglycemia, hypernatremia, or hyper- pituitary-gonadal axis is intact. A serum estra- magnesemia. (Kaplan and Sadock, pp. 403–405) diol or CT scan of the pituitary are, therefore, unnecessary. A laparoscopy is a reasonable 16. (B) Wernicke’s encephalopathy, which, if left procedure to remove the testes in a 46,XY untreated, can result in a chronic amnestic con- female with testicular feminization, but is an dition known as Korsakoff’s syndrome, is inappropriate and unnecessary diagnostic tool caused by thiamine deficiency in patients with for this patient. (Speroff and Fritz, pp. 419–423) alcohol dependence. If a patient with alcohol (c) ketabton.com: The Digital Library

322 10: Practice Test 4

19. (C) The absence of a uterus can only be due to 20. (E) Creation of the vagina will enable this Müllerian agenesis or testicular feminization woman to have satisfactory coitus, including (androgen insensitivity syndrome). The presence orgasm. Daily use of vaginal dilators of increas- of mature breasts indicates estrogen secretion ing length and diameter is a nonoperative means (unless there is a history of long-standing estro- of creating a vagina. This usually requires daily gen therapy); normal breast development is pres- use of the dilators for 6–12 months to achieve ent in both disorders. The presence of an adult adequate vaginal depth. The most common pubic hair pattern excludes testicular feminiza- operative method to create a vagina is to sew a tion because the deficiency or absence of andro- split-thickness skin graft taken from the buttocks gen receptors in the pubis will result in sparse or over a mold. The mold with covering skin graft absent pubic hair. Thus, the most likely diagno- is then placed into a space created between the sis is Müllerian agenesis. Because these women bladder and rectum. This is called the McIndoe have normally functioning ovaries, a test to procedure. Removal of the gonads is inappro- detect ovulation also establishes the diagnosis of priate, because they are normally functioning Müllerian agenesis: a biphasic basal temperature ovaries. Any form of hypertension is inappro- graph or a serum progesterone concentration priate for the same reason. Glucocorticoid over 5 ng/mL. Women with anorexia nervosa therapy is indicated only in women with cer- have deficient hypothalamic secretion of gonado- tain forms of congenital adrenal hyperplasia tropin-releasing hormone (GnRH) and second- (21-hydroxylase deficiency, 11-hydroxylase arily decreased secretion of FSH, luteinizing deficiency, 17α-hydroxylase deficiency). (Speroff hormone (LH), and ovarian estradiol and prog- and Fritz, pp. 420–421) esterone. The presence of breast growth indicates that this woman was exposed to estrogen at some 21. (A) Congenital adrenal hyperplasia may be time. The exposure may have been in the past caused by many different defects of steroido- and may have been from exogenous sources. genesis. The most common defect, 21- However, anorectic women have a cervix and hydroxylase deficiency, accounts for about 95% uterus. The normal height and the presence of of these disorders and is transmitted as an breasts excludes gonadal dysgenesis (e.g., Turner’s autosomal-recessive trait. These patients have syndrome), as these women are never taller than aldosterone deficiency, which results in 157.5 cm (62 in.), and the streak gonads are inca- hyponatremia, hyperkalemia, hypotension, pable of estrogen production. There is one excep- and shock. They also have glucocorticoid defi- tion to this height limit: women with 46,XX or ciency and, therefore, have hypoglycemia. The 46,XY gonadal dysgenesis will have a normal electrolyte abnormalities are the key to the height. However, women with any form of diagnosis. None of the other diseases listed gonadal dysgenesis (streak gonads) will have a would cause this degree of electrolyte abnor- uterus and cervix, even those with 46,XY gonadal mality. (Rudolph et al., pp. 2038–2042) dysgenesis. This is so because the gonads are so dysgenetic (literally translated: malformed) that 22. (A) The U.S. Preventive Services Task Force the streak gonads of a woman with 46,XY (USPSTF) currently has updated their guide- gonadal dysgenesis are so malformed that they lines for screening for AAA by recommending never produce Müllerian duct regression factor. one-time screening by ultrasonography in all Women with 17α-hydroxylase deficiency have male smokers aged 65–75. Risk factors for the hypertension and are sexually infantile, because development of an AAA include advanced age this steroid enzyme deficiency does not permit (>65 years), a history of smoking (>100 ciga- normal cortisol, androgen, or estrogen biosyn- rettes in a lifetime), a first-degree family history thesis. Deficient cortisol biosynthesis results in of AAA requiring surgical repair, and male sex. increased ACTH secretion, which stimulates an There are currently no recommendations for increased adrenal secretion of mineralocorticoids AAA screening for males who do not have a and results in hypertension. (Speroff and Fritz, history of smoking or for females. (USPSTF) pp. 419–423) (c) ketabton.com: The Digital Library

Answers: 19–28 323

23. (B) DDH, formerly called congenital disloca- Explanations 25 through 27 tion of the hip, is a relatively common problem, with a frequency of 1.5–10 per 1000 live births. Persons who have significantly disabling Genetic factors seem to have a major role, with obsessions (persistent, intrusive thoughts or increased risk occurring in children with an impulses) or exhibit disruptive compulsive affected parent or sibling. Girls are affected six behavior (stereotyped, purposeful, repetitive to eight times more often than are boys. behavior that is felt as no longer controllable by Neuromuscular disorders also predispose the the individual) are suffering from OCD. fetus to hip dysplasia. Uterine crowding and Obsessive thoughts can be distinguished from mechanical problems in the uterus appear to psychotic thoughts by the affected person’s predispose to DDH. For example, DDH is more recognition that these thoughts are their own common with breech presentation and oligo- and have not emanated from an external hydramnios. Type of delivery is not related to source. Affected persons typically display few cognitive deficits except those imposed by the the incidence of DDH. (Rudolph et al., pp. 2434–2436) concentration and attention demands of the OCD. Treatment of OCD can be multifaceted. 24. (D) Aspirin is well absorbed from both the Effective pharmacotherapeutic agents include stomach and small intestine. Removal from the the tricyclic antidepressants, especially body involves both hepatic and renal path- clomipramine, and the selective serotonin reup- ways. Treatment is indicated when the ingested take inhibitors (SSRIs), which include fluoxe- dose exceeds 100 mg/kg, and intensity of ther- tine, sertraline, paroxetine, citalopram, and apy is guided by the severity of the ingestion as fluvoxamine. These are the drugs of first estimated by serum salicylate levels in rela- choice. If these are partially effective, other tionship to time since ingestion. Therapeutic medications may be added. An anxiolytic (e.g., intervention is aimed at limiting absorption of buspirone) or a benzodiazepine (e.g., alprazo- aspirin from the gastrointestinal tract (effec- lam or clonazepam) may be tried, especially if tively accomplished by gastric lavage and anxiety is prominent. Zolpidem may be added administration of activated charcoal) and to improve sleep if there is a problem with enhancing renal excretion of the drug. insomnia as well. Lithium may be added for Maintenance of high urine flow aids rapid prominent affective symptoms such as depres- clearance and alkalinization of the urine results sion. Antipsychotics (e.g., risperidone) may in an ionized form of the drug that is poorly also be used in the event of psychotic symp- reabsorbed from the renal tubule, thus increas- toms. Behavioral therapy, designed to reduce ing excretion. N-acetylcysteine has no place in anxiety, also can be effective. The technique of the treatment of aspirin poisoning. It is, how- progressively delaying compulsive responses ever, quite effective in preventing the hepato- to anxiety or obsessional thoughts has been toxicity associated with severe acetaminophen helpful in many cases. Group therapy can be overdose. Induced emesis is rarely used in the useful in providing support, sharing successful emergency department because it tends to intervention techniques, and reducing the fear in delay the administration of activated charcoal. persons with OCD that they are crazy. TMS is Several studies have not shown any benefit a noninvasive technique for stimulating the from the addition of cathartics to charcoal in cerebral cortex with an electrical magnetic field the management of acute aspirin ingestions. without inducing seizures. TMS is currently (Ellenhorn, pp. 210–221) being studied for the treatment of depression. (Kaplan and Sadock, pp. 616–623, 1144) 25. (D) 28. (C) Subluxation of the radial head, also known 26. (A) as “nursemaid’s elbow,” is a very common con- dition of young children between the ages of 27. (B) 1 and 4 years. It is caused by sudden traction on (c) ketabton.com: The Digital Library

324 10: Practice Test 4

the forearm, resulting in dislocation of the sebum, which contains triglycerides, squalene, radial head from the capitulum of the humerus. wax esters, sterol esters, and phospholipids. The child holds the affected arm in flexion at Propionibacterium acnes, an anaerobic pleo- the elbow with the forearm pronated. X-rays morphic skin organism, produces a lipase that reveal no abnormalities. Treatment is by swift releases free fatty acids from the sebum mix- supination of the forearm; further treatment is ture, including highly irritating medium-chain usually not necessary with the first occurrence. triglycerides. Individuals prone to acne seem (McMillan et al., p. 1037) to have an increased turnover of abnormally cohesive keratinized cells in the sebaceous duct; 29. (B) these accumulate, plug the duct, and inhibit the release of sebum to the skin surface. Acne tends 30. (A) to persist until the late teens or early twenties. Too frequent or too vigorous face washing can Explanations 29 and 30 be irritating to the skin and actually worsen the acne. Diet seems to be of little importance in the The presentation of a painful swelling involving etiology of acne. The treatment goals are to the nipple areolar complex associated with decrease the risk of scarring and to alleviate the erythema and subareolar fluid in a young psychological stress during these critical years woman are the typical findings of periductal of social and sexual development. Adolescents mastitis and abscess. The organisms responsible with mild to moderate acne usually respond to for the disease are staphylococcal species and topical treatment. Therapy should be individu- anaerobes. The diagnosis can be confused with alized, but benzoyl peroxide and tretinoin is a inflammatory carcinoma, the presentation of reasonable combination for initial therapy. (Rudolph which includes peau d’orange, a solid mass, et al., pp. 1208–1210) and other findings that usually involve larger areas of skin of the breast rather than localized 32. (D) to the nipple areolar complex. Paget’s disease of the nipple is characterized by an eczematoid 33. (C) lesion; Mondor’s disease usually affects the peripheral breast and represents superficial Explanations 32 and 33 thrombophlebitis; and intraductal papilloma is usually nonpalpable and presents with a nipple Malignant hypertension is defined by the discharge without inflammatory changes. An presence of papilledema and end-organ abscess can be treated with aspiration and changes rather than the absolute BP level. antibiotics that cover both aerobic and anaero- Headache may not be the result of his ele- bic bacteria. Antibiotics alone would not be suf- vated BP, and heart size can be related to fac- ficient for an abscess, but can be used in patients tors other than hypertension, such as heart with periareolar inflammation without abscess failure and cardiomyopathy. Impairment of formation. Incision and drainage can be used, renal function provides the best index to prog- particularly if the abscess does not respond to nosis in malignant hypertension. The level of nonoperative measures, but the patient must systolic or diastolic BP and size of the heart are care for an open wound that may require weeks features that are readily reversible and do not for healing. Lumpectomy and excisional biopsy relate quantitatively to prognosis. A larger heart and more florid retinopathy do not nec- are not appropriate therapies. (Brunicardi et al., pp. 465–466; Greenfield et al., p. 1266) essarily portend a poorer prognosis, but numerous studies have documented that the 31. (E) Acne is a skin disorder with its onset in worse the renal impairment, the poorer the puberty, when testosterone derived from the prognosis. Compliance may play a role in BP gonads and adrenal glands stimulates sebaceous control, but is not reliable for predicting prog- gland activity. This results in the formation of nosis. (Kasper et al., p. 1478, 1480) (c) ketabton.com: The Digital Library

Answers: 29–39 325

34. (B) The NNT has been increasingly used in 36. (C) C. difficile is a spore-forming, gram-positive evidence-based medicine to assess the true clin- anaerobic bacillus that produces two exotoxins ical effects of studies in the literature. It is easily (toxin A and toxin B). C. difficile-associated diar- calculated by taking the reciprocal of the rhea is increasing in frequency and is the most absolute risk reduction. The NNT is clinically common cause of antibiotic-associated diar- significant because it gives a practitioner an rhea. Risk factors for developing C. difficile diar- idea of the effort required to achieve a particu- rhea include previous antibiotic exposure, lar clinical outcome. While in general, a small hospitalization, advanced age, and underly- NNT suggests an effective therapy, a large ing chronic medical conditions. (CDC web site, NNT can be clinically significant depending Overview of Clostridium difficile Infections, 2007. on the therapy and the outcomes being con- Available at: http://www.cdc.gov/ncidod/ dhqp/id_ sidered. Thus, there are no established cutoffs Cdiff.html) for determining an effective intervention. The NNT can also be useful in comparing the same 37. (E) Compression ultrasonography is the diag- outcomes for different interventions. The NNT nostic test of choice for assessment of DVT. can also be used to express the occurrence of Sensitivity and specificity are more than 95% adverse effects or harms of therapy and is for proximal DVT but sensitivity falls to around termed the number needed to harm or NNH. 70% for isolated DVT in the calf. Imaging of calf The NNH is calculated in a similar fashion veins often is not routinely performed. Given as the NNT using the reciprocal of the absolute the clinical suspicion, follow-up ultrasonogra- increase in risk in adverse events. (McQuay H, phy in 1 week is advisable to detect possible Moore R. Using numerical results from systematic extension from the calf into proximal veins. reviews in clinical practice. Ann Intern Med 1997; If the test is negative at 1 week, proximal 126(9):712–720. Available at: http://www. annals.org/ extension then is highly unlikely. (Kaspar et al., cgi/content/full/126/9/712) p. 691, 1491)

35. (E) Patients with Alzheimer’s disease often have 38. (D) Most patients with stage II hypertension symptoms that may mimic depression (lack of (systolic BP >160 or diastolic >100) require at enthusiasm, moodiness) along with their intel- least two drugs for control of their blood pres- lectual decline and, therefore, may score poorly on sure. In the absence of contraindications, one of a depression screening inventory. In early stages, these should be a thiazide diuretic such as they may be able to perform well on formal hydrochlorothiazide. Monotherapy for stage I mental status testing. CT and MRI scans are not hypertension should also be a thiazide diuretic specific for Alzheimer’s and may be normal early in the absence of compelling indications for in the course of the disease. The earliest metabolic other agents. (JNC 7, pp. 2560–2572) changes in Alzheimer’s occur in the parietal cortex and can be seen with PET scanning. Tremor is 39. (B) The initial evaluation of hirsutism should uncommon with Alzheimer’s, but is seen with include measurement of serum 17-hydrox- Parkinson’s. Pathologic features include brain yprogesterone, testosterone, and perhaps atrophy, neurofibrillary tangles, and granulo- dehydroepiandrosterone sulfate (DHEAS). vacuolar neuronal degeneration. Initial symp- 17-Hydroxyprogesterone tests for 21-hydroxylase toms generally begin after the age of 45 years, deficiency causing congenital adrenal hyper- with progressive memory loss leading to global plasia. Appropriate tests to evaluate her anovu- dementia. Some familial clustering is reported, latory menstrual cycles also include serum but most cases are sporadic. Choline acetyl- prolactin and thyroid function. A pelvic ultra- transferase activity is reduced, not increased, sound is appropriate if the bimanual part of and hallucinations and delusions are common. the pelvic examination is unsatisfactory due to Electroencephalogram (EEG) may be normal obesity or voluntary guarding. An ACTH stimu- or show nonspecific slowing. (Kasper et al., pp. lation test is appropriate only in those women 2393–2400) who may have congenital adrenal hyperplasia, (c) ketabton.com: The Digital Library

326 10: Practice Test 4

suggested by a positive family history of hir- infections, and pneumonia. MRSA infections sutism and oligomenorrhea, and significantly typically occur in patients who have undergone increased DHEAS levels. One milligram of dex- invasive medical procedures, are immunocom- amethasone at 11:00 p.m. should suppress the promised, or have chronic medical conditions next morning’s cortisol concentration to less such as end-stage renal disease. There are also than 5 g/dL. Failure to do so suggests the pos- increasing reports of community-acquired sibility of an adrenal tumor, and CT scan of the MRSA infections (typically skin infections) in adrenal glands should be done. Measurement otherwise healthy individuals. (MRSA in of FSH should be done only if the woman has Healthcare Settings) any menopausal symptoms. (Speroff and Fritz, pp. 504–513) 42. (B) In patients with resistant hypertension (hypertension poorly controlled on three or 40. (A) Polycystic ovary syndrome was first more drugs), it is important to look for identi- described in 1935 by Stein and Leventhal. It is fiable causes. Although pheochromocytoma, important to understand the features of the dis- hyperthyroidism, and Cushing’s syndrome are order as a syndrome that likely has multiple all identifiable causes of hypertension, they are causes. There is growing sentiment to discard relatively uncommon, and his history of the terms polycystic ovary syndrome and headaches and daytime sleepiness suggest Stein-Leventhal syndrome in favor of a more sleep apnea. Other identifiable causes include descriptive term, such as chronic estrogenic hyperaldosteronism, renal artery stenosis, and anovulation. In this way, the disorder can more coarctation of the aorta. (JNC 7, pp. 2560–2572) easily be recognized as one with a spectrum of causes and clinical manifestations. At least 75% 43. (A) The Institute of Medicine Committee on of women with chronic estrogenic anovulation Nutritional Status during Pregnancy and will have what has been called polycystic ovary Lactation indicates that women who are over- syndrome. The normal 17-hydroxyprogesterone weight or obese at the onset of pregnancy concentration effectively excludes attenuated should be advised to gain less total weight 21-hydroxylase deficiency and an adrenal ade- during the pregnancy. The appropriate weight noma. The normal prolactin concentration gain for a woman with a prepregnancy BMI tends to exclude a pituitary adenoma, although over 29 is roughly 13–18 lbs. The fetus, some pituitary tumors do not appear to secrete expanded blood volume, uterine enlargement, any hormones. The history of oligomenorrhea breast tissue growth, and other products of and hirsutism since puberty, plus the normal conception generate an estimated 13–17 lbs. bimanual examination, makes a Sertoli-Leydig Because weight gain during pregnancy is not cell tumor unlikely. Abrupt cessation of based solely on these tissues, acceptable weight menses, followed by the onset of hirsutism plus gain in women with normal BMIs may be unilateral ovarian enlargement, is the usual greater. (Cunningham et al., 2005, pp. 126–127, 213–219) presentation of a virilizing ovarian tumor. (Speroff and Fritz, pp. 470–476) 44. (B)

41. (D) The main mode of transmission of MRSA is 45. (D) through health care providers’ hands. Providers’ hands can become colonized with MRSA and if Explanations 44 and 45 appropriate hand hygiene (through washing with soap and water or using an alcohol-based The patient above presents with a history hand sanitizer) is not performed prior to patient and physical examination most consistent contact, MRSA can be transmitted to other with angioedema. Angioedema related to ACE patients. MRSA infections in health care settings inhibitors usually does not involve urticaria or can cause potentially life-threatening infections rash as it is not medicated via mast cell activa- including bloodstream infections, surgical site tion. Inhibition of ACE results in increased (c) ketabton.com: The Digital Library

Answers: 40–46 327

levels of bradykinin causing vasodilatation and ural course of disease. Selection bias refers to increasing vascular permeability. Most cases differences in individuals who choose to enroll occur within 7 days of initiation of the ACE in studies compared to those who do not. inhibitor but a substantial number occur months Enrollment bias refers to the different assign- to years later. (Kostis et al., pp. 1637–1642) ment of patients to separate arms of a study. Recall bias occurs when individuals are more 46. (B) Lead-time bias occurs when screening tests likely to remember certain exposures or events identify diseases at an earlier stage which when they have a disease. Observational bias increases the time from diagnosis to death. This occurs when study participants may change can make screening tests appear to prolong their responses to questions or behavior if they survival when they are merely advancing the know they are being observed. (Wallace and time of diagnosis and are not altering the nat- Doebbeling, p. 19) (c) ketabton.com: The Digital Library

BIBLIOGRAPHY

Brunicardi FC, Andersen DK, Billiar TR, et al., eds. McMillan JA, DeAngelis CD, Feigin RD, et al. Oski’s Schwartz’s Principles of Surgery, 8th ed. New York, Pediatrics: Principles and Practice, 4th ed. NY: McGraw-Hill, 2005. Philadelphia, PA: JB Lippincott, 2006. CDC web site, MRSA in Healthcare Settings, 2007. Rudolph CD, Rudolph AM. Pediatrics, 21st ed. New Available at: http://www.cdc.gov/ncidod/dhqp/ York, NY: McGraw-Hill, 2003. ar_MRSA_spotlight_2006.html. Scott JR, Gibbs RS, Karlan BY, et al., eds. Danforth’s Centers for Disease Control and Prevention. MMWR Obstetrics and Gynecology, 9th ed. Philadelphia, PA: Morbidity and Mortality Weekly Report. Recom- Lippincott Williams & Wilkins, 2003. mendations and Reports. Sexually Transmitted Speroff L, Fritz MA. Clinical Gynecologic Endocrinology Diseases Guidelines, Vol. 55, No. RR-11. August 4, and Infertility, 7th ed. Philadelphia, PA: Lippincott 2006, pp. 54–56. Williams & Wilkins, 2005. Cunningham FG, Leveno KJ, Bloom SL, et al. Staheli LT. Fundamentals of Pediatric Orthopedics, 2nd ed. Williams Obstetrics, 22nd ed. New York, NY: Philadelphia, PA: Lippincott Raven, 1998. McGraw Hill, 2005. Stoudemire A, ed. Human Behavior: An Introduction Ellenhorn MJ. Ellenhorn’s Medical Toxicology: Diagnosis for Medical Students. Philadelphia-New York: and Treatment of Human Poisoning, 2nd ed. Baltimore, Lippincott-Raven, 1998. MD: Williams & Wilkins, 1997. The 7th Report of the Joint National Committee (JNC 7) Greenfield LJ, Mulholland M, Lillemoe KD, et al., on the Prevention, Detection, Evaluation, and Treatment eds. Surgery: Scientific Principles and Practice, 4th ed. of High Blood Pressure. NIH Publication No. 03-5233, Philadelphia, PA: Lippincott-Raven, 2005. May 2003. Income, Poverty, and Health Insurance Coverage in Townsend CM Jr, Beauchamp RD, Evers BM, et al., the United States: 2005. U.S. Census Bureau, eds. Sabiston Textbook of Surgery: The Biologic Basis of August 2006. Modern Surgical Practice, 17th ed. Philadelphia, PA: Kaplan HI, Sadock BJ. Synopsis of Psychiatry: Behavioral W.B. Saunders, 2004. Sciences/Clinical Psychiatry, 9th ed. Baltimore, MD: USPSTF, The Guide to Clinical Preventive Services, 2006. Williams & Wilkins, 2003. Wallace RB, Doebbeling BN, eds. Maxcy-Rosenau-Last Kostis JB, Kim HJ, Rusnak J, et al., Incidence and Textbook of Public Health & Preventive Medicine, Characteristics of Angioedema Associated with 14th ed. Stamford, CT: Appleton & Lange, 1998. Enalapril. Arch Intern Med 2005 Jul 25; 165(14): 1637–42.

328 (c) ketabton.com: The Digital Library

Subject List: Practice Test 4

Question Number and Subject 23. Pediatrics 24. Pediatrics 1. Obstetrics-gynecology 25. Psychiatry 2. Preventive Medicine 26. Psychiatry 3. Pediatrics 27. Psychiatry 4. Pediatrics 28. Pediatrics 5. Obstetrics-gynecology 29. Surgery 6. Obstetrics-gynecology 30. Surgery 7. Psychiatry 31. Pediatrics 8. Psychiatry 32. Internal Medicine 9. Psychiatry 33. Internal Medicine 10. Surgery 34. Internal Medicine 11. Surgery 35. Internal Medicine 12. Surgery 36. Internal Medicine 13. Surgery 37. Internal Medicine 14. Preventive Medicine 38. Internal Medicine 15. Psychiatry 39. Obstetrics-gynecology 16. Psychiatry 40. Obstetrics-gynecology 17. Internal Medicine 41. Preventive Medicine 18. Obstetrics-gynecology 42. Internal Medicine 19. Obstetrics-gynecology 43. Obstetrics-gynecology 20. Obstetrics-gynecology 44. Internal Medicine 21. Pediatrics 45. Internal Medicine 22. Preventive Medicine 46. Preventive Medicine

329 (c) ketabton.com: The Digital Library

This page intentionally left blank (c) ketabton.com: The Digital Library

CHAPTER 11 Practice Test 5 Questions

1. Patients with end-stage renal disease who 4. Magnesium sulfate may be the preferred treat- receive dialysis are typically covered by what ment in this patient if she is not at least at 37 form of insurance? weeks’ gestation, and there is no evidence of fetal distress. What is the purpose of magne- (A) Medicare sium sulfate? (B) Medicaid (C) Private Insurance (A) tocolysis to prevent preterm labor (D) Due to the high costs associated with (B) decrease her blood pressure into the dialysis, most insurance companies are normal range not able to cover dialysis patients (C) reduce the risk of eclampsia (E) Blue Cross/Blue Shield (D) increase uteroplacental blood flow (E) prevent fetal hypertension Questions 2 through 4 5. A 70-year-old man with history of hyperlipi- A 19-year-old primigravida is at 39 weeks’ gesta- demia presents to your clinic for follow-up. tion. Her prenatal course had been normal since her The goal of treatment is to reduce the systolic first visit at 9 weeks’ gestation. Her blood pressure blood pressure to less than 140 mm Hg. On a is now 144/96 mmHg. She has 2+ proteinuria. Her previous visit you noted a blood pressure of patellar reflexes are hyperactive. 165/80 and a pulse of 60. Based on your advice he had been checking his blood pressure at 2. Which of the following is the most likely home with a reliable machine and it has diagnosis? remained in this range. At his visit today he is (A) acute glomerulonephritis 165/85. He feels well and his examination is (B) essential hypertension otherwise normal. He already eats a diet low in salt and with plenty of fruits and vegetables (C) pheochromocytoma and walks daily. What do you recommend (D) preeclampsia regarding his blood pressure? (E) polycystic kidneys (A) Begin low-dose diuretic therapy with 3. Which of the following is the most appropriate follow-up in 1 month. treatment of this patient? (B) Do nothing. (C) Begin an evaluation for secondary (A) bed rest causes of hypertension. (B) oral magnesium sulfate (D) Begin therapy with a beta–blocker. (C) a thiazide diuretic (E) Begin therapy with an alpha–blocker. (D) propranolol (E) induction of labor

331

Copyright © 2008 by The McGraw-Hill Companies, Inc. Click here for terms of use. (c) ketabton.com: The Digital Library

332 11: Practice Test 5

DIRECTIONS (Questions 6 through 12): For each a moderate amount of alcohol. Examination numbered item, select the ONE best lettered reveals pulse rate of 90/min, BP of 105/65 option that is most closely associated with it. Each mmHg, and unremarkable heart and lung find- lettered option may be selected once, more than ings. She has fullness and mild tenderness in once, or not at all. the right lower quadrant. Blood tests reveal a hematocrit of 21. For each patient with gastrointestinal bleeding, select the most likely diagnosis. 9. A 65-year-old man is transferred to the emer- (A) diverticulosis of the colon gency department from an outside facility for (B) ulcerative colitis evaluation of hematemesis and melena. Past his- tory is pertinent for recently discovered cholelithi- (C) Meckel’s diverticulum asis, for which he has not yet received a surgical (D) ischemic colitis consultation. He has complained of abdominal (E) aortoenteric fistula pain for the last 3–4 weeks and has been taking (F) peptic ulcer ibuprofen four times daily for 3 weeks as well as (G) carcinoma of the colon alcohol for the pain. Examination reveals a pulse (H) esophageal varices rate of 100/min and BP of 100/60 mmHg fol- (I) gastric ulcer lowing resuscitation. Abdominal examination reveals tenderness in the right upper quadrant 6. A 60-year-old man is brought to the emergency and epigastrium. Rectal examination reveals department following a fainting episode at melanotic stool. home. He complains of bright red blood per rectum for the last 6 hours. He denies abdom- 10. A 50-year-old man presents to the emergency inal pain or vomiting. Examination reveals a department following an episode of hemateme- pulse rate of 110/min; BP of 95/50 mmHg; and sis. Past history is pertinent for a 40 pack-year no significant heart, lung, or abdominal find- smoking history and chronic alcoholism. ings. He has gross blood on rectal examination. Examination reveals a pulse rate of 120/min, BP of 90/60 mmHg, and an unremarkable 7. A 56-year-old woman presents to the emer- heart and lung examination. Skin examination gency department following an episode of reveals spider angiomata and palmar ery- melanotic stool 48 hours ago. She underwent thema. Abdominal examination reveals a non- resuscitation at another facility before transfer. tender abdomen that is distended. She has no current complaints. The melena has not recurred. Past surgical history is pertinent 11. A 35-year-old man presents to the emergency for appendectomy at age 25 and abdominal department with abdominal pain, hematemesis, aortic aneurysm repair 2 years ago. She has a 30 and black, tarry stools. He complains of abdom- pack-year smoking history and drinks alcohol inal pain for 2 weeks, for which he has taken socially on weekends. Upper gastrointestinal antacids with partial relief. Past history is perti- (UGI) endoscopy to the duodenal bulb is nent for hypertension, 20 pack-year smoking his- normal, as well as colonoscopy. tory, and occasional alcohol intake. Examination reveals a pulse rate of 110/min, BP of 95/60 8. A 65-year-old woman presents to the emergency mmHg, tenderness in the epigastrium, and department complaining of severe fatigue and melanotic stool on rectal examination. weakness over the last week. Past history is per- tinent for hypertension, diabetes, and coronary 12. A 35-year-old woman presents to the emergency artery disease. Medications include a calcium department with a 1-week history of bloody channel blocker, insulin, and daily aspirin. She diarrhea. She complains of intermittent abdom- has a 40 pack-year smoking history and drinks inal pain for 1 month that is now constant and (c) ketabton.com: The Digital Library

Questions: 6–18 333

weight loss of 10 lbs. Past history is unremark- (C) chlorpromazine able. Examination reveals a temperature of (D) benztropine 101°F, pulse rate of 85/min, and BP of 110/60 (E) lithium carbonate mmHg. Heart and lung examinations are unre- markable. Abdominal examination reveals dif- 16. A 17-year-old female is brought to the emer- fuse abdominal tenderness. gency department because she ingested vita- mins that contained iron. Which of the following Questions 13 and 14 symptoms you might see early in the course of A 39-year-old multiparous woman has a retained acute iron poisoning? placenta 60 minutes after the vaginal birth of a 3650-g (A) bleeding diathesis healthy boy. There was no episiotomy and no lacer- (B) gastrointestinal hemorrhage ations of her perineum, vagina, or cervix. She now (C) liver failure has profuse vaginal bleeding and her blood pres- sure is 80/50 mmHg with a pulse rate of 120/min. (D) prolonged QT interval Her uterine fundus is firm. (E) respiratory failure

13. Which of the following is the probable etiology 17. A 12-year-old girl with malaise, fatigue, sore of her postpartum hemorrhage? throat, fever, hepatosplenomegaly, and gener- alized lymphadenopathy is diagnosed as (A) cervical laceration having Epstein-Barr virus-related mononucle- (B) uterine atony osis. Which of the following complications is (C) Couvelaire uterus most likely in this patient? (D) acute thrombocytopenia (A) azotemia (E) placenta accreta (B) chronic active hepatitis (C) encephalitis 14. Which of the following is the most appropriate treatment to control the hemorrhage? (D) lymphoproliferative disease (E) pancreatitis (A) transfusion with whole blood (B) uterine packing 18. All of the following are considered coronary (C) intravenous pitocin heart disease (CHD) risk equivalents with a (D) uterine artery embolization 10-year risk of CHD of greater than 20% and (E) intravenous administration of necessitating a low-density lipoprotein (LDL) methylergonovine cholesterol goal of less than 100 except which one? 15. A 26-year-old man presents to the emergency (A) diabetes mellitus room with complaints of seeing and feeling (B) abdominal aortic aneurysm bugs crawling on him (there are none per your (C) peripheral arterial disease thorough examination). He is tremulous, diaphoretic, disoriented, and his pulse rate is (D) cigarette smoking 120/min with a BP of 200/100 mmHg. Which (E) symptomatic carotid disease of the following psychotropic drugs is he most likely withdrawing from? (A) amitriptyline (B) alprazolam (c) ketabton.com: The Digital Library

334 11: Practice Test 5

19. A 60-year-old woman presents to your clinic for (A) An estimated 15–20% of boys and 2–9% follow-up care. She has a history of high blood of girls under age 18 have the disorder. pressure, for which she takes medication, and (B) Low levels of plasma dopamine beta- cigarette smoking. In anticipation of this visit hydroxylase have been implicated in she had had fasting laboratory tests done some children with this disorder. which reveal the following lipid profile: total (C) There is no correlation between parental cholesterol 230, triglycerides 150, high-density alcoholism and the emergence of this lipoprotein (HDL) 35, LDL 165. Her blood pres- disorder. sure is 135/88. She has already instituted ther- (D) Attention deficit disorder cannot, by apeutic lifestyle changes and her transaminases definition, coexist with this disorder. are normal. She is afraid to take medications that might “hurt her liver.” Which of the fol- (E) The average age of boys is the same as lowing do you recommend? that of girls at the onset of this disorder. (A) Advise niacin, reassuring her that it will not hurt her liver. 22. On January 1, 2006, Medicare began to offer (B) Advise simvastatin after reviewing risk prescription drug coverage for Medicare of hepatotoxicity. enrollees. Which of the following descriptions of the program is correct? (C) Advise Ezetimibe reassuring her that it will not hurt her liver. (A) Prescription drug coverage is only avail- (D) Advise fish oil and continued dietary able to qualified individuals with intervention. incomes below the poverty level. (E) Advise gemfibrozil. (B) Prescription drug coverage is available to qualified individuals who need Questions 20 and 21 expensive, brand name medications. (C) Prescription drug coverage is available A 10-year-old boy is brought to your office with a to everyone with Medicare who enrolls 3-year history of behavioral problems. Last week, in the program and pays a monthly pre- he was suspended from school for bullying kids at mium for the coverage. the bus stop. Last year, he killed his family’s cat by (D) The government provides coverage for putting it in the dryer. His mother reports he ran prescription drugs for certain Medicare away from home twice last year, each time for enrollees. more than 2 days. He was previously arrested for burning down a warehouse. His mother says hav- (E) All Medicare prescription drug coverage ing him born was a mistake. The boy reports his is the same for all enrollees. mood is “good.” 23. A 62-year-old man with a past medical history 20. Which of the following is the most likely of only hypertension presents to the emergency diagnosis? department with a 2-week history of worsening shortness of breath and palpitations. He denies (A) antisocial personality disorder chest pain. He does report orthopnea and (B) oppositional defiant disorder paroxysmal nocturnal dyspnea. On examina- (C) no axis I disorder tion, you note a blood pressure of 105/70 and (D) Tay-Sachs disorder a pulse of 145. His jugular venous distention (JVD) is elevated at 14 cm. He has crackles 1/3 (E) conduct disorder of the way up in both lung fields. His heart examination reveals tachycardia with an irreg- 21. Which of the following statements is correct ularly irregular rate and no murmurs. You regarding this disorder? diagnose congestive heart failure and order an electrocardiogram (ECG), which reveals the following: (c) ketabton.com: The Digital Library

Questions: 19–27 335

V1 25. Which of the following is the most appropriate initial step in management? (A) sedation with IV midazolam (B) IV morphine for analgesia (C) administration of 100% oxygen by face mask (D) debridement of the lower-extremity burns What diagnosis does this ECG suggest? (E) IV fluid bolus of 20 mL/kg (A) atrioventricular (AV) nodal re-entrant 26. The patient improves and is transferred to the tachycardia intensive care unit for further management. (B) atrial flutter Which of the following is the appropriate initial (C) sinus arrhythmia fluid resuscitation? (D) atrial fibrillation (A) IV dextrose/0.5 normal saline at 150 mL/h (E) multifocal atrial tachycardia (B) IV lactated Ringer’s at maintenance rate per hour, with boluses of albumin as 24. While you are administering oxygen and draw- required ing laboratory findings, the patient develops (C) IV lactated Ringer’s at twice mainte- chest pain, and follow-up blood pressure is nance rate per hour noted to be 70/50. Your next step is to do which (D) 4800 mL of lactated Ringer’s, given at of the following? 200 mL/h (A) Administer a beta-blocker such as (E) IV maintenance fluids plus an addition- metoprolol IV. al 4800 mL of lactated Ringer’s, one-half (B) Administer amiodarone IV. given over the first 8 hours and the (C) Perform immediate cardioversion remainder given over 16 hours followed by heparin. (D) Administer procanamide IV. 27. Within several hours of admission, the patient begins to complain of pain in the right foot. He (E) Start an IV nitroglycerine drip. is noted to have swelling of the foot and ankle, decreased capillary refill of the right toes, and Questions 25 through 27 decreased sensation. Which of the following is A 22-year-old, previously healthy, 176-lb man was the treatment of choice? rescued from a house fire, after being trapped in a (A) elevation of the affected extremity small bedroom of the house for several hours. When (B) initiation of antibiotics brought to the emergency department, he is noted to be combative and disoriented. His lungs are clear (C) hyperbaric oxygen to auscultation bilaterally. His respiratory rate is (D) administration of 100% oxygen by face 30/min, BP 100/70 mmHg, and heart rate 115/min. mask He has sustained 15% second- and third-degree (E) escharotomy burns on the lower extremities, with a circumferen- tial full-thickness injury below the right knee. (c) ketabton.com: The Digital Library

336 11: Practice Test 5

28. A 77-year-old male is visiting you for an annual (D) Advise him that smoking makes it checkup. He has a history of mild hypertension about 15 times more likely that he and elevated cholesterol but otherwise does will develop cancer of the lung. not have any other chronic medical conditions. (E) Advise him that he should quit Six months ago he was hospitalized for 3 days smoking now. with a community-acquired pneumonia. He would like to review his vaccinations during 30. A worker complains of paresthesias, numb- this visit. You note that he has received an ness, and tingling that started distally in the annual influenza vaccine in the fall, had a lower extremities but now is starting to affect tetanus booster 2 years ago, and received a his hands. He is developing muscle weakness. pneumococcal vaccine 10 years ago. What is Electromyographic abnormalities suggest the most appropriate statement about his vac- axonal degeneration and demyelination. Which cination status with respect to the pneumococ- exposure is the most suspect cause of this clin- cal vaccine? ical picture? (A) He should receive the pneumococcal (A) lead vaccination now as this should be (B) n-hexane repeated every 5 years. (C) benzene (B) He does not need any further pneumo- (D) cadmium coccal vaccinations as he received one dose after the age of 65. (E) carbon disulfide (C) He should receive the pneumococcal 31. A 55-year-old lady with a history of hyperten- vaccination now as this should be sion presents for follow-up to your clinic. She repeated every 10 years. takes 100 mg of atenolol for her hypertension. (D) He should receive a repeat pneumococ- Her blood pressure is 130/70. She feels well. cal vaccination today because he has On physical you note a regularly irregular hypertension and elevated cholesterol. heartbeat. Her physical is otherwise normal. (E) He should receive a repeat pneumococ- ECG is done and reveals type I second-degree cal vaccination today as he was recently heart block (Wenckebach). The ECG is other- hospitalized for pneumonia. wise normal with a heart rate in the 70s. What is the next most appropriate step? 29. A 40-year-old man calls your office asking for a chest x-ray. He is worried because a friend (A) Admit to the hospital for pacemaker was just diagnosed with lung cancer. Both he placement. and his friend smoke two packs of cigarettes a (B) Admit to the hospital to rule out day and have chronic coughs. He admits he myocardial infarction. seldom goes to the doctor unless he has a seri- (C) Order stress test with myocardial ous injury. You order an x-ray, and it is normal. imaging. Which of the following is the most appropriate (D) Reduce the dose of atenolol to 50 mg course of action at this point? and schedule a follow-up ECG. (A) Advise the patient that he really needs a (E) Refer her to cardiology for pacemaker computed tomography (CT) scan, since placement. x-rays can miss early cancers. (B) Reinforce his concern by showing him 32. A 50-year-old woman presents with weakness pictures of victims of lung cancer and of her right hand, accompanied by a tingling emphysema. sensation on awakening each morning. She is found to have hypesthesia of the thumb, index, (C) Assess his readiness to quit smoking. and middle fingers, along with atrophy of the (c) ketabton.com: The Digital Library

Questions: 28–37 337

thenar muscles. Which of the following is the Questions 35 and 36 most likely diagnosis? A 26-year-old, previously healthy man is brought to (A) compression of the digital nerve to the the emergency department with a stab wound to thumb the fifth left intercostal space in the midclavicular (B) compression of the median nerve in the line. He is found to have a pulse of 140/min, sys- hand tolic BP of 80 mmHg, and respiration of 20/min. His (C) compression of the median nerve in the trachea is midline, heart sounds appear distant, and carpal tunnel breath sounds are equal bilaterally. (D) compression of the median nerve by the pronator teres 35. Which of the following is the most likely diagnosis? (E) compression of the median nerve in the axilla (A) transected descending aorta (B) cardiac tamponade 33. Which of the following treatments for conges- (C) massive left hemothorax tive heart failure is not associated with pro- (D) tension pneumothorax longed survival? (E) phrenic nerve paralysis (A) digoxin (B) beta-blockers 36. Which of the following is the most appropriate (C) angiotensin-converting enzyme (ACE) next step in management? inhibitors (A) insertion of a left chest tube (D) spironolactone (B) emergency department thoracotomy (E) angiotensin receptor blockers (C) pericardiocentesis (D) rapid infusion of an IV fluid bolus 34. You are working in a student health center and seeing a member of the track team for a phys- (E) intubation and assisted ventilation ical. The history is unremarkable but on exam- ination you note a systolic murmur that begins 37. A muscular 25-year-old man presents to the well after the first heart sound. You ask the emergency room with pressured speech, patient to perform a Valsalva maneuver and decreased need for sleep, grandiosity, hyper- note that the murmur increases in intensity. sexuality, and racing thoughts worsening over What is the next most appropriate step? 2 weeks. He has no family history of mental ill- ness, but you find out he has been taking a (A) Clear the student for participation in medication he obtained illegally for the past sports as this is innocent murmur. month. Which one of the following medica- (B) Obtain an ECG and if normal clear the tions is the most likely cause of this patient’s student for participation. symptoms? (C) Prohibit participation in sports and (A) morphine order an echocardiogram. (B) cannabis (D) Ask if the student has a history of (C) alprazolam rheumatic fever. (D) anabolic steroid (E) Advise that the student does not need endocarditis prophylaxis for dental (E) lithium carbonate procedures. (c) ketabton.com: The Digital Library

338 11: Practice Test 5

Questions 38 and 39 39. You return to reassess the patient and while palpating his radial pulse you note that it dis- 38. A 45-year-old man presents to the emergency appears on inspiration and returns with expi- department with a complaint of chest pain, ration. What is the next most appropriate step? fatigue, and dyspnea. He was well until 2 days prior to this visit when he noted the onset of (A) urgent echocardiogram and considera- sharp, substernal chest pain radiating to the tion of pericardiocentesis left arm and back. The pain has gotten pro- (B) urgent cardiac catheterization gressively worse and is increased when he (C) placement of a chest tube coughs. He denies any past medical history (D) stat cardiothorasic surgery consultation and takes no medications. Examination reveals for repair of dissecting aneurysm a patient in mild distress. His blood pressure is (E) consideration of thrombolytic therapy 88/50, pulse 97, respiratory rate 20. His chest is for large saddle pulmonary embolism clear and you hear what sounds like a scratchy systolic murmur. ECG reveals ST-segment 40. Thromboangiitis obliterans (Buerger’s disease) elevation in the limb leads and V -V . His 2 6 is characterized by which of the following? chest x-ray is displayed in Figure 11-1. (A) typically occurs in smokers (B) occurs most often in young females (C) typically affects large arteries (D) noninvolvement of veins (E) highest worldwide prevalence in the southern United States

41. A 72-year-old lady with a history notable for pre- viously well-controlled hypertension is admit- ted to the hospital for evaluation of severe hypertension and a rise in her serum creati- nine from her usual baseline of 1 to 2.2. She takes only hydrocholorothiazide 25 mg. Your evaluation reveals a blood pressure of 175/105 and a pulse of 88. Lungs are clear and her car- diac examination is notable for left ventricular lift. Urinalysis revealed a bland sediment. While auscultating her abdomen you note the presence of a bruit periumbilically. While talk- ing with her she begins to cough and over the course of 10–15 minutes becomes dyspenic and FIG. 11-1 CXR. (Reproduced, with permission, from Kasper DL et al (eds). Harrison's Principles of Internal Medicine, 16th Ed. New hypoxic. Follow-up examination reveals bilat- York: McGraw-Hill, 2005:1345) eral rales and chest x-ray confirms pulmonary edema. What is the most likely cause of this What is the most likely diagnosis? patient’s presentation? (A) acute myocardial infarction (A) acute glomerulonephritis (B) dissecting aortic aneurysm (B) systemic lupus erythematosus (C) acute pericarditis (C) renal artery stenosis (D) acute pulmonary embolism (D) hyperaldosteronism (E) pneumothorax (E) hypothyroidism (c) ketabton.com: The Digital Library

Questions: 38–46 339

42. A 31-year-old White woman is pregnant at 31 (C) multiple endocrine neoplasia weeks’ gestation. She and her husband just type I (MEN I) read an article in Good Housekeeping maga- (D) multiple endocrine neoplasia type IIa zine about cystic fibrosis (CF) and ask you what (MEN IIa) their chances are of having a child with CF. (E) parathyroid carcinoma Neither have been tested for the CF gene. You know that the gene frequency in a White pop- 45. You screen a 30-year-old woman for chlamydia ulation is approximately 1 in 25. You tell them with a nucleic acid amplification test that has a that the probability of having a child with CF is sensitivity of 90% and a specificity of 95%, and approximately the result comes back positive. What is the like- (A) 0, because neither of the couple has lihood that she has chlamydia? symptoms of CF (A) You cannot tell without knowing what (B) 1 in 50 the likelihood is that other women like (C) 1 in 625 her have chlamydia. (D) 1 in 1350 (B) There is about a 95% chance that she has it. (E) 1 in 2500 (C) There is about a 5% chance that she has it. (D) You cannot tell without knowing the 43. All of the following are reasons to consider reproducibility of the test. aortic valve replacement (AVR) in a patient (E) You cannot tell without obtaining the with severe aortic stenosis except which one? results of a polymerase chain reaction (A) syncope (PCR) test to make a definitive diagnosis. (B) congestive heart failure 46. A 17-year-old girl has moderate dysmenorrhea (C) onset of angina that began about 1 year after menarche. 2 (D) reduction of valve area to less that 0.5 cm Physical examination, including the pelvic (E) declining left ventricular function examination, is normal. Which of the following medications most often provides effective relief 44. A 65-year-old woman presents to the physi- for her primary dysmenorrhea? cian’s office for evaluation of depression. Past history is pertinent for passage of a kidney (A) fluoxetine stone 1 month ago. Physical examination is (B) acetaminophen unremarkable. Screening blood tests reveal a (C) bromocriptine calcium of 12.8 and albumin of 4.0. Parathyroid (D) ibuprofen hormone (PTH) assay reveals an elevated value (E) hydrocortisone of 328. Which of the following is the most likely etiology of her symptoms? (A) parathyroid adenoma (B) parathyroid hyperplasia (c) ketabton.com: The Digital Library

Answers and Explanations

1. (A) Medicare is typically known as a health preeclampsia. Blood pressure is not reduced by insurance program for individuals over the age magnesium sulfate; intravenous hydralazine of 65; however, it also provides coverage for or labetalol are safe antihypertensive agents patients of all ages who have end-stage renal when given in small, intermittent doses for disease (permanent renal failure requiring dial- diastolic blood pressures greater than 105–110 ysis or a kidney transplant). (Centers for Medicare torr. Uteroplacental blood flow, decrease in and Medicaid Services (CMS) web site, 2007. Available preeclampsia, is unaffected by magnesium sul- at: http://www.cms.hhs.gov/ MedicareGenInfo) fate. Magnesium sulfate should be continued for 24 hours after delivery because of the con- 2. (D) In a previously healthy woman with an tinued risk of eclamptic seizures. (Cunningham uncomplicated pregnancy, the appearance of et al., pp. 267–268) hypertension and proteinuria in the third trimester is preeclampsia. The other disorders 5. (A) The goal of treatment is to reduce the sys- usually are present throughout the pregnancy, tolic blood pressure to less than 140 mm Hg. may develop earlier than the third trimester, Isolated systolic hypertension (ISH) is defined and may present with hypertension or protein- as a systolic blood pressure of above 160 and a uria, but not necessarily both. (Cunningham et al., diastolic blood pressure below 90 and com- pp. 259–263) monly occurs in the elderly. It is an important risk factor for cardiovascular disease and treat- 3. (E) The treatment of preeclampsia with a certain ment has been shown to reduce both cardio- term pregnancy (>37 weeks’ gestation) is deliv- vascular and stroke risk. Close follow-up is ery of the infant. Induction of labor with oxytocin important and care should be taken not to (Pitocin) is the preferred method of delivery, pro- reduce the diastolic below 65 as this may also vided the preeclampsia is not severe, the HELLP increase risk. Given his low pulse initiating syndrome (hemolysis, elevated liver enzymes, therapy with a beta-blocker in the absence of a low platelets) does not develop, or fetal distress compelling indication such as previous does not occur. A cesarean section should be per- myocardial infarction would not be the first formed if any of these develop, and vaginal choice. Alpha-blockers are not recommended delivery is not imminent. In the presence of as monotherapy due to increased risk of heart severe preeclampsia or the HELLP syndrome, failure noted in the ALLHAT (Antihypertensive delivery should be accomplished after 34 and Lipid-Lowering Treatment to Prevent Heart weeks. When gestation is less than 32 weeks, Attack) trial. (SHEP, pp. 3255–3264) expectant management for at least 48 hours should be attempted while glucocorticoids (e.g., 6. (A) The leading cause of massive bleeding per betamethasone or dexamethasone) is adminis- rectum in the older population is angiodyspla- tered to the mother to accelerate fetal lung mat- sia and diverticulosis of the colon. The absence uration. (Cunningham et al., pp. 259–263) of abdominal findings suggests these possibil- ities rather than an inflammatory process. 4. (C) The only effect of magnesium sulfate is to (Brunicardi et al., pp. 1065–1066, 1084; Greefield et al., reduce the risk of seizures with eclampsia. pp. 1056–1063) Although magnesium sulfate is commonly used as a tocolytic in idiopathic preterm labor, 7. (E) A sentinel, or herald, bleed is common in this is not its purpose in pregnant women with patients with an aortoenteric fistula. The key

340 (c) ketabton.com: The Digital Library

Answers: 1–16 341

point in the history is prior aneurysm repair eliminates the possibility of a cervical lacera- with a graft. The location of the fistula is the tion, although the lower genital tract should be third or fourth portion of the duodenum, so UGI reinspected to be certain that a laceration was endoscopy to the duodenal bulb would miss not overlooked. With uterine atony, the fundus the lesion. Immediate operation is indicated is boggy and larger than expected, unlike the because these patients will suffer a subsequent firm uterus found in this woman. A Couvelaire life-threatening bleed. (Brunicardi et al., pp. 739–740; uterus occurs as a complication of placental Greenfield et al., pp. 1056–1063; Townsend et al., p. 1255) abruption with concealed hemorrhage. Blood intravasates between myometrial fibers and 8. (G) Colon carcinoma may be the source of diminishes the capacity of the myometrium to chronic blood loss that often presents with contract. Although acute blood loss may lead to fatigue and orthostasis, particularly from a right- thrombocytopenia, the bleeding is the cause sided lesion. Abdominal fullness and tender- rather than the consequence of the low platelet ness is suggestive of a large, right-sided colon count. (Cunningham et al., pp. 369–370) cancer. (Brunicardi et al., pp. 1090–1091; Greenfield et al., pp. 1056–1063; Townsend et al., pp. 1457–1458) 14. (D) Uterine artery embolization, hypogastric artery ligation, and hysterectomy are accept- 9. (I) Bleeding from acute gastric ulcer may occur able treatment options. The choice depends on and become life threatening, especially in the the experience of the obstetrician, the rapid elderly taking nonsteroidal anti-inflammatory availability of an interventional radiology team, drugs (NSAIDs). Immediate operation is indi- and the hemodynamic status of the patient. If a cated following resuscitation. (Brunicardi et al., hypogastric artery ligation is unsuccessful, then pp. 959–969; Greenfield et al., pp. 1056–1063) a hysterectomy is necessary. Blood should be transfused to maintain hemodynamic stability 10. (H) Varices account for 10% of UGI bleeding but is never the sole treatment with placenta and are suggested in patients with an alcohol accreta. In one study, one-fourth of the women history and examination findings suggestive died who had a uterine pack inserted after of cirrhosis. (Greenfield et al., pp. 1056–1063) manual removal of the placenta. This was four times higher than in those who had an imme- 11. (F) Peptic ulcer disease is the most common diate hysterectomy. Administration of ergot cause of acute UGI hemorrhage. Duodenal alkaloids is a treatment option only when post- ulcers occur slightly more frequently than gas- partum hemorrhage is attributable to uterine tric ulcers. Brisk hemorrhage occurs from the atony. (Cunningham et al., pp. 369–371) gastroduodenal artery from a posterior pene- trating ulcer in the duodenal bulb. (Brunicardi et al., 15. (B) Short-acting benzodiazepines especially, such pp. 967–969; Greenfield et al., pp. 1056–1063) as alprazolam, can produce significant symptoms when abruptly discontinued, a sign of physio- 12. (B) The features of ulcerative colitis include age logic dependence. Mild symptoms can include of onset usually between 15 and 40 years, anxiety, irritability, sleep disruption, tremor, and bloody diarrhea, abdominal pain, and fever. nausea; the development of paranoia, depres- Bloody diarrhea is the major symptom in 25% of sion, delirium, and seizures are serious compli- patients. (Brunicardi et al., pp. 1065–1066, 1078–1080; cations of sudden drug withdrawal. The Greenfield et al., pp. 1067–1068; Townsend et al., p. 1428) likelihood of significant withdrawal symptoms increases with dosage, length of treatment, and 13. (E) A placenta accreta must be suspected if the shorter half-life. (Kaplan and Sadock, pp. 1027–1028) placenta does not separate spontaneously by 30 minutes after delivery of the infant, especially 16. (B) The clinical symptoms of iron toxicity are if a plane of dissection cannot be identified divided into stages. In a significant ingestion, with attempts to remove the placenta manu- the first stage begins immediately after the inges- ally. The examination immediately after birth tion and lasts 6–24 hours. Vomiting, diarrhea, (c) ketabton.com: The Digital Library

342 11: Practice Test 5

abdominal pain, pallor, lethargy, and hypoten- is recommended. Myopathy is a concern with sion can be seen. These result from the direct tox- statins. Myalgias occur with a frequency of icity to the GI tract and GI hemorrhage can 2–11% but rhabdomyolysis occurs with fre- occur. Liver failure and an associated coagu- quency of less than 0.1%. Hepatotoxicity can lopathy can occur but not typically until 12–24 occur with niacin. Ezetimibe does provide a hours after the ingestion. Arrhythmias and res- reduction in LDL but mortality benefit has yet piratory symptoms are not seen with iron inges- to be shown. Fish oil and dietary intervention tions. If a patient does not develop symptoms are beneficial but not sufficient therapy in this within 6 hours of ingestion, it is unlikely that case. Gemfibrozil is primarily helpful in lower- iron toxicity will develop. (Ellenhorn, pp. ing triglycerides. (Rosenson, pp. 408–416) 1558–1562) 20. (E) Symptoms of conduct disorder must begin 17. (C) Numerous complications of Epstein-Barr before the age of 13 for the diagnosis to be made, virus-associated mononucleosis have been and 3 of 15 behaviors must have been present described, such as hematologic complications, including: bullying, truancy, cruelty to or hurt- hemolytic anemia, granulocytopenia, and ing animals, threatening or intimidating others, thrombocytopenia. Neurologic complications often initiating fights, cruelty to people, stealing include aseptic meningitis, encephalitis, optic or raping, purposeful firesetting, lies to or cons neuritis, Guillain-Barré syndrome, transverse others, staying out at night despite parents pro- myelitis, and Bell’s palsy. Splenic rupture is hibiting, and running away from home. rare but potentially fatal. Often, it follows mild Differential diagnoses include mood disorders, trauma. X-linked lymphoproliferative disease psychotic disorders, and oppositional defiant is a rare but fatal complication that occurs in disorder. Children with oppositional defiant dis- males. Although liver involvement is not order do not typically violate the rights of others, uncommon acutely, chronic active hepatitis has whereas those with conduct disorder do. not been described. Pancreatitis and azotemia Children with conduct disorder are more likely have not been described. (Rudolph et al., pp. to develop antisocial personality disorder, but 1035–1038) this disorder cannot be diagnosed until adult- hood. Tay-Sachs disease is a disorder which is 18. (D) Diabetes mellitus, abdominal aortic caused by a defect of the enzyme hex- aneurysms, peripheral arterial disease, and osaminidase A and leads to seizures and spas- symptomatic carotid stenosis all carry a 10-year ticity. (Kaplan and Sadock, pp. 1167, 1232–1238) risk of >20% and are considered CHD risk equivalents. Cigarette smoking is a risk factor 21. (B) Almost 10% of boys 18 years of age or for CHD but is not by itself a CHD risk equiv- younger meet the diagnostic criteria for con- alent. When combined with other risk factors it duct disorder. Although this prevalence rate is may contribute to a risk of >20%. The LDL significantly higher than in girls, the incidence goal for CHD and CHD risk equivalents is less of the disorder actually is higher in postpuber- than 100. tal girls than in postpubertal boys. Parental alco- holism, sociopathy, neglect, or absence can 19. (B) This patient’s risk for CHD is elevated. Her predispose to the development of conduct dis- LDL goal is less than 130. The statins, such as order in their children. In some children, hyper- simvastatin, lower LDL the most, compared to activity precedes the emergence of conduct other agents and have the best evidence for a disorder. Various etiologic factors contributing therapeutic reduction in mortality. They are to the development of conduct disorders have first-line therapy for both primary and second- been explored. These include attempts to define ary prevention of CHD. Risk of hepatotoxicity specific neurologic factors. One such theory is low with only around 1% of patients treated states that there is decreased noradrenergic evidencing persistently elevated transaminases. functioning in conduct disorder. The enzyme Checking transaminases at 12 weeks of therapy dopamine beta-hydroxylase, which converts (c) ketabton.com: The Digital Library

Answers: 17–27 343

dopamine to norepinephrine, has been found to reducing angina but lowers blood pressure and be in low levels in the plasma of some children would likely worsen this patient’s hypotension. with conduct disorder. In addition, it has been found that attention deficit disorder and con- Explanations 23 and 24 duct disorder often coexist in some children. Other studies indicate the probability of psy- The patient above presents with a history and chosocial etiologic factors, including having physical examination most consistent with parents with serious psychopathology, sub- angioedema. Angioedema related to ACE stance abuse, and alcohol abuse disorders. The inhibitors usually does not involve urticaria or number of boys with the disorder is higher, and rash as it is not medicated via mast cell activa- the age of onset is younger (for boys, 10–12 tion. Inhibition of ACE results in increased levels of bradykinin causing vasodilatation and years vs. 14–16 years for girls). (APA, pp. 93–99; Kaplan and Sadock, pp. 1234–1238) increasing vascular permeability. Most cases occur within 7 days of initiation of the ACE 22. (C) On January 1, 2006, Medicare began to offer inhibitor but a substantial number occur months prescription drug coverage (also known as to years later. (Kostis et al., pp. 1637–1642) Medicare Part D) for all Medicare enrollees. All individuals who are covered by Medicare are 25. (C) eligible for the prescription drug coverage pro- gram regardless of income level. Medicare pre- 26. (E) scription drug coverage is insurance that may help lower prescription drug costs of both brand 27. (E) name and generic medications and is provided by private companies. Beneficiaries choose from Explanations 25 through 27 a list of drug plans and pay a monthly premium This patient has carbon monoxide poisoning, to enroll. The monthly premium, required co- first requiring oxygen. Initial fluid resuscitation payments, and covered medications are all dif- in burn injury is isotonic crystalloid. Colloid is ferent depending on the particular drug plan not recommended in the first 24 hours because picked by an individual. (Centers for Medicare and of the profound capillary leak associated with Medicaid Services (CMS) web site, 2007. Prescription the early injury phase. The most common for- Drug Coverage: Basic Information. Available at: http:// mula for fluid resuscitation in burn injury is the www.medicare.gov/pdp-basic-information.asp) Parkland formula. This is based on body weight and percentage of body surface area (BSA) 23. (D) burned; that is, 4 mL × body weight (kg) × %BSA = mL resuscitation fluid/first 24 h. One- 24. (C) The ECG shown in question 23 is consistent half of this amount is given in the first 8 hours with atrial fibrillation. In a patient with rapid after the burn, with the remainder over the next atrial fibrillation that becomes hemodynamically 16 hours. This fluid is given in addition to the unstable immediate cardioversion is indicated patient’s maintenance fluid requirements. despite the risk of thromboembolism. In a stable Patients with circumferential burn injury are at patient electrical or chemical cardioversion risk of developing compartment syndrome. should not be attempted until risk for embolism Burn wound edema increases the enclosed com- has been fully assessed and heparin initiated. A partment pressure above systemic perfusion stable patient with >48 hours in atrial fibrilla- pressures, resulting in tissue ischemia. The tion should either be anticoagulated for 4 weeks patient will have pain in the affected extremity, or assessed for presence of atrial clot with a trans- especially with passive stretch. The limb will esophageal echocardiogram and anticoagulated demonstrate signs of decreased distal perfu- prior to cardioversion. An alternative is not to sion, including pallor, decreased sensation, and attempt conversion but continue rate control and motor weakness. Compartment pressure is anticoagulation. IV nitroglycerine is effective in relieved with emergency escharotomy. Elevation (c) ketabton.com: The Digital Library

344 11: Practice Test 5

of the extremity is not appropriate as it will fur- not cause central nervous system (CNS) symp- ther diminish distal perfusion. Supplemental toms as might additionally be expected for oxygen by mask is an ineffective means of carbon disulfide. (LaDou, pp. 391–392; Wallace and increasing oxygen delivery because of dimin- Doebbeling, p. 510) ished perfusion to the affected limb. (Brunicardi et al., pp. 192–194; Greenfield et al., pp. 245–250; Townsend 31. (D) Type I second-degree heart block is often et al., pp. 576–578) due to block at the AV node and does not usu- ally progress to third-degree heart block. On 28. (B) If a patient has received a pneumococcal ECG it is characterized by a gradually increas- vaccination after the age of 65, repeat revacci- ing PR interval followed by a dropped beat. In nation is not usually required. Revaccination an asymptomatic patient with no signs of with the pneumococcal vaccine should occur ischemia, treatment consists of looking for a one-time after 5 years for patients with chronic reversible cause of decreased AV nodal con- renal failure or nephrotic syndrome, functional duction. In this case that is the beta-blocker or anatomic asplenia, immunosuppressive atenolol. (Kasper, pp. 1337–1338) conditions (such as HIV infection, leukemia, or congenital immunodeficiency) or who are 32. (C) Carpal tunnel syndrome is secondary to receiving chemotherapy or long-term steroids. compression of the median nerve at the wrist. Patients with cardiovascular disease, elevated This compression neuropathy affects women cholesterol, or who have a history of pneumo- more often than men. The patient may experi- nia do not need revaccination. (Recommended ence weakness of the hand, which is typically Adult Immunization Schedule. CDC web site: http:// worse at night and relieved by shaking the hand www.cdc.gov/nip/recs/adult-schedule-11x17.pdf) on awakening. There is hypesthesia in the median nerve distribution, and when long- 29. (E) The single most effective intervention a standing, there may be atrophy of the thenar physician can make in smoking cessation is muscles. Management includes wrist splinting, delivering an unambiguous, nonjudgmental, anti-inflammatory agents, steroid injection, and informative statement on the need to quit avoiding repetitive movement of the hand and smoking. It is especially helpful to deliver this wrist. Surgical decompression may be required message consistently over successive visits, and if nonoperative measures do not relieve symp- if effective and adapted, assistance and follow- toms. Compression of the digital nerve of the up are provided. Physicians can help establish thumb produces paresthesias limited to the a “quit date,” prepare the patient for with- affected digit. The motor branch of the median drawal symptoms, and supply positive rein- nerve may be compressed in the palm second- forcement on successive visits. Nicotine ary to direct trauma. Median nerve compression products may be useful for many smokers by the pronator teres is usually associated with withdrawing from tobacco products. It is sur- forearm pain in addition to paresthesias in the prising how often clinicians fail to counsel nerve distribution. (Brunicardi et al., pp. 1766–1770; smoking patients to stop using tobacco prod- Greenfield et al., p. 2024; Townsend et al., p. 2221) ucts. As per the year 2000 Public Health Service Guideline, the five “A’s” of tobacco cessation 33. (A) Ace inhibitors, angiotensin receptor block- are: ask (whether smokes), advise (not to use ers, spironolactone, beta-blockers, and nitrates/ tobacco), assess (readiness to quit), assist (in hydralazine have all been shown to prolong quit attempt), and arrange (for follow-up). (Lang survival in patients with CHF. Digoxin and and Hensrud, 2004; U.S. Preventive Services Task Force; furosemide do improve symptoms but have U.S. Public Health Service, www.surgeongeneral.gov; not been shown to prolong survival. (Kasper, Wallace and Doebbeling, p. 812) pp. 1372–1375)

30. (B) n-Hexane is a classic cause of peripheral 34. (C) Hypertrophic obstructive cardiomyopa- neuropathy, in the pattern described. It does thy (HOCM) is an important cause of sudden (c) ketabton.com: The Digital Library

Answers: 28–39 345

cardiac death in young adults and children. It transferred to the operating room for defini- is an intermittent left outflow tract obstruction tive management of the underlying penetrating that is worsened by exercise. Given the risk of cardiac injury. This patient does not have res- sudden cardiac death the student should not be piratory compromise and would, therefore, not cleared for participation in sports until defini- require assisted ventilation or chest tube inser- tive evaluation via echocardiogram has been tion. The patient’s hypotension may not done. The ECG may show left ventricular respond to IV fluid bolus, but requires urgent hypertrophy but may also be normal. During pericardial decompression to improve venous the Valsalva maneuver the reduction in ven- return and cardiac output. Emergency depart- tricular volume worsens the outflow obstruc- ment thoracotomy is reserved for patients with tion and intensifies the murmur. In aortic penetrating chest trauma who arrive in the stenosis the reduction in ventricular volume emergency department with measurable vital results in a decreased intensity of the murmur. signs and subsequently suffer cardiopul- Rheumatic fever may result in mitral and aortic monary arrest. The thoracotomy permits open valve disease but is a not a risk factor for cardiac massage, and attempt at manual con- HOCM. (Kasper, pp. 1410–1412) trol of blood loss from the site of injury, in preparation for rapid transfer to the operating 35. (B) room. (Brunicardi et al., pp. 133–135; Greenfield et al., pp. 412–413; Townsend et al., pp. 506, 508–510) 36. (C) 37. (D) Abuse of anabolic steroids is most likely to Explanations 35 and 36 be associated with the development of such manic-like symptoms as euphoria, irritability, Cardiac tamponade results from rapid accu- and reckless behavior. Depression can result mulation of fluid in the pericardial sac, com- from drug withdrawal, and suicides have promising cardiac filling and resulting in occurred. Abuse of anabolic steroids can pro- decreased cardiac output. The clinical findings duce drug dependence. (Kaplan and Sadock, pp. of tachycardia, hypotension, and distant heart 466–468; Stoudemire, 1998a, p. 414) sounds in the presence of penetrating chest trauma should alert the physician to this diag- 38. (C) nosis. Patients may also demonstrate JVD from decreased venous return to the right heart, and 39. (A) a diminished pulse pressure. Penetrating trauma with an entry wound in this location Explanations 38 and 39 (fifth intercostal space midclavicular line) would be unlikely to injure the descending Acute pericarditis often presents with sharp aorta. From anatomic landmarks, the left substernal chest pain radiating to the back and atrium and left ventricle are directly in the path sometimes left arm. The pain can be pleuritic of penetration. A massive hemothorax or ten- and is often worse with movement. Examination sion pneumothorax would be associated with findings can include a pericardial friction rub respiratory compromise, decreased breath that is a scratchy and may have presystolic, sys- sounds on the side of injury, and tracheal shift tolic, and early diastolic components. ECG to the contralateral side. Phrenic nerve paraly- reveals diffuse ST-segment elevation in the limb sis would affect diaphragmatic function and and precordial leads. The chest film in this case lead to altered ventilatory mechanics secondary reveals a large “water bottle” cardiac silhouette to paradoxical movement of the hemidi- consistent with pericardial effusion. Paradoxical aphragm. Pericardiocentesis is both diagnostic pulse is defined as a drop of 10 mmHg in arte- and therapeutic and should be the first step in rial pressure with inspiration and is a sign of management. Aspiration of as little as 10–20 mL cardiac tamponade. When tamponade is severe, may improve the patient’s hemodynamic as in this case, the radial pulse may disappear status. Once stabilized, the patient should be with inspiration. The treatment of cardiac (c) ketabton.com: The Digital Library

346 11: Practice Test 5

tamponade is pericardiocentesis. (Kasper, pp. carry a CF gene is (1/25)2, or 1 in 625. As an 1414–1418) autosomal-recessive disorder, 1 in 4 children will have CF if both parents carry a CF gene. 40. (A) Buerger’s disease has a worldwide distri- Thus, the empiric probability that this couple bution but is more prevalent in the Middle East will have an affected child is the product of the and Far East. Typically, symptoms begin before probability that both carry a CF gene and the the age of 40–45 in young male smokers. Small- 25% probability that the child will inherit the CF and medium-sized arteries and veins are typi- gene from both parents: 1/625 times 1/4 or 1 in cally affected and pathologically are affected by 2500. Routine testing for CF carrier status has inflammatory thrombi. The only proved strat- become a standard of care for all pregnant egy to prevent disease progression is discon- women. (Creasy and Resnick, pp. 250–255) tinuation of the use of tobacco, including exposure to second hand smoke, in any form. 43. (D) Syncope, congestive heart failure, and (Olin, pp. 864–869) angina all predict time to death of between 1.5 and 3 years making the risk of surgical AVR 41. (C) The new onset of hypertension below age worth considering. Declining left ventricular 35 or above age 55, a sudden worsening of pre- function on echocardiogram also identifies a vious well-controlled hypertension, hyperten- patient group that should be considered for sion resistant to three or more drugs, and AVR. Patients with severe aortic stenosis (valve sudden worsening of renal function in a patient area less that 0.5 cm2) who are asymptomatic with atherosclerotic vascular disease all raise and have normal left ventricular function concern for a secondary cause of hypertension. should be followed as the rate of surgical mor- This patient had a sudden worsening of hyper- tality outweighs that of sudden cardiac death. tension, decline in renal function, and onset of (Kasper, pp. 1397–1398) flash pulmonary edema which all suggest renal artery stenosis. The presence of a periumbilical 44. (A) The highest incidence of primary hyper- bruit, especially diastolic is suggestive of renal parathyroidism occurs in women over the age artery stenosis. Hyperaldosteronism is a sec- of 60. Most patients are asymptomatic and ondary cause of hypertension but not directly found on screening tests. Those patients with associated with acute renal failure or flash pul- symptoms most often manifest emotional dis- monary edema. Acute glomerulonephritis can orders (e.g., depression) and hypercalciuria. present with hypertension, renal failure, and Five to ten percent of patients with first-time volume overload but the bland urine sediment renal colic have primary hyperparathyroidism. in this case makes it unlikely. (Kasper, pp. Most patients with primary hyperparathy- 1707–1708) roidism have disease limited to one gland rather than multigland disease. MEN syndromes I and 42. (E) CF is an autosomal-recessive disorder. The IIa manifest parathyroid disease, but are rare CF gene has been mapped to chromosome and are associated with other endocrinopathies. 7q3.1. More than 600 gene mutations causing Carcinoma is rare. (Brunicardi et al., pp. 1434–1439; CF have been identified, although the mutation Greenfield et al., pp. 1319–1323) designated delta F508 accounts for approxi- mately 70% of the gene mutations in a White 45. (A) To determine the likelihood that a person population. Most laboratories analyze 38–31 with a positive test truly has the disease (to mutations for diagnosis or carrier screening. determine the positive predictive value of the This is sufficient to identify 90–95% of muta- test), you must know the sensitivity of the test tions in a White population. The frequency of and also must know the prevalence. If no one mutations in a Black population is approxi- has the disease, no one testing positive will mately 1 in 65, and is rare in an Asian popula- have the disease. If everyone has the disease, tion. Each of this couple has a 1 in 25 chance of everyone testing positive will have the disease. carrying a CF gene. The probability that both Positive predictive value = true positives (c) ketabton.com: The Digital Library

Answers: 40–46 347

divided by true positives plus false positives. is instructed to begin the NSAID with the onset (Greenberg et al., p. 79) of premenstrual symptoms and continue on a scheduled (not prn) basis until she is far enough 46. (D) Prostaglandin inhibition is the key to con- into her menstrual cycle that the dysmenorrhea trol of primary dysmenorrhea. Even the dys- usually disappears. Bromocriptine and corti- menorrhea associated with endometriosis is costeroids have no discernible effect on ovula- thought to be secondary to excess prostaglandin tion or the production of prostaglandins and are formation. NSAIDs such as ibuprofen and ineffective in relieving primary dysmenorrhea. others are usually effective in controlling the Although fluoxetine or other selective serotonin pain of primary dysmenorrhea (pain with men- reuptake inhibitors (SSRIs) are useful to treat struation in the absence of pelvic pathology). the premenstrual syndrome (PMS) or the pre- Inhibition of ovulation with oral contraceptives, menstrual dysphoric disorder (PMDD), these depot medroxyprogesterone acetate, or a are ineffective in relieving dysmenorrhea. The gonadotropin-releasing hormone agonist is presence of pelvic pathology, such as a pelvic also effective. Oral contraceptives decrease mass or nodularity of the uterosacral ligaments prostaglandin synthesis by causing atrophy and suggesting endometriosis, should be evaluated decidualization of the endometrium. Relief of before prescribing medication. The preferred dysmenorrhea may be enhanced by the com- method of evaluation of suspected pelvic bined use of oral contraceptives and an NSAID pathology is a diagnostic laparoscopy. (Speroff taken on a scheduled basis, that is, the woman and Fritz, pp. 539–540)

347 (c) ketabton.com: The Digital Library

BIBLIOGRAPHY

American Psychiatric Association (APA). Diagnostic angioedema associated with enalapril. Arch Intern and Statistical Manual of Mental Disorders, 4th ed. Med 2005 Jul 25;165(14):1637–42. Text Revision. Washington, DC: American LaDou J. Current Occupational & Environmental Psychiatric Association, 2000. Medicine, 3rd ed. New York, NY: McGraw-Hill, Brunicardi FC, Andersen DK, Billiar TR, et al., eds. 2003. Schwartz’s Principles of Surgery, 8th ed. New York, Lang RS, Hensrud DD. Clinical Preventive Medicine, NY: McGraw-Hill, 2005. 2nd ed. Chicago, IL: AMA Press, 2004. Centers for Medicare and Medicaid Services (CMS) web Olin JW, Thromboangiitis obliterans (Buerger’s dis- site, 2007. Prescription Drug Coverage: Basic Information. ease) N Engl J Med 200:343 (12):864–869. Available at: http://www.medicare.gov/pdp-basic- Prevention of stroke by antihypertensive drug treat- information.asp ment in older persons with isolated systolic hyper- Creasy RK, Resnik R, Iams JD. Maternal–Fetal tension. Final results of the Systolic Hypertension in Medicine: Principles and Practice, 5th ed. the Elderly Program (SHEP). SHEP Cooperative Philadelphia, PA: W.B. Saunders, 2004. Research Group. JAMA 1991 Jun 26;265(24):3255–64. Cunningham FG, Leveno KJ, Bloom SL, et al. Williams Recommended Adult Immunization Schedule, October Obstetrics, 22nd ed. New York, NY: McGraw Hill, 2006–September 2007. CDC web site: http:// 2005. www.cdc.gov/nip/recs/adult-schedule-11x17.pdf Greenberg RS, Daniels SR, Flanders WD, et al. Medical Rosenson RS, Current overview of statin-induced Epidemiology, 3rd ed. New York, NY: McGraw-Hill, myopathy; Am J Med 2004 Mar 15;116(6):408–16. 2001. Speroff L, Fritz MA. Clinical Gynecologic Endocrinology Greenfield LJ, Mulholland M, Lillemoe KD, et al., and Infertility, 7th ed. Philadelphia, PA: Lippincott eds. Surgery: Scientific Principles and Practice, 4th Williams & Wilkins, 2005. ed. Philadelphia, PA: Lippincott-Raven, 2005. Stoudemire Kaplan HI, Sadock BJ. Synopsis of Psychiatry: Townsend CM Jr, Beauchamp RD, Evers BM, et al., Behavioral Sciences/Clinical Psychiatry, 9th ed. eds. Sabiston Textbook of Surgery: The Biologic Baltimore, MD: Williams & Wilkins, 2003. Basis of Modern Surgical Practice, 17th ed. Kasper DL, Braunwald E, Fauci A, et al. Harrison’s Philadelphia, PA: W.B. Saunders, 2004. Principles of Internal Medicine, 16th ed. New York, Wallace RB, Doebbeling BN, eds. Maxcy-Rosenau-Last NY: McGraw-Hill, 2005. Textbook of Public Health & Preventive Medicine, 14th Kostis JB; Kim HJ; Rusnak J; Casale T; Kaplan A; ed. Stamford, CT: Appleton & Lange, 1998. Corren J; Levy E; Incidence and characteristics of

348 (c) ketabton.com: The Digital Library

Subject List: Practice Test 5

Question Number and Subject 23. Internal Medicine 24. Internal Medicine 1. Preventive Medicine 25. Surgery 2. Obstetrics-gynecology 26. Surgery 3. Obstetrics-gynecology 27. Surgery 4. Obstetrics-gynecology 28. Preventive Medicine 5. Internal Medicine 29. Preventive Medicine 6. Surgery 30. Preventive Medicine 7. Surgery 31. Internal Medicine 8. Surgery 32. Surgery 9. Surgery 33. Internal Medicine 10. Surgery 34. Internal Medicine 11. Surgery 35. Surgery 12. Surgery 36. Surgery 13. Obstetrics-gynecology 37. Psychiatry 14. Surgery 38. Internal Medicine 15. Psychiatry 39. Internal Medicine 16. Pediatrics 40. Preventive Medicine 17. Pediatrics 41. Internal Medicine 18. Internal Medicine 42. Obstetrics-gynecology 19. Internal Medicine 43. Internal Medicine 20. Psychiatry 44. Surgery 21. Psychiatry 45. Preventive Medicine 22. Preventive Medicine 46. Obstetrics-gynecology

349 (c) ketabton.com: The Digital Library

This page intentionally left blank (c) ketabton.com: The Digital Library

FIG. 3-2 (Courtesy of Neil S. Prose.) (Question 53) FIG. 9-1 (Courtesy of Neil S. Prose.) (Question 6)

FIG. 12-1 (Reproduced, with permission, from Zinner MJ. Maingot’s FIG. 12-2 (Courtesy of Neil S. Prose.) (Question 45) Abdominal Operations, 10th ed., Vol. 1. Stamford, CT: Appleton & Lange, 1997.) (Questions 10 and 11)

FIG. 13-6 (Reproduced, with permission, from Hurst JW. Medicine for the Practicing Physician, 4th ed. Stamford, CT: Appleton & Lange, 1996.) (Question 24) (c) ketabton.com: The Digital Library

FIG. 14-4 (Courtesy of M. L. Williams. Reprinted with permission FIG. 14-5 (Courtesy of M. L. Williams.) (Question 34) of Yearbook Medical Publishers.) (Question 33)

FIG. 14-7 (Courtesy of Neil S. Prose.) (Question 45) FIG. 14-8 (Courtesy of I. Frieden.) (Question 46) (c) ketabton.com: The Digital Library

CHAPTER 12 Practice Test 6 Questions

Questions 1 and 2 (A) acute bronchiolitis (B) acute laryngotracheobronchitis A healthy 23-year-old woman has her first prenatal visit. She has regular menses at 28- to 30-day inter- (C) a foreign body in the upper airway vals. Her last menstrual period (LMP) began (D) laryngomalacia December 1, 2006. (E) peritonsillar abscess

1. Which of the following is her estimated due Questions 4 and 5 date (EDD)? A 26-year-old man complains to his family physi- (A) August 24, 2007 cian that he finds it difficult to perform adequately (B) September 1, 2007 in his job as a truck driver for the past 3 months. (C) September 8, 2007 Since returning from combat in Iraq 1 year ago, (D) September 15, 2007 where he drove trucks for the U.S. Army, he has avoided driving under bridges, and when he passes (E) September 22, 2007 garbage on the side of the road he feels severe anxi- ety coming on and is compelled to pull his truck 2. This woman has a pelvic examination at this over. He says he does this because he has thoughts prenatal visit. Her uterus is palpated midway that the garbage is an explosive device. He com- between her symphysis and umbilicus. By plains of anxiety, irritability, hypervigilence, and he physical examination, which of the following is refuses to talk about his experience in Iraq. He has her gestation age? nightly nightmares about the combat, and he has (A) 6 weeks stopped seeing his former friends, and feels his (B) 12 weeks future is short. He also tells you he has been drink- (C) 16 weeks ing heavily when he is not driving. (D) 20 weeks 4. Which of the following is the most likely (E) 28 weeks diagnosis? 3. A 5-month-old child has a history of 2 days of (A) generalized anxiety disorder rhinorrhea and 1 day of fever and worsening (B) panic disorder without agoraphobia cough. She has been well prior to this time and (C) posttraumatic stress disorder (PTSD) has not previously had similar symptoms. On (D) psychosis NOS (not otherwise specified) physical examination, she has inspiratory stri- (E) obsessive-compulsive disorder dor. Which of the following should be consid- ered in the differential diagnosis?

351

Copyright © 2008 by The McGraw-Hill Companies, Inc. Click here for terms of use. (c) ketabton.com: The Digital Library

352 12: Practice Test 6

5. Which of the following medications is most passed meconium at 48 hours, but only after receiv- appropriate to treat this individual’s anxiety? ing a glycerin suppository. He has not tolerated oral feeds, and urine output has decreased over the pre- (A) lorazepam ceding 12 hours. (B) sertraline (C) clonazepam 8. Which of the following diagnostic and/or ther- (D) haloperidol apeutic interventions is essential prior to trans- (E) risperidone porting the child to a regional pediatric hospital? (A) nasogastric tube decompression and IV 6. A 16-year-old girl comes to the clinic for a fluid resuscitation checkup. Her parents are concerned about her (B) sweat chloride determination weight. Over the past year, she has lost 20 lbs. She has not been ill. She is an excellent student (C) plain abdominal radiographs and active in many after-school activities. On (D) blood and urine cultures, followed by physical examination, she is emaciated, initiation of broad-spectrum antibiotics hypothermic, and bradycardic. Which of the (E) barium enema following is the most likely diagnosis? 9. On arrival at the receiving pediatric hospital, (A) anorexia nervosa which of the following is the most appropriate (B) bulimia approach to establishing the diagnosis? (C) hyperthyroidism (A) anorectal manometry (D) diabetes mellitus (B) plain radiographs, followed by a barium (E) depression enema, and subsequent rectal biopsy 7. An asymptomatic sexually active woman aged (C) abdominal ultrasound less than 25 sees you in clinic for a Pap smear. (D) sweat chloride She tells you that she is in a monogamous rela- (E) laparotomy tionship. Which of the following best describes correct advice to her concerning sexually Questions 10 and 11 transmitted diseases? A 55-year-old man presents to the physician’s office (A) I recommend that you get a chlamydia complaining of persistent heartburn of 10 years’ test. duration. He also complains of a chronic cough and (B) You should consider getting a chlamy- regurgitation at night for 3 years despite raising the dia test. head of his bed 30 degrees. He self-medicated inter- (C) If you are pregnant, you should be mittently for 7 years, initially with antacids and screened for chlamydia. then H2 blockers, but the severity of his symptoms (D) If you have burning with urination, I increased and he was placed on a proton-pump definitely recommend that you be inhibitor 3 years ago. His heartburn is partially screened for chlamydia. improved, and he still complains of nighttime regurgitation and cough. His physical examination (E) Screening for chlamydia is of uncertain is normal, and he is referred for upper endoscopy benefit. that reveals superficial linear ulcerations of his dis- tal esophagus with 3 cm of epithelial changes con- Questions 8 and 9 sistent with Barrett’s esophagus (Figure 12-1). A 3-day-old term infant presents with progressive Biopsies reveal esophagitis and high-grade dyspla- vomiting and abdominal distention. On question- sia of the Barrett’s epithelium. ing the nursery staff, they report that the child (c) ketabton.com: The Digital Library

Questions: 5–15 353

diagnosis is confirmed with the findings of tri- cuspid vegetations on echocardiogram and positive blood cultures. Which of the following organisms is most likely responsible? (A) Streptococcus viridans (B) Staphylococcus aureus (C) Streptococcus bovis (D) Pseudomonas aeruginosa (E) coagulase negative staphylococci

Questions 13 through 15

A 55-year-old homeless woman with a long history of schizophrenia is seen for monitoring of her med- ications. As she talks, she frequently protrudes her tongue and grimaces. Her arms and hands jerk and FIG. 12-1 Also see color insert. (Reproduced, with permission, from Zinner MJ. Maingot’s Abdominal Operations, 10th ed., Vol. 1. Stamford, bizarrely gesticulate as she talks. She is constantly CT: Appleton & Lange, 1997.) squirming as she sits. She has been on fluphenazine 20 mg PO bid for many years. 10. Early treatment with which of the following would most likely have prevented this condition? 13. The abnormal movements strongly suggest which of the following? (A) H2 blockers (B) diet modification (A) an exacerbation of catatonic schizophrenia (C) Nissen fundoplication (B) tardive dyskinesia (D) proton-pump inhibitor (C) akathisia (E) regular schedule of antacids (D) pseudoparkinsonism (E) Huntington’s disorder 11. Which of the following should be recom- mended for this patient? 14. Which of the following is a risk factor for the above condition? (A) double dose proton-pump inhibitor and repeat endoscopy in 3 months (A) young age

(B) proton-pump inhibitor and H2 blocker, (B) being male followed by repeat endoscopy in 3 months (C) low dose of medication (C) antibiotic treatment for Helicobacter pylori (D) being homeless with proton-pump inhibitor (E) having a mood disorder (D) Nissen fundoplication (E) esophagectomy 15. Which of the following is a primary initial treat- ment strategy in this case? 12. A 25-year-old woman presents with high fever, (A) increasing the dose of fluphenazine cough, pleuritic chest pain, and dyspnea. She admits to heavy alcohol use and IV heroin (B) add clonazepam to the regimen abuse. Chest x-ray (CXR) reveals nodular pul- (C) switch from fluphenazine to quetiapine monary infiltrates. You astutely suspect infec- (D) add benztropine tious endocarditis as the etiology and the (E) add propranolol (c) ketabton.com: The Digital Library

354 12: Practice Test 6

16. In 1990, a study was conducted to determine 18. Over the course of an evening, 10 individuals the relationship between outdoor air pollution present to an emergency department com- and bronchitis in postal workers employed plaining of abrupt onset of severe nausea, vom- during the period 1970–1980. Information on iting, and abdominal cramps. Many were the occurrence of bronchitis since employment prostrate with sweats and dizziness. Some of and other relevant factors were gathered from the individuals developed diarrhea. On exam- the medical records of 2500 letter carriers deliv- ination, they were afebrile. All of them noted ering mail outdoors as well as those of 500 the onset of symptoms within 1–3 hours of comparable post office employees who had eating at the same restaurant, and all of them worked indoors. The risk of bronchitis was had eaten minced barbecue from the restau- found to be higher among those working out- rant buffet. The health department conducted doors. Which of the following describes this an inspection the next day. Which of the fol- study design? lowing findings is most likely associated with the illness? (A) ecologic study (B) historical cohort (A) obtaining a history of vomiting and (C) cross-sectional diarrhea from the food handler prepar- ing the barbecue (D) prospective cohort (B) inadequate reheating of previously (E) randomized, controlled trial refrigerated food 17. A 19-year-old student at a rural state univer- (C) an infected cut on the hand of a food sity is admitted to a hospital with rapid onset handler who prepared the barbecue of fever, headache, photophobia, and a stiff (D) an outbreak of gastroenteritis among neck. The student soon develops an extensive restaurant staff 1 week ago purpuric skin rash and becomes obtunded. (E) the meat for the barbecue came from a Gram’s stain of the cerebrospinal fluid (CSF) supplier previously implicated in an shows numerous gram-negative diplococci. E. coli 0157:H7 outbreak Within 3 days, two other students are admit- ted with fever, headache, and stiff neck. 19. You are in a shopping mall and you notice two Gram’s stains of their CSF also show gram- people next to a trash container. One is lying on negative diplococci. One of the students had the ground with involuntary twitching. The attended a seminar with the first student. The other is on her knees, in obvious physical dis- third has no known relationship with the tress, including respiratory difficulty, and is others. You are the public health officer for starting to exhibit involuntary movements of the area, and school officials are calling you the extremities. Which of the following is the for advice. There are 2 months of school left. most appropriate action to take? Which of the following do you recommend to (A) Call others for help, get someone to call control the outbreak? 911, check airways, breathing, and pulse. (A) Administer rifampin, 500 mg twice (B) Check airways, breathing and pulse, daily for 2 days, to all students and then call for help and call 911 if needed. faculty. (C) Call 911 and then check airways, breath- (B) Close the school and send the students ing, and pulse. home until new cases cease to appear. (D) Leave them where they are, tell every- (C) Conduct a mass immunization for one to evacuate the area, leave the area meningococcal disease. yourself, and call 911. (D) Identify the source of infection using a (E) If possible, quickly transfer the two peo- quick case-control study. ple to a protected area where they can (E) Conduct a mass media campaign on the be cared for without interference. signs and symptoms of meningitis. (c) ketabton.com: The Digital Library

Questions: 16–26 355

Questions 20 through 22 of asthma in the test population will have an effect on which of the following characteristics A 55-year-old man presents to the emergency depart- of the screening tool? ment with an acute onset of excruciating substernal chest and epigastric pain following a prolonged (A) sensitivity episode of vomiting. His past history is significant (B) specificity for hypertension, diabetes, and a 30 pack-year smok- (C) positive predictive value (PPV) ing history. Vital signs show a blood pressure (BP) of (D) accuracy 90/60 mmHg, pulse rate 100/min, respiratory rate (E) reliability 18/min, and temperature of 100°F. Examination reveals an ill-appearing man with subcutaneous 24. A high-risk cardiac patient has been counseled emphysema and distant breath sounds on the left. A repeatedly by his physician to alter his cur- CXR reveals pneumomediastinum and a left rently unhealthy diet. He considered changing hydropneumothorax. A left chest tube is placed, and his diet in the past, but thought that it would be IV fluids are administered with resolution of his too difficult. At his last routine health visit, he hypotension. was told that he had gained an additional 10 lbs. The patient realizes that his lifestyle is a prob- 20. Which of the following is the most likely lem and makes a commitment to make a diagnosis? change within the next 6 months. According (A) bronchopleural fistula to the transtheoretical model of behavior (B) esophageal perforation change, which of the following stages best fits (C) Mallory-Weiss tear this patient? (D) spontaneous pneumothorax (A) precontemplation (E) gastric perforation (B) contemplation (C) preparation 21. Which of the following is the most appropriate (D) action diagnostic test? (E) maintenance (A) computed tomography (CT) scan (B) esophagogram 25. Which of the following is not one of the crite- (C) analysis of the pleural fluid ria for diagnosis of the metabolic syndrome? (D) esophagoscopy (A) body mass index (BMI) (E) tracheobronchogram (B) waist circumference (C) high-density lipoprotein (HDL) 22. Which of the following factors has the most cholesterol level influence on outcome? (D) BP (A) choice of antibiotics (E) fasting glucose >110 (B) time to definitive treatment (C) intensive care management 26. A child is diagnosed with acute lymphoblastic leukemia (ALL). Which of the following is a (D) early intubation and mechanical favorable prognostic factor in ALL in childhood? ventilation (E) history of smoking (A) age >7 years (B) female gender 23. A pharmaceutical company sponsors the devel- (C) initial WBC of 10,000 opment of a new screening tool for asthma and (D) presence of a mediastinal mass plans to test it in a clinic where there are a large (E) platelet count of 100,000 number of asthmatics. The high prevalence rate (c) ketabton.com: The Digital Library

356 12: Practice Test 6

DIRECTIONS (Questions 27 through 31): For each growth on her face, chest, and abdomen. She numbered item, select the ONE best lettered now shaves every other day. She has no family option that is most closely associated with it. Each history of irregular menses or hirsutism. Her lettered option may be selected once, more than BP is normal. She has no galactorrhea. The once, or not at all. examination confirms the hirsutism. Her pelvic examination is normal except for a male For each patient with amenorrhea, select the most escutcheon. Her serum dehydroepiandros- likely diagnosis. terone sulfate (DHEAS) concentration is (A) anorexia nervosa normal. (B) endometrial sclerosis (C) polycystic ovary syndrome 30. A 38-year-old woman is now 6 months post- partum, but has not resumed menstruation. She (D) postpill amenorrhea did not breast-feed; in fact, she was unable to do (E) pregnancy so because she did not form milk. Preeclampsia (F) premature ovarian failure and uterine atony causing an estimated blood (G) prolactin-secreting pituitary tumor loss of 1200 mL within 3 hours after the birth of (H) Sertoli-Leydig cell tumor her child complicated her pregnancy. The uter- (I) 17α-hydroxylase deficiency ine atony responded to uterine massage and (J) Sheehan’s syndrome intramuscular ergot injections. (K) testicular feminization syndrome 31. A 16-year-old girl has never menstruated, nor (L) Turner’s syndrome has she developed breasts. She has Tanner stage (M) 21-hydroxylase deficiency 2 pubic hair. A trial of oral medroxyprogesterone (N) true hermaphroditism acetate failed to induce menstruation. She is 148 (O) Müllerian agenesis cm tall and weighs 46 kg (101 lbs). Examination confirms the absence of breasts and scant devel- 27. A 24-year-old nulligravida last menstruated opment of pubic hair. A small cervix is seen on 3 years ago. One month ago, she took medrox- speculum examination, and a small uterus is yprogesterone acetate for 10 days but failed to palpated on bimanual examination. She has a bleed vaginally. Her medical history is unre- webbed neck, cubitus valgus, and widely markable except for chronic schizophrenia, for spaced nipples. Her serum follicle-stimulating which she takes an antipsychotic medication. hormone (FSH) level is 86 mIU/mL. Her BP is normal. She is 165 cm (5 ft 6 in.) tall and weighs 59 kg (130 lb). Nipple secretion is 32. A 65-year-old woman with a history of hyper- present bilaterally. Her vagina is dry, and there tension and hyperlipidemia is admitted with is no mucus within the cervical canal. Her substernal chest pain. The initial episodes pelvic examination and the remainder of her occurred with heavy exertion and resolved examination is normal. with rest. For the past week they have occurred more frequently and even with mild exertion. 28. A 19-year-old woman has never menstruated. The discomfort is dull, pressure like and is She has never developed breasts or pubic hair. associated with some shortness of breath and Physical examination confirms that she has diaphoresis. Today she noted the onset while Tanner stage 1 breast development and pubic shopping and came into the emergency depart- hair. Her BP is 142/94 mmHg. Her pelvic ment for evaluation. In the emergency depart- examination is normal for a prepubertal female. ment her electrocardiogram (ECG) revealed 1 mm ST-segment depression in the inferior 29. A 22-year-old woman began to menstruate at leads. Her chest pain resolved with sublingual the age of 13 years, but has menstruated irreg- nitroglycerine and her ECG normalized. You 1 admitted her to the general medical ward and ularly at 1 /2- to 5-month intervals since. For the past 6 years, she has noted increasing hair have ruled out myocardial infarction with (c) ketabton.com: The Digital Library

Questions: 27–38 357

serial enzymes and ECGs. What is the next step (A) fluoxetine in her evaluation? (B) triazolam (A) Reassure her that she has not had a (C) clonazepam heart attack and discharge her. (D) diazepam (B) Obtain a stress test with myocardial (E) temazepam imaging. (C) Schedule her for a cardiac 36. A 15-year-old presents with a 3-week history of catheterization. headache, drowsiness, and fever. On physical (D) Intensify antianginal, BP, and choles- examination, there is mild nuchal rigidity and terol therapy and schedule outpatient a third cranial nerve palsy on the left. A lumbar follow-up. puncture is performed and shows an elevated WBC count with primarily monocytes, a pro- (E) Reassure her that her symptoms are tein of 750 g/100 mL, and a low glucose. The likely gastrointestinal (GI) in origin. most likely organism is which of the following? 33. All of the following have been shown to result (A) Mycobacterium tuberculosis in a significant reduction in mortality in the (B) Neisserria meningitis setting of acute myocardial infarction except (C) Streptococcus pneumoniae which one? (D) Haemophilus influenzae (A) beta-adrenoreceptor blockers (E) Salmonella typhimurium (B) aspirin (C) nitroglycerine 37. A 52-year-old man is receiving a preoperative (D) angiotensin-converting enzyme (ACE) evaluation before elective surgery. He is inhibitors asymptomatic and has a normal examination, but is noted to have a hemoglobin of 10.8, (E) heparin hematocrit of 33, with a mean corpuscular volume (MCV) of 70 (normal, 82–92), and 6.1 34. A 71-year-old woman is receiving parenteral million RBCs (normal, 4.5–5.0). Which of the methicillin for leg cellulitis. Over 2 days, she following is the most likely diagnosis? develops macroscopic hematuria, oliguria, and marked deterioration in renal functioning. (A) sickle cell anemia Which of the following is suggestive of (B) iron-deficiency anemia methicillin-induced acute interstitial nephritis? (C) alpha-thalassemia major (A) protein in the urine (D) beta-thalassemia minor (B) eosinophils in the urine (E) anemia of chronic disease (C) RBC casts in the urine (D) hyaline casts in the urine 38. A 27-year-old woman presents with bloody stools. She is found to have multiple, irregular (E) myoglobin in the urine dark brown macules on her lips, buccal mucosa, hands, and feet. What is the most 35. A 50-year-old professor was traveling across likely cause of her GI bleeding? the country to attend a conference where he was to make a presentation. He took a medi- (A) esophageal carcinoma cine to help him get some sleep on the flight. (B) jejunal hamartomas The day following his presentation (which wit- (C) gastric telangiectasia nesses report he did give), he has no memory (D) intestinal neurofibromas of making the presentation. Which of the fol- lowing medications most likely caused his (E) colonic hemangioma memory loss? (c) ketabton.com: The Digital Library

358 12: Practice Test 6

39. An 11-month-old infant presents with poor 41. Subsequent testing is positive for acute Lyme growth, alopecia, large volume stools, and skin disease in this patient. Which of the following rash. You recall that this is consistent with acro- is a likely outcome in Lyme disease? dermatitis enteropathica. The findings in this (A) Thirty percent of patients have a slowly disorder are caused by an inability to absorb expanding skin lesion (erythema which of the following? migrans [EM]) at the site of the tick bite. (A) zinc (B) Within weeks, 75% of untreated patients (B) vitamin K will develop neurologic manifestations.

(C) vitamin B12 (C) Fifty percent of untreated patients may (D) copper develop chronic neuroborreliosis. (E) iron (D) Months after onset of illness, 60% of untreated patients will have intermittent 40. An unmatched case-control study of the rela- attacks of joint swelling and pain. tionship between colon cancer and exposure to (E) Joint involvement is primary in small ionizing radiation reported the following data: joints (hands).

42. Which of the following persons is most in need Colon cancer of vaccination against Lyme disease? Ionizing radiation Cases Controls Total (A) a 20-year-old living in Chicago, IL Ye s 1 1 3 5 4 6 (B) an 85-year-old retiree living in Phoenix, No 50 209 259 Total 61 244 305 AZ (C) a 35-year-old living in the North Carolina mountains These results indicate which of the following? (D) a 56-year-old who vacations in the Maine woods each summer (A) The lifetime risk of colon cancer in the total population is approximately 20%. (E) a 45-year-old living in Boca Raton, FL (B) Among those without colon cancer, the 43. At a routine company physical examination, odds of ionizing radiation exposure are an asymptomatic 46-year-old man is found to about 1:5. have a BP of 150/110 mmHg, but no other (C) Among those who were exposed to ion- abnormalities are present. Which of the fol- izing radiation, the estimated risk of lowing is the most appropriate next step in colon cancer is 20.8%. management? (D) The relative risk of colon cancer associ- ated with exposure to ionizing radiation (A) reassure the patient and repeat the is approximated by an odds ratio of 1:3. physical examination in 12 months (E) The risk of colon cancer in the control (B) order an ECG group is 14.3%. (C) initiate antihypertensive therapy (D) obtain repeated BP recordings in your Questions 41 and 42 office and/or the patient’s home or work site A 42-year-old man camping in northern Wisconsin (E) counsel the patient on dietary sodium finds a tick embedded in his scalp. He presents to reduction clinic a few days later after returning from his trip. He camps multiple times each summer and is fear- ful of Lyme disease. (c) ketabton.com: The Digital Library

Questions: 39–46 359

44. A 30-year-old man tells you that he believes (A) leprosy his headaches are a result of exposure to an (B) secondary syphilis aerosolized adhesive at work. Which of the fol- (C) chancroid lowing questions is most important to ask to (D) acquired immune deficiency syndrome determine whether the exposure caused the (AIDS) headaches? (E) tinea (A) Are the headaches worse when you use more of the adhesive? 46. A 58-year-old man carries the diagnosis of (B) What is in the adhesive (request the pseudogout (chondrocalcinosis). Which of the Material Safety Data Sheet correspon- following is a common feature? ding to the product)? (A) uric acid deposits within joints (C) Do any coworkers have similar (B) episodic attacks of pain symptoms? (C) small joints affected more often than (D) When did the symptoms start? large joints (E) Do you have the headaches on days off? (D) predilection for women (E) predilection for young people 45. A 23-year-old female presents to your office with a rash. She reports that she is experiencing low-grade fevers and headache, in addition to the rash. She also relates having noticed a small ulcer in her genital area approximately 6 weeks earlier that has now healed. On physical exam- ination, diffuse lymphadenopathy is noted along with the rash pictured in Figure 12-2. Of the following disorders, which is most likely to be the correct diagnosis?

FIG. 12-2 Also see color insert. (Courtesy of Neil S. Prose.)

359 (c) ketabton.com: The Digital Library

Answers and Explanations

1. (C) The EDD is calculated according to Naegele’s abscess is seen in older children and adoles- rule by adding 7 days to the first day of the LMP, cents. (Kliegman, pp. 83–84) then subtracting 3 months. For this to be accu- rate, the woman must have reasonably regular 4. (C) PTSD is characterized by hyperarousal symp- menstrual intervals of 28–32 days. The mean toms including hypervigilance, anxiety, irritabil- duration of pregnancy is 266 days from concep- ity, increased startle response, and insomnia; tion or 280 days from the first day of the last reliving symptoms including flashbacks, recur- menses if menstrual intervals are about 28 days. rent intrusive thoughts of the trauma and night- An ultrasound should be done between 14 and mares; and avoidance symptoms including direct 20 gestational weeks (estimated from uterine avoidance of reminders of the trauma, avoidance size) in women with an uncertain LMP or irreg- of talking about the experience, social avoidance. ular menstrual intervals. (Scott et al., p. 5) A sense of a foreshortened future and emotional numbing. One reliving symptom, three avoid- 2. (C) The uterine fundus in pregnancy does not ance symptoms, and two hyperarousal symp- grow beyond the pelvis until after 12 postmen- toms are required to make the diagnosis. strual weeks. Subsequently, gestational age can Generalized anxiety disorder is characterized by be estimated from fundal height in 2 ways. At chronic, excessive, and uncontrollable worry 14, 16, and 18 weeks, the uterus is palpable one- about multiple life circumstances, often accom- fourth, one-half, and three-fourths the distance panied by concentration problems and muscle from the symphysis pubis to the umbilicus. The tension, but unrelated to trauma. Panic disorder fundus is palpable at the level of the umbilicus is characterized by discrete episodes of intense at 20 gestational weeks. Between 16 and 36 anxiety, often coupled with a fear of dying, which weeks, the distance from the symphysis pubis may occur out of the blue. They are sometimes to the uterine fundus (in cm) approximates the accompanied by agoraphobia (a fear of going number of weeks of gestation. (Scott et al., p. 8) into areas, such as shopping malls, where the panic attacks have occurred). Psychosis NOS 3. (B) Any condition that causes narrowing of the involves a loss of a sense of reality involving hal- upper airway may result in inspiratory stridor. lucinations or delusions. Obsessive-compulsive This includes acute laryngotracheobronchitis disorder is an anxiety disorder that involves or croup. This patient has the typical history recurrent intrusive thoughts that lead to repetitive and examination seen in patients with croup. behaviors that the person feels driven to perform. Laryngomalacia is a condition of unusual soft- (Kaplan and Sadock, pp. 616–631) ness of the larynx, resulting in inspiratory col- lapse and stridor. The condition is present early 5. (B) Sertraline and other selective serotonin reup- in infancy and usually resolves by 3–5 years. take inhibitors (SSRIs) are effective in the treat- Patients with laryngomalacia have chronic stri- ment of the anxiety in PTSD. Benzodiazepines dor. Conditions that cause narrowing of the such as lorazepam and clonazepam may reduce conducting system of the lower respiratory anxiety in patients with PTSD, but the patient’s tract, such as bronchiolitis, result in wheezing. heavy drinking may be a contraindication for Foreign bodies are more common in children using a benzodiazepine. Such antipsychotic between the ages of 1 and 4 years, when they medications as haloperidol and risperidone are tend to put things in their mouths. Peritonsillar not effective alone in the treatment of PTSD, but

360 (c) ketabton.com: The Digital Library

Answers: 1–12 361

in selected patients with flashbacks, the addition would be indicated only if there were perinatal of an antipsychotic to an SSRI may be useful. risk factors for sepsis. Progressive abdominal (Kaplan and Sadock, pp. 622–631) distention, vomiting, and delayed passage of meconium are suggestive of Hirschsprung’s 6. (A) Anorexia nervosa is a disorder of unknown disease. Plain abdominal radiographs will cause that primarily affects young women. Its demonstrate multiple dilated bowel loops. A onset most frequently occurs during adolescence contrast enema is important to exclude other and its highest incidence is among White causes of distal bowel obstruction in the females of Western countries. The clinical picture neonate. Hirschsprung’s disease is then con- is predominated by signs of starvation, and firmed with rectal biopsy. Anorectal manometry obsessive-compulsive traits are often present. is a useful diagnostic tool in older children Affected individuals are frequently good stu- because it requires the subject’s cooperation and dents who have been characterized as achievers. communication skills. (Brunicardi et al., pp. A loss of 25% or more of total body weight is 1496–1497; Greenfield et al., pp. 1906–1914; Townsend almost always included in the diagnostic crite- et al., pp. 2113–2114) ria. Hypothermia and bradycardia are seen with severe weight loss. Patients with bulimia tend to 10. (C) binge and then purge. They are often of normal weight. Hyperthyroidism is not common in ado- 11. (E) lescents but can occur. It can cause weight loss, but this patient does not have other signs of Explanations 10 and 11 hyperthyroidism, such as tachycardia. Diabetes in adolescents typically is of abrupt onset, with The complications of gastroesophageal reflux the classic symptoms of polyphagia, polydip- disease (GERD) include esophagitis, esophageal sia, and polyuria. Depression can cause weight ulceration, stricture, and Barrett’s esophagus, loss, but patients tend to withdraw from their among others. Although some patients with activities, and mood changes are the dominant GERD respond well to medical therapy, such as a proton-pump inhibitor, regurgitation, and symptom. (Rudolph et al., pp. 232–233, 2074–2077, 231, 2112–2119) coughing attacks, some may not improve with medical treatment alone. The results of treatment 7. (A) The third U.S. Preventive Services Task by antireflux surgery, such as Nissen fundopli- Force (USPSTF) strongly recommends that cli- cation, are generally good under these circum- nicians routinely screen all sexually active stances and may prevent the progression of the women aged 25 and younger, and other asymp- Barrett’s. Once present, Barrett’s rarely regresses tomatic women at increased risk, for chlamydia. following surgery. Once high-grade dysplasia is (USPSTF) found within Barrett’s epithelium, up to half of esophagi removed for such a condition demon- 8. (A) strate foci of invasive cancer. Therefore, the rec- ommended treatment for high-grade dysplasia 9. (B) within Barrett’s is esophagectomy. There is no evidence to suggest that H. pylori is involved Explanations 8 and 9 in GERD or esophagitis. (Townsend et al., pp. 1142–1144, 1163) This infant presents with the clinical picture of a bowel obstruction. Initial management should 12. (B) Infectious endocarditis in IV drug users is include IV fluid resuscitation and nasogastric usually caused by S. aureus and often involves decompression. These interventions are essen- the tricuspid valve. Left-sided lesions in IV tial before safe transport of the child to another drug users usually involve previously dam- center for further diagnostic studies. A septic aged valves and can be caused by P. aeruginosa. workup, with initiation of antibiotic therapy, S. viridans typically causes a more indolent (c) ketabton.com: The Digital Library

362 12: Practice Test 6

presentation called subacute bacterial endo- individual subjects was collected. It was not a carditis. S. bovis usually enters the bloodstream cross-sectional design, because new occur- via the GI tract and is often associated with GI rences of bronchitis since employment were malignancies. Coagulase negative staphylo- used to calculate risk. It was not a randomized, cocci are commonly associated with prosthetic controlled trial, because the investigator did heart valve infections. (Kasper, pp. 731–734) not have anything to do with who works inside or outside of the post office. It was not a 13. (B) Tardive dyskinesia is a frequent side effect prospective cohort design, because the cohort of long-term administration of typical antipsy- was defined in the past. (Greenberg et al., p. 116, chotics and presents with involuntary, irregu- 117; Wallace and Doebbeling, p. 18, 19) lar choreoathetoid movements of the tongue, mouth, and sometimes limbs and the trunk. 17. (C) A mass immunization program is indicated. (Kaplan and Sadock, p. 995) Preventive treatment with rifampin is usually limited to close contacts such as household 14. (E) Being homeless is not a known risk factor members, day care center contacts, and people for tardive dyskinesia, but length of treatment exposed to the patient’s oral secretions. Sending and increased dose of dopamine receptor students home 2 months early will only redis- antagonist medication, increasing age, female tribute exposed persons. A case-control study sex, and the presence of mood disorder all are will be difficult, since there are so many expo- risk factors for development of tardive dyski- sure variables and it will take time to do. An nesia. (Kaplan and Sadock, p. 995) education campaign can help with early detec- tion, not with prevention. Immunizations will 15. (C) Vitamin E, propranolol, diazepam, and be more effective. Most of the sporadic out- others all have varied effects on the disordered breaks in this country are due to serotype C movements of tardive dyskinesia, but none have meningococcus, which is covered by the vac- been entirely effective. Benztropine is not effec- cine. Rates of asymptomatic carriage are high, tive for tardive dyskinesia but is used quite and it may be impractical to eliminate all car- effectively for other extrapyramidal effects of riage in a population. It is not clear why a gen- the neuroleptics, such as tremors, pseudoparkin- eral population with many carriers may sonism, and the dystonias. Switching from a suddenly erupt with several cases of disease. dopamine receptor antagonist to a serotonin- Preventing disease through vaccination remains dopamine antagonist (quetiapine) would be the the best method in the face of an outbreak. Note best treatment option of those given. Newer that immunization is not protective for the antipsychotics, such as olanzapine, risperidone, serotype that generally affects children of less ziprasidone, aripiprazole, and quetiapine, seem than 1 year of age. (Lang and Hensrud, 2004; Wallace to have little extrapyramidal effects and may and Doebbeling, p. 206) offer successful treatment of psychosis without risk of tardive dyskinesia. Ideally, a discontinu- 18. (C) The most likely illness is staphylococcal ation of all antipsychotics would prevent the food poisoning, resulting from contamination worsening of the movement disorder. The dire by a food handler. Often, the handler has evi- effects of this on the psychotic disorder, how- dence of an infection. The victims exhibit clas- ever, make it a rare option. (Kaplan and Sadock, pp. sic symptoms of abrupt, often violent nausea, 995, 1051–1066; Stoudemire, 1998, pp. 178–179, vomiting, abdominal cramps, and prostration. 645–647) They frequently feel feverish and perspire, but they are not febrile. Diarrhea is frequent, but 16. (B) In a historical cohort study, the cohort is not the dominant complaint. The illness is formed in the past, classified by exposure caused by a heat-stable enterotoxin produced status, and followed forward in time to deter- from S. aureus. Regarding the food handler, S. mine the development of disease. This study is aureus is a common organism found in muco- not ecologic in design, because information on cutaneous infections. In outbreaks like this, (c) ketabton.com: The Digital Library

Answers: 13–25 363

infected lesions on food handlers can some- whom a delay in treatment of more than 24–48 times be found. Transmission is usually from an hours has occurred. (Townsend et al., pp. 1110–1115) infected food handler to foodstuffs that are pre- pared ahead of time and then held at subopti- 23. (C) When the prevalence of the screened-for- mal temperatures, which allows the organism disease increases in a population, the PPV of time to replicate and produce enterotoxin. the screening test also increases. (If no persons Because the toxin is heat stable, rewarming the in the population have the disease, the PPV food and keeping it hot on a serving table will will be 0%. If all persons in the population have not neutralize it. The toxin is preformed when the disease, the PPV will be 100%.) The sensi- ingested, so there is no incubation time required tivity and specificity of the test are not affected in the victim. Therefore, onset is usually rapid. by the prevalence of the disease. Accuracy (Wallace and Doebbeling, p. 724, 727) refers to the specificity and sensitivity of the test, which are characteristics of the test itself, 19. (D) If you observe a pattern consistent with and thus is not affected by prevalence. A reli- poisoning by a chemical weapon of mass able test reproduces the same results when it is destruction, leave the scene and call 911 repeated. (Just as a very reliable gun will immediately. Do not become the next helpless always hit a target in exactly the same place, victim. Rapidly developing (over the course although it may not be hitting the bull’s eye.) of minutes) twitching, vomiting, difficulty (Wallace and Doebbeling, pp. 907–908) breathing, pinpoint pupils, coma, and death are typical of poisoning with the potent 24. (B) According to the transtheoretical model of organophosphate nerve gas sarin. (Wallace and behavior change, a patient in the contempla- Doebbeling, pp. 1169–1173) tion stage of behavior change is a person who has some motivation to change and has made a 20. (B) commitment to do so within the next 6 months. The precontemplation stage represents a stage 21. (B) at which an individual has no consideration of change and this may be because he/she is not 22. (B) aware of the behavior or its consequences. The action stage occurs when individuals modify Explanations 20 through 22 their activities, experiences, or environment in order to change behavior. It represents the first Clinical features of spontaneous esophageal 6 months of actual behavior change. The main- perforation (Boerhaave’s syndrome) include tenance stage represents the period during acute onset of excruciating pain after straining, which the individual works toward preventing such as in the case of vomiting. Associated clin- relapse. It usually represents the period after ical findings may include subcutaneous emphy- the first 6 months of behavior change. A prepa- sema, fever, hypotension, tachycardia, and ration stage is sometimes added to refer to those dyspnea. Initial treatment includes resuscita- planning definitive change within 1 month and tive efforts to stabilize the patient. The diag- having already initiated some changes in nostic test of choice is a contrast study of the behavior. Relapse also is sometimes referred to esophagus. A CT scan is helpful in atypical pre- as a stage. (Lang and Hensrud, pp. 61–63; Wallace and sentations or to demonstrate related complica- Doebbeling, p. 812) tions. Esophagoscopy is rarely needed to diagnose a perforation. Analysis of the pleural 25. (A) Criteria for diagnosis of metabolic syn- fluid may be suggestive of a perforation, but drome include waist circumference of >40 in. radiologic studies would be required to make a (102 cm) in men and >35 in. (88 cm) in women, definitive diagnosis. Time to definitive treat- triglycerides of >150, HDL <40 in men and ment is the factor that most influences outcome. <50 in women, BP of >135/80, and fasting glu- Poorer survival has been noted in patients in cose >110. Any three of these constitutes the (c) ketabton.com: The Digital Library

364 12: Practice Test 6

metabolic syndrome. Abdominal girth is androgen, and estrogen secretion is deficient. included as a measure of obesity rather than The gene for 17α-hydroxylase is on chromo- BMI because it is more highly correlated with some 10q24, and a variety of mutations result in metabolic risk factors. (Kasper, pp. 1431–1432) the clinical picture seen in this woman. The mineralocorticoid pathway is the only steroid 26. (C) The following factors contribute to a favorable biosynthetic pathway that is not impaired. prognosis for children with ALL: age between 3 Mineralocorticoid (11-desoxycorticosterone, and 7 years, absence of lymphadenopathy, female corticosterone, and aldosterone) secretion is gender, initial WBC of 10,000, hemoglobin of 7 increased because the decreased cortisol causes g/100 mL, and platelet count of at least 100,000. an increased, adrenocorticotropic (ACTH) hor- All of these factors except gender lose their mone secretion. Serum pregnenolone and prog- prognostic significance after 2 years of com- esterone concentrations will be elevated, while plete and continuous remission. Unfavorable 17-hydroxypregnenolone and 17-hydrox- prognostic factors include the presence of a yprogesterone concentrations will be low. mediastinal mass, Ph1 chromosome, decreased Treatment is glucocorticoid replacement titrated immunoglobulins, T- or B-cell surface mark- to suppress high levels of ACTH to within the ers on lymphoblasts, more than 25% blast normal range. (Speroff and Fritz, p. 352) cells in the bone marrow 14 days after treat- ment, and L2 or L3 lymphoblasts. (McMillan et al., 29. (C) Irregular menstrual intervals in women pp. 1493–1501) with signs of androgen excess (hirsutism, tem- poral balding, voice changes, clitoral enlarge- 27. (G) Amenorrhea and galactorrhea are usually ment, and so forth) suggest a functional rather concurrent findings in women with signifi- than a neoplastic disorder if the problem is cantly increased prolactin concentrations. The chronic and dates from the time of puberty. differential diagnosis of hyperprolactinemia is: Polycystic ovary syndrome is the most likely thyroid dysfunction resulting in increased diagnosis because of the combination of erratic hypothalamic thyrotropin-releasing hormone menstrual intervals since menarche, hirsutism, secretion, certain psychotropic and antihyper- negative family history, normal BP, and a tensive medications, a pituitary tumor, and idio- normal DHEAS concentration. The absence of a pathic. Although it is uncertain that this patient family history, normal BP, and a normal DHEAS has a pituitary adenoma, this is the most likely level eliminate 21-hydroxylase deficiency. diagnosis in her situation of anestrogenic amen- Deficiency of 17α-hydroxylase causes sexual orrhea and galactorrhea. A serum prolactin con- infantilism, not hirsutism. Moderate increases centration exceeding 100 ng/mL (normal is in serum prolactin levels may cause irregular usually less than 20) is associated with a pitu- menses, but not hirsutism. Sertoli-Leydig cell itary adenoma in at least 80% of these women. tumor is the only other hyperandrogenic disor- This woman should have a serum prolactin con- der listed. Clinical presentation of women with centration measured, and then a CT scan or MRI this tumor is usually abrupt onset of amenor- of her pituitary if the prolactin concentration is rhea and unilateral ovarian enlargement. (Speroff this high. Tumors develop as a result of chronic and Fritz, pp. 465–475, 506–519) lactotrope stimulation (or chronic suppression of dopamine secretion from the hypothalamus). 30. (J) Postpartum amenorrhea in a woman with a First-line treatment is oral bromocriptine history of hypertension and hemorrhage is (Parlodel) or cabergoline (Dostinex). (Speroff and most likely the result of pituitary necrosis and Fritz, pp. 429–437) inability to secrete gonadotropins (FSH and luteinizing hormone). Prolactin secretion is also 28. (I) Hypertension in a sexually infantile woman impaired, and lactation is not possible in these with primary amenorrhea is 17α-hydroxylase women. ACTH and thyroid-stimulating hor- deficiency until proven otherwise. This is an mone (TSH) secretion may also be impaired, autosomal-recessive disorder in which cortisol, and this woman should be evaluated with an (c) ketabton.com: The Digital Library

Answers: 26–37 365

AM serum cortisol and TSH concentration. Methicillin, penicillin, diuretics, nonsteroidal anti- (Speroff and Fritz, p. 438) inflammatory drugs (NSAIDs), and allopurinol all have been implicated. An immune basis is 31. (L) Key to making the diagnosis of Turner’s syn- postulated, and the acute azotemia may be asso- drome (gonadal dysgenesis) is the recognition ciated with signs of an allergic reaction: fever, that 148 cm (4 ft, 10 in.) is short. Streak gonads arthralgias, and rash. Laboratory findings may (i.e., gonadal dysgenesis) should be the first con- include pyuria without bacteriuria, WBC casts, sideration in any woman with primary amen- and serum or urine eosinophilia. Discontinuing orrhea who is under 5 ft tall. Other signs of the offending agent may reverse the renal failure, gonadal dysgenesis in this young woman are so a high degree of suspicion and early diagnosis the webbed neck, cubitus valgus, shield chest is vital. Steroids are commonly given and may and wide-spaced nipples, heart murmur, and further improve renal function. RBC casts in the others. A karyotype should be obtained to con- urine are diagnostic of glomerulonephritis and firm the diagnosis and ensure that there is no Y are not associated with interstitial nephritis. chromosome. The presence of a Y chromosome Proteinuria is a nonspecific finding that may be requires removal of the gonads because these due to glomerular or tubular injury, hyperten- women have about a 30% chance of developing sion, chronic renal failure, or the presence of a gonadal tumor, such as gonadoblastoma. The abnormal circulating proteins such as Bence- increased FSH concentration confirms that this Jones proteins found in multiple myeloma. woman’s amenorrhea is the result of gonadal Hyaline casts are a normal finding on urinalysis. failure, not hypothalamic-pituitary failure. (Kasper et al., pp. 247–252, 1705) (Speroff and Fritz, pp. 345–348) 35. (B) Triazolam is a high potency benzodiazepine 32. (C) This patient presents with a history consis- and short-acting benzodiazepine that has been tent with accelerated angina. She is considered associated with anterograde amnesia. (Kaplan a high-risk patient due to the presence of angina and Sadock, p. 1027) at low levels of activity and ST-segment depres- sion on ECG. As a high-risk patient, she would 36. (A) Tuberculous meningitis is characterized do best with an early invasive strategy such as pathologically by a thick, gelatinous exudate in cardiac catheterization. In this case, stress test- the subarachnoid space. The predilection of this ing would be unlikely to change management. material for the base of the brain explains the fre- Pretest probability for myocardial ischemia is quency of cranial nerve findings in afflicted chil- high and a negative test would not result in a dren. Although the concentration of protein in significantly post test probability to avoid fur- the CSF usually is elevated, levels of 1 g/100 ther testing. Intensifying medical management mL or greater generally indicate obstruction of as in choice D is appropriate but insufficient. the ventricular system. The tuberculin test is (Kasper, pp. 1444–1448) negative in about 10% of patients, especially those with advanced disease. Although the 33. (C) Essential pharmacotherapy of acute onset may occasionally be acute, signs and myocardial infarction includes beta-blockers, symptoms generally begin gradually or insidi- antithrombotic therapy including antiplatelet ously. The prognosis is guarded, and for those agents, heparin, consideration of thrombolytic who present with advanced neurologic findings therapy, and ACE inhibitors. These have all such as coma, the prognosis is poor, even with been shown to reduce mortality. Nitrates may appropriate therapy. (Rudolph et al., pp. 953–959) be useful for the relief of anginal pain but have only a slight effect on mortality. (Kasper, pp. 37. (D) A low hematocrit can be seen with all of the 1454–1455) anemias listed, but is usually much lower (low 20s) in sickle cell anemia. A low MCV is asso- 34. (B) Drug-induced acute interstitial nephritis is a ciated with iron-deficiency anemia or beta- frequent cause of reversible renal failure. thalassemia, but a normal or high number of (c) ketabton.com: The Digital Library

366 12: Practice Test 6

RBCs is characteristic of beta-thalassemia 40. (D) From a case-control study, the incidence minor. In iron deficiency, the red cell count is (risk) of colon cancer cannot be calculated usually below normal, and there is an elevated directly, because the cases and controls have red cell distribution width. Cells are usually been selected based on their disease status. normocytic normochromic with anemia of Thus, choices (A), (C), and (E) should be chronic disease. Patients with alpha-thalassemia excluded as possible correct answers. Choice major have moderate-to-severe hemolytic (B) gives the odds of ionizing radiation expo- anemia early in life. (Kasper et al., pp. 329–335, sure in those with colon cancer (rather than 594–600) without colon cancer), and so cannot be selected as an answer. Thus, the only correct answer is 38. (B) The patient in this question with pigmented (D), in which the relative risk of colon cancer oral lesions and lower GI bleeding most likely associated with exposure to ionizing radiation has Peutz-Jeghers syndrome, which is a syn- is approximated by an odds ratio. The odds ratio drome of mucocutaeous pigmentation and is the odds that a case was exposed to divided intestinal polyps. The polyps associated with by the odds that a control was exposed to. Here Peutz-Jeghers syndrome are hamartomatous. the odds ratio is 1.3. (Gordis, p. 33) The polyps occur most frequently in the jejunum but may be found anywhere between the stom- 41. (D) ach and the rectum. Malignant transformation of the polyps occurs in 2–3% of cases. There are 42. (D) many other syndromes with cutaneous pig- mented lesions and GI neoplasia. Tylosis Explanations 41 and 42 (hyperkeratosis of palms and soles) is associ- ated with esophageal carcinoma. Hereditary The clinical manifestations of Lyme disease can hemorrhagic telangiectasia (Rendu-Osler-Weber be divided into three phases: localized infec- disease) often involves bleeding of the GI tract. tion, disseminated infection, and persistent The blue rubber bleb nevus syndrome also infection. After an incubation period of 3–32 causes bleeding due to GI hemangiomas. days, 80% of patients develop EM, a slowly Neurofibromatosis (von Recklinghausen dis- expanding annular lesion at the site of the tick ease) is characterized by café au lait spots with bite. Within days or weeks patients may develop systemic symptoms including fever, cutaneous and intestinal neurofibromas. (Kasper et al., pp. 198, 301–303) severe headache, chills, arthralgias, and fatigue. After weeks or months, 15% of untreated 39. (A) Acrodermatitis enteropathica is an patients will develop neurologic manifesta- autosomal-recessive disorder characterized tions including meningitis, encephalitis, clinically by dermatitis, alopecia, malabsorp- chronic axonal polyneuropathy, and cerebel- tion, infections, growth retardation, and, rarely, lar ataxia. Months after the onset of infection, hypogonadism. Children with this disorder approximately 60% of untreated patients manifest the clinical findings soon after being develop frank arthritis. The typical pattern is weaned from breast-feeding because of the that of intermitten attacks of oligoarticular decreased bioavailability of zinc in cow’s milk. arthritis involving large joints such as the Malabsorption is characterized by severe steat- knees. Other potential disease manifestations orrhea, lactose intolerance, and malnutrition. include atrioventricular block and pericardi- Some patients are known to develop cuta- tis. Lyme disease typically occurs in three dis- neous candidiasis, bacterial infections, and eye tinct areas: the Northeast from Maine to manifestations, including blepharitis and pho- Maryland, the Midwest in Wisconsin and tophobia. Chronic zinc deficiency is the cause Minnesota, and in the West in northern of the disorder. The treatment of choice is the California and Oregon. The disease is trans- administration of 10–45 mg/day of elemental mitted by the ticks of the Ixodes ricinus com- plex. (Kasper et al., pp. 995–999) zinc. (Rudolph et al., p. 1176, 1177) (c) ketabton.com: The Digital Library

Answers: 38–46 367

43. (D) Before any laboratory evaluation or ther- serologic testing (serologic test for syphilis apy, the presence of hypertension must be care- [STS], Venereal Disease Research Laboratory fully documented. When characteristic [VDRL], fluorescent treponemal antibody- end-organ changes are not apparent on physi- absorption test for syphilis [FTA-ABS]). Leprosy cal examination, the presence of hypertension may present with hypopigmented macules, can best be documented by demonstrating a plaques, or papules, but fever and headache persistent elevation of BP. Although a single are not typical symptoms. Chancroid presents observation of mildly elevated BP does not jus- with an inflammatory macule that forms a pus- tify either an evaluation for secondary causes tule and then ulcerates. Adenopathy may be or initiation of treatment, it should not be noted with chancroid, but fevers and headaches ignored. The patient should be rescheduled for are not typical. The gram-negative organism additional BP measurements on several occa- responsible for chancroid (H. ducreyi) may be sions within the next few weeks. There is no obtained by culture. The history of a genital need to obtain an ECG or begin counseling if ulcer followed by the rash pictured should raise this is not true hypertension. (Kasper et al., pp. the suspicion of syphilis as the most likely 1468–1471) cause. (Creasy and Resnik, pp. 772–776)

44. (E) Dose response and biological plausibility 46. (B) Pseudogout (chondrocalcinosis) is an are helpful in determination of causation, but osteoarthritis-like disorder often associated with the clearest criterion for causation is temporal- hemochromatosis. Patients experience stiffness ity. The cause must precede the effect. The other and pain caused by proliferation and fibrosis of proposed answers are also relevant. (LaDou, synovial tissue. In approximately half of patients pp. 232–233; Rom, p. 173) there is calcium pyrophosphate deposition dis- ease (CPPD). Pseudogout afflicts middle-aged 45. (B) Syphilis is a sexually transmitted infection and elderly persons, with males affected slightly caused by the bacterium Treponema pallidum. more often. Asymptomatic periods punctuated After an average incubation period of 21 days, by acute attacks are common. Large joints are a painless papule forms and gradually forms a affected more than the small ones, with the most clean ulcer (chancre). The secondary stage frequently involved site being the knee. This begins about 4–8 weeks after the appearance of arthropathy affects 20–40% of patients with the primary chancre. Malaise, sore throat, hemochromatosis and may be the first clinical fever, and headache are common. The diagno- feature of the disease in some patients. (Kasper sis can be made by dark-field examination and et al., pp. 2057–2058) (c) ketabton.com: The Digital Library

BIBLIOGRAPHY

Brunicardi FC, Andersen DK, Billiar TR, et al., eds. Lang RS, Hensrud DD. Clinical Preventive Medicine, Schwartz’s Principles of Surgery, 8th ed. New York, 2nd ed. Chicago, IL: AMA Press, 2004. NY: McGraw-Hill, 2005. McMillan JA, DeAngelis CD, Feigin RD, et al. Oski’s Creasy RK, Resnik R, Iams JD. Maternal–Fetal Medicine: Pediatrics: Principles and Practice, 4th ed. Philadelphia, Principles and Practice, 5th ed. Philadelphia, PA: W.B. PA: JB Lippincott, 2006. Saunders, 2004. Rom WN, ed. Environmental and Occupational Medicine, Gordis L. Epidemiology, 3rd ed. Philadelphia, PA: W.B. 3rd ed. Philadelphia, PA: Lippincott-Raven, 1998. Saunders, 2004. Rudolph CD, Rudolph AM. Pediatrics, 21st ed. New Greenberg RS, Daniels SR, Flanders WD, et al. Medical York, NY: McGraw-Hill, 2003. Epidemiology, 3rd ed. New York, NY: McGraw-Hill, Scott JR, Gibbs RS, Karlan BY, et al., eds. Danforth’s 2001. Obstetrics and Gynecology, 9th ed. Philadelphia, PA: Greenfield LJ, Mulholland M, Lillemoe KD, et al., Lippincott Williams & Wilkins, 2003. eds. Surgery: Scientific Principles and Practice, 4th Speroff L, Fritz MA. Clinical Gynecologic Endocrinology ed. Philadelphia, PA: Lippincott-Raven, 2005. and Infertility, 7th ed. Philadelphia, PA: Lippincott Kaplan HI, Sadock BJ. Synopsis of Psychiatry: Behavioral Williams & Wilkins, 2005. Sciences/Clinical Psychiatry, 9th ed. Baltimore, MD: Stoudemire A, ed., Human Behavior: An Introduction Williams & Wilkins, 2003. for Medical Students. Philadelphia-New York: Kasper DL, Braunwald E, Fauci A, et al. Harrison’s Lippincott-Raven, 1998. Principles of Internal Medicine, 16th ed. New York, Townsend CM Jr, Beauchamp RD, Evers BM, et al., NY: McGraw-Hill, 2005. eds. Sabiston Textbook of Surgery: The Biologic Kliegman RM, Greenbaum L, Lye P. Practical Basis of Modern Surgical Practice, 17th ed. Strategies in Pediatric Diagnosis and Therapy. Philadelphia, PA: W.B. Saunders, 2004. Philadelphia, PA: Elsevier, 2004. USPSTF, The Guide to Clinical Preventive Services, 2006 LaDou J. Current Occupational & Environmental Wallace RB, Doebbeling BN, eds. Maxcy-Rosenau-Last Medicine, 3rd ed. New York, NY: McGraw-Hill, 2003. Textbook of Public Health & Preventive Medicine, 14th ed. Stamford, CT: Appleton & Lange, 1998.

368 (c) ketabton.com: The Digital Library

Subject List: Practice Test 6

Question Number and Subject 23. Preventive Medicine 24. Preventive Medicine 1. Obstetrics-gynecology 25. Internal Medicine 2. Obstetrics-gynecology 26. Pediatrics 3. Pediatrics 27. Obstetrics-gynecology 4. Psychiatry 28. Obstetrics-gynecology 5. Psychiatry 29. Obstetrics-gynecology 6. Psychiatry 30. Obstetrics-gynecology 7. Preventive Medicine 31. Obstetrics-gynecology 8. Surgery 32. Internal Medicine 9. Surgery 33. Internal Medicine 10. Surgery 34. Internal Medicine 11. Surgery 35. Psychiatry 12. Internal Medicine 36. Pediatrics 13. Psychiatry 37. Internal Medicine 14. Psychiatry 38. Internal Medicine 15. Psychiatry 39. Pediatrics 16. Preventive Medicine 40. Preventive Medicine 17. Preventive Medicine 41. Internal Medicine 18. Preventive Medicine 42. Internal Medicine 19. Preventive Medicine 43. Internal Medicine 20. Surgery 44. Preventive Medicine 21. Surgery 45. Obstetrics-gynecology 22. Surgery 46. Internal Medicine

369 (c) ketabton.com: The Digital Library

This page intentionally left blank (c) ketabton.com: The Digital Library

CHAPTER 13 Practice Test 7 Questions

1. A 25-year-old man presents with a large malig- recurrent episodes over the last 2 years, for which nant melanoma on his back. There is no appar- she has been treated nonoperatively, the last episode ent lymphadenopathy (clinical stage 1). Which occurring 6 weeks ago. On examination, she weighs of the following is the most important prog- 198 lbs and is febrile to 101°F, with a BP of 120/80 nostic factor? mmHg and pulse rate of 85/min, and has mild right upper quadrant abdominal tenderness. Laboratory (A) tumor thickness data reveal a WBC of 18,000 and mildly elevated (B) tumor diameter alkaline phosphatase and bilirubin. (C) tumor location (D) the patient’s gender 2. Which of the following is the most appropriate (E) mitotic rate diagnostic imaging procedure for this patient? (A) radionuclide gallium scan Questions 2 and 3 (B) abdominal ultrasound A 55-year-old woman presents to the emergency (C) computed tomography (CT) scan department with malaise, fever, chills, anorexia, and (D) radionuclide technetium-99m sulfur nausea of 2 weeks’ duration. Her past history is per- colloid liver scan tinent for a cholecystectomy 5 years ago and chronic (E) magnetic resonance imaging (MRI) diverticulitis of the sigmoid colon, manifesting as

371

Copyright © 2008 by The McGraw-Hill Companies, Inc. Click here for terms of use. (c) ketabton.com: The Digital Library

372 13: Practice Test 7

FIG. 13-1 (Reproduced, with permission, from Zinner MJ. Maingot’s Abdominal Operations, 10th ed., Vol. 2. Stamford, CT: Appleton & Lange, 1997.)

3. An imaging procedure (Figure 13-1) demon- 4. What metabolic abnormality is the typical strates multifocal hepatic abscesses of differ- cause for the electrocardiogram (ECG) changes ent sizes. Which of the following is the most seen in Figure 13-2? appropriate course of initial management? (A) hypokalemia (A) broad-spectrum antibiotics alone (B) hypercalcemia (B) percutaneous drainage alone (C) hypomagnesemia (C) surgical drainage alone (D) hyperkalemia (D) broad-spectrum antibiotics and percuta- (E) hypocalcemia neous drainage (E) broad-spectrum antibiotics and surgical 5. Several people in a village in a developing nation drainage have contracted brucellosis. You recommend

V2 V5

V3 V6

FIG. 13-2 (c) ketabton.com: The Digital Library

Questions: 3–8 373

that pasteurization be used to eliminate Brucella (A) Inhalation of spores is followed by a sp. from milk products consumed in the com- symptom-free incubation period of 1–6 munity. Which of the following best describes days and then without warning, the this sort of preventive measure? sudden onset of a catastrophic illness, including hemoptysis that generally (A) primary prevention results in death. (B) secondary prevention (B) The spore form is commonly found in (C) tertiary prevention the intestines of cattle, which generally (D) prophylactic prevention are not made ill by anthrax. (E) quaternary prevention (C) A person with inhalational anthrax is contagious, and respiratory isolation of 6. A 34-year-old male is admitted with fever and potentially infected persons is an essen- shortness of breath to a Chicago hospital. Chest tial aspect of infection control. x-ray reveals a lobar infiltrate and blood cul- (D) Immunity following recovery from tures grow Streptococcus pneumoniae. Which of anthrax is lifelong, and applies to all the following antibiotics is best empiric choice known strains. for treatment? (E) Inhalational anthrax is treatable with (A) ceftriaxone doxycycline or ciprofloxacin or amoxi- (B) penicillin cillin if administered early. (C) vancomycin Questions 9 and 10 (D) amoxicillin (E) azithromycin A 33-year-old woman presents to the physician’s office with a 2-month history of a left breast mass. The mass 7. An 18-month-old boy presents with a 2-day is nontender, and there is no associated nipple dis- history of intermittent vomiting and irritability. charge or skin dimpling. Her past history is unremark- On physical examination, he looks uncomfort- able. Her last menstrual period (LMP) started 2 weeks able, has right lower quadrant fullness, and a ago. Current medications include birth control pills. “currant jelly” stool in his diaper. Which of the Family history is positive for breast cancer in a mater- following is the most likely diagnosis? nal grandmother. Physical examination reveals a 2-cm, firm, well-defined, very mobile, oval mass in the upper (A) constipation outer quadrant of the left breast without regional (B) gastroenteritis adenopathy. Mammography shows dense breasts. (C) intussusception Ultrasonography demonstrates a well-marginated (D) Meckel’s diverticulum solid mass (Figure 13-3). (E) volvulus

8. An envelope containing a fine powder was opened by a secretary in an office area. She fears that it may contain anthrax spores. Which of the following is true about anthrax as a weapon of mass destruction?

FIG. 13-3 (c) ketabton.com: The Digital Library

374 13: Practice Test 7

9. Which of the following is the most likely denies ever being depressed or manic, except diagnosis? while on crack. Which of the following per- sonality disorders best describes this man? (A) invasive ductal carcinoma (B) ductal carcinoma in situ (A) histrionic personality disorder (C) fibrocystic changes (B) narcissistic personality disorder (D) fibroadenoma (C) avoidant personality disorder (E) intraductal papilloma (D) borderline personality disorder (E) antisocial personality disorder 10. Which of the following should be recom- mended as part of management of the mass? 13. A 40-year-old man last used IV drugs more than 10 years ago. Since that time he has been (A) discontinuation of birth control pills a moderate drinker of beer and wine. In the (B) re-examination after the next menstrual American population at large, what is the cycle approximate prevalence of the viral illness that (C) needle biopsy is most likely to cause him to need a liver (D) excisional biopsy transplant? (E) repeat ultrasound in 6 months (A) 0.1% (B) 2% 11. A young couple is going on their honeymoon (C) 4% to Mexico and wish to avoid travelers’ diar- rhea. You advise them to avoid water, ice, (D) 7% salads, and raw vegetables. What organism is (E) 10% the usual culprit? 14. A patient smokes two packs of cigarettes per (A) Shigella day, and is an alcoholic. You tell him that he (B) Salmonella should stop smoking and that he should stop (C) Vibrio parahaemolyticus drinking alcohol. You explain that smokers (D) Escherichia coli have an average reduction in life expectancy of (E) Campylobacter jejuni 6.6 years. What is the average change in life expectancy for alcoholics? 12. A 27-year-old man is psychiatrically evaluated (A) 0.3 years in jail for making threats to kill himself. He has (B) no change been arrested for violation of parole, operat- (C) 1 year ing a drug house, and selling cocaine. He is enraged with the police for doing this, his fif- (D) -5 years teenth or “maybe twentieth” arrest since age 15. (E) -20 years He feels they are out to get him. He admits to using alcohol and “crack” since age 12; they 15. A 3-day-old infant is brought to the clinic were easy to obtain because both parents used because she is constipated. She was born at them. The highest grade achieved was 10th, term and went home within 24 hours. She has during which he quit because “both the teach- not had any stools since she was born. She has ers and principal had it in for him.” He was been breast-feeding well and has not had any truant most of the time, anyway. Currently, he vomiting. Her mother thinks that her abdomen operates his own “chop shop” and talks with is bloated. On physical examination, her weight pleasure about the “kids” who provide him is equal to her birth weight. Her abdomen is with cars for the shop and about his dealers distended, and there are no other abnormal who buy parts. “We take care of each other.” He findings. Which of the following is the most admits to hearing voices “occasionally” but likely diagnosis? (c) ketabton.com: The Digital Library

Questions: 9–20 375

(A) duodenal stenosis 18. This 23-year-old woman has a negative Pap (B) esophageal atresia smear, negative cultures for gonorrhea and (C) functional constipation chlamydia, and a negative wet smear for tri- chomonas, yeast, and bacterial vaginosis. (D) Hirschsprung’s disease Which one of the following recommendations (E) breast-feeding failure is most appropriate.

16. A 30-year-old, previously healthy woman slips (A) repeat Pap smear in 2–3 years on the ice while crossing the street and strikes (B) use of condoms with sexual intercourse her head on the pavement. Bystanders report (C) immunization with HPV vaccine that she has loss of consciousness for 2 minutes, (D) test for the presence of HPV following which she is lucid but complaining of (E) recommend sexual abstinence a headache. She is taken to a nearby emergency department. Over the next few hours, the 19. This patient returns 1 year later for a routine patient develops a decreased level of con- annual gynecologic examination. She has been sciousness, a dilated right pupil, and left hemi- in a mutually monogamous relationship since paresis. Which of the following is the most her first visit 1 year ago. Her pelvic examina- likely diagnosis? tion is normal. However, her Pap smear is now (A) right occipital intracranial hematoma interpreted as ASCUS (atypical squamous cells (B) right subdural hematoma of undetermined significance). Hybrid capture (C) right epidural hematoma testing for HPV is negative for high-risk strains. Appropriate management is (D) left epidural hematoma (E) subarachnoid hemorrhage (A) repeat Pap smear in 6 months (B) colposcopy Questions 17 through 20 (C) random cervical biopsies without colposcopy A 23-year-old woman sees you for her first visit to a (D) LEEP (loop electroexcision procedure) gynecologist. She is sexually active, exclusively cone biopsy with males. She first had intercourse at age 19 and has had 8 lifetime sexual partners. She has sexual (E) topical estrogen cream for 2 weeks fol- intercourse three to four times per week. She uses a lowed by a repeat Pap smear combination oral contraceptive. None of her part- ners have ever used a condom. She has never had a 20. Had this patient’s Pap smear returned as LSIL Pap smear. She requests a Pap smear and screening (low-grade squamous intraepithelial lesion), for sexually transmitted diseases. appropriate management of this patient is (A) vaccination with quadrivalent HPV 17. Which one of the following is the most vaccine common sexually transmitted disease? (B) colposcopy with biopsy of any colpo- (A) Trichomonas vaginalis scopic lesion (B) Candida albicans (C) repeat Pap in 3 months (C) bacterial vaginosis (D) LEEP conization (D) Chlamydia trachomatis (E) testing for high-risk HPV strains (E) human papilloma virus (HPV) (c) ketabton.com: The Digital Library

376 13: Practice Test 7

21. A 73-year-old man has been experiencing (A) pityriasis rosea increasing drowsiness and incoherence. He has (B) erythema nodosum a history of arrhythmias and has fallen twice in (C) erythema multiforme the past 2 weeks. There are no focal deficits on (D) psoriasis neurologic examination. A contrast CT scan of the head is shown in Figure 13-4. Which of the (E) bullous pemphigoid following is the treatment of choice? 23. A 32-year-old woman presents for an annual gynecologic examination and requests screen- ing for sexually transmitted infections. Her Pap smear is subsequently reported as normal, but a cervical culture is positive for gonorrhea (Neisseria gonorrhoeae). Which one of the fol- lowing is appropriate treatment? (A) ofloxacin 300 mg orally twice daily for 7 days (B) ceftriaxone 125 mg intramuscularly one time

FIG. 13-4 (C) ciprofloxacin 500 mg orally one time (D) doxycycline 100 mg orally twice daily (A) give parenteral antibiotics for 7 days (B) give antifungal therapy (E) ceftriaxone 125 mg IM and azithromycin (C) drill burr holes 1 g orally (D) observe the patient and repeat the CT scan in 1 month 24. An 18-year-old college student presents with (E) give fibrinolytic therapy sudden onset of severe pain in both legs. Her hematocrit is 26. Her peripheral blood smear is 22. A 42-year-old man has the fingernails shown in shown in Figure 13-6. Which of the following is Figure 13-5. He also has arthritis involving the the most likely diagnosis? fingers. Which of the following is the most (A) sickle cell anemia likely associated skin condition? (B) iron-deficiency anemia (C) alpha-thalassemia major (D) beta-thalassemia minor (E) anemia of chronic disease

FIG. 13-5 (Reproduced, with permission, from Zais N.The Nail in Health and Disease, 2nd ed. East Norwalk, CT: Appleton & Lange, 1990.) (c) ketabton.com: The Digital Library

Questions: 21–26 377

FIG. 13-6 Also see color insert. (Reproduced, with permission, from Hurst JW. Medicine for the Practicing Physician, 4th ed. Stamford, CT: Appleton & Lange, 1996.)

25. You apply a purified protein derivative (PPD) Questions 27 and 28 skin test to a 60-year-old healthy male from a rural area in the United States who is applying A 20-year-old college student seems very uneasy as for a job in a hospital. Between 48 and 72 hours you walk in the examination room at the student later, you measure induration to be 6 mm and health office. He tells you he has painful sores on his tell him the result was negative. Three weeks penis and admits that he has had intercourse with later, the test is repeated and induration meas- several different women in the past several months. ures 13 mm. Which of the following is the most The lesions are shown in Figure 13-7. appropriate next step in management? (A) Repeat again in 2 months. (B) Treat the new infection. (C) Consider this to probably be a booster effect. (D) Obtain sputum for culture. (E) Check again immediately.

26. A 6-month-old boy is brought to the clinic with a 1-day history of rectal bleeding. He has not had any pain or discomfort and has had no other symptoms. On physical examination, he looks well. His abdominal examination is normal. The diaper that the parents brought in is filled with dark red blood. Which of the fol- lowing is the most likely diagnosis? (A) anal fissure (B) intussusception FIG. 13-7 (Reproduced, with permission, from Wolff K, Johnson RA, (C) Meckel’s diverticulum Suurmond R. Fitzpatrick’s Color Atlas and Synopsis of Clinical (D) milk allergy Dermatology, 5th Ed. New York: McGraw-Hill, 2005:901.) (E) volvulus (c) ketabton.com: The Digital Library

378 13: Practice Test 7

27. Which of the following is the most likely (A) Seek transfer to a sit-down job on the diagnosis? light assembly line. (A) herpes zoster (B) Wear a back belt when lifting and carry- ing parcels. (B) syphilis (C) Use both hands to hold the parcel when (C) herpes simplex holding a parcel out in an extended (D) condyloma acuminatum reach. (E) lymphogranuloma venereum (D) Do light stretching, including back extensions, several times per day. 28. The student has lots of questions about this (E) Take ibuprofen 400 mg PO three times a infection. Which of the following is important day with food. for him to know about this condition? (A) He can continue sexual activity with the DIRECTIONS (Questions 31 through 34): For each lesions present as long as he uses a numbered item, select the ONE best lettered condom. option that is most closely associated with it. Each (B) Shedding of the organism will cease lettered option may be selected once, more than when the lesions are gone. once, or not at all. (C) Doxycycline is an effective treatment. For each patient with rectal bleeding, select the most (D) He is at very low risk of having a recur- likely diagnosis. rent episode if he uses condoms. (A) ischemic colitis (E) Infection can be transmitted even when (B) peptic ulcer he is asymptomatic. (C) Meckel’s diverticulum 29. What is the current recommended strategy for (D) diverticulosis of the colon lung cancer screening? (E) colonic polyp (F) ulcerative colitis (A) chest x-ray every other year in a patient with greater than 50 pack-years of (G) carcinoma of colon smoking (B) chest x-ray every other year for every- 31. A 70-year-old woman presents with vague one over 50 years old abdominal pain and bloody diarrhea 3 days after an abdominal aortic aneurysm repair. (C) chest CT scan every year in a patient with greater than 50 pack-years of 32. A 50-year-old alcoholic man with epigastric smoking pain and tarry black, foul-smelling stools. (D) chest x-ray every year for everyone over 50 years old 33. A 60-year-old previously healthy man presents (E) screening is not recommended with massive rectal bleeding.

30. A worker is returning to more active duty after 34. A 75-year-old woman with recent onset of con- having recovered from a difficult episode of stipation, small-caliber stools, and intermittent radicular low back pain. She wants to avoid a dark red blood per rectum. recurrence. Her job will be to walk around on a concrete shop floor delivering parcels that 35. A 3-year-old boy is scheduled for a tonsil- weigh 15 lbs. Which of the following recom- lectomy. As part of his preoperative evalua- mendations is most likely to help her avoid a tion, coagulation studies are obtained. They recurrence? are normal except for an increased bleeding time and a prolonged partial thromboplastin (c) ketabton.com: The Digital Library

Questions: 27–42 379

time (PTT). Which of the following is the most 40. A 38-year-old accountant with bilateral frontal likely diagnosis? headache several days per week that worsens over the course of the day. (A) idiopathic thrombocytopenia purpura (B) von Willebrand disease Questions 41 and 42 (C) classic hemophilia (D) deficiency in factors II, VII, IX, and X A 17-year-old previously healthy teenager suffers a direct blow to the upper abdomen during a hockey (E) deficiency in factor IX game. He complains of moderate epigastric pain at the time of the injury. Forty-eight hours later, the DIRECTIONS (Questions 36 through 40): For each boy develops progressively increasing bilious eme- numbered item, select the ONE best lettered sis. On arrival in the emergency department, his option that is most closely associated with it. Each abdomen is nondistended, with mild tenderness on lettered option may be selected once, more than palpation in the epigastrium and right upper quad- once, or not at all. rant. Laboratory investigations demonstrate a For each clinical setting described below, select the hematocrit of 36, WBC of 11,000, and an amylase of diagnosis that fits. 235 IU.

(A) migraine headache 41. Which of the following is the most likely (B) muscle contraction (tension) headache diagnosis? (C) subarachnoid hemorrhage (A) pancreatitis (D) subdural hematoma (B) gastritis (E) brain tumor (C) acute cholecystitis (F) cluster headache (D) duodenal hematoma (G) sinus headache (E) small-bowel perforation 36. A 42-year-old secretary with pain in her fore- 42. Which of the following is the treatment of head that is worse when she bends over. choice? 37. A 32-year-old football coach with the worst (A) immediate exploration headache of his life on the top of his head. (B) nasogastric decompression and par- enteral alimentation 38. A 50-year-old alcoholic man with pain around (C) retrocolic gastrojejunostomy his right eye and excessive tearing. (D) antacids and H2 blockers 39. A 16-year-old girl with unilateral pain in the (E) cholecystectomy frontotemporal area associated with nausea. (c) ketabton.com: The Digital Library

380 13: Practice Test 7

FIG. 13-8

43. A 44-year-old man undergoes evaluation for 45. The appropriate diagnostic studies are worsening headaches. His posteroanterior and obtained. Which of the following organisms is lateral arteriograms are shown in Figure 13-8. most likely to cause systemic infection in this What is the most likely complication? infant? (A) hypopituitarism (A) group A Streptococcus (B) subarachnoid hemorrhage (B) group B Streptococcus (C) hypercalcemia (C) Haemophilus influenzae, type B (D) tentorial herniation (D) Listeria monocytogenes (E) chronic meningitis (E) Staphylococcus aureus

Questions 44 and 45 46. Which of the following organisms is most likely to cause acute cervical adenitis in an 8-month- 44. A 1-week-old infant is brought to the emer- old child? gency department with a 1-day history of fever. He has also been irritable and been eating less (A) group A Streptococcus than usual. On physical examination, he has a (B) group B Streptococcus temperature of 39°C. He is irritable and incon- (C) H. influenzae, type B solable. Which of the following diagnostic (D) Pasteurella multocida studies should be done? (E) S. pneumoniae (A) complete blood count (CBC) (B) CBC, blood culture (C) CBC, blood culture, urinalysis (UA) (D) CBC, blood culture, UA, urine culture (E) CBC, blood culture, UA, urine culture, spinal tap (c) ketabton.com: The Digital Library

Answers and Explanations

1. (A) Among patients with clinical stage 1 pri- 4. (A) Leads V2 and V3 show U waves. U waves mary melanoma, tumor thickness has been are associated with prolonged ventricular repo- consistently shown to be the best indicator of larization and are seen in patients who are prognosis. This is true even when regional hypokalemic. Hyperkalemia is associated ini- lymph node metastases are subsequently dis- tially with a tall peaked T-wave and shortened covered (clinical stage 1 but pathologic stage 2). QT interval. Hypercalcemia can also shorten Tumors less than 0.85 mm thick are associated the QT interval. Hypomagnesemia can lead to with the most favorable prognosis, and those QRS prolongation and peaked T waves. greater than 3.65 mm in thickness are associ- Hypocalcemia is another cause of QT prolon- ated with the least favorable. Tumor thickness gation. (Kasper et al., pp. 1311–1319) is also related to the rate at which death occurs. Tumor location, mitotic rate, and the patient’s 5. (A) Primary prevention aims to preserve health gender are less powerful determinants of prog- by preventing disease before it has occurred. nosis, and tumor diameter is relatively unim- Pasteurization removes the risk of illness before portant. (Kasper et al., pp. 498–503) a population is exposed to potential disease. Secondary prevention aims to detect disease 2. (C) at an early stage in order to minimize morbid- ity. Most screening programs are considered 3. (D) secondary prevention. Tertiary prevention involves the care of established disease, with Explanations 2 and 3 attempts made to minimize the negative effects of disease, and prevent disease-related com- Pyogenic hepatic abscesses may occur fol- plications. The terms “quaternary prevention” lowing intra-abdominal sepsis, such as diver- and “prophylactic prevention” were added as ticulitis. A typical presentation may include distractors. (Lang and Hensrud, pp. 3–4) fever, abdominal pain, anorexia, and nausea of relatively short duration, along with an ele- 6. (A) Beta-lactam antibiotics are the cornerstone of vated WBC and elevation in liver function therapy for serious pneumococcal infection. At tests. The most sensitive imaging procedure is present, approximately 20% of pneumococcal the CT scan. An ultrasound is a very useful isolates in the United States have only interme- screening test, especially for suspected biliary diate susceptibility to penicillin. Among these, tree abnormalities, but may be limited in most are susceptible to third-generation obese patients or those with nonhomogeneous cephalosporins such as cefotaxime, ceftriaxone, livers. Radionuclide imaging studies are not cefepime, and cefpodoxime of penicillin resist- as useful as CT or ultrasound. The treatment ance. After culture sensitivities return, antibiotic of pyogenic abscesses includes both antibi- therapy can be tailored depending on antibiotic otics and drainage. Percutaneous drainage of susceptibilities. The newer fluoroquinolones most hepatic abscesses is safe and effective. remain highly effective against pneumococci, Surgical drainage is advised for patients with equal efficacy against penicillin-susceptible exhibiting signs of continued sepsis despite and -resistant strains. (Kaspar et al., pp. 2449–2550) percutaneous drainage and appropriate antibiotics. (Greenfield et al., pp. 909–910; Townsend 7. (C) Intussusception, a telescoping of one por- et al., pp. 1534–1539) tion of the gut into another, is the most

381 (c) ketabton.com: The Digital Library

382 13: Practice Test 7

common cause of intestinal obstruction frequent in this age group. Ductal carcinoma in between the ages of 2 months and 6 years. situ is usually nonpalpable, but may on occa- Typically, there is the onset of severe paroxysmal sion present as a mass. Fibrocystic changes can pain in a previously healthy child. Stools con- present as a mass, but a well-defined mass iden- taining both blood and mucus, known as cur- tified by sonography is more suggestive of rant jelly stools, are characteristic of this fibroadenoma. Intraductal papillomas have an disorder and are passed by 60% of these average size of 3–4 mm and are rarely palpable. patients. Constipation can cause abdominal Excisional biopsy is not necessary for all pain and can be palpated in the abdomen if fibroadenomas, and the patient should be given severe enough. It does not cause bloody stools. the option of observation if the physical exam- Patients with infectious gastroenteritis can have ination, imaging studies, and needle biopsy blood in their stools, but this patient does not (triple test) are all benign. (Donegan and Spratt, have diarrhea. Meckel’s diverticulum causes pp. 71–73, 329–330; Greenfield et al., pp. 1261–1266) painless rectal bleeding. (Kliegman, pp. 258–259; Townsend et al., pp. 2112–2113; Way and Doherty, pp. 11. (D) Diarrhea is the leading cause of illness in 1323–1324) travelers. Destination is the most important determinant of risk with rates of diarrhea as 8. (E) Inhalational anthrax occurs very rarely in high as 55% among travelers to parts of Africa, persons working with animals. It potentially Central and South America, and Southeast Asia. can result from inhalation of weaponized Most cases of travelers’ diarrhea are due to tox- spores, which can be made readily respirable. igenic E. coli, however Campylobacter appears One to six days of incubation are followed by to dominate in Asia and North Africa. Other about 3 days of mild symptoms such as low- common organisms include Salmonella, Shigella, grade fever, malaise, myalgia, fatigue, nonpro- Rotavirus, and the Norwalk agent. Except for ductive cough, and sometimes precordial Giardiasis, parasitic infections are uncommon oppression. This is followed by sudden onset of causes of traveler’s diarrhea. Antibiotic resist- dyspnea, profuse sweating, and cyanosis, which ance is a growing problem including quinolone generally proceeds to death unless treated early resistance among Campylobacter strains and in its course. Widening of the mediastinum and strains of E. coli, Shigella, and Salmonella is resist- pleural effusions are seen on chest x-ray. ant to trimethoprim-sulfamethoxazole. (Kasper, Doxycycline or ciprofloxacin or amoxicillin are pp. 727–728) approved for treatment of anthrax. Recovered persons develop immunity, but the generaliz- 12. (E) Antisocial personality disorder is character- ability and durability of this are uncertain. ized by an ongoing disregard for and violation Inhalational anthrax is not contagious as an of the rights of others. Patterns of these behav- inhalational disease. Cattle do tend to get an intes- iors and attitudes begin in early childhood or tinal form of anthrax, but this results in cata- adolescence. Persons with such a disorder fail to strophic illness for the animal. (CDC Anthrax; conform to social norms and are frequently Wallace and Doebbeling, p. 357) deceitful and manipulative to achieve their aims. They are impulsive and fail to plan ahead. 9. (D) They also tend to be irritable and may get into physical fights as well as assaultive behaviors. 10. (C) (APA, pp. 701–706; Stoudemire, 1998a, pp. 218–221)

Explanations 9 and 10 13. (B) About 1.6% (4.1 million persons) of Americans have been infected with hepatitis Fibroadenoma represents the most common C. No vaccine is available for hepatitis C, the breast tumor in young women. They are usu- results of medical treatment of hepatitis C are ally well defined, mobile, and firm on exami- mediocre after it has become chronic, and nation. Invasive ductal carcinoma is much less (c) ketabton.com: The Digital Library

Answers: 8–18 383

chronic active hepatitis C is the leading indica- A subdural hematoma develops from injury to tion for liver transplants. Persons with chronic the dural venous sinuses, with an acute sub- active hepatitis C should absolutely avoid dural hematoma presenting with signs of consumption of alcoholic beverages, as alco- rapidly increasing intracranial pressure and hol accelerates progression to liver failure. herniation. Subarachnoid hemorrhage is asso- (http://www.cdc.gov/ncidod/diseases/hepatitis/c/fact.htm; ciated with a severe headache and photopho- Wallace and Doebbeling, pp. 182–185) bia, without a significant alteration in level of consciousness. (Way and Doherty, pp. 875–876) 14. (E) Mortality associated with alcoholism often occurs relatively early in life, whether due to 17. (E) Genital HPV is now the most common sex- liver failure or suicide or motor vehicular acci- ually transmitted infection with an estimated dents. Injuries precipitated by alcoholics may 6.2 million persons newly infected annually. It also result in the deaths of others. Alcoholism is estimated that more than 80% of sexually results in large losses of productive life years. active women will acquire genital HPV by the The average change in life expectancy for alco- age of 50. Most infections are asymptomatic and holics is −20 years. This reinforces the need for self-limited. Approximately 100 strains of HPV prevention and for treatment of alcoholism. have been identified. Of these, approximately (Wallace and Doebbeling, pp. 818–849) 15 are considered high-risk strains that may cause abnormal Pap smears, including cervical 15. (D) Hirschsprung’s disease, or congenital agan- dysplasia. (CDC, No. 56, p. 4) glionic megacolon, is associated with an absence of ganglion cells in part of the colon. It 18. (C) A quadrivalent vaccine for HPV is Food is the most common cause of intestinal obstruc- and Drug Administration (FDA) approved for tion of the colon and accounts for 33% of all HPV negative women between ages 9 and 26. neonatal obstructions. Patients present with It protects against the high-risk strains 16 and constipation as neonates. Duodenal stenosis 18 that account for approximately 70% of and esophageal atresia would cause vomiting. abnormal Pap smears (ASCUS, LSIL, and high- Breast-feeding failure can also cause constipa- grade squamous intraepithelial lesion [HSIL]; tion, but this baby has regained her birth atypical squamous cells of undetermined sig- weight, so it is obvious that her intake is ade- nificance, LSIL, and HSIL). It also protects quate. Functional constipation does not occur against strains 6 and 11, the most common in the first few days of life. (Kliegman, pp. 258–259; ones associated with genital warts. It is not Townsend et al., pp. 2113–2114; Way and Doherty, pp. FDA approved for males. Condoms appear to 1321–1322) reduce the risk of HPV infection by about 70% and should be recommended, except perhaps 16. (C) An epidural hematoma most commonly in women in a long-term mutually monoga- occurs in the temporoparietal region as a result mous relationship. The American College of of hemorrhage from the middle meningeal Obstetricians and Gynecologists recommends artery. There may be an initial brief loss of con- annual screening with Pap smears in women sciousness secondary to a concussive event, under the age of 30 years, while the American followed by a variable lucid interval. As the Cancer Society recommends annual or bien- hematoma expands and exerts a mass effect, nial screening. Some organizations have sug- there is a deteriorating level of consciousness, gested that the combination of Pap smear and with tentorial herniation and eventual mid- HPV testing can safely reduce the frequency of brain compression. The patient develops an cervical cancer screening to every 2–3 years, ipsilateral dilated fixed pupil and contralateral provided both are negative. However, there are hemiparesis. Therefore, this patient with a currently little data to support this. Sexual dilated right pupil and left hemiparesis has a abstinence is the only certain way to prevent right epidural hematoma. An occipital intracra- infection with HPV, but is not practical in nial hematoma will present with visual defects. today’s society. (CDC, No. 56, p. 3, 7, 8) (c) ketabton.com: The Digital Library

384 13: Practice Test 7

19. (A) A repeat Pap smear in 6 months in women slowly. Symptoms may follow the inciting under the age of 30 years with an ASCUS Pap trauma by several weeks. Altered mental status and a negative test for high-risk HPV strains is is often more prominent than focal signs and the most appropriate management. The major- may progress from confusion to stupor to ity of young women with ASCUS Pap smears coma. Treatment consists of evacuation of the will subsequently have normal Pap smears clot via burr holes. Antibiotics and antifungal without therapeutic intervention A colposcopy agents have no role, and fibrinolytic therapy or is a consideration but is very unlikely to dis- delay in treatment could be harmful. (Kaspar close a dysplastic lesion, is expensive, and is et al., pp. 812–813) unpleasant to the patient. Random cervical biopsies in the absence of a visible cervical 22. (D) Psoriasis is a chronic skin disease charac- lesion is never indicated. A LEEP biopsy or terized by well-demarcated, erythematous conization is reserved for dysplastic lesions of papules and plaques covered by flakes or scales. the cervix. Estrogen cream is not indicated Common sites of involvement are the scalp, because young women who menstruate or who back, extensor surfaces of the knees and elbows, take an oral contraceptive will not have perianal region, and genitalia. Psoriasis is asso- atrophic epithelial cells. (ACOG Practice Bulletin, ciated with nail dystrophy (pits, grooves, or 2005) crumbling), arthritis (usually monoarthritic, involving the digits), and acute anterior uveitis 20. (B) Between 15 and 30% of women with an (common to the human lymphocyte antigen LSIL Pap smear will have high-grade cervical [HLA]-B27–related diseases). The other skin dysplasia (cervical intraepithelial neoplasia findings do not have any association with nail [CIN] 2, CIN 3) at the initial colposcopic exam- pitting. (Kasper, pp. 291–292) ination. This level of risk is similar to those whose Pap smears are ASCUS with positive 23. (E) Women whose cervical cultures are positive HPV testing or those with ASC-H (atypical for gonorrhea should be presumed to be posi- squamous cells, cannot rule out high-grade tive for chlamydia (C. trachomatis) and be lesion). This level of risk justifies a colposcopy treated for both. Currently accepted screen- with directed biopsy of an colposcopic lesion. ing of therapy is the combination of ceftriax- HPV testing in women with LSIL has no value one 125 mg IM and azithromycin 1 g orally, in screening of these women for cervical both given once. An added benefit of this reg- pathology because 83% of these women will imen is greater certainty that the woman have a positive test for high-risk HPV strain(s). receives the treatment. Doxycycline is accept- A LEEP conization should be reserved for able therapy for chlamydia, but not gonorrhea. those women whose cervical biopsies are inter- Fluorquinolones are no longer recommended preted as high-grade dysplasia. Since 83% of treatments for sexually transmitted disease women with an LSIL Pap are positive for HPV, because of the increasing prevalence of resist- vaccination is not warranted. (ACOG Practice ance to these drugs. (CDC, No. 55, p. 58, 65; CDC, Bulletin, 2005) No. 56, pp. 1–8)

21. (C) The CT scan shown in Figure 13-4 demon- 24. (A) This smear demonstrates sickling of red strates a smooth, biconvex lens-shaped mass in blood cells. Sickle cell anemia is caused by the the periphery of the right temporoparietal homozygous state for the abnormal beta-globin region. This picture is characteristic of a sub- chain gene, resulting in the production of dural hematoma that is a result of laceration of hemoglobin S (Hb S) rather than normal Hb veins bridging the subdural space. Unlike an A. During deoxygenation, polymerization of epidural hematoma, which expands quickly Hb S occurs, resulting in sickled red cells. These and progresses rapidly to coma, a subdural cells lead to hemolysis, splenic and hepatic hematoma is initially limited in size by red cell sequestration, and occlusion of increased intracranial pressure and expands microvasculature, the latter being responsible (c) ketabton.com: The Digital Library

Answers: 19–30 385

for the pain symptoms. Long bone pain is a Explanations 27 and 28 classic symptom of acute painful crisis. (Kasper, pp. 595–597) In Figure 13-4, a group of vesicles with early central crusting on a red base can be seen on 25. (C) If a person has not had a skin test for tuber- the shaft of the penis. This is characteristic of culosis within the past year, and is getting a genital herpes simplex. Syphilis is usually a baseline PPD, it is wise that it be done as a painless chancre that develops into an ulcer. “two step.” In the standard Mantoux skin test, Lymphogranuloma venereum is an organism 5 tuberculin units of PPD is injected intrader- that belongs to the Chlamydia group of parasites. mally, and the diameter of induration is read The primary genital lesion is a small painless 48–72 hours later. Induration of up to 15 mm papule that heals spontaneously. Condyloma can be considered negative in low-risk persons, acuminatum is a wart caused by HPV that may up to 10 mm in persons from areas where be pinpoint or cauliflower-like. Herpes zoster tuberculosis is endemic, and up to 5 mm in represents reactivation of the varicella-zoster persons likely to be infected or with a compro- virus and is not sexually transmitted. Patients mised immune response. A person who was with genital herpes need to be warned that infec- exposed to tuberculosis in the past, but who tion can be transmitted even when asympto- does not have active infection, may have a matic. Shedding of the organism does not stop small response to the initial PPD, but has a when the lesions are gone. Patients should stronger response after that response has been refrain from sexual activity while the lesions are boosted by exposure to PPD. A positive present because condoms are not reliably pro- response to PPD 6 months after an initial neg- tective. Patients also need to be told about the ative may, therefore, be misconstrued to be due risk of having a recurrent episode. Acyclovir, to a new infection. A “two step” better estab- valacyclovir, and famciclovir are antivirals used lishes a baseline by repeating the PPD to see to treat herpes simplex. Doxycycline is the treat- whether the immune response was strength- ment of choice for lymphogranuloma venereum. ened by the initial PPD. This is particularly (Kasper, pp. 771–773) indicated if the response to the initial PPD was equivocal and if it has been years since the 29. (E) Screening trials using chest x-ray and cyto- person last had a skin test for tuberculosis. logic sputum evaluation have failed to show a (Wallace and Doebbeling, p. 211) decrease in lung cancer mortality. Screened groups had the same number of deaths from 26. (C) Meckel’s diverticulum is present in 2–3% of lung cancer as control groups. Although there the population. Symptoms and signs can occur have been some recent small studies showing at any age but usually are manifested in the some benefits of CT scans, larger studies are cur- first 2 years of life. The most common sign is rently underway to investigate the benefit of painless rectal bleeding. Anal fissures com- using CT scanning for lung cancer screening. monly cause rectal bleeding in infants, but it is The U.S. Preventive Services Task Force a small amount of blood that typically coats (USPSTF) does not currently recommend screen- the stool. Intussusception can cause bloody ing asymptomatic patients for lung cancer with stools but usually also causes intermittent CT scans, chest x-ray, or sputum cytology. abdominal pain. Milk allergy would not occur (USPSTF, 2006) this acutely. If the patient had a bloody stool with a volvulus, he would be ill-appearing and 30. (D) Sitting actually increases the pressure on lumbar disks relative to standing. National have abdominal pain. (Kliegman, pp. 323–331; Townsend et al., pp. 1366–1368) Institute for Occupational Safety and Health (NIOSH) has not found backbelts to be helpful 27. (C) in prevention of low back pain. Holding weight out away from the body greatly increases pres- 28. (E) sure on the lumbar disks and should be avoided. (c) ketabton.com: The Digital Library

386 13: Practice Test 7

Weight should be kept close to the body. Being bleeding time is normal in hemophilia and active, including light stretching (generally back factor IX deficiency, and the PTT is normal in extension exercises such as McKenzie exercises, idiopathic thrombocytopenic purpura. Factors if this does not produce radicular pain), and II, VII, IX, and X are vitamin-K–dependent and maintenance of good posture may help control may be deficient in a variety of disorders, includ- symptoms. (LaDou, pp. 56–57; McCunney, p. 324; ing liver disease, malabsorption, and altered Wallace and Doebbeling, pp. 656–657) bowel flora. (Behrman et al., pp. 1660–1661)

31. (A) 36. (G)

32. (B) 37. (C)

33. (D) 38. (F)

34. (G) 39. (A)

Explanations 31 through 34 40. (B)

Ischemic colitis after aortic aneurysm repair is a Explanations 36 through 40 result of ligation of the inferior mesenteric artery, with compromise to the perfusion of the distal Sinus headache is usually acute in onset, worse colon. It is associated with crampy lower on awakening, better on standing, and can abdominal pain, and bloody stools from worsen on bending over. Often, there is a puru- mucosal sloughing. Bleeding from a gastroin- lent nasal discharge and pain over the involved testinal source above the ligament of Treitz (such sinus. Subarachnoid hemorrhage is often as a peptic ulcer) will result in melena, which is described as “the worst headache of my life” characterized by tarry, foul-smelling stools. and is very abrupt in onset. Cluster headache Meckel’s diverticulum presents with painless predominantly occurs in middle-aged men, rectal bleeding in children. Diverticulosis is the and its typical presentation involves intense most common cause of massive rectal bleeding, unilateral headache that is searing, stabbing, which is usually bright red and painless. Colonic and accompanied by ipsilateral lacrimation, polyps may be singular or multiple, and usually nasal stuffiness, and facial flushing. Alcohol is bleed at the time of stooling, or when the polyp believed to be a precipitant, although alcohol is sloughs spontaneously. Patients with ulcerative well tolerated between attacks. Migraine colitis may have diarrhea, abdominal cramps, headache is more common in women, and and weight loss, with a clinical course that may usually begins in childhood or young adult be associated with a severe exacerbation. With life. Migraine may or may not have an aura, but severe colitis, there is increased frequency of is unilateral, of pulsating quality, and accom- bloody stools, and signs of systemic toxicity panied by nausea or vomiting, photophobia, or (fever and leukocytosis). Carcinoma of the colon phonophobia. Subdural hematoma is also a should be suspected in a patient presenting with subtle condition with earlier head trauma that rectal bleeding and a history of a change in may be forgotten until the patient displays bowel habits. (Greenfield et al., pp. 1051–1064; Way mental status changes or focal neurologic and Doherty, pp. 717–720) deficits. A patient with a brain tumor may be asymptomatic other than headache initially, but 35. (B) von Willebrand disease is an autosomal- usually neurologic deficits will be found as the dominant disorder. von Willebrand disease is time progresses. Tension headache is typically characterized by a prolonged bleeding time, worse as the day goes on, bilateral in the fron- various abnormalities of the factor VIII com- totemporal area, and described as a tight band plex, and a prolonged PTT. In contrast, the around the head. (Kasper, pp. 85–94) (c) ketabton.com: The Digital Library

Answers: 31–46 387

41. (D) resulting in a subarachnoid hemorrhage with sudden onset of severe headache and 42. (B) meningeal symptoms and signs (e.g., nuchal rigidity, photophobia). Rapid progression to Explanations 41 and 42 stroke, coma, or death may follow. Intracranial aneurysms are not associated with hypercal- A direct blow to the epigastrium compresses cemia, hypopituitarism, or chronic meningitis the duodenum against the vertebral column, and rarely cause tentorial herniation without with a shearing injury resulting in an intramu- rupturing. Surgical approaches to such large ral hematoma. With the breakdown of hemo- saccular intracranial aneurysms (>2.5 cm) globin in the hematoma, there is an increase in include excision and ligation. The risk of rupture the oncotic pressure and imbibement of fluid is about 6% in the first year after identification into the hematoma, leading to progressive and may remain high indefinitely. (Kasper, pp. obstruction. This can contribute to the delay in 2387–2392) presentation, which is commonly several days after the initial injury. Bilious vomiting is sec- 44. (E) Infants with fever, especially those less than ondary to obstruction at the junction of the 1 month of age, are at high risk for serious bac- second and third parts of the duodenum. There terial infection. Approximately 9% of these may be mild hyperamylasemia, related either infants who look well on examination will have to the duodenal wall injury or to an associated a serious bacterial infection. History and phys- pancreatic injury. Pancreatitis from blunt ical examination alone are not sensitive in abdominal trauma usually presents at the outset detecting these infections. Laboratory evalua- with progressive abdominal tenderness and sig- tion is essential. Bacteremia, urinary tract infec- nificant elevation of serum amylase. There is tion, and meningitis are all possibilities in this often a delay in the diagnosis of small-bowel age group, and so all of these tests must be perforation, with the development of tachycar- done. (Kliegman, pp. 1066–1067) dia, fever, and peritonitis on abdominal exami- nation. Gastritis is usually secondary to an acute 45. (B) Group B Streptococcus is the most common infectious agent, and cholecystitis occurs in the cause of neonatal sepsis and meningitis in presence of cholelithiasis. Patients with obstruc- many medical centers. Gram-negative organ- tion from intramural duodenal hematoma are isms, L. monocytogenes, and Salmonella are also managed with nasogastric decompression and possible, but less common. H. influenzae, type b, parenteral nutrition. In the majority of cases, the causes invasive disease, but the incidence has obstruction will resolve in 7–10 days. Operative decreased dramatically since universal immu- intervention is not usually required. (Greenfield et nization was instituted. S. aureus would be an al., p. 435; Townsend et al., pp. 515–517; Way and unusual organism unless the infant had some Doherty, pp. 232–233) type of break in the skin. (Kliegman, pp. 1066–1067)

43. (B) The arteriograms in Figure 13-8 demon- 46. (A) Group A Streptococcus is the most likely strate a large aneurysm arising from the basilar organism. The next most common would be artery. Intracranial aneurysms occasionally S. aureus. H. influenzae and S. pneumoniae do not present with new onset or worsening of cause adenitis. P. multocida is an organism found headaches or may be asymptomatic and found in the mouths of cats, which can cause a wound coincidentally during evaluation of an unre- infection after a cat bite. (Kliegman, p. 973, 974) lated disorder. Frequently, they leak or rupture, (c) ketabton.com: The Digital Library

BIBLIOGRAPHY

ACOG Committee on Practice Bulletins. ACOG Kasper DL, Braunwald E, Fauci A, et al. Harrison’s Practice Bulletin No. 66: Management of abnor- Principles of Internal Medicine, 16th ed. New York, mal cervical cytology and histology. Obstet Gynecol NY: McGraw-Hill, 2005. 2005;106:645–64. Kliegman RM, Greenbaum L, Lye P. Practical Strategies American Psychiatric Association (APA). Diagnostic in Pediatric Diagnosis and Therapy. Philadelphia, PA: and Statistical Manual of Mental Disorders, 4th ed. Text Elsevier, 2004. Revision. Washington, DC: American Psychiatric LaDou J. Current Occupational & Environmental Medicine, Association, 2000. 3rd ed. New York, NY: McGraw-Hill, 2003. Behrman RE, Kliegman RM, Jenson HB. Nelson Lang RS, Hensrud DD. Clinical Preventive Medicine, Textbook of Pediatrics, 17th ed. Philadelphia, PA: 2nd ed. Chicago, IL: AMA Press, 2004. W.B. Saunders, 2006. McCunney RJ. A Practical Approach to Occupational Centers for Disease Control and Prevention. Anthrax. and Environmental Medicine, 3rd ed. Philadelphia, 2007. Available at: HYPERLINK “http://www. PA: Lippincott Williams & Wilkins, 2003. cdc.gov/” www.cdc.gov/ Musher Centers for Disease Control and Prevention. MMWR Ropper AH, Brown RH, Chang AE Oncology: An Morbidity and Mortality Weekly Report. Evidence Based Approach, New York, Springer- Quadrivalent Human Papillomavirus Vaccine: Verlag, 2006. Recommendations of the Advisory Committee on Stoudemire A, ed., Human Behavior: An Introduction Immunization Practices (ACIP) No. 56(RR02). for Medical Students. Philadelphia-New York: March 23, 2007, pp. 1–24. Lippincott-Raven, 1998. Donegan WL, Spratt JS, eds. Cancer of the Breast, 5th ed. Townsend CM Jr, Beauchamp RD, Evers BM, et al., Philadelphia, PA: W.B. Saunders, 2002. eds. Sabiston Textbook of Surgery: The Biologic Basis of Greenberg RS, Daniels SR, Flanders WD, et al. Medical Modern Surgical Practice, 17th ed. Philadelphia, PA: Epidemiology, 3rd ed. New York, NY: McGraw-Hill, W.B. Saunders, 2004. 2001. USPSTF, The Guide to Clinical Preventive Services, 2006. Greenfield LJ, Mulholland M, Lillemoe KD, et al., Wallace RB, Doebbeling BN, eds. Maxcy-Rosenau- eds. Surgery: Scientific Principles and Practice, 4th ed. Last Textbook of Public Health & Preventive Medicine, Philadelphia, PA: Lippincott-Raven, 2005. 14th ed. Stamford, CT: Appleton & Lange, 1998. Kaplan HI, Sadock BJ. Synopsis of Psychiatry: Behavioral Way LW, Doherty GM, eds. Current Surgical Sciences/Clinical Psychiatry, 9th ed. Baltimore, MD: Diagnosis and Treatment, 124th ed. New York, NY: Williams & Wilkins, 2003. McGraw-Hill, 2006.

388 (c) ketabton.com: The Digital Library

Subject List: Practice Test 7

Question Number and Subject 23. Internal Medicine 24. Internal Medicine 1. Internal Medicine 25. Preventive Medicine 2. Surgery 26. Pediatrics and Surgery 3. Surgery 27. Internal Medicine 4. Internal Medicine 28. Internal Medicine 5. Preventive Medicine 29. Preventive Medicine 6. Internal Medicine 30. Preventive Medicine 7. Surgery 31. Surgery 8. Preventive Medicine 32. Surgery 9. Surgery 33. Surgery 10. Surgery 34. Surgery 11. Internal Medicine 35. Pediatrics 12. Psychiatry 36. Internal Medicine 13. Preventive Medicine 37. Internal Medicine 14. Preventive Medicine 38. Internal Medicine 15. Pediatrics and Surgery 39. Internal Medicine 16. Surgery 40. Internal Medicine 17. Obstetrics-gynecology 41. Surgery 18. Obstetrics-gynecology 42. Surgery 19. Obstetrics-gynecology 43. Internal Medicine 20. Obstetrics-gynecology 44. Pediatrics 21. Surgery 45. Pediatrics 22. Internal Medicine 46. Pediatrics

389 (c) ketabton.com: The Digital Library

This page intentionally left blank (c) ketabton.com: The Digital Library

CHAPTER 14 Practice Test 8 Questions

1. A 9-year-old girl with a history of intermittent 3. Over the next 6 hours, the patient does not wheezing for several years is brought to the pedi- improve. A plain abdominal radiograph is atrician. The child has not been on medications for shown in Figure 14-1. Which of the following is some time. Physical examination reveals a febrile the most appropriate next step in management? child who is agitated and has perioral cyanosis. Intercostal and suprasternal retractions are pres- ent. The breath sounds are quiet, and wheezing is audible bilaterally. Which of the following is the most appropriate initial intervention? (A) Prescribe IV aminophylline. (B) Obtain a chest film. (C) Prescribe nebulized cromolyn sodium. (D) Obtain a complete blood count (CBC) and blood culture. (E) Prescribe nebulized albuterol.

Questions 2 and 3

A 53-year-old male with a prior history of a sig- moid colon resection for diverticular disease presents to the emergency department with a 2-day history of crampy abdominal pain and progressive abdomi- nal distention. He has had several episodes of bile-stained vomiting. His last bowel movement was 24 hours before presentation. On examination, FIG. 14-1 (Reproduced, with permission, from Zinner MJ. Maingot’s the patient has moderate abdominal distention, Abdominal Operations, 10th ed., Vol. 2. Stamford, CT: Appleton & with intermittent high-pitched bowel sounds. Lange, 1997.)

2. Initial management and evaluation of this (A) to continue serial examinations and patient should include which of the following? repeat abdominal radiographs in the morning (A) IV fluids and nasogastric decompression, (B) insertion of a rectal tube for colonic followed by plain abdominal radiographs decompression (B) IV fluids and analgesics (C) insertion of a Cantor tube for small- (C) plain abdominal radiographs bowel decompression (D) a barium enema (D) colonoscopy (E) a Fleet’s enema and stool softeners (E) urgent exploratory laparotomy

391

Copyright © 2008 by The McGraw-Hill Companies, Inc. Click here for terms of use. (c) ketabton.com: The Digital Library

392 14: Practice Test 8

4. An investigator studies breast cancer. She enrolls (A) often found as multiple lesions 300 disease-free women to take part in a 10-year (B) often associated with periodontal randomized prospective investigation designed disease to determine whether specific risk factors are (C) often due to aerobes responsible for disease. (D) often due to gram-negative bacilli Which of the following issues are inherent dis- (E) second-generation cephalosporin is the advantages of this type of study? drug of choice

(A) It has a large potential for loss to 6. At the conclusion of the study, the investigator follow-up. calculates the relative risk of developing breast (B) It is difficult to choose an appropriate cancer for a particular risk factor. Which of the control group. following calculations should she do to calcu- (C) It is not good for studying more than late relative risk? one outcome. (A) (incidence in exposed persons + (D) It not good for studying rates. incidence in unexposed persons) – (E) It is not good for studying rare risk (incidence in unexposed persons) factors. (B) (incidence in exposed persons) – (incidence in unexposed persons) 5. A 42-year-old alcoholic woman is admitted to (C) (incidence in exposed persons – the hospital after suffering a seizure. She is incidence in unexposed persons)/ febrile, cachectic, ill-kempt, and has poor oral incidence in exposed persons hygiene. A chest examination is normal; her (D) (incidence in exposed persons)/ chest x-ray is shown in Figure 14-2. Which of (incidence in unexposed persons) the following is true regarding this lesion? (E) (incidence in exposed persons × incidence in unexposed persons)/100

Questions 7 and 8

A 61-year-old man is admitted to the cardiac care unit (CCU) with crushing chest pain and the elec- trocardiogram (ECG) shown in Figure 14-3. He is agitated, pale, and diaphoretic. Peripheral pulses are weak, and systolic BP is 90 mmHg. Neck veins are distended, Kussmaul’s sign is present, but lungs

are clear. There is an S3 gallop but no murmur.

FIG. 14-2 (c) ketabton.com: The Digital Library

Questions: 4–10 393

FIG. 14-3

7. Which of the following is the most likely cause (A) Injectable vaccine was less expensive of the hypotension? than oral vaccine due to cost of preparation. (A) acute mitral regurgitation (B) The risk of contracting vaccine- (B) aortic dissection associated paralytic polio from oral (C) bacterial endocarditis vaccine is now higher than the risk of (D) right ventricular infarct contracting wild-strain polio. (E) pericardial tamponade (C) No cases of wild-strain paralytic polio had been reported in 15 years. 8. Which of the following is the most appropriate (D) Injectable vaccine is more protective next step in the patient’s management? than oral vaccine. (A) computed tomography (CT) scan of the (E) Injectable polio vaccine is available chest worldwide. (B) intubation and mechanical ventilation (C) pericardiocentesis 10. A 4-year-old girl is brought to the office 2 days after she was bitten by her neighbor’s cat. The (D) IV fluids and emergent cardiac bite is on her hand and occurred when she was catheterization teasing the cat. On physical examination, there (E) blood cultures and empiric antibiotics are two closed puncture sites with erythema and induration around the wound. Which of 9. In 2000, the Public Health Advisory Committee the following organisms is most likely to cause on Immunization Practices changed its recom- the infection in this wound? mendation for immunizing children under 4 years old against polio. The old recommen- (A) Rochalimaea henselae dation included use of oral polio vaccine. The (B) Eikenella corrodens new recommendation is for a series of four (C) Pasteurella multocida doses of injectable polio vaccine. This recom- (D) Francisella tularensis mendation reflected which of the following (E) Spirillum minus facts? (c) ketabton.com: The Digital Library

394 14: Practice Test 8

11. A healthy 28-year-old woman begins prenatal (C) She is in the active phase of the first care at 8 weeks’ gestation. Her hemoglobin con- stage of labor. centration is 13.5 g/dL. She began to take an (D) She is in the second stage of labor. over-the-counter multivitamin about 3 months (E) She is in the third stage of labor. before conception. Which of the following statements about folic acid is correct? 14. At a +1 station, which one of the following (A) 0.4–1.0 mg daily reduces the risk of a statements best describes the location of the neural tube defect (NTD). fetal vertex? (B) It prevents a microcytic anemia. (A) It is floating above the pelvic inlet. (C) The amount should be increased to (B) It is below the pelvic inlet and above the 1–2 mg/day in a woman who previously ischial spines. had a child with spina bifida. (C) It is at the level of the ischial spines. (D) Prenatal vitamins should be started (D) It is below the ischial spines and above when pregnancy is recognized. the perineum (E) Prescription prenatal vitamins have (E) It is visible at the perineum during a 0.4 mg of folic acid. contraction.

12. In the emergency room, you see a 25-year-old 15. An 8-month-old child has a temperature of female, with no history of human immunode- 38.7°C. Physical examination reveals a non- ficiency virus (HIV) infection, who was just toxic appearing child with no focal signs of sexually assaulted. She wants an HIV test infection on physical examination. A blood cul- immediately, which you provide. You propose ture is obtained and is positive in 35 hours. follow-up testing with an enzyme-linked Which of the following organisms is most likely immunosorbent assay (ELISA) test based on to be found in the blood culture? the delay between infection and appearance of detectable antibodies. (A) Escherichia coli (B) Haemophilus influenzae In general, ELISA detectable antibodies appear (C) Staphylococcus aureus how long after infection with HIV? (D) Streptococcus pneumoniae (A) 2–10 days (E) Neisseria meningitidis (B) 5–21 days (C) 10 days to 1 month 16. A 7-year-old girl is brought to the clinic with a 2-day history of right ear pain. She has not had (D) 2–9 weeks fever, congestion, or other symptoms. On phys- (E) 4 weeks to 3 months ical examination, she is afebrile. She has pain when you move the pinna. There is drainage in Questions 13 and 14 the right ear canal that obscures the tympanic 13. A healthy, 28-year-old, primigravid woman at membrane. Which of the following is the most 40 weeks’ gestation has had regular uterine likely diagnosis? contractions for 7 hours. Her cervix is 7-cm (A) acute otitis media with perforation dilated and 70% effaced. The vertex is at a +1 (B) contact dermatitis station. Which one of the following statements (C) foreign body in the ear canal is correct? (D) mastoiditis (A) She is having false labor pains. (E) otitis externa (B) She is in the latent phase of the first stage of labor. (c) ketabton.com: The Digital Library

Questions: 11–21 395

17. A worker from a company that uses and ships 20. A 35-year-old granite quarry worker presents a lot of organic solvents sees you because he to his primary care physician with the com- feels very fatigued. He appears to be anemic, plaint of a persistent productive cough. A chest and a CBC reveals anemia, leukopenia, and x-ray reveals nodules in the middle and upper thrombocytopenia. Which of the following is lung fields. A tuberculin skin test is done and is most likely to cause this profile? read as positive. In addition to this patient’s tuberculosis, what comorbid condition is highly (A) acetone likely in this patient? (B) mushroom toxin (C) toluene (A) asbestosis (D) benzene (B) silicosis (E) diesel exhaust (C) coal worker’s pneumoconiosis (D) berylliosis 18. You refer a 45-year-old male with type I dia- (E) siderosis betes to an ophthalmologist because he is com- plaining of blurred vision. The visit to the 21. A fox bites a man camping in a rural area of ophthalmologist’s office is considered which Texas. The fox escapes after the incident and of the following types of preventive measures? the man is unable to capture or track him. Which of the following is the most appropriate (A) primary prevention follow-up therapy for this individual? (B) secondary prevention (C) tertiary prevention (A) postexposure therapy with both rabies vaccine and immunoglobulin (D) quaternary prevention (B) postexposure therapy for tetanus based (E) predisease prevention on immunization status only, fox rabies is uncommon in North America 19. Based on evidence-based practices you are asked to prioritize who will be receiving (C) postexposure therapy with rabies vac- Papanicolaou (Pap) tests. For which one of the cine and tetanus toxoid regardless of following women would you be most likely to immunization status perform a Pap test? (D) postexposure therapy with rabies vac- cine, rabies immunoglobulin, and (A) A 78-year-old female who has three tetanus prophylaxis based on the indi- grown children, a history of normal Pap vidual’s immunization status tests, and no Pap test in the past 5 years. (E) postexposure therapy with rabies (B) A 27-year-old female who had a total immunoglobulin and tetanus toxoid hysterectomy 1 year ago, and who had a history of noninvasive cancer of the cervix. (C) A 38-year-old female who has never been sexually active, and who has not had a Pap test in the past 10 years. (D) A 37-year-old female who is sexually active, has had normal Pap tests every 3 years since age 20, and had a normal Pap test 1 year ago. (E) A 30-year-old female who is sexually active, has a history of normal Pap tests, had a total hysterectomy for uterine fibroids, and has not had a Pap test in 10 years. (c) ketabton.com: The Digital Library

396 14: Practice Test 8

DIRECTIONS (Questions 22 through 26): Each set her psychiatrist for a routine visit and com- of matching Questions in this section consists of a plains that she might be getting “the flu.” She list of lettered options followed by several num- describes symptoms of nausea and diarrhea. bered items. For each item, select the ONE best let- As she speaks, the psychiatrist notes that she tered option that is most closely associated with it. slurs her words and is tremulous and ataxic. Each lettered option may be selected once, more than once, or not at all. 26. A young man with a history of depression has responded well to an antidepressant. During a Certain side effects are typically caused by psychi- routine appointment with his primary care atric medication. In the case reports below, identify physician, he was noted to be hypertensive. the medication that is most likely the cause of the side He had no previous history of hypertension. effect described. 27. A 15-year-old boy comes to the emergency (A) phenelzine department with sudden onset of respiratory (B) fluvoxamine distress. He states that he was studying 1 hour (C) haloperidol ago, when he developed chest pain and dysp- (D) imipramine nea. He has no history of any previous respi- (E) lithium carbonate ratory problems. On physical examination, he (F) lorazepam is tall and thin, tachypneic, has moderate retractions, and decreased breath sounds over (G) venlafaxine the left lung fields. On percussion, the left lung (H) trazodone field is more tympanic than the right. Which of the following is the most likely diagnosis? 22. A patient with a history of atypical depression and treatment resistance presents to the emer- (A) aspiration of a foreign body gency room with a severe headache, fever, and (B) acute bacterial pneumonia severe hypertension. She reports she ate some (C) empyema aged cheese for lunch less than 1 hour before (D) acute bronchitis the headache started. (E) pneumothorax 23. A young man was successfully medically Questions 28 and 29 treated for feelings of discouragement, an early morning awakening sleep disturbance, poor A 2-week-old boy is brought to your office for a appetite, and sense of hopelessness. Several checkup. He has been doing well at home. The results months after starting this medication, he sought of his newborn screen indicate that he has sickle cell help in the emergency department because of a disease (hemoglobin SS disease). penile erection that would not subside. 28. Which of the following is the most important 24. A 35-year-old woman with a history of major next step in his management? depression with psychosis appeared in the emergency department. Her eyes were, as she (A) avoiding heat exposure said, “locked up in my head.” She had started a (B) immunizing with pneumococcal vaccine drug approximately 2 weeks before this episode. (C) prescribing folic acid supplements (D) prescribing iron supplements 25. A 45-year-old woman with a history of erratic, (E) prescribing prophylactic penicillin hostile behavior, pressured speech, flight of ideas, and impulsiveness has had her symp- toms controlled with medication. She is seen by (c) ketabton.com: The Digital Library

Questions: 22–32 397

29. Six months later, this same patient returns to 31. You are reviewing data in order to determine your office with a 3-day history of lethargy and whether there is an association between meas- fever. He has also had rhinorrhea and a cough. ured systolic and diastolic blood pressures in On physical examination, he is pale, tachy- the general population, and what is the mag- cardic, and has a left upper quadrant mass. His nitude of the trend. Which of the following sta- hemoglobin is 4 g/dL, platelet count is 100,000, tistical analysis techniques would be most and white blood cell (WBC) is 15,000 with 50% appropriate? segmented neutrophils. His reticulocyte count (A) Wilcoxon Two-Group Rank Sum test is 15%. Which of the following is the most likely diagnosis? (B) Mann-Whitney U-test (C) Wilcoxon Signed-Ranks test (A) acute chest syndrome (D) linear model of regression (B) acute splenic sequestration (E) odds ratio (C) aplastic crisis (D) sepsis 32. A 12-month-old boy is brought to the emer- (E) vasoocclusive crisis gency department after having a generalized seizure at home. He has also vomited three 30. The concerned mother of a 13-year-old girl times and been very irritable. He has never had states that her daughter has been sleeping late, a seizure previously and in general is in good easily becomes irritated with her younger health and has received all of his routine immu- brother, and insists on locking her bedroom nizations. On examination, he has a tempera- door. She is doing well in school and is on the ture of 40°C. He is irritable and does not seem middle school soccer team. On physical exam- to recognize his mother. A cerebrospinal fluid ination, she is Tanner stage 4 and the rest of her (CSF) sample obtained shows the following: physical findings are normal. Which of the fol- 2000 WBCs (98% segmented neutrophils), pro- lowing is the most appropriate next step in her tein of 155, and glucose of 20 mg/dL with a management? serum glucose of 95 mg/dL. Which of the following organisms is the most likely cause of (A) checking her thyroid function his infection? (B) obtaining a urine drug screen (C) ordering a pregnancy test (A) H. influenzae (D) questioning the patient about possible (B) group A Streptococcus abuse (C) group B Streptococcus (GBS) (E) reassuring the mother (D) S. pneumoniae (E) Mycobacterium tuberculosis (c) ketabton.com: The Digital Library

398 14: Practice Test 8

33. A 2-month-old infant is brought to the clinic with a diaper rash. Her mother states that it started as a red rash in the diaper area 1 week ago. She treated it with over-the-counter diaper cream, but it continued to spread and then began to peel 2 days ago. On physical exami- nation, she is afebrile, has a diaper rash (Figure 14-4), and otherwise has a normal physical examination. Which of the following is the most likely diagnosis?

FIG. 14-5 Also see color insert. (Courtesy of M. L. Williams.)

(A) contact dermatitis (B) erythema marginata (C) erythema migrans (D) erythema multiforme (E) tinea corporis FIG. 14-4 Also see color insert. (Courtesy of M. L. Williams. Reprinted with permission of Yearbook Medical Publishers.) 35. During a daytime church picnic, a wild fox suddenly appears from the woods and attacks a young woman, biting her several times on (A) allergic dermatitis the leg. The fox then proceeds to wander (B) bullous impetigo around the picnic, attacking brightly colored (C) Candida dermatitis coolers. A man attempts to catch the fox and is (D) irritant dermatitis bitten several times on his hands and head in the process. Animal control officers arrive and (E) seborrheic dermatitis safely trap the fox. Which of the following is the best course of action? 34. A 7-year-old girl is brought to the clinic with a rash. Her mother states that the rash has devel- (A) Observe the fox for 10 days. oped over the last several days. It began as a (B) Submit the fox head for rabies testing small red bump and has continued to grow and treat victims based on results. (Figure 14-5). She has also had fever, headache, (C) Begin treatment with tetanus antitoxin. and malaise over the last several days. On (D) Begin treatment of bite victims with physical examination, she has the rash and oth- rabies vaccine. erwise has a normal examination. Which of the (E) Begin treatment of bite victims with following is the most likely diagnosis? rabies immune globulin and rabies vaccine. (c) ketabton.com: The Digital Library

Questions: 33–40 399

36. A 2-year-old boy is diagnosed with severe (A) mean mental retardation after a referral for lack of (B) mode any language development. Which of the fol- (C) median lowing epidemiologic characteristics is seen in (D) midrange severe mental retardation? (E) midmode (A) female predominance (B) about 80% correlation with advanced Questions 39 and 40 maternal age A young man develops wheezing after eating lobster (C) prevalence nearly evenly distributed and presents to the emergency department 15 minutes across social classes later with persistent breathing difficulty. (D) life expectancy equal to the general population 39. Which of the following is the most important (E) less than half of cases are genetic in initial treatment? origin (A) oral beta-blocker 37. A researcher is conducting a study to deter- (B) topical steroid mine whether a new medication causes insom- (C) IV steroid nia. He surveys a group of study individuals (D) subcutaneous epinephrine taking the medication. The survey tool asks (E) intramuscular diphenhydramine individuals to score their responses on a scale, rating the quality of their sleep as being poor, 40. Which of the following is associated with this fair, good, or excellent. The data that are being type of immediate hypersensitivity? collected by the researcher can be categorized as which of the following? (A) immunoglobulin A (IgA) binding to mast cells or basophil membranes (A) dimensional (B) delayed skin rash (B) ordinal (C) immediate appearance of symptoms, (C) binary with self-limited resolution within one- (D) nominal half hour (E) categorical (D) hypertensive response (E) symptoms including laryngeal edema 38. An individual is studying length of survival and bronchospasm associated with chemotherapy for a particular cancer. He has looked at 25 individuals who have survival times ranging from 2 months to 2 years. Three of his study participants are still alive so their true survival time is still unknown. Which of the following measures of central tendency can be used at this point to analyze the data? (c) ketabton.com: The Digital Library

400 14: Practice Test 8

41. A 21-year-old female comes to you for infor- (C) daily doses of bismuth subsalicylate mation about HIV and condom use. She has (Pepto-Bismol) and ciprofloxacin intermittently had sexual intercourse with sev- (D) administration of a dose of eral different men over the past 3 years. She immunoglobulin immediately after consistently assures you that condoms are returning used. She wants to know whether condoms (E) vaccination with hepatitis A vaccine will protect her from HIV transmission from an 10 years earlier HIV positive partner. Which of the following about condoms and HIV is true? 43. Your next patient is a 4-month-old infant who (A) If a woman has frequent sexual inter- is returning to have her ear checked. You diag- course with an HIV positive man for a nosed her with otitis media 2 weeks ago, and period of years, the risk of transmission she has taken amoxicillin for 10 days. She is of HIV with consistent condom use feeling well, and her mother’s only concern is becomes similar to the risk of transmis- that she has developed a diaper rash over the sion without condoms. last 3 days. She has been using emollient (B) Latex condoms are less effective than creams on it, which have not helped. On phys- natural membrane condoms in preven- ical examination, there are no abnormal find- tion of HIV transmission. ings except for the rash (Figure 14-6). Which of the following is the most likely diagnosis? (C) Condoms treated with nonoxynol-9 are more than twice as effective at prevent- ing transmission of HIV when com- pared with condoms not treated with this spermicide. (D) Abstinence is the only way to prevent HIV transmission. (E) Lubrication with petroleum jelly increases the likelihood that latex condoms fail.

42. A 25-year-old male develops low-grade fever, loss of appetite, dark urine, and then jaundice 3 weeks after a 2-week long humanitarian relief tour requiring extensive backcountry travel in an underdeveloped country. During the relief FIG. 14-6 Also see color insert. (Courtesy of Neil S. Prose.) effort, he had close contact with young chil- dren and lived in the homes of area residents. He relied on locally prepared food and water (A) allergic dermatitis sources. A laboratory test shows a high titer of (B) bullous impetigo immunoglobulin M (IgM) antihepatitis A virus (C) Candida dermatitis (HAV). Which of the following would have (D) irritant dermatitis been the best method to prevent this from (E) seborrheic dermatitis happening? (A) administration of a dose of immunoglobulin 1 week before leaving (B) ingesting only foods cooked to high temperature and bottled water (c) ketabton.com: The Digital Library

Questions: 41–46 401

44. A scientist working with ionizing radiation 46. A 6-year-old girl is brought to the office with a experiences a malfunction with his personal rash. Her mother states that 1 week ago she protective equipment and is exposed to about had a fever, headache, and malaise, which three Sieverts of radiation. What is the most lasted for 3 days and then resolved. Now she likely form of acute radiation syndrome that he feels well except for some pruritus with the will experience? rash. Her mother has also noted that the rash becomes more prominent when she is outside (A) cerebral form in the sun. On examination, she has no abnor- (B) gastrointestinal form mal findings, except for the rash. Her cheeks (C) hematopoietic form are intensely red, and on her extremities she (D) pulmonary form has the rash shown in Figure 14-8 (see color (E) renal form plate 7). Which of the following is the most likely diagnosis? 45. A 7-day-old infant is brought to the clinic with a rash. He was born at term and is the first child in the family. He had no problems at birth and has been doing well at home. On exami- nation, he is afebrile. Other than the rash (Figure 14-7, see color plate 6), which is found only on the top of his scalp, his examination is normal. Which of the following organisms is most likely the cause of his rash?

FIG. 14-8 Also see color insert. (Courtesy of I. Frieden.)

(A) erythema infectiosum (B) Kawasaki disease

FIG. 14-7 Also see color insert. (Courtesy of Neil S. Prose.) (C) measles (D) roseola (E) scarlet fever (A) enterovirus (B) GBS (C) herpes simplex virus (D) S. aureus (E) varicella-zoster virus (c) ketabton.com: The Digital Library

Answers and Explanations

1. (E) This patient is in severe distress and needs and electrolyte abnormalities. Continued non- immediate therapy. Inhaled albuterol is the ini- operative management is appropriate only tial treatment of choice. Cromolyn sodium is a in patients with partial obstruction who drug with efficacy in the prevention of acute show continued clinical improvement when exacerbations of allergen- and exercise-induced re-examined. A rectal tube or colonoscopy is asthma. It has no bronchodilatory activity and not indicated for a small-bowel obstruction, should not be used in the treatment of acute and a “long tube” (Cantor tube) has no addi- asthma. Aminophylline is rarely used in the tional advantage over standard nasogastric acute treatment of asthma, because studies decompression. (Townsend et al., pp. 1334–1340; have shown that it does not add to the efficacy Way and Doherty, pp. 662–667) of inhaled albuterol. Diagnostic studies may need to be done later when the patient’s res- 4. (A) Disadvantages seen commonly with cohort piratory distress has been relieved. (Rudolph studies are that they often use large sample sizes, et al., pp. 1959–1961) they often require lengthy follow-ups, and loss of follow-up can bias results. Cohort studies 2. (A) have the advantage of being able to evaluate multiple outcomes. Choice of control is more of 3. (E) an issue when using a case-control method of study. Ethical and legal issues are more com- Explanations 2 and 3 monly a concern in randomized-controlled trials. (Wallace and Doebbeling, pp. 18–19) This patient presents with symptoms and signs of a small-bowel obstruction, most probably 5. (B) Lung abscesses are localized cavities with from intra-abdominal adhesions related to pre- pus. They are usually a result of aspiration of vious abdominal surgery. The diagnosis is infected material from the upper airway and strongly suspected on the basis of the clinical often associated with periodontal disease or presentation. Appropriate initial management poor oral hygiene. The most common pathogens should, therefore, include nasogastric decom- are oral anaerobes. Although Bacteroides fragilis pression to prevent further distention from is isolated in up to 10% of cases, S. aureus may swallowed air, and initiation of IV fluid resus- be found as well. Most anaerobic pneumonia citation before obtaining abdominal radi- still respond to penicillin, but in life-threatening ographs. Analgesics may mask physical signs situations resistance should be presumed and of impending bowel ischemia. A barium enema the addition of quinolones, monobactams, and may be helpful in patients with suspected aminoglycosides is advised. (Kasper, pp. 942–945, colonic obstruction, but should not be obtained 1536–1538) before resuscitation and plain radiographs. This patient does not have a history of consti- 6. (D) Relative risk is calculated by dividing the pation, and, therefore, enemas and stool sof- incidence in exposed persons by the incidence in teners are not indicated. This patient has unexposed persons. Total incidence - incidence in clinical and radiographic signs of a complete unexposed is the calculation used for determin- small-bowel obstruction. Because of the risk of ing population attributable to risk. Incidence in intestinal ischemia, surgery is indicated after exposed - incidence in unexposed persons is the initial resuscitative maneuvers to correct fluid calculation for attributable risk and (incidence in

402 (c) ketabton.com: The Digital Library

Answers: 1–12 403

exposed–incidence in unexposed)/incidence in Immunization, Recommendations of the Advisory exposed is used to identify attributable risk per- Committee on Immunization Practices. Available at: cent. (Lang and Hensrud, pp. 16–17) http://www.cdc.gov/mmwr/PDF/rr/rr5515.pdf; Wallace and Doebbeling, p. 124) 7. (D) 10. (C) Wounds resulting from animal bites may 8. (D) become infected with the typical skin and soft-tissue infectious agents, S. aureus and Explanations 7 and 8 Streptococcus pyogenes. Occasionally, they are infected with normal flora of the animal’s The ECG demonstrates T-wave inversions in mouth. Cat bite wounds may become infected II, III, and AVF, inferior wall leads. About one- with P. multocida. These infections can be quite third of patients with inferoposterior wall left severe and may require surgical drainage ventricular infarction have some degree of because the long slender teeth of cats enable right ventricular necrosis, and an occasional deep inoculation of the organism. Dog bite patient has extensive right ventricular wounds may become infected with E. corrodens. myocardial infarction (MI). The predominant Rat bite wounds may result in rat bite fever, of clinical feature of right ventricular MI is severe which S. minus is one causative agent. R. hense- right ventricular failure with jugular venous lae causes cat scratch disease. F. tularensis is the distention but no pulmonary vascular con- etiologic agent for tularemia. (McMillan et al., gestion, Kussmaul’s sign (increased jugular pp. 1208–1209) venous distention with inspiration), and often hypotension. The mainstay of treatment is 11. (A) Folic acid in doses of 0.4–1 mg daily reduce volume expansion. Pressor agents, preload the risk of a NTD, such as spina bifida, from and afterload reducing drugs, and intra-aortic approximately 1 in 1000 to 1 in 2000 when balloon counterpulsation may also be begun 1–3 months before conception and con- required. All patients with suspected acute tinued through the first 3 months of pregnancy. MI should have access to primary percuta- Over-the-counter multivitamins typically con- neous coronary intervention (primary PCI). tain 0.4 mg folic acid, whereas prescription pre- Primary PCI has been shown to be more effec- natal vitamins typically contain 1 mg. Folic acid tive than fibrinolysis and is associated with prevents or corrects a macrocytic, megaloblas- better short-term and long-term outcomes. tic anemia. Women who previously delivered Not all hospitals have the personnel and facil- an infant with a NTD should begin 4 mg/day ities necessary to perform primary PCI at all folic acid at least 1 month before conception times. (Kasper, pp. 1414–1417, 1448–1462) and continue this dose until 12 weeks’ gestation to reduce the risk of a recurrent NTD. The dose 9. (B) Disease from wild poliovirus was elimi- may then be reduced to 1 mg daily because nated from the United States. However, closure of the embryonic neural tube is com- approximately eight cases of paralytic polio plete by this time. The primary reason to pre- associated with oral vaccine were occurring scribe a prenatal vitamin is to provide 60–65 mg every year. Since this complication does not elemental iron daily, rather than the 25–30 mg arise with injectable inactivated poliovirus vac- in most over-the-counter multivitamins. (Creasy cine (IPV), the recommendation was made to and Resnick, p. 978) change the schedule to four doses of IPV. (When polio was still endemic, oral vaccine 12. (E) ELISA detectable antibodies to HIV gen- had been preferable, since oral vaccines were erally appear within 4–12 weeks after infection more efficient at preventing intestinal infec- with HIV. They are detectable within 6 months tion without disease, and therefore transmis- in more than 95% of cases. Note that since sion of polio that can occur in vaccinated 2002, rapid HIV tests are available, which persons.) (CDC, General Recommendations on require blood or an oral swab. Positive results (c) ketabton.com: The Digital Library

404 14: Practice Test 8

require confirmatory testing with Western of age, and the incidence declines after the Blot of immunofluorescent assay. (CDC Revised second birthday. Occult bacteremia occurs Recommendations for HIV Testing for Adults, with essentially the same frequency in all Adolescents, and Pregnant Women in Health-Care socioeconomic groups. The most common Settings, September 2006. Available at: http:// organism found is S. pneumoniae followed by www.cdc.gov/mmwr/preview/mmwrhtml/ Salmonella and N. meningitidis. The treatment of rr5514a1.htm) children with occult bacteremia is dependent on laboratory findings and clinical status. 13. (C) False labor or Braxton Hicks contractions (Kliegman, pp. 1063–1066) occur at irregular intervals and usually last 30 seconds or less. Although they may cause 16. (E) External otitis is an infection of the external slight cervical dilation and effacement, only auditory canal. The primary symptom is ear true labor will cause the amount of cervical pain, which is worsened when the pinna is dilation and effacement present in this woman. moved. This distinguishes it from otitis media. The first stage of labor is defined as the inter- A foreign body is easily seen in the canal, unless val from onset of regular uterine contractions there is secondary infection and drainage. sufficient to cause cervical dilation to complete Mastoiditis is a complication of otitis media. cervical dilation, 10 cm. The first stage is sub- Patients have fever, and as the process pro- divided into a latent phase and active phase. gresses, displacement of the pinna. Contact The latent phase is characterized by slow cer- dermatitis of the ear is usually due to an ear- vical dilation of 1 cm/h or less. When the rate ring and would occur on the pinna. (Rudolph of cervical dilation increases to 1.5 cm/h or et al., pp. 1255–1256) greater, the woman is in the active phase of the first stage of labor. The second stage of labor is 17. (D) Chronic exposure to benzene, which is the interval from complete cervical dilation to used in many industrial processes, is a very delivery of the infant. The third stage is the well-documented cause of bone marrow interval from delivery of the infant to delivery depression. Benzene-induced aplastic anemia of the placenta. (Cunningham et al., pp. 152–157) can be fatal. Chronic exposure also is known to cause an increased risk of leukemia. Acute 14. (D) Station of the presenting part is divided exposure to high concentrations will cause cen- between −3 and +3. At −5, the presenting part is tral nervous system (CNS) depression. (LaDou, above the pelvic inlet. At 0 station, the presenting p. 224; Wallace and Doebbeling, p. 514) part is at the level of the ischial spines. Stations − 4 to −1 are between the pelvic inlet and ischial 18. (C) Tertiary prevention activities involve the care spines. At +5 station, the presenting part is visi- of established disease, with attempts made to ble at the perineum, eventually even between restore to highest function, minimize the neg- contractions. Stations +1 to +4 are between the ative effects of disease, and prevent disease- ischial spines and perineum. Descent of the pre- related complications. In this scenario, the senting part progresses from −5 to +5 stations. patient has established disease or diabetes, and (Cunningham et al., pp. 152–157, 426–427) referral to the ophthalmologist’s office is an attempt to minimize the negative effects of his 15. (D) Occult bacteremia refers to the finding of a diabetes-induced retinopathy. Primary pre- positive blood culture in a child who appears vention is used to prevent disease before it has well enough to be treated as an outpatient. The occurred, that is, immunization programs. response to antipyretics or lack of response does Secondary prevention aims to detect presymp- not change the risk for bacteremia. The risk of tomatic disease at an early stage in order to occult bacteremia correlates statistically with prevent or minimize morbidity. Quaternary an elevated WBC (15,000) and fever (39°C and predisease prevention are not terms used [102.2°F]). Occult pneumococcal bacteremia is to refer to the three main types of prevention. most common between 6 months and 2 years (Lang and Hensrud, pp. 3–4) (c) ketabton.com: The Digital Library

Answers: 13–25 405

19. (C) Pap testing should be done for all women MAO A and B. When a patient on an MAOI who have been sexually active and who have a ingests foods containing dietary amines (tyra- cervix. It should be started when sexual activ- mine), because MAO in their digestive tract is ity is initiated, be repeated at least every blocked, the amines are absorbed and can lead 3 years, and probably can be stopped at age 65 to an adrenergic crisis. Patients on the irre- if all previous Pap tests have been normal. If a versible MAOIs need to watch their diet care- woman has had a hysterectomy for invasive fully. Aged cheeses and fermented or dried cancer of the cervix or has a history of diethyl- meats are rich in tyramine. Individuals on stilbestrol (DES) exposure, Pap tests (cell cytol- MAOIs also need to avoid sympathomimetic ogy) should be continued despite absence of agents, serotonergic medications, and meperi- the cervix (evidence for this practice is not dine. (Stern and Herman, p. 345) strong, but it is the current recommendation). Cancer of the cervix seems to occur almost 23. (H) Trazodone, an antidepressant drug, is used exclusively or exclusively in association with not only for depression, but also for sleep active infection with a pathogenic strain of because of its sedating properties. It is associ- HPV. (USPSTF) ated with the rare occurrence of priapism, a prolonged erection in the absence of sexual 20. (B) Silicosis is often seen in quarry workers stimulation. If not treated early, the condition working with compounds with high free crys- may develop into an emergent situation. (Kaplan talline silica such as quartz, sandstone, granite, and Sadock, pp. 1123–1125) and slate. Involvement of the lungs by silicosis increases susceptibility to tuberculosis infec- 24. (C) Haloperidol is a butyrophenone antipsy- tion. There is no added risk of tuberculosis after chotic drug with a high degree of extrapyra- exposure to asbestos or other nonsilica dust. midal effects. The oculogyric syndrome is an Coal worker’s pneumoconiosis, as the name example of an extrapyramidal effect called a implies, is more commonly seen in coal work- dystonia. Thioridazine, a piperidine phenoth- ers. Asbestosis is more commonly seen in those iazine, has some potential to cause extrapyra- who have a history of exposure to asbestos- midal symptoms, but this is significantly less containing compounds or materials, or the than that seen with haloperidol. (Kaplan and mining or milling of asbestos. Siderosis refers Sadock, p. 497, 1053) to the lung disease caused by inhalation of iron dust. Siderosis is not considered to increase 25. (E) Lithium carbonate is a naturally occurring susceptibility to tuberculosis. (LaDou, p. 337; salt used as a mood stabilizer in the treatment Wallace and Doebbeling, p. 487) of bipolar disorders. Because of the potential for toxicity, periodic monitoring of blood level 21. (D) Fox rabies is still a serious problem in is required. In addition, monitoring is required North America. Furthermore, foxes act as a to confirm a blood level within the therapeu- common reservoir of rabies in parts of Texas. tic range, usually 0.6–1.2 meq/L. As serum The appropriate management of this patient levels rise above 1.5 meq/L, symptoms of would include postexposure prophylaxis with toxicity emerge in a somewhat predictable both rabies vaccine and immune globulin. The manner from early signs such as nausea, vom- patient’s tetanus immunization status should iting, and diarrhea to more severe signs of also be reviewed and postexposure prophy- coarse tremor, ataxia, lethargy, and coma. laxis administered if the immunization status Sensitivity to the significance of observable is incomplete or unknown. (Wallace and Doebbeling, clinical signs provides a means for the clinician pp. 349–352) to identify toxicity, even though a blood level is not immediately available. (Kaplan and Sadock, 22. (A) Phenelzine is a monoamine oxidase pp. 1067–1074) inhibitor which relatively irreversibly blocks (c) ketabton.com: The Digital Library

406 14: Practice Test 8

26. (G) Venlafaxine has been associated with a sus- prevalent adolescent problems, such as abuse, tained increase in diastolic blood pressure in up pregnancy, and drug abuse, can also result in to 10% of individuals, especially at higher withdrawal from the family, some change in doses. (Stern and Herman, p. 344) their functioning at school would be expected. If she were hypothyroid, you would expect to 27. (E) Spontaneous pneumothorax can be seen in see some changes in her physical examination. otherwise healthy adolescents. It presents very (Rudolph et al., pp. 224–226) suddenly with chest pain and dyspnea. Acute bronchitis does not present with these acute 31. (D) Linear regression with calculation of the findings. Asthma would present with wheez- regression coefficient is commonly used to ing or decreased breath sounds bilaterally. assess whether there is a relationship between Pneumonia and empyema would not present two variables and to assess the magnitude of over such a short time. He would also be that trend. The regression coefficient is the febrile. A foreign body aspiration is unlikely slope of the line. If the line is essentially hori- in this age group. (Kliegman et al., p. 65) zontal, the slope is close to zero, and the vari- ables have little relationship to each other. It 28. (E) Patients with sickle cell disease are at risk must be remembered that a high level of cor- for overwhelming infection, especially with relation indicates strong association, but does encapsulated organisms. This is especially true not necessarily mean that there is a causal rela- in infants and young children. The use of pro- tionship between the two variables. The odds phylactic penicillin has dramatically decreased ratio is used to contrast two proportions. The this risk. It is started as soon as the diagnosis is Wilcoxon Two-Group Rank Sum test, Mann- made. He will need a pneumococcal vaccine, Whitney U-test, and Wilcoxon Signed-Ranks but immunization is started at 2 months of age. test are nonparametric rank tests that use ranks Neonates with sickle cell disease are not at any of observed data rather than original values. greater risk for iron deficiency than normal (Feinstein, p. 372, 381) infants. These patients are at higher risk for folate deficiency. However, controlled studies 32. (D) The patient has the typical CSF findings of have not documented the need for routine folic bacterial meningitis, with elevated WBCs and a acid supplementation, so some centers have predominance of neutrophils, elevated protein, elected to stop routine supplementation. The and decreased glucose. These values would not only caution in avoiding heat for these patients be expected in patients with tuberculosis or is that dehydration can exacerbate sickling. enteroviral meningitis. Typical bacteria that When exposed to heat, patients need to be dili- cause meningitis in this age group are S. pneu- gent about adequate fluid intake. (Rudolph et al., moniae and N. meningitidis. GBS is the most pp. 1531–1534, 2233–2234) common bacterial cause of meningitis in neonates. Group A Streptococcus does not com- 29. (B) This is the classic presentation of splenic monly cause meningitis. H. influenzae meningi- sequestration, which occurs in these patients tis is rarely seen secondary to prevention with in the first few years of life, before the spleen immunization. (Kliegman et al., pp. 955–965) autoinfarcts. Acute chest syndrome, intussus- ception, and vasoocclusive crisis are not asso- 33. (B) Diaper dermatitis is a very common prob- ciated with severe anemia. If this were an lem in infants. This infant’s rash is due to bul- aplastic crisis, a very low reticulocyte count lous impetigo. The classic presentation of this would be expected. (Rudolph et al., pp. 1532–1532) illness is erythema of the skin, followed by the development of bullae that rupture. Allergic 30. (E) One of the major developmental tasks of dermatitis and irritant dermatitis are most adolescence is the acquisition of independence prominent on the convex areas and are intensely from the parents. Spending time in their own red. Candida dermatitis is red without bullae personal space is a part of this. Although and has satellite lesions at the margins. In (c) ketabton.com: The Digital Library

Answers: 26–38 407

seborrheic dermatitis, children tend to have 36. (C) The prevalence of severe mental retardation the rash on the scalp, neck, and face also. It is is spread about equally across all social classes. scaly and more prominent in the intertriginous Mild mental retardation is seen more predom- areas. (Hurwitz, pp. 14, 34–38) inantly in children of lower social classes. Severe mental retardation has also been found 34. (C) Erythema migrans is the exanthem seen in to have a male excess with a male to female Lyme disease. It starts as a red macule or ratio of 1.1 to 1.4. About 30% of severe mental papule and then expands to an annular plaque retardation is attributed to trisomies, and thus, with a raised border. It is seen 3–5 weeks after although advanced maternal age is a strong the tick bite at the site of the bite. Patients often predictor of trisomy, the correlation is much have systemic symptoms at the same time. less than 80%. The life expectancy of persons Erythema marginatum is the exanthem in acute with severe mental retardation is considerably rheumatic fever. It is also an annular lesion that shorter than that of the general population. usually occurs on the trunk. This patient does (Wallace and Doebbeling, pp. 1049–1051) not have the other symptoms consistent with rheumatic fever. Contact dermatitis is not 37. (B) The main characteristic of the ordinal scale annular but intensely erythematous and is that the categories have a logical or ordered shaped according to the offending article, such relationship to each other. This type of scale as in a necklace shape if the patient is allergic permits the measurement of degrees of differ- to a metal necklace. Erythema multiforme clas- ence, but not the specific amount of difference. sically has target lesions. Tinea corporis is also The intervals between dimensional scales have annular, but has central clearing and is typi- distinct measurable differences between them cally not as large as erythema migrans. that look at specific quantifiable amounts of (American Academy of Pediatrics, pp. 428–433; Hurwitz, difference, that is, 1, 2, 3, 4 hours. A binary scale pp. 306–310) has only two categories, that is, male, female, yes, no, and so forth. In a nominal scale, the cat- 35. (E) Although testing of an animal specimen egories have no rankings or relationship to can give a diagnosis in less than 24 hours, each other, that is, blue, black, brown, cate- immediate treatment is indicated because the gories for eye color. Categorical is similar to fox’s behavior is clearly abnormal and highly nominal. (Feinstein, pp. 10–13) suggestive of rabies. It is appropriate after such an attack to begin preventive treatment imme- 38. (C) The median is useful when looking at lon- diately for presumptive rabies exposure. The gitudinal data when the data set are still incom- Immunization Practices Advisory Committee plete. The median survival time is the length of (ACIP) (last updated in 1999) confirmed that survival that is met or exceeded by 50% of the recommended postexposure prophylaxis is a study population. The median does not incor- five-shot series of immunizations given over porate the values at the extremes and there- 28 days. A dose of rabies-specific immunoglob- fore will not be affected by the remaining ulin, calculated by weight, is given with the values that are still not recorded. The mean first immunization. One-half of the dose is takes into account all values and therefore administered in the area of the bite(s), the cannot be calculated at this time. There is not remainder in the gluteus. In the United States, enough information provided to determine no cases of rabies have occurred after prompt whether a mode can be calculated on this and complete postexposure prophylaxis. example. If there are more than three study Observation of wildlife for rabies is never indi- participants with a particular survival time, the cated because the period in which rabies lengths of the remaining study participants will may be communicable is unknown. (CDC be irrelevant, but if the maximum number of web site: http://www.cdc.gov/nip/publications/acip- participants with a particular survival time is list. htm#Comprehensive; Wallace and Doebbeling, less than 3, we will have to wait and find out pp. 351–352) what the remaining survival times are. The (c) ketabton.com: The Digital Library

408 14: Practice Test 8

midrange is calculated by dividing the sum of in preventing the sexual transmission of HIV the minimum and maximum items by two. The infection. The strongest evidence for this comes maximum value is still unknown, therefore the from studies of couples in which one member midrange cannot be calculated yet. Midmode is is known to be infected with HIV and the other not a statistical measure of central tendency. is known to be initially uninfected. In a 2-year (Feinstein, p. 31) study, 124 couples of this sort reported consis- tent condom use during about 15,000 episodes 39. (D) of intercourse. None of the seronegative part- ners became infected with HIV. In contrast, in 40. (E) couples who used condoms inconsistently, 10% of the initially uninfected partners acquired Explanations 39 and 40 HIV infection over the same time period. That rate is similar to the rate of HIV transmission in Treatment must be prompt, because death can nonusers of condoms. Condoms made from occur within minutes with a severe anaphy- natural membrane (lamb cecum) are more lactic reaction. Administration of subcutaneous porous to viral particles than are latex con- or IM epinephrine (0.3–0.5 mL of 1:1000 epi- doms. Condoms treated with nonoxynol-9, nephrine) should be given as soon as possible contrary to expectations, have not been demon- to control symptoms. Repeated doses every strated to decrease likelihood of HIV trans- 20 minutes may be required for severe reac- mission. Exposure of latex to petroleum products tions. Beta-blockers are contraindicated. Topical causes it to deteriorate and to tear more easily. steroids are of no benefit, and IV steroids are (Wallace and Doebbeling, pp. 1196–1197) not effective for the acute event, but may help reduce the likelihood of later recurrence of 42. (E) Although prior administration of human bronchospasm. Diphenhydramine (50–100 mg immunoglobulin containing anti-HAV before IV/IM) is appropriate for urticaria-angioedema or within 2 weeks after exposure is 75–85% and bronchospasm, but is second-line therapy effective, prior immunization against hepatitis after epinephrine. Type 1 immediate hyper- A is even more effective. A two-dose vaccina- sensitivity reactions are characterized by early tion schedule is 95% effective in preventing symptoms such as urticaria, laryngeal edema, disease in adolescents and adults. Immunity bronchospasm, and vasomotor collapse. They is believed to last at least 10 years, and perhaps result from mast cell or basophil binding by 20 or 30 years. Consuming untreated water and IgE (not IgA) or activated complement frag- uncooked food in endemic areas is a risk factor; ments C3a or C5a (so-called anaphylatoxins). A therefore, relying on bottled water and cooked persistent delayed rash is not seen with type food is reasonable. However, this will not pro- 1 reactions. The early cutaneous hypersensi- tect against contact with younger children who tivity response of wheal and flare is seen within often have subclinical infections and remain 15 minutes and resolves in about 90 minutes. significant sources of disease transmission. Foods, bee stings, drugs, and pollens are exam- Pepto-Bismol and ciprofloxacin are preventive ples of inciting agents. Vascular collapse and treatments for travelers’ diarrhea. (Wallace and hypotension due to loss of intravascular Doebbeling, pp. 76–77, 175–178) volume can lead to death, therefore fluid resus- citation is often needed. (Kasper, pp. 1947–1950) 43. (C) Diaper dermatitis is a very common prob- lem in infants. This infant’s rash is due to 41. (E) The most effective strategies for preventing Candida. Candida dermatitis is red without heterosexual transmission of HIV are absti- bullae and has satellite lesions at the margins. nence or having sex with an uninfected partner It is common in infants, especially when they in a mutually monogamous relationship. have been on antibiotics. In bullous impetigo, Studies have also shown that consistent and the skin is initially erythematous and then correct use of latex condoms is highly effective bullae develop. Allergic dermatitis and irritant (c) ketabton.com: The Digital Library

Answers: 39–46 409

dermatitis are most prominent on the convex of the infection to the infant is much more areas and are intensely red. In seborrheic der- likely if the infection is primary. The charac- matitis, children tend to have the rash on the teristic lesions are grouped vesicles on an ery- scalp, neck, and face also. It is scaly and more thematous base. They are likely to occur at a prominent in the intertriginous areas. (Hurwitz, traumatized location such as the scalp if a mon- pp. 27–29) itor has been used during delivery. Varicella also is vesicular, but one would expect to see 44. (C) The hematopoietic form is most commonly numerous lesions, which usually start on the seen at radiation levels of 2–10 Sieverts. The trunk. GBS and Staphylococcus can cause cel- main phase of the illness is seen about 3–6 lulitis in the newborn, but the rash would not weeks after exposure and can consist of be vesicular. Exanthems are common with leukopenia and thrombocytopenia. If bone enterovirus but are macular and are present marrow suppression is significant, then result- all over the body. (American Academy of Pediatrics, ing death from infection and/or hemorrhage pp. 361–370) may occur. The cerebral form of acute radiation syndrome occurs at exposures of greater than 46. (A) Erythema infectiosum (fifth disease) is 50 Sieverts. This form usually presents with caused by parvovirus. This patient has the nausea, vomiting, diarrhea, and headache. typical presentation with a prodromal illness, Coma, convulsions, and death may also occur followed by the characteristic “slapped- on the first day. The gastrointestinal form is cheek” rash on the face and the reticular rash seen at exposure rates of 10–20 Sieverts and on the extremities. Kawasaki disease is diag- can have effects that present up to 2 weeks after nosed clinically. Patients have fever, conjunc- exposure. The pulmonary form occurs at expo- tivitis, adenopathy, oral changes, and rash and sures of greater than 6 Sieverts of irradiation to are usually ill. Measles presents with the clas- the lung tissue. Effects are most commonly seen sic triad of the three C’s—conjunctivitis, about 30–90 days after exposure. If significantly coryza, and cough—followed by the rash. extensive, it can result in respiratory failure, Roseola usually occurs in infants. Patients pulmonary fibrosis, or cor pulmonale. Renal have several days of high fever, followed by a forms are not commonly recognized as one of rash on the trunk, which is macular or macu- the main forms of acute radiation syndrome. lopapular. The rash in scarlet fever is a fine (Wallace and Doebbeling, pp. 621–624) papular rash (sandpaper-like). (Rudolph et al., pp. 1058–1059, 1221–1224) 45. (C) Neonates are exposed to herpes at birth if the mother has a genital infection. Transmission (c) ketabton.com: The Digital Library

BIBLIOGRAPHY

American Academy of Pediatrics. Committee on LaDou J. Current Occupational & Environmental Injury and Poison Prevention. Selection and using Medicine, 3rd ed. New York, NY: McGraw-Hill, the most appropriate car safety seats for growing 2003. children: guidelines for counseling parents. Lang RS, Hensrud DD. Clinical Preventive Medicine, Pediatrics 1996 2nd ed. Chicago, IL: AMA Press, 2004. Creasy RK, Resnik R, Iams JD. Maternal–Fetal Medicine: McMillan JA, DeAngelis CD, Feigin RD, et al. Oski’s Principles and Practice, 5th ed. Philadelphia, PA: Pediatrics: Principles and Practice, 4th ed. Philadelphia, W.B. Saunders, 2004. PA: JB Lippincott, 2006. Cunningham FG, Leveno KJ, Bloom SL, et al. Williams Rudolph CD, Rudolph AM. Pediatrics, 21st ed. Obstetrics, 22nd ed. New York, NY: McGraw Hill, New York, NY: McGraw-Hill, 2003. 2005. Stern TA, Herman JB. Massachusetts General Hospital Feinstein AR. Principles of Medical Statistics. Washington, Psychiatry Update and Board Preparation, 2nd ed. DC: Chapman & Hall, 2002. New York, NY: McGraw-Hill, 2000. Hurwitz S. Clinical Pediatric Dermatology: A Textbook of Townsend CM Jr, Beauchamp RD, Evers BM, et al., Skin Disorders of Childhood and Adolescence, 2nd ed. eds. Sabiston Textbook of Surgery: The Biologic Basis of Philadelphia, PA: W.B. Saunders, 1993. Modern Surgical Practice, 17th ed. Philadelphia, PA: Kaplan HI, Sadock BJ. Synopsis of Psychiatry: Behavioral W.B. Saunders, 2004. Sciences/Clinical Psychiatry, 9th ed. Baltimore, MD: USPSTF, The Guide to Clinical Preventive Services, 2006. Williams & Wilkins, 2003. Wallace RB, Doebbeling BN, eds. Maxcy-Rosenau-Last Kasper DL, Braunwald E, Fauci A, et al. Harrison’s Textbook of Public Health & Preventive Medicine, Principles of Internal Medicine, 16th ed. New York, 14th ed. Stamford, CT: Appleton & Lange, 1998. NY: McGraw-Hill, 2005. Way LW, Doherty GM, eds. Current Surgical Diagnosis Kliegman RM, Greenbaum L, Lye P. Practical Strategies and Treatment, 124th ed. New York, NY: McGraw- in Pediatric Diagnosis and Therapy. Philadelphia, PA: Hill, 2006. Elsevier, 2004.

410 (c) ketabton.com: The Digital Library

Subject List: Practice Test 8

Question Number and Subject 23. Psychiatry 24. Psychiatry 1. Pediatrics 25. Psychiatry 2. Surgery 26. Psychiatry 3. Surgery 27. Pediatrics 4. Preventive Medicine 28. Pediatrics 5. Internal Medicine 29. Pediatrics 6. Preventive Medicine 30. Pediatrics 7. Internal Medicine 31. Preventive Medicine 8. Internal Medicine 32. Pediatrics 9. Preventive Medicine 33. Pediatrics 10. Pediatrics 34. Pediatrics 11. Obstetrics-gynecology 35. Preventive Medicine 12. Internal Medicine 36. Preventive Medicine 13. Obstetrics-gynecology 37. Preventive Medicine 14. Obstetrics-gynecology 38. Preventive Medicine 15. Pediatrics 39. Internal Medicine 16. Pediatrics 40. Internal Medicine 17. Preventive Medicine 41. Preventive Medicine 18. Preventive Medicine 42. Preventive Medicine 19. Preventive Medicine 43. Pediatrics 20. Preventive Medicine 44. Preventive Medicine 21. Preventive Medicine 45. Pediatrics 22. Psychiatry 46. Pediatrics

411 (c) ketabton.com: The Digital Library

This page intentionally left blank (c) ketabton.com: The Digital Library

Index

A Acyclovir, 48, 66, 87, 107 Alzheimer’s disease Abdominal aneurysm, ruptured, 216, 242 Addison’s disease, 15, 34 dementia, 168, 183, 280, 289 Abdominal aortic aneurysm (AAA), Adenocarcinoma of the endometrium, screening for, 316, 325 226–227, 250, 314, 322 48, 66 symptoms, 17, 36 Abdominal pain Adenocarcinoma of the pancreas, 8, 29 Amaurosis fugax (See Transient in early adolescent girl, 61, 81 ADHD (See attention deficit monocular blindness) during labor, 60, 81 hyperactivity disorder) Amenorrhea and ureteral calculus, 201–202, 231 Adolescents HGC test for, 257, 266 various diagnoses, 205, 235 abdominal pain in girl, 61, 81 and Müllerian agenesis, 314, 321–322 Abdominal ultrasound, 314, 322 behavior changes, 397, 406 primary, 101, 121 ABG (See Arterial blood gas) breast enlargement, in males, 89, secondary, 51, 70 Abortion/breast cancer study, 142, 109–110 and serum DHEAS concentration, 159–160 head trauma, 254, 262 50, 68–69 Absence seizures, 259, 268 pityriasis rosea in, 87, 107 and serum FHS level, 51, 70 Absolute risk reduction, 316, 325 SLE in, 92, 111 various diagnoses, 356, 364–365 Acamprosate, 178, 193 testicular torsion, 93, 113 Amitriptyline, 178, 192 Accelerated angina, 356–357, 365 African Americans Amlodipine, 15, 35 ACE inhibitors (See Angiotensin- infant mortality rates, 145, 162 Amniocentesis, 56–57, 76 converting enzyme inhibitors) iron-deficiency anemia, 94, 114–115 Amoxicillin Acne, 316, 324 rickets, 93, 113 for chlamydia, 256, 264 Acquired immunodeficiency syndrome tinea capitis in, 92, 111–112 for otitis media, 94, 104, 114, 124 (AIDS) (See AIDS) Age as prophylactic antibiotic for dental Acquired vitiligo, 8, 29 and breast cancer risk, 208, 236–237 procedures, 2, 25 Acrodermatitis enteropathida, 358, 366 and crude death rate, 137, 152–153 Amoxicillin-clavulanic acid, 104, 124 Acromegaly, 15, 34 and delirium, 170, 185 Anabolic steroid abuse, 337, 345 Acting out, 179, 193, 312, 320 and depression, 174, 189 Analgesics Active surveillance system, 145, 162 Age-specific death rates, 137, 152–153 for acute sinusitis, 2, 26 Acute anteriolateral wall myocardial Age-standardized death rates, 144, 161 for terminally ill patients, 258, 266–267 infarction (MI), 296, 303 Agoraphobia, 169, 184 Anaphylactic reaction, 399, 408 Acute anterior iridocyclitis, 6, 29 Agranulocytosis, 172, 179, 187, 194 Anaphylactoid purpura (See Henoch- Acute appendicitis, 205, 235 AIDS (acquired immunodeficiency Schönlein purpura) Acute cholecystitis, 205, 235 syndrome), 11–12, 31–32 (See also Anastomosis, primary, 213, 241 Acute intracranial hemorrhage, 94, 114 Human immunodeficiency virus) Androgen insensitivity syndrome, 61, 81 Acute iritis, 6, 29 Akathisia, 165, 180 Anemia Acute laryngotracheobronchitis, 351, 360 Albuterol, 391, 402 in infants, 298, 304–305 Acute lymphoblastic leukemia (ALL), Alcohol abuse menstrual periods and, 23, 42–43 355, 364 and acute pancreatitis, 205, 235 symptomatic, 226, 249 Acute myocardial infarction, 357, 365 and Mallory—Weiss tear, 218, 243 Angina Acute oliguria, 13, 33 Alcohol dependence, 178, 193 accelerated, 356–357, 365 Acute pancreatitis, 5, 27, 205, 235 Alcohol withdrawal delirium, and cardiovascular disease in women, Acute pericarditis, 338, 345 170–171, 185 130, 146–147 Acute radiation syndrome, 401, 409 Alcohol-induced persisting amnestic Angioedema, 318, 326–327 Acute sickle dactylitis (See Hand-foot disorder, 313, 321 Angiotensin-converting enzyme (ACE) syndrome) Alcoholism, 374, 383 inhibitors Acute sinusitis, 2, 26 Alkalosis, 23, 28, 42 adverse reactions to, 35 Acute splenic sequestration, 103, 123, ALL (See Acute lymphoblastic leukemia) chronic cough from, 314, 321 397, 406 Allergic rhinitis, 95, 115 for diabetes management, 261 Acute suppurative lymphadenitis, Alpha-fetoprotein (AFP), 75 facial swelling and, 318, 326–327 206, 235 Alprazolam, 333, 341 Anhedonia, 175, 189–190

413

Copyright © 2008 by The McGraw-Hill Companies, Inc. Click here for terms of use. (c) ketabton.com: The Digital Library

414 Index

Animal bites (See Bat bite; Cat bite; Asterixis of the hands, 17, 36 Bipolar II disorder, 177, 191 Fox bite) Asthma Birth defects Anion gap metabolic acidosis, 222, 246 in children, 104, 125, 391, 402 coumadin and, 57, 76 Anopheles aegypti, 138, 154 DSM axis system, 297, 304 and drug use, 58, 78 Anopheles sp., 138, 154 renal disease and, 98, 117 phenytoin and, 58, 78 Anorexia nervosa, 4, 27, 168, 182–183, Asymptomatic primary Bladder outlet obstruction, 13, 33 352, 361 hyperparathyroidism, 274, 283 Bleeding esophageal varices, 17, 36 Antegrade amnesia, 357, 365 Atelectasis, 293, 302 Blood ammonia level, 17, 36 Anterior/posterior cul-de-sacs, 54, 73 Atenolol, 336, 344 Blood bank, HIV testing, 294, 302–303 Anthrax, 138, 153, 373, 382 Atrial septal defect (ASD), 21, 40, 276, 284 Blood dyscrasia side effects, 179, 194 Anthropoid pelvis, 57, 76–77 Atropine, 300, 306–307 Blood sugars, 53, 72 Antibiotic-associated colitis, 279, 287–288 Attention deficit disorder, 175, 190 Blood transfusion, 134, 149–150 Antibiotics (See also IV antibiotics) Attention deficit hyperactivity disorder Bloody diarrhea, 97, 116 for periductal mastitis with abscess, (ADHD), 176, 178, 191, 193 Bloody stools, 94, 115 316, 324 Atypical antipsychotic medications, Boerhaave’s syndrome, 355, 363 for PPROM, 54, 72 172, 187, 193 Bone marrow depression, 278, 286 before tooth extraction, 2 Atypical mycobacterial lymphadenitis, Borderline personality disorder, 171, for urinary tract infections, 22–23, 42 101, 122 178, 186, 193, 297, 304 Anticholinergic delirium, 166, 181 Atypical squamous cells of Bordetella pertussis, 301, 308 Anticholinergic drugs, 63, 83 undetermined significance (ASCUS) Botulism, in infants, 101, 121 Anticholinergic side effects, 174, 188–189 (See ASCUS) Bovine spongiform encephalitis (BSE), Anticoagulant use during pregnancy, Autoimmune thyroiditis (See 145, 162 57, 76 Hashimoto’s thyroiditis) Bowel obstruction, 352, 361 Anti-D immune globulin, 62, 82–83 Automobile crash victim (See Motor Brachytherapy, 58, 77 Antidepressant medications vehicle crash victim) Bradycardia, 4, 101, 121 effects of, 177–178, 192 Avoidant personality disorder, 178, 193 Brain MRI, for MS diagnosis, 277, 285 mirtazapine, 171, 186 AVR (See Aortic valve replacement) Branchial cleft cyst, 206, 235–236 side effects/characteristics, 179, 193, Axis I/II/III/IV (DSM-IV-TR), 297, 304 BRCA1/2 mutation, 208, 237 253, 261 Azithromycin, 376, 384 Breast, 59, 78, 207–210, 236–238 tricyclic, 189 Breast cancer, 208–210, 236–238 Antidepressant therapy, low-dose, 11, 31 B abortion link study, 142, 159–160 Antigens, 144, 161 Bacillus cereus, 311, 319 age as risk factor, 59, 78–79, 208, Antihistamines, 98, 117 Bacterial endocarditis, 2, 25 236–237 Antihypertensive agents, 15, 31, 35 Bacterial meningitis, 17, 36–37, 397, 406 and BRCA1/2 mutation, 208, 237 Antihypertensive therapy, 10 Bacterial sinusitis, 256, 264 epithelial hyperplasia and, 61, 82 Antipsychotic medications, Bacterial vaginosis, 53, 60, 72, 79–80 fine-needle aspiration for complications /side effects from, Barium enema, 352, 361 management, 56, 62, 75–76, 82 165, 180, 274–275, 283 Barrett’s esophagus, 352–353, 361 hormone replacement therapy as risk Antipyretics, 2, 26 Basal cell carcinoma, 2, 25 factor, 257, 265 Antireflux surgery, 102, 122 Basal skull fracture, 205, 234 image-guided percutaneous biopsy Antisocial personality disorder, 374, 382 Bat bite, 143, 161 for, 59, 78 Anxiety disorder, 178, 193 Bed-wetting, 89, 109 metastatic, 4–5, 27 Anxiolytics, 169, 184 Bee sting, 88, 108–109 modified radical mastectomy for Aortic dissection, 217, 242 Beef, hemolytic-uremic syndrome and, management, 62, 82 Aortic root dilation, 3, 26 142, 160 needle biopsy, 210, 238 Aortic stenosis, 339, 346 Behavioral Risk Surveillance system, psychological complications, 312, 320 Aortic valve disease, 13, 32 145, 162 SLN biopsy, 208–209, 237–238 Aortic valve replacement (AVR), 339, 346 Behavioral therapy, 315, 323 survival predictor, 59, 79 Aortoenteric fistula, 332, 340–341 Benign pneumoconiosis, 256, 264 Breast development, premature, 47, 65 Appendicitis, acute, 205, 235 Benzene, 395, 404 Breast discharge, blood-tinged, Arm (subluxation of the radial head), Benzodiazepines, 333, 341 199–200, 229 101, 121–122, 315, 323–324 Benzoyl peroxide, 316, 324 Breast enlargement, in adolescent male, Arrhythmia, 20, 39 Benztropine, 165, 174, 180, 188 89, 109–110 Arterial blood gas (ABG), 15, 34–35 Beta-hemolytic Streptococcus, 141, 158 Breast imaging, 207, 236 Arterial embolism, 216, 242 Betamethasone, 51, 69 Breast mass, 373–374, 382 Arthritis, rheumatoid, 22, 41 Beta-thalassemia minor, 357, 365–366 Breast pain Asbestos exposure, mesothelioma and, Bethanechol, 188 and fibrocystic disease of the breast, 280, 288 Bias, in testing, 137, 153 61, 82 ASCUS (atypical squamous cells of Bicornate uterus, 52, 71 periductal mastitis with abscess, undetermined significance), 375, 384 Bilateral hypogastric artery ligation, 62, 83 315–316, 324 ASD (See Atrial septal defect) Bilateral parietal lobe activity, 316, 325 tamoxifen for, 49, 67 Asperger disorder, 170, 185 Biparietal diameter, 62, 82 Breast-feeding Aspirin Bipolar disorder, 175, 190 and infant weight gain, 273, 282 avoiding, before tonsillectomy, 98, 117 Bipolar I disorder, 176, 178, 191, 193, and rickets, 93, 113 overdose, 315, 323 279, 288 and vitamin D, 275, 284 (c) ketabton.com: The Digital Library

Index 415

Breast-feeding jaundice, 99, 119, 255, 263 for meningococcal meningitis night terrors, 95, 116 Breath holding spell, 274, 283 outbreak, 131, 148 obstructed airway, 100, 120 Breech presentation, 314, 323 for pneumococcal infection, 373, 381 orbital cellulitis, 93, 113 Broad-spectrum antibiotics Central diabetes insipidus (DI), 6, 28, otitis externa, 394, 404 for diverticular disease, 254, 262 204–205, 234 pneumonia, 96–97, 116 for multifocal hepatic abscess, 372, 381 Cerebral aneurysm, ruptured, 227, post-traumatic epilepsy, 94, 114 Bromocriptine, 172, 186–187 250–251 proteinuria, 299, 306 Bronchitis, chronic, 19 Cerebrovascular system, 131, 147 psychological disorders, 176, 191 Bronchospasm, 399, 408 Cervical adenitis, 380, 387 rickets, 93, 113 Brucellosis, 130, 138, 146, 153, Cervical biopsy, 56, 74 scarlet fever, 90, 111 372–373, 381 Cervical cancer sickle-cell anemia, 98–99, 102–103, BSE (See Bovine spongiform FIGO staging of, 48, 58, 66–67, 77 118, 123 encephalitis) invasive squamous cell carcinoma, streptococcal meningitis, 90, 111 Bubonic plague, 132, 148 48, 66–67 subluxation of the radial head, 101, Buerger’s disease (See Thromboangiitis prognostic factors for 5-year survival, 121–122 obliterans) 49, 67 surrogate guardian, 134, 149–150 Bullous impetigo, 398, 406–407 therapies for, 58, 77 tinea capitis, 92, 111–112 Bupropion, 175, 190 Cervical cerclage, 56, 74–75 tonsillectomy, 98, 117 Bupropion hydrochloride, 133, 149 Cervical lesion, 56, 74 tonsillectomy preparation, 98, 117 Burch procedure, 75 Cervicitis, 64, 84 toxic synovitis, 260, 269 Burn injury, 335, 343 Cesarean section tuberculosis, 87, 107 Burr holes, 376, 384 emergency, 55, 74 vesicoureteral reflux, 102, 122 Buspirone, 178, 193 for placental abruption management, von Willebrand disease, 378–379, 386 Butchers, 138, 153 61, 81 Wilms tumor, 89, 110 and septic pelvic thrombophlebitis, Chi-square test, 134–135, 150 C 278, 286 Chlamydia CAD (See Coronary artery disease) CHD (See Coronary heart disease) and gonorrhea, 376, 384 Calcium (See Hypercalcemia) Chelation, 136, 151 LCR test for, 256, 264 Calcium channel blockers, 35 Chemical weapon poisoning, 354, 363 screening for, 133, 149, 352, 361 Calcium oxalate, 23, 42 Chest pain, in children, 104, 125 Chlamydia trachomatis, 93, 113, 301, 308 Campral (See Acamprosate) Chicken pox Choanal atresia, 98, 118 Cancer death rates among women, in children, 96, 116 Cholecystitis, acute, 205, 235 134, 150 incubation period, 140, 158 Choledocholithiasis, 207, 236 Candida albicans, 312, 320 VZIG for, 87, 107 Cholera, 135, 151, 258, 267 Candida dermatitis, 96, 116, 400, 408–409 Childhood depression, 176, 191 Cholesterol reduction, 334, 342 Car seats, 88, 108 Childhood schizophrenia, 176, 191 Chondrocalcinosis (See Pseudogout) Carbamazepine, 179, 194 Children Chronic bronchitis, 19 Carbon disulfide, 133, 149 ALL, 355, 364 Chronic cough, from ACE inhibitors, Carbon monoxide, 139, 156 allergic rhinitis, 95, 115 314, 321 Carbon monoxide diffusing capacity asthma, 391, 402 Chronic glomerulonephritis, 53, 71–72 (DLCO), 18 atypical mycobacterial lymphadenitis, Chronic lymphocytic thyroiditis (See Carbon monoxide poisoning, 335, 343 101, 122 Hashimoto’s thyroiditis) Carboplatin therapy, 49, 68 bacterial sinusitis, 256, 264 Chronic obstructive pulmonary disease Carcinoma of the colon (See Colon cancer) bed-wetting, 89, 109 (COPD), 19, 37–38, 260, 268 Carcinoma of the head of the pancreas, breath holding spell, 274, 283 Chronic paranoid schizophrenia 8, 29 chicken pox, 96, 116 and anticholinergic delirium, 166, 181 Cardiac catheterization, 357, 365 conduct disorder, 334, 342–343 and antipsychotic medication Cardiac enzyme test, 104, 125 coxsackie virus, 92, 112 complications, 171–172, 186 Cardiac tamponade, 337, 345 dental caries, 92, 111 atypical antipsychotics for, 172, 187 Cardiogenic shock, 20, 39 epilepsy risk, 102, 123 DSM axis system, 297, 304 Cardiovascular disease erythema infectuosum, 92–93, 112, risperidone for, 178, 193 exercise to prevent, 133, 149 401, 409 Chronic relapsing infection presentation in women, 130, 146–147 erythema migrans, 398, 407 brucellosis, 130, 146 Carotid duplex ultrasound, 274, 282 eye injury treatment, 88, 108 Chronic renal failure, 5, 28 Carpal tunnel syndrome, 336–337, 344 foreign body aspiration, 256, 264 Chronic renal insufficiency, 5, 28 Case-control study, 137, 153, 259, 268, HSP, 298, 305 Cigarettes (See Smoking) 358, 366 incubation periods for infections Ciprofloxacin Cat bite, 95, 116, 393, 403 diseases, 140, 158 to control meningococcal meningitis Cataract surgery, 129, 146 inspiratory stridor, 94, 114 outbreak, 131, 148 Catheterized urine specimen, 89, 110 intelligence assessment, 278–279, 287 for inhalational anthrax, 373, 382 Causation, 359, 367 Kawasaki disease, 92, 112 for travelers’ diarrhea, 1, 25 CD4+ T-cell count, 13, 32 lead screening for, 131, 147 Cirrhosis, 206, 236, 280, 288 Ceftriaxone Meckel’s diverticulum, 202–203, Cisplatin, 255, 263–264 for chlamydia, 376, 384 231–232 Citalopram, 178, 192 for gonorrhea, 53, 72, 376, 384 meningitis, 98, 117–118 CJD (See Creutzfeldt-Jakob disease) (c) ketabton.com: The Digital Library

416 Index

Class III contaminated surgery, 293, 302 rectal bleeding and, 378, 386 Cranial computed tomography (CT) Classical typhus fever (See Epidemic staging of, 213, 240 scan, 88, 108 typhus) surgery for, 293, 302 Creatine phosphokinase (CPK), 1, 8, 25, Cleft lip/palate, 58, 78 Colon polyps, 211, 239 172, 186 Clinical breast examination, screening Colon resection, 293, 302 Creutzfeldt-Jakob disease (CJD), 5, 28 schedules, 140, 156–157 Colonic decompression tube, 258, 266 Cricopharyngeal muscle myotomy, Clinical studies Colonic obstruction, 277, 286 218, 243 abortion—breast cancer link study, Colorado tick fever, 138, 154 Cricothyroidotomy, 227, 250 142, 159–160 Colorectal cancer, 3, 26 Crohn’s disease, 203, 232–233 case-control study, 137, 153, 259, 268, Colposcopy, 375, 384 Cross-sectional study, 294, 303 358, 366 Combat veterans, PTSD among, 351–352, Croup, 114, 254, 262 chi-square test, 134–135, 150 360–361 Crude death rate, 137, 144, 152–153, 161 cohort study, 141, 159, 392, 402 Community-acquired pneumonia, 8, 29 Crush injury, 98, 118 cross-sectional study, 294, 303 Compartment syndrome, 335, 343 Cryptomenorrhea, 81 historical cohort study, 354, 362 Compression ultrasonography, 317, 325 Cryptorchidism, 19, 38, 223–224, 247 HIV screening study, 141, 158–159 Computed tomography (CT) scan CT scan (See Computed tomography scan) infant mortality rates, 145, 162 for colon cancer diagnosis, 220, 244 Cushing’s syndrome, 15, 34 lead-time bias, 318, 327 for diverticular disease, 254, 262 Cutaneous anergy to skin test antigens, linear model of regression, 397, 406 for DVT management, 278, 286 12, 32 lung disease screening, 136–137, 152 for nonsmall cell lung cancer Cyanosis and NNT, 316, 325 screening, 215, 241–242 and choanal atresia, 98, 118 ordinal scale, 399, 407 for pyogenic hepatic abscess, 371–372, in newborn, 95, 115 positive predictive value (PPV), 339, 381 and transposition of the great arteries, 346, 355, 363 Conception, 57, 77 276, 284 prevalence, 298, 305 Condoms, HIV transmission and, Cyst of the canal of Nuck, 51, 70 for prostate cancer, 136, 151–152 400, 408 Cystic fibrosis randomized, 298, 305 Conduct disorder, 334, 342–343 and childbearing, 51, 70 and relative risk, 392, 402–403 Condyloma acuminatum, 300–301, 307 odds of offspring having, 339, 346 stomach cancer—sugar substitute link, Confidence interval, 143–144, 160 sweat chloride test for, 311, 319 143–144, 160 Congenital adrenal hyperplasia, Cystoscopy, for cervical cancer staging, Clinical trials, lack of cost/benefit 314, 322 48, 66–67 analysis in, 144, 161 Congenital aganglionic megacolon Cytologic analysis, 200, 229 Clomiphene, 60, 80 (See Hirschsprung disease) Clorazil, 179, 194 Congenital heart disease D Clostridium botulinum, 139, 155 Marfan syndrome, 91, 111 Danazol, 59, 67, 79 Clostridium difficile Turner syndrome and, 91, 111 Dantrolene, 233 antibiotic-associated colitis, 279, various diagnoses, 276, 284–285 Daycare employees, hepatitis A in, 287–288 Congenital syphilis, 274, 282–283 88, 108 diarrhea, 316–317, 325 Congestive heart failure, 337, 344 DCIS (See Ductal carcinoma in situ) Clostridium perfringens, 139, 141, 155, 158 Conjunctivitis, 6, 142, 159 DDH (See Developmental dysplasia Clozapine, 172, 177, 187, 192 Constipation, 99, 102, 119, 122 of the hip) Clue cells, 80 Constitutional delay, 104, 105, 124–125 Death rates (See Mortality rates) Cluster A personality disorder, 169, Contaminated water, 135, 151 Decongestants, 2, 26 184–185 Contemplation stage of behavior, Deep venous thrombosis (DVT), 257, Cluster headaches, 14, 33, 379, 386 355, 363 265, 277, 286, 317, 325 Coagulase-positive S. aureus, 141, 158 Contraception, 52, 71, 301, 307–308 Defense mechanisms, 179, 193 Coal damp, 139, 156 Conversion disorder, 166, 181 Dehydration, in infants, 89, 110 Coal mining, methane exposure and, COPD (See Chronic obstructive Dehydroepiandrosterone sulfate 139, 156 pulmonary disease) (DHEAS), 50, 69 Coarctation of the aorta Corneal abrasion, 88, 108 Delayed puberty, 63, 83–84 and trisomy 21, 273, 282 Coronary artery disease (CAD), 5, 22–23 Delirium NOS (not otherwise specified), and Turner syndrome, 91, 111 Coronary heart disease (CHD) 170, 185 Cocaine use, MI and, 296–297, 303–304 diabetes mellitus and, 16, 36 Delirium Tremens, 313, 321 Cognitive function, impaired, 174, 189 risk equivalents for, 333, 342 Delusions Cognitive psychotherapy, 175, 190 Cortical dementia, 175, 190 and psychosis, 255, 263 Cohort study, 141, 159, 392, 402 Corticosteroids, 277, 285 and psychotic depression, 169, 184 Coliform bacteria Cost/benefit analysis, 144, 161 symptoms, 167, 182 and cholera, 258, 267 Coughing Dementia, 280, 289 in well water, 144, 161 from ACE inhibitors, 314, 321 Dengue fever, 138, 154 Colon cancer to clear obstructed airway, 100, 120 Dental caries, 92, 111, 131, 148 adjuvant therapy for, 213, 240–241 Coumadin, 57, 76 Depression (See also Major depressive cohort study, 141, 159 Countertransference, 312, 320 disorder) and DVT, 277, 286 Cow’s milk, 101, 121 in children, 176, 191 gastrointestinal bleeding and, 332, 341 Coxsackie virus, 92, 112 consultation before diagnosis, management of, 219–220, 244 CPK (See Creatine phosphokinase) 276, 285 (c) ketabton.com: The Digital Library

Index 417

interview questions for evaluation, Dopamine hypothesis, 168, 182–183 Enalapril, 253, 261 173–174, 188 Double uterine fundus, 52, 71 Encephalitis, 333, 342 and parathyroid adenoma, 339, 346 Down syndrome (See Trisomy 21) Encephalopathy, 98, 118 Dermacentor andersoni, 138, 154 Doxycycline, 373, 382 Endocarditis, 2, 25, 353, 361–362 Dermatomyositis, 1, 25 Drinking water, fluoridation of, 131, 148 Endometrial cancer, 47, 48, 52, 65, 66, 71 Detrusor dyssynergia, 63, 83 Drug use (illicit), birth defects and, 58, 78 Endometriosis, 54, 73, 255, 263 Detrusor overactivity, 260, 268 Drug-induced acute interstitial nephritis, Endoscopy, 211, 238–239 Developmental dysplasia of the hip 357, 365 End-stage renal disease, Medicare for, (DDH), 314, 323 DSM-IV-TR (See Diagnostic and 331, 340 Developmental milestones, 298, 304 Statistical Manual of Mental Enlarged uterus, 54, 73 Dextroamphetamine, 178, 193 Disorder, 4th Edition, Text Revision) Enoxaparin, 277, 286 DHEAS (See Dehydroepiandrosterone DTaP (diphtheria and tetanus toxoids Environmental Protection Agency (EPA), sulfate) and acellular pertussis) vaccination, 138, 139, 155 Diabetes 98, 118, 132–133, 149 Enzyme-linked immunosorbent assay and ABG, 15, 35 Duct stones (See Choledocholithiasis) (ELISA), 294, 302–303, 394, 403–404 and duodenal ulcer, 277, 285–286 Ductal carcinoma in situ (DCIS), Eosinophils, 357, 365 enalapril for, 253, 261 209–210, 238 EPA (See Environmental Protection and endometrial cancer, 52, 71 Duloxetine, 178, 192 Agency) and hyperkalemia, 13 Duodenal atresia, 222–223, 246, 280, Epidemic typhus, 138, 154 and hypertension, 10, 31, 32–33 288–289 Epidural hematoma, 375, 383 ideal LDL reading, 253, 261 Duodenal hematoma, 379, 387 Epiglottitis, 94, 114 and toe infections, 274, 282 Duodenal ulcer, 219, 244, 277, 285–286 Epilepsy, 102, 123 Diabetes insipidus, 6, 28, 204–205, 234 Duplex scanning, for DVT, 257, 265, Epinephrine Diabetes mellitus, 16, 36 277, 286 for anaphylactic reaction, 399, 408 DSM axis system, 297, 304 DVT (See Deep venous thrombosis) for bee sting, 88, 108–109 gestational (See Gestational diabetes Dysmenorrhea for croup, 254, 262 mellitus) endometriosis, 54, 73, 255, 263 Epi-pen Jr, 88, 108–109 and nephrotic syndrome, 22, 41 ibuprofen for, 339, 346–347 Epithelial hyperplasia, 61, 82 and pregnancy, 53, 72 secondary, 255, 263 Epithelial ovarian cancer, 49, 68 and urinary incontinence, 56, 75 Dysphagia, 218, 243–244 Epoxy compounds, 139, 156 Diagnostic and Statistical Manual of Dyspnea, 18, 104, 125 EPS (See Extrapyramidal side effects) Mental Disorder, 4th Edition, Text Dysthymic disorder, 177, 191 Epstein-Barr virus (EBV) titer test, Revision (DSM-IV-TR), 297, 304 101, 121 Diaper rash, 96, 116 E Epstein-Barr virus-related Diaphragm, 52, 71 Ear pain mononucleosis, 333, 342 Diarrhea otitis externa, 99, 119 Erosive gastritis, 226, 249 Clostridium difficile, 316–317, 325 Streptococcus pneumoniae infection, Erythema infectuosum, 92–93, 112, E. coli, 25, 141, 158 103, 123–124 401, 409 Giardia lamblia, 141, 158 Ear plugs, 299, 306 Erythema migrans, 398, 407 Legionella, 8, 30 Early onset neonatal sepsis, 299, 306 Erythromycin Diastolic murmur, 9, 30 EBV (See Epstein-Barr virus) for chlamydia, 256, 264 Diffuse malignant mesothelioma, 3, 26 ECG (Electrocardiogram), 12–13 for whooping cough, 100, 120–121 Digoxin, 337, 344 Echocardiogram Escharotomy, 335, 343 Diphenhydramine, 88, 108 for acute pericarditis, 338, 345 Escherichia coli Diphtheria, 140, 158 for Marfan syndrome, 3, 26 infection in infants, 89–90, 110 Diphtheria vaccination (See DTaP Ecological study, 131, 148 and nursery epidemic diarrhea, vaccination) ECT (See Electroconvulsive therapy) 141, 158 Disaster response, management of, Ectopic pregnancy (See Tubal ectopic and pyelonephritis, 300, 307 132, 149 pregnancy) and travelers’ diarrhea, 25, 374, 382 Displacement, 173, 188 EDD (See Expected delivery date) and urinary tract infections, 23, 42 Dissociative amnesia, 167, 182 Effect modifier, 145, 162 Escherichia coli 0157:H7, 97, 116, 142, 160 Dissociative fugue, 171, 186 Effusion fluid, aspiration of, 3, 26 Esophageal carcinoma, 218, 243–244 Diuretic therapy, for ISH, 331, 340 Ejaculate volume, 60, 80 Esophageal perforation, 355, 363 Divalproex, 177, 192 Elderly Esophageal varices, 17, 36, 332, 341 Diverticular disease, 254, 262 cataract surgery, 129, 146 Esophagectomy, 353, 361 Diverticulitis, 212, 239 fractures, 130, 147 Esophagogram, 355, 363 Diverticulosis of the colon, 8, 30, health funding for, 130, 147 Esophagoscopy, 218, 243 332, 340, 378, 386 Electric arc, 139, 156 Estrogen plus progestin therapy, 63, 84, DLCO (Carbon monoxide diffusing Electrocardiogram (ECG), 12–13 257, 265 capacity), 18 Electroconvulsive therapy (ECT), 165, Ethics (cesarean section decision), 55, 74 DNA polymorphism, 70 174, 180, 189 Exanthem, 102, 123 Donepezil, 168, 183 ELISA (See Enzyme-linked Exanthem subitum (See Roseola Dopamine, 9 immunosorbent assay) infantum) Dopamine beta-hydroxylase, 334, Emergency contraception, 257, 265 Excisional biopsy, for melanoma, 215, 342–343 Emergent cardiac catheterization, 393, 403 241 (c) ketabton.com: The Digital Library

418 Index

Exercise-induced asthma, 104, 125 E. coli 0157:H7 infection, 142, 160 GnRH agonists, 54, 73 Expected delivery date (EDD), 55, 74, and food-handling methods, 130, 147, Gonadal dysgenesis (See Turner’s 351, 360 354, 362–363 syndrome) Experience rating, 144, 160 incubation periods, 139, 155 Gonorrhea, 53, 72, 376, 384 Extrapyramidal side effects (EPS), prevention of, 142, 160 Griseofulvin, 92, 111–112 165, 180 Food-handling methods, 130, 147 Gross hematuria, 224–225, 248 Eye pain, imipramine and, 172, 187 Forceps, for occiput posterior delivery, Ground beef, 142, 160 Eyelid, swollen, 93, 113 57, 76–77 Groundwater, coliform bacteria in, Eyes Foreign body aspiration, 256, 264 144, 161 injury in children, 88, 108 Formaldehydes, 139, 156 Group A Streptococcus, 380, 387 subluxation of the lenses, 2, 26 45,X karyotype, 63, 84 Group B Streptococcus (GBS) viral conjunctivitis, 28–29 45,XO karyotype, 101, 121 bacterial infection in infants, 380, 387 Fox bite, vaccinations following, 395, childbirth, 52, 70–71 F 398, 405, 407 and early onset neonatal sepsis, 299, Facial swelling, ACE inhibitors and, Fractures, among elderly, 130, 147 306 318, 326–327 Framingham Study, 131, 147 GSI (See Genuine stress incontinence) Failure to breathe, 98, 118 Frontotemporal cortex, 168, 183 Guillain-Barré syndrome, 97, 117 False negative/positive, 141, Frostbite, 227, 250 Gunshot wounds, 197, 228, 275, 283–284 158–159 Frotteurism, 166–167, 181 Gynecomastia, 89, 109–110 Febrile nonhemolytic transfusion FSH (See Follicle-stimulating hormone) reaction, 226, 249 Functional asplenia, 118 H Fecal occult blood testing (FOBT), Functional constipation, 99, 119 Haemophilus influenzae, 114 140, 157 Furunculosis, 141, 158 Haemophilus influenzae b (Hib) vaccine, Fecundability, of normally fertile 132, 148–149, 273, 282 couples, 58, 77 G Haloperidol, 171–172, 186, 396, 405 Femoropopliteal bypass, 258–259, 267 Garage worker, carbon monoxide Hand, foot, and mouth disease, 92, 112 Fertility/infertility, 60, 80 exposure, 139, 156 Hand washing, 132, 149, 317, 326 Fetal alcohol syndrome, 100, 120, 296, 303 Gas exposure symptoms, 139, 156 Hand-foot syndrome, 103, 123 Fetal biophysical profile, 53, 72 Gastric bypass, 273, 282 Hashimoto’s thyroiditis, 8, 14, 24, 29, Fetal heart rate, 62, 82 Gastric ulcers 33, 43 Fetal hypoxia, 55, 74 Heliobacter pylori, 141, 158 HBIG (See Hepatitis B immune globulin) Fetal red blood cells, 50, 68 recurrence, 219, 244 Head trauma, 94, 114, 254, 262 Fetal vertex, 394, 404 symptoms, 211, 238, 332, 341 Headache (various diagnoses), 379, 386 Fibroadenoma, 373–374, 382 Gastroesophageal reflux disease (GERD), Health care planning methods, 138, 154 Fibrocystic disease of the breast, 59, 352–353, 361 Health funding, 130, 147 61, 79, 82 Gastrointestinal bleeding Health insurance Fibromyalgia, 10–11, 31 and jejunal hamartomas, 357, 366 experience rating, 144, 160 FIGO stage, of cervical cancer, 58, 77 various diagnoses, 332–333, 340–341 marketing of, 132, 149 Fine-needle aspiration, for breast cancer Gastrointestinal hemorrhage, 333, percentage of Americans without, management, 56, 59, 62, 75–76, 341–342 313, 321 78, 82 GBS (See Group B Streptococcus) Health Insurance Portability and Fish, PCBs in, 139, 155 General anesthesia, malignant Accountability Act (HIPAA), Fluconazole, 312, 320 hyperthermia risk from, 204, 233 259, 267 Fluorescein dye stain of the cornea, Generalized anxiety disorder, 178, 193 Health Maintenance Organizations 88, 108 Generalized seizures, 313, 321 (HMOs), 138, 144, 154, 161 Fluoridation, of drinking water, 131, Genetic amniocentesis, 56–57, 76 Health risk profile, stratified random 148 Genital herpes, 48, 66 sample for, 134–135, 150 5-fluorouracil chemotherapy with Gentamicin, 5 Health survey, 142, 145, 159, 162 leucovorin, 213, 240 Genuine stress incontinence (GSI), 51, Hearing loss, 299, 306 Fluoxetine, 165, 177, 179, 180, 192, 193 56, 70, 75 Heavy metal exposure, chelation for, Fluphenazine, 353, 362 GERD (See Gastroesophageal reflux 136, 151 Fluvoxamine, 315, 323 disease) Height, prediction of, 299, 306 FOBT (See Fecal occult blood testing) Gestation age, 351, 360 Helicobacter pylori, 141, 158 Folic acid, 394, 403 Gestational diabetes mellitus, 53, 72 Hematopoietic acute radiation Follicle-stimulating hormone (FSH), Gestational trophoblastic disease, 64, 84 syndrome, 401, 409 51, 61, 70, 81 GFR (See Glomerular filtration rate) Hemodialysis, 5, 28 Follicular conjunctivitis (See Viral Giant cell arteritis, 22, 40 Hemolytic-uremic syndrome, 97, 116, conjunctivitis) Giardia lamblia, 140, 141, 158 142, 160 Follow-up studies, 392, 402 Giardiasis, 140, 158 Hemophilus type B infection, 273, 282 Food allergies, 101, 121 Glioma, 4, 27 Hemoptysis, 19, 38, 260, 269 Food and Drug Administration, 139, 155 Glomerular filtration rate (GFR), 53, Henoch-Schönlein purpura (HSP), 298, Food-borne illness 71–72 305 active surveillance system for BSE, Glomerulonephritis, 141, 158 Heparin, 57, 76 145, 162 Glomerulonephritis, chronic, 53, 71–72 Heparin-induced thrombocytopenia Bacillus cereus, 311, 319 Glucocorticoids, 51, 69 (HIT), 257, 258–259, 265, 267 (c) ketabton.com: The Digital Library

Index 419

Hepatic abscess, 371–372, 381 Hydrochlorothiazide, 15, 35 Imperforate hymen, 61, 81 Hepatic encephalopathy, 17, 36 Hydrogen sulfide, 139, 156 Incarcerated femoral hernia, 215, 241 Hepatitis A Hydrostatic barium enema, 220–221, 245 Incontinence in daycare employee, 88, 108 17α-Hydroxylase deficiency, 356, 364 and detrusor dyssynergia, 63, 83 IG for, 140, 157 21-Hydroxylase deficiency, 50, 69 and detrusor overactivity, 260, 268 symptoms, 14, 33–34 17-Hydroxyprogesterone, 317, 325–326 and diabetes mellitus, 56, 75 in underdeveloped countries, 400, 408 Hypercalcemia GSI, 51, 56, 70, 75 Hepatitis A immune globulin, 88, 108 and metastatic breast cancer, 4, 27 overflow incontinence, 56, 75 Hepatitis A vaccine, 88, 108, 136, 152 and primary hyperparathyroidism, Incubation periods Hepatitis B 13, 32 for food-borne illness, 139, 155 IG for, 140, 157 and squamous lung cancer, 19, 38 for infectious diseases of childhood, protection of newborn from, 87, 107 Hypercholesterolemia screening, 275, 284 140, 158 screening schedules, 140, 157 Hyperlipidemia, 331, 340 Individual psychotherapy, 167, 182 symptoms, 14, 34 Hyperparathyroidism, 13, 15, 32, 34 Indwelling bladder catheter, 254, 262 Hepatitis B immune globulin (HBIG), Hyperproteinemia, 22, 41 Infant(s) 87, 107 Hypersensitivity, to vaccine, 99, 119 acrodermatitis enteropathida, 358, 366 Hepatitis C Hypertension acute laryngotracheobronchitis, IG for, 140, 157 control of, for aortic dissection 351, 360 and IV drug use, 374, 382–383 management, 217, 242 anemia, 298, 304–305 symptoms, 14, 34 and diabetes, 32–33 bacterial infection, 380, 387 Hepatosplenomegaly, 102, 122–123 and intracerebral hemorrhage, 10, 31 botulism from honey, 101, 121 Herpes simplex, 377–378, 385, 401, 409 ISH, 331, 340 bowel obstruction, 352, 361 n-hexane, 336, 344 lisinopril/hydrochlorothiazide breast-feeding jaundice, 255, 263 hGC (human chorionic gonadotropin) combination for, 317, 325 bullous impetigo, 398, 406–407 test, 253, 257, 261, 266 malignant, 316, 324 Candida dermatitis, 96, 116, 400, High-altitude pulmonary edema, 16, 35 and placental abruption, 61, 81 408–409 HIPAA (See Health Insurance Portability and renal artery stenosis, 338, 346 cervical adenitis, 380, 387 and Accountability Act) renal disease and, 98, 117 choanal atresia, 98, 118 Hirschsprung disease, 102, 122, 361, and rosacea, 5 congenital adrenal hyperplasia, 374–375, 383 screening for, 358, 367 314, 322 Hirsutism, 50, 68–69, 317, 325–326 side effects of medication, 35 congenital syphilis, 274, 282–283 Historical cohort study, 354, 362 sleep apnea and, 318, 326 constipation, 102, 122 HIT (See Heparin-induced venlafaxine side effect, 253, 261 CT scan for trauma, 88, 108 thrombocytopenia) Hyperthermia, malignant, 204, 233 DDH, 314, 323 HIV (See Human immunodeficiency Hypertriglyceridemia, 5, 27 duodenal atresia, 280, 288–289 virus) Hypertrophic obstructive E. coli infection, 89–90, 110 HMOs (See Health Maintenance cardiomyapathy (HOCM), fever, 89, 110 Organizations) 337, 344–345 functional constipation, 99, 119 HOCM (See Hypertrophic obstructive Hyperventilation, 98, 117–118 hepatitis B immunization, 87, 107 cardiomyapathy) Hypochloremic metabolic alkalosis, 95, 116 herpes simplex exposure, 401, 409 Holosystolic, blowing murmur, 2 Hypochondriasis, 166, 173, 181, 187–188 Hirschsprung disease, 374–375, 383 Honey, and botulism in infants, 101, 121 Hypokalemia, 6, 28, 372, 381 hypochloremic metabolic alkalosis, Hormone replacement therapy, 257, 265 Hypomanic state, 177, 192 95, 116 HPV (See Human papillomavirus) Hypoparathyroidism, 15, 34 immunizations, 98, 118 HSP (See Henoch-Schönlein purpura) Hypotension, 392–393, 403 intussusception, 103, 124, 220–221, 245 Human chorionic gonadotropin (hCG), Hypotonia, 278, 287 Meckel’s diverticulum, 377, 385 19, 38, 50, 69 Hyrdralazine, 15, 35 occult bacteremia, 394, 404 Human immunodeficiency virus (HIV) otitis media, 93–94, 113–114 blood bank testing, 294, 302–303 I perineal hypospadias, 102, 122 CD4+ T-cell count, 13, 32 Ibuprofen, 339, 346–347 progressive cholestatic jaundice, ELISA test for, 394, 403–404 Identical twins, 52, 70 225, 248 incubation period, 16, 35–36 Idiopathic hypercalciuria, 23, 42 pyelonephritis, 300, 307 medication options, 16, 35–36 Idiopathic scoliosis, 100, 120 pyloric stenosis, 299, 305–306 opportunistic infections, 21, 32, 40 Idiopathic thrombocytopenic purpura retinal hemorrhages, 298, 304 prevention of transmission, 400, 408 (ITP), 97, 123, 225, 248–249 roseola infantum, 311, 319–320 screening study, 141, 158–159 IG (See Immune globulin) rotavirus, 90, 110–111 Human lymphocyte antigens, 52, 70 ILD (See Interstitial lung disease) sickle-cell disease, 396–397, 406 Human papillomavirus (HPV), 300–301, Image-guided percutaneous biopsy, sleep position for, 87, 108 307, 375, 383 59, 78 sweat chloride test for cystic fibrosis, Human papillomavirus (HPV) vaccine, Imipramine, 172, 187 311, 319 55, 73–74, 375, 383 Immune globulin (IG), various clinical trauma, 88, 108 Humerus, supracondylar fracture of, indications for, 140, 157 vitamin K deficiency, 94, 115 221, 246 Immunocompromised patients whooping cough, 100, 120–121 Hydatiform mole, 48, 66, 311, 319 vaccinations for, 97, 99, 116–119 Infant botulism, 101, 121 Hydralazine, 5, 27 VZIG for, 87, 107 Infant mortality rates, 145, 162 (c) ketabton.com: The Digital Library

420 Index

Infantile autism, 176, 191 J Lithium carbonate, 178, 179, 193, 194, Infectious endocarditis, 353, 361–362 Jaundice 396, 405 Infectious mononucleosis, 101, 121, 140, 158 breast-feeding jaundice, 99, 119, Liver, fetal red blood cell production in, Inferior wall myocardial infarction, 7, 29 255, 263 50, 68 Infertility, 60, 77, 80 various diagnoses, 206–207, 236 Liver metastases, 207, 236 Infiltrating ductal carcinoma, 59, 79 Jehovah’s Witnesses, blood transfusions Livestock workers, 138, 153 Influenza A virus, 129, 146 and, 134, 149–150 Lobular carcinoma in situ (LCIS), 208, Influenza vaccinations, 144, 161 Jejunal atresia, 223, 247 237 Inguinal hernia, 204, 224, 233, 247–248 Jejunal hamartomas, 357, 366 Lorazepam, 169, 171, 184, 186 INH (See Isoniazid) Juice, dental caries and, 92, 111 Louse-borne typhus (See Epidemic Inhalational anthrax, 373, 382 typhus) Injectable polio vaccine, 393, 403 K Low back pain, 20, 39, 378, 385–386 Inpatient psychiatric hospitalization Karyotype Low-density lipoprotein (LDL), 253, 261 for anorexia nervosa, 168, 182–183 for Müllerian agenesis screening, Low-dose antidepressant therapy, 11, 31 for suicidal ideation, 173, 187 314, 321 Lower gastrointestinal bleeding, 8, 30 Insecticides (organophosphorous for Turner’s syndrome, 63, 84 LSIL (low-grade squamous compounds), 139, 155–156 Kawasaki disease, 92, 93, 105, 112, 126 intraepithelial lesion), 375, 384 Inspiratory stridor, 94, 114 Kidney stones, 23, 42 Lung abscesses, periodontal disease and, Institutional planning, 138, 154 Knee, pain/swelling in, 3, 26, 105, 126 392, 402 Insurance (See Health insurance) Korsakoff’s syndrome, 313, 321 Lung cancer Intelligence assessment, 278–279, 287 death rates among women, 134, 150 Intermenstrual spotting, 55, 74 L lack of screening for, 378, 385 Internuclear ophthalmoplegia, 277, 285 Labor metastatic, 18, 37 Interstitial lung disease (ILD), 18, 21, first stage, 281, 289, 394, 404 small cell carcinoma, 6, 28 37, 40 induced, 253, 260, 261, 268, 331, 340 Lung tumor, 3, 26 Intoeing, 100, 120 and postpartum hemorrhage, 260, 268 Lupus, 22, 41 Intracerebral hemorrhage, 10, 31 and preeclampsia, 331, 340 Luteinizing hormone (LH), 44, 51, 69–70 Intracranial aneurysms, 380, 387 rh immune globulin administration, Lyme disease Intracranial hemorrhage, acute, 94, 114 56–57, 76 clinical manifestations, 358, 366 Intractible pelvic hemorrhage, 62, 83 spontaneous onset of, 50, 68 and erythema migrans, 398, 407 Intraductal carcinoma, 4 vaginal bleeding during, 60, 81 geographic risk factors, 135, 151 Intraductal papilloma, 199–200, 229 vertex presentation, 62, 82 as occupational hazard, 138, 153–154 Intrauterine pregnancy (See Tubal ectopic Laboratory tests (See Screening tests) Lymphadenitis, 101, 122 pregnancy) Lactated Ringer’s, 335, 343 Lymphedema, 68 Intrauterine sac, 51, 69 Laparotomy, 216, 242, 391, 402 Lymphocytic infiltration, 24, 43 Intubation, 98, 117–118 Laryngeal carcinoma, 206, 236 Intussusception, 103, 124, 220–221, 245, Laryngeal edema, 399, 408 M 373, 381–382 Laryngotracheobronchitis, acute, Macular erythroderma, 16, 35 Invasive carcinoma of the breast, 210, 238 351, 360 Magnesium sulfate, 331, 340 Invasive squamous cell carcinoma of the Late-onset schizophrenia, 167, 182 Major depressive disorder (See also cervix, 48, 66 LBBB (Left bundle branch block), 13 Depression) Involuting fibroadenoma, 198–199, 229 LCIS (See Lobular carcinoma in situ) cognitive function, 174, 189 Iritis, acute, 6, 29 LCR test (See Ligase chain reaction test) cognitive psychotherapy for, 175, 190 Iron poisoning, 333, 341–342 LDL (See Low-density lipoprotein) electroconvulsive therapy for, 174, 189 Iron siderosis, 256, 264 Lead poisoning, 97, 117, 136, 151 recurrent, 176, 191 Iron-deficiency anemia, 23, 42–43, 94, Lead screening, 131, 147 sertraline for, 178, 193 114–115 Lead-time bias, 318, 327 sleep abnormalities, 174, 188 Ischemic colitis, 378, 386 Left bundle branch block (LBBB), 13 symptoms, 165, 175, 180, 189 Isolated systolic hypertension (ISH), Left tension pneumothorax, 202, 231 Malaria, 138, 154 331, 340 Left ventricular failure, 20, 38 Malignant glioma, 4, 27 Isoniazid (INH), 87, 107, 143, 160 Legionella, 8, 18–19, 30, 37 Malignant hypertension, 316, 324 ITP (See Idiopathic thrombocytopenic Leiomyoma, 54, 73, 217, 243 Malignant hyperthermia, 204, 233 purpura) Leukemia, 99, 118–119, 355, 364 Malignant melanoma, 371, 381 IV antibiotics Leukocytosis, 179, 194 Malingering, 166, 181 for infant, 90, 110 Levonorgestrel, 257, 265 Mallory—Weiss tear, 218, 243 for pyelonephritis, 300, 307 Lewy body dementia, 179, 194 Mammography IV fluids LH (See Luteinizing hormone) cross-sectional study, 294, 303 for aspirin overdose, 315, 323 Ligase chain reaction (LCR) test, for LCIS management, 208, 237 for bowel obstruction, 352, 361 256, 264 risk/benefit analysis, 144, 161–162 for right ventricular infarct, 393, 403 Linear model of regression, 397, 406 screening schedules, 140, 156–157 for small-bowel obstruction, 391, 402 Lipoma, 206, 235 Management, of disaster response, IV hydration Liquid-based cervical cytology, 298–299, 132, 149 for infant, 90, 110 305 Mandated vaccination, 273, 282 for pyelonephritis, 300, 307 Lisinopril, 15, 35, 317, 318, 325–327 Mania, 178, 193 IV penicillin, for GBS, 299, 306 Listeria monocytogenes, 17, 37 Manic episode, 279, 288 (c) ketabton.com: The Digital Library

Index 421

MAO inhibitor (See Monoamine oxidase Milk allergies in children, 101, 121 Nasal erythema, 5, 27 inhibitor) Mini-Mental State Examination (MMSE), Nasogastric decompression Marfan syndrome, 2–3, 26, 91, 111 174, 189 for bowel obstruction, 352, 361 Marshall-Marchetti-Krantz procedure, 75 Minor head injury (grade 1), 254, 262 for duodenal hematoma, 379, 387 Mass immunizations, for meningococcal Miosis, organophosphate poisoning and, for small-bowel obstruction, 391, 402 disease, 354, 362 101, 121 Neck mass Mastectomy, 62, 82, 208, 237 Mirtazapine, 171, 186 pleomorphic adenoma of the parotid, Maternal serum alpha-fetoprotein Mitral stenosis, 9, 30 220, 245 (MSAFP), 49, 56, 67–68, 75 Mitral valve prolapse, 4, 26–27, 91, 111 and thyroid cancer, 217, 243 Maternal/child health care system, Mixed parotid tumor, 206, 236 thyroid carcinoma, 312–313, 320–321 138, 154 MMSE (See Mini-Mental State various diagnoses, 206, 235–236 Maxilofacial trauma, 227, 250 Examination) Neck muscle spasm, 165, 180 McIndoe procedure, 322 Modal, 136, 152 Necrotizing soft-tissue infection, 221, 245 Mean, 136, 152 Modified radical mastectomy, 62, 82 Needle biopsy Measles, 140, 157 Monilial vaginitis, 59, 79–80 for breast cancer management, 210, Meat, health standards for, 139, 155 Monoamine oxidase (MAO) inhibitor, 238 Meckel’s diverticulum, 198, 202–203, 229, 178, 192 for fibroadenoma, 374, 382 231–232, 377, 385 Mononucleosis (See Infectious Needle decompression, 202, 231 Medial tibial torsion, 100, 120 mononucleosis) Nefazodone, 178, 192 Median, 136, 152, 399, 407–408 Moraxella catarrhalis, 114 Neonatal cholestasis, 99, 119 Medicaid, 143, 160 Morbid obesity, 15, 34 Neonatal sepsis, 52, 70–71, 299, 306 Medicare, 331, 334, 340, 343 Mortality rates, 137, 144, 152–153, 161, Nephrogenic diabetes insipidus (DI), Medicare Part B, 301, 307 374, 383 204–205, 234 Medullary thyroid carcinoma, 312–313, Mother, death of, 297, 304 Nephrotic syndrome, 22, 41 320–321 Motor vehicle crash victim Neural tube defect (NTD), 394, 403 Megaloblastic anemia, 20, 39 anion gap metabolic acidosis, 222, 246 Neurally-mediated syncope, 254, 261 Melanoma, 214–215, 241, 371, 381 delirium tremens in, 313, 321 Neurofibromatosis, 59, 79 Meningitis (See also Bacterial meningitis) DVT screening, 317, 325 Neuroleptic malignant syndrome (NMS), in adults, 17, 36–37 exploratory laparotomy for, 204, 234 171–172, 186–187, 274–275, 283 in children, 98, 117–118 maxilofacial trauma, 227, 250 Neuropathy, 256, 264 and meningococcemia, 100, 120 nutritional needs with severe head Newborns, 95, 115, 222–223, 246–247 from type B H. influenzae, 273, 282 injuries, 202, 231 Night terrors, 95, 116 Meningococcal disease, 354, 362 soft-tissue injury, 221, 245–246 Nissen fundoplication, 353, 361 Meningococcal meningitis, 131, 148 zinc deficiency, 226, 249–250 Nitroglycerine, 357, 365 Meningococcal vaccination, 276, 285 MRSA infections (See Methicillin- Nitrosamines, 139, 156 Meningococcemia, 100, 120 resistant staphylococcal aureus NMS (See Neuroleptic malignant Menstrual bleeding, 54, 73 infections) syndrome) Menstrual periods MS (See Multiple sclerosis) NNT (See Number needed to treat) and anemia, 23, 42–43 MSAFP (See Maternal serum alpha- Nocturnal enuresis, 89, 109 and polycystic ovarian disease, 24, 44 fetoprotein) Nonobstructing sigmoid colon Mental retardation Mucopurulent rhinorrhea, 256, 264 carcinoma, 293, 302 DSM axis system, 297, 304 Müllerian agenesis, 314, 322 Nonsmall cell lung cancer, 215, 241–242 fetal alcohol syndrome and, 100, 120 Multiaxial diagnostic system (mental Nontuberculous lymphadenitis (See prevalence across social classes, 399, health), 297, 304 Atypical mycobacterial 407 Multifocal hepatic abscess, 372, 381 lymphadenitis) Turner’s syndrome and, 278, 287 Multinodal goiter, 14, 33 Nortriptyline, 253, 261 Mesothelioma, 3, 26, 280, 288 Multiple sclerosis (MS), 276–277, 285 Nostril, foreign body lodged in, 89, 109 Metabolic alkalosis, 23, 42 Mumps, 140, 158 NTD (See Neural tube defect) Metabolic syndrome, 355, 363–364 Murine (endemic) typhus, 138, 153 Number needed to treat (NNT), 316, 325 Metastatic adenocarcinoma of the Muscle contraction headache, 379, 386 Nursemaid’s elbow (See Subluxation of prostate, 204, 233 Muscle fasciculation, 101, 121 the radial head) Metastatic breast cancer, 4–5, 27 Mycobacterium tuberculosis, 357, 365 Nursery epidemics, 141, 158 Metastatic disease to the lungs, 18, 37 Mycoplasma pneumoniae, 141, 158, 301, Methane, 139, 156 308 O Methicillin-induced acute interstitial Myelosuppression, 256, 264 OA (See Osteoarthritis) nephritis, 357, 365 Myocardial infarction (MI) Obesity, 15, 34, 273, 282 Methicillin-resistant staphylococcal acute, 357, 365 Observational study designs, 141, 159 aureus (MRSA) infections, 317, 326 cocaine use and, 296–297, 303–304 Obsessive-compulsive disorder (OCD), Methotrexate, 278, 286 inferior wall MI, 7, 29 315, 323 Metronidazole, 60, 80 Myoglobinuria, 8, 30 Obstructed airway, 100, 120 MI (See Myocardial infarction) Occiput posterior delivery, 57, 76–77 Microalbuminuria, 253, 261 N Occult bacteremia, 394, 404 Midgut volvulus, 223, 246 Naegele’s rule, 55, 74 OCD (See Obsessive-compulsive Migraine headache, 22, 23, 41–42, 379, Narcolepsy, 172, 187 disorder) 386 Nasal discharge, 95, 115 Odds ratio (OR), 137, 142, 153, 160 (c) ketabton.com: The Digital Library

422 Index

Olanzapine, 168, 177, 182–183, 192, Parathyroid adenoma, 339, 346 Pleomorphic adenoma, 206, 236 274–275, 283 Parathyroid hormone (PTH) test, Pleomorphic adenoma of the parotid, Oligomenorrhea, 317, 325–326 274, 283 220, 245 Oliguria, 13, 33 Parenteral alimentation, 379, 387 Pneumococcal infection, 373, 381 Omphalocele, 198, 228–229 Park Rangers, 138, 153–154 Pneumococcal vaccinations, 276, 285, Oocytes, 52, 70 Parkinson’s disease, 9, 30 336, 343–344 Open fracture, 197, 228 Paroxetine, 169, 175, 184, 189 Pneumocystis carinii pneumonia (PCP), Open-breast biopsy, fine-needle Paroxysmal supraventricular 21, 40 aspiration vs., 59, 78–79 tachycardia, 20, 39 Pneumonia OR (See Odds ratio) Passive-aggressive behavior, 179, 193 after appendectomy, 14, 33 Oral contraceptives, 47, 65, 259, 267 Pasturella multicoda, 95, 116, 393, 403 in children, 96–97, 116 Orbital cellulitis, 93, 113, 114 PCBs (See Polychlorinated biphenyls) and COPD, 260, 268 Orchiectomy, radical, 224, 246 PCP (See Pneumocystis carinii pneumonia) and respiratory rate, 8, 29 Ordinal scale, 399, 407 Pediculus humanus corporis, 138, 154 Pneumothorax, 19, 38, 396, 406 Organic compounds, toxic effects of, Pelvic hemorrhage, 62, 83 Polio vaccine, injection vs. oral, 97, 117, 139, 155–156 Penetrating wound, 204, 221, 233–234, 393, 403 Organophosphate poisoning, 101, 121, 245 Poliomyelitis, 129, 146 300, 306–307 Penicillin Polychlorinated biphenyls (PCBs), 139, Organophosphorous compounds, 139, for GBS, 299, 306 155 155–156 for sickle-cell anemia, 98–99, 118 Polycystic ovarian disease, 15, 24, 34, 44, Oropharyngeal airway, 98, 118 for sickle-cell disease, 396, 406 317, 326, 356, 364 Osgood-Schlatter disease, 105, 126 for streptococcal meningitis, 90, 111 Polymyalgia rheumatica, 21, 39 Osteoarthritis (OA), 22, 41 Penis, 294, 303, 377–378, 385 Polypectomy, 211, 239 Osteomalacia, 5, 28 Peptic ulcer, 332, 341, 378, 386 Polyuria, 6, 204–205, 234 Osteoporosis, 130, 147 Peptic ulcer, perforated, 205, 235 Population-based planning, 138, 154 Otitis externa, 99, 119, 141, 158, 394, 404 Percutaneous drainage, 372, 381 Positive predictive value (PPV), 142, 159, Otitis media, 93, 94, 96, 104, 113–114, Perforated duodenal ulcer, 219, 244 339, 346, 355, 363 123–124 Perforated peptic ulcer, 205, 235 Positron emission tomography (PET) Ovarian cancer, 47, 65 Pericardiocentesis, 337, 338, 345 scan, 316, 325 Ovaries, 54, 73 Pericarditis, acute, 338, 345 Postcoital spotting, 55, 74 Overflow incontinence, 56, 75 Periductal mastitis with abscess, Posterior fossa brain tumor, 259, 267–268 Ovulation, 52, 57, 70, 77 315–316, 324 Postoperative fever, 293, 302 Oxybutin chloride (Ditropan), 63, 83 Perineal hypospadias, 102, 122 Postpartum depression, 50, 68 Oxytocin, 253, 261 Periodontal disease, 392, 402 Postpartum fever, 278, 286 Periorbital cellulitis, orbital cellulitis vs., Postpartum hemorrhage, 58, 78, 226, P 93, 114 249, 260, 268 Paclitaxel, 256, 264 Periorbital edema, 97, 116 Postpartum pituitary failure, 260, 268, Pancreas, 8, 29 Peripheral neuropathy, 336, 344 356, 364–365 Pancreatic carcinoma, 206–207, 236 Peripheral pulmonic stenosis, 276, Post-traumatic epilepsy, 94, 114 Pancreatitis 284–285 Post-traumatic stress disorder (PTSD), acute, 5 Peritonsillar abscess, 105, 125 176, 190, 351–352, 360–361 and jaundice, 207, 236 Pertussis (See Whooping cough) Posture, idiopathic scoliosis and, 100, 120 and polyuria, 205, 234 Pesticides, regulation of, 138, 139, 155 PPD skin test (See Purified protein and zinc deficiency, 280, 288 PET scan (See Positron emission derivative skin test) Panic attack, 169, 183–184 tomography scan) PPROM (See Preterm premature rupture Panic disorder, 166, 181 Petechial rash, 100, 120 of amniotic membranes) Pap smear Petroleum jelly, and condom use, 400, 408 PPV (See Positive predictive value) ASCUS, 375, 384 Peutz-Jeghers syndrome, 357, 366 Prader-Willi syndrome, 170, 185, 295, 303 and HPV vaccine candidates, 73–74 PFT (See Pulmonary function testing) Precocious puberty, 94–95, 115 liquid-based cervical cytology, Pharyngoesophageal diverticulum, 218, Preeclampsia, 48, 66, 278, 286–287, 298–299, 305 243 331, 340 LSIL, 375, 384 Phenelzine, 178, 179, 192, 193, 396, 405 Pregnancy priority of patients needing, 395, 405 Phenytoin, 58, 78 chlamydia treatment during, 256, 264 screening interval, 133, 149 Pheochromocytoma, 15, 34, 313, 320–321 and chronic glomerulonephritis, 53, screening schedules, 140, 157 Phobia, 173, 188 71–72 Papilledema, 21, 39 Physostigmine, 166, 181 and diabetes mellitus, 53, 72 Paraesophageal hernia, 200–201, 229–230 Pick’s disease, 168, 175, 183, 190 EDD, 351, 360 Parainfluenza virus, 301, 308 Pill-rolling tremor, 165, 180 gestation age, 351, 360 Paranoid personality disorder, 178, 193 Pityriasis rosea, 87, 107 HCG test, 253, 261 Paranoid schizophrenia, 167, 182–183 Placenta accreta, 64, 85, 333, 341 hydatiform mole, 48, 66, 311, 319 (See also Chronic paranoid Placenta previa, 64, 84 loss of, 56, 74–75 schizophrenia) Placental abruption, 60–61, 64, 81, 84 mean duration of, 55, 74 Parapneumonic effusion, 24 Plain abdominal radiograph, 391, 402 and preeclampsia, 278, 286–287 Parasomnias, 95, 116 (See also Sleep Planning methods, for health care, supine hypotensive syndrome, 62, 82 disturbance) 138, 154 weight gain, 318, 326 (c) ketabton.com: The Digital Library

Index 423

Premature thelarche, 47, 65–66 Q Risperidal, 174, 189 Prenatal care, 131, 147–148 Quetiapine, 255, 262, 353, 362 Risperidone, 174, 178, 188, 193 Prescription drug coverage, Medicare, Rosacea, 5, 27 334, 343 R Roseola infantum, 102, 123, 311, 319–320 Preterm premature rupture of amniotic Rabies, 143, 161, 395, 398, 405, 407 Rotavirus, 90, 110–111 membranes (PPROM), 53, 72–73 Racemic epinephrine, 254, 262 “Round pneumonia,” 96–97, 116 Prevalence, in surveys, 134–135, 142, 150, Radiation syndrome, acute, 401, 409 Routine immunization 159, 298, 305 Radiation therapy, for cervical cancer, DTaP, 132–133, 149 Priapism, as trazodone side effect, 253, 58, 77–78 Haemophilus influenzae b (Hib) vaccine, 261 Radical orchiectomy, 224, 246 132, 148–149 Primary adrenocortical deficiency (See Random sampling, 134–135, 150 as primary prevention, 130, 147 Addison’s disease) Randomized clinical study, 298, 305 RSV (See Respiratory syncytial virus) Primary amenorrhea, 101, 121 Range, of readings, 136–137, 152 RTA, type 4, 13 Primary anastomosis, 213, 241, 254, 262 Rape, PTSD and, 176, 190 Rubella, 140, 158 Primary hyperparathyroidism, 13, 32 RBRVSs (See Resource-Based Relative Ruptured abdominal aneurysm, 216, 242 Primary infertility, 60, 80 Value Scales) Ruptured cerebral aneurysm, 227, Primary nocturnal enuresis, 89, 109 Reaction formation, 179, 193 250–251 Primary prevention, 130, 147, 275, 284, Rectal biopsy, 352, 361 372–373, 381 Rectal bleeding S Primary syphilis infection, 294, 303 diverticulosis of the colon, 8, 30 Salicylate intoxication, 15, 34 Prions, 5, 28 Meckel’s diverticulum, 202–203, Salmonella, 139, 155 Private foundations, HIPAA regulations 231–232 Sarcoidosis, 22, 41 for, 259, 267 in toddler, 198, 229 Sarcoptes scabiei, 138, 154 Proctoscopy, 213, 214, 240, 241 various diagnoses, 378, 386 Sarin, 354, 363 Program-based planning, 138, 154 Rectal carcinoma, 214, 240–241 Scaploid fracture, 226, 249 Progressive cholestatic jaundice, 225, 248 Red urine, 104, 125 Scarlet fever, 90, 111 Projectile vomiting, 95, 116 Reducible inguinal hernia, 224, 247–248 Schizoid personality disorder, 169, 178, Prolactin-secreting pituitary tumor, 356, Regional enteritis, 20, 39 184–185, 193 364 Regression, 312, 320 Schizophrenia (See also Paranoid Prolonged QT, as nortriptyline side Relapsing-remitting type MS, 277, 285 schizophrenia) effect, 253, 261 Relative risk, 137, 143–144, 153, 160, 392, negative symptoms, 175, 189–190 Prophylactic bilateral mastectomy, 208, 402–403 and suicide risk, 172, 187 237 REM latency, 174, 188 Schizotypal personality disorder, 178, 193 Propranolol, 165, 180 Renal artery stenosis, 338, 346 Sciatica, 294, 303 Proptosis, 93, 114 Renal disease, hypertension and, 98, 117 Scoliosis, 100, 120 Prostate cancer, 136, 151–152, 204, 233 Renal failure, chronic, 5, 28 Screening tests Prostatic hypertrophy, 13, 33 Renal function, malignant hypertension and positive predictive value, 142, 159 Proteinuria, 103, 124, 299, 306 and, 316, 324 risk/benefit analysis, 144, 161–162 Proximal muscle weakness, 1, 25 Renal insufficiency, chronic, 5, 28 schedules for women, 140, 156–157 PSA test, 136, 151–152 Renal stones, 23, 42 sensitivity of, 142, 159 Pseudogout, 359, 367 Renal toxicity, 255, 263–264 SD (See Standard deviation) Pseudomonas aeruginosa, 99, 119 Renal ultrasound, 90, 102, 110, 122, Seborrheic keratoses, 12, 32 Pseudotumor cerebri, 21, 39–40 300, 307 Secondary amenorrhea, 51, 70, 257, 266 Psoriasis, 376, 384 Resistant hypertension, 318, 326 Secondary dysmenorrhea, 255, 263 Psychiatric medications, side effects of, Resource-Based Relative Value Scales Secondary gain, 173, 188 396, 405–406 (RBRVSs), 301, 307 Secondary prevention, 147, 275, 284 Psychosis, 177, 192, 254–255, 262–263 Respiratory alkalosis, 6, 28 Secondary syphilis, 359, 367 Psychotherapy, individual, 167, 182 Respiratory distress, 256, 264, 391, 402 Second-degree heart block, Type I, Psychotic depression, 169, 184 Respiratory illness, in infants, 301, 308 336, 344 PTH test (See Parathyroid hormone test) Respiratory syncytial virus (RSV), 141, Sedimentation rate test, 11, 31 PTSD (See Post-traumatic stress disorder) 158, 301, 308 Selection bias, 137, 153 Puberty, 49, 63, 68, 84 Resting tremor, 30 Selective serotonin reuptake inhibitor Pulmonary edema, 20, 38 ret protooncogene screening, 313, 321 (SSRI), 178, 189, 192, 193 Pulmonary function testing (PFT), 18, 37 Retinal hemorrhages, 298, 304 Selenium sulfide shampoo, 92, 111–112 Pulmonary immaturity, 51, 69 Retropubic urethropexy, 56, 75 Semen analysis, 60, 80 Pulmonary stenosis, 95, 115 Rh immune globulin, 56–57, 76 Semen fructose test, 60, 80 Pulmonary tuberculosis, 11, 31 Rhabdomyolysis, 8, 30 Seminoma, 19, 38, 224, 247 Puncture wound, 95, 116 Rheumatoid arthritis, 22, 41 Sensitivity, positive predictive value and, Purified protein derivative (PPD) skin Rhinorrhea, mucopurulent, 256, 264 142, 159, 339, 346 test, 377, 385 Rickets, 93, 113 Sensorimotor peripheral neuropathy, Purulent drainage, 89, 109 Rifampin, 131, 148 133, 149 Pyelonephritis, 14, 33, 110, 300, 307 Right epidural hematoma, 375, 383 Sentinel lymph node (SLN) biopsy, Pyloric stenosis, 95, 116, 201, 230, 299, Right ventricular infarct, 393, 403 208–209, 237–238 305–306 Rigid proctoscopy, 258, 266 Separation anxiety disorder, 170, 185 Pyogenic hepatic abscess, 371–372, 381 Risk/benefit analysis, 144, 161–162 Septic pelvic thrombophlebitis, 278, 286 (c) ketabton.com: The Digital Library

424 Index

Septic shock, 20, 39 Smoking cessation, 133, 149, 275, Supracondylar fracture of the humerus,

Serotonin type 2 (5-HT2) receptor 284, 336, 344 221, 246 blockade, 178, 192, 193 Smoking education, 275, 284 Surface water, coliform bacteria and, Sertraline, 178, 193, 352, 360–361 Snare polypectomy, 211, 239 144, 161 Serum 17-hydroxyprogesterone, 317, Soft-tissue injury, 221, 245–246 Surgical resection, 220, 244 325–326 Somatization disorder, 166, 181 Surrogate guardian, 134, 149–150 Serum creatine phosphokinase (CPK), Specificity, positive predictive value and, SVR (See Systemic vascular resistance) 8, 172, 186 142, 159, 339, 346 Sweat chloride test, 311, 319 Serum dehydroepiandrosterone sulfate Sperm concentration, 60, 80 Symptomatic anemia, 226, 249 (DHEAS), 50, 69 Spermatogenesis, 60, 80–81 Symptomatic lead-poisoning disease, Serum estradiol concentration test, 47, 65 Splenectomy, 225, 248–249, 276, 285 136, 151 Serum FSH concentration, 61, 81 Splenic sequestration (See Acute splenic Symptoms, treatment of, 2, 16, 26 Severe aortic stenosis, 339, 346 sequestration) Syncope, 254, 261 Severe aortic valve disease, 13, 32 Splenomegaly, 92, 112 Syphilis, 274, 282–283, 294, 303, 359, 367 Severe depression, 177, 192 Spontaneous esophageal perforation, Systemic lupus erythematosus (SLE), 22, Severe hypertension, 338, 346 355, 363 41, 92, 111 Shaken baby syndrome, 88, 108, 298, 304 Sports injury (head trauma), 254, 262 Systemic vascular resistance (SVR), 20, 39 Sheehan’s syndrome (See Postpartum Sports participation, systolic murmur Systolic ejection murmur, 276, 284 pituitary failure) and, 337, 344 Systolic murmur, 337, 344 Sickle-cell disease Squamous lung cancer, 19, 38 in children, 102–103, 123 SSRI (See Selective serotonin reuptake T in newborns, 396–397, 406 inhibitor) Tamoxifen, 5, 49, 67 peripheral blood smear for diagnosis, Stab wound, 337, 345 Tardive dyskinesia, 177, 192, 353, 362 376, 384–385 Stage II hypertension, 317, 325 Taxol (See Paclitaxel) prophylactic penicillin for, 98–99, 118 Staging, of colon cancer, 213, 240 Tension headache, 379, 386 Side effects, of antipsychotic Standard deviation (SD), 137, 152 Tension pneumothorax, 255, 263 medications, 165, 180 Staphylococcal food poisoning, 354, Teratogenesis, 57, 76 SIDS (See Sudden infant death 362–363 Terminally ill patients, 258, 266–267 syndrome) Staphylococcus aureus, 139, 155, 353, Tertiary prevention, 147, 275, 284, 395, 404 Sigmoid colon resection, 254, 262 361–362 Testicular cancer, 19, 38 Sigmoid diverticulitis, 205, 235 Statistical analysis (See Clinical studies) Testicular torsion, 93, 113, 203, 233 Sigmoid volvulus, 212–213, 239–240, Status epilepticus, 4 Testosterone, 61, 81 258, 266 Stillbirth, 53, 72, 274, 282–283 Tetanus, 140, 158 Sigmoidoscopy, 216, 242 Still’s murmur, 276, 284 Tetanus immune globulin (TIG), 98, 118 Silicosis, 135, 151, 395, 405 Stratified random sample, 134, 150 Tetanus toxoid, 221, 245–246, 395, 405 Silver nitrate eye drops, 103, 123 Streptococcal meningitis, 90, 111 Thiamine, 313, 321 Simple febrile seizure, 102, 123 Streptococcal pharyngitis, 90, 111 Thiazide diuretics, 15, 35 Simvastatin, 334, 342 Streptococcal pyoderma, 141, 158 Thioridazine, 174, 188 Sinus arrhythmia, 297–298, 304 Streptococcus, 2, 25 Thoracotomy, 217, 243 Sinus headache, 379, 386 Streptococcus aureus, 35 Thromboangiitis obliterans, 338, 345 Sinusitis Streptococcus pneumoniae Thrombocytopenia, 179, 194 acute, 2, 26 and bacterial meningitis, 17, 37, 397, 406 Thrombocytosis, 112 bacterial, 256, 264 and COPD, 260, 268 Thromboembolectomy, 216, 242 Sixth disease (See Roseola infantum) ear infection, 103, 123–124 Thrombolytic therapy, 7 Skin lesion, melanoma and, 214–215, 241 and occult bacteremia, 394, 404 Thyroglossal duct cyst, 206, 235 SLE (See Systemic lupus erythematosus) and otitis media, 114 Thyroid carcinoma, 206, 217, 236, 243, Sleep apnea, 318, 326 and pneumonia, 97, 116 312–313, 320–321 Sleep disturbance Subarachnoid hemorrhage, 10, 31, 379, Thyroid hormone, 24, 43 and depression, 174, 175, 188, 189 380, 386, 387 Thyroidectomy, 313, 321 and fibromyalgia, 11, 31 Subdural hematoma, 9, 30, 376, 384 TIA (See Transient ischemic attack) night terrors, 95, 116 Sublimation, 179, 193 Tick(s), 135, 151 Sleep position, SIDS and, 87, 108 Subluxation of the lenses, 2, 26 TIG (See Tetanus immune globulin) SLN biopsy (See Sentinel lymph node Subluxation of the radial head, 101, Tinea capitis, 92, 111–112 biopsy) 121–122, 315, 323–324 Tobbaco (See Smoking) Small cell lung carcinoma, 6, 28 Submucosal fibroids, 54, 73 Toddlers Small-bowel obstruction, 391, 402 Substance-induced mood disorder, bacterial meningitis, 397, 406 Small-bowel radiography, 203, 233 177, 191 hepatitis A risk, 88, 108 Smoking Sudden infant death syndrome (SIDS), intussusception, 373, 381–382 and cerebrovascular system illness, 87, 108 iron-deficiency anemia, 94, 114–115 131, 147 Suicide, 172, 175, 187, 189 lead poisoning, 97, 117 and CHD, 333, 342 Sumatriptan, 22, 42 purulent drainage, 89, 109 cohort study, 141, 159 Supine hypotensive syndrome, 62, 82 rectal bleeding, 198, 229 and mesothelioma, 280, 288 Suppression, 179, 193 subluxation of the radial head, 315, and oral contraceptive use, 259, 267 Suppurative lymphadenitis, acute, 323–324 and thromboangiitis obliterans, 338, 345 206, 235 and vaccinations, 99, 119 (c) ketabton.com: The Digital Library

Index 425

Toe infections, 274, 282 United States Department of Agriculture Ventricular infarct, 393, 403 Tonic-clonic seizure, 102, 123 (USDA), 139, 155 Ventricular septal defect, 276, 285 Tonsillectomy, 98, 117 Upper gastrointestinal (UGI) Vertex presentation, 62, 82 Tooth extraction, 2 hemorrhage, 218, 243 Vesicoureteral reflux, 102, 122 Toxic shock syndrome (TSS), 16, 35 Upper respiratory infection (URI), 16, Viral conjunctivitis, 6, 28–29 Toxic synovitis, 260, 269 35, 264 Viral upper respiratory infection (URI), Transesophageal echocardiography, Ureteral calculus, 201–202, 231 264

217, 242 Urethrocystometry, 63, 83 Vitamin B12 deficiency, 20, 39 Transient ischemic attack (TIA), 274, 282 Urethral catheter, 13 Vitamin D, 113, 275, 284 Transient monocular blindness, 11, 31 URI (See Upper respiratory infection) Vitamin K deficiency, 94, 115 Transposition of the great arteries, 276, Urinalysis, 103, 124 Vitiligo, 8, 29 284 Urinary hesitancy, 174, 188–189 Voiding cystourethrogram (VCUG), 90, Transtheoretical model of behavior Urinary incontinence (See Incontinence) 110, 300, 307 change, 355, 363 Urinary tract infections, 22–23, 42 Vomiting, by infants, 89, 90, 110–11 Travelers’ diarrhea, 374, 382 Urine collection, from infant, 89, 110 von Willebrand disease, 378–379, 386 Trazodone, 253, 261, 396, 405 Urine discoloration, 104, 125 Vulvar carcinoma, 48, 66–67 Treponema pallidum, 102, 122–123 Urine specimen, catheterized, 89, 110 Vulvar lesions, 300–301, 307 Tretinoin, 316, 324 USDA (See United States Department VZIG (See Varicella immune globulin) Triazolam, 357, 365 of Agriculture) Trichotillomania, 256, 264–265 Uterine anomalies, 52, 71 W Tricyclic antidepressant medication, 189 Uterine atony, 58, 64, 78, 85 Warehouse workers, 138, 153 Trifluoperazine, 177, 192 Uterine fibroids, 54, 73 Wastewater treatment plant overflow, Triglycerides, 5, 27–28 Uterine fundus, 49, 58, 67, 78 132, 149 Trimethoprim-sulfamethoxazole, 300, Uterine packing, 333, 341 Water, fluoridation of, 131, 148 307 Uterine rupture, 64, 85 Water pollution, poliomyelitis and, 129, Trisomy 21, 56, 75, 273, 278, 282, 287 146 TSS (See Toxic shock syndrome) V Wechsler Intelligence Scale for Children Tubal ectopic pregnancy, 50, 64, 69, Vaccine-associated paralytic (WISC), 278–279, 287 84–85 poliomyelitis, 97, 117 Wegener’s granulomatosus, 22, 40 Tuberculin skin test (TST), 87, 107 Vagina, surgical creation of, 314, 322 Weight gain during pregnancy, 318, 326 Tuberculosis Vaginal bleeding Well water, coliform bacteria and, 144, in AIDS patients, 11 after childbirth, 333, 341 161 in children, 87, 107 during labor, 60, 81 Wernicke’s encephalopathy, 313, 321 isonazid for management of, 143, 160 postpartum, 58, 78 Whipple disease, 16–17, 36 management of, 377, 385 various diagnoses, 64, 84–85 White-footed mouse, 135, 151 and silicosis, 395, 405 Vaginal discharge WHO (See World Health Organization) silicosis and, 135, 151 and bacterial vaginosis, 60, 79–80 Whooping cough, 100, 120–121, 140, 158 Tuberculous meningitis, 357, 365 Candida albicans, 312, 320 Wilms tumor, 89, 110 Turner syndrome (gonadal dysgenesis) monilial vaginitis and, 59, 79–80 WISC (See Wechsler Intelligence Scale and congenital heart disease, 91, 111 Vaginal laceration, 64, 84–85 for Children) and delayed puberty, 63, 84 Vaginal spotting, hCG test for, 253, 261 Women, cancer death rates, 134, 150 diagnosis, 356, 365 Vaginismus, 49, 67 Workplace issues and intelligence, 278, 287 Vaginitis, monilial, 59, 79–80 hearing protection, 299, 306 primary amenorrhea, 101, 121 Vaginosis, bacterial, 53, 60, 72, 79–80 low back pain, 378, 385–386 and serum FSH concentrations, 61, 81 Valporate, 179, 194 and prenatal care, 131, 147–148 Twins Valproic acid, 279, 288 sciatica hazards, 294, 303 and bipolar disorder, 175, 190 Valvular heart disease, 20, 38 World Health Organization (WHO), determining if identical, 52, 70 Varicella immune globulin (VZIG), 139, 155 Type I second-degree heart block, 336, 87, 107 344 Varicella vaccine, immunocompromised Y Type II diabetes mellitus, 297, 304 patients and, 87, 97, 99, 107, 116–119 Yellow fever, 138, 154 Typhoid fever vaccination, 129, 146 Vasa deferentia, congenital absence of, 60, 80 Z U Vasectomy, failure rate of, 301, 307–308 Zenker’s diverticulum (See UGI hemorrhage (See Upper Vasoocclusive crisis, 103, 123 Pharyngoesophageal diverticulum) gastrointestinal hemorrhage) VCUG (See Voiding cystourethrogram) Zinc, acrodermatitis enteropathida and, Ulcerative colitis, 18, 37, 197, 228, Venereal Disease Research Laboratory 358, 366 332–333, 341 (VDRL) test, 87, 107 Zinc deficiency, 226, 249–250, 280, 288 Uncompetent cervix, 56, 74–75 Venlafaxine, 253, 261, 396, 406 Zoloft (See Sertraline) Get more e-books from www.ketabton.com Ketabton.com: The Digital Library